Вы находитесь на странице: 1из 3151

C V S

Coronary Artery Diseases


Risk Factors
Traditional
- LDL, HTN, smoking.
- HDL, obesity, DM,
- age >45, >55,
- FH of premature CHD < 55 , < 65 ,
- others; hyperuricemia, CRF. Stress, personality.
Non traditional;
Lp(a) , homocysteine , PAI-1, fibrinogen, cytokines, CRP, apo B ,
small dense LDL, visceral obesity , fasting hyperinsulinemia,
microalbuminuria.
NB; Effect of smoking;
- Bl viscosity, carboxyHb, & Hct.
- plat adh/agg,
- HDL.
Leading to ischemic stroke, SA Hge, CAD, HTN, AA,
thromboembolism.
Protective factors;
1- cholesterol in diet.
2- HDL by; estrogen, exercise, alcohol.

Effect of Estrogen.
1. HDL, LDL, Lpa.
2. LDL coronary uptake
3. coagulation factors.
4. insulin sensitivity.
Beneficial effect of small amount of alcohol; HDL, insulin
sensitivity, anti-thrombotic (TPA), anti-plat (PC/ tx), anti-oxidant
(flavinoids & polyphenolin red wine).
Normal adult 12-lead ECG
1)Hyperacute T

2)Raised ST 3)Q wave, T inversion

4)Q wave, inverted T 5)Q wave


Cardiac enzymes;
Enzyme Onset (hs) peak Return(day)
CPK-MB 4 18 2
Troponin 3 24 10
LDH 10 24 14

CPK-MB ratio>2.5% of total CPK,


new rise infarction extension.
Troponin;
- most sensitive, more specific than CPK.
- indicator of severity.
- n=<0.4 ng/ml.
- false +verenal, PE, myocarditis baseline & 12 hs after.
definitions
Angina pectoris;
- transient attacks of chest pain caused by myocardial ischemia.
- It is a symptom of CAD.
- Ccc by burning,or boring substernal pressure or heaviness.
- Precipitated by exertion, stress, cold.
- Releived by rest, nitroglycerine.
- ECG; subendocardial ischemia=
*transient ST depression in ant, inf or all leads ) during the attack.
*sometimes T wave inversion.

Prinzmetal angina;
- non infarction ischemia due to coronary artery spasm leading to
transient transmural ischemia.
- Atypical angina due to; ST elevation not depression, occur at rest or
at night.
- Rt coronary.
- patients coronary atherosclerosis.
- ECG; transient ST elevation, without Q or t wave inversion.
Subendocardial infarction;
- ECG; non Q wave infarction= NSTEMI=
persistent ST depression
&/or T wave inversion.

Transmural infarction
- ECG; STEMI = Q wave infarction =
1) Hypercute T,
2) ST elevation & reciprocal ST depression
3) Q wave,
4) T wave inversion.
NB;
- ECG may be normal in acute MI
especially early infarction.
- it is better to say Q or non-Q infarction.
Diagnosis of recent MI
- enzymes + (sympt. or ST)
or postmortum pathological finding.
Diagnosis of Full thickness MI =
+ akinetic area or scar on ECHO .
Acute anterior myocardial infarction
ST elevation in the anterior leads V1 - 6, I and aVL
reciprocal ST depression in the inferior leads
Old anterior MI
Q wave infarction
Non-infarction transmural infarction
transmural ischemia. Hypercute T, ST elevation, Q wave,
(Prinzmetal angina) T wave inversion.
transient ST elevation

Myocardial ischemia
non Q wave infarction
subendocardial infarction
Non-infarction
persistent ST depression
subendocardial ischemia
(classic angina) &/or T wave inversion
Transient ST depression without path Q
DD of ST changes
Raised ST;
- transmural infarction (>2 in chest leads, >1 in limb leads), with
evolutional changes & reciprocal depression.
- prinzmetal angina; transient without evolutional changes or reciprocal
depression.
- early repolarization pattern; young, stable ST elevation without evolutional
changes or reciprocal depression.
- pericarditis; special pattern, in all leads.
- LBBB; with characteristic QS pattern in V1,2 & notched R in V5,6.
- Depressed ST;
- angina.
- LVH with strain.
- non-Q MI=NSTEMI= subendocardial infarction.
- reciprocal depression in transmural infarction.
- digitalis; scooping of ST-T.
- Post. Wall infarction.
DD of T wave inversion
Normally in avR, V1.
Ischemia.
Ventricular strain pattern.
Pericarditis.
Subarachnoid Hge (deep, wide + prolonged
QT, U wave), may be due autonomic dysf.
BBB.
Ventricular Pacemaker.
Stable angina;
- signs (+/- S4, MR).
- Management.;
- ECG during the attack, transient ST depression &/or T wave inversion.
- TTT During the attacknitroglycerine tab, 0.4 mg not more than 3 tab.
- Exercise Stress test, if normal ECG in between the attacks;
*The test is stopped if the patient develop angina, fatigue, diagnostic ST changes,
*+ve results = failed or BP, ischemic ECG changes.
* if early + ve results angio.
If achieve 9 min medical ttt (BB, CCB, aspirin 150mg, exercise).
- Thallium stress, to increase the diagnostic accuracy of Exercise Stress test,
- coronary angio,
indication; - unsettled diagnosis.
- refractory to medical ttt.
- suspect Lt main stem or 3 vessels dis.
- pt > 40 yrs before valve replacement.
*if proximal stenosis dilatation by PTCA (may need repeated procedure).
*CABG is done if - restenosis after PTCA.
- severe 3 vs
- severe Lt main stem.
- DM + 2 vs.
Unstable angina.
angina is said to be unstable if;
- accelerated (severe, frequent, longer).
- at rest.
- post MI, PTCA, CABG.
TTT ;
1. Medical; Aspirin (or clopidogril), Heparin (or
LMWH),GP IIb IIIa(absiximab, tirofiban), nitroglycerin
(PO or IV), BB.( during the attack), .
2. early percutaneous coronary intervention (PCI), with
or without stenting, is recommended in all except low
risk patients.
3. In low risk patients discharge with outpatient stress
testing within 72 hours.

4. After dischargeBB, Aspirin, nitrates, lipid lowering,


ACEI.
NSTEMI= Non Q wave infarction=
subendocardial infarction ;
- ECG; persistent ST depression &/or T wave
inversion.
- evidence indicates that subendocardial infarction
may have as bad long term prognosis as
transmural infarction with higher 1 yr mortality.
- should be investigated early & aggressively.
- TTT; GP IIb IIIa.
- no benefit from thrombolysis.
Syndrome X;
- Middle aged female presented with atypical chest pain.
- May be related to estrogen deficiency.
- Due to small vessel disease or Lt ventricular dysfunction.
- ST on exercise test.
- Normal angio.
- TTT; medical ttt, Laser percutaneous transmyocardial revascularization;
= Laser holes in the epicardium to form channels connected to vent
cavities, benefit in distal dis e.g. DM.
coronaries
Rt coronary supplies the inferior (diaphragmatic)
portion of the HT.
LAD supplies septum & most LT ventricle.
LT circumflex supplies the lateral wall of Lt ventricle.
Frequency of occlusion; LAD, RCA, Lt circumflex.
Transmural Myocardial infarction

Definition; ischemia & necrosis of a portion


of the entire thickness of the LT ventricular
wall.
ECG changes;
- Acute phase; Hypercute T, ST elevation +
reciprocal depression.
- Evolving phase; Q wave,T wave inversion.
Localization of infarction;

Anterior wall infarction;


- anteroseptal; in V1,2.
- Strictly ant= in v3,4.
- Anterolateral; in V5,6.
Acute anterior myocardial infarction
ST elevation in the anterior leads V1 - 6, I and aVL
reciprocal ST depression in the inferior leads
Complications of anterior wall Infarction;
1 - septal perforation & VSD.---- lt vent failure.
2- cardiac rupture (tamponade).
3- vent aneurysm;
- double apex,
- persistent ST without reciprocal depression.
- complication; 1.VT- syncope ,
2.thromboembolism,
3. rupture .
- TTT: warfarin for 3-6 ms, surgery if significant dec in COP.
4- thromboembolism in 1/3 cases.
5- dressler $= post cardiotomy $
ESR, fever, anemia, pericardial eff,
anticardiac ms ab
ttt=NSAIDs, steroids.
6- Mobitz II.
7- BBB.
inf. Infarction
inferior (diaphragmatic) portion of the LT ventricle.
Changes in lead II, III, AVF.
Due to occlusion of RT coronary & less commonly Lt
circum.
Complications of inf. Infarction;
- Papillary ms dysfunction (rupture is rare).
- conduction abn; CHB, Mobitz I (SAN is supplied by
RCA in 60%).
- RT vent infarction. fluid & thrombolysis.
- Posterior wall inf.
Acute inferior myocardial infarction
ST elevation in the inferior leads II, III and aVF
reciprocal ST depression in the anterior leads
Papillary ms dysfunction ( with Inf. MI);
- in 0.1% of MI.
- 80% the post. Papillary ms (supplied
by RT coronary, so more common with Inf.
MI.
- several days after MI.
- TTT; emergency MVR.
- Prognosis; 24 hs survival = 25% with
medical ttt, 50% with emergency MVR.
Posterior wall infarction

Posterior wall of the LT ventricle.


ECG; tall R, ST depression & tall +ve T in
V1,2.
May extend to the lateral or inferior wall
(postero-inferior).
Due to occlusion of RT coronary.
Acute posterior myocardial infarction
(hyperacute) the mirror image of acute injury in leads V1 - 3
(fully evolved) tall R wave, tall upright T wave in leads V1 -3
usually associated with inferior and/or lateral wall MI
RV infarction;
- in 1/3 of inferior infarction.
- BP, congested neck, clear lung bases.
- diag.; ECG ST in V1, V3R, V4R,V5R.
- confirmed; ECHO, coronary angio,
thallium scan.
- TTT= monitor by PCWP, IV fluid,
thrombolysis but never nitroprusside or VD.
inf-post-lat
NB;
* Cause of Post-MI VT;
- scar EPS & ablation.
- ischemia angio & thallium.
if associated with LVEF < 40%ICD.
* Indications for temporary pacing post MI (for
2 wks ) - ant wall & CHB or new BBB.
- inf wall & CHB if HD unstable.

- if no sinus rhythm within 2 wks permanent


pacing.
Shock after MI;
1 - RV infarctionfluids.
2- VSD Vs. papillary ms rupture (severe
MR) ECHO & IABP.
3- LV extensive infarction or re-infarction
(loss of > 40% of LV) IABP.
4- free wall rupture (fatal) & tamponade.
PCWP = Lt atrial pressure = LVDP.

Post MI CVP PCWP


LVF
MR N
VSD N
Tamponade
RVI
Management of transmural infarction
1) Thrombolysis
Indication ( pain & ECG)
- pain < 12 hs + ST in 2 leads.
- pain + new BBB.
Most benefit;
- early 1 hr.
- poor LVF or syst BP < 100.
- high ST.
- large ant. infarction.
Thrombolysis
Contraindications;
1) Absolute;
- active internal, uncontrolled ext. bleeding.
- suspected Aortic dissection.
- uncontrolled BP > 200/100.
- head trauma < 2w, stroke< 2 m.
- cranial or abd neoplasm.
- pregnancy.
2) Relative;
- CPR > hr
probable IC thrombus e.g. (AF+MS)
Complications
- reperfusion arrhythmia within 2 hs.
-Hge.
NB; - 1ry angioplasty is better than thrombolysis, preferred if HD unstable
2) Drugs which decrease mortality in MI;
1) Aspirin (not dypyridamol).
2) thrombolytics/ angioplasty.
3) BB (FU with PR < 0.24, HR >45, SBP > 100)3 doses
of 5 mg metoprolol
4) ACEI esp. CHF, EF< 40%, cardiomegaly.
5) Statin ( mortality & recurrence) even with
average cholesterol prior to discharge.
- LV function is the prognosticator post MI.
NB: - nitrate morbidity not mortality.
- DHP CCBs inc.CVS risk after MI.
- In type II DM, must be kept on insulin infusion for
24 hs then SC for 3 months mortality.
3) PTCA
Indication;
- discrete lesion. - proximal.
- non calcified . - non-occluded
- short history of angina.
Complication;
1) acute occlusion.
2) restenosis.
* for stent restenosis (for high risk patients as DM).
drug eluting stents + GP IIbIIIa + aspirin +
clopidogril for 1 m .

GP IIbIIIa used in; - high risk PTCA - stents


- NSTMI - unstable angina
4) CABG
Indication;
- symptomatic LT main stem dis.
- symptomatic proximal 3 vs dis.
- 2 vs including LAD.
Most benefit; moderately impaired LVF.
Complication;
1) mortality: < 2% , 10% for 2ry procedure.
2)periop. Graft occlusion; 10%
esp venous grafts.
( art. Grafts have higher patency).
3) post cardiotomy $ up to 6 m after.
5) Laser percutaneous transmyocardial
revascularization;
= Laser holes in the epicardium to form channels
connected to vent cavities, benefit in distal dis
e.g. DM.

6) Elective balloon pump insertion;


- cardiogenic shock
- VSD
- papillary ms rupture.
Post MI rehabilitation
1 month abstinence from sex & driving.
2 months off work.

Abstinence from driving after revascularisation


- 1 m for ordinary driver
- 3 m for vocational drivers.
- Loss of license after ICD.
perioperative cardiac complication
cardiac complication are the most common cause of
perioperative mortality & morbidity esp. after vascular
surgery.
Investigations;
So if high risk of ischemia (history of CAD)coronary
angio.
If intermediate risk ( DM, PVD) dipyridamol thalium
scan or dopamine stress ECHO ( both are equivalent,
NPP > 95%.
if low risk exercise stress test.
TTT;
Perioperative BB risk of morbidity by 50% &
recommended for high & intermediate risk.
Temporary pacing for trifascicular block.
Myocardial diseases
Myocarditis
Infective Non- Infective

*Viral -Rheumatic
Coxsakie, ECHO, mumps, measles, influenza, EBV, -CT dis; SLE,
HIV, adeno, rubella, robeola. Rheumatoid,
*Bacterial myocarditis
TB, brucella, Hemophilus. recurrent
refractory VT.
* spirochital;leptospirosis, borellia burgdorferi -Drugs;
*Fungal (aspergillus, histoplasmosis) adriamycin,
*parasitic; trypanosomes cruzi, trichinosis, chloroquine,
toxoplasma, amoeba, malaria phenothiazine,
lithium, sulfa,
paracetamol.
-irradiation
Myocarditis DCM
History -Young -Older
-Acute onset - chronic
-Prodrome (fever, flu like,arthralgia)
Investigation -Neutrophilia -ve
-viral Ab titre e.g. cox B titre1:160. -ve
-CXR; Slight cardiomegaly. - CXR; Huge
-ECG; episodic VT, HB, ST/T. cardiomegaly

ttt Avoid BB, digitalis


Cardiomyopathies
These are conditions that;
1ry affect Ht muscle.
Of unknown etiology.
Characterised by myocardial dysfunction
After exclusion of
- volume & pressure overload.
- IsHD
- pericardial disease.

NB; ischemic CM= IsHD that has no other manifestations


as angina or MI but only present with Ht failure.
Hypertrophic cardiomyopathy (HCM)
AD in 50%.
Mutation of myosin gene on chr 1, 11, 14, 15.
Bimodal 1st peak 2nd decade.
2nd peak 4-6 decade.
LV outlet obstruction occur in late systole.
Associations;
- Friedreich ataxia.
- WPW.
- pheochromocytoma
Symptoms;
- exertional dyspnea
- chest pain.
- palpitation.
- syncope.
- sudden death VT (dt acc. Path- associated AF)
massive infarction
outflow obst
(HCM)
Signs;
general,
- prominent a wave in neck veins

Local,
palpdouble apical impulse (S4), LS thrill.
auscmurmur; late or pansystolic over LSE or apex
more than A1, not over carotid dt outflow obst & MR.
by obst e.g. digitalis, inotropics, volume e.g.
standing, valsalva, diuretics, VD as nitrates.
by squatting, hand grip.
(HCM)
Invest;
ECHO- Asymetrical septal hypertrophy 60%
concentric 30%, apical 10%
- septum/post wall>1.5.
- >30% LV outflow gradient
- obliteration of LV cavity.
- systolic ant. Motion of mitral valve leaflet.
Catheterbanana or spade like LV.
Poor prognostic features;
- young age < 30 yrs.
- FH of sudden death.
- syncopal sympt.
- LVH > 3 cm.
- VT on Holter ECG.
- hypotension on peak exercise test.
NB; no correlation with outflow tract obst gradient.
(HCM)
Management;
1. Avoid volume depletion
2. avoid intense physical exertion and competitive sport.
3. Treatment of dyspnea and chest pain generally begins
with medical therapy. negative inotropic agent;
- verapamil or
- beta blocker, +/- disopyramide,
- cautious addition of a diuretic,
4. If medical therapy fails non-pharmacologic therapy;
surgical myectomy or alcohol septal ablation.
5. Prophylaxis for endocarditis.
Dilated cardiomyopathies (DCM)
,
Causes;
Alcohol.
Nutritional; thiamine, selenium, Ca, P.
Endocrinal; hypo &, DM, acromegaly.
Metabolic; glycogen storage diseases.
Infiltrative; hemochromatosis.
Toxic; cocaine, adriamycin.
Peripartum.
Viral, cox, HIV.
autoimmune,; SLE, Sclero.
C/P; biventricular failure.
ECG biventricular ++ ; LBBB , poor R prog., +/- AF.
Catheter; CO, EDP in both vent.
TTT; antifailure= ACEI, digoxin.
anticoag= AF, mural thrombosis.
antiarrhyth=
Restrictive CM
Causes;
- myocardial= idiopathic, amyloidosis,
sarcoidosis, scleroderma,
hemochromatosis, glycogen
storage, Gaucher.
- Endomyocardial =fibrosis, hyperesinophilic S,
carcinoid, malignancy,
irradiation, toxin-related.
Sympt; SVC, PVC, low COP, AF.
Signs; steep x, y.
pulsus paradoxus, AF.
TTT; antifailure + anticoag + ttt of etiology.
Pericardial diseases
Pericarditis
Infective Non- Infective ( T U M O R)

*Viral -TB, trauma.


Coxsakie. -Uremia
*Bacterial -Malignancy, medication (those causing SLE as
Strept, staph, TB. hydralazine, procainamide, INH), MI
-Other infections, viral , bacterial .
-Rheumatic fever, rheumatoid &Other CT,
radiation, recurrent

Types
1- acute dry= fibrinous.
2- associated with pericardial effusion;
serous CT.
serofibrinous T U M O R
hgic TB, tumors, trauma
3- constrictive.
Pericarditis
Constrictive pericarditis (TB)
Causes = T U M O R without ( medications, MI, RF).
Symptoms;
- Rt vent failure (cachexia, ascites, hepatomegaly, oedema, JVP)
- + dyspnea, AF due to atrial enlargement.

Signs;
- X, y descent deep .
- Kaussmaul sign (inspiratory filling of neck v).
- Impalpable apex.
- Pericardial knock.
- AF.
- Ascites precox.

CXR; pericardial calcification.

ECHO; thick bright pericardium, biatrial enlargement, normal systolic function,


poor diastolic function, peak systolic & diastolic values during inspiration.

ECG; PR (early & specific), ST, inverted T.

CT scan , pericardial rim of calcification.


pericardial effusion
Types;
1) serous;
- transudateoedematous states.
- Exudate hypothyroid, CT, viral, TB, malignancy.
2) hgic; TB, tumors, trauma, CRF + heparin on HD.
3) bloody; ruptured aneurysm, dissecting AA, MI.
4) chylous; lymphedema .

Signs; JVP, pulsus paradoxus, loss of y descent dt atrial compression,


Ewart sign= bronchial breathing at Lt lung base dt compression of lingual
lobe.
CXR; globular cardiomegaly( flask shaped).

ECG; electrical alternans, electromechanical dissociation in tamponade.

TTT; drainage or pericardial window in chronic cases.


Restrictive CM Constrictive P P.effusion/tampon
Neck veins Common; JVP, Deep X & Y Y obliterated

palpation Systolic bulge Systolic retraction Impalpable apex

auscultatio S3 gallop, TR +/-MR (cardiomyopathy as Pericardial knock Distant Ht sounds


n Ht failure) (as S3 due to catching effect
of pericardium on the relaxing
ventricle)
liver Common; congested non pulsating

CXR Ht failure without cardiomegaly pericardial calcification Flask shaped


cardiomegaly
ECG Common; AF
Catheter - Difference bet 2 ventricle -Less
LT VEDP > RT
-Difference of pressure in the chamber
during cardiac cycle -Less

ECHO Common; normal systolic function

Confirmed biopsy CT, MRI ECHO


by
Endocardial diseases
Rheumatic Fever
Diagnosis;
evidence of recent strept infection (ASOT >250 adult or
333 children, scarlet fever, +ve throat swap, other
antistrep Ab)
+ 2 Major or 1M & 2m criteria.

Major minor

1-carditis fever

2- arthritis arthralgia

3- chorea CRP, ESR

4-erythema marginatum PR

5- SC nodules (non tender, Previous RF


on knee, elbow, spine)
Prophylaxis from recurrence;
1- TTT of strep infection with penicillin.
2-long term long lasting penicillin (Benzathine penicillin 1.2
million unit IM/month (erythromycin/12 hr if penicillin
allergy) till
- 25 yrs or
- 30 yrs if RHD or
- 5 yrs after the last attack.
Treatment;
1. Benzathine penicillin 1.2 million unit/wk IM for 3 wks
or oral penicillin 500 mg/6 hrs for 10 days.
2. Salicylates (5-6 g/d) for 6 wks e.g. indomethazin,
diclofenac.
or steroids 50 mg/d for 4 wks if intolerance to salicylates.
FU with ESR.
Endocarditis
Causes of endocarditis
Infective Non- Infective

*Bacterial -Rheumatic
-Strept viridans (subacute), -Lebmansac
fecalis (=enterococci after urinary cath) endocarditis
bovis (colorectal cancer) -Carcinoid
-Staph aureus (skin, drug abuse, acute), -Marantic
albus (cardiac surgery),
epidermidis ( valve replacement)
-Pseudomonus.
-Hemophillus (HACEK, ceftrioxone very effective)
* Ricketsia, coxsiella (=Q fever)
*Chlamydia
*Fungal (candida, aspergillus)
IEC
IEC; 40% no valve lesion.
10%IV drug, cong, prothetic
50% strept viridans
Signs of IEC;
- toxic manifestation + clubbing, subconj Hge, - spleen, soft small
palpable ( -ve in renal, old, debilitated, HT failure),
- cardiac( varying murmur, new murmur, conduction defect, dt abscess
formation, Ht failure, infarction dt emboli0.
- Thromboembolic ;
*acute septic emboli pyemia, mycotic aneurysm in the brain
subarachnoid Hge
*SBE infarctions e.g. CNS, mesenteric, - renal, infarction, CRAO ,UL, LL.
Immunological
Kidney focal proliferative , MPGN.
- skin, osler nodes ( tender, cutaneous), splinter He,, Janway spots.
- eye; Roth spots,,.
Poor prognostic factors.
- staph,
- culture neg. ,
- proth.valve,
- low complement.
Indications for surgery
- relapses,
- septic emboli,
- septal abscess,
- fungal,
- large vegetations,
- extensive valve incompetence.
FU by CRP (ESR fall slowly).
Medical ttt till culture results.
Pen G 2-4 mill U/4h
+ gentamycin 1 mg/kg/8h
+ nafcillin or oxacillin 1.5 g IV/4h
Medical ttt after C& S;
Strep benzyl pen (or vanco) + low dose genta.
Staph. Flucloxacillin (or vanco)
Rickettsia -> Rifampicin + doxycyclin.
Pseudomonus carbinicillin 10 g/d + genta 240mg

Medical ttt in special situations;


Penicillin hypersensitivity vancomycin 15 mg/kg/12hr.

Prothetic valve- vancomycin 15 mg/kg/12hr.


- gentamycin 1 mg/kg/8h for 2 wks.
- Rifampicin 300 mg/8 hrs.
Rt sided;
- 50% staph, 15% pseudom,
- diagnosed with transthoracic ECHO,
- flucloxacillin (vanco or teicoplanin) + gentamycin for 2 wks.
- no valve replacement if pulm septic emboli.

2-6 months Post operative (staph, 85% MRSA) vancomycin.


Antibiotic prophylaxis
NICE guidelines 2008 recommended that Antibiotic prophylaxis is
no longer offered routinely for defined inteventional procedure.
Antibiotic prophylaxis has not been proven to be effective & there
is no clear association between episodes of IEC & interventional
procedures.
Benefits of antibiotic prophylaxis must be weighed against the
adverse effects for the patients & the risk of developing antibiotic
resistance.

People at risk are;


1. Acquired valvular HD with stenosis or regurge.
2. Valve replacement.
3. structural congenital HD except isolated ASD, fully repaired VSD
or PDA, closure devices that are judged to be endothelialised.
4. HCM
5. Previous IEC.
Advice people at risk about;
- Good oral hygiene.
- Symptoms of IEC.
- Risks of invasive procedures
- Why ab prophylaxis is no longer indicated.

Do not offer ab prophylaxis for;


- All dental procedure. except.
- Upper & lower GIT, respiratory, genitourinary.
- Do not offer chlorhexidine mouth wash before dental
procedures.
- Investigate & treat any episode of infection.
- Offer ab that cover organisms causing IEC for those
pts undergoing procedures at a site where there is a
suspected infection.
Antibiotic prophylaxis
* not indicated for;
- cardiac cath
- diagnostic upper GI endoscopy.
- large secondum ASD.
- MVP without regurge.
- after 6 month of valve repair.
- TEE.

indicated for;
-- all surgery.
- rigid bronchoscopy.
- therapeutic upper GI endoscopy.

regimen
Upper (e.g. dental)
BeforeAmox 2 g oral or 1 g IV 1 hr
After genta 1.5 mg/kg IV + 500 mg oral amox 6 hr if general anathesia.
Lower (colonoscopy and biopsy) Amox 1 g IV + genta 1.5 mg/kg IV + Amox 1g
oral 6 hrs
Cardiac tumors
50% -Myxomas * 2ries from breast & lung
But tumor that most commonly metastasis to the heart= melanoma.
Atrial myxoma;
- interatrial septum, LT atrium.
- .
- sympt.constitutional (IL-6fever, clubbing)
systemic embolism.
postural syncope.
- signsMS , early diastolic plop (murmur).
PHT, sinus rhythm.
- invest; WCC.,
ESR in 60%,
IG (hypergammaglobulinemia) .
- diag.=TEE, avoid catheter.
- TTT= surgical resection without delay ( rapidly growing, & embolize)
Valvular Diseases
MS MR AR AS
Normal average =4.5 cm, 2.5 cm,
valve MS<2cm, tight MS<1cm. AS<0.7 cm, severe AS<0.5
area
Causes 1- organic;
1- organic; 1-organic; 1- organic;
-Rh -Rh
-Rh, chronic -Cong marfan -Cong
-cong -Rh -syphilis, RA, valv; bicuspid
2- functional; -Cong e.g. Ank. Sp. suprav; wiliam,
Marfan, MVP -Acute; IEC,
- Austin flint (with AR). coarct
-Ischemic dissecting
Cary comb(Rh carditis subv; HCM
Acute aneurysm
- MR, VSD, PDA. calcific;
(infarction, IEC,
- tumors 2- functional;
post valvotomy)
Hemic
functional; HTN
-LVF AR
-HCM
MS MR AR AS
Symptoms 1- PVC 1-palpitation 1-palpitation 1-Angina
2-Low COP 2- PVC 2- angina 2-syncope(Low
3-SVC 3- Low COP 3-PVC dt LVF COP)
3-dyspnea(PVC)
Signs - Malar flush Peripheral
1- general signs e.g.
corrigan,
DeMussey,
water-
hummer
pulse,
Duroziez,
Hills
2-local -Palpable S1 Soft S1 Early soft Soft S2
-Loud S1,OS, Soft blowing blowing diast Harsh ejection
-Loud S2 murmur systolic murmur at
pansystolic
A1
-Mid-diastolic murmur
rumbling murmur.
MS MR AR AS

Signs of -OS near S2 -PH, -LVF -Syncope, LVF


severity
-long murmur -Thrill -Peripheral signs -Pulsus T&P
-PH, loud S2, PR, -LV++ - S3. -Low pulse pr,
RV heave, -Long murmur low BP
-S3
TR -Funct MS (austin -Thrill
-Func MS flint) -Single soft A2,
- ECHO; M area <1cm
-Cath; PCWP>25. reversed splitting
pr gr>15. -S3,S4
COP<2.5L/min/m2. -Long murmur
-ECHO;
V area<0.5
SP gr>60
Indication Severe symptoms Severe symp symptoms symptoms
for Severe MS Severe MR Severe AR Severe AS
surgical Thromboembolism ECHO; -ECHO; EF<50%,
interventi despite adequate EF<60%,
on AV root>50mm,
anticoag inr=2.5-3.5) LVESD>50mm
MS MR AR AS
TTT 1- Prophylaxis from 1- Prophylaxis -If asymtomatic -surgery
IEC. from IEC. FU with ECHO. - No role for
2- diuretics 2-ttt of AF -surgery medical ttt
3- ttt of AF 3-VD
4-+/-surgery 4-+/-surgery

NB; Silent MS; Predominant MI without surgery,


1-lutembacher (+ASD) in Double M; sudden death 73%.
2-PH 1-soft S1
3-RVF 2-S3 in Asympt. Adult,
Calcified 3-displaced sudden death3-5%.
1-soft S! hypertrophied
apex child, sudden death
2-no OS
S1+ MR; in 6-9%.
NB; huge Lt atrum in
MR, AF in MS 1-MS Complications
2-post. Leaflet (S Death, CHB,
calcific emboli, CAD
in 50% of severe
AS)
Bicuspid AV
- Most common cong HT
- Sporadic, familial in 10%.
- calcify with age AS.
- Surgery is likely to be required.
Aortic sclerosis
- Only localized murmur
- No LVH
- Normal pulse volume
Cause of AS according to the age;
<60yrs Rh, cong
>60yrs + calcified valve.
TR
Causes;
- organic; Rh, cong (Ebstein anomaly), IEC, carcinoid.
- Functional; RVF.

Symptoms; palp, SVC, low COP.


Signs;
general cyanoecterous, prominent V, absent X,
ascites precox.
local pansystolic murmur to the Lt of the sternum,
carvello sign(murmur by inspiration).
PS
Causes
- organic cong (fallot)..
- Functional; ASD, hemic , PHT.
Prothetic valve
Types
1. Mechanical valve;
adv; durable.
disadv; lifelong anticoagulation.
NB; INR target for AV= 2-3, for MV=2.5-3.5.
2. Tissue valve (porcine, homograft)
indications; old age.
adv; no anticoagulation.
disadv; calcify with restenosis, replaced after 5-10 yrs.

Indications;
1. Severe AS, AI, MI.
2. MS in the following conditions;
- MV score >8 (according to mobility, thickness, calc., subvalvular apparatus.
- associated MR 2/4.
- Lt atrial thrombus.

Signs;
MV prothesis; metalic S1, metalic OS+ systolic murm+/-diastolic murmur.
AV prothesis; metalic S2, metalic E click + systolic murmur only.
NB; any early diastolic mrmur at AV area malfunctioning V.
tissue valves does not produce metalic sounds.
Complications;
- infection; mortality 60%.
early -1 y staph epidermidis. later fungal.
if PR septal abcess.
- dehescencesurgery.
- hemolysis.
- thromboembolism.

Anticoagulation + surgery;
- stop warfarin & give heparin/6h, 3 days before.
- stop heparin 6 h before till 24 h after major surgery or to 6 h after
minor surgery.

Anticoagulation + pregnancy;
- 1st tri.heparin, 2nd warfarin, 3rdheparin, lactationwarfarin.
- warfarinfetal hge
teratogenic 5-30% acc. To dose; mental R, optic A, nasal
hypoplasia, chondrodysplasia.
MVP
Associations;
- WPW, ASD, PDA, LA myxoma.
-CT disorders. pseudoxanthoma, osteogenesis imperfecta, Marfan,
- Symptoms; palpitation, pain due to stress ischemia of papillae.
Signs;
- midsystolic click by squating,
- midsystolic murmur by standing .
Complications;
- long QT, arrhysthmias, death.
- rupture of chordae.
- embolism.
- neurosis.

ECG;depressed ST, T inversion in inferior leads, long QT,


ECHO; systolic posterior motion of 1 or2 leaflet, mainly post. Leaflet).
Treatment;
- BB for palpitation, pain.
- anticoag for embolic manifest.
- prophylaxis of IEC if audible murmur or thick leaflets.
- valve repair (not for the click only).
Pulmonary hypertension
Mean P pressure =20mmHg,
Mean P pressure=pulm. Diastolic pressure+1/3 pulm puls
pressure
PH = mean PP> 30 mmHg
C/P; inspection; Pulm. pulsation.
palpation; palpable P2,
auscultation, 2 sounds= P2, S4;
2 murmur = PS, PR (graham steel),
1 click= ejection systolic at Pulm.area.
Causes;
A) 2ry PH
- Passive; MVD, const p, restrictive CM, Lt atrial
myx.
- Hyperkinetic; ASD, VSD
- reactive=VC e.g. high altitude, COPD.
- obliterative= sclerosis, fibrosis (EAO e.g. B).
- obstructive = embolic ,vasculitis , Bilarz.
TTT of PH;
1-Digoxin (even in sinus rhythm)
2-diuretic in RVF
3-Ttt of the cause,
B) 1ry PH (<1% of all cases)
- Def; mean PAP>25 at rest without any cause.
- Female /male = 2:1.
- 1/10 may be familial, other possible causes; CT dis, vasculitis, HIV, drugs as appetite
suppressor (fenfluramine).
- CXR; dilated proximal artery & pruned (very small diameter) peripheral.
- Median survival if untreated = 3 yrs.
- Management;
1-Do 1st PFTs, ECHO, immunologic markers, V/Q scan, Rt catheter &
VD test with adenosine or inhaled NO
if dec. in PAP without COP CaCB
if no SC pump of epoprostenol

2-avoid pregnancy, exercise.


3-Digoxin (even in sinus rhythm)
4-diuretic in RVF
5-O2
6-Anticoag
7- surgery; - HL transplant if; RVF, PASP>60 mmhg, expected 2 yr survival <50%.
- atrial septostomy in no resting hypoxia.
Congenital heart diseases
without shunt + shunt

Acyanotic -AS (potentially cyanotic)


-coarctation Mixed blood
- dextrocardia -ASD, VSD, PDA
-Coarctation of Aorta

cyanotic -P atresia -Fallot


-Severe PS -Ebstein
-Hypoplastic Lt Ht -Complete transposition
of great vessels

NB:
- Most common cong HT= biscuspid AV.
- Most common isolated cong HT= VSD ((30%)
- Most common cong in adult = ASD.
ASD
Most common cong in adult.
Complication; Paradoxical embolism-> stroke in young.
Associated;
- fetal alcohol S, Down S , cong rubella, Noonan

Types;
- 70% Osteum secondum
- ass with MVP (10-20%)
- RT vent dilatation RBBB + RAD
- 15% Osteum Primum
- ass with MI, TI , VSD
- - affect conduction system RBBB, LAD
- picked early in childhood.
- 15% sinus venosus
- defect in upper septum
- ass with anomalous pul venous drainage.
Signs;
-P; fixed splitting of S2, functional PS (ejection systolic murmur).
- Lt parasternal heave.
- PS.
- in OP (MI, TI, VSD).

Invest.;
- ECG; biatrial enlargement, RBBB, RAD.
- ECHO; paradoxal septal motion (bidirectional movement through the defect),
septal defect.
- CXR; pulmonary plethora.
- catheter; step up O2 in RT atrium(mean increased oxygen concentration in
the RT atrium), PH, RVP.
NB; Best is TEE.

TTT;
- antibiotic prophylaxis from IEC only in O Primum.
- indication for surgery; pulm/systolic flow ratio > 1.5/1
- may be closed with a prothetic patch through cardiac catheter.
Patent foramen oval;
- 25% of population.
- slit- like dehiscence in fossa oval.
- unlike ASD; no equalization of pressure between 2 atria.
- like ASD; Paradoxical embolism.

Holt-oram S
- Rare S; ASD, triphalyngeal thumb, absent upper arm.
- AD.

Lutembacher S.
ASD + Rh MS silent MS(ASD decreased the load over the
stenosed mitral area ,so no early diastolic murmer of MS)
VSD
Types;
1. Muscular (Rogers dis); small defect.... close
2. Membranous; most common is memb., 30-50%
spont. Closure.
Signs;
- apex; hyperdynamic
- Palpable P2, parasternal thrill
- Auscult; inc. P2, parasternal pansystolic murmur;
if eisenmenger murmur & thrill disappear, signs of
PH +/- RVF.
NB; VSD COP.
TTT; antibiotic prophylaxis from IEC (high risk)
surgery at 3-6 yrs if pulm/systolic flow ratio > 1.5.
PDA
Causes;
- prematurity
- cong rubella
Most common distal to LT subclav.
Signs;
- general; peripheral signs of AI.
- local; Lt infraclav thrill, enlarged lt Ht.
ausc, Lt infraclav machinary mur.
if eisenmenger diff cyanosis in LL, mur become short
& soft. PH.

TTT;
- Premature infantindomethacin (90% closure).
- antibiotic prophylaxis
Eisenmenger
Sign;
-general; clubbing, cyanosis, V wave dt TR.
-local;
Palp P2, Rt vent heave.
ausc; pansyst murm & flow mur., PR (graham steel).

Catheter; RA, RV, PA pressures.


Complications;
- RVF,
- hemoptysis,
- cerebral embolism, abscess,
- polythythemia, thrombosis, gout,
- IEC (rare)

TTT; - medical for HT F.


- Ht lung transplantation
F3 F4 F5

-ASD ASD
Fallot -PS -PS + F4
-RVH -RVH
-Overridding aorta
-VSD
Cyanotic spill=hypoxic syncope= PS,periph resistance.
Signs;
- general; cyanosis, squatting, clubbing, stunded growth
- local; RV heave, PS murmur. inversly related to pulm gradient, single loud
A2.
Invest;
- CXR; pulm oligemia, coeur en sabot
- ECG; RVH.
Complications;
-IEC,
-polythythemia, thrombosis, gout.
-paradoxical embolism, cerebral abcess.
-Vent arrhysthmia.
TTT; blalock operation (BL from Lt subclav a to P) weak radial pulse.
NB; Ebstein anomaly;
- Maternal intake of lithium in 1st trimester.
- Cyanotic.
- pulmonary atresia,TR,
- ASD, Rt to lt shunt
Coarctation
2 - 5 times more common in males.
Most common distal to LT subclav.
Symptoms;
- in infants Ht failure.
- adult UL; shoulder pain, headache, HTN, epistaxis.
LL; cold, intermittent claudications, weakness.
Signs.
- general; HTN, radiofemoral delay or absent femoral pulse
- Local;
ins/palp; suzman sign= visible, palp interscapular collateral
pulsation.
auscul; - HTN; A2, S4 , A ejection click, AS ,
-Lt interscapular late syst mur.
- syst or continuous mur of collaterals over the back.
Invest;
- CXR; - Roesler sign; 3-8 rib notching
- - Ba swallow; reversed 3 or E sign of oesophagus.
Association;
cardiac; - Bicuspid AV - PDA - mitral V dis.
extra cardiac; - Berrys aneurysm - renal abn. - Turner S.

Complications;
- IEC - dissecting AA - subarachnoid Hge - LVH.

TTT
- surgical resection at 4-6 yrs.
- balloon angioplasty if recurrent.
- antibiotic prophylaxis & ttt of HTN.
Differential diagnosis
Of systolic murmur over the apex;
1. MI
2. AS, PS (propagated from above).
3. VSD (all over the precordium)
Of diastolic murmur over the apex;
1. MS
2. AI

Of systolic murmur over the A1;


1. AS (with all its causes; organic, functional)

Of systolic murmur over the P;


1. PS (organic, functional).

Of Lt parasternal systolic murmur


1. Posterior leaflet MI.
2. TI
3. VSD
4. Subval. AS (HCM)
5. Subvalv. PS.
Pressure in cardiac chambers;
Values in mmHg

4-12

0-6
90-140
/4-12
15-30
/0-6

120 (90-140)
/80 (60-90)
25 (15-30)
/10 (8-15)
Vascular diseases
DVT
Invest;
- venous duplex rarely venogram
- ankle-brachial pressure index +/-
arteriogram to detect ischemia which could
contraindicates compression.
TTT;
- recent DVT anticoagulation.
- old DVT ttt of oedema, infection,
compression after exclusion of ischemia.
PE
C/P;
1. Massive pulm embolism&Submassive pulm embolism
- when > 50% of pulm vascular area is obstructed PAP.
- retrosternal pain, dyspnea, shock, cyanosis, Rt sided failure.
2. Pulm infarction; uncommon, must have occlusion of bronchial vs &
airways cough, hemoptysis, fever, jaundice, signs of atelectasis, rub,
effusion.
3. Chronic pulm HTN; recurrent small pulm emboli no symptoms, cor
pulmonale, recurrent episodes of dyspnea, fever, arrhythmias.
NB; V/Q scan;
- v=xenon, Q= macroaggregated albumin.(no iodine used.)
- can be done during pregnancy (perfusion only is adequate).
- sens =98%, specificity = 40% so if ve virtually exclude PE.
- in COPD & emphesema matched defects.
D Dimer in PE, sepsis, MI, DIC.
Invest;
1. D Dimer (non specific), good ve no PE except if clinical
suspicion.
2. CT angio; only show large artery, same sens & specificity as VQ & angio.
3. ECG; an S1, Q3, T3 pattern
sinus tachycardia
T wave inversion in leads V1 - V3
RBBB, RAD, poor R progression.
4. ABG; low or normal PCO2, hypoxia, resp. alkalosis..DD; acute severe asthma but
PEFR is normal.
5. V/Q; mismatch, not adequate if basal consolidation shown.
6. Pulm angio; gold standard.

Diagnosis;
1. clinical probability + V/Q mismatch diagnostic.
2. clinical probability + V/Q mismatch exclude.
3. If any other investigate more.

Ttt; anticoag for high & moderate clinical probability;


- heparin 5-10,000 unit IV bolus then 1000 IU/h (adjusted to keep PTT =
1.5-2.5 for 5 days at least or stop when INR = 2-3. (or twice daily SC LMWH).
Acute pulmonary embolus
ECG; The following, often transient, changes
may be seen in a large pulmonary embolus;
an S1 Q3 T3 pattern
sinus tachycardia
T wave inversion in leads V1 - V3
Right Bundle Branch Block
low amplitude deflections
Acute pulmonary embolus
PE
TTT;
Massive pulm embolism
- TTT of DVT
- Thrombolysis (or embolectomy)
2. Submassive
- Anticoagulation
- ttt of Rt sided HT failure.
3. Chronic recurrent;
- Anticoagulation
- IVC filters.
NB; in pulm septic emboli; # thrombolysis for PE & surgery
for the valve for IEC.
PE
TTT;
1. Anticoagulation;
- LMWH twice daily SC.
- Or heparin 5-10.000 u IVbolus then 1300 u/h adjusted to keep
PTT=1.5-2.5 times for at least 5 days & stopped when INR=2-3
- Warfarin for 3-6 months.
6 wks if post poerative PE.
1 yr or lifelong if recurrent.
2. Thrombolysis;
- indications; collapse dt massive embolism.
- contraindicated if septic emboli as in RT sided IEC in drug
abusers.
- done through a peripheral vein to pulm artery, as effective as
embolectomy (which is rarely done now).
3. IVC filters;
- indications; when anticoagulation is # or failed (=continuous
showers of emboli)
NB; Shock

With PCWP=
With PCWP= -RV infarction
-LVF -PE
-Temponade -Hypovolemia
-MS -Septic shock
Aortic aneurysm
Causes;
1. Atherosclerosis (commonest).
2. Congenital;
- cystic medial necrosis (Marfan, Ehler Danlos )
- osteogenesis imperfecta.
- repair of coarct in Turner.
3. Trauma.
4. Inflammatory;
- infectious; Mycotic (IEC), syphilis, TB.
- Non- infectious; rheumatoid arthritis, takayasu, Giant cell
sero-ve, Behcet, Reiter, ankylosing S.
Site of Atherosclerotic aneurysm; 75% abdominal, 25% thoracic.
Site of dissection; 2/3 ascending A, 1/5 descending.
C/P;
1. Thoracic; asympt, mediastinal compression, signs;
supresternal pulsation.
2. Abdominal; asympt, majority below renal arteries,
signs; abd mass, peripheral ischemia.
Prognosis; Mortality rates;
3% in elective excision.
18% in emergency excision.
50% in acute rupture.
TTT;
1. If < 5 cm, asymtomatic annual U/S, BB to achieve
HT rate < 60b/min, optimal BP control.
2. If > 5 cm abd or > 6 cm thoracic, rapidly expanding,
symptomatic, A thrombosis & peripheral embolism
surgical excision.
NB; in Marfan operate if > 5 cm thoracic.
Aortic dissection
Type A= ascending.
Type B= descending.
Causes; cystic medial necrosis.
1. HTN (70%)
2. Atherosclerosis
3. Congenital; coarct, bicuspid AV, Turner, Noonan.

Sympt;
1. Sudden severe chest pain radiate to the back.
2. Vasovagal manifestations.

Signs;
1. HTN
2. Loss of arterial pulse.
3. Occlusion of important vs (stroke, paraparesis)
4. Compression (mediastinal S).

Complications of ascending; AR, inferior MI, pericardial


effusion, carotid dissection, subclavian pulse.
Invest;
1. CXR; wide mediastinum.
2. TOE (most imp); for diagnosis, severity of AR, LV function,
pericardial effusion.
3. CT does not identify site of tear, AR, coronary involvement.
4. MRI does not allow monitoring, # if prothesis.
5. Coronary angio to assess the need for concomitant CABG.

TTT;
1. Early; releive pain, Na nitroprusside & BB to syst BP<120,
cardiac contractility shear stress of Aorta.
2. Later;
- Ascending surgery.
- Descending
-surgery if; impending rupture, compress major Vs,
uncontrolled pain, continuing dissection, Marfan/ Aortic root> 5cm.
- medical ttt with BB, CaCB & CT/6months if old
dissection>2wks, stable, isolated arch, uncomplicated
descending.
Classification of hypertension
Classification According of BP level:
Normal <120/80
Prehypertension 120/80 139/89
Hypertension >140/90

Staging Hypertension:
Stage 1: 140-159/90-99
Stage 2 :> 160/100

Isolated systolic hypertension:


Grade 1 140-145/ < 80
Grade 2 > 160/ < 80
Causes of hypertension

Essential Secondary

Renal Endocrine Vascular Pregnancy Drugs Diet


Renovascular Hyperaldosteronism Coarctationn See HTN in
Reno-parenchymal Pheochromocytoma pregnancy
Glomerulonephritis Hypo/Hyperthyroidism
Collagen Disease, Cushing
Henoch-Schonlein Purpura,
Chronic Nephritis
Licorice
Contraceptive pills Tyramine rich food
Corticosteroids Salty food
Mineralcorticoids
Sympathomimetics
-nasal decongestants
-appetite suppressants
Phenothiazine
Antidepressants
Cyclosporine
Erthropiotin
Standing
BP should be taking in:
First visit evaluation.
Elderly patients above 60 years.
Diabetic patients.
Patients with postural symptoms.
Patients on potent VD or large doses of
diuretics.
Standing BP should be measured 2
minutes after standing.
Investigations (1st . visit & annually)

Blood ECG
Fundus
Urine exam K+
Bl.sugar
proteinuria Lipid
S.Creatinine
Uric acid
HB
Hypertension Treatment
Life style modification

Stage 1: (140-159 / 90-99)


Start with thiazide or in special situations you can start with ACEI ,
ARB , BB ,CCB .
Stage 2: 160/100
Two drugs combinations
If patient with diabetes or chronic kidney disease you start
with 2 or more antihypertensive drugs to achieve goal BP

After 1 month if target BP is not reached: reach optimum


doses or add additional drug

Inadequate response to antihypertensive drugs


BP < 10/5 after 15 days therapy
Resistant
Hypertension
Office blood pressure >140/90 or 130/80 mm Hg in patients with diabetes
or chronic kidney disease
and
Patient prescribed 3 or more antihypertensive medications at optimal
doses, including if possible a diuretic
Arrhythmias

Premature beats
- loud S1.
- TTT;
reassure if no sympt, young, not
frequent, not multifocal, no structural HT
dis.
if not or failed BB.
Supraventricular tachy;
Atrial rate 150-250.
Types; 1) Atrial tachy (repeated abn. P)
2) MAT
- 3 diff P
- HR >100.
- COPD due to hypoxia.
- TTT= verapamil & TTT of COPD.
- NB; digoxin (arrythmogenic) unless AF.
MAT
3) Reenterant tachy ;
a) AVNRT
- reenterant in or around AVN.
- sudden onset- offset.
- P wave inverted, just before or after or burried.
- TTT; adenosine, carotid sinus massage terminate it.

b) AVRT (in WPW)


1- orthodromic ( retrograde through acc path)Narrow complex.
2- antidromic ( retrograde through AVN) wide complex + delta
wave.
- P wave inverted & some time after complex.
- TTT; amiodarone ( BB, flecainide)
- NB; digoxin & verapamil .
AVNRT
AVRT
Incisional tachy
- PAT.
- scar of corrected ASD.
- gradual onset.
- TTT= propafenone.
Wondering atrial pacemaker;
- as MAT but rate =95.
- physiological.
WPW
, non familial.
Types A tall R in V1 dt LT path.
ECG ( short PR, delta wave, broad complex.
Associations ( MVP, HCM, EBstein, thyrotoxicosis).
TTT; Radiofrequency ablation
if asymp no TTT, can participate in competitive sports.
Ttt of Complications;
- treat narrow complex tachycardia (rate is usually 190) with vagal
stimulation, cardioversion, verapamil or adenosine- same as any SVT.
* AVRT orthadenosine.
*AVRT antidromicamiodarone, flecainide, BB.

- But never treat Acute AF or A flutter with digoxin, verapamil or BB. (


may lead to VT through acc path) DC or lignocaine or procainamide.
but digitalis, verapamil, adenosine, are .
Rt pathway activate
from Rt to Lt from ant to post
-ve in V1

Rt
LT

Lt pathway activate
from Lt to Rt from post to ant
+ve in V1
WPW
Atrial flutter
A. Rate=250-350.
NB; Regular tachy 150/min, narrow complex A.flutter 2 :1
untill proved otherwise.
Best seen in inferior leads.
Uncover by adenosine or massage.
TTT;
- DC ( most likely arrhyth. To respond)
- Class III ( Ibutilide, amiodarone, sotalol)= medical
cardioversion 60 % sinus.
- DC + Ibutilide 100%.
- Radiofrequency ablation 90%.
Low risk of thromboembolism but anticoag if prolonged.
AF

A. Rate=400-600.
Most common, with age.
C/P ( varing S1, absent a, pulsus deficit> 10, with
exercise .
Causes;
- MVD, LA > 4.5 cm.
- IsHD, HTN, LVH,
- ASD, SSS, WPW, a myxoma
- myocarditis, const. peri.,
- PE, pneumonia, hypoxia, br. carcinoma.
- thyrot, alcohol, coffee
- lone, idiopathic
TTT;
- Paroxysmal
if young event monitoring.
if old patient, previous TIA anticoag, INR=2-2.5
+ sotalol or amiodarone.

ttt of the attack


stable unstable

Old> 1 y DC
recent
Anticoag + digoxin
TTE
Atria > 5 cm atria< 5 cm
Anticoag + digoxin
> 2 days < 2 days
Cardioversion
Anticoag 3 wks TEE without anticoag
Cardioversion
then anticoag 3 wks thrombus No thrombus
Anticoag 6 wks Cardioversion without anticoag
then repeat then anticoag
AF
Anticoag 3 wks before & after cardioversion.
after cardioversion Anticoag (most imp) +
amiodarone or sotalol.
Propranolol or verapamil instead of digoxin in
young or hyperdynamic unless Ht failure.
rate control is better than rhythm.
rhythm control mortality.
Pt with slow AF without TTT tend to be chronic.
Risk factors for recurrence after cardioversion = indication for digoxin &
anticoag.
- long duration > 1 y.
- Lt atrium > 5cm
- old age > 75 y
- Rh MVD
- LV impairment.
Risk factors for thromboembolism.(5-7% annually);
- Rh MVD
- DM, HTN
- LV impairment, Lt atrium enlargement.
- previous CVA/TIA.
if anyanticoag at all ages(risk to 1.6%)
If not acc to the age(<65 = aspirin, 65-75= aspirin or warfarin, >75= warfarin)
Digoxin level by erythromycin, thiazides, amiodarone, verapamil, quinidine.
Digoxin in AVB, WPW, HCM, MAT, constrict. Cardio, myocarditis, prior to
elective cardioversion.
VT
1) Monomorphic
Causes ( acute MI, DCM, chronic CAD, HCM,
myocarditis.
Signs (regular pulse, 160, variable S1dt variable
AVV position, abn splitting).
TTT;
if unstable DC.
if stableIV amiodarone (of choice) or lignocaine.
if recurrent pacing.
if post-arrest ICD.
Monomorphic VT
Features favoring VT
Concordance.
P waves.
Capture & fusion beats.
Very broad > 3.5 .
+
History of IsHD.
Variable S1, no decrease in rate with carotid
sinus masage or adenosine.
Must be ttt as VT.
Non-sustained VT
if chronic recurrent

not sustained
sustained ttt only if poor LVF
& structural HT dis

Poor LV funct. Good funct VT induced by


amiodarone + BB VT induced by EPS
ICD exercise test
Or EPS + radiofreq ablation

ICD BB
+/- amiodarone +/- amiodarone

NB;
- if not sure VT/SVT with aberration adenosine diagnostic dose (SVT) &
TTT as VT.
- Sustained VT=> 3 runs of V beats<30 sec.
- CCBs are absolutely BP. Do not ever use verapamil in wide
complex tachy in the emergency setting (30% of those with V tac rapidly
deteriorate)
Avoid verapamil with;
1. AF or a flutter in WPW.
2. Wide complex tachy
3. With BB.

OK verapamil with;
1. AF or a flutter in healthy HT.
2. MAT
3. PSVT (2nd choice ater adenosine)
4. WPW with narrow complex tachy
5. symptomatic ttt of HCM
6. Severe concentric LVH
7. HTN.
VT
2) Polymorphic (Torsade de points):
Causes
- K, Mg, Ca, hypothyroid, hypothermia.
-TCA, antipsychotics (phenothiazines)
- clarithro, erythro, quinolones, cotrimoxasole, Ketoconasole, pentamidine.
- antiarr; amiodarone, sotalol, class 1a.
- IsHD, cong long QT S.
TTT
1- avoid or withdraw class Ia, c, III.
2- IV MG 1 gm or K channel opener.
3- BB ( not sotalol) IV in cong. Forms
4- temporary pacing & isoprenaline ( prevent brady & hence VT).
2ry prevention.
1) congBB & permanent pacing.
ICD if life threatening arrhyth while on BB, FH od death in young.
Lt stellate gangliectomy.
2) acquired ttt of the cause.
NB; Torsade de points is resistant to DC.
-amiodarorone & class I a, c are .
Torsade De pointes
Bradyarrhysmias
SSS.
WPW
AV block;
- signs ( weak S1, variable in CHB, abn splitting, regular a
wave, +/- prolonged V wave)
- TTT;
1) atropine, isoprenaline.
2) ttt of the cause.
3) temporary pacing.
4) permanent pacing ( Mobitz II, CHB with wide complex.
-NB; LT BBB is almost always associated with HT disease.
2nd Degree AV Block, Type I
LBBB and 2nd degree AV Block,
Mobitz Type II
CHB;
- causes;
1) cong, unknown cause, minority due to maternal AID
with anti Ro permanent pacing.
2) acquired; RCA dis or extensive ant MI temporary
& rarely permanent pacing.

NB:
- most common cause of permanent pacing.
- mortality 50% at 1 year esp. if age> 80y or
non rheumatic.
SSS:
1. bradycardia.
2.sinuatrial block.
3. sinus node arrest.
4. Tachybrady syndrome.
5. AF.
Pacemakers
Nomenclature;
1st letter= chamber paced (V, A, D).
2nd letter= chamber sensed (V, A, D).
3 rd letter= pacemaker response to sensed impulse (T, I,
T/I=atrially triggered, vent inhibited)
Uses of temporary pacing.
1) brady asystole, hemodynamic unstable, severe not
responding to drug, post MI CHB, 2nd degree HB.
2) Tachy overdrive termination e.g.A flutter, VT
prevention e.g. torsade.
3) before cardiac intervention to high risk ( LBBB, RCA
angioplasty) vascular surgery, preop to trifascicular.
4) Post MI with trifascicular or RBBB+ LAH or LPH.
Uses of permanent pacing;
1) brady;
- SSS ( symptomatic S arrest > 3 sec or asymptomatic
S arrest > 5 sec)AAI.
- chr AV block, post AV ablation DDD or VVI.
- syncope DDD or VVI.
- post cardiac transplant
2) others; HCM, DCM, long QT.

NB; most common pacemaker is DDD; most physiologic & provides


better exercise tolerance. DDD maintain AV synchrony, + rate
responsive models (R) if young energetics for Chronotropic
incompetence.Most clinicians use DDD, unless the pt is in chronic
slow AF
- AAI are used e.g. in symptomatic sinus brady.
-
Pacemaker S;
- with VVI.
- short of breath at rest, relieved with exertion.
- ECG= retrograde P.
- TTT= DDD.
Pacing in ant. MI; any except MOBITZ1
- 2nd or CHB.
- new bifascicular.
- new RBBB with preexisting 1st, LAH, LPH.
- in LBBB with preexisting 1st AVB.
Pacing in Ht failure cardiac
resynchronization therapy =CRT= bivent.or
multisite pacing.
if - EF<35% with optimal drug therapy.
ICD

Overdrive pacing.
Indications;
- cardiac arrest dt VF,VT.
- sustained VT + LVEF< 40%
- non sustained VT + LVEF< 40%
+ syncope or post MI.
- long QT; post arrest or FH of sudden death.
Antiarrhysmic drugs:
Class effect site use
Class I A Quinidine (- ) Na channels A, V, acc WPW+ AF
Disopyramide AP
procainamide
B Lignocaine (- ) Na channels V Monomorph VT
Phenytoin AP
Mexiletin
C Flecainide (- ) Na channels A, V, acc AF
propafenone No effect on AP
Class II Propranolol BB nodes Stress, thyroid,
Atenolol sinus, cong long
QT, VT
Class III Amiodarone (- )Na, Ca, K A, acc ,V AF, A flutter
Sotalol channels AVRT+ wpw
Ibutilide plateau VT
Class IV verapamil (- )Ca channels AV node SVT, MAT
Others Adenosine K opener, no(-) AV node SVT, diag of
digoxin inotrop. flutter
Adenosine
Short acting.
IV, 6-12 mg
Uses (diag diff bet VT & SVT, therapy SVT)
Enhanced by disopyramide & inhibited by theophyllin.
Side effect; chest pain, bronkospasm, flushing, hypotension.
Amiodarone
Side effects.
most common after oral neurotoxicity,
most common after IV hypotension.
Lung toxicity (1-10%) IPF, ARDS, BOOP, pl effusion.
others; hepatitis, throiditis, optic neuritis, corneal opacities,
photosensitivity, digoxin & anticoag level, PR, QRS,
QT esp with 1a.
in Torsade de points.
NB; no prophylactic use of lignocaine after MI mortality.
HT failure
EF= 50-70%.
5 ys survival;
65% if EF<40.
95% if EF>50.
Causes of High COP failure.
Diastolic HF;
- preserved syst. function ( >40%).
- ischemia.
- transient in MI , permanent in restrictive cardiomyopathy
& Vent Hypertrophy.
Systolic HF;
EF< 40%.
TTT
1) Physical activity.
2) Diet.
3) Exercise.
4) VD;
NB; ACEI mortality & hospitalization.
of no benefit in RVF.
- obstructive valve lesion.
- in HCM.
5) Digoxin- renin, symp, hospital but not mortality.
- used in Syst HF, S3, severe CHF.
- AVB, asymtomatic HF, acute MI.

6)Diuretics e.g. spironolactone mortality & hospitalization & symptoms.


used in severe CHF.

7) BB ( Meto, Biso, Carvi) mortality & hospitalization & symptoms.


not used in volume overloaded patients.
Syncope
Definition; sudden transient loss of conscious dt cerebral
ischemia.
A) Neurocardiogenic
1) Vasovagaltilt table test, never biting tongue.
2) situational ( cough, micturition).
3) carotid sinus syndrome carotid sinus massage, ttt=DDD
or sympathomimetic, fludro.
4) Pyschgenic (panic, hysteria).

NB; carotid sinus massage;


- 5 sec massage Rt then lt 30 sec apart while pt supine, if
ve repeat in upright.
- +ve= >3 sec asystole, SBP > 50mmHg.
- # in carotid bruit, recent MI (3 m), recent stroke (3m),
previous VT.
B) Orthostatic hypotension
30% of syncope in elderly.
Definition;
HR> 30b/min, SBP>20mmHg, DBP>10mmHg after 3min up.
(n= HR =10 b/min, SBP=3mmHg, DBP= 5mmHg,stabilize in 45 sec.)
causes; - hypovolemia.
- drugs; anti HTN, diuretics.
- addisson.
- autonomic dysf.
- HF or stenotic lesions.
tilt table test,
TTT;
non pharmacological; avoid heavy meals, alcohol, postural
training.
pharmacological; sympathomimetics, MAOI-A, fludro.
C) Neurological
e.g. Migraine, hyperventilation,
vertebrobasilar TIA.
D) Cardiac disorders
at rest e.g. arrhyth, MI.
exertional= mechanical obst e.g. AS, MS,
HCM, dissection,PE, PS, PH, temponade.
positional, ball & valve thrombus, myxomas.
JVP
Normally.
Kussmauls sign.
Absent HJR.
Absent & giant a wave.
Canon a; regular, irregular, regular but not
constant.
Absent x, steep x.
Absent Y, rapid y.
Temponade compress ventricles, constrictive P
compress atria.
Cardiac examination
Apex;
Heaving, thrusting/hyperdynamic, Tapping.
Dyskinetic/displaced, diffuse.
Double apex, pericardial knock.
Parasternal heave, palpable S3.

Diastolic shock.
S1; loud, soft, split, variable.
NB; PAC has long PR & loud S1 as it is premature=short diastole.

S2; loud, soft, wide split, fixed split, reversed split or single S2.
NB;wide split, soft S2=PS.
narrow split, loud S2=PH.
wide split, loud S2=ASD.
Murmurs
Systolic murmurs over Apex, A, P.
Diatolic murmurs over Apex, A2.
Continuous murmurs.
NB; all murmurs by standing exept MVP &
HCM.
- Effect of respiration.
HT transplantation
Survival; 80% at 1 yr.
75% at 3 yrs.
505 at 10 yrs
Complications;
- accelerated coronary small vs dis.
- lymphoma, skin cancer
- CRF dt cyclosporin.
Indications; 44% CHD, CM, myocarditis.

NB; in myocarditis, transplantion does not worse the


prognosis but may recur in the transplant.
ECG & arrhythmias
Comment on;
1) Rhythm.
2) Rate
3) Axis
4) P wave
5) PR
6) QRS
- R wave progression
- amplitude.
- duration.
- Pathological Q.
7) ST.
8) T wave.
9) QT.
10) U wave.
Normal adult 12-lead ECG
1) Rhythm
Regular;
- sinus (+ve P in II, -ve P in avR)
- nodal ( no P or -ve P in II, +ve P in avR).
- idoventricular (regular brady, broad
complexes, variable PR).
Irregular
- AF,
- extrasystole,
- respiratory sinus arrhyth.
Sinus Rhythm
Atrial Premature Beat
Ventricular premature beats
Long QT interval
Atrial Bigeminy
Ventricular bigeminy
There are also features of an acute inferior myocardial
infarction.
Atrial fibrillation
Atrial fibrillation. Right axis
deviation.
2) Rate
if regular- 300/n. of large bet R-R.

N of complexes in 3 sec x 20 or in 2
sec x 30 or 6 sec x 10 (1 sec=5 )
if irregular.
3) Axis
N I II III avR avF

Rt axis

LT r
S
Confirm r
LT
S

LAH
RBBB, old inferior MI
left anterior hemiblock ,First degree heart block
Left Anterior Fascicular Block
RBBB + LAFB(M pattern in v1,v2+lt
axis deviation)
Bifascicular Block: RBBB + LAFB
RBBB + LPFB(M pattern in
v1.v2+Rt axis deviation)
NB; Trifascicular
LAH + RBBB + long PR.
Rt axis(LPH) + RBBB + long PR.

Bifascicular
LAH + RBBB
Rt axis(LPH) + RBBB
Trifasicular' block
Complete Right Bundle Branch Block
Left Anterior Hemiblock(lt axis deviation)
Long PR interval
4) P wave
N (2.5 x2.5 small )
Mitral
Pulmonal
Biatrial enlargement. in V1 (biphasic
with larger ve deflection in V1)
absent
Right atrial hypertrophy
A P wave in lead II taller then 2.5 mm (2.5 small squares).
The P wave is usually pointed.
P mitrale
5) PR
N (3-5 )
Short (WPW, nodal, LGL).
Long ( 1st degree Ht block).
Variable ( CHB, Mobitz I)
WPW
Lown Ganong Levine Syndrome
6) QRS(red flag for tall R wave in
v1)
1. R wave progression

V1 V2 V3 V4 V5 V6

Causes of tall R in V1; RVH, RBBB, dextrocardia,


WPW type A,
Posterior MI.
Posterior MI and Right
Ventricular MI
2. Amplitude;
N
LVH S in V1 + R in V6> 7.
S+ R in any 2 chest leads > 5 .

+/- inverted T in lat leads (strain)

RVH tall R in V1 & deep S in V6.


NB:the sum of any tall R&deepS express VH
Left ventricular hypertrophy (LVH)
LVH with "Strain"
3. Duration
n<2.5 .
Causes of wide complexes;
- BBB
- WPW
- vent beats
- pacemaker
- hyperkalemia.
RBBB(when seen,detect the axis)
r
R
in V1

+ depressed ST & inverted T in Rt leads.


(secondary T wave invertion)

- RBBB + RAD or LAD=bifascicular.


RBBB With ST-T Wave
Abnormalities
Right Bundle Branch Block
LBBB
in V1, in V6.
- no R.
- T against QRS.(pseudo ischemia in
V1,T wave upward &Q wave
downward)
NB; true ischemia T same=ischemia.
ie:(inverted T with pathological Q)
LBBB
Acute inferior myocardial infarction in
the presence of left bundle branch block
Ventricular
Pacemaker
Rhythm
Broad complex.
Pacemaker; pacing spikes (best seen here in V4 - V6)
will be seen - they may be subtle
the paced QRS complexes are abnormally wide
Wolf-Parkinson-White
syndrome left lateral pathway
WPW(delta wave &slurred upstrok
of R wave)
Hyperkalemia
Brady
Absent P
Broad complexes
Hyperacute T
Depressed ST
Hyperkalaemia
4. Pathological Q ( > R)
normal Q only in avR & V5.
differential diagnosis;
- transmural infarction(old).
- LBBB.(psudo infarction in case of
upward T wave or T-same if T wave
inverted in case of MI)
- WPW.(slurred upstrok of R wave)
7) ST
Raised ST;
- transmural infarction (>2 in chest leads, >1 in limb
leads).
- pericarditis.(also there is PR elevation &ST depression in
aVR)
- LBBB.
- Depressed ST;
- NST MI, angina.
- digitalis.the concavity of ST is directed upward
- RBBB.V1& V2
- post. MI(in V1)
DD of ST changes
Raised ST;
- transmural infarction (>2 in chest leads, >1 in limb leads), with
evolutional changes & reciprocal depression.
- prinzmetal angina; transient without evolutional changes or
reciprocal depression.
- early repolarization pattern; young, stable ST elevation without
evolutional changes or reciprocal depression.
- pericarditis; special pattern, in all leads.
- LBBB; with characteristic QS pattern in V1,2 & notched R in V5,6.
- Depressed ST;
- angina.
- LVH with strain.
- non-Q MI=NSTEMI= subendocardial infarction.
- reciprocal depression in transmural infarction.
- digitalis; scooping of ST-T.
- Post. Wall infarction.
Acute inferior myocardial infarction
ST elevation in the inferior leads II, III and aVF
reciprocal ST depression in the anterior leads
Acute anterior myocardial infarction
ST elevation in the anterior leads V1 - 6, I and aVL
reciprocal ST depression in the inferior leads
Old anterior MI
Old inferior myocardial infarction
Acute posterior myocardial infarction
(hyperacute) the mirror image of acute injury in leads V1 - 3
(fully evolved) tall R wave, tall upright T wave in leads V1 -3
usually associated with inferior and/or lateral wall MI
inf-post-
Pericarditis(rasised ST in the chest
leads &raised PR WITH depressed
ST in aVR)
Digitalis effect
shortened QT interval
characteristic down-sloping ST depression, reverse stick appearence, (shown
here in leads V5 and V6)
Digitalis effect
8) T wave
Hyperacute Hyperkalemia.
Flat Hypokalemia.
Inverted ischemia.
DD of T wave inversion
Normally in avR, V1.
Ischemia.
Ventricular strain pattern.
Pericarditis.
Subarachnoid Hge (deep, wide +
prolonged QT, U wave), may be due
autonomic dysf.
BBB.
Ventricular Pacemaker.
9) QT
> RR = prolonged;
In;
- Congenital;
Jerwell-Lange-Nielson (AR,deafness).
Romano ward (AD)
- Electrolytes (Hypokalemia, hypocalcemia, hypoMg,
hypothermia, hypothyroid).
- Drugs ; antiarrhyth (Ia as quinidin &disopyramid, III
as amiodaron &d-sotalol), antibiotics ( macrolids,
quinolones, quinine, sutrim, antifungal, pentamidine
- Subarachnoid Hge.
- MVP, Ischemia, Rh carditis.
Long QT Interval
Long QT Interval
10) U wave
Hypokalemia

NB; Hypokalemia;
- prolonged QT.
- flat T.
- U wave.
Hypokalemia
Bradyarrhythmias

Irregular
Regular If Absent P=slow AF.
const PR =Mobitz II
Variable PR=Mobitz I

Variable PR Fixed PR=


= CHB -sinus.
-Nodal. no P wave.
2nd Degree AV Block, Type I(progressive
prolongation of PR interval)
LBBB and 2nd degree AV Block, Mobitz Type
II(dropped complex &not prececeded by progressive
PR prolongation+M-shaped broad complex)
RBBB plus Mobitz II 2nd Degree AV Block(M-shaped R
wave,depressed ST segment &inverted T wave)plus
every 2 P waves followed by one complex
Trifascicular Block: RBBB, LAFB, and Mobitz II 2nd
Degree AV Block(M-pattern in V1&rS pattern in II so
LT axis deviation and LAFB&1 P:2R in both II&V1)
Mobitz II 2nd Degree AV Block
With LBBB
Complete Heart Block(regular bradycardia
with variable PR interval in II)
Sinus bradycardia
Acute inferior MI(receprocal changes seen in I&aVL) and Right Bundle
Branch Block(M-pattern in V1,V2 with depressed ST &inverted T) are
also present
SSS:
1. bradycardia.
2.sinuatrial block.
3. sinus node arrest.
4. Tachybrady syndrome.
5. AF.
Tachyarrhythmias

Broad
Narrow -VT
-SVT + aberration
-WPW + AF or SVT

Regular
Irregular -Sinus tachy
= rapid AF -SVT
-A flutter 2:1 block
Sinus tachycardia
AVNRT
AVRT(WPW)short PR &delta wave
MAT
Atrial fibrillation with pre-existing left bundle
branch block(absent P in II&M pattern in V5,V6)
Atrial flutter with 2:1 AV conduction
Atrial flutter
Broad Complex tachy.
VT
SVT + aberration
WPW + AF or SVT
Wolf-Parkinson-White
syndrome with atrial fibrillation
Ventricular tachycardia
Monomorphic VT
Features favoring VT
Concordance.(the same polarity of QRS coplex in all
chest leads)
P waves.(AV dissociation may result in visible P which
appear to march through the tachycardia)
Capture(intermittent narrow QRS complex owing to
normal ventricular activation through AVN and
conducting system) & fusion beats.
Very broad > 3.5 .
+
History of IsHD.
Variable S1, no decrease in rate with carotid sinus
masage or adenosine.
Must be ttt as VT.
Polymorphous ventricular tachycardia
(Torsade de pointes).

This is a form of VT where there is usually no difficulty


in recognising its ventricular origin.
wide QRS complexes with multiple morphologies
changing R - R intervals
the axis seems to twist about the isoelectric line
Torsade De pointes
Ventricular fibrillation
bizarre, irregular, random waveform
no clearly identifiable QRS complexes or P waves
wandering baseline
Acute pulmonary embolus(SIQIIITIII
&poor progression of R wave &strain
pattern in chest leads&RBBB in V1)
The following, often transient, changes
may be seen in a large pulmonary
embolus.
an S1 Q3 T3 pattern
a prominent S wave in lead I
a Q wave and inverted T wave in lead III
sinus tachycardia
T wave inversion in leads V1 - V3
Right Bundle Branch Block
Arrhythmias

Premature beats
- loud S1.
- TTT;
reassure if no sympt, young, not
frequent, not multifocal, no structural
HT dis.
if not or failed BB.
Supraventricular tachy;
Atrial rate 150-250.
Types; 1) Atrial tachy (repeated abn. P)
2) MAT
- 3 diff P
- HR >100.
- COPD due to hypoxia.
- TTT= verapamil & TTT of COPD.
- NB; digoxin (arrythmogenic) unless AF.
3) Reenterant tachy ;
a) AVNRT
- reenterant in or around AVN.
- sudden onset- offset.
- P wave inverted, just before or after or burried.
- TTT; adenosine, carotid sinus massage terminate it.

b) AVRT (in WPW)


1- orthodromic ( retrograde through acc path)Narrow
complex.
2- antidromic ( retrograde through AVN) wide complex +
delta wave.
- P wave inverted & some time after complex.
- TTT; amiodarone ( BB, flecainide)
- NB; digoxin & verapamil .
Incisional tachy
- PAT.
- scar of corrected ASD.
- gradual onset.
- TTT= propafenone(class Ic).
WPW
, non familial.
Types A tall R in V1 dt LT path.
ECG ( short PR, delta wave, broad complex).
Complications;
- AVRT orth(narrow complex tachy)adenosine.
- AVRT antidromic(broad complex)amiodarone,
flecainide, BB.
- AF or flutter ( may lead to VT through acc path)
DC or lignocaine or procainamide.
but digitalis, verapamil are .
Associations ( MVP, HCM, thyrotoxicosis).
TTT; Radiofrequency ablation
if asymp no TTT, can participate in competitive sports.
WPW
Atrial flutter
A. Rate=250-350.
NB; Regular tachy 150/min, narrow complex A.flutter 2 :1
untill proved otherwise.
Best seen in inferior leads.
Uncover by adenosine or massage.
TTT;
- DC ( most likely arrhyth. To respond)
- Class III ( Ibutilide, amiodarone, sotalol)= medical
cardioversion 60 % sinus.
- DC + Ibutilide 100%.
- Radiofrequency ablation 90%.
- atrial pacing.
Low risk of thromboembolism but anticoag if prolonged.
AF
A. Rate=400-600.
Most common, with age.
C/P ( varing S1, absent a, pulsus deficit> 10)
Causes;
- MVD, LA > 4.5 cm.
- IsHD, HTN, LVH,
- ASD, SSS, WPW, a myxoma
- myocarditis, const. peri.,
- PE, pneumonia, hypoxia, br. carcinoma.
- thyrot, alcohol, coffee
- lone, idiopathic
TTT;
- Paroxysmal
if young event monitoring.
if old patient, previous TIA anticoag, INR=2-2.5
+ sotalol or amiodarone.

ttt of the attack

stable unstable
DC
TTE

Atria > 5 cm atria< 5 cm


Anticoag + rate controlling
> 2 days < 2 days
Cardioversion
Anticoag 3 wks TEE without anticoag
Cardioversion
then anticoag 3 wks thrombus No thrombus
Anticoag 6 wks Cardioversion without anticoag
then repeat then anticoag
AF
Anticoag 3 wks before & after cardioversion.
after cardioversion Anticoag (most imp) +
amiodarone or sotalol.
Chronic AF
Propranolol or verapamil instead of digoxin in young
or hyperdynamic unless Ht failure.

rate control is better than rhythm.


rhythm control mortality.

Pt with slow AF without TTT tend to be chronic.


Risk factors for recurrence after cardioversion =
indication for digoxin & anticoag.
- long duration > 1 y.
- Lt atrium > 5cm
- old age > 75 y
- Rh MVD
- LV impairment.
Risk factors for thromboembolism.(5-7% annually);
- Rh MVD
- DM, HTN
- LV impairment, Lt atrium enlargement.
- previous CVA/TIA.
if anyanticoag at all ages(risk to 1.6%)
If not acc to the age(<65 = aspirin, 65-75= aspirin or
warfarin, >75= warfarin)
Digoxin level by erythromycin, thiazides, amiodarone,
verapamil, quinidine.
Digoxin in AVB, WPW, HCM, MAT, constrict. Cardio,
myocarditis, prior to elective cardioversion.
VT
1) Monomorphic
Causes ( acute MI, DCM, chronic CAD, HCM,
myocarditis.
Signs (regular pulse, around 160, variable S1
due to variable AVV position).
TTT;
if unstable DC.
if stableIV amiodarone (of choice) or lignocaine.
if recurrent pacing.
if post-arrest ICD.
Non-sustained VT
if chronic recurrent

not sustained
sustained ttt only if poor LVF
& structural HT dis

Poor LV funct. Good funct VT induced by EPS


ICD amiodarone + BB
Or radiofreq ablation

ICD
+/- amiodarone
NB;
- if not sure VT/SVT with aberration adenosine diagnostic dose (SVT) &
TTT as VT.
- Sustained VT=> 3 runs of V beats<30 sec.
- CCBs are absolutely BP.
if unstable DC.
if stableIV amiodarone (of choice) or lignocaine.
if recurrent pacing.
if post-arrest ICD.
VT
2) Polymorphic (Torsade de points):
Causes
-familial 90%
- K, Mg, Ca, hypothyroid, hypothermia.
-TCA, antipsychotics (phenothiazine)
- clarithro, erythro, quinolones, cotrimoxasole, Ketoconasole,
pentamidine.
- antiarr; amiodarone, sotalol, class 1a.
- IsHD, cong long QT S.
TTT
1- avoid or withdraw class Ia, c, III.
2- IV MG 1 gm .
.
2ry prevention.
BB ( not sotalol) IV in cong. Forms.
Lt stellate gangliectomy
ICD if life threatening arrhyth while on BB, FH of death in young.

.
2) acquired ttt of the cause.
NB; Torsade de points is resistant to DC.
-amiodarorone & class I a, c are .
The recommendations guidelines:
Lifestyle modification was recommended for all patients
. Beta blockers Beta blocker therapy was recommended for patients with QT
prolongation and suggested for patients with a molecular diagnosis of congenital
LQTS but a normal QT interval. Beta blockers are a mainstay of therapy in both
asymptomatic and symptomatic patients with congenital LQTS, since they reduce
both syncope and SCD [29] . Beta blockers interrupt the "trigger" for TdP and
may shorten the QT interval by decreasing activation from the left stellate
ganglion.

ICD implantation was recommended for survivors of a cardiac arrest who have a
reasonable expectation of survival with a good functional status for at least one
year . ICD implantation was suggested for patients who experience sustained VT
and/or a syncopal event consistent with a tachyarrhythmia while on beta blocker
therapy.

Beta blocker therapy should be initiated or continued in all patients who receive
an ICD.

The guidelines also stated that evidence was less well established for the
following: Left cardiac sympathetic denervation for patients with syncope, TdP, or
cardiac arrest while receiving beta blockers [58] . ICD implantation combined with
beta blocker therapy for patients in categories at an increased risk of SCD (eg,
LQT2 or LQT3)
Bradyarrhysmias

SSS.
WPW

AV block;
- signs ( weak S1, variable in CHB, abn splitting, regular a
wave, +/- prolonged V wave)
- TTT;
1) atropine, isoprenaline.
2) ttt of the cause.
3) temporary pacing.
4) permanent pacing ( Mobitz II, CHB with wide
complex.
-NB; LT BBB is almost always associated with HT disease.
CHB;regular bradycardia with variable PR
- causes;
1) cong, unknown cause, permanent
pacing.
2) acquired; RCA dis or extensive ant MI
temporary & rarely permanent pacing.

NB:
- most common cause of permanent pacing.
- mortality 50% at 1 year esp. if age> 80y or
non rheumatic.
Pacemakers
Nomenclature;
1st letter= chamber paced (V, A, D).
2nd letter= chamber sensed (V, A, D).
3 rd letter= pacemaker response to sensed impulse (T, I,
T/I=atrially triggered, vent inhibited)
Uses of temporary pacing.
1) brady asystole, hemodynamic unstable, severe not
responding to drug, post MI CHB, 2nd degree HB.
2) Tachy overdrive termination e.g.A flutter,
VT prevention e.g. torsade.
3) before cardiac intervention to high risk ( LBBB, RCA
angioplasty) , preop to trifascicular.
4) Post MI with trifascicular or RBBB+ LAH or LPH.
Uses of permanent pacing

1) brady;
- SSS ( symptomatic S arrest > 3 sec or asymptomatic
S arrest > 5 sec)DDD.
- chr AV block DDD or VVI.
- Neurocardiogenic syncope DDD or VVI.
- post cardiac transplant
2) others; HCM, DCM, long QT.

NB; most pacemakers are VVI.


- AAI are used e.g. in symptomatic sinus brady.
- DDD maintain AV synchrony, + R if young.

Chronotropic incompetence mainly in young energetics rate


responsive models (R).
Pacemaker S;
- with VVI.
- short of breath at rest,
- ECG= retrograde P.
- TTT= DDD.
Pacing in ant. MI;
- 2nd or CHB.
- new bifascicular.
- new RBBB with preexisting 1st, LAH, LPH.
- in LBBB with preexisting 1st AVB.
Pacing in Ht failure
if - class 3, 4.
- QRS > 130.
- EF<35% with optimal drug therapy.
ICD
Overdrive pacing.
Indications;
- cardiac arrest dt VF,VT.
- sustained VT + LVEF< 40%
- non sustained VT + LVEF< 40%
+ syncope or post MI.
- long QT; post arrest or FH of sudden
death.
Antiarrhysmic drugs:
Class effect site use
Class I A Quinidine (- )Na channels A WPW+ AF
Disopyramide AP V
procainamide
acc

B Lignocaine (- )Na channels V Monomorph VT


Phenytoin AP
Mexiletin

C Flecainide (- )Na channels A AF


propafenone No effect V
acc

Class II Propranolol BB nodes Stress, thyroid,


Atenolol sinus, cong long
QT, VT

Class III Amiodarone (- )Na, Ca, K channels A AF, A flutter


Sotalol plateau acc AVRT+ wpw
Ibutilide V VT

Class IV verapamil (- )Ca channels AV node SVT, MAT

Others Adenosine K opener, no(-) AV node SVT, diag of


digoxin inotrop. flutter
Adenosine
Short acting.
IV, 6-12 mg
Uses (diag diff bet VT & SVT , therapy SVT)
Enhanced by disopyramide & inhibited by theophyllin.
Side effect; chest pain, bronchospasm, flushing, hypotension.
Amiodarone
Side effects.
most common after oral neurotoxicity,
most common after IV hypotension.
Lung toxicity (1-10%) IPF, ARDS, BOOP, pl effusion.
others; hepatitis, throiditis, optic neuritis, corneal opacities,
photosensitivity, digoxin & anticoag level, PR,
QRS,
QT esp with 1a.
in Torsade de points.
NB; no prophylactic use of lignocaine after MI mortality.
Respiration
PFTs
1. PEFR (Peak exp flow rate); L/min for airway obst in
asthma.
2. Spirometer; Flow rate.
Forced expiratory volume in one second (FEV1)
and forced vital capacity (FVC), FEV1/ FVC = 70-
80%.
obstruct FEV1/ FVC < 70%.
Restrict FEV1/ FVC preserved , FVC
3. lung volumes
- total lung capacity (TLC),
- residual volume (RV).
Decreased lung volumes suggest restrictive disease
if accompanied by a normal FEV1/FVC ratio.
4. Gas transfer; Measurement of diffusing capacity
for carbon monoxide (DLCO) assesses gas
exchange.
restrictive disease + DLCO intrinsic lung disease,
restrictive disease + normal DLCO a non-pulmonary
cause of restriction. (Extrinsic lung disease).
Markedly DLCO + obstructive emphysema,
NB: KCO = DLCO/ VA (alveolar volume = Gas transfer
corrected for lung volume.
KCO-emphysema, ILD, pulm vascular as Pulm HTN, emboli.
KCO pulm hge, polycythemia, Lt to Rt shunt, exercise, Br asth.
Flow-volume loops can identify upper
airway obstruction, which can be
impossible to detect from standard FVC
measurements.
Interpretation of DLCO

Low DLCO with obstruction


Emphysema
Cystic fibrosis
Bronchiolitis

Low DLCO with restriction


Interstitial lung disease
Pneumonitis

Increased DLCO
Polycythemia
Pulmonary hemorrhage
Left-to-right intracardiac shunting
Respiratory failure
Inability to maintain adequate oxygenation &
CO2 excretion.
Type 1; O2 with normal CO2, in restrictive
dis ( diffusion defect). (neuro, muscular,
alveolar), ttt by O2 in inspired air.
Type 2; O2 & CO2, in obstructive dis, ttt
by low flow O2 to maintain hypoxic drive,
doxapram, may need mechanical ventilation
Age related changes of PFTs
Decrease in
FEV1
VC
Po2
Increase in
RV

NO increase of Pco2
Obstructive sleep apnea
C/P; snoring, AM headaches, unrefreshing
sleep, day time drowsiness.
O/E; systemic/pul HTN, ventricular arrhythmias,
LL oedema 2ry to pul HTN/cor pulmonal.
ABG; pO2, pCO2.
Polycythemia 2ry to pO2.
RVF LVF in late stages.
TTT nasal CPAP.
CONTINUOUS POSITIVE AIRWAY PRESSURE
Continuous positive airway pressure (CPAP) functions as a pneumatic splint to
maintain upper airway patency throughout all phases of sleep breathing. It
operates by means of a flow generator which delivers pressure through air
tubing to a nasal or face mask worn overnight. Most patients require lifelong
Asthma
Def; chronic inflammatory disorder of the
airways ccc by variable reversible airway obst
with hyperresponsiviness to a variety of stimuli
leading to recurrent attacks of wheezes, SOB,
cough.
Precipitating factors; drugs (NSAIDs, BB),
infection, cigarettes, fumes, perfumes, pets,
pollen, grass, exercise, cold weather, humidity.
C/P; sympt= variable reversible wheezes,
cough, SOB.
signs=tachypnea, hyperinflation, exp rhonchi.
Investigations
> 25% PEFR variability on at least 3 days per week for 2
weeks.
> 15% PEFR improvement after BD.
Skin prick test Type I hypersens, IgE.
Spirometer normal ---but on metacholine challenge test,
FEV1by >20%. obstructive---BD-- FEV1by 12-15%.
lung volumes.
FEV1/FVC< 70%.
TTT;
- Cough variant asthma inhaled steroids (Budesonide)
- exercise variant asthma diag. by spirometer before & after
TTT; preB2, leukotriene recep antagonist., pre exercise
warming
- TTT of acute severe asthma nebulized Ipratropium &
salbotamol, O2, steroids30-40mg. Mg sulphate single dose.
Acute severe asthma.
Signs; 1- speech impairment.
2- pulse >110 b/min.
3- RR> 25 br/min.
4- PEFR 33-50%predicted.
TTT; high flow O2(40-60%), Nebulised Ipratrobium (0.5 mg 4-6 hourly)
, Nebulised salbutamol, steroids tablets (40-50 mg /day prednisolone)or(100 mg
/6hrs hydrocortisone) for at least 5 days, Single dose magnesium sulfate +/-
Ab.
Life threatening asthma.
Signs; 1-confusion , exhaustion.
2- brady <60b/min.
3- BP.
4- silent chest , cyanosis .
5-O2, n or CO2, HCO3 is considered if base def > 8.
6- PEFR < 33%predicted.
Do ABG & consult ICU
NB; ABG done if O2 sat < 92%.
CXR is necessary to exclude pneumothorax.
A 25-year-old gentleman, known asthmatic , presents with an acute
dyspnea. He is usually well controlled on inhaled fluticasone and
salbutamol inhaler. The patient admits to have been getting progressively
short of breath over the last few days.

Arterial blood gases on air show:


PaCO2 = 3.8 kpa
PaO2 = 7.5 kpa
pH = 7.49
Standard bicarbonate= 23 mmol/L

The casualty officer starts the patient on high flow oxygen and
continuous nebulised salbutamol. He also gives the patient 200 mg of
hydrocortisone IV and one dose of nebulised ipratropium bromide. No
improvement is seen after 30 minutes of treatment.
BTS Guidelines on the Management of chronic
Asthma
Step I inhaled short acting B2 agonist on demand.

Step II Add inhaled steroid 200-800 mcg/d (small


dose)

Step III Add inhaled long-acting B2 agonist .

Step IV as before + inhaled steroid up to 2000


mcg/d (large dose) or leukotriene receptor antagonist
or SR theophylline or B2 agonist tablet

Step V Use daily steroid tablet in lowest dose


occupational asthma
asthma induced or aggravated by work.
The highest groups at risk include:
spray painters (isocyanate)
plastic makers (resins)
farmers (grain dust)
bakers (flour)

Peak expiratory flow rate Serial measurement of


(PEFR) is useful in the investigation and
assessment of occupational asthma (OA) .
Monitoring is carried out by recording PEFR at
least 4 times per day for a period of at least 2 weeks
at work and during a similar period away from work;
COPD
Definition;
Chronic slowly progressive dis ccc by airway
obst that does not markedly change over
several months.
Types;
Chronic bronchitis, Asthmatic bronchitis.
Emphysema.
Bronchiectasis.
Chronic bronchitis chronic cough
&expectoration for most of the days, for at
least 3 m, for 2 consecutive yrs.
Emphysema; permanent enlargement of
the airways distal to terminal bronchioles.
Causes of COPD;
Smoking, dust, air pollution (cadmium
emphysema, coal, cotton, cement),
1 antitrypsin def emphysema
Diagnosis The presence of airflow obstruction
should be confirmed by performing post-
bronchodilator spirometry
- FEV1< 80% predicted, mild= 50-80%
moderate 30-50%
severe < 30%
- FEV1/FVC< 70%
- Large lung volumes.
Differentiating COPD from
asthma
Clinical features COPD Asthma
Smoker or ex-smoker Nearly all Possibly
Symptoms under age 35 Rare Often
Chronic productive cough Common Uncommon

Breathlessness Persistent and Variable


progressive
Night time waking with Uncommon Common
breathlessness and or wheeze

Significant diurnal or day to uncommon Common


day variability of symptoms
[2004]
Differentiating COPD from
asthma: 2
If diagnostic uncertainty remains, the following
findings should be used to help identify asthma:
FEV1 and FEV1/FVC ratio return to normal with drug
therapy
a very large (>400ml) FEV1 response to
bronchodilators or to 30mg prednisolone daily for 2
weeks
serial peak flow measuremenst showing significant
(20% or greater) diurnal or day-to-day variability.
TTT COPD
The most imp. Is smoking cessation.
Br.dilators ,Ipratrobium bromide ,B2 agonists, oral
theophyllin
Steroids 30 mg /d only for responsive.
LTOT survival.
Lung transplant does not survival, only improve
quality of life
TTT of acute exacerbation
NB; should not receive more than 24-28% O2
without ABG monitoring, keeping PO2=50-60.
Managing stable COPD: Promote
effective inhaled therapy
In people with stable COPD who remain breathless
or have exacerbations despite using short-acting
bronchodilators as required, offer the following as
maintenance therapy:
if FEV1 50% predicted: either LABA or LAMA
if FEV1 < 50% predicted: either LABA+ICS in a combination
inhaler, or LAMA

Offer LAMA in addition to LABA+ICS to people with COPD


who remain breathless or have exacerbations despite taking
LABA+ICS,
ICS = inhaled corticosteroid
irrespective of their FEV1 LABA = long-acting beta agonist
2
LAMA = long-acting muscarinic agonist
long-term oxygen therapy (LTOT)
Indications for patients with chronic lung disease
include :
Arterial PO2 (PaO2) < 7.3 kpa, or an arterial oxygen
saturation (SaO2) < to 88 %.
PaO2 < 8 kpa , if there is evidence of cor pulmonale,
or erythrocytosis (hematocrit >55 %).
BENEFITS (LTOT improve any thing) .
LTOT improve outcome measures, including quality
of life, cardiovascular morbidity, depression, cognitive
function, exercise capacity, and frequency of
hospitalization . improved survival among patients
that received LTOT.
Bronchiectasis
Def; Abn. persistent dilatation of bronchi + suppuration.
RT middle lobe S= TB, measles, whooping cough.
Br sicca hemmorragica = Apical =
DD: TB, Friedlander pneumonia (Klebsiella).
Causes;
1. cong;
- ciliary dysf immotile cilia = infert, sinusitis.
kartagner = + dextro, infert, sinusitis.
Youngs = + infert, sinusitis.
- 1ry hypogamaglobulinemia.
- cystic fibrosis.
2. Acquired;
- infection= abs, TB, pneumonia ABPA
- obst = LN, FB, tumor, chr bronchitis..
3. Idiopathic = 60%
C/P; chronic mucopurulent sputum.
Inv;
1. high resolution CT diagnostic in > 90%.
2. CXR.
3. PFTs; obst/restrictive.
4. IG level deficiency of humoral immunity.

TTT;
Most important postural drainage.
Nebulized tobramycin for pseudomonas.
immunization
surgery, transplant.
complications;
- hemoptysis.
- chest pain
- 2ry amyloidosis.
NB; causes of upper lobe bronchiactasis;
- ABPA.
- cystic fibrosis.
- TB.
Cystic fibrosis
common AR, carrier rate (1/25).
CFTR gene on chromosome 7. DF 508
median survival = 35 yrs.
abnormal CL transport across cell membrane;
- in sweat gland failure to reabsorb Cl .
- in airway failure to secrete Cl (thick mucous), ( Mucovisidosis)
C/P;
Pulmonary dis;
colonization by
-children H influenza.
-teenage & adults pseudomonas, ttt by ceftazidime.
upper lobe.
pneumothorax in 10%.
90% of death due to respiratory failure.
GIT;
pancreatic insufficiency in 90%, meconium ileus & adult equivalent S,
pancreatitis, Biliary cirrhosis, gall stones, DM in 1/3, male infertility.
NB;
- CF survival adv from cholera due to abn Cl channels.
Invest;
sweat test; pilocarpine iontopheresis at least 50 gm
collected.
Genetic study.
TTT;
antibiotics nebulized tobramycin, tobramycin, continuous
antistaph.
most important is postural drainage.
pancreatic enzyme supplements
meconium ileus equivalent good hydration,
regular oral gastrografin.
Transplantation; double lung or Ht/lung survival.
Restrictive lung diseases
Extrinsic. (chest wall, pleural, neuromuscular).
Intrinsic. ( IPF, pneumonitis).

PFTs (Lung volumes, vital capacity ,FEV1,DLCO)


Interstitial pulm fibrosis
Causes;
1. 80% cryptogenic (ANA +ve)
2. Collagen dis; SLE, RA & AS.
3. Drugs; (Be tough) Bleomycin, Busulfan,
cyclophosphamide, (not vincristine),
methotrexate, melphalan, amiodarone,
sulfasalazine, Gold, nitrofurantoin.
4. Irradiation
5. occupations
Cryptogenic Fibrosing alveolitis
= idiopathic pul fibrosis
Clinical; elderly men, Dyspnea + 5 C (dry cough, cyanosis,
clubbing, crackles).
CXR / HRCT ---- early (ground glass appearance)
give steroids 30mg / d.
Late (reticulonodular shadowing)
in the periphery & base
ABG= type 1 RF.
PFTs= restrictive.
ANA, RF +ve.
Diagnosis; typical C/P + CT or Lung Biopsy (VATS or open) .
NB: Transbronchial not helpful
5 yr survival = 50%.
TTT; 20% respond to steroids, aza , cycloph , INF (all medical
ttt does not survival), lung rehabilitation, O2 therapy,
single lung transplantation......... survival.
Sarcoidosis
multisystem non caseating granulomatous dis.
granuloma composed of MQ, lymphocytes,
epitheloid cells, multinucleated giant cells.
chronic or acute.
Patients who present with a classical Lofgren's
syndrome of fever, erythema nodosum,
arthralgias, and bilateral hilar lymphadenopathy
do not require biopsy .
Biopsy should be performed in most other
cases of suspected sarcoidosis.
Acute chronic
Lofgrens= fever, arthritis, EN Lung; stage 1; BHL
stage 2; BHL + Pul infil
stage 3; Pul infil
Uveitis, parotid enlargement Ht = restrictive Cardiomyopathy.

Bilateral hilar LN Abd = HSM, LN, renal stones,


granulomatous IN.
Neuro =meningitis, mononeuritis,,bil facial

Locomotor = bone cysts in digits, arthritis

Skin = lupus pernio, nodules

Eye = uveitis, sjogren

Endocrine= Ca, thyroid or pituitarism,


DI. TSH,

Hypergammaglobulinemia
Bad prognostic criteria;
age > 40. Male gender , sympt > 6 months.
Splenomegaly, absence of EN, > 3 organs
Moderate to severe loss of lung function at presentation
stage 3. or A predominance of honeycomb changes on
(HRCT) scan
Diagnosis;
CXR (BHL) + EN
CT chest
pulmonary function tests.
Transbronchial biopsy
24 hrs urine Ca
serum ACE, peripheral blood counts, serum chemistries
-ve tuberculin .
TTT;
stage 1; resolve spontaneous
stage 2; ttt with steroid if progressive
TTT with steroid or other immunosupressives as azathioprine
or methotrexate if Ca, Calcuria, eye, cardiac,neuro .
combination therapy;
azathioprine (2 to 3 mg/kg per day),
low-dose glucocorticoids (usually less than 20 mg / d).
high-dose acetylcysteine (600 mg administered as
effervescent tablets orally three times per day).
Assess response to treatment after 3-6 months,
reassess every 3 months
continue treatment for 1-2 year.
LTOT.
early referral for lung transplantation evaluation, even
before the outcome of medical therapy has been
determined (Grade 2C).
Occupational IPF

Inorganic dust Organic


=Allergic pneumonitis
Extrinsic allergic alveolitis=
1. Asbestos (, clubbing, Br carc, mesothelioma) Bird fancier (protein)
2. Silicosis (TB, PMF, egg shell) Malt worker (Asp. Clavatus)
3. Coal worker pneumoconiosis Farmer (Hay, micropolyspora fani)
4. Berryliosis ( as sarcoid, BALber lymph. ) Baggasosis (Sacchari)
Mushrom worker
Ventilation pneum or humidifier fever

Other occupational dis due to Inorganic dust ;


Occupational asthma
Bysinosis= Monday fever =asthma without IPF
Asbestos-related disease
Ship builder, fireproof.
Effects;
1. Pl plaques; calcify, not premalignant,
asymptomatic or restrictive.
2. Pl thickening; lung bases, dyspnea, restrictive,
KCO normal.
3. Pl effusions.
4. Asbestosis (clubbing, lower zone fibrosis,
restrictive, KCO.) Untreatable, resp failure
5. Malignancy Br carc, mesothelioma,
6. Br carc by smoking but not mesothelioma.
Silicosis
silicon dioxide.
hard rock miners, civil engineers
associated with TB.
acute dry cough, dyspnea.
chronic nodules, upper lobe, PMF, egg shell
calcification of hilar LN.
no effective TTT, compensatible.
NB; DD of egg shell calcification Silicosis &
sarcoidosis.
Coal worker pneumoconiosis
Simple CWP asymtomatic, no IPF, small
nodules<1.5 cm.
PMF;( progressive massive fibrosis) > 3cm.
- Upper zones
- Cavitate.
- Resp. F.
- PFTs obst/ restrictive
- +ve ANA , +ve Rh F
Berylliosis
prolonged even slight amount beryllium as
high technology electronics, alloys.
chronic interstitial infiltrates affecting
mainly upper lobes +/- bil hilar LN as
sarcoidosis ( non caseating grannuloma)
Beryllium lymphocyte transformation test.
extrinsic allergic alveolitis
Attacks of fever, cough, and dypsnea several hours after
exposure to the agent.
No wheezes.
Clubbing.
With repeated attacks---IPF
Type III hypersensitivity.
Invest.
- no eosinophilia
- CXR-fleeting infiltrates then upper zone fibrosis, no
cavitation, no calcification, non caseating granuloma.
- serum precipitins are non specific, indicate only exposure.
- Increase of immunoglobulins except Ig E.
- RF is +ve
PFTs-- restrictive pattern
TTT: avoidance + Steroids.
lower lung zone fibrosis;
1. Asbestos.
2. Cryptogenic FA.
3. CT dis.
4. Drugs.
any thing else -- upper zone fibrosis

Diseases affecting upper lung zones; examples


1. ABPA
2. Extrinsic Allergic alveolitis
3. Cystic fibrosis
4. Sarcoidosis
5. Silicosis
6. TB
7. PMF
Byssinosis
= Monday fever
cotton dust
no IPF, only asthma.
no VC.
FEV1 during work specially on the first
day.
spontaneous pneumothorax,
In general, treatment depends upon the size of the pneumothorax, the
extent of respiratory distress, and the presence or absence of
underlying lung disease. The goals of treatment are to remove air from
the pleural space and to prevent recurrence.
Observation Observation should be the treatment of choice for
small closed pneumothoraces without signicant breathlessness.
Patients with small (<2 cm) (25 % of the hemithorax )
primary pneumothoraces not associated with breathlessness should be
considered for discharge with early outpatient review. + Supplemental
oxygen: The administration of humidified 100 % oxygen increased the
rate of air resorption 6 times
Aspiration of the pleural space Simple aspiration is
recommended as rst line treatment for all primary pneumothoraces
requiring intervention. [ Evacuation of air is required for pneumothoraces
that occupy (> 2Cm) more than 25 % of hemithorax or for patients with
severe dyspnea, hypoxemia, or significant pain.
Thoracostomy tube Patients who fail aspiration treatment or who
have a recurrent spontaneous pneumothorax thoracostomy tube.
Breathless and/or rim of air>2 cm on chest
radiograph..Aspiration.... If fail ..
Consider repeat aspiration. Intercostal
drain if fail. Referral to thoracic
surgeon
2ry sp. Pneumothorax ---- Breathless
+ age > 50 years + rim of air>2 cm on
chest radiograph I C tube if not try aspiration
Simple aspiration is less likely to succeed in
secondary pneumothoraces and, in this
situation, is only recommended as an initial
treatment in small (<2 cm) pneumothoraces in
minimally breathless patients under the age of
50 years. Patients treated successfully with
simple aspiration should be admitted to hospital
for at least 24 hours before discharge.
A lateral chest or lateral decubitus
radiograph should be performed if the
clinical suspicion of pneumothorax is high,
but a PA radiograph is normal.
Intercostal tube drainage
If simple aspiration or catheter aspiration
drainage of any pneumothorax is unsuccessful
in controlling symptoms, then an intercostal tube
should be inserted. [B]
Intercostal tube drainage is recommended in
secondary pneumothorax except in patients who
are not breathless and have a very small (<1 cm
or apical) pneumothorax. [B]
Chest drain suction
Suction to an intercostal tube should not be applied directly after tube
insertion, but can be added after 48 hours for persistent air leak or failure
of a pneumothorax to re-expand. [B]
High volume, low pressure (10 to 20 cm H2O)suction systems are
recommended. [C]
Patients requiring suction should only be managed on lung units where there
is specialist medical and nursing experience. [C]

Referral to thoracic surgeons


In cases of persistent air leak or failure of the lung to re-expand, the
managing respiratory specialist should seek an early (35 days) thoracic
surgical opinion. [C]
Open thoracotomy and pleurectomy remains the procedure with the lowest
recurrence rate for difcult or recurrent pneumothoraces. Minimally
invasive procedures, thoracoscopy (VATS), pleural abrasion, and surgical
talc pleurodesis are all effective alternative strategies.
Pleural effusion
Exudate Transudate

Criteria > 1 of the following;


Pl fl/serum TP> 0.5.
Pl fl /serum LDH > o.6
Pl fl LDH> 2/3 upper limlt
normal serum

Causes Malignancy Hypoalb


TB CHF, nephrotic, LC
Pneumonia Contrictive pericarditis
PE PE
Meigs iatrogenic
chylothorax
A low pleural fluid glucose concentration (less than
60 mg/dL (3.33 mmol/liter), or a pleural fluid/serum
glucose ratio less than 0.5) narrows the differential
diagnosis of the exudates to the following
possibilities

Rheumatoid pleurisy
Empyema
Malignant effusion
Tuberculous pleurisy

All transudates and all other exudates have pleural fluid


glucose concentration similar to that of blood glucose.
parapneumonic effusion

In general, a parapneumonic effusion should be


sampled if it meets any of the following criteria:
layers >10 mm on a lateral decubitus film.
It is loculated .
It is associated with thickened parietal pleura on
CT scan.
In general, a pH <7.20 or a glucose <60 mg/dL
is an indication for drainage of the effusion
The indications for chest tube insertion in
patients with infected pleural effusions are:
presence of organisms on a Gram stain of
the pleural fluid , a frankly purulent pleural
fluid, pleural pH < 7.2 in the setting of an
infected pleural effusion, loculated pleural
effusions and poor clinical progress
despite antibiotic treatment.
TTT of hepatic hydrothorax
Diuretic & salt restriction.
If resp distress large volume
paracentesis.
TIPSS & liver transplantation.
Vidiothoracoscopy & talc pleurodesis.
But never Tube
Aspergillosis.
Allergic bronchopulmonary aspergillosis
Aspergilloma
Invasive aspergillosis
Allergic bronchopulmonary aspergillosis
The clinical picture of ABPA is dominated by asthma complicated by
recurrent episodes of bronchial obstruction, fever, malaise, expectoration
of brownish mucous plugs, peripheral blood eosinophilia. With fleeting
shadows on CXR.
A skin prick test should be the first step in an asthmatic being evaluated for
ABPA. A negative prick skin test virtually excludes ABPA from
consideration.

A history of asthma
Immediate skin test reactivity to Aspergillus antigens
Precipitating serum antibodies to A. fumigatus
Serum total IgE concentration > 1000 ng/mL
Peripheral blood eosinophilia >500/mm(3)
Lung infiltrates on chest x-ray or chest HRCT
Central bronchiectasis on chest CT
Elevated specific serum IgE and IgG to A. fumigatus.
Pulmonary function testing Most patients have airflow obstruction with
reduced FEV1 and increased residual volume. Individuals with
bronchiectasis or fibrosis may exhibit a mixed obstructive and restrictive
pattern.
glucocorticoid therapy.
Diagnosis is suspected based on history and
imaging tests and confirmed by Aspergillus
skin testing and measurement of IgE levels,
circulating precipitins, and A. fumigatus
specific antibodies.
Treatment is with corticosteroids and, in
patients with refractory disease, itraconazole .
Aspergilloma.
An Aspergilloma usually arises in a preexisting
cavity in the lungs. Any condition that causes
cavitation e.g tuberculosis or cystic fibrosis.

Aspergillomas are masses of fungal mycelia that


grow in preexisting lung cavities.

Haemoptysis is a common symptom.

They do not require treatment with either anti-


fungals or steroids
Pneumonia
Causes of community acquired Pneumonia;
Strept pneumonie
Atypical organisms.
- Mycoplasma
- legionella
- chlamydia
- coxciella
- Others; staph aureus, G ve as H influenza,
viral, anaerobic, fungal (esp in IC,
nosocomial).
Mycoplasma Pneumonia
C/P;
Pneumonia; Walker pneumonia, dry cough, in epidemics.
+ exrapulmonary (4 M, rashes, anemias) .
- Myocarditis, pericarditis,
- meningitis, encephalitis,
- Myalgia , arthralgia.
- Myringitis.
- Erythema nodosum, E multiforms.
- AAHA, thrombocytopenia

Diagnosis; cold aglutinin in 50%, confirmed by rising Ab titre.


TTT=macrolides, Doxy.
Legionella pneumonia
Outbreaks related to contaminated water cooling system,
showers & air conditioners.
C/P; (5 L)
- Lung
- Level of conscious disturbed.
- Liver enzymes, elevated.
- Lytes; hyponatremia dt SIADH.
- Loose motions & ARF.
- Lymphopenia or pancytopenia.

Diagnosis; urine legonella Ag.


TTT= macrolides+/- rifampicine, levofloxacin.
Coxciella
Rickettssia like.
Minimal exposure to dust or milk of farm animals (cattle).
By ticks
C/P;
- acute; fever, pneumonia.
- Chronic; cardiac (myo & endocarditis), hepatitis,
osteomyelitis, uveitis, clubbing.

Diagnosis; Complement fixation test.


TTT; doxyciclin (+ rifampicin or clindamycin in chronic
cases)

7 C cattle, chronic, cardiac, clubbing, CFT, doxyciclin,


clindamycin.
Chlamydia psitassi
C/P
-Birds exposure.
- As coxciella but LFTs, Na (as legionella)

TTT; tetracyclin, Azithromycin


Indication of hospitalizations (Signs of severe pneumonia
(CURB-65 ))

- altered mental state (confusion )


- vital data; BP < 90/60
- RR > 30 B/MIN
- Fever > 39.5
- AF
- age > 65yr, comorbidities.
- Lab; WBC > 30, <4. urea > 7.
- PO2 < 60
Poor prognostic factors;
- PO2< 8 KPa.
- multilobular.
- comorbidities
Suppurative pneumonia
staph au, Klebs, pseudo
Lung abcess, anaerob
Actinomycis, invasive aspergillosis
Lobar pneumonia;
- Strept pn.
- Klebs.
- H infl.
Pneumonia + effusion
- Bact (20% mycopl)
- Fungal.
- Parasitic.
NB; not pneumocystis carinii.
Pneumocystis carinii pneumonia. The history of exercise
induced dyspnoea in a HIV positive male with no signs on
chest X-ray with little other data would suggest PCP until
proved otherwise.
Patients who present with pneumonia after
influenzae or measles are at risk of
Staphylococcal pneumonia. Look for
cavitation on CXR. The BTS
guidelines: state that amoxicillin should be
first line therapy for all pneumoniae, with
addition of flucloxacillin if there is risk of it
being staphylococcal. Gentamicin may be
indicated in severe hospital acquired
pneumonia, rifampicin in severe atypical
infections (e.g. legionella).
TTT
community acquired Pneumonia
Mild amox + erythro
Severe 3rd generation cephalo +/- clarithrom
If risk of staph( after influenza, measles) add fluclox.
Shift to oral after 48 hrs, continue for 7-10 d, 14 d for
G-ve, 21 d for legionella.

Nosocomial = 3 rd cephalosporin.
-Moderatecefotrioxone
-Severe ceftazidine
or Imipinem + genta
COPD, bronchiectasis H influenza,T= cefotrioxone.
Cystic fibrosispseudomonus, TTT= ceftazidine
Aspiration clinda, metronidazole.
Lung Abcess;
Org; Anaerobes,
strept,
staph aureus,
Klebs.
TTT;
- Clindamycin for 6 wks + postural drainage + Xray FU.
- if failed --. Percutaneus drainage.
Empyema;
- In 15% of comm-acq pn. slow to improve, persistant
fever, WBCs.
- Pl fluid PH < 7.2.
- TTT; drainage , IV Ab , intrapleural streptokinase.
if failed decortication.
Pulmonary embolism.

All patients with possible PE should have clinical


probability assessed and documented.
D-dimer
Blood D-dimer assay should only be considered
following assessment of clinical probability.

D-dimer assay should not be performed in those with


high clinical probability of PE.
A negative D-dimer test reliably excludes PE in patients
with low or intermediate clinical probability; such
patients do not require imaging for VTE.
Imaging
CTPA is now the recommended initial lung
imaging modality for non-massive PE.
In patients with coexisting clinical DVT, leg
ultrasound as the initial imaging test is
often sufficient to confirm VTE. [B]
CTPA or echocardiography will reliably
diagnose clinically massive PE. [B]
Thrombolysis is the first line treatment for
massive PE [B] and may be instituted on
clinical grounds alone if cardiac arrest is
imminent [B]; a 50 mg bolus of alteplase is
recommended.
Treatment
Thrombolysis should not be used as first line treatment
in non-massive PE. [B]
Heparin should be given to patients with intermediate or
high clinical probability before imaging. [C]
Unfractionated heparin (UFH) should be considered (a)
as a first dose bolus, (b) in massive PE,
Otherwise, low molecular weight heparin (LMWH)
should be considered as preferable to UFH, having
equal efficacy and safety
and being easier to use. [A]
Oral anticoagulation should only be commenced once
VTE has been reliably confirmed. [C]
The target INR should be 2.03.0; when this is achieved,
heparin can be discontinued. [A]
,
primary pulmonary hypertension presents
mostly after age 30s, and more commonly
affects women. There is dyspnea ,
cyanosis,chest pain & right heart failure .
treatment includes prostaglandin analogues
(iloprost, epoprostenerol), sildenafil, bosentan
and calcium channel blockers.
ARDS

Causes.
Uremia.
Pancreatitis
Infection(viral pneum, pneumocystis,G-ve
sepsis)
Embolism (air, fat, amniotic).
Irritant (gas, O2, drugs; opiates, aspirin
overdose, IV B2 agonist, aspiration,
drowning).
Type I RF
DD Lung cavitation
Cancer
Autoimmune (wegner)
Vascular (PE)
Infection (staph au, Klebs, pseudo,
anaerob, TB, histoplasma, nocardia)
TB
Causes of pulmonary eosinophilia
Extrinsic;
1. Parasitic; Tropical pulmonary eosinophilia immune response to the
bloodborne microfilarial stages of the lymphatic filariae, Wuchereria
bancrofti,
2. Fungal; ALLERGIC BRONCHOPULMONARY ASPERGILLOSIS .

3. MEDICATIONS AND TOXINS NSAIDs and antimicrobials (eg,


nitrofurantoin, minocycline, sulfonamides and ampicillin) are the most
common classes of drugs associated with pulmonary eosinophilia,
Intrinsic;
1. CHRONIC EOSINOPHILIC PNEUMONIA symptoms including cough,
fever, progressive breathlessness, weight loss, wheezing, CEP occurs
predominantly in women and nonsmokers; The chest radiographic finding
of bilateral peripheral infiltrates described as the "photographic negative" of
pulmonary edema is pathognomonic but found in fewer than 1/3 of cases.
2. CHURG-STRAUSS SYNDROME a vasculitic disorder often characterized
by sinusitis, asthma, and prominent peripheral blood eosinophilia
3. Hypereosinophilic syndrome
Tuberculosis
1 ry asymtomatic, EN, mild cough,
wheeze.
Reactivation or reinfection persistant
cough, wt loss, hemoptysis, pneumonia, pl
eff.
Miliary HSM, fever, wt loss, choroidal
tubercles.
Diagnosis of tuberculosis
Clinical presentation
Radiography
Two stage Tuberculin skin testing or Interferon- assays
measure IFN-g released by T cells in response to
stimulation by MBT antigens not present in BCG
ZN staining & sputum cultures by
1) Lowestein Jensen media (LJ)
2) BACTEC technique;
It is a technique used in culture from liquid media, by
measuring radioactive 14 CO2 liberated in the medium, gives
results within 10-14 days, compared to average 24.03 days by
conventional (LJ).
adenosine deaminase test in pleural effusions. typically
greater than 50 U/L in tuberculous pleural effusions, with
a sensitivity of 90 to 100% and a specificity of 89 to 100%.
Bronchoscopy & Biopsies; pl, LN, liver, BM.
PCR test; urine, pl, sputum, CSF.
a positive test requires:
15 mm----- In people with no known risks for TB.
10 mm----- in people with known risks for tuberculosis
-diabetes or kidney failure, leukemia, lymphoma,
-carcinoma of H & N or lung, silicosis,
-history of gastrectomy or jejunal bypass,
-in health care workers,
- drug abusers,
-recent immigrants from countries with high TB
prevalence).
5 mm----------in people -who have HIV,
-who are taking steroid therapy (>15 mg pred. for 1 month),
-organ transplant recipients,

false-negative test :
-up to 20% of people infected with TB.
- IC (cancer, recent chemotherapy, AIDS) .
-booster phenomenon
A) Definitions of Pulmonary Tuberculosis
1- Smear-positive pulmonary TB
1. 2 sputum +ve for AFB;
2. 1 sputum +ve + radiographic abnormalities
consistent with pulmonary TB.
3. 1 sputum +ve + culture +ve for M. TB.
2- Smear-negative pulmonary TB radiographic
abnormalities + lack of clinical response
despite one week of a broad-spectrum
antibiotic
Treatment of tuberculosis
First-line anti-tuberculous drug
All for 2 m
rifampicin (rifampin ), RMP, 600 mg (10-20 mg/kg) Then R & INH for 4 m
isoniazid , INH, 300 mg (5-10 mg/kg) If P not used 9 m
Ethambutol, EMB, 1 gm (25 mg/kg)
Pyrazinamide,PZA. 1 gm (20-35 mg/kg)
Second-line drugs (SLDs)
(less effective e.g., p-aminosalicylic acid) or, toxic e.g.,
cycloserine; or unavailable in many developing countries
(e.g.,fluoroquinolones):
aminoglycosides: e.g., amikacin (AK), kanamycin;
Streptomycin, STM, 1 gm IM daily for 1-2 m.
polypeptides: e.g., capreomycin,
fluoroquinolones: e.g., ciprofloxacin (CIP), moxifloxacin (MXF);
p-aminosalicylic acid (PAS or P).
NB;
streptomycin active # extracellular org., while PZA active #
intracellular org.
Rifabutin
substitute for RIF for patients who have experienced
intolerance to rifampin.
Rifapentine
Rifapentine may be used once weekly with INH in the
continuation phase .

Side effects;
Idiosyncrasy
NeuropathyINH (so add pyridoxine)
Hepatitis R, INH, PZA.
Optic neuritis ethambutol
Irreversible ototoxicity (esp vestibular) streptomycin.
Gout PZA.
At the beginning of therapy, LN may enlarge dt hypersensitivity,
TTT by steroids
The continuation phase of treatment is given for either 4
or 7 months.
-The 4-month continuation phase should be used in
majority of patients.
-The 7-month continuation phase is recommended only
for three groups:
1- cavitary pulmonary tuberculosis .
2- patients whose initial phase of treatment did not
include PZA;
3- patients being treated with once weekly INH and
rifapentine .
Drug-induced hepatitis

defined as a serum AST level >3 times in


the presence of symptoms,
or >5 times in the absence of symptoms.
INH, RIF, and PZA should be stopped
immediately. 2 or more anti- TB without
hepatotoxicity, such as EMB, SM, amikacin
may be used .

in patients with known liver disease


avoiding PZA + monitored regularly.
Renal disease

1) If GFR between 30-60ml no change use


standard drugs in lower ranges of dose .
2) If GFR <30 1/2 EMB (or avoided). 3/4
PZA dose .
3) on dialysis, daily during the initial phase, at
the end of each haemodialysis session & 3
times /wk in the continuation phase.
Pregnancy
Rifampicin is enzyme inducer for hormonal
contraception.
Untreated TB in pregnancy is associated with an
increased risk of miscarriage and major foetal
abnormality, and treatment of pregnant women should
continue.
The initial treatment regimen should consist of INH,
RIF, and EMB+Pyridoxine supplementation (25
mg/day). For 9 months. avoid Streptomycin
congenital deafness .
avoid PZA.
Breastfeeding should not be discouraged for women
being treated with the first-line antituberculosis agents
because the small concentrations of these drugs in
breast milk do not produce toxicity in the nursing
newborn.
Contact tracing
Tuberculin test
-ve +ve

Repeat after 1 wk Sputum & CXR

+ve
-ve +ve
Repeat after 6 wks
TTT

+ve -ve

-ve chemoprophylaxis

BCG
Preventive chemotherapy
daily isoniazid for 6 month (5 mg/kg) or INH+ R
for 3 m
Target groups for preventive treatment
1. Infants of mothers with PTB
2. Children under 5 years of age
3. HIV-infected individuals
4. for contact with recent tuberculin +ve conversion with
no XR or clinical evidence of TB.

NB; BCG protect against leprosy & ttt of some cancer.


MDR-TB
When to suspect.
Exposure to a person with active TB from areas with high
prevalence of drug resistance.
with active TB with treatment failure or relapse.
Treatment of MDR-TB must be done on the basis of sensitivity
testing: it is impossible to treat such patients without this
information.
If treating a patient with suspected MDR-TB, the patient should
be started on
One of 1st agents drugs to which the pt is partially sensitive .
One injectable agent.
One quinolone.
For one year after culture negative. Or MDR resistant2 yrs.
Pulmonary nodules
Benign Malignant

size < 2.5-3 cm > 2.5-3 cm

Margins smooth Irregular, speculated

calcifications Central/ diffuse, Eccentric, sparse


laminated
growth - +

Satellite lesions - +

CT with C Less likely to enhance more likely to


enhance
PET No uptake +
Bronchogenic carcinoma
Predisposing factors;
A; asbestos, aluminium, arsenic
C; cryptogenic fibrosis, scleroderma, smoking.
Types;
Adenocarcinoma; most common now (30-40%)
Squamous cell carcinoma; was most common,
related to smoking. (20-30%)
Small cell carcinoma; (20%).
Large cell (10%).
Bronchoalveolar carcinoma; (5%).
Adenocarcinoma; (P)
Most common
Slowly growing
Peripheral.
Not smoking dependent
Following asbestos
HPOA
Isolated brain, bone metastasis.

Squamous cell carcinoma; (c, H)


was most common,
related to smoking.
Horner, hyperCa, hyperthyroidism
central, cavitating, clubbing.
Late metastasis.
Pancoast tumor Horner, shoulder pain, waisting of
small ms of the hand.
Small cell carcinoma; (20%).
Central
Rapidly growing
Many paramalignant; SIADH, ACTH, Eaton lambert
SVC obstruction by LN.
Others; polymyositis, PN,cerebellar S, acanthosis
nigricans, erythema gyratum repens.
Large cell (10%) gynecomastia.
Bronchoalveolar carcinoma;
Scleroderma, lung fibrosis.
Spread through bronchial tree
Present as chronic pneumonia (with air bronchogram)
Solitary nodule or diffuse nodular lesions.
mucous secretion , Bronchorrhea.
Suptype of adeno.
Intrathoracic complications
Pressure
Effusion
Destruction; collapse, pneumothorax.
Diagnosis;
Staging;
I - hilar LN.
II + hilar LN.
IIIA ipsilateral med/subcarinal> 2cm from carina
LN, pleura, pericardium, chest wall but not maignant
cells in pleural effusion.
IIIB contralateral or supraclavicular LN or
malignant pl effusion (local invasion; HT, oesoph,
vessels, carina or<2 cm.
IV distant dis.
NB; Horner does not mean extension beyond the lung.
Sputum cytology. A diagnosis may be made
on sputum cytology which should be the
initial investigation with a positive yield of
approximately 70-80% four samples.
TTT
Small cell limited (I-IIIA) chemo + radio
extensive chemo only
Non small I-IIIA surgery
or chemo then surgery
IIIB extensive chemo, radio
Surgery is # if preop. FEV1<2L or expected post-op FEV1
<0.8L.
If SVC obstruction radio, or high dose dexamethazone, or
chemo if small cell. best ttt now is stent.
Prognosis;
Overall 5 yr survival is 15 %.
Small cell, poor prognosis; few months
Mesothelioma
, 50-70 yrs.
85% asbestos ,latent period>30yrs.
Diagnosis; pl biopsy by VATS.
DD; adeno, pl thickening.
TTT; unsatisfactory; drainage, pleurodesis,
radio
Median survival; 1 yr for pleural & 6 month
for peritoneal.
Br adenoma
= br carcinoid
Young, male = female, non smoker.
Central, broad base , highly vascular .
Produce ACTH, GHRH, rarely 5HT.
Complication; hemoptysis, collapse.
Octreotide scan. urinary HIAA.
ttt; resection, resist radio & chemo.
Pulmonary causes of clubbing
ILD; IPF, asbestos
Suppurative lung dis; lung abcess, asbestos, empyema,
bronchiectasis, cystic fibrosis.
Malignant lung dis; Br carc, mesothelioma
not TB
not COPD
Fleeting shadows;
1. ABPA.
2. Chronic esinophilic pneumonia.
Calcifications
1. LN; TB, Silicosis, Sarcoidosis.
2. Pleural; Asbestos plaques, old hemothorax.
3. Parenchymal;
- infections; TB, fungal, old varicella
- Ca; PTH, CRF, Vit D.
- benign tumors.
Bronchiolitis obliterans with organizing
pneumonia (BOOP)
Seen when Bronchiolitis (inflammation &
fibrosis) spread to adjacent parenchymal tissue.
Causes; RA, viral infections, toxic inhalations,
BMT, lung transplantation.
CXR parenchymal infiltrates
PFTs; restrictive pattern. Or mixed.
Ventilatory support
NIPPV (non-invasive +ve pressure ventilation);
- use securely fitting nasal or full face mask & a portable
bilevel pressure support ventilators in general wards.
- require conscious, alert cooperative pt.
NIV should be considered in all patients with an acute
exacerbation of COPD in whom a respiratory acidosis
(pH <7.35, PaCO2 >6 kPa) persists despite immediate
maximum standard medical treatment on controlled
oxygen therapy for no more than one hour.
CPAP (continuous +ve airways pressure):
- use tightly fitting face mask pneumatic splint to the
airway, used in obstructive sleep apnea, may cause
hypotension.
conventional ventilators.
NIV is particularly indicated in:
COPD with a respiratory acidosis pH 7.257.35 (H+4556
nmol/l)
Hypercapnic respiratory failure secondary to chest wall
deformity (scoliosis, thoracoplasty) or neuro-muscular diseases
NIV is not indicated in:
Impaired consciousness
Severe hypoxaemia
Patients with copious respiratory secretions
The benefits of an acute NIV service are likely to be:
Fewer patients referred to intensive care for intubation
Shorter stays on intensive care
Fewer deaths of patients with acute respiratory failure
A 65 year old man has had 5 kg weight loss. He
is an ex smoker and used to work in a coal mine.
Chest X ray shows a large right sided pleural
effusion and several pleural plaques in both lung
peripheries. Pleural aspiration reveals an
exudate. What is the next best investigation?
A.Bronchoscopy
B. Lung function tests
C. Spiral CT of the chest
D. Thoracoscopy and biopsy
E. Sputum for AFB
a malignant effusion., the best option
would be thoracoscopy and biopsy to drain
the fluid as well as confirm a diagnosis
simultaneously. A video assisted
thoracoscopy would help to do this.
A 50 year old Afro-Caribbean man has a cough. He
also has generalised arthralgia. There is an eruption of
tender red areas on his shin. Which of the following is
important to determine whether this condition is self
limiting?
A. Hilar lymphadenopathy and eosinophilia
B. Response to Kveim test
C. Hilar lymphadenopathy and erythema nodosum
D. Serum calcium levels
E. Serum ACE levels
A 35 year old man has significant wheezing and
breathlessness. Recordings of peak flows shows
diurnal variation. He was prescribed with
salbutamol but continues to have frequent
wheezy episodes. What is the next step in
management?
A.Phosphodiesterase inhibitors .
B. Leukotriene antagonists .
C. Oral antibiotics .
D. Inhaled corticosteroids.
E. Oral steroids.
An 18 year old girl with severe difficulty in
breathing to A+E. She has had a history of
asthma with two previous ITU admissions.
Her peak flow is currently 100. She is unable
to say more than a few words. She was
given nebulised salbutamol and iv
hydrocortisone 200mg by the
paramedics.Which treatment should she now
have?
A. Further iv hydrocortisone 200 mg
B. Iv magnesium 2g
C. Intubation and ventilation
D. Iv salbutamol
E. Iv ipatropium
A 35 year old lady with systemic sclerosis has
breathlesness on exertion. She has bilateral basal
crepitations in the chests and corresponding
interstitial shadowing on the CXR. Which is likely to
be found on her lung function tests?
A. P02 of 11 desaturating to 10 on exertion
B. FEV1 to FVC ratio of 65% .
C. Diffusion capacity (DLCO) of 17 (predicted 23)
D. Increased residual volume
E. FEV1 of 5 L
A 45 year old woman has progressive
shortness of breath. On examination she has
O2 sats of 93% on air and finger clubbing was
noticed. In assessing the potential diagnosis,
which one of the following is a typical feature
of cryptogenic fibrosing alveolitis?
A. Dry cough
B. B. Haemoptysis
C. C. Haematuria
D. D. Wheeze
E. E. Abdominal pain
An 13 year old male has recent onset
breathlessness. He wheezes particularly during
the summer when the pollen count is high.
Which of these tests would help to confirm the
diagnosis?
A. Trial of inhaled corticosteroids
B. Trial of anticholinergics
C. Serial peak flow measurements
D. Lung function tests
E. Chest X ray
3. A 25-year-old man presents with right-sided
facial weakness and parotid swelling. There is
a right LMN facial nerve palsy but no other
abnormalities on examination. Lumbar puncture
findings are: Opening pressure18 cm, CSF
Protein 0.9 g/l Glucose3.5 mmol/l, Microscopy
85 lymphocytes. What is the most likely
diagnosis?
A. Lyme disease
B. Multiple sclerosis (MS)
C. Guillain Barre syndrome (GBS)
D. Neurosarcoidosis
E. Ramsey Hunt syndrome
4. A 35-year-old woman complains of repeated painful
rash on her legs. She also gives a 11 month history
of dyspnoea on exertion. Her doctor arranged for her
to have a chest X-ray and this shows bilateral hilar
prominence & basal reticulonodular shadowing
Examination of her legs reveals a purplish-red,
maculopapular tender rash on her shins. Apart from
few bi-basal crepitations on auscultation, systemic
examination is normal. A pulmonary function test
shows a moderately impaired DLCO. Which one of
the following is the best investigation?
A. Bronchoscopy
B. High-resolution CT scan of the thorax
C. Open lung biopsy
D. Kveim test
E. Serum ACE level
5. A 46-year-old woman is admitted to hospital with
a left basal, community-acquired pneumonia.
She is on the appropriate antibiotics. She is still
pyrexial four days after admission and a chest
X-ray confirms a left pleural effusion. The house
officer has performed a diagnostic tap. Which of
the following is an indication to insert a chest
drain?
A. Pleural fluid protein level more than 50% of
serum protein level
B. Pleural fluid LDH more than 60% of serum LDH
C. Haemorrhagic pleural fluid
D. Pleural fluid pH < 7.2.
E. Pleural fluid glucose < 1.6 mmol/l
A 60 year old miner has been in the occupation for 20
years. He presents with a cough and breathlessness.
Chest XR shows diffuse interstitial shadowing. A sputum
sample is positive for acid fast bacilli. Which of the
following dusts is most likely to have predisposed the
patient to tuberculosis?
A. Beryllium
B. Cadmium
C. Coal
D. Silica
E. House dust
19) A 73-year-old male smoker presents with
haemoptysis of 3 weeks duration. Examination
reveals left supraclavicular lymphadenopthy. A
chest radiograph reveals a left sided hilar mass.
Which is the appropriate inital diagnostic test
for this patient?
1 )Bronchoscopy
2 )CT guided biopsy
3 )Lymph node biopsy
4 )PET scanning
5 )Sputum cytology
15.A 43-year-old man with a past history of
hypothyroidism complains he has had pain and
stiffness in the joints of his hand and feet for three
months. He also gives a history of dyspnoea of that
started six weeks ago. Examination shows swollen
hand joints and signs of a right pleural effusion. A
diagnosis of rheumatoid arthritis is made. Which one
of the following is characteristic of a rheumatoid
pleural effusion?
A. It is a transudate
B. It has a low glucose level
C. Long-standing rheumatoid effusions have low
cholesterol levels
D. It has a high pH
E. It should be treated by decortication
17.A 25-year-old male presents to A+E with shortness of
breath. One week ago, he developed influenza &
become more short of breath & fatigued in last 24
hours. His temperature was 38.5oC, his SaO2 = 90%
on 2L of oxygen, a Bl pressure 100/60 mmHg & heart
rate 120/min. The CXR shows patchy consolidation.

Which antibiotic therapy should you select for this man?


1 )Amoxicillin
2 )Amoxicillin Flucloxacillin
3 )Amoxicillin Gentamicin
4 )Amoxicillin Rifampicin
5 )Flucloxacillin
33- A teenage girl presents with Guillain-Barre
syndrome. Her weakness continues to worsen
after admission to hospital. Which of the
following should be used to monitor her?
1) arterial blood gases
2) chest expansion size
3) FEV1/FVC ratio
4) PEFR
5) vital capacity
24) Which of the following statements is true of the
PFTs VC;
a) cannot be measured by spirometer alone.
b) in emphesema & in IPF.
c) Is the max amount of air which can be exhaled
after max. inspiration.
d) Is the sum of tidal volume & inspiratory
capacity.
e) When reduced, is a specific indication of
restrictive lung dis.
In asbestos related disorders which of the
following statements is correct?
1 )basal fibrotic shadowing on CXR suggests
coincidental idiopathic fibrosing alveolitis
2 )increased incidence of primary lung cancer
3 )pleural effusion develops more than 20 years
after causative asbestos exposure
4 )pleural plaques are recognized precursors of
mesothelioma
5 )the risk of malignant mesothelioma is greatly
increased in smokers compared to non-
smokers
39) A 63-year-old woman presents a 5 day history of
progressive shortness of breath. Her family brought
her in because she was increasingly sleepy during the
last 24 hours. She was diagnosed as Chronic
Obstructive Pulmonary Disease 3 years ago with a
FEV1 less than 50% of predicted. She has an oxygen
concentrator at home. Examination revealed
depressed consciousness & a respiratory rate of 24
br/min shallow breaths. There were decreased breath
sounds with minimal air movement. If an arterial Bl
gas on room air were to be performed, which of the
following results would you expect?
1 )pH 7.16 paCO2 70 paO2 50 HCO3 24
2 )pH 7.24 paCO2 80 paO2 55 HCO3 30
3 )pH 7.32 paCO2 60 paO2 70 HCO3 30
4 )pH 7.41 paCO2 40 paO2 50 HCO3 24
5 )pH 7.48 paCO2 30 paO2 85 HCO3 24
25) A 20-year-old male student is assessed for shortness
of breath that occurs whilst running. He has no other
symptoms & does not smoke. Examination, full count,
chest X-ray are normal. Which of following likely will
be helpful in confirming the suspected diagnosis?
1 )Arterial Bl gas studies before & after exercise
2 )Determination of lung volumes & diffusing capacity
3 )Measurement of venous lactate before & after
exercise
4 )Spirometry before & after administration of
bronchodilators
5 )Spirometry before & after exercise
A 60-year-old farmer presented to the emergency department with a two week history of
progressively worsening cough, breathlessness and myalgia. His symptoms had
become particularly worse after he recently finished baling hay indoors, ready for the
winter.He had no other previous medical history of note and was a lifelong non-
smoker. He recently purchased a budgerigar from an internet site. O/E his
temperature was 38.4C, bl.pr 160/100 mmHg, pulse 130 beats per minute and
oxygen saturations of 86% on air. His heart sounds were normal and there were no
audible murmurs. Auscultation of the chest revealed bronchial breath sounds at the
right base. Abdominal examination revealed a soft and non-tender abdomen with no
palpable masses or abnormal enlargement of organs.Laboratory investigations
revealed:Hb14.5 g/dL(13.0-18.0)WBCs20.0 x109/L(4-11 x109)Neutrophils18.0
x109/L(1.5-7 x109)Lymphocytes1.0 x109/L(1.5-4.0 x109)Monocytes0.8 x 109/L(0-
0.8 x109)Eosinophils0.2 x109/L(0.04-0.4 x109)Basophils0.01 x109/L(0-0.1
x109)Platelets390 x109/L(150-400 x109) ESR (Westergren)90 mm/1st hour(0-15
mm/1st hour)S.Na140 mmol/L(137-144)S. K4.2 mmol/L(3.5-4.9)S. urea13.7
mmol/L(2.5-7.5)S. cr.120 mol/L(60-110)Serum avian precipitins Positive
Micropolyspora faeni precipitins Negative
A chest radiograph revealed patchy consolidation in the right lower zone.

What is the most likely diagnosis?


1. Avian influenza
2. Chlamydia psittaci pneumonia
3. Coxiella burnetii pneumonia
4. Extrinsic allergic alveolitis
5. Farmer's lung
True 2. This patient has acute pneumonia with a high white cell count,
high inflammatory markers and chest radiograph changes
suggestive of an infective process. The most likely pathogen in this
case is Chlamydia psittaci, a pathogen acquired most commonly
from birds.
Extrinsic allergic alveolitis would be less likely to cause such acute
symptoms, and chest radiograph changes are usually bilateral.
Positive serum avian precipitins are not diagnostic of extrinsic allergic
alveolitis (EAA), and only suggest the patient has had exposure to
birds.
Farmer's lung, a form of hypersensitivity pneumonitis, is now very rare
since the majority of farmers no longer bale hay in wet conditions.
When present, it is most commonly associated with positive
Micropolyspora faeni antibodies.
A 35 year old woman is short of breath on exertion. Her
oxygen saturation at rest is 99% but on exertion is 90%.
Her CXR shows prominent pulmonary arteries.
Echocardiography shows enlarged right heart and a right
heart cathether measures the pulmonary arterial
pressure to be 35mmHg. D-dimers are negative and
inflammatory markers are not raised. What is the likely
diagnosis?
A. Pulmonary embolus
B. Hereditary hemorrhagic telangiectasia
C. Pulmonary hypertension secondary to anorectics
D. Primary pulmonary hypertension
E. Pneumocystis pneumonia
29) Which of the following statements is true of
the diffusion capacity of carbon monoxide?
1 )Is a specific measure of lung perfusion.
2 )Depends on thickness of alveolar wall.
4 )Is increased in cigarette smokers.
5 )Is increased in emphysema.
1) The Mantoux reaction an example of
which type of hypersensitivity reaction ?
1 )Type I hypersensitivity
2 )Type II hypersensitivity
3 )Type III hypersensitivity
4 )Type IV hypersensitivity
5 )Humoral Immune Response
Comments:
4 )Type IV hypersensitivity
The tuberculin skin test is an example of a Type IV hypersensitivity,
a delayed type hypersensitivity (DTH) reaction. This reaction
develops when primed Th1 cells encounter their specific antigen. An
inflammatory response evolves over 24-72 hours. In tuberculin skin
test, injected antigen is protein derived from M.Tuberculosis. Th1
cells recognize peptide bound to MHC on APCs, are activated &
secrete pro-inflammatory cytokines including IL2, IFN-gamma, TNF,
chemokines as GM-CSF. There is recruitment of inflammatory cells
predominantly macrophages at the site of antigen deposition,
activation of phagocytes. Some cytokines (TNF) as well as
macrophage derived lytic enzymes cause local tissue destruction.
CD8+ T-cells have also been implicated in DTH responses. The
result is an indurated erythematous lesion at the site of injection
which indicates previous exposure to TB.
32) A 35-year-old man presents after 3 months of chronic
cough with purulent sputum & shortness of breath on
exertion. He gives a history of at least two sinus &
bronchial infections per year requiring treatment with
antibiotics. He also says he & his wife have been
unable to have children. He smokes 15 cigarettes per
day. Examination was normal except for some
wheezing & an area of focal crackles at the left lung
base. Chest X-ray shows patchy infiltrates at both bases.
Investigations revealed FEV1 2.0 L, FVC 2.7 L, pH7.38,
PaCO2 40 mmHg, PaO2 82 mmHg. What is the likely
diagnosis?
1 )alpha-1-Antitrypsin (Antiprotease) deficiency
2 )Asthma
3 )Cystic fibrosis
4 )Hypogammaglobulinemia
5 )Immotile cilia syndrome
3.A 24-year-old man presents to the emergency room
complaining of shortness of breath and right-sided
chest pain. The symptoms began abruptly about 2
hours previously. The pain is worse with inspiration.
He denies fevers or chills and has not had any leg
swelling. He has no past medical history but smokes 1
pack of cigarettes daily. On physical examination, he
is tachypneic with a respiratory rate of 24 breaths/min.
His oxygen saturation is 94% on room air. Breath
sounds are decreased in the right lung, and there is
hyperresonance to percussion. A chest radiograph
confirms a 3.5 cm pneumothorax of the right lung.
What is the best approach for treatment of this patient?
A. Needle aspiration of the pneumothorax.
B. Observation and administration of 100% oxygen.
C. Placement of a large-bore chest tube.
D. Referral for thoracoscopy with stapling of blebs .
2.A 63-year-old male with a long history of cigarette
smoking comes to see you for a 4-month history of
progressive shortness of breath and dyspnea on
exertion. The symptoms have been indolent, with no
recent worsening. He denies fever, chest pain, or
hemoptysis. He has a daily cough of 3 to 6
tablespoons of yellow phlegm. The patient says he
has not seen a physician for over 10 years. Physical
examination is notable for normal vital signs, a
prolonged expiratory phase, scattered rhonchi,
elevated jugular venous pulsation, and moderate
pedal edema. Hematocrit is 49%.
Which of the following is likely to prolong his survival?
A. Atenolol
B. Enalapril
C. Oxygen
D. Prednisone
E. Theophylline
1. A 52-year-old female presents with a community acquired
pneumonia complicated by pleural effusion. Athoracentesis is
performed, with the following results: Appearance Viscous,
cloudy.
pH 7.11.
Protein 5.8 g/dL.
LDH 285 IU/L.
Glucose 66 mg/dL.
WBC 3800/mm3, PMNs 93%.
Gram stain Many PMNs; no organism seen
Bacterial cultures are sent, but the results are not currently
available. Which characteristic of the pleural fluidis most
suggestive that the patient will require tube thoracostomy?
A. Presence of more than 90% polymorphonucleocytes (PMNs).
B. Glucose less than 100 mg/dL
C. Presence of more than 1000 white blood cells
D. pH less than 7.20.
E. Lactate dehydrogenase (LDH) more than two-thirds of the
normal upper limit for serum.
40) A 24-year-old asthmatic patient was admitted with
acute severe asthma. Which of the following
statements regarding the diagnosis is correct?
1 )Agitation should be managed with a
benzodiazepine.
2 )A high inspired Oxygen concentration should be
used routinely.
3 )Inhaled salmeterol is indicated as first line therapy.
4 )Normal arterial pCO2 is reassuring .
5 )Pulsus paradoxus is a reliable sign of severity.
10.A 38-year-old homosexual male who is known to be
infected with HIV virus presents with a week of fever &
tachypnoea. Chest x-ray reveals bilateral alveolar
infiltrates. Arterial Bl gas determination reveals a pO2 of
55mmHg, pCO2 of 33 mmHg on room air.
Bronchoalveolar lavage reveals positively stained
organisms with methenamine silver staining.

Which of following statements is correct concerning his


management?
1 )Tranbronchial biopsy should be carried out to confirm
the diagnosis.
2 )Corticosteroids are contraindicated given the risk of
other opportunistic infections.
3 )Aerosolised Pentamidine therapy would be appropriate
if the patient had a known allergy to sulphonamides.
4 )Trimethoprim-sulfamethoxazole alone should be
administered.
Comments:4 )Trimethoprim-sulfamethoxazole alone
should be administered. Pneumocystis carinii
pneumonia is an opportunistic infection which occurs in
immunosuppressed populations, particularly AIDS. The
classic presentation is non-productive cough,
dyspnoea, fever, bilateral interstitial infiltrates &
hypoxemia. Diagnostic methods of choice include
sputum induction & bronchoalveolar lavage. The drug of
choice for treatment & prophylaxis is trimethoprim-
sulfamethoxazole, but alternatives are often needed
because of adverse effects or, less commonly, treatment
failure. Adjunctive corticosteroid therapy may improve
survival in moderate severe cases.
A 62 yrs old smoker with 2 days history of cough &
fever was admitted with a diagnosis pneumonia & of Lt
sided pleural effusion up to the six intercostal space.
Pleural fluid is aspirated & send for tests. Which of the
following is an indication for insertion of a chest drain?
1. Bl stained pleural fluid.
2. pleural fluid pH< 7.2.
3. Serous pleural fluid .
4. pleural fluid LDH> 200IU/l.
5. pleural fluid glucose > 2 mmol/l.
42) Regarding pneumonia caused by
Legionella pneumophilia, which of the
following is true?
1)is associated with hyponatremia
2 )is best treated with intravenous amoxicillin
& clavulanic acid
4 )is readily diagnosed by standard aerobic
culture of sputum
5 )should be managed on the ward in a
respiratory isolation cubicle
36) A 16-year-old girl presents with a 2 day
history of deteriorating breathlessness &
dyspnoea. Blood gas analysis shows a pH of
7.25, a PCO2 of 7.0kPa, a PO2 of 8.5kPa, a
base excess of 4. Which of the following
interpretations is correct?
1 )Results are consistent with IPF.
2 )Blood gases suggest type1 respiratory failure
3 )Immediate intubation is required.
4 )Results are consistent with late severe asthma.
5 )Bicarbonate may be necessary to correct the
acidosis.
16.A 45-year-old solicitor had an onset of severe, crushing,
substernal chest pain while attending a football match. He
collapsed on his way to the car. Bystander
Cardiorespiratory Resuscitation was begun immediately
and continued until arrival in Casualty where an
endotracheal tube was inserted , ventilation was
maintained on 100% oxygen. Investigations revealed:pH
7.13 PaO2 560 mmHg PaCO2 18 mmHg Bicarbonate
5.8 SaO298%. Based on these laboratory values, which
of the following statements best fit his current
pathophysiology?
1 )He is demonstrating a primary respiratory alkalosis
2 )He probably developed a large right to left
intracardiac shunt
3 )His anion gap is probably normal
4 )His pulmonary artery pressure is probably elevated
21) A 63 year-old diabetic presents with a pyrexia,
productive cough shortness of breath of 5
days. She has RLL consolidation & a small
unilateral pleural effusion on CXR. Which is a
marker of poor prognosis?
1 )Temp >38C
2 )WCC > 15
3 )Her age
4 )Her CXR signs
5 )Her diabetes
14.A 26-year-old man with a history of alcohol &
drug abuse was admitted with a 14 day history
of fever, cough & fatigue. He was emaciated.
His temperature was 39.4C. Cervical & axillary
lymphadenopathy were present. Chest X-ray
showed bilateral areas of pulmonary
shadowing. Which of the following is the likely
diagnosis?
1 )alcoholic cardiomyopathy
2 )pneumococcal pneumonia
3 )pneumocystis pneumonia
4 )pulmonary tuberculosis
5 )tricuspid endocarditis
8. A 35-year-old man, seen 6 months after a cadaveric
renal allograft. He receives azathioprine & prednisolone.
He is feeling generally unwell past week , a pyrexia
of 38.6oC, anorexia & a cough productive of thick green
sputum. Chest x-ray reveals a left lower lobe nodule of
approximately 5cm diameter & central cavitation.
Analysis of sputum reveals long, crooked, branching &
beaded gram-positive filaments. Which of the following
antimicrobials is appropriate initial therapy for this
patient?
1 )Ceftazidime
2 )Co-amoxiclav
3 )Co-trimoxazole
4 )Erythromycin
5 )Rifampicin & Isoniazid
Comments:3 )Co-trimoxazole [100]
The likely diagnosis Norcardiasis. Nocardia are
aerobic, Gram-positive branching filamentous
bacteria which often appear beaded on staining.
Nocardiosis can be diagnosed rapidly by
examination of sputum, pus with Gram stain
& a modified acid-fast stain. Pneumonia is
typically found in the immunocompromised as in
this case, may be a single lesion or extensive
pneumonic consolidation. The drug of choice is
trimethoprim-sulfamethoxazole.
13.A 54-year woman was admitted with acute
breathlessness. On examination she had a
temperature of 37oC, a respiratory rate of 32 breaths
per minute, a pulse of 120 beats per minute, a Bl
pressure of 100/60 mmHg, a peak expiratory flow
rate of 250 litres per minute. Auscultation of heart &
chest was normal. The Chest X-ray was normal,
AB gases on air showed: pH 7.35 (7.36 - 7.44), Pa02
6.0 kPa (11.3 - 12.6), PaC02 3.9 kPa (4.7 - 6.0),
Serum bicarbonate 24 mmol/l (20 - 28). She was
started on high flow oxygen. What is the important
next treatment?
1 )amoxicillin intravenously
2 )aminophylline intravenously
3 )intravenous fluids
4 )low molecular weight heparin
5 )nebulised salbutamol
15-A 54-year woman was admitted with acute
breathlessness. On examination she had a
temperature of 37oC, a respiratory rate of 32
breaths per minute, a pulse of 120 beats per
minute, a Bl pressure of 100/60 mmHg, a peak
expiratory flow rate is normal . Auscultation of
heart & chest was normal. The Chest X-ray was
normal, AB gases on air showed: pH 7.35
(7.36 - 7.44), Pa02 6.0 kPa (11.3 - 12.6), PaC02
3.9 kPa (4.7 - 6.0), Serum bicarbonate 20
mmol/l (20 - 28). She was started on high flow
oxygen. What is the important next treatment?
1 )amoxicillin intravenously
2 )aminophylline intravenously
3 )intravenous fluids
4 )low molecular weight heparin
5 )nebulised salbutamol
9. A 42 year-old restauranter who is HIV positive
from 8 years presents with progressive
shortness of breath on exercise. The chest X-
ray was normal except for prominent
pulmonary arteries. Pulse oximetry
demonstrates that he desaturates on exercise.
Which is the likely diagnosis?
1 )Pneumocystis carinii pneumonia
2 )Primary pulmonary hypertension
3 )Intracardiac shunt across an atrial septal
defect
4 )Pulmonary embolic disease
5 )Anaemia
44) Which of the following statement is true
of infections with Mycobacterium
tuberculosis:
1 )non-sputum producing patients are non-
infectious
2 )a positive tuberculin test indicates active
disease
3 )lymph node positive disease requires
longer treatment than pulmonary disease
4 )in pregnant women treatment should be
given until after delivery
5 )pyrazinamide has high activity against
active extracellular organisms
Comments: 1 )non-sputum producing patients are non-
infectious
Only untreated smear positive pulmonary TB is likely to
be infectious. Active disease may be indicated by grade
III/IV response tuberculin. 80% of individuals have
history of BCG vaccination have grade I/II response. All
forms of pulmonary TB may be treated equally except
tuberculous pleural effusion which may require drainage
(with large effusions causing breathlessness) with
adjunct corticosteroids to delay reaccumulation. Length
of treatment in other forms are bone TB 9 months,
meningitis 1 year, drug resistance 2 years. Streptomycin
has high activity against extracellular organisms whilst
pyrazinamide has high activity against intracellular
organisms.
A 65 year old woman with breathlessness has fibrotic
changes on her CXR. She has arterial blood gases and
lung function tests. Which one of the following features
suggests progressive pulmonary sarcoid?
A. high pCO2 .
B. A fall in lung compliance
C. A reduced maximum lung elastic recoil pressure
D. An restrictive flow volume loop
E. A normal helium dilution total lung capacity
Acid Base Balance
Arterial Blood Gases
pH 7.35 -7.45
pCO2 4.6 6.1 Kpa
pO2 9 -10.5 Kpa
HCO3 22-28 mmol/l.
Cl 96-105 mmol/l.
AG 12 4 meq/L .
osmolar gap 10-15 mmol/L
Stepwise approach to interpreting the
arterial blood gas.
1. History & Presentation: The H&P usually
gives an idea of what acid base disorder might
be present even before collecting the ABG
sample.
2. Look at the pH:
- If pH < 7.35, then acidemia
- if pH > 7.45, then alkalemia
- If pH within normal range:
then acid base disorder not likely present
pH may be normal in the presence of a mixed
acid base disorder, particularly if other
parameters of the ABG are abnormal.
3. Look at PCO2, HCO3:

- If PCO2 is the initial chemical change,


then process is respiratory.
- if HCO3- is the initial chemical change,
then process is metabolic.
Respiratory Acidosis PCO2 HCO3

Respiratory Alkalosis PCO2 HCO3

Metabolic Acidosis HCO3 PCO2

Metabolic Alkalosis HCO3 PCO2


Acute Respiratory Acidosis :
The [HCO3] will increase by 1 mmol/l for
every 10 mmHg elevation in pCO2
above 40 mmHg.

Chronic Respiratory Acidosis :


The [HCO3] will increase by 4 mmol/l for
every 10 mmHg elevation in pCO2
above 40mmHg.
Acid-Base Physiology
Bedside Rules for Assessment of
Compensation
Metabolic Acidosis:
Calculate expected pCO2
respiratory Acidosis :
Calculate expected HCO3.
Metabolic Acidosis
A metabolic acidosis is an abnormal
primary process or condition leading to
an increase in fixed acids in the blood.
A decrease in plasma bicarbonate can be
caused by two mechanisms:
A gain of strong acid
A loss of base
Anion Gap and Delta Ratio
The anion gap is defined as the quantity
of anions not balanced by cations.
This is usually equal to 12 4 meq/L and
is usually due to the negatively charged
plasma proteins as the charges of the
other unmeasured cations and anions tend
to balance out.
Na + Unmeasured cations = Cl- +
HCO3- + Unmeasured anions
Anion gap = [Na] ([Cl-] + [HCO3-])
Classification by Anion Gap
A: High Anion-Gap Acidosis
Ketoacidosis .Lactic Acidosis
Diabetic ketoacidosis

Renal Failure Toxins

Ethylene glycol
Methanol
Salicylates
B:Normal Anion-Gap Acidosis (or Hyperchloraemic
acidosis)
Renal Causes
Renal tubular acidosis
Carbonic anhydrase inhibitors

GIT Causes
Severe diarrhoea
Uretero-enterostomy
Drainage of pancreatic or biliary secretions
Small bowel fistula
When high AG

Delta AG
Delta HCO3 > 1 ...... Concomitant
metabolic alkalosis .

Delta AG
Delta HCO3 < 1 ...... Concomitant NG
metabolic acidosis.
Osmolar Gap

The Osmolar Gap is another important diagnostic


tool that can be used in differentiating the causes
of elevated anion gap metabolic acidosis. The
major osmotic particles in plasma are Na+ , Cl- ,
HCO3-, urea and glucose and as such, plasma
osmolarity can be estimated as follows:
Plasma osmolarity = 2(Na) + glucose + BUN.
Note that because Cl- and HCO3- are always
bound to Na, their contributions to osmolarity are
estimated by doubling the Na concentration.
Plasma osmolality (Posm) can also be measured
directly by freezing point depression.
The osmolar gap is the difference between the
calculated serum osmolarity and the measured
serum osmolarity
Osmolar Gap = Measured Posm Calculated
Posm
The normal osmolar gap is 10-15 mmol/L H20 .The
osmolar gap is increased in the presence of low
molecular weight substances that are not included
in the formula for calculating plasma osmolarity.
Common substances that increase the osmolar
gap are ethanol, ethylene glycol, methanol,
acetone, isopropyl ethanol.
In a patient suspected of poisoning, a
high osmolar gap (particularly if 25)
with an otherwise unexplained high
anion gap metabolic acidosis is
suggestive of either methanol or
ethylene glycol intoxication.
Poisonings & Intoxications

Although many drugs and toxins can be removed with extracorporeal


techniques, few are indicated for this therapy e.g
- Alcohol (methanol,ethylene glycol, alcohol)
- Lithium,salicylate,theophylline,metformin
Poisonings & Intoxications
Methanol
- solvents , varnish ,de-icing solutions
-N/V,abdominal pain (pancreatitis), blurred
vision (papilledema), CNS symptoms
,seizures ,coma, metabolic acidosis (increase
anion gap).
Poisonings & Intoxications

Methanol
Treatment:
- empty stomach contents
- Ethanol/fomepizole (inhibit ETOH DeH)
-Folic acid
- Hemodialysis: severe metabolic acidosis, very
high osmolar gap, methanol level>50 mg/dl &
visual changes (disc hyperemia).
Poisonings & Intoxications

Ethylene Glycol
- Antifreeze ,solvents
- not protein bound
- EGglycolic acidglyoxylic acidoxalic acid.
- early : as methanol.
- later : flank pain ,renal failure ,ca oxalate crystals.
Poisonings & Intoxications
Ethylene Glycol
Treatment:
- Empty stomach contents
- Ethanol/fomepizole (inhibit ETOH DeH)
- Thiamine 100 mg IM qid, B6 50 mg IM qid
; (glyoxalate glycine)
- Hemodialysis, CVVHD/F (severe
metabolic acidosis , very high osmolar gap,
level>20 mg/dl)
Poisonings & Intoxications
Lithium
- Acute /acute on chronic/ chronic
- Not protein bound
- C/P: lethargy ,tremor ,confusion ,seizure
Treatment:
- Empty stomach
- Replete volume/forced diuresis
- Hemodialysis/ CVVHDF, Indications:
- level > 4 mEq/L
- Level > 2.5, severe CNS symptoms
Poisonings & Intoxications

Salicylate
- Acute or chronic intoxication
- C/P: headache,tinnitus,confusion,n/v, hyperventilation, metabolic
acidosis,cardiovascular collapse
Treatment;
- Gastric emptying,activated charcoal.
- IV fluids, alkaline diuresis.
- Hemodialysis:
level > 80 mg/dl with symptoms
seizure, coma
severe metabolic acidosis + renal failure
Metabolic Alkalosis
A metabolic alkalosis is a primary acid-base
disorder which causes the plasma bicarbonate
to rise to a level higher than expected.
Causes of a metabolic alkalosis can
be classified into several groups

Gain of alkali in the ECF


from an exogenous source (eg IV NaHCO3
infusion, citrate in transfused blood)
Loss of H+ from ECF
via kidneys (eg use of diuretics)
via gut (eg vomiting, NG suction)
Respiratory Acidosis
Respiratory failure type 2.
COPD, CO2 retension.
Respiratory Alkalosis
Hyperventilation.
Tachypnea. ( fever ,pneumonia , PVC)
Biochemistry results: Na+ 127, K+ 5.2,
Cl- 79, HCO3- 20, urea 50.5 & glucose 9.5
mmols/l. Anion gap 33 mmols/l
Arterial Blood Gases
pH 7.58
pCO2 21 mmHg
pO2 154 mmHg
HCO3 19 mmol/l
the acid-base diagnosis:
1. pH: pH>7.44 so an alkalaemia is
present. The cause is an alkalosis
2. Pattern: pCO2 & bicarbonate are both
low suggesting either a metabolic acidosis
(with respiratory compensation) or a
respiratory alkalosis (with renal
compensation). As we know an alkalosis
must be present then we would accept a
respiratory alkalosis.
3. Clues: The anion gap is noted to be very high
so there must be a high-anion gap metabolic
acidosis present as well. To explore the
causes of a HAGMA:
4. Compensation:
Assessing the compensation for a respiratory
alkalosis . The expected HCO3 is..........
Arterial Blood Gases (on supplemental O2
by face mask)
pH 7.20
pCO2 39 mmHg
pO2 277 mmHg
HCO3 14.9 mmol/l
A previously well 55 year old woman is
admitted with a complain of severe
vomiting for 5 days. Physical
examination reveals postural
hypotension, tachycardia, and
diminished skin turgor. The
laboratory finding include the
following:
Electroyes: Na 140 , K 3.4, Cl 77
HCO3 9 , Cr 2.1
ABG: pH 7.23 , PCO2 3.3 kpa
2-A 44-year-old woman presents with 24
hours of unremitting emesis. She is
dehydrated and hypotensive.
Tests of her ABG show the following
values: PaO2 = 11 Kpa , PaCO2 = 6 Kpa,
HCO3- = 39 mEq/L, and pH = 7.49. What is
the acid-base disturbance here?
A 50 year old insulin dependent diabetic woman was
brought to the ED by ambulance. She was semi-
comatose and had been ill for several days.
Current medication was digoxin and a thiazide
diuretic for CHF.
Lab results
Serum chemistry: Na 132, K 2.7, Cl 79,
HCO3 19 Glu 815,
Lactate 0.9 urine ketones 3+

ABG: pH 7.41 PCO2 32 HCO3- 19 pO2 82


A 60 year old homeless man presents with
nausea, vomiting and poor oral intake 2
days prior to admission. The patient reports
a 3 day history of binge drinking prior to
symptoms.
Labs : Serum chemistry: Na 132, K 5.0, Cl
104, HCO3- 16 , BUN 25, Cr 1.3, Glu 75
ABG: pH 7.30 , PCO2 3.6 ,
HCO3- 16, PO2 9.2.
22 year old female with type I DM, presents
to the emergency department with a 1 day
history of nausea, vomiting, polyuria,
polydypsia and vague abdominal pain.
O/E. noted for deep sighing breathing,
orthostatic hypotension, and dry mucous
membranes.
Labs: Na 138 , K 6.0, Cl 93, HCO3- 11
glucose 720, BUN 38, Cr 2.6.
UA: pH 5, SG 1.010, ketones negative,
glucose positive . Plasma ketones trace.
ABG: pH 7.2 HCO3- 10 PCO2 2.7
Potasium Homeostasis
98 %of total body K is intracellular.
Only 2% (about 70 mEq), is in the
extracellular fluid,
where the normal concentration of 3.5-5
mEq/L.
Hyperkalemia
Pseudohyperkalemia
Hemolysis via small needle (traumatic Venipuncture)
The most common cause is lysis of red cells in a
phlebotomy specimen.
Delayed analysis.
Excessive tourniquet .
Severe Thrombocytosis. Potassium released from platelets
Severe Leukocytosis.

Pseudohyperkalemia can be excluded by repeating the sample


as atraumatically as possible .
Redistribution (ICF to ECF)
Acidosis
Hyperkalemic periodic paralysis.
Insulin deficiency (DKA).
Tissue damage (Tumor lysis ,Burn injury ,Crush injury,
Surgery , Massive Hemolysis )

Medications:
Beta Blockers
Digitalis intoxication (Digoxin Toxicity)
Decreased renal excretion

Renal Insufficiency or Renal Failure.


Renal Tubular Acidosis(Type4) .
Adrenal Insufficiency (Addison's).
Medication.
Heparin .
Cyclosporine
ACE Inhibitor
Potassium sparing Diuretics
NSAIDS
The initial diagnostic approach begins with
the clinical history, review of medications,
and physical examination.
Symptoms and signs include muscular
weakness or flaccid paralysis, paralytic
ileus, and characteristic ECG changes.
Symptoms and Signs
3 types of muscle fibres.
May be asymptomatic
Irregular heartbeat
Fatigue
Tingling, numbness, or paraethesia
Weakness
Paralysis
Difficult breathing
ECG
Mild hyperkalemia 5.5-6 mmol/L peaked T
Moderate 6-8 mmol/L, peaked T + absent P wave
Severe > 8 mmol/L , peaked T absent P wave ,
wide QRS , Vent arrhythmia,VF,asystole
TARGET OF TREATMENT

Minimize membrane depolarization


Shifting k into cells
Promote k loss
Correction of acidosis
D/C exogenous k sources , antikaliuretics.
Serious hyperkalemia =
Plasma K> 7.5 mmol/L
Profound weakness
Absent P wave , wide QRS , vent arrhythmia.
Intravenous calcium is administered to stabilize
the myocardium;
it lowers the threshold potential, thus
counteracting the toxic effect of high potassium.
Calcium does not have any effect on the serum
potassium level. Improvement in the ECG
changes should be visible within two to three
minutes of administration of calcium
Medications Used in
Acute Treatment of Length of
Hyperkalemia Dosage Onset effect Mechanism of action Cautions
10 to 20 mL of 10 Immediate 30 minutes Protects myocardium Can worsen
Calcium percent solution IV over from toxic effects of K; digoxin toxicity
two to three minutes no effect on serum
gluconate potassium level

Regular insulin 10 units 15 to 30 minutes Two to Shifts potassium out of Consider 5


Insulin IV with 50 mL of 50 six hours the vascular space and percent
percent glucose into the cells; no effect dextrose
on total body solution
potassium infusion at 100
mL per hour to
prevent
hypoglycemia
with repeated
doses. Glucose
unnecessary if
blood sugar
elevated above
250 mg per dL
10 to 20 mg by nebulizer 15 to 30 minutes Two to three Shifts potassium into the May cause a
Albuterol over 10 minutes (use
concentrated form, 5 mg
hours cells, additive to the
effect of insulin; no
brief initial
rise in serum
(Ventolin) per mL) effect on total body
potassium
potassium

20 to 40 mg IV, give with 15 minutes to one hour Four hours Increases renal excretion Only effective
Furosemide saline if volume of potassium if adequate
depletion is a concern renal response
(Lasix) to loop diuretic

Oral: 50 g in 30 mL of One to two hours Four to six Removes potassium from Sorbitol may
Sodium sorbitol solution (rectal route is faster) hours the gut in exchange for be associated
polystyrene Rectal: 50 g in a 1gm binds 1mmol k sodium with bowel
retention enema necrosis. May
Cation lead to sodium
retention
exchange resin

hemodiatysis
The most rapid and PD is 15% as
effective effective as HD
Hyporeninemic hypoaldosteronism( RTA
type 4 ) should be considered in patients
with diabetes and hyperkalemia, a trial of
oral fludrocortisone to establish this
diagnosis;

potassium levels will return to normal in a


day or two after initiation of fludrocortisone.
Hypokalemia
Manifestation of hypokalemia
1)Cardiac-ECG changes
2) Neuromuscular
a) weakness of the limb muscles
b) weakness of the trunk and resp. m.
c) Smooth m. dysfunction ( gastric distension,
ileus, urinary retention)
d) Rhabdomyolysis.
3. Renal
a) DI (polyuria)
b) tubulointerstesial nephritis.
c) Metabolic alkalosis.
d) Hyponatremia
Etiology of hypokalemia
(Loss or Shift)
1) Dietary Deficiency.
2) Redistribution.
3) Non renal losses.
4) Renal losses.
Redistribution -Cellular uptake

Cellular uptake is controlled by the activity of


Na+/K+ ATPase and H+ion conc.
Alkalosis
Insulin-
adrenergic stimulation.
Extrarenal losses of K+

B.Gastrointestinal losses
Vomiting
GI suction
Diarrhea
Rectal tumors
Laxative abuse
Renal K+ loss
Urinary K+ excretion > 20 meq/L
Either associated with:
Normotension
Hypertension
Renal K+ loss

Normotensive.
1) DrugsDiuretics ,Amphotericin,cisplatine.
2) Renal tubular acidosis (type 1&2).
3) Bartters Syndrome,Gitelman Syndrome.
4) Magnesium deficiency.
5) Hypercalcemia.
Renal K+ loss
Hypertensive

A. Increased renin
B. Decreased renin
C. Normal renin
Renal K+ loss
Hypertensive
A.Increased renin
Malignant hypertension
Renal artery stenosis
Renin secreting tumor.
B. Decreased renin
1. Increased Aldosterone.
Primary hyperaldosteronism (Conns disease).
adrenal adenoma.
adrenal hyperplasia.
Adrenal carcinoma.
2. Decreased Aldosterone.
(ap.miner.excess)
Exogenous mineralocorticoid.
Licorice ingestion.
Liddles syndrome.
11 BHSD deficiency .
11-hydroxylase deficiency.
Normal Renin( Cushing)

Cushings Syndrome (Adrenal


adenoma, hyperplasia, carcinoma).
Pitutary (Cushings Disease).
Ectopic ACTH.
Renal tubular acidosis
Renal tubular acidosis (RTA) hyperchloremic
(normal anion gap) metabolic acidosis .
high urine pH, bicarbonaturia, and reduced net
acid excretion. frequently accompanied by
hypokalemia.

Distal RTA Type 1 AR or acquired


defect in H+ excretion & urinary acidification by the
DCT hypercalcuria, Nephrocalcinosis.

Proximal RTA Type 2 AR or acquired


Defect of HCO3 reabsorption from PCT, generally
occurs as part of the renal Fanconi S. osteomalacia.
Bartter Syndrome
AR
affecting Na+-K-2Cl in TLOH salt wasting and
hypokalemic, metabolic alkalosis. lasix like
Failure to thrive, growth retardation,
dehydration, low blood pressure, muscle
weakness, tetany, paresthesias.
Nephrocalcinosis.
hypercalciuria and normomagnesemia
Nephrocalcinosis.
Gitelman Syndrome
is a milder disorder.
diagnosed in adolescents and adults.
AR trait, inactivating mutations of thiazide
sensitive Na-Cl cotransporter.
Thiazide like.
hypocalciuria due to Ca2+ transport in PCT.
Hypomagnesemia remains unexplained.
Apparent Mineralocorticoid Excess

Deficiency in 11-hydroxysteroid dehydrogenase


enzyme. (11 HSD), responsible for the conversion of
cortisol to the inactive metabolite , therefore protecting the
mineralocorticoid receptors from cortisol intoxication.
Liddle Syndrome (psuedohyperaldosteronism).

Congenital Adrenal Hyperplasia(11hydroxylase def)

In AME . failure to thrive, salt retention, hypertension, and


hypokalemic alkalosis, with a suppression of the RAA
system.
Liddle Syndrome
AD,
HTN + hypokalemia
low plasma renin and aldosterone.
Activating mutations in the amiloride-sensitive
epithelial Na+ channel in the CD.
Treatment
Hypertension is not improved by spironolactone
but can be corrected by ENaC antagonists
(amiloride or triamterene).
Glucocorticoid-Remediable
Hyperaldosteronism

AD
Fusion between aldosterone synthase and 11-
hydroxylase, regulated by ACTH secrete
aldosterone in response to ACTH,
However, some subjects have early onset severe
hypertension, hypokalemia, and metabolic
alkalosis.
diagnosis ; 18-hydroxycortisol metabolites in the
urine.
TTT= glucocorticoid administration can suppress
excessive aldosterone secretion.
Acid-base and potassium balance

metabolic alkalosis and hypokalemia suggest diuretic


use or vomiting.

metabolic acidosis and hypokalemia suggest diarrhea or


laxative abuse or RTA type 1&2.

metabolic acidosis and hyperkalemia suggest adrenal


insufficiency or RTA type 4 .

On the other hand, the plasma bicarbonate and


potassium concentrations are typically normal in the
SIADH
Hyponatremia
Hyponatremia

Defined as serum sodium less than 135.


Very common in hospitalized patients.
Severity of symptoms depends on acuity
of change (1-3 days).
neurological signs due to brain edema.
Clinical presentation
Clinically significant hyponatremia present with
nonspecific symptoms attributable to cerebral
edema.
Nausea and vomiting .
Difficulty concentrating.
Confusion.
Lethargy.
Agitation.
Headache.
Seizures. (Focal or generalized seizure).
In those patients with acute severe
hyponatremia, signs of brainstem herniation,
including coma, unilateral, dilated fixed pupil.
Hyponatremia
Hypovolemic
Orthostatic changes Poor skin turgor
BP/HR Flat jugular veins
Dry mucous membranes.

Hypervolemic
Peripheral edema
Elevated jugular venous pressure Ascites

(signs of the cause, congestive heart failure , cirrhosis , or nephritic syndrome)

Euvolemic
Absence of physical exam findings consistent with above .
Hypovolemic hyponatremia
Excess fluid losses (eg, vomiting, diarrhea, excessive sweating, GI
fistulas or drainage tubes, pancreatitis, burns) that have been replaced
primarily by hypotonic fluids (GIT) .

Salt-wasting nephropathy (Renal) .

Cerebral salt-wasting syndrom (Renal).

Check urine sodium

<20 mEq/L --- vomiting, diarrhea, third space loss

>20 mEq/L ---Diuretics, Mineralocorticoid deficiency,


Bicarbonaturia, Salt losing nephritis
Euvolemic hyponatremia
1. Psychogenic polydipsia.
2. Administration of hypotonic intravenous fluids or
irrigation fluids to hospitalized pts.
3. SIADH .
4. hypoadrenalism & hypothyroidism.

Check urine osmolality

<100 mOsm/kg Primary poydipsia

>100 mOsm/kg - check TSH/FT4 & short synacten test .


Hypervolemic hyponatremia
acute or chronic renal failure.
hepatic cirrhosis, congestive heart failure,
or nephrotic syndrome.
Check urine sodium
<20 mEq/L ------CHF, Cirrhosis, Nephrotic syndrome
>20 mEq/L ----ARF, CRF
Syndrome of Inappropriate antidiuretic
Hormone (SIADH)
occurs when ADH is released in amounts far in
excess of those indicated by plasma osmotic
pressure.
this syndrome is associated with diseases that
affect osmoreceptors in the hypothalmus.
is more common in the elderly.
Causes: brain (hypothalamus) , pulmonary ,
Drugs.
SIADH is characterized by

1. Euvolemic hyponatremia.
2. serum hypo-osmolality.
3. concentrated urine.
4. Urinary sodium concentrations > 20 mEq/L.
5. Normal renal function.
6. Normal adrenal & thyroid function.
Treatment

Acute hyponatremia (duration <72 h) can be safely


corrected more quickly than chronic hyponatremia.

Correction of serum sodium that is too rapid can


precipitate severe neurologic complications, such as
central pontine myelinolysis.

More rapid therapy (1.5 to 2 mEq per hour) can be used


in the first few hours if there is serious cerebral
dysfunction. Otherwise, maximum rate of elevation: 10-
12 mEq during the first 24 hours
Treatment of hyponatremia
Hypovolemic hyponatremia
Normal saline 0.9 % till euvolemic
Hypervolemic hyponatremia
Water restriction+ furesamide
Euvolemic hyponatremia
Mild --.Water restriction+ ttt of cause
.Demeclocycline
.V2 receptor antagonist (vaptans)
severe -- hypertonic saline
hypertonic saline
Hypertonic saline is indicated if
1. Symptomatic hypona.
2.s.Na < 120 mEq

Na needed = (120 s.Na) x TBW


Time needed = Na to be raised by 0.5 mEq /hr
= 2 X (120 s.Na)
Na needed / hr = Na needed
Time needed
Hypertonic solution needed/hr= Na needed / hr x2
Central Pontine Myelinolysis
Due to rapid correction of Hyponatremia
Possibly due to disturbance of blood brain
barrier and damage by cytokines
No effective therapy
Presentation( neuro)

Dysarthria-Dysphagia
Lethargy and coma
Seizures-Nystagmus
Ataxia
emotional lability-akinetic mutism
gait disturbance -myoclonus
Behavioral disturbances-
Paraparesis or quadriparesis
Hypernatremia
Hypernatremia
Hypernatremia is defined as a plasma
[Na+] >145 mEq/L .
Hypernatremia are almost caused by water
loss rather than increase in total body
sodium .
So Sodium concentration is often used as
an indicator of fluid status rather than salt
imbalance .
Causes of Na
Causes of Hypernatremia
I)Hypovolemic Hypernatremia

:A) Non Renal Causes.


Ur.Osmol >400 mosmol /kg .

1 ) GIT .
2 ) Skin :- High fever , heavy exercise etc.
3 ) Respiratory : -Intubated Pts.

B) Renal causes.
Polyuric States ( Ur.Osmol. May be low or high )

E.C.F. depletion & Intravascular depletion


Skin turgor, sunken eyeballs, Dry mucus membranes.
Hemodynamic effects.(BP, HR, JVP).
Cool extremities..
II) Isovolemic Hypernatremia
Central Diabetes Insipidus
Any CNS Insult
1 ) Infiltration of post.pitutary.
2 ) Post Hypophysectomy.
3) Vascular Insult .
4)Encephalitis.
5)Neoplasms.
6)Taumatic.
Ur.Osmol.<250 mosmol/kg .

Treatmen Vasopressin + -Arginine -


Carbamazepine .
B) Nephrogenic Diabetes
Insipidus
1 ) Inherited .
2 ) Acquired :-
Intrinsic renal disease :
Sickle cell anemia , medullary cystic diseases.
Obst. Uropathy.
Drug Induced.
Electrolyte Disturbance.
Causes of Diabetes Insipidus
Central DI Nephrogenic DI
Aquaired
Pituitary surgery 1. Drugs - Lithium,
Head trauma demeclocycline,
Tumors, Suprasellar or intrasellar amphotericin B,
leukemia
2. Electrolyte disturbances
Vascular (cerebral aneurysm,
K, Ca.
thrombosis, hemorrhage,
Sheehan syndrome) 3. Conditions that impair
medullary hypertonicity;
Granulomas
obstructive uropathy,
(sarcoidosis,Histocytosis,
Wegener granulomatosis, diuretic phase of ATN,
tuberculosis) osmotic diuresis.
Infectious (encephalitis, Congenital,
meningitis, Guillain-Barr - XLR V2 mutation.
syndrome)
Idiopathic (autoimmune) in 30%.
Diabetes Insipidus

Na

Large urine volume


Uosm excretion rate <700 mosm/d
=DI

<300
300-700 = partial
Water deprivation

U osm <300 =complete DI 300-700 = partial

10 g d DAVP IN 10 g d DAVP IN

U osm > 50% If less


>10% =cranial <10% =nephrog..
= cranial =nephrogenic
III ) Hypervolemic Hypernatremia-

A)Iatrogenic
B) Miniralocorticoid Excess :-
Lab Studies
There are 3 essential laboratory tests in the
evaluation of patients with hypernatremia:
1. Serum osmolality
2. Urine osmolality
3. Urinary sodium
Treatment of hypernatremia
Hypovolemic hypernatremia
Normal saline 0.9 % till euvolemic
Hypervolemic hypernatremia
D5W + furesamide
Euvolemic hypernatremia
Hypotonic fluids (D5W, 0.45% Saline)--
water Deficit = TBW(60%BW) X s.Na -140
140
Administer deficit over 48 h
Treatment of hypernatremia

Treatment of hypernatremia

Hypovolemic Euvolemic Hypervolemic

Isotonic saline(0.9%) Hypotonic fluids Frusemide + D5W or


till euvolemia (D5W, 0.45%) Oral
Furosemide - Na K 2 CL channels
medullary hypertonicity
free water loss from CD.
Specific management issues:
central DI intranasal desmopressin 5- 20g
(intranasal spray) x1-2/d.
Nephrogenic DI
- low Na diet.
- NSAIDs ( action of AVP).
- Thiazide.
CALCIUM
Ionized Calcium regulation is critical for:
nerve conduction,
muscle contraction, and muscle relaxation.
membrane stability,
bone & teeth mineralization ,
blood coagulation,
Body Calcium Distribution

Calcium

(99%) <1%
intracellular (4/5)
Bone and extracellular fluid (1/5)

10%
50% 40% complexed with anions
bound to protein
free (active) ionized form
(predominantly albumin) (eg, phosphate, carbonate,
citrate, lactate, sulfate).
Calcium regulation
Calcium regulation is maintained by
parathyroid hormone (PTH), vitamin D,
and calcitonin through complex feedback
loops.
These compounds act primarily at bone,
renal, and GI sites.
Calcium also is affected by magnesium
and phosphorus.
HYPOCALCEMIA
Causes of hypocalcemia
Hypoalbuminemia
Pseudohypocalcemia.
Parathyroid hormone related
Hypoparathyroidism
Pseudohypoparathyroidism
Hypomagnesemia
Vitamin D related
Nutritional deficiency
Impaired absorption
Hepatic disease
Renal failure
Anticonvulsant
vitamin D resistant
Ca deposition
Hungry bone syndrome
Acute pancreatitis
phosphate administration
Hyperphosphatemia ( Rhabdomyolysis , tumor lysis, Renal failure)
Critical illness and severe sepsis
Medications and other causes
Parathyroid hormone related
Hypoparathyroidism
Acquired hypoparathyroidism
Neck irradiation
surgical removal
Infiltrative disease ; hemachromatosis, granulomatous disease (sarcoidosis),
amyloidosis, or metastatic malignant infiltration
autoimmune disorder; adrenal failure. Mucocutaneous candidiasis,
alopecia&vitiligo are associated (polyglandular autoimmune disease)(PGA I)
Hereditary hypoparathyroidism .
Sporadic, late-onset
DiGeorge syndrome, associated with congenital heart disease, cleft palate/lip,
and abnormal facies.
Kearns-Sayre syndrome, which presents with heart block, retinitis pigmentosa,
and ophthalmoplegia.
Pseudohypoparathyroidism
end-organ resistance to the effects of PTH. E.g. AHO.
Vitamin D related
Nutritional deficiency.
Impaired absorption ; Small bowel diseases, such as celiac disease,
gastric bypass, steatorrhea, and pancreatic diseases.
Hepatic disease .
impaired 25-hydroxylation of vitamin D,
decreased bile salts with malabsorption of vitamin D,
decreased synthesis of vitamin Dbinding protein, or other factors.
Anticonvulsants

Renal failure
decreased conversion of 25-hydroxyvitamin D to its active form 1,25-
dihydroxyvitamin D.This results in an increase in PTH. Inherited conditions
vitamin D dependent rickets (type I) or 1-alpha-hydroxylase deficiency .
vitamin D dependent rickets (type II )--- Hereditary vitamin D resistance
rickets.
Medications and other causes
Proton pump inhibitors (PPIs)
Cinacalcet (calcimimetic agent)--- inhibition of PTH release
Amphotericin, Aminoglycosides.
Chemotherapy
Cisplatin -----Mg wasting Mg wasting
Combination therapy with 5-fluorouracil and leucovorin ----
decreasing calcitriol production
Bisphosphonates
.zoledronic acid -----suppressing the formation and function of
osteoclasts
Anticonvulsant therapy.
Foscarnet---complexes ionized calcium
Some radiographic contrast dyes may contain EDTA, which
chelates calcium in serum, thereby reducing serum ionized ca.
Clinical picture
Acute hypocalcemia
The patient may complain of muscle cramping, shortness of breath
secondary to bronchospasm, tetanic contractions, distal extremity
numbness, and tingling sensations.
Latent tetany findings
Chvostek sign: Tap over the facial nerve about 2 cm anterior to the tragus of
the ear : twitching first at the angle of the mouth, then by the nose, the eye,
and the facial muscles.
Trousseau sign: Inflation of a blood pressure cuff above the systolic pressure
causes local ulnar and median nerve ischemia, resulting in carpal spasm.
Peroneal sign.

Chronic hypocalcemia
cataracts, dry skin, coarse hair, brittle nails, psoriasis, chronic pruritus,
and poor dentition, papilledema. basal ganglia calcification and extrapyramidal
neurologic symptoms (in idiopathic hypoparathyroidism).
Brittle nails,

Alopecia
Dry skin
Coarse air
Lab Studies
Serum albumin
Corrected calcium (mg/dL) = measured total Ca (mg/dL) +
0.8 (4- serum albumin [g/dL]).
Serum ionized calcium
Serum phosphorus
If low-------magnesium depletion (nutritional), vitamin D
deficiency and hungry bone disease
If high----hypoparathyroidism,pseudohypopara, or renal failure
Serum magnesium
Alkaline phosphatase:
normal ------ PTH deficiencies .
elevated ------------- osteomalacia and rickets.
Elevated BUN and creatinine levels may indicate renal
dysfunction.
Imaging Studies
Skeletal x-rays.
Disorders associated with rickets or osteomalacia present
with the pathognomonic Looser zones, better observed in
the pubic ramus, upper femoral bone, and ribs.
Osteoblastic metastases from certain tumors (eg, breast,
prostate, lung).
CT scan of the head . may show basal ganglia calcification
(in idiopathic hypoparathyroidism).
Treatment(Treat symptoms, not numbers)
Acute hypocalcemia
Promptly correct symptomatic or severe hypocalcemia with
cardiac arrhythmias or tetany with parenteral administration.
Administer 1-2 ampules 10% calcium gluconate (93 mg/10
mL) in 50-100ml over 5-10 minutes.
Calcium chloride 10% solution (273 mg/10-mL ampule)
delivers higher amounts of calcium and is advantageous when
rapid correction is needed, but it should be administered via
central venous access.
NB:Patients with cardiac arrhythmias or patients on digoxin
therapy need continuous ECG monitoring during calcium
replacement because calcium potentiates digitalis toxicity.
Chronic hypocalcemia: Treatment of chronic
hypocalcemia depends on the cause .(ca carbonate)
Hypercalcemia
Management of Acute Hypercalcemia
1.Hydration.
Saline (0.9%) infusion, 2 -4 L over 24 hours

2.Inhibition of Bone Resorption.


Bisphosphonate.
Pamidronate(60 to 90 mg over 4 hours IV) or.
Zoledronate(4 mg over 15 min IV).
Calcitonin.
4 IU/kg to 8 IU/kg q12h sc or IM x 1 to 2 days
3.Calciuresis.
Loop diuretics Furosemide10 to 20 mg IV every 6 to 12 hours
4.Glucocorticoids(when indicated)
Hydrocortisone 200 -300 mg IV daily for 3 to 5 days
5.Dialysis in renal failure.
6. TTT. OF THE CAUSE.
PHOSPHORUS
Homeostasis
Factors Regulating Phosphorus Homeostasis

Parathyroid hormone.
1 ,25(OH)2D3.
Extracellular fluid volume.
Dietary phosphate intake.
phosphatonins .
Renal function.
HYPERPHOSPHATEMIA
. CAUSES:
Increased renal reabsorption:
Renal causes : Hyperphosphatemia occurs almost exclusively with
impaired glomerular filtration rate.
Endocrinal causes :
hypoparathyroidism, pseudohypoparathyroidism,
acromegaly, thyrotoxicosis.
Extracellular shift : DKA, Acidosis(Metabolic ,Respiratory).
Execessive tissue damage : Massive release from intracellular stores:
tumor lysis syndrome, rhabdomyolysis.

Iatrogenic causes : Overdose of vitamin-D derivatives, phosphate-


containing enemas.

pseudohyperphosphatemia: in paraprotenemias .
Lab Studies
Serum total & ionized calcium
Serum magnesium
Alkaline phosphatase:
BUN and creatinine levels .
Parathyroid hormone levels.
Vitamin D metabolites
Urinary cAMP
HYPERPHOSPHATEMI
A
Treatment:
Acute symptomatic hyperphosphatemia
Intravenous volume repletion with normal saline.
ins/glu to enhancecellular uptake;
dialysis but this is limited due to intracellular loca.

Chronic:
Dietary restriction and phosphate binders.
Treatment (chronic)
if possible treat the underlying cause .

1.phosphate Dietary restriction


2. Phosphate binders.
3. Dialysis .
Phosphate binders
Calcium based

Non-Calcium based

such as aluminum hydroxide,, lanthanum,


and sevelamar (Renagel), reduce the
amount of available phosphorus, thereby
decreasing phosphate absorption and
serum phosphate levels.
Hypophosphatemia
Major causes of hypophosphatemia
Internal redistribution
Increased insulin secretion,
Acute respiratory alkalosis, Hungry bone syndrome

Decreased intestinal absorption


Inadequate intake, Antacids containing aluminum or magnesium,
Steatorrhea and chronic diarrhea,
Vitamin D deficiency or resistance
Increased urinary excretion
Primary and secondary hyperparathyroidism
Vitamin D resistance,Hereditary hypophosphatemic rickets
Oncogenic osteomalacia,Fanconi syndromeOther - osmotic
diuresis, acetazolamide, acute volume expansion
Signs and symptoms of hypophosphatemia
CENTRAL NERVOUS SYSTEM metabolic encephalopathy
:Typical presenting symptoms include irritability and paresthesias, which can
progress to confusion, seizures, delirium, and coma .

CARDIOPULMONARY SYSTEM Myocardial contractility


may be impaired with ATP depletion leading to congestive heart failure and
respiratory failure due to weakness of the diaphragm .
SKELETAL AND SMOOTH MUSCLE
proximal myopathy (affecting skeletal muscle) and dysphagia and ileus (affecting
smooth muscle). severe phosphate depletion can lead to rhabdomyolysis .

HEMATOLOGIC DYSFUNCTION (severe phosphate depletion )


Red blood cells A reduction in intracellular ATP levels increases erythrocyte
rigidity, predisposing to hemolysis .
White blood cells Diminished intracellular ATP levels reduces both phagocytosis
and granulocyte chemotaxis .
Platelets Defective clot retraction and thrombocytopenia.
Magnesium Homeostasis
Mg is present mainly intracellular.
No hormonal regulation of intestinal or renal (re)absorption.
LOH is the main site for Mg homeostasis.
All diuretics cause Mg especially loop diuretics, except acute thiazide or
amilorideMg.
Hyperaldosteronism, volume expansion Mg.
Chronic metabolic acidosis Mg.
& PTH Mg PTH.
Mg K and Ca.
Mg is a Ca CB, Mg intracellular Ca bronchospasm
VC
arrhythmias
metabolic S.
Mg has neuro-inhibitory action.
Mg has favorable lipid profile, same action of statins.
Mg has many therapeutic indications esp. eclampsia, acute severe asthma,
arrhythmias, ms cramps, renal stones, antacids.
Hypomagnesemia

In most cases of , hypomagnesemia is a result of renal or


intestinal magnesium wasting ,which eventually leads
tototal body magnesium depletion.

In the majority of patients ,hypomagnesemia is the result of


an acquired magnesium wasting disorder.

Hereditary causes of hypomagnesemia are less frequent.


Causes of hypomagnesemia
1)Causes related to redistribution from
extracellular to intracellular
Hungry bone syndrome . H
Treatment of diabetic ketoacidosis .D
Ethanol withdrawal & dependence . E
Refeeding syndrome .F
Acute pancreatitis .B
Total parenteral nutrition B
2)Causes related to gastrointestinal losses.
Diarrhea .D
Vomiting and nasogastric suction . G
Gastrointestinal fistulas and ostomies .G
Starvation ( intake ) . I
3)Causes related to renal losses

Inherited renal tubular defects


Gitelman syndrome .G
Classic Bartter syndrome . B
Familial hypomagnesemia with hypercalciuria or hypocalcuria.
Drugs
Diuretics -Loop diuretics, osmotic diuretics, and chronic use of thiazides
Antimicrobials -Amphotericin B, aminoglycosides, capreomycin, vancomycin,
foscarnet and pentamidine, . A & B
Chemotherapeutic agents -Cisplatin .C
Immunosuppressants -Tacrolimus and cyclosporine .C
Volume expansion. .E
Primary hyperaldosteronism.( aldo ) .A
Polyuric phase of acute tubular necrosis . A
Post obstructive diuresis. D
hypoparathyroidism .H
Neuromuscular irritability & CNS hyperexcitability.
-Hyperactive deep tendon reflexes.
-Muscle cramps.
-Muscle fibrillation.
-Trousseau and Chovstek signs.
-Dysarthria and dysphagia.
-Irritability.
-Disorientation.
-Psychosis.
-Ataxia, vertigo, nystagmus, and seizures.
DONT FORGET
*HYPOKALEMIA CANNOT BE
CORRECTED UNLESS
HYPOMAGNESEMIA HAS BEEN
CORRECTED
Hypermagnesemia
Causes of hypermagnesemia
Acute renal failure .
chronic renal failure with Ingestion of Mg-containing
substs such as vitami, antacids, cathartics, enemas .
Excessive intravenous infusions of Mg in patients
treated for eclampsia, asthma or cardiac arrhythmias.
Release of intracellular magnesium e.g. tumor
lysis syndrome, Rhabdomyolysis, massive hemolysis.
Lithium intoxication decreases urinary excretion.

C/P : Neuromuscular depression & inhibition.


GIT
Dysphagia

To solid & fluid Intermittent


To solid & progressive =functional
=mechanical =oesophagial spasm (+ chest pain)
-Peptic stricture (+H burn) Achalasia (+ resp distress)
-Carcinoma (old)
Achalasia of the cardia
Failure of relaxation dt degeneration of the myenteric
(Auerbachs) plexus.
In South America dt Chagas dis. ( Trepanosoma Cruzi).
C/P; , Intermittent functional dysphagia To solid & fluid.
Inv;
CXR absent gastric air bubble.
Ba swallow parot beak appearance.
manometry LOS pressure > 30 mmhg.
Complications; SCC.
TTT;
1. Botulinum injections are most effective of all the options for
relieving a lower oesophageal sphincter.
2. Nifedipine, nitrates or sildenafil are used but less effective.
3. Surgically, Hellers oesophageal myotomy via abdominal
incision or laparascopically.
Achalasia
GERD
C/P; Ht burn, dysphagia, cough, asthma.
Oesophagitis is present in half of GERD patients.
Approach to pt with GERD;
1. Trial of PPI for 1 month,
2. If failed Upper GI endoscopy.
3. PH monitoring; done pre &post-operative ,refractory.
TTT:
1. PPI.
2. Fundoplication.
Complications; Barrets oesophagus.
Alarm symptoms to do upper GIT endoscopy;
Anemia, wt loss, dysphagia, odynophagia.
Barrett's oesophagus
Columnar replaces Squamous in Barretts oesophagus. This
is also known as small intestinal (columnar) metaplasia with
goblet cells.
There is increased risk of oesophageal adenocarcinoma.
Screening;
1. GERD with persistant symptoms at 50 years esp white male.
2. Metaplasia /3 yrs.
3. Low grade dysplasia /1 yrs.
4. High grade dysplasia radiofrequency ablation or
endoscopic mucosal resection.

NB: does not regress with anti H pylori ttt.


The most common causes of infectious esophagitis
include candida, cytomegalovirus (CMV), and
herpes simplex virus.
Candida esophagitis endoscopy white mucosal
plaque-like lesions are noted. Confirmatory biopsy
yeasts and pseudohyphae .culture reveals Candida.

herpes simplex virus: upper endoscopy the


earliest manifestation is a vesicle, although it is rarely
seen. The lesions coalesce to form ulcers (usually less
than 2 cm), frequently with normal-appearing
intervening mucosa. The ulcers are well circumscribed
.

CMV infection, ulcers tend to be linear or longitudinal


and deeper & giant.
medication-induced esophagitis
Queen paid N Cash
quinidine
potassium chloride
Alendronate .
iron sulfate .
doxycycline, tetracycline.
NSAIDs.
C vitamine.
Mallory-Weiss tear
caused by forceful or prolonged vomiting
lower oesophageal tear.
vomiting bright red blood or by passing blood
in the stool.
If vitally stable. Discharge.
Cancer esophagus
Predisposing factors;
1. GERD. Barretsadeno.
2. Achalasia SCC. Plummer vinson $.
3. smoking, alcohol.
4. Coeliac dis
5. Tylosis (familial palmar/planter icthyosis).
C/P; wt loss, progressive dysphagia To solid .
Pathology; cases in the lower 1/3 , all scc except with barrets.

Inv;
Ba swallow shouldering .irregular filling defect, rat tail appearance,
Oesphogoscopy & Biopsy
Ttt surgery in only 1/3 suitable, if resectable.
Palliative ttt for dysphagiaThe best is stent insertion. rapid &
effective than radio & chemo
Cancer oesophagus
Peptic ulcer diseases
Causes;
1. H pylori.
2. NSAIDS.
3. Infections; CMV, HSV,
4. Drugs; bisphsphonate, KCl, steroids, MMF, cocaine, chemotherapy.
5. Others; chrons, ZES, Ca, ischemia, irradiation.
TTT;
1. Acid supressing drugs; antacids, H2 blockers, PPI.
2. Mucosal protecting agents; sucralfate, PG analogue.
3. TTT of H pylori.
4. Avoid NSAIDS.
5. Surgery if complicated;

PPI irreversible inhibition of HK ATPase, before breakfast,


side effect; 1-Cahip fracture,
2- anemia dt fe absorption, B12 dt IF.
3-omeprazole, lanzo enz inhibitors.
4-pneumonia
5- gynecomastia.
Complications of surgery;
1. Blind loop S; bact overgrowth.
2. Dumping;
- early 15-30 min hypotension dt fluid shift
2ry to hyperosm. Content.
- late 1-2 hr dt hypoglycemia.
4. Bile reflux gastropathy.
Complications of PU;
1. Bleeding
2. Perforation
3. Penetration
4. Gastric outflow obstruction.
H pylori
is a gram-negative microaerophilic rod
produce urease. Urease produces ammonia from
urea.
Incidence; 80% in developing countries, 20% in
industrialized.
Complications;
1. Chronic active gastritis.
2. Peptic ulcer disease.
3. MALT lymphomas.
4. Adenocarcinoma.
5. Extragastric; IsHD, enteropathic arthropathy, acne
rosesea, urticaria.
Diagnosis;
1. Non invasive;
- stool Ag.
- urea breath test(14C urea ingestion & breath Co2)FU after ttt.
- serology; best screening but remain +ve after ttt for > 6 months
so not a good indicator of acute infection.
2. invasive (endoscopy);
- rapid urease (urea on specimen & pH colour indicator).
- pathologymicroscopic examination.
-culture.
TTT;
Triple therapy e.g. PPI 20 mg x2 + clarithromycin 500 mg x2 +
amoxacillin1 g x2.
Or quadriple therapy (+ tetra).
For 14 days then continue PPI for another 2 wks in DU & 6 wks
in GU.
New onset dyspepsia
Ask about alarm symptoms

-ve
+ve
Non invasive test
Refer for Ba or endoscopy
for H pylori


+ve - ve
Eradication Ttt for 4 wks
H2 blocker or
& confirm eradication by UBT
If failed Ba or endos PPI for 1 month If no response
New onset dyspepsia or GERD
(ask if NSAIDs)
If not do Non invasive test
for H pylori

-ve H pylori +ve H pylori


Eradication Ttt for 4 wks
Life style advise & & confirm eradication by UBT
Give PPI for 1months If failed Ba or endos

+ve response
Give the least dose which relief the
symptoms
-Ve response
or refer for Ba or endos
On demand TTT.
MALT lymphoma
Low grade B cell lymphoma.
most commonly found in the gastrointestinal tract
(stomach is the commonest).
associated Helicobacter Pylori infection.
TTT; respond in over 80% of cases to helicobacter
eradication.
A proportion of patients will not respond to
eradication therapy alone and will go on to more
conventional anti-lymphoma therapies such as
such as cyclophosphamide, chlorambucil,
nucleoside analogues, or radiotherapy.
NSAID- PU
COX 2 inhibitors such as celecoxib are used as
analgesics instead of NSAIDS in patients who are
at high risk of upper GI dyspepsia or ulceration.
However, there remains an increased risk of
ulceration, though less so compared to NSAIDS.
COX 2 inhibitors shown to increase the mortality
rate of patients with CHD by blocking the
antithrombotic effects of certain prostaglandins
Misoprostol (prostaglandin analogue) used in
treatment of NSAIDs gastric ulceration.
.
ZES (Gastrinoma)
GastrinomaPUD ( panc. head 60% & duod. 30%)
Sporadic or MEN1.
60% malignant.
40% metastatic at presentation or multiple.
C/P: Abdominal pain, Ulcers, diarrhea .
When to suspect.
Recurrent DU.
Postbulbar or multiple ulcers
Resistant to TTT.
Pt. with stigmata of MEN I.
Inv;
1. Fasting gastrin level > 1200pg/ml+ low PH. (most sens. & specific).
2. Secretin stimulation test.(secritin paradoxically incr. gastrin response).
3. Tumour localization; CT, MRI, OCtreotide scan, Endo.US, laparotomy.
TTT ; PPI, octreotide, surgery, chemotherapy.
Pernicious anemia
Autoimmune disease Ab to parietal cells
HCl, intrinsic factor gastric atrophy, achorhydria,
macrocytic anemia dt B12 def., pancytopenia.
Incidence; 1 % of the population > 50 years.
C/P; , FH, >50 years old, macrocytic anemia dt
B12 def., pancytopenia other autoimmune
manifestations; vitiligo, Addison, thyroid dis.
Complication; gastric cancer.
Chronic gastritis
Type A; autoimmune, anemia, affect gastric body.
Type B; H pylori, antrum
Menetriers disease
Hyperplasia of mucin producing cells.
C/P; protein loosing enteropathy, hypochlorhydria.
Ttt; antacids for ulcerations, high protein diet, total
gastrectomy.
DD of thick gastric folds;
- Menetriers, ZES,
- Lymphocytic, esinophilic , granulomatous gastritis.
- Malignancy, MALT lymphoma.
Protein loosing enteropathy.
Gastric polyps; (rare & mostly benign).
1. Hamartomatous ; PJS..
2. Adenomatous; pre-malignant
Gastric carcinoma
Predisposing factors;
1. H pylori.
2. Pernicious anemia.
3. Adenomatous polyps.
4. Partial gastrectomy.
5. Bl group A.
6. Smoking.
7. Alcohol, spicy.
Adenocarcinoma, antrum.
Paramalignant; acanthosis, dermatomyositis.
Gastroparesis
Causes;
1. DM.
2. Autonomic Neuropathy.
3. Vagotomy.

C/P ;delayed gasric emptying , vomiting


Inv; isotope empting studies.
TTT; prokinetics; metoclopramide,
erythromycin, low fat diet ,low fibre , small
frequent meals. feeding jejunostomy.
Vomiting
Higher centers
(see,smell)

ICT
CTZ VC
Bact toxins
5HT3 D2 M1

chemotherapy
5 HT3 H1
Metabolic Visceral aff Vestibular syst
Drugs; -acidosis (gastric, IO, ishemia) preg, motion sickness
-morphin, Hypoxia
-digitalis, Uremia
-estrogen, LCF
-ethanol ehtanol
Malabsorption
Causes;
1. Preabsorptive;
1- gastric; post-gastrectomy.
2- pancreatic;e.g. chronic pancreatitis, Cystic fibrosis.
3- bile acid e.g.cholestasis, Ileal resection,Drugs as cholestyramine.

2) Absorptive; (Impaired mucosal absorption)


- Intestinal resection or bypass
- Inflammation, infiltration: Crohn's disease, Celiac sprue, Amyloidosis,
Lymphoma,Radiation,
- Infections; giardiasis, Tropical sprue, Whipple's disease
- Genetic , Agammaglobulinemia, abetaliproteinemia.

3) Post absorptive; (Impaired nutrient delivery from intestine)


- Lymphatic obstruction, ectasia, Lymphoma .
- Circulatory disorders; Constrictive pericarditis, Mesenteric artery ath.
- Endocrine Adrenal insufficiency Hyperthyroidism.
Management of malabsorption
1. Lab; CBC, Ca, Fe, folic acid, B12, cholesterol, Alb.
2. To prove steatorrhea;
- qualitative; sudan III
- quantitative; stool fat>7 g/d or 6% of dietary fat.
3. endoscopic biopsy.
- non caseating granuloma crhons
- congo red +ve amyloidosis.
- Malignant cells lymphoma.
- PAS +ve org whipple, DD; M avium intracellular.
-foamy intestinal mucosa
- clubbed villi, dilated lymph int lymphangectasia.
- absent villi, no plasma cells Agammaglob
- - absent villi, mononuclear infiltrate, hypertrophy of crypts celiac.
4- Urinary D Xylose test; assess CHO absorption from proximal SI, 25 g D
xylose oral & collect urine for 5 hr, n=>4.5g D xylose in urine.

NB; causes of villous atrophy;


Celiac, tropical sprue, bact overgrowth, drugs as neomycin, laxative.
Whipple, hypogamma, lymphoma.
Other specific tests;
5- for Bacterial overgrowth;
6- for exocrine pancreatic function;
- therapeutic test with pancreatic enzymes
- secretin test (most sens)
- fecal elastase.
- serum trypsinogen.
Bacterial overgrowth
Definition; Symdrome ccc by diarrhea, steatorrhea, macrocytic anemia dt
proliferation of colonic bacteria in the small intestine e.g. E coli, bacteroids.
Pathogenesis;
- decongugation of bile acidsmicelle formation steat.
- consume B12 (not folate)
- enterotoxins diarrhea
Causes;
1- Anatomical stasis; e.g. Afferent loop stasis/ blind loop/ strictures / fistulae.
2- functional stasis; DM, Scleroderma intestinal pseudoobstruction.
Diagnosis;
- macrocytic anemia (B12, folate).
- jejunal aspirate for quantitative culture (> 105 ) .(gold standard)
- H+ breath test (oral lactulose for early H+ detection in breath)
- radioactive Carbon C14 breath test (oral bile salts for early CO2 in breath).
TTT; surgical correction
antobiotics; tetracyclin, metronidazole, ceph, amox/clav for 3 wks or 1
wk/month if recurrent.
probiotics
Coeliac Disease
Coeliac disease (gluten sensitive enteropathy) is relatively common in
Ireland, caucasiens, incidence1%.
Gluten is found in wheat, rye and barley .
C/P; asymptomatic diarrhea, steatorrhea, 2ry lactase def, anemia, bone
dis.
Complications; GI & non GI cancer, intestinal lymphoma.
Associations; Dermatitis herpetiformis, IDDM, Ig A def, infertility, IBD,
thyroiditis, fibrosing alveolitis, hyposplenism, Ig A nephropathy.
Lab;
- Folate levels are usually normal but not B12 in coeliac disease (in Crohns
both are decreased).Fe, Ca
- CBC: Macro or micro, howel jolly bodies.
- antigliadin, antiendomesial, tissue transglutaminase, antireticulin
Jejunal biopsy in coeliac disease typically shows:
hypertrophied crypts of Lieberkuhn,
villus atrophy,
an increase in intraepithelial lymphocytes.
TTT; gluten free diet , use rice, oat, potatoes, Fe Ca, Folic supplementation,
if resistant steroid.
Dermatitis herpetiformis
- manifests as a pruritic rash.
- Anti-endomysial antibodies may be present.
- TTT; Dapsone (diaminodiphenyl sulfone)
often is used initially;
sulfapyridine is substituted in patients
unable to tolerate dapsone.
Tropical sprue

Cause; infection as klebsiella, E coli .


Diagnosis; history of living in tropical areas +
biopsy ( as celiac but less villous atrophy, more
infiltration, same severity throughout intestine.
Ttt; tetracyclin for up to 6 months, folate.
Whipple disease
Chronic multisystem dis ass with;
- GIT:diarrhea, steatorrhea., wt loss,
- Arthoropathy: arthralgia, migratory large joint
- CNS; oculomusticatory myorrhysmia,
myoclonus, tremors, extra & dementia.
- Lymphadenopathy.
Cause; trophoryma whipplei (G+ve bacilli)
Biopsy; PAS +ve MQ in lamina containing the
bacilli. (DD M. avium)
Ttt; sutrim DS for 1 year, .
Short Bowel S
Causes e.g. crohns, mesenteric Vascular dis, trauma,
bariatric surgery.
C/P;
1. Diarrhea dt bile acid, transit time, bact overgrowth
2. Lactose intolerance
3. Enteric hyperoxaluria dt
- bile acid colonic permeability
- FA bind Ca colonic Oxalate absorption.
4. GB cholesterol stone dt bile acid
5. Dumping S
TTT; opiate, low fat, medium chain triglycerides
(MCTs), low lactose, vit replacement,
int. transplantation, trophic hormones as GLP-2.
Ileal dis

Mild
Bile acid diarrhoea Severe
dt bile acid induced
Cl secretion from the colon
FA steatorrhea
Ttt; cholestyramine Ttt; low fat diet, MCT
NB; Long chain FA> 12 C require micelle formation for
absorption.
Medium chain FA 8-12 C does not need micelle
formation, used in ttt of malabsorption e.g. coconut oil.
Short chain FA <8C
- formed by colonic bacteria
- e.g. acetate, propionate, butyrate.
- Nutrient to the colon,
- with antibiotic use Ab induced colitis & diarrhea.
Lactase deficiency;
Causes;
- 2ry e.g. celiac dis.
- 1ry; 15% of Caucasians, 85-100% in
blacks.
C/P; Asymptomatic, bloating, abd cramps
TTT; lactose free diet.
DD; IBS.
<2 wks.
Acute diarrhoea
90% infectious.
Toxigenic (watery); staph, B cereus , vibrio chol,.cl bot, cl
perfring, E coli,
Invasive (bloody); (YES CAN CAUSE Vibrio
parahemolyticus, vulnificus) yersinia, E coli, E coli o157, E hist,
salm, shig, campylo, cl diff, vibrio parahemol, vulnificus, CMV
colitis.
Traveler diarrhea E coli, campyl, shigella, salmonella, ttt;
cipro, sutrim.
Hamburger E coli o157:H7
Seafood vibrio, yersinia.
Fried rice B cereus.
Ttt;
- symptomatic
- metronidazole
- cipro + culture
Campylobacter; bloody, GBS ,ttt =erythro
Yersinia; Rt ileac, reactive arthritis, EN,
myocarditis.ttt; quinolone
E coli o157:H7 HUS.
Amebic desentry metro .
Antidiarrheal should be avoided in febrile
dysentery.
Chronic diarrhea
1. Watery;
A) secretory i.e. does not with fasting, stool OG
<25. e.g. stimulant laxative, toxins, resection,
hormone producing tumors as VIPoma.
B) osmotic; with fasting, stool OG > 50 eg. Osm
laxative as Mg, lactase def, lactulose, sorbitol.
2. Inflammatory; pain, fever, bleeding, fecal
leucocytes e.g. IBD, microscopic colitis, radiation,
ischemic, malignancy.
3. Steatorrhea; sudan III, fecal fat > 7 g/d.
4. Dysmotility as IBS, DM, thyrotoxicosis.
5. Factitious e.g. Munchausen, eating disorders.
NB
1) stool osmolar Gap
Measured Stool osmolarity= 290-300 mosm/kg
water
Calculated= 2(stool Na +K)
Stool osmolar Gap= Measured- Calculated
if < 25= secretory diarrhea
25-50= mixed
> 50= osmotic
2) Prolonged fasting > 24 hrs;
if diarrhea osmotic (diet)
if not secretory.
Investigations for laxative abuse;
1. S. K
2. Melanosis coli on endoscopy. DD;
carcinoid.
3. Ba enema; loss of haustration
Inflammatory Bowel disease
UC CD
site Colon, wash back ilitis All GIT, perianal except rectum
complications Toxic megacolon Fistulas, strictures
Macroscopic pseudopolyps transmural
Narrow short colon with Cobble stone
loss of haustrations Skip lesions
Microscopic Crypt abcesses Non caseating granuloma
lab P ANCA 70% sens. ASCA ; 90% specific, 70% sens.
Ba enema loss of haustrations Cobble stone, string sign
TTT 5ASA oral or rectal Antibiotics
Steroid rectal, oral, IV Budezonide, Steroid oral, IV
Aza or 6MP Aza or 6MP, Methotrexate
IV cyclosporin or Infleximab, IV cyclosporin
infleximab
Ulcerative colitis
Crohns
affect any
portion of the
gastrointestinal
tract from the
mouth to the
perianal area.
Its transmural
inflammatory
nature
If suspecting IBD upper GI series with small bowel follow-
through (UGI/SBFT) and colonoscopy, with biopsies.
Features diagnostic of Crohn's disease include;
- evidence of small bowel involvement on UGI/SBFT, or
- non-caseating granulomas.
- Significant perianal disease.
Extraintestinal feature include;
-Skin., EN, clubbing, Pyoderma Gang.,
- Eye. uveitis, episcleritis,
-Articular., osteoporosis, ankylosing spondylitis, sacroiliitis,
peripheral arthritis,.
-GIT., gall stones, PSC, cholangiocarcinoma,
- Renal ,Ca oxalate urinary stone , amyloidosis. --- ---
Thromboembolism.
Cancer risk in IBD;
Risk factors; duration, extent, FH. Screening; after 8-y of
pancolitis or 15 y of lt sided colitis colono + biopsy/1-3 yr.
C/p; majority has bloody diarrhea and mild cramping, with minimal systemic
symptoms such as fever, weight loss, and anorexia.
TTT
depends upon both the extent of colonic involvement and the severity of the disease
process at presentation.
1. for ulcerative proctitis = topical 5-aminosalicylate suppositories or steroid foams.
2. Patients with left-sided colitis often respond to topical enemas.
3. Pan colitis ( extends beyond the splenic flexure).
A) mild to moderate;
- oral 5-ASA or sulfasalazine + topical 5-ASA or steroid enemas .
- add oral prednisone (40 to 60 mg/day) in: 1- more severe symptoms and
2- failed oral 5-ASA and topical therapy.
These agents often require 3-6 weeks to exert their maximal benefit.
Once remission is achieved, the drug dose can be tapered to maintenance levels.
Disease severity Medication Daily dose

Mild-to-moderate disease
Mesalamine suppository 1000mg at night

Mesalamine enema 4 g at night


Hydrocortisone enema 100 mg at night
Mesalamine e.g. Pentasa 500to 1000 mg PO 4 times /d
Sulfasalazine 1-1.5 g PO four times/ d
oral 5-ASA + 5-ASA enemas/steroid enema
Prednisone 40 to 60 mg PO /d
Maintenance therapy in UC
to reduce the risk of relapse
Disease type Medication Dose

Proctitis Mesalamine suppository 1000 mg /night to /3 night

Distal colitis Mesalamine enema 4 g /night to / 3 night

Left-sided & pancolitis Sulfasalazine 500 mg PO four times daily


Mesalamine EC (Asacol*) 400 to 800 mg PO 3 times/d
Olsalazine 500 mg PO twice daily
Sulfasalazine/oral 5-ASA + 5-ASA enema

Steroid-dependent colitis Mercaptopurine 50 mg up to 1.5 mg/kg PO/day


Azathioprine 50 mg up to 2.5 mg/kg PO/day
Methotrexate 10 to 15 mg PO once per week
B) Severe active disease
B) Severe active disease is a Disease Medicatio Daily dose
medical emergency defined as > 6 bloody
stools/ day & signs of systemic toxicity (HR severity n
> 90, temp. 37.8 , Hb 10,5, ESR >30) Severe active disease
- Hospitalization &intensive inpatient TTT .
- pulse IV steroid, either hydrocortisone 100 On steroids Methylprednisolone 40
mg 4 times daily ,or methyle prednisolone recently to 60 mg IV once daily
60 mg daily .if no improvement by day 3 or Hydrocortisone 100
there is subsequent deterioration mg IV/ 6 hrs or IVI
-IV cyclosporine. As a rescue therapy before Cyclosporine
surgery.2 mg / kg / day. Infliximab
-infliximab if cyclosporine is #contra. Toxic IV corticosteroids
-IV fluids & electrolytes replacem. megacolon Broad-spectrum
-IV antibiotics if infection is considered. antibiotics
-surgery. Chronic Mercaptopurine,
active Azathioprine
disease Infliximab
(steroid
refractory)
Toxic megacolon
Colonic diameter > 5.5cm or Caecum > 9 cm in
abd plain X ray .
Management medical for up to 48 hrs
NPO.
Clinical, laboratory, & radiological monitoring.
IV fluids & electrolytes replacement .
IV antibiotics if infection is considered.
If no response within 48hrssubtotal colectomy
Surgery;
UC Subtotal colectomy& end ileostomy or
ileal pouch anal anastomosis (IPAAS).
Indication;
intractable( not responding to intensive medical ttt.)
Fulminant , acute severe on top of chronic .
Poorly controlled Complicated extraintestinal.
Toxic mega colon,
Impact of colectomy on UC manifestations
Stay the same; sacroilitis, sclerosing cholangitis, uveitis.
Positive response; pyoderma, peripheral arthritis, perianal dis.
Complication; pouchitis in 1/3. C/P; diarrhea, arthralgia, fever. Biopsy
differentiate bet CD & true pouchitis. Prevention with probiotics. Ttt of
pouchitis; antibiotics.Metronidazole 1x3or ciprofloxacin 250 1x2
CD; - surgical resection is contraindicated.
- stricturoplasty, diversion colostomy for perianal dis
- total proctocolectomy & ileostomy (but IPAAS is
contraindicated dt pouch failure)
Antiinflammatory drugs in CD, UC;
1. 5ASA; for induction & maintenance
- e.g. - sulfasalazine (sulfapyridine + 5 ASA).
Side effect; oligospermia, allergic, idiosyncrasy,
agranulocytosis, rash, folate def.
- Enteric coated; mesalazine (Pentasa,
Asacol)2-4g.
2.Azathioprine 50 mg up to 2.5 mg/kg PO/day
Is the best maintenance ttt for steroid dependent
3.Steroids;
for induction only not maintenance.
Infliximab;
chimeric monoclonal antibody to TNF.
Used in;
1- refractory IBD, not fit for surgery.
2-perianal Fistulizing Crohn's;
3-RA, Psoriasis, cryo, not AS.
Dose; IV 5 mg/kg at 0, 2, 6 then /8 wks.
Side effects;
- Ab to infliximabe resistance & reaction so given with
Aza or methotrexate + hydrocortisone before the dose.
- lymphoma.
- allergic reactions, serum sickness
- infection esp TB
- demylenating disorders.
Fistulizing Crohn's;
- elemental diet is as effective as steroids in induction of
remission in Crohn's not in UC, better than bowel rest & TPN.
- antibiotics metronidazole or cipro.
- Aza, metho
- infliximab
- TPN

Oral lesions ; Aphthous ulcerations are the most common ,


usually occur with coexistent intestinal disease.
TTT;
- respond to treatment directed at the intestinal disease.
- Topical therapy consisting of Topical hydrocortisone or
carboxymethylcellulose or topical tacrolimus or topical
sucralfate may be effective for local symptom relief.
IBD & pregnancy.
- Dis activity not affected by pregnancy.But pt
should be in remission for 6 months before
conception.
- CS is preferred in perianal dis.
- Drugs;
- 5 ASA, steroids safe but use folate with sulfas.
- Aza, 6MP safe with consent.
VIPomas
VIPomas [vasoactive intestinal peptide (VIP)] .
C/P;WDHA
- watery diarrhea, stool volume > 700 ml/d.
- hypokalemia, N-AG acidosis
- achlorhydria,
+ hyperglycemia , hypercalcemia in 2/3,
flushing and hypokalaemia due to diarrhoea.
Treatment; correcting volume and electrolyte
abnormalities by using potassium chloride and
sodium bicarbonate. Octreotide controls diarrhea
in 80% of cases. Glucocorticoids reduce symptoms
in 50% of patients with VIPoma.
Carcinoid tumours
C/P After hepatic metastasis;
Forgut cushing, acromegaly.
Midgut (ABCDEF); asthma, belly abd, cor pulm
TR>TS, diarrhea, endocardial fibrosis, flushing
(Carcinoid syndrome)
Bronchus; ACTH, GHRH. (cushing, acromegaly).
Diagnosis; raised urinary 5-HT levels. A precursor of
5HT, tryptophan is highly metabolised and
consequently niacin deficiency (pellagra) occurs. The
three D's dementia, dermatitis and diarrhoea occur.
Ttt; resection, octreotides, methysergide,
cyproheptadine, phenoxybenzamine for flushing.
Glucagonoma; WARDS
- wt loss
- anemia
- rash. NME
- DM, diarrhea
- DVT.

Somatostatinoma SSS
- sugar (DM)
- steatorrhea
- stones GB
Ischemic colitis
Non- occlusive
Rectosegmoid
Elderly, bloody diarrhea& pain .
XR abd; thumb printing dt mucosal &
submucosal oedema.
colonoscopy; narrowing, ulceration.
Angio; not usually indicated.
Pseudomembranous colitis
Antibiotics as clindamycin, ampicillin,
cephalosporin, tetracyclin flurishing of Cl
difficile toxins A, B.
C/P; fever , bloody diarrhea
Diagnosis; C difficile toxin in stool.
Ttt; discontinue the antibiotic.
Give metronidazole, oral vancomycin.
Irritable bowel syndrome
C/P; pain relieved by defecation, Never bleeding
/rectum, Never wt loss, Never nocturnal.

(ROME II criteria) 12 weeks or more in the last year


of abdominal discomfort or pain that has two of the
following three features:.
(1)Relieved by defecation
(2) Associated with a change in frequency of stool
(3) Associated with a change in consistency of stool
Ttt;
- dietary therapy; lactose free, high fibre.
symptomatic ttt.
Chronic Colonic pseudo-obstruction

C/P; of mechanical obstruction but without occlusion


of the lumen.
Causes;
1. Endocrinal causes; DM, hypothyroidism,
hypoparathyroidism, hypoadrenalism.
2. Neurologic; parkinsonism, ms dystrophy, MS.
3. Rheumatologic; progressive systemic sclerosis.
4. Infection; Chagas dis.
5. Medications; TCA, clonidine, phenothiazines, narcotics
6. Amyloidosis
7. Paramalignant.
Colonic polyps

Non-neoplastic
1. Hamartomatous
Neoplastic
(jej polyps, PJS, cowden)
1. Adenomatous (FAP, Gardner)
2. Hyperplastic.
2. Papillary adenoma
3. Inflammatory.
3. Villous papilloma
(Bilh, lymphoid, pseudopol of UC)
4. carcinoid
4. C.Tissue polyps
(lipoma, fibroma, leimyoma)
Familial adenomatous polyposis;
- AD, chr 5
- > 100 polyps
- 100% premalignant.
- extracolonic tumours; thyroid, pancreatic, duodenal.
- 1/3 has no FH.
- polyps may occur in UGIT.
- total colectomy should be performed as soon as possible following
diagnosis.
Gardner S= FAP when extracolonic tumours predominates as
osteomas, fibromas, neurofibromas, lipomas, dermoid cysts, desmoid
tumours, retroperitoneal fibrosis.
cowden (1/3 has goitre,10%cancer, fibrocystic dis of breast)
Peutz-Jeghers syndrome;
- AD.
- dt gene mutation of the STK11 in most cases.
- hamartomatous polyps + mucocutaneous melanocytic macules, perioral
freckles, alopecia, nail dystrophy.
HNPCC
AD, chr 2,3
Associations; other cancers; endometrial, ovarian, gastric
Amesterdam criteria for genetic testing;
1. 3 or more 1st degree family members with colorectal ca.
2. Colon ca over 2 generations.
3. At least 1 before<50yrs.
4. Pt with CRC or endometrial ca below<40yrs
5. Pt with CRC with 1st degree CRC or extracolonic ca
If genetic testing +ve colonoscopy
Colorectal cancer
Predisposing factors;
1. High fat diet, low fibre
2. FAPC
3. HNPCC
4. UC, CD
5. Adenomatous polyps
6. Black race, male sex
Preventive measures;
1. NSAIDs
2. High Ca diet
3. HRT (E)
C/P; include iron def anemia, sepsis with streptococcus bovis.
Inv; double contrast Ba enema, colonoscopy, CEA.
Ttt; resection anastomosis, abdominoperineal resection &
colostomy for distal rectal tumours.
Cancer colon
Screening of colorectal cancer
People > 60 yrs
- colonoscopy/10 yr or
- occult B in stool/ y + sigmoidoscopy /5 yr.
People who have adenoma removal & biopsy then
according to the size & number
1 or 2 polyps < 1 cm colonoscopy/5 yr .
3 or 4 polyps < 1 cm or one > 1cm colonoscopy/3yr.
> 5 polyps < 1 cm or 3 or more > 1cm colonoscopy/3yr.
PH of Ca colon,
- colonoscopy after 1 year then after 3years then /5 yr.
FH of two 1st degree relatives with colorectal cancer or FH
of one 1st degree relative with colorectal cancer < 60 yr.
colonoscopy / 3-5 yr beginning 10 yrs younger than the
youngest affected relative.
PH of UC (8 yr pancolitis )or( > 15 yr Lt sided)
colonoscopy then acc.
Extensive colitis with mild inflammation colonoscopy every 3 yrs.
Extensive colitis with moderate to severe inflamm colonoscopy /1 yr
Lt sided colitis colonoscopy every 5 yrs.
(colonoscopy + biopsy )
FH of FAP
- Colonoscopy / 1-2 yr from puberty till 40 yr old then as
average.
- if polyps develop or genetic test +ve colectomy.
FH of HNPCC
- colonoscopy 1-2 yrs from 21 yr ..
Constipation
Types of Constipation
and Causes
Recent onset
DD;
Colonic obstruction Neoplasm; stricture: ischemic, diverticular, inflammatory
Anal sphincter spasm Anal fissure, painful hemorrhoids
Medications Fe, Ca, Al HO, anticholinergics, opiates,diuretics
Chronic
Irritable bowel Constipation-predominant, alternating
Medications blockers, antidepressants +2Ca
Disorders of rectal Slow-transit constipation, megacolon, Pelvic floor
evacuation (dysfunct,; rectal prolapse; rectocele.
Endocrinopathies Hypothyroidism, hypercalcemia, pregnancy
Psychiatric disorders Depression, eating disorders, drugs
Neurologic disease Parkinsonism, multiple sclerosis, spinal cord injury
muscle disease Progressive systemic sclerosis
Abdominal pain
Medical causes;
- alcohol, DKA, hyperlipidemia pancreatitis
- Hyperparathyroid, adrenal insufficiency
- FMF, porphyria, angioneurotic edema
- Lead toxicity, morphin withdrawal.
- Vasculitis as HSP.
- sickle cell an..
Gut Hormones
Secretin is produced by the jejunum. It
relaxes the oesophageal sphincter and also
stimulates pancreatic enzyme secretion.
Gastrin is secreted by the G cells of gastric
antrum. It stimulates parietal cells produce
hydrochloric acid.
Vasoactive intestinal peptide (VIP) promotes
intestinal water and electrolyte secretion.
Chief cells pepsin.
Parietal cells HCl, intrinsic factor.
Surface cells-> HCO3, mucous.
GIT
Dysphagia

To solid & fluid Intermittent


To solid & progressive =functional
=mechanical =oesophagial spasm (+ chest pain)
-Peptic stricture (+H burn) Achalasia (+ resp distress)
-Carcinoma (old)
Achalasia
Failure of relaxation dt degeneration of the myenteric
(Auerbachs) plexus.
In South America dt Chagas dis. ( Trepanosoma Cruzi).
C/P; , functional dysphagia To solid & fluid.
Inv;
CXR absent gastric air bubble.
Ba swallow parot beak appearance.
manometry LOS pressure > 30 mmhg.
Complications; SCC.
TTT;
1. Botulinum injections are most effective of all the options for
relieving a lower oesophageal sphincter.
2. Nifedipine, nitrates or sildenafil are used but less effective.
3. Surgically, Hellers oesophageal myotomy via abdominal
incision or laparascopically.
Achalasia
GERD
C/P; Ht burn, dysphagia, cough, asthma.
Oesophagitis is present in half of GERD patients.
Approach to pt with GERD;
1. Trial of PPI for 1 month,
2. If failed Upper GI endoscopy.
3. If normal Bernstein test.
4. PH monitoring; done pre & post-operative
refractory.
TTT:
1. PPI.
2. Fundoplication
Complications; Barrets oesophagus.
Alarm symptoms to do upper GIT endoscopy;
Anemia, wt loss, dysphagia, odynophagia.
Barrett's oesophagus
Columnar replaces Squamous in Barretts oesophagus. This
is also known as small intestinal (columnar) metaplasia with
goblet cells.
There is increased risk of oesophageal adenocarcinoma.
Screening;
1. GERD with persistant symptoms at 50 years esp white male.
2. Metaplasia /3 yrs.
3. Low grade dysplasia /1 yrs.
4. High grade dysplasia FU /3 months.

NB: does not regress with anti H pylori ttt.


The most common causes of infectious esophagitis include
candida, cytomegalovirus (CMV), and herpes simplex virus.
The diagnosis of Candida esophagitis is usually made at
endoscopy when white mucosal plaque-like lesions are noted.
Confirmatory biopsy shows the presence of yeasts and
pseudohyphae .culture reveals Candida.

The diagnosis of herpes simplex virus: is usually established


with an upper endoscopy. Lesions affect the squamous mucosa
where the earliest manifestation is a vesicle, although it is rarely
seen. The lesions coalesce to form ulcers (usually less than 2
cm), frequently with normal-appearing intervening mucosa. The
ulcers are well circumscribed .

CMV infection, ulcers tend to be linear or longitudinal and


deeper & giant.
medication-induced esophagitis
Queen paid N Cash
quinidine
potassium chloride
Alendronate .
iron sulfate .
doxycycline, tetracycline.
NSAIDs.
C vitamine.
Mallory-Weiss tear
occurs in the mucous membrane typically
in the lower oesophagus.
usually caused by forceful or prolonged
vomiting or coughing.
vomiting bright red blood or by passing
blood in the stool.
Cancer esophagus
Predisposing factors;
1. GERD.
2. Achalasia SCC.
3. smoking, alcohol.
4. Plummer vinson.
5. Barretsadeno.
6. Coeliac dis.
7. Tylosis (familial palmar/planter icthyosis).
C/P; wt loss, progressive dysphagia To solid .
Pathology; cases in the lower 1/3 , all scc exept
with barrets.
Inv; Ba swallow shouldering
Ttt surgery in only 1/3 suitable, improve 5 yr surv to
10%.
Cancer oesophagus
Peptic ulcer diseases
Causes;
1. H pylori
2. NSAIDS
3. Infections; CMV, HSV,
4. Drugs; bisphsphonate, KCl, steroids, MMF, cocaine, chemotherapy.
5. Others; chrons, ZES, Ca, ischemia, irradiation.
TTT;
1. Acid supressing drugs; antacids, H2 blockers, PPI.
2. Mucosal protecting agents; sucralfate, PG analogue.
3. TTT of H pylori.
4. Avoid NSAIDS.
5. Surgery if complicated;
PPI irreversible inhibition of HK ATPase, before breakfast,
side effect; 1-Cahip fracture,
2- anemia dt fe absorption, B12 dt IF.
3-omeprazole, lanzo enz inhibitors.
4-pneumonia
5- gynecomastia.
Complications of surgery;
1. Blind loop S; bact overgrowth.
2. Dumping;
- early 15-30 min hypotension dt fluid shift
2ry to hyperosm. Content.
- late 1-2 hr dt hypoglycemia.
4. Bile reflux gastropathy.
Complications of PU;
1. Bleeding
2. Perforation
3. Penetration
4. Gastric outflow obstruction.
H pylori
, is a gram-negative microaerophilic rod
produce urease. Urease produces ammonia from
urea,
Incidence; 80% in developing countries, 20% in
industrialized.
Complications;
1. Chronic active gastritis.
2. PUD
3. MALT lymphomas
4. Adenocarcinoma
5. Extragastric; IsHD, enteropathic arthropathy, hep
enceph, acne rosesea, alopecia, urticaria,
Raynaulds.
Diagnosis;
1. Non invasive;
- stool Ag
- urea breath test(14C urea ingestion & breath Co2)FU after ttt.
- serology; best screening but remain +ve after ttt for > 6 months
so not a good indicator of acute infection.
2. invasive (endoscopy);
- rapid urease (urea on specimen & pH colour indicator).
- pathology microscopic examination.
-culture.
TTT;
Triple therapy e.g. PPI 20 mg x2 + clarithromycin 500 mg x2 +
amoxacillin1 g x2.
Or quadriple therapy (+ tetra)
For 14 days then continue PPI for another 2 wks in DU & 6 wks
in GU.
New onset dyspepsia
(Exclude GERD,
biliary, IBS, drugs)
>40 yrs, alarm symp

-ve
+ve
Non invasive test
Refer for Ba or endoscopy
for H pylori

+ve - ve
Ttt for 4 wks
& confirm eradication by UBT
H2 blocker or
If failed Ba or endos refer for Ba or endos

alarm symp; of ulcer complications, malig


MALT lymphoma
Low grade B cell lymphoma.
most commonly found in the gastrointestinal
tract (stomach is the commonest).
associated Helicobacter Pylori infection.
TTT; respond in over 80% of cases to
helicobacter eradication.
A proportion of patients will not respond to
eradication therapy alone and will go on to more
conventional anti-lymphoma therapies such as
such as cyclophosphamide, chlorambucil,
nucleoside analogues, or radiotherapy.
NSAID- PU
COX 2 inhibitors such as celecoxib are used as
analgesics instead of NSAIDS in patients who are at
high risk of upper GI dyspepsia or ulceration. However,
there remains an increased risk of ulceration, though
less so compared to NSAIDS.
Misoprostol (prostaglandin analogue) and cimetidine are
used in treatment of gastro-oesophageal ulceration.
COX 2 inhibitors have also recently been shown to
increase the mortality rate of patients with coronary
artery disease by blocking the antithrombotic effects of
certain prostaglandins.
ZES (Gastrinoma)
GastrinomaPUD ( panc. head 60% & duod. 30%)
Sporadic or MEN1.
60% malignant.
40% metastatic at presentation or multiple.
C/P: Abdominal pain, Ulcers, diarrhea .
When to suspect.
Recurrent DU.
Postbulbar or multiple ulcers
Resistant to TTT.
Pt. with stigmata of MEN I.
Inv;
1. Fasting gastrin level > 1200pg/ml+ low PH. (most sens. & specific).
2. Secretin stimulation test.(secritin paradoxically incr. gastrin response).
3. Tumour localization; CT, MRI, OCtreotide scan, Endo.US, laparotomy.
TTT ; PPI, octreotide, surgery, chemotherapy.
Pernicious anemia
Autoimmune disease Ab to parietal cells
HCl, intrinsic factor gastric atrophy, achorhydria,
macrocytic anemia dt B12 def., pancytopenia.
Incidence; 1 % of the population > 50 years.
C/P; , FH, >50 years old, macrocytic anemia dt
B12 def., pancytopenia other autoimmune
manifestations; vitiligo, Addison, thyroid dis.
Complication; gastric cancer.
Chronic gastritis
Type A; autoimmune, anemia, affect gastric body.
Type B; H pylori, antrum
Menetriers disease
Hyperplasia of mucin producing cells.
C/P; protein loosing enteropathy, hypochlorhydria.
Ttt; antacids for ulcerations, high protein diet, total
gastrectomy.
DD of thick gastric folds;
- Menetriers, ZES,
- Lymphocytic, esinophilic , granulomatous gastritis.
- Malignancy, MALT lymphoma.
Protein loosing enteropathy.
Gastric polyps; (rare & mostly benign).
1. Hamartomatous ; PJS..
2. Adenomatous; pre-malignant
Gastric carcinoma
Predisposing factors;
1. H pylori.
2. Pernicious anemia.
3. Adenomatous polyps.
4. Partial gastrectomy.
5. Bl group A.
6. Smoking.
7. Alcohol, spicy.
Adenocarcinoma, antrum.
Paramalignant; acanthosis, dermatomyositis.
Gastroparesis
Causes;
1. DM.
2. Autonomic Neuropathy.
3. Vagotomy.

C/P ;delayed gasric emptying , vomiting


Inv; isotope empting studies.
TTT; prokinetics; metoclopramide,
erythromycin, low fat diet ,low fibre , small
frequent meals. feeding jejunostomy.
Vomiting
Higher centers
(see,smell)

ICT
CTZ VC
Bact toxins
5HT3 D2 M1

chemotherapy
5 HT3 H1
Metabolic Visceral aff Vestibular syst
Drugs; -acidosis (gastric, IO, ishemia) preg, motion sickness
-morphin, Hypoxia
-digitalis, Uremia
-estrogen, LCF
-ethanol ehtanol
Malabsorption
Causes;
1. Preabsorptive;
1- gastric; post-gastrectomy.
2- pancreatic;e.g. chronic pancreatitis, Cystic fibrosis.
3- bile acid e.g. CLD, cholestasis, Bacterial overgrowth, Ileal
resection,Drugs as neomycin, cholestyramine.

2) Absorptive; (Impaired mucosal absorption)


- Intestinal resection or bypass
- Inflammation, infiltration: Crohn's disease, Celiac sprue, Amyloidosis,
Lymphoma,Radiation,
- Infections; giardiasis, Tropical sprue, Whipple's disease
- Genetic , Agammaglobulinemia, abetaliproteinemia.

3) Post absorptive; (Impaired nutrient delivery from intestine)


- Lymphatic obstruction, ectasia, Lymphoma .
- Circulatory disorders; Constrictive pericarditis, Mesenteric artery
atherosclerosis, Vasculitis.
- Endocrine Adrenal insufficiency Hyperthyroidism.
Management of malabsorption
1. Lab; CBC, Ca, Fe, folic acid, B12, cholesterol, Alb.
2. To prove steatorrhea;
- qualitative; sudan III
- quantitative; stool fat>7 g/d or 6% of dietary fat.
3. endoscopic biopsy.
- non caseating granuloma crhons
- congo red +ve amyloidosis.
- Malignant cells lymphoma.
- PAS +ve org whipple, DD; M avium intracellular.
-foamy intestinal mucosa
- clubbed villi, dilated lymph int lymphangectasia.
- absent villi, no plasma cells Agammaglob
- - absent villi, mononuclear infiltrate, hypertrophy of crypts celiac.
4- Urinary D Xylose test; assess CHO absorption from proximal SI, 25 g D
xylose oral & collect urine for 5 hr, n=>4.5g D xylose in urine.

NB; causes of villous atrophy;


Celiac, tropical sprue, bact overgrowth, drugs as neomycin, laxative.
Whipple, hypogamma, lymphoma.
Other specific tests;
5- for Bacterial overgrowth;
6- for exocrine pancreatic function;
- therapeutic test with pancreatic enzymes
- secretin test (most sens)
- fecal elastase, chemotrypsin
- serum trypsinogen.
- bentiromide test.
Bacterial overgrowth
Definition; Symdrome ccc by diarrhea, steatorrhea, macrocytic anemia dt
proliferation of colonic bacteria in the small intestine e.g. E coli, bacteroids.
Pathogenesis;
- decongugation of bile acidsmicelle formation steat.
- consume B12 (not folate)
- enterotoxins diarrhea
Causes;
1- Anatomical stasis; e.g. Afferent loop stasis/ blind loop/ strictures / fistulae.
2- functional stasis; DM, Scleroderma intestinal pseudoobstruction.
Diagnosis;
- macrocytic anemia (B12, folate).
- jejunal aspirate for quantitative culture (> 105 ) .(gold standard)
- H+ breath test (oral lactulose for early H+ detection in breath)
- radioactive Carbon C14 breath test (oral bile salts for early CO2 in breath).
TTT; surgical correction
antobiotics; tetracyclin, metronidazole, ceph, amox/clav for 3 wks or 1
wk/month if recurrent.
probiotics
Coeliac Disease
Coeliac disease (gluten sensitive enteropathy) is relatively common in
Ireland, caucasiens, incidence1%.
Gluten is found in wheat, rye and barley .
C/P; asymptomatic diarrhea, steatorrhea, 2ry lactase def, anemia, bone
dis.
Complications; GI & non GI cancer, intestinal lymphoma.
Associations; Dermatitis herpetiformis, IDDM, Ig A def, infertility, IBD,
thyroiditis, fibrosing alveolitis, hyposplenism, Ig A nephropathy.
Lab;
- Folate levels are usually normal but not B12 in coeliac disease (in Crohns
both are decreased).Fe, Ca
- CBC: Macro or micro, howel jolly bodies.
- antigliadin, antiendomesial, tissue transglutaminase, antireticulin
Jejunal biopsy in coeliac disease typically shows:
hypertrophied crypts of Lieberkuhn,
villus atrophy,
an increase in intraepithelial lymphocytes.
TTT; gluten free diet , use rice, oat, potatoes, Fe Ca, Folic supplementation,
if resistant steroid.
Dermatitis herpetiformis
- manifests as a pruritic rash.
- Anti-endomysial antibodies may be present.
- TTT; Dapsone (diaminodiphenyl sulfone)
often is used initially;
sulfapyridine is substituted in patients
unable to tolerate dapsone.
Tropical sprue

Cause; infection as klebsiella, E coli .


Diagnosis; history of living in tropical areas +
biopsy ( as celiac but less villous atrophy, more
infiltration, same severity throughout intestine.
Ttt; tetracyclin for up to 6 months, folate.
Whipple disease
Chronic multisystem dis ass with;
- GIT:diarrhea, steatorrhea., wt loss,
- Arthoropathy: arthralgia, migratory large joint
- CNS; oculomusticatory myorrhysmia,
myoclonus, tremors, extra & dementia.
- Lymphadenopathy.
Cause; trophoryma whipplei (G+ve bacilli)
Biopsy; PAS +ve MQ in lamina containing the
bacilli. (DD M. avium)
Ttt; sutrim DS for 1 year, .
Short Bowel S
Causes e.g. crohns, mesenteric Vascular dis, trauma,
bariatric surgery.
C/P;
1. Diarrhea dt bile acid, transit time, bact overgrowth
2. Lactose intolerance
3. Enteric hyperoxaluria dt
- bile acid colonic permeability
- FA bind Ca colonic Oxalate absorption.
4. GB cholesterol stone dt bile acid
5. Dumping S
TTT; opiate, low fat, medium chain triglycerides
(MCTs), low lactose, vit replacement,
int. transplantation, trophic hormones as GLP-2.
Ileal dis

Mild
Bile acid diarrhoea Severe
dt bile acid induced
Cl secretion from the colon
FA steatorrhea
Ttt; cholestyramine Ttt; low fat diet, MCT
NB; Long chain FA> 12 C require micelle formation for
absorption.
Medium chain FA 8-12 C does not need micelle
formation, used in ttt of malabsorption e.g. coconut oil.
Short chain FA <8C
- formed by colonic bacteria
- e.g. acetate, propionate, butyrate.
- Nutrient to the colon,
- with antibiotic use Ab induced colitis & diarrhea.
Lactase deficiency;
Causes;
- 2ry e.g. celiac dis.
- 1ry; 15% of Caucasians, 85-100% in
blacks.
C/P; Asymptomatic, bloating, abd cramps
TTT; lactose free diet.
DD; IBS.
Acute diarrhoea
<2 wks.
90% infectious.
Toxigenic (watery); staph, B cereus , vibrio chol,.cl bot, cl
perfring, E coli,
Invasive (bloody); (YES CAN CAUSE Vibrio
parahemolyticus, vulnificus) yersinia, E coli, E coli o157, E
hist, salm, shig, campylo, cl diff, vibrio parahemol, vulnificus,
CMV colitis.
Traveler diarrhea E coli, campyl, shigella, salmonella, ttt;
cipro, sutrim.
Hamburger E coli o157:H7
Seafood vibrio, yersinia.
Fried rice B cereus.
Ttt;
- symptomatic
- metronidazole
- cipro + culture
Campylobacter; bloody, GBS ,ttt =erythro
Yersinia; Rt ileac, reactive arthritis, EN,
myocarditis.ttt; quinolone
E coli o157:H7 HUS.
Amebic desentry metro .
Antidiarrheal should be avoided in febrile
dysentery.
Chronic diarrhea
Causes;
1. Watery;
A) secretory i.e. does not with fasting, stool OG <25.
e.g. stimulant laxative, toxins, resection, hormone
producing tumors as VIPoma.
B) osmotic; with fasting, stool OG > 50 eg. Osm laxative
as Mg, PO4, lactase def, lactulose, polyethelene glycol,
sorbitol.
2. Inflammatory; pain, fever, bleeding, fecal leucocytes e.g.
IBD, microscopic colitis, radiation, ischemic, malignancy.
3. Steatorrhea; sudan III, fecal fat > 7 g/d.
4. Dysmotility as IBS, DM, thyrotoxicosis.
5. Factitious e.g. Munchausen, eating disorders.
NB
1) stool osmolar Gap
Measured Stool osmolarity= 290-300 mosm/kg water
Calculated= 2(stool Na +K)
Stool osmolar Gap= Measured- Calculated
if < 25= secretory diarrhea
25-50= mixed
> 50= osmotic
2) Prolonged fasting > 24 hrs;
if diarrhea osmotic (diet)
if not secretory.
Investigations for laxative abuse;
1. S. K
2. Melanosis coli on endoscopy. DD;
carcinoid.
3. Ba enema; loss of haustration
Inflammatory Bowel disease
UC CD
site Colon, back flush ilitis All GIT, perianal except rectum
Skip lesions -ve +ve
Macroscopic pseudopolyps transmural
Narrow short colon with Cobble stone
loss of haustrations
complications Toxic megacolon Fistulas, strictures
Microscopic Crypt abcesses Non caseating granuloma
lab PANCA70% sens. ASCA; 90% specific, 70% sens.
Ba enema loss of haustrations Cobble stone, string sign
TTT 5ASA oral or rectal Antibiotics
Steroid rectal, oral, IV Budezonide, Steroid oral, IV
Aza or 6MP Aza or 6MP, Methotrexate
IV cyclosporin or Infleximab, IV cyclosporin
infleximab
Ulcerative colitis
Crohns
affect any
portion of the
gastrointestinal
tract from the
mouth to the
perianal area.
Its transmural
inflammatory
nature
If suspecting IBD upper GI series with small bowel follow-
through (UGI/SBFT) and colonoscopy, with biopsies.
Features diagnostic of Crohn's disease include;
- evidence of small bowel involvement on UGI/SBFT, or
- non-caseating granulomas.
- Significant perianal disease.
Extraintestinal feature include;
- EN, clubbing, Pyoderma Gang.,
- uveitis, episcleritis,
- osteoporosis, ankylosing spondylitis, sacroiliitis, peripheral
arthritis,.
- gall stones, PSC, cholangiocarcinoma,
- Ca oxalate urinary stone , amyloidosis, thromboembolism.
C/p; majority has bloody diarrhea and mild cramping, with minimal systemic symptoms
such as fever, weight loss, and anorexia.
TTT
depends upon both the extent of colonic involvement and the severity of the
disease process at presentation.
1. for ulcerative proctitis = topical 5-aminosalicylate (5-ASA) suppositories or
steroid foams.
2. Patients with left-sided colitis often respond to topical enemas.
3. Pan colitis ( extends beyond the splenic flexure).

A) mild to moderate;
- oral 5-ASA or sulfasalazine + topical 5-ASA or steroid enemas .
- add oral prednisone (40 to 60 mg/day) in:
1- more severe symptoms and
2- failed oral 5-ASA and topical therapy.

B) Severe active disease


- pulse IV steroid,
- IV cyclosporine
- surgery.
These agents often require 3-6 weeks to exert their maximal benefit.
Once remission is achieved, the drug dose can be tapered to maintenance levels of
sulfasalazine (2 g/day), mesalamine (1.2 to 2.4 g/day), or olsalazine (1 g/day) .
Induction therapy of active UC
Disease severity Medication Daily dose
Mild-to-moderate disease
Mesalamine suppository 1000mg at night
Mesalamine enema 4 g at night
Hydrocortisone enema 100 mg at night
Mesalamine e.g. Pentasa 500to 1000 mg PO 4 times /d
Sulfasalazine 1-1.5 g PO four times/ d

oral 5-ASA + 5-ASA enemas/steroid enema


Prednisone 40 to 60 mg PO /d
Severe active disease
On steroids recently Methylprednisolone 48 to 60 mg IV once daily
Hydrocortisone 100 mg IV/ 6 hrs or IVI
Cyclosporine
Infliximab
Toxic megacolon IV corticosteroids
Broad-spectrum antibiotics
Chronic active disease Mercaptopurine, Azathioprine
(steroid refractory) Infliximab
Maintenance therapy in UC
Disease type Medication Dose

Proctitis Mesalamine suppository 1000 mg /night to /3 night

Distal colitis Mesalamine enema 4 g /night to / 3 night

Left-sided & pancolitis Sulfasalazine 500 mg PO four times daily


Mesalamine EC (Asacol*) 400 to 800 mg PO 3 times/d
Olsalazine 500 mg PO twice daily
Sulfasalazine/oral 5-ASA + 5-ASA enema

Steroid-dependent colitis Mercaptopurine 50 mg up to 1.5 mg/kg PO/day


Azathioprine 50 mg up to 2.5 mg/kg PO/day
Methotrexate 10 to 15 mg PO once per week
Fistulizing Crohn's;
- elemental diet is as effective as steroids in induction of
remission in Crohn's not in UC, better than bowel rest & TPN.
- antibiotics
- Aza, metho
- infliximab
- CAI
- TPN

Oral lesions ; Aphthous ulcerations are the most common ,


usually occur with coexistent intestinal disease.
TTT;
- respond to treatment directed at the intestinal disease.
- Topical therapy consisting of hydrocortisone in a carrier of
pectin gelatin and carboxymethylcellulose or topical sucralfate
may be effective for local symptom relief.
Antiinflammatory drugs in CD, UC;

1. 5ASA;
- e.g. - sulfasalazine (sulfapyridine + 5 ASA).
Side effect; oligospermia, allergic, idiosyncrasy,
agranulocytosis, rash, folate def.
- Enteric coated; mesalazine (Pentasa, Asacol)2-4g.

2. Steroids;
for induction only not maintenance, less effective
than topical 5 ASA.
Infliximab;
chimeric monoclonal antibody to TNF.
Used in;
1- refractory IBD, not fit for surgery.
2-perianal
3-RA, Psoriasis, cryo, not AS.
Dose; IV 5 mg/kg at 0, 2, 6 then /8 wks.
Side effects;
- Ab to infliximabe resistance & reaction so given with
Aza or methotrexate + hydrocortisone before the dose.
- lymphoma.
- allergic reactions, serum sickness
- infection esp TB
- demylenating disorders.
Surgery;
UC ileal pouch anal anastomosis (IPAAS).
Indication; intractable, fulminent, toxic, complicated,
extraintestinal.
Stay the same; sacroilitis, sclerosing cholangitis, uveitis.
Improve; pyoderma, peripheral arthritis, perianal dis.
Complication; pouchitis in 1/3. C/P; diarrhea, arthralgia,
fever. Biopsy differentiate bet CD & true pouchitis.
Prevention with probiotics. Ttt; antibiotics.

CD; - surgical resection is contraindicated.


- stricturoplasty, diversion colostomy for perianal dis
- total proctocolectomy & ileostomy (but IPAAS is
contraindicated dt pouch failure)
IBD & pregnancy;
- sulfasalazine infertility dt oligospermia.
- Fetal complication dt dis activity not medications.
- Dis activity not affected by pregnancy.
- But pt should be in remissionfor 6 months before
conception.
- CS is prefered in perianal dis.
- Drugs;
- 5 ASA, steroids safe but use folate with sulfas.
- Aza, 6MP safe with consent.
- infleximabe; propably safe.
Cancer risk in IBD;
Risk factors; duration, extent, FH, PSC,
stricture, pseudopolyps, bypassed seg
in CD.
Screening; after 8-10 y of pancolitis or
12-15 y of lt sided colitis colono +
biopsy/1-3 yr.
Gut Hormones
Secretin is produced by the jejunum. It
relaxes the oesophageal sphincter and also
stimulates pancreatic enzyme secretion.
Gastrin is secreted by the G cells of gastric
antrum. It stimulates parietal cells produce
hydrochloric acid.
Vasoactive intestinal peptide (VIP) promotes
intestinal water and electrolyte secretion.
Chief cells pepsin.
Parietal cells HCl, intrinsic factor.
Surface cells-> HCO3, mucous.
VIPomas
VIPomas [vasoactive intestinal peptide (VIP)] .
C/P;WDHA
- watery diarrhea, stool volume > 700 ml/d.
- hypokalemia, N-AG acidosis
- achlorhydria,
+ hyperglycemia , hypercalcemia in 2/3,
flushing and hypokalaemia due to diarrhoea.
Treatment; correcting volume and electrolyte
abnormalities by using potassium chloride and
sodium bicarbonate. Octreotide controls diarrhea
in 80% of cases. Glucocorticoids reduce symptoms
in 50% of patients with VIPoma.
Carcinoid tumours
C/P After hepatic metastasis;
Forgut cushing, acromegaly.
Midgut (ABCDEF); asthma, belly abd, cor pulm
TR>TS, diarrhea, endocardial fibrosis, flushing
(Carcinoid syndrome)
Bronchus; ACTH, GHRH.
Diagnosis; raised urinary 5-HT levels. A precursor of
5HT, tryptophan is highly metabolised and
consequently niacin deficiency (pellagra) occurs. The
three D's dementia, dermatitis and diarrhoea occur.
Ttt; resection, octreotides, methysergide,
cyproheptadine, phenoxybenzamine for flushing.
Glucagonoma; WARDS
- wt loss
- anemia
- rash. NME
- DM, diarrhea
- thromboembolism

Somatostatinoma SSS
- sugar (DM)
- steatorrhea
- stones GB
Constipation
Types of Constipation
and Causes
Recent onset
DD;
Colonic obstruction Neoplasm; stricture: ischemic, diverticular, inflammatory
Anal sphincter spasm Anal fissure, painful hemorrhoids
Medications Fe, Ca, Al HO, anticholinergics, opiates,diuretics
Chronic
Irritable bowel Constipation-predominant, alternating
Medications blockers, antidepressants +2Ca
Disorders of rectal Slow-transit constipation, megacolon, Pelvic floor
evacuation (dysfunct,; rectal prolapse; rectocele.
Endocrinopathies Hypothyroidism, hypercalcemia, pregnancy
Psychiatric disorders Depression, eating disorders, drugs
Neurologic disease Parkinsonism, multiple sclerosis, spinal cord injury
muscle disease Progressive systemic sclerosis
Abdominal pain
Medical causes;
- alcohol, DKA, hyperlipidemia pancreatitis
- Hyperparathyroid, adrenal insufficiency
- FMF, porphyria, angioneurotic edema
- Lead toxicity, morphin withdrawal.
- Vasculitis as HSP.
- sickle cell an..
Ischemic colitis
Non- occlusive
Rectosegmoid
Elderly, bloody diarrhea& pain .
XR abd; thumb printing dt mucosal &
submucosal oedema.
colonoscopy; narrowing, ulceration.
Angio; not usually indicated.
Pseudomembranous colitis
Antibiotics as clindamycin, ampicillin,
cephalosporin, tetracyclin flurishing of Cl
difficile toxins A, B.
C/P; fever , bloody diarrhea
Diagnosis; C difficile toxin in stool.
Ttt; discontinue the antibiotic.
Give metronidazole, oral vancomycin.
Irritable bowel syndrome
C/P; pain relieved by defecation,
alternating bowel habits, mucous (ROME
II criteria).
Never bleeding /rectum,
Never wt loss,
Never nocturnal.
Ttt;
- dietary therapy; lactose free, high fibre.
symptomatic ttt.
Chronic Colonic pseudo-obstruction

C/P; of mechanical obstruction but without occlusion


of the lumen.
Causes;
1. Endocrinal causes; DM, hypothyroidism,
hypoparathyroidism, hypoadrenalism.
2. Neurologic; parkinsonism, ms dystrophy, MS.
3. Rheumatologic; progressive systemic sclerosis.
4. Infection; Chagas dis.
5. Medications; TCA, clonidine, phenothiazines, narcotics
6. Amyloidosis
7. Paramalignant.
Colonic polyps

Non-neoplastic
1. Hamartomatous
Neoplastic
(jej polyps, PJS, cowden)
1. Adenomatous (FAP, Gardner)
2. Hyperplastic.
2. Papillary adenoma
3. Inflammatory.
3. Villous papilloma
(Bilh, lymphoid, pseudopol of UC)
4. carcinoid
4. C.Tissue polyps
(lipoma, fibroma, leimyoma)
Familial adenomatous polyposis;
- AD, chr 5
- > 100 polyps
- 100% premalignant.
- extracolonic tumours; thyroid, pancreatic, duodenal.
- 1/3 has no FH.
- polyps may occur in UGIT.
- total colectomy should be performed as soon as possible
following diagnosis.
Gardner S= FAP when extracolonic tumours predominates as
osteomas, fibromas, neurofibromas, lipomas, dermoid cysts,
desmoid tumours, retroperitoneal fibrosis.
cowden (1/3 has goitre,10%cancer, fibrocystic dis of
breast)
Peutz-Jeghers syndrome;
- AD.
- dt gene mutation of the STK11 in most cases.
- hamartomatous polyps + mucocutaneous melanocytic
macules, perioral freckles, alopecia, nail dystrophy.
HNPCC
AD, chr 2,3
Associations; other cancers; endometrial, ovarian, gastric
Amesterdam criteria for genetic testing;
1. 3 or more 1st degree family members with colorectal ca.
2. Colon ca over 2 generations.
3. At least 1 before<50yrs.
4. Pt with CRC or endometrial ca below<40yrs
5. Pt with CRC with 1st degree CRC or extracolonic ca
If genetic testing +ve colonoscopy
Colorectal cancer
Predisposing factors;
1. High fat diet, low fibre
2. FAPC
3. HNPCC
4. UC, CD
5. Adenomatous polyps
6. Black race, male sex
Preventive measures;
1. NSAIDs
2. High Ca diet
3. HRT (E)
C/P; include iron def anemia, sepsis with streptococcus bovis.
Inv; double contrast Ba enema, colonoscopy, CEA.
Ttt; resection anastomosis, abdominoperineal resection &
colostomy for distal rectal tumours.
Cancer colon
Screening of colorectal cancer
People > 50 yrs
- colonoscopy/10 yr or
- occult B in stool/ y + sigmoidoscopy /5 yr.

PH of adenoma, acromegaly
- after 1 year then /5-10 years according to the size.
PH of Ca colon,
- colonoscopy after 1 year then after 3years then /5 yr.
FH of two 1st degree relatives with colorectal cancer or adenoma or
FH of one 1st degree relative with colorectal cancer or adenoma < 60 yr.
colonoscopy / 3-5 yr beginning 10 yrs younger than the youngest
affected relative.

PH of UC (8 yr pancolitis )or( > 15 yr Lt sided)


- colonoscopy + biopsy /1-3 yr
FH of FAP
- Colonoscopy / 1-2 yr from puberty till 40 yr old then as average.
- if polyps develop or genetic test +ve colectomy.
FH of HNPCC
- colonoscopy /2 yrs from 21 yr till 40 yrs then annually thereafter.
Hematology
Anemia
Hemoglobin= 13-16 gm/dl
Causes of anemia according to MCV
Microcytic anemia: (MCV <78 fL)
Iron deficiency anemia.
Thalassemic disorders.
Anemia of chronic disease(late;uncommon 30%
Sideroblastic anemia
Copper deficiency, Lead poisoning, zinc
poisoning, Al toxicity (rare)
Punctate basophilia = chr lead poisoning.
Normocytic anemia:
(MCV 78 to 100 fL)

Acute bl loss.
Hemolytic anemia. ( except thalassemia)
Aplastic A.
Anemia of chronic disease.
1- acute & chronic inflammation & malignancy.
2- chronic kidney disease.
3- - endocrine disease (pituitarism, thyroidism,
adrenalism)
Bone marrow infiltration. ( mylophthiathic An.)
macrocytic anemia:
(MCV >100 fL)
Megaloblastic A;
- Folic acid & B12 deficiency .
- others; drugs DNA synthesis ( Arac, 6MP,
hydroxyurea, Zidovudine, Methotrexate,
Azathioprine, Capecitabine, Imatinib).
Non- Megaloblastic.
- CLD, Myxedema , Alcohol, MDS,
Myeloproliferative, Reticulocytosis, some
aplastic, sideroblastic.
Shapes of RBCs

Target cell (central mass & peripheral


ring of Hb) in thala, liver dis, post-splenectomy.
Acanthocyte post-splenectomy, liver dis.
Schistocytes mechanical hemolysis,TMA.
Tear drop cell myelofibrosis.
Poikilocytosis=irregular shape.
Anisocytosis=diff in size.
Reticulocytosis
Hemolysis.
Hge .
Fe def. anemia. after starting ttt.
B12 def. anemia. after starting ttt.
leukoerythroblastic blood picture: presence of
immature myeloid & erythroid cells in blood
smear: causes
1.bone marrow infiltration with malignant cells
(carcinoma, lymphoma, myeloma, leukemia) or
fibrous tissue (myelofibrosis).
2.Severe hemolysis or severe hemorrhage .
Dimormphic Bl.Picture
Presence of both normocytic cells &
microcytic cells in blood film at the same time.
Causes:
combined def of fe ,B12 & folic a. (Coeliac disease).
Iron def. An. treated by hematinics.
B12 def. An. treated by hematinics.
Iron def. An. treated by blood transfusion.
Sideroblastic An.
Fe def anemia
50% of anemia.
Causes;
demand; infant, preg, epo therapy.
loss; bl loss, donation.
intake; diet, malabs, inflammation
Investigations;
1. (Ferritin)
2. serum Fe, TIBC.
3. TSAT
4. Anaemia early (normo), later (Micro)
4. BM stainable Fe., BM sideroblasts, n=20-40%,
in Fe def <10
5. Serum transferrin receptor prot (TRP) which is
released after interaction with transferrin.
Stages of Fe deficiency
Fe indices Normal Fe
deficiency
anemia

MCV NN Micro/hypo

S. Fe (g/dl) 50-150 <30

TIBC (g/dl) 300-360 >400

TSAT 30-50% <20%

S Ferritin 30-200 <15


(ng/ml)

Fe deficiency anemiaearlyNN, Hb=10-13g/dl,


laterMH, Hb<10g/dl.
In CKD, laboratory criteria of Iron deficiency are markedly
different from those in patients with relatively normal renal
function.

Lab Fe deficiency in Fe deficiency in CKD


Normal
population
TSAT <20% <20%
(n=20-50%)
Ferritin <15 ng/ml <100 ng/mL predialysis
< 200 ng/ml in HD
This discrepancy reflect an underlying inflammatory state
associated with advanced renal failure and dialysis.
TTT;
Bl transfusion if;, CVS instability.
1unit packed RBCs Hb 1 g/dl= 250 mg Fe.
oral (Grade 1B), 200 mg elemental iron/d before meals
Or IV total dose regimen= Fe sucrose(ferrosac) or
dextran(cosmofer) BW x 2.3 x (15-pt Hb) .
Retics in 4-7 days, peak 1-2 wks.
Side effects;.
1. Anaphylaxis (Fe dextran due to anti-dextran Ab).
Contraindications;
1. Active infection.
2. Fe overload.
Refractory Iron deficiency anemia
Wrong diagnosis.
DD; 1- An of chr illnessTIBC.
2- thalath serum Fe, Hb electroph.
3- sideroblastic normal Fe, BM exam.
Malabsorption.
Chronic bl. Loss
Chronic infection or inflammation
Non compliance

NB.
Hypochromia(decreased HB) more marked in iron def. An.
Microcytosis(decreased MCV) more marked in thalassemia.
An of chronic illness
Causes;
- chronic infection as TB
- Chronic inflammation as RA, vasculitis.
- Malignancy.
mechanism; IL-1, TNF-, INF-.
leading to; defective iron mobilization from its stores,
defective iron utilization by bone marrow.(IMP)
hepcidin release from the liverFe absorption & release from
stores.
Epo production
Sideroblastic anemia
The MCV may be (non megaloblastic an),
or n
congenital pyridoxine deficiency.
Acquired: lead, alcohol, drugs eg INH.
Pappenheimer iron-protein complex
(sideroblastic granules) in Fe overload,
hyposplenism.
Fe indices Fe deficiency thalassemia An. Of Sideroblastic
anemia chronic An.
disease

S.Fe (g/dl) decreas increase decrease increase


TIBC (g/dl) increase decrease decrease Decrease

TSAT decrease increase decrease increased

S Ferritin decrease increase increased increased

MCV decrease decrease normonor variable


mo
Megaloblastic Anemia
Causes of B12 def; (the most IMP)
Gastric; gastrectomy, pernicious A.
Intestinal; bact overgrowth (bacteria consump B 12 to produce
folic acid) ileal dis; resection, TB, chrons, transcobolamin II def.

Causes of Folate def;


intake;
- diet; elderly, alcohol.
- malabsorption; tropical sprue, celiac.
2. loss;
A) excessive utilization, loss;
- physiologic preg, lact, premature.
- hemat/malig CHA, leuk, MM, carc.
- inflam Malaria, crhons, psoriasis, dermatitis.
- HD, peritoneal dialysis.
B) antifolate drugs; Alcohol, phenytoin, DHF reductase inhibitors
(methotrexate, trimethoprine, pyrimethamine), sulphasalazine.
DHF reductase Methionine

DHF Methyl THF


THF
(FOLIC ACID) (Folinic acid)

Cob (B12) Methyl cob


Homocysteine

MM co A

-Give Folinic acid in case of using DHF reductase inhibitors as (methotrexate,


trimethoprine, pyrimethamine),
-Cob (B12) def lead to extracellular folate but intracellular folate.
-In folate or B12 def Homocysteine & MM co A.
C/P;
Macrocytic anemia.
GIT; glossitis, diarrhea, wt loss.
Neural tube defect(in infants), SCD.
Hyperhomocystinemia.
In pernicious an other autoimmune dis (
vitiligo, MG, DM, addisson).
Investigations;

Megaloblastic Anemia;
CBC;
RBCs MCV > 100, anisopoikilocytosis.
wbcs n, Hypersegmented neutrophils (> 5)(it is the earliest)
plat n but > 40,000.
BM; RBCs large erythroblast, large nuclei, normal
hemoglobinization.
S. folic acid & B12.
RBCs folate, high if Bl transfusion, retics.
MMA , Hyperhomocysteinemia. unconj bil, LDH.(due to
intramedullary heamolysis)
Pernicious Anemia; s.gastrin, Antiparietal cell ab, Anti intrinsic
factor ab, Check TFTs, High risk of gastric malignancy.
TTT;
Cob 1000 Ug IM x 6 times in 3 wks then /3
month for life.
+ folate 5-15 mg for 4 months or for life or
folinic acid.
+ kCl & aspirin during ttt response for
expected hypokalemia & thrombocytosis.
Haemolytic anemia
Haemolytic anemia

Intracorpuscular extracorpuscular

congenital acquired immune Non immune

Membrane defect; sphero Alloimmune


Enzyme defect; G6PD, pyruvate K. Drug; Penicillin
Hb def; sickle, thalath. Methyldopa ,INH Mechanical; valve,
Autoimmune march, TMA, burn
PNH cold Ab; Chemical; lead,
Mycoplasma, IMN,NHL benzene,drugs
Cold aglut dis, PCH. Infections; clost,
warm Ab; 3L & MDopa malaria
SLE, HL, CLL
Haemolysis can be intravascular or extravascular.
Intravascular haemolysis: G6PD, PNH, cold
Autoimmune, Non immune extracorpuscular .
- bilirubin may be normal or mildly elevated.
- hemosiderinuria, hemoglobinuria.
- Fe loss.
Extravascular haemolysis: in the reticuloendothelial
system, destroyed by macrophages; warm
Autoimmune & congenital except G6PD,.
Laboratory findings
Evidence for Haemolysis
Increased red cell breakdown:
Elevated serum bilirubin (unconjugated)
Excess urinary urobilinogin
Reduced plasma haptoglobin
Raised serum lactic dehydrogenase (LDH)
Increased red cell production:
- Reticulocytosis
- Erythroid hyperplasia of the bone marrow
- CBC; MCV due to retics & folic acid def. nucleated cells.
Specific Investigations
Positive Coombs test
Hemoglobinopathies
Hb A Hb 2, 2 (appear at 38 wks gestation)
Hb A2 Hb 2, 2
Hb F Hb 2, 2 (appear at 10 wks gestation)
Inheritance in hemolytic anemia;
SpheroAD
Pyruvate KinaseAR
Thal, sickle AR
G6PDx linked.
Ineffective erythropoisis;
thal,
myelofibrosis,
megaloblastic,
MDS.
Thalathemia syndromes
thalath
Type Gene Type of Hb Hb MCV
def HbA H Barts
Silent 1 98- - - 15 90
100%
Trait 2 85-95% rare - 12-13 70-80
(microcyt
without
anemia)
Hb H 3 70-95% 5-30% 6-10 60-70
(severe
an)
Hydrops 4 - 5-10% 90-95% fatal
fetalis

Hb H is diagnosed by staining RBCs with brilliant Cresel Blue


. thalath is diagnosed by globin chain synthesis assay not by Hb electro
thalath (10-15% of med & asia, 0.8% of africans)
Type Genetic abn C/P TTT

thalath major Homozygous. -During -Need Bl transf


childhood for survival
-Hemosiderosis -Folic acid
due to Bl transf -splenectomy
-pneumovax
vaccine
thalath Homozygous. -As major - May survive
intermedia + persistant fetal Hb or -Hemosiderosis. without Bl
thal transfusion

thalath minor Heterozygous. Micro/hypochr< -


75 Fl,
Hct<30%,Hb
A2 or F
(3.57.5%)
Co-inheritance of & thal trait
precipitated globin chain severity of
anemia.
-TTT; repeated trans. Of packed RBCS
1. Desferoxamine before 5-8 yrs in thal
major; side effects; deafness, cataract,
yersinia, mucormycosis.
2. BM transplantation
3. Gene therapy.
Sickle cell dis
Abn Hb due to subst of glutamine by valine at
position 6 from N terminal of beta chain.
Types;
1. Hb SS; homozygous (Sickle cell dis), sickle at
PO2 5-6 KPa.
2. Hb AS; hetezygous (Sickle cell trait) sickle at
PO2 2.5-4 KPa.
3. Hb SC; sickle at PO2 4 KPathrombosis,
avascular necrosis, retinopathy & fetal loss.
4. Hb S thal.
C/P;
Thrombotic stoke.
Acute chest S.
Splenic infarction ( hyposplenism) , mesenteric
vascular occlusions,Protected from malaria.
Kid; RTA, DI , Interst. Nephritis, FSGS, papillary
necrosis, renal infarction, RCC.
Priapism.
Fetal loss.
Avascular necrosis, sickle dactilitis, osteomyelytis
(salm, staph,Klebs). Gout, pseudogout, osteoarthritis.
Leg ulcers.
Proliferative retinopathy.
Diag; Hb electropheresis, sickling test (Na
bisulfite test) turbidity in Hb s only.
Diag of AS (trait) by Avascular necrosis &
isothenuria.
ttt; hydroxyurea, transfusion in aplastic or
sequestration crises,
exchange transfusion in sickle crises till
Hb S<30%.in
Hb SC patients are more prone to proliferative
retinopathy and chest crises, but otherwise
run a more benign course than SS and have
correspondingly better survival.
Hb E; 22 (26 glulys) is the most common Hb
in south east asia & 2nd most common variant
worldwide;
- Hb E hetero microcytosis, no anemia.
- Hb E homo mild microcytic anemia.
Spherocytosis
AD
May present in adultJaundice, Spl, Gall
stones.
Diagnosed by bl film & osmotic fragility.
Ttt; splenectomy, folic acid, Bl transfusion.
G6PD;
- Intravascular hemolysis upon exposure to; fava beans,
infections, drugs as;
1-antimalarials; primaqine, Dapson, chlorproguanil.
2-sulfa; s.methoxazole, dapson.
3- ab; nalidixic acid, nitrofurantoin,quinolones.
4- analgesic; phenazopyridine.
Others; naphtalene, methylene blue.
- Enzyme test done in between attacks as retics has higher
enzyme level.
Pyruvate kinase; 1:10.000
in newborn or delayed.
extravascular hemolysis, Spleen.
ttt=folic acid, bl transf, , BM transpl
Heinz bodiesprecipitated globin chain in G6PD.
Paroxysmal nocturnal
hemoglobinuria (PNH)
Def; Red cell memb defect that render it liable to complement mediated
lysis. Affected cell does not express glycosyl phosphatidyl inositol (GPI) &
Decay accelerating factor (DAF) which degrade complement.
Triad; intravascular hemolysis.
pancytopenia or bicytop.
thrombosis.
Incidence; 1/million.
C/P; morning hematuria, recurrent abd pain, budd chiari, aplastic an, AML.
Inv; - may be microcytic
- BM; hypercellular+/-dyserythropeitic feature
- flow cytometry CD55,59
- sucrose hemolysis test & Ham test are unreliable.
ttt; BM transpl, eculizumab (anti C5 A), if AA ATG & cyclosporine.
PCH;
- rare, children, viral infection.IGg
- diagnosed by; Donath-Landsteiner Ab test .
Cold Agglutinin dis;
- Elderly
- React only at low temp
- Ig M in high titre, regarded as waldenstrom or
low grade B cell lymphoma.
- Ttt; rituximab.
Treatment of Autoimmune hemolytic anemia.

corticosteroids (Grade 1B)


For patients not responding to corticosteroids, or for those who
require large doses of corticosteroids to maintain their
response, we suggest elective splenectomy (Grade 2B).
For patients unwilling or unable to undergo splenectomy, we
suggest the institution of immunosuppressive or
cytotoxic agents (eg, azathioprine, cyclophosphamide,
cyclosporine, rituximab) (Grade 2C).
IVIG is sometimes effective in the treatment of autoimmune
hemolytic anemia refractory to conventional therapy with
prednisone and splenectomy
Aplastic anemia
Aplastic anemia is due to injury to the pluripotent stem cell.
Causes
Congenital; fanconi An.
Acquired;
Idiopathic.
irradiation
drug induced, benzene, chemo, chloramph,
phenylbutazone, carbamazepine.
viral infection; EBV, HIV, Parvo, hepatitis.
Immune; SLE, Thymoma, GVHD.
pregnancy
PNH
C/P;
- bleeding secondary to thrombocytopenia,
- fatigue and pallor due to anemia,
- fever and bacterial infections resulting from
neutropenia.
- No Spleen, or LN.
- Short stature, caf au lait patchesfanconi.
Inv;
- pancytopenia, retics.
- BM; fatty pale hypocellular of all three cell lines.
Fate relapse, MDS, leuk, PNH.
Treatment
- withdrawal of offending agents (if any),
- supportive care, Blood and platelet transfusions
(keep Hb > 7, Plat> 10.000). Any blood products
should be irradiated and CMV-negative, and
should not be from family members.
- Hematopoietic cell transplantation (HCT)
- intensive immunosuppressive therapy
(antithymocyte globulin, cyclosporine,
prednisone and recombinant human G-CSF or
GM-CSF, sutrim )
Pure red cell aplasia
(PRCA)
Def; Hb, retics, BM= rare erythroid precursor cells.
Causes;
congenital; Diamond-Blackfan S.
Acquired;
- idiopathic.
- thymoma
- drugs; Epo, chloramphenicol, phenytoin, Azathioprine.
- viruses; Parvo in IC, HTLV1, EBV, hepatitis.
- CT dis; SLE, RA.
- pregnancy.
TTT; steroids & other immunosuppressant.
Parvo B 19;
- In children 5th dis.
- In adult polyarthralgia, repeated abortion.
- In CHA aplastic crises.
- In IC PRCA,
- diagnosed by
- BM. giant pronormoblasts.
- PCR from Bl.
- Serology (but Ig G & M are commonly absent),
- ttt by IVIG for 5 days. & symptomatic
Major causes of pancytopenia

1. With hypocellular BM
Aplastic anemia.Others, aleuk leuk, lymphoma &
fibrosis of BM.
2. With hypercellular BM
1ry; MDS, Megaloblastic, Myelophthysis,
PNH, hairy cell L.
2ry; SLE, sepsis, hypersplenism.
Myelodysplastic syndromes
Def; dysmorphic cellular BM, ineffective erythropoiesis, peripheral
cytopenias.
C/P; old , Spleen in 20%.
Cause;
cong; Down, FH.
acq; irradiation, benzene, chemo.
Invest;
Bl. Pic. RBC=MCV, dysmorphic.
wbc=granules, abn segmentation.
plat=large, granules.
BM (cells) erythroid=Megaloblastoid (large nuclei, Hb)+/-
TTT; stem cell transplantation
immunosuppressant (linalidomide)
G CSF, Epo.
Classification; only reading
RA =refractory anemia,
RARS =refractory anemia with ring sideroblast,
RCMD= refractory cytopenias with multilineage dysplasia
RAEB= refractory anemia with excess blasts.
Hypersplenism
Any cause of splenomegaly > pancytopenia ,
bicytopenia or monocytopenia
Hereditary hemolytic anemias.
Autoimmune cytopenia.
Infection. (SBE, IMN, Malaria).
Inflammation. (Felty).
Infiltation. (amyloid, Gaucher, glycogen storage).
Congestion. (Budd Chiari, CHF, LC, PVT).
Hyposplenism

This peripheral blood smear shows;


Howell-Jolly bodies in two red cells (black arrows),
Howel Jolly bodies remnant of nuclear mat removed by the spleen.
aim cells are also seen (blue arrows), another consequence of splenectomy.
Hyposplenism
Diseases cause these appearances
A : Amyloidosis, asplenia
P: post splenectomy, post irradiation.
C: Sickle , collagen, celiac, crhons, UC.
D : dermatitis herpetiform.
E : Essential thrombocytosis.
Subjects with functional asplenia may show any of the
following:

Mild degrees of thrombocytosis and leukocytosis.


target cells, acanthocytes.
Howell-Jolly bodies in circulating red cells
Heinz bodies (precipitated globin chain,also in G6PD).
People with an absent or dysfunctional spleen
are prone to infections from encapsulated
organisms such as Streptococcus pneumonie,
Haemophilus influenzae and Neisseria
meningitides.
They are also prone to Capnocytophaga
canimorsus, a gram-negative bacillus which is
present in the mouth of dogs. rapidly fatal.
Protozoal illness like Falciparum malaria and
babesiosis.
The risk can be reduced by vaccination
strategy, prophylactic antibiotics and health
education.
blood lymphocytes
1.5-3.5.000
20-40% of WBCs

T-cells B-cells natural killer (NK)


60 to 80 % 10 to 20 % 5 to 10 %.

Suppressor/
Helper T-cells
cytotoxic T-cells
60 to 70 %
30 to 40 %
(CD4+ cells)
(CD8+ cells).
Causes of lymphocytosis
Infections
Viral Infections (EBV) , (CMV) , Mononucleosis syndrome
(adenovirus type 12, herpes virus-6),Mumps, varicella,
influenza, hepatitis, rubella, Infectious lymphocytosis
(Coxsackie virus B2, enteroviruses including poliovirus)
Bacterial Infections; Pertussis, tuberculosis, brucellosis.
Protozoal; Toxoplasmosis & Babesiosis.

Malignancy;
CLL,ALL, NHL, lymphocyte counts >20,000/L...referal.

Stress-induced lymphocytosis;Post trauma ,Post splenectomy


Autoimmune;Rheumatoid arthritis
Endocrine; Hyperthyroidism, addison.
Lymphocytosis (>4000/cmm)
Reactive lymphocytosis medical
conditions associated with lymphocytosis in
which lymphocyte count normalizes within <2
months after resolution of this condition.
Examples are viral infection and pertussis.

Malignant lymphocytosis refers to an


acute or chronic lymphoproliferative
disorder as CLL, ALL.
Atypical lymphocytosis; IMN, CMV, Hepatitis,
NHL, leuk.
Neutropenia with relative lymphocytosis; IMN,
CMV, Hepatitis, Typhoid, TB, brucellosis,
agranulocytosis.
Lymphopenia
1.HIV.
2.Legionaire disease.
3.Steroid therapy , cytotoxics.
4.Hodgkin L.
5. Congenital immune def.
Monocytosis(>1000/cmm)
n= 200-800
Infections:
Viral Infections (EBV) , (CMV), Mononucleosis
syndrome (adenovirus type 12, herpes virus6)
Bacterial Infections; Typhoid, tuberculosis, brucellosis,
IEC
Protozoal Infections Toxoplasmosis, malaria,
leishmania, trypanosomiasis.

Chronic Inflammation : RA, UC, sarcoid.

Heamatological malignancies: Acute monocytic


leukemia, chronic myelogenous leukemia, Hodgkin
lymphoma , MDS.
Neutropenia (<1500/cmm)
n= 2000- 7.500
40-80% of WBCs

severe < 500/microL.(agranulocytosis).


The risk of infection begins to increase at an
ANC below 1000/microL.
Causes of neutropenias( re. pancytopenia)
Postinfectious neutropenia the most common
cause of acquired isolated neutropenia e.g. viral,
typhoid.
Drug-induced neutropenia & agranulocytosis;
the second most common cause of neutropenia,
Including ( clozapine, antithyroid,sulfasalazine)
Primary immune disorders as SLE, Felty.
Hypersplenism
Nutritional deficiency ; Alcoholism , Vitamin B12 or
folate deficiency
Bone marrow disorders ; aplastic anemia, the
leukemias & MDS.
familial.
Effect of neutropenia
pyogenic or enteric bacteria or certain fungi. from
the skin and from the gastrointestinal and urinary tract.
does not increase the susceptibility to viral or parasitic
infection.
Common sites of infection include the oral cavity and ,
the skin, genital areas & lung. bacteremia
patients with ANC of less than 500/microL admit &
give parenteral antibiotics.
Neutropenic fever = fever > 38 measured 3
times/24h in a neutropenic patient . TTT ; broad
spectrum b lactam Ab with antipseudomonus
(ceftazidine) +/- vanco or Tienam , antifungal if fever
persists after 7 days.
Neutrophilia
Infections : Bacterial, fungal.
Inflammation : gout, RA, UC, crhons,vasculitis,
uremia, R fever, preeclampsia.
Infarction ; PE, MI, sickle crises.
Trauma; surgery, burns.
Physiologic ; preg , exercise.
Drugs ; steroids .
BM emergency; bleeding , hemolysis .
Myeloproliferative; CML, PRV, MF, ET.
Malignancy; AML, CML, HL.
Leukomoid reaction
excessive or reactive leuckocytosis.
TLC simulating leukemia usually not > 50.000.
blast cells never > 5%.
Shift to the left = staff cells.
Leucocytic Alk phosphatase due to mature WBC.
The leukocyte alkaline phosphatase (LAP) is
high in;
1. infection,
2. inflammation, and
3. polycythemia vera (PV)
low in;
1. chronic myelogenous leukemia (CML)
2. paroxysmal nocturnal hemoglobinuria
Eosinophilia (>500/cmm)
n= 20-500
ALLERGIC DISORDERS .
commonly from atopic conditions and drug reactions.
Skin;contact, eczema, urticaria, dermatitis herp.
Resp; Allergic rhinitis & Asthma.

Drug-induced eosinophilia;
NSAIDs, Dantrolene ,gold, antimicrobials , tetracyclines,
minocycline, sulfonamides penicillins, cephalosporins ,
Nitrofurantoin. and ampicillin Allopurinol, phenytoin ,)
are the most common.
Other causes of eosinophilia;
Infection; helminthes.
Systemic dis; Adrenal insufficiency, Atheroembolic disease ,
celiac dis. Churg- Stauss S.
Hematologic & neoplastic disorders;
CML, eosinophilic leuk, HL, histiocytosis, Hypereosinophilic
syndrome, Mastocytosis.
pulmonary eosinophilia
Extrinsic;
Loafler pneumonia(Ascaris, toxocara, strongyloid.
Tropical pulmonary eosinophilia filaria,
ABPA.
2. Intrinsic;.
Chronic eosinophilic pneumonia;
Churg- Stauss S.
Idiopathic hyperesinophilic S.
Hypereosinophilic syndrome marked peripheral
eosinophilia > 1.500 for > 6 months and involvement of multiple
organs such as the heart, gastrointestinal tract, lungs, brain,
and kidneys. Some cases are myeloproliferative disorders as
eosinophilic variants of CML, or T cell lymphomas that produce
large amount of IL-5. TTT; steroids, hydroxyurea.

Chronic eosinophilic pneumonia; cough, fever, progressive


breathlessness, weight loss, wheezing, predominantly in
women and nonsmokers; following radiation therapy for breast
cancer. CXR "photographic negative" of pulmonary edema is
pathognomonic for the disease but found in <1/3of cases

Mastocytosis Systemic mastocytosis is charact. by


proliferation and accumulation of mast cells in various organs
including the skin, liver, spleen,LN
Patients present with organomegaly, hematologic or cutaneous
abnormalities.bl. eosinophilia in up to 20 %.
Myeloproliferative disorders;
1. CML
2. PRV
3. Essential thrombocytosis.
4. Myelofibrosis
Lymphoproliferative dis;
1. CLL
2. Hairy cell leuk
3. Prolymphocytic leuk
4. Lymphomas
Immunoproliferative dis = monoclonal gamopathies=
plasma cell dyscrasias;
1. Benign (MUGS).
2. Malignant; MM, plasmacytomas, waldenstrom, heavy
chain dis, cryo, amyloid, lymphomas.
Polycythaemia
Polycythaemia

A)True B) Relative
= RC mass =normal RC mass &
plasma volume

2ry to;
1ry Hypoxia; altitude, COPD, cyanotic HT dis,Hb F.
=PRV epo; hydroneph, renal cysts, RCC,
tumors as liver, uterine, cerebellum.
cushing

plat, TLC, LAP, B12,


epo, O2
epo
Normal O2
PRV
Criteria for diagnosis of PRV;
3A or 2A + 2B.
A1=RC mass > 36ml/kg plat
A2= normal ABG WBC
A3= splenomegaly NAP
B12

C/P HTN, pruritis, thrombosis, PU, gout, 10% AML.


ttt; venesection, hydroxyurea, aspirin.
CML
Middle age, male, massive spleen, tender sternum.
Visual dist, priapism, deafness.
TLC=100-500.000, Myelocytes, < 5% blast.
Basophilia, esinophilia.
B12 (due to transcobolamin 1 released from neutrophils, but
bind cob tightly, does not enhance tissue utilization).
NAP.
90% philadelphia chr t(9:22) +ve (good prognosis), bcr-abl
gene in 5% of ph-ve cases.
Ttt; Imatinib inhibit the abn tyrosine kinase produced by bcr-
abl gene, induce phil ve haematopoisis, BMT.
Hydroxyurea & Myleran (busulfan)count but does not
modify the cytogenetics.
FU by PCR for bcr-abl gene.
Thrombocytosis

normal platelet count =150,000 to


450,000/microL.
Thrombocytosis defined as a platelet
count >500,000/microL.
reactive or a marker for the presence of a
hematologic disorder.
1ry essential THROMBOCYTOSIS
- Plat count > 0.6 million
- C/P; hge (plat dysfunction), thrombosis,
hyposplenism (due to splenic infarcts)
- TTT; if asymtomatic no ttt.
young,high riskanagrilide
thrombosisAspirin.
life threatening plasmapharesis, N2 mustard
single dose.
REACTIVE THROMBOCYTOSIS
much more frequent than essential thrombocytosis
The causes of RT include:
Chronic Infection; osteomyelitis, TB
Malignancy; HCC, myeloproliferative
inflammatory conditions; RA, vasculitides , IBD.
Hge, hemolysis, surgery, Acute pancreatitis .
Iron deficiency .
Drugs; Vincristine, Epinephrine, steroids
Postsplenectomy state.
BM fibrosis

1ry 2ry
= Myelofibrosis = Myelophsisic
1. Tumors; br, lung, prost
2. Infections;TB, fungal,HIV
3. Sarcoid, Gaucher, osteopetrosis
4. Hemat; CML, MM, lymph, hairy cell

Path; BM fibrosis leukoerythroblastic blood picture-pancytopenia,


dry tap of BM.
ineffective erythropoesis.
extramedullary erythropoesis

CBC; RBCsnucleated, tear drop cells


wbcs myelocytes, promyelocytes & myeloblasts.
plat giant.
Acute leukemia
Wbcs=20-100.00, > 30% blasts.

ALL;
- children
- CNS involv common. Methotr & CNS irrad
- immunophenotyping T & B
- CytochemestryPAS +ve.

- >60% cure rate after chemo .


- poor prognostic factors; age<2, >10, TLC >100,
plat<25, L3-CNS infilt, , cytogenetic abn, Tcell,
response to ttt.
AML;
- adult.
- CNS involv uncommon.
- morphologyM0----M7.
M0 undiff , M1minimal diff , M2 diff
M3 promyelocyt, M4 Myelomonocytic, M5 mono, M6
erythroblastic, M7 megakarioblastic.
- Cytochemestry sudan Black, esterase.
- M3DIC during ttt, prevented by retinoic acid, Auer
rods, t (15;17)RARA, ATRA, FFP, plat,good prognosis
- M5= gum hyperplasia.
- M4,5monocytosis.
- ttt; AraC .
- 80% remission, 30% cure rate.
immunophenotypingCD on lymphocytes e.g.
CD10,19,
Cytogenetics (chr abnormality);
sig; - markers of the dis, if relapseBM transpl.
- prognostic,
if good conventional ttt,
badtransp.
ALL philad chr t(9;22) in of cases bad
prognosis.
CMLphilad chr t(9;22) good prognosis.
AML; t (8,21)M2 good prognosis.
t (15:17)M3 good prognosis.
CLL
Commonest cause of lymphocytosis in elderly.
95% B cells, 5% t cells (poor prognosis).
May confirm diagnosis by immunophenotyping from blood.
May be associated with;.Richter lymphoma.
- monoclonal gammopathy 5%.
- hypogammaglob infection with encapsulated org.
- AIHA 10%, AI thrombocytopenia 2%.
Stages; A lymphocytosis& < 3
B; > 3 groups of LN.
C. cytopenia.9BM failure
Ttt indicated for; stage c, bulky dis > 10cm, short lymphocyte
doubling time < 6 mth, disabling B symtoms.
chlorambucil or fludarabine + sutrim for pseudomonus
prophylaxis.
Ttt of cytopenias.
Most common cause of death is infection.
Hodgkin lymphoma
Lymphoproliferative dis, prognosis depend on;
1. Bulk of the dis; >10cm LN mass=bulky.
2. Staging;(1-4)A or B according to B symptoms.
B symptoms= F, sweats, wt loss.
NB; pruritis, abd pain are not B symtoms.
-Stage 11 group of LN
-Stage 22 group of LN, same side of diaphragm.
-Stage 3LN on both side of diaphragm ( the spleen is
considered a LN, not extranodal).
-Stage 4extranodal
3. Histopathological type;
- Lymphocyte predominant best prognosis.
- Lymphocyte depleted worst.
- Nodular sclerosis most common 2/3 of cases.
- Mixed picture more extranodal;
C/P; age peak; 15-34 yrs.
Investigations;
-CBC; Neutrophilia, esinophilia, monocytosis, early
lymphocytosis but no basophilia.
- FU by LDH.
- Staging clinically & CT +/- PET scan. No need for
staging splenectomy with available imaging.
TTT; early radio.
late chemo ABVD (adria,
bleom,vinblast,dacarbazine; MOPP (mustine or
chlorambucil= less vomiting, onco, procarb,
prednizone) side effect; sterility, leukemogenic.
NHL
T cell lymphomas makes up to 10-20% of NHL & have
worst prognosis.
Divided clinically acc to maturity of cells into;
Low grade High grade

Mature cells Immature, rapidly progressive without


ttt

Older age Any age

More bone marrow involv Less bone marrow involv

Less curable. 40% cure rate

Radio for stage 1 CHOP monthly for 6 courses


Chlorambucil for diffuse dis. (cycloph, hydroxvdaunorubicin,
IFN oncovin, pred)
Gammopathies
= diseases of globulins
=immunoglobulin overproduction

Alb.= 3.3- 4.7 g/dl.


1 glob.=0.1-0.4 g/dl.
2 glob.=0.3-0.9 g/dl.
2 glob.=0.7-1.5 g/dl.
glob.=0.5-1.4 g/dl.
Monoclonal Gammopathy
= Plasma cell dyscrasias
Polyclonal Gammopathy =immunoproliferative diseases
Due to proliferation of a single
Due to proliferation of many Clone of Ig-forming cells that
B cell clones produce homogenous excess of
e.g. CLD, Chr. Inflammation, light, heavy chains or complete IG
infection. molecule. (M-protein, where the
"M" stands for monoclonal).
MM
Def; a malignant plasma cell tumour distributed at multiple sites of
BM
Most frequent hematologic malig (10%).
Most common cause of death is infection.
5 yr survival 25%, 2 yrs from diagnosis.
Cytokines released by MM cells; IGF1, IL-6, VEGF.
Classification of plasma cell neoplasms;
1. Symptomatic; monoclonal prot > 3 g/dl & plasma cell in BM >
10%, end organ damage (CRAB= Ca, renal failure, anemia,
bone osteolytic lesions).
2. Asymptomatic; monoclonal prot > 3 g/dl & plasma cell in BM >
10%, but no CRAB, no ttt, progress in 20%/yrs.
3. MUGS monoclonal prot < 3 g/dl, remain stable for long period,
other Ig are normal & plasma cell in BM < 10%, but no CRAB, no
ttt, progress in 2%/yrs.
Criteria for diagnosis of MM;(3 major or
1major & 1 minor)
Major Minor

BM plasma cell > 30% BM plasma cell 10- 30%

Monoclonal Gammopathy (Ig Monoclonal Gammopathy (Ig


G > 3.5, IgA >2, urine >1 g/24 G < 3.5, IgA <2, urine <1 g/24
h) h)

Plasmacytoma (tissue biopsy) Normal other Ig

Osteolytic lesions
C/P;
C; Ca,
renal ;
- Tubular; cast nephropathy, infiltration, Ca,
uric C, contrast, nephrotoxic Antibiotics.
- glomerular; hyperviscosity, cryo, amyloid,
LCDD.
anemia, pancytop, infection
bone osteolytic lesions, pathologic fractures,
soft tissue mass
Others; hyperviscosity, PN, AN.
lab
1. CBC; anemia, pancytop.
2. Ca, P, normal Alk P.
3. TP, Alb, uric acid.
4. Bone scan=normal, bone XR survey.
5. B2 microglobilin (prognostic factor).
6. Detection of monoclonal LC ( Lamda), most common Ig G> Ig A. Ig M.
I. Serum protein electrophoresis
May not pick up small bands or bands outside of the gamma region
II. Urine protein electrophoresis
Both serum and urine should be tested to increase detection to 95%
III. Immunofixation
A .detection of monoclonal protein after the proteins are separated by electrophoresis
B. More sensitive than electrophoresis (detection limits 150-500 mg/L)
IV. Serum free light chains (FLC) assay
A. Most sensitive (detection limit of 0.5 mg/L)
B. Assay does not detect monoclonality of the light chain but rather an abnormal
ratio of kappa versus lambda (for AL amyloidosis FLC is 91% sensitive vs 69%
with serum immunofixation and 83% for urine immunofixation)
C. Sensitivity is 99% when FLC is combined with serum and urine immunofixation
Poor prognostic factors;
1. Old age.
2. Ig A isotype
3. Chromosomal abn.
4. Hb <10, s. creat, LDH, CRP, B2 microglob.
5. Alb.
V. Treatment
A. Restore intravascular volume
B. Remove offending agents and nephrotoxic drugs
. Hypercalcemia
i. Volume repletion
ii. Bisphosphonates in refractory cases.
C. Reduce light chain levels
1. Chemotherapy
i. Thalidomide plus dexamethasone,
or
ii. Bortezomib plus dexamethasone
iii. Melphalan/pred, VAD.
2. D. Stem cell transplantation +/- kidney transplant is
an option in selected patients.
Waldestroms macroglobulinemia
a malignancy of lymphoplasmacytoid cells that secreted IgM. The light chain
isotype is kappa in 80% of the cases.
like myeloma; secret IgM, involves the bone marrow,
In contrast to myeloma, Kappa LC in 80%, LN, HSM, hyperviscosity
syndrome, no bone lesions or hypercalcemia, the size of the IgM paraprotein
results in little renal excretion. Therefore, renal disease is not common.
C/O; weakness, fatigue, and recurrent infections, epistaxis, visual
disturbances, and neurologic symptoms such as peripheral neuropathy,
dizziness, headache, and transient paresis .

O/E; LN, HSM, fundus vascular dilatation &engorge of the retinal veins

Inv; normocytic, normochromic anemia, rouleaux formation and a positive


Coombs' test.10% of macroglobulins are cryoglobulins.

TTT; serious hyperviscosity symptoms plasmapheresis, Rituximab,


bortezomib and lenalidomide
median survival is >3 years.
Bleeding tendency
1. Platelet defect;
Thrombocytopenia
Decreased platelet production ; viral infections, chemotherapy , radiation
therapy, congenital, Vitamin B12 and folic acid deficiency, Myelophthysic.
Increased platelet destruction; ITP, SLE, post-transfusion, TMA , drugs,
Hypersplenism.
Thrombathenia;
Hereditary defective adhesionBernard soulier.
defective aggreg glansman
defective adhesion VWD.
Acquired; aspirin, uremia.
Thrombocytosis (essential)
2. Coag factor defect;
cong; hemophilia A, B, VWF.
acquired; Ab mediated, CLD, Vit K def, drugs; anticoag, thrombolytic, DIC.
3. Fibrinolysis;
cong; PAI-1def.
4. Vascular;
cong; HHT, EDS.
acquired; palpable (vasculitic), non palpaple
Purpura
1. Thrombocytopenia
Decreased platelet production ; viral infections, chemotherapy ,
radiation therapy, congenital, Vitamin B12 and folic acid deficiency,
Myelophthysic.
Increased platelet destruction; ITP, SLE, post-transfusion, TMA , drugs,
Hypersplenism.
2. Thrombathenia;
Hereditary defective adhesionBernard soulier.
defective aggreg glansman
defective adhesion VWD.
Acquired; aspirin, uremia.
3. Vascular purpura;
palpable (vasculitic); HSP, cryo, Small vs vs.
non palpaple; traumatic; senile, steroid, scurvy, easy bruising.
inflammatory; sepsis, purpura fulminans, meningococ .
hematologic Malig;
drug reaction
embolic; atheroemb, fat emb.
Thrombocytopenia
Decreased platelet production
bone marrow suppression by;
viral infections (eg, rubella, mumps, varicella, parvovirus, HCV, and EBV).
Decreased , live attenuated measles vaccination.
Following chemotherapy , radiation therapy or drugs as thiazide, alcohol, E.
congenital or acquired BM aplasia, such as Fanconi anemia & (TAR) syndrome.
Vitamin B12 and folic acid deficiency.
Myelophthysic.

Increased platelet destruction


Immune mediated; ITP, SLE, post-transfusion due to anti-platelet Abs
Consumption; TMA (TTP, HUS, DIC, APA , HELLP syndrome),
Certain drugs, heparin, quinine, quinidine, and valproic acid
Hypersplenism.
Dilutional thrombocytopenia post
transfusional with packed RBC so give platelet
concentrates to patients receiving > 20 units of
packed RBC in a 24 hour period.
Pseudothrombocytopenia
inadequate anticoagulation of the blood sample .
EDTA- dependent agglutinins which can lead to
platelet clumping and spurious thrombocytopenia
inherited thrombocytopenia
Bernard-Soulier syndrome The Bernard-
Soulier syndrome is an autosomal recessive
disorder presenting with mild thrombocytopenia,
circulating "giant" platelets, marked platelet
dysfunction, and bleeding.

Wiskott-Aldrich syndrome is a rare X-linked


disorder which presents as a clinical triad of
immunodeficiency, eczema, and moderate to
severe thrombocytopenia with small (3 to 5 fL)
platelets.

TAR $
Drug induced thrombocytopenia
Heparin
Valproic acid
Gold salts
Quinine and quinidine
Trimethoprim-sulfamethoxazole &other sulfonamides
Interferons
Nonsteroidal anti-inflammatory drugs (NSAIDs)
Penicillin
Ranitidine
Measles-mumps-rubella vaccine
Glycoprotein IIb/IIIa inhibitors (eg, abciximab)
Diuretics.
Diagnosis of HIT
Diagnosis of HIT depends on:
1. History of heparin administration for > 5 days.
2. Exclusion of other causes of thrombocytopenia.
3. Demonstration of HIT antibodies.

TTT
lepirudin (Refludan) (Thrombexx) 0.05 mg/kg (3
mg in total)
Idiopathic thrombocytopenic purpura
(ITP)
There are only two criteria required for
making the diagnosis of ITP:
1. Thrombocytopenia, with an otherwise
normal complete blood count and white
blood cell differential, including a normal
peripheral blood smear
2. No clinically apparent associated medical
condition(s) or medications that may
cause thrombocytopenia
Treatment
Treatment should be restricted to those patients with
moderate or severe thrombocytopenia with bleeding
symptoms.
Patients with mild, asymptomatic thrombocytopenia,
discovered incidentally on a routine blood count,
should not be treated.
1. glucocorticoids either prednisone (1 mg/kg per day
PO) or dexamethasone
2. In patients with persistent symptomatic and severe
thrombocytopenia (ie, a platelet count
<10,000/microL) after initial treatment with
glucocorticoids, or in those whose disease relapses
after their withdrawal IVIG, or splenectomy (Grade
1B).
3. Refractory cases: Rituximab.
4. If life-threatening bleeding
Platelet transfusions,
IVIG at a dose of 1 gm/kg IV, repeated the
following day if the platelet count remains
<50,000/microL, Pulse methylprednisolone at
a dose of 1 gm IV, repeated daily for three
doses .
IV recombinant human factor VIIa.
C/P; spont, bl per orifices plat defect.
deep, trauma coag f .
Invest;
1. PT, PTT, plat count;
- if PT only factor 7 def.
- if PTT only with bleeding hemophilia, VWD.
without bleedingfact 12 def.
thrombosis lupus anticoag.
- if PT, PTT vit K def, oral anticoag.
- if PT, PTT, plat CLD, DIC.
if PTT, plat lupus anticoag
- if all normal thrombathenia.
2. Bleeding time;
- if normal factor 13 def, HHT.
- if high plat funct tests.
3. Platelet function tests.
- Restocitin test test adhesion.
- ADP aggregation test test aggreg.
- Collagen aggregation test test for plat release.
PRINCIPLES OF (Fischer,
Hemodialysis international, 2007)
COAGULATION

- -
citrate -
anti Th III
LMWH + heparin &
- indirect thrombin
Direct thrombin inhibitors
inhibitors
Hemophilia
A= factor 8 def; synthesized by the liver, spleen, kid, placenta,
carried in the circulation non covently to VWF.
B= factor 9 def.
XLR.
C/P; deep hematoma, hemoarthrosis after trauma.
Diag; APTT, factor 8 or 9.
TTT;
1. FFP
2. Coag factor conc.
3. factor 8 conc.
4. DDAVP; factor 8 & VWF.
5. Education, physiotherapy, vaccination.
VWD
AD
Female, menorrhagia.
Bleeding dt coagulation defect & platelet
defect.
Inv; PTT, bleeding time.
TTT; factor 8, cryoppt, FFP.
DIC
Def; intravascular fibrin deposition due to activation
of coag factors.
Causes;
1. Sepsis
2. Malig.
3. Obstetric; dead fetus, abruptio, septic abortion,
amniotic fluid embolism.
4. trauma; rabdomyol, fat mbolism, burn.
5. Vascular; giant hemangioma.
6. Immunology; acute hemolytic transfusion react,
GVHD, transp rejection.
7. Liver dis; acute fatty liver of preg.
8. Drugs; fibrinolytics, warfarin, amphetamines.
Chronic DIC; giant hemangioma, malignancy, dead
fetus, LC.
Invest; PT, PTT, plat, FDPs, D dimers.
DD;
1. severe CLD; fact 8 is synthesized by spleen, kid so does
not in CLD.
2. TTP/HUS normal clotting factors & no fibrinolysis.
TTT;
if no bleeding Ttt of underlying cause only.
1. Control bleeding by; (target PT, PTT < 1.5 normal,
fibrinogen > 100 mg/dl.)
- FFP ( 1 U coag F 3%, dose 5-10 ml/kg)
- cryoppt (fibrinogen, 8, VWF/ dose 10 U/2-3 FFP)
- plat 1-2 U/10Kg.
3. prot C conc.
4. Fibrinolytic inhibitors; tranexamic acid (in bleeding &
FDPs, APA, giant hemangioma).
Antithrombotic drugs

Antiplatelets Anticoagulants Fibrinolytics

1) Aspirin
2) anti ADP receptors Oral
Coumarin derivatives IV - strep., urokinase,
(clopidogril [plavix],
- Anistreplase
ticlopidine)
- alteplase
3) GPIIbIIIa antagonists
(Abceximab,
Tirofiban [Agrastat])
4) Anti Phosphodiesterase; Indirect
Dipyridamol [Aggrenox] Direct
thrombin inhibitors
thrombin inhibitors
Heparin
LMWH e.g. enoxaparine Lepirudin (Thrombexx)
Heparinoid e.g. Danaparoid Argatroban
Fondaparinux [Arixtra] Binalirudin
(Harrison's
PRINCIPLES OF INTERNAL MEDICINE, 2008)
Causes of Hypercoagulable states
Inherited
Factor V Leiden mutation 40 to 50 %
Prothrombin gene mutation
Protein S deficiency
Protein C deficiency
Antithrombin (AT) deficiency 4%
Dysfibrinogenemia
Hyperhomocysteinemia

Acquired disorders
1. coag factors; pregnancy, OCP, HRT, Tamoxifen, smoking.
2. activity of coag factors; APA, PNH, vasculitis, Malignancy, Behcet $.
3. Loss of anticoag; nephrotic S.
4. viscosity; Myeloproliferative disorders, paraproteinemia (Waldenstrom's
macroglobulinemia, Multiple myeloma), diabetic hyperosmolarity.
5. Stagnant bl; immobility, Surgery, old age > 45yrs, VV.
Thrombophilia;
= recurrent thrombosis without predisposing factors (verchow triad;)
- May be due to cong or acquired cause as APA.
- Should be tested for if;
* recurrent venous
* arterial
* young < 45 yrs.
* FH.
* unusual site; CNS, Budd chiari, PV.
Patients are either "strongly" or "weakly" thrombophilic:
Strongly thrombophilic:
First idiopathic venous thrombosis before 50 years of age OR
History of recurrent thrombotic episodes OR
First-degree relative(s) with documented thromboembolism before age 50 .
, preg, or OCP, DVT suspect Factor V leiden.
Factor V leiden.

, preg, or OCP, DVT.


1/3 of cases.
Diagnosed by activated prot C resistance
test.
- measure PT, PTT normal.
- add activated prot C normally PT, PTT.
- in Factor V leiden no PT, PTT.
Assay for Factor V by PCR.
Hyperhomocysteinemia may be;
cong; cystathionine synthase, methionine
synthase
C/P; down lens disloc, Marfanoid, livido, MR,
thrombosis.
Acquired; aging, smoking, renal failure,
myxedema, IBD, psoriasis, drugs;
methotrexate, INH, antiepileptics.
C/P; atherogenic & thrombophilic.
Diag; plasma methionine & homocyst.
TTT; vit B6, B12, folate.
Graft versus host disease
GIT : diarrhea , ANV.
Liver : elevated liver enzymes.
Skin: indurated erythema on palms & soles.
Immunology
Complement
Complement pathway
Classic pathway
Mannose + lectin
from bact cell wall C1qrs - C1 inhibitor, + immune complex

proteases
C2,4 - C4 binding protein

C3a, 5 a
chemotaxis C3 Alternate pathway

C3b
Factor B,
opsonization
(C3bBb)
+ bact. Lipopolysac
C3 nephritic factor

- Membrane inhibitor - Decay accelerating factor


of reactive lysis (MIRL) C5,6,7,8,9
(MAC) cell lysis (DAF), factor H.
Complement deficiency
Def of Complement components;
1. Congenital def. of C2,4 IC clearance SLE, vasculitis, polymyositis.
2. C3 def Life threatening infection with encapsulated org. e.g. pneumococci.
3. MAC (C5,6,7,8,9) disseminated neisserial infection.
4. Acquired consumption occur in SLE, cryo, IEC, post infectious GN, MPGN.

Def of regulatory proteins;


1. Def of C1 inhibitor hereditary angioneurotic edema dt vasoactive C2
fragment, ttt with epinephrine in the acute attack & danazole for prevention
as it hepatic synthesis of C1 inhibitor & C4, SLE dt consumption of C2, 4.
2. PNH; def of membrane GP (GPI) which bind the inhibitory comp DAF &
MIRL compl mediated cell lysis.
3. HUS; familial type dt def of Factor H compl activation & end. Injury.
4. MPGN type 2; C3 nephritic factor, def factor H.
5. ACEI C1 inhibitor def angioedema.
6. Cryo; C4, preserved C3 dt C4 binding protein inact of C3.
Immunoglobulin
Ig G;
- Monomer formed of a divalent molecule ( 2Fab).
- Most abundant. Fab
- 2ry immune response.
- The only Ig that cross the placenta.
- MW; 150.000 dalton (25 for each LC & 50 for HC).
- 4 subclasses according to Fc.

Ig A;
- Monomer in serum, dimeric in secretion.
- Def chronic diarrhea, respiratory infection, autoimmune dis.
- IgA1 contain hinge region.
- Level undetectable at birth, reach adult level at puberty.

Ig M
- pentameric.
- 1ry immune response. Fc
- Secretion do not need Th2.
- Include Bl group Ab.
- Against bacteria.
- Level undetectable at birth, reach adult level at 1 yr.

Ig D on B cell surface.

Ig E; produced by plasma cell, attach to mast cell & basophils,


responsible for type 1 hypersensitivity reaction.
Cells of the immune system
PMN;
1. Neutrophils phagocytosis, bacteria.
2. basophils & mast cells; type 1 hypersensitivity,
parasites.
3. Eosinophils; allergy, helminthes, phagocytose
IC.
Mononuclear;
1. Lymphocytes,
2. monocytes.
blood lymphocytes
1.5-3.5.000
20-40% of WBCs

T-cells B-cells natural killer (NK)


60 to 80 % 10 to 20 % 5 to 10 %.

Helper T-cells Suppressor/


60 to 70 % cytotoxic T-cells
(CD4+ cells) 30 to 40 %
Bind to (CD8+ cells).
class 2 MHC Ag
on APC as MQ
IL1,6, TNF

Active
CD4 Th Bind to
class 1 MHC Ag
IL2 On viral infected cells, cancer cells
IFN
IL12, 18 IL4

th1 IL10 th2


Chronic granulomatous disease;
- It is a defect in the production of NADPH H
peroxide phagocytic function of neutrophils
(normal phagocytosis but defective intracellular
killing) chronic granulomas & microabcesses
in skin, bone & liver.
- Diagnosed by nitroblue tetrazolium test
(screening for reduced phagocytic capacity).
- Ttt; INF
Hepatology
Functions of the Liver
Main functions include:
Metabolism of CHO, protein, fat
Storage/activation vitamins and minerals
Formation/excretion of bile
Detoxifier of drugs/alcohol
Action as (bacteria) filter and fluid chamber
Conversion of ammonia to urea
Generally LFTs reflects 2 patterns of liver
disease;
1. Cholestatic Alk P, GGT.
2. Hepatocellular AST, ALT.

Factors favouring hepatic origin of Alk P;


1. GGT.
2. 5-nucleotidase.
Type of Hepatitis
A B C D E

Route of fecal-oral percutaneous percutaneous percutaneous fecal-oral


transmission permucosal permucosal permucosal

Chronic no yes yes yes no


infection

Prevention pre/post- pre/post- blood donor pre/post- ensure safe


exposure exposure screening; exposure drinking
immunization immunization risk behavior immunization; water
modification risk behavior
modification
Hepatitis A - Clinical Features
Is an RNA enterovirus
Incubation period: Average 30 days ( 2 - 6 wks )

Complications: Fulminant hepatitis.


Cholestatic hepatitis.
Relapsing hepatitis
Prevention;
- Immune Globulin IM, 0.02 mL/kg
- Pre-exposure (travelers to endemic regions/4-6months) &
- Post-exposure (within 2-4 wks)
- Hepatitis A vaccines .
- for persons > 2 years old .
- provides long-lasting protection (20 years).
- (Havrix) use 3 doses at 0, 1, 6 months).

TTT; bed rest, symptomatic ttt e.g. antiemetics, avoid hepatotoxic


drugs.
Hepatitis B - Clinical Features
Hepa DNA virus.
Mode of transmission; Bl, sexual, perinatal.
Incubation period: Average (2 -6 ms).
Chronic infection: 30%-90%.
Association/ complications of HBV;
- Aplastic anemia.
- Arthralgia/itis.
- PAN.
- Glomerulonephritis; membranous., MP.
- HCC. - urticaria, acrodermatitis.
Acute Hepatitis B Virus Infection with Recovery
Typical Serologic Course
Symptoms
HBeAg anti-HBe

Total anti-HBc
Titre

HBsAg IgM anti-HBc anti-HBs

0 4 8 12 16 20 24 28 32 36 52 100

Weeks after Exposure


Progression to Chronic Hepatitis B Virus Infection
Typical Serologic Course
Acute Chronic
(6 months) (Years)
HBeAg anti-HBe
HBsAg
Total anti-HBc
Titre

IgM anti-HBc

0 4 8 12 16 20 24 28 32 36 52 Years

Weeks after Exposure


Diagnosis
serological tests ;
HBsAg - used as a general marker of active infection, persistance
> 6 m= chronic infection.
HBsAb - used to document recovery and/or immunity to HBV
infection.
HBc Ag is not detected in the blood.

anti-HBc IgM - marker of acute infection, may be the only


marker in the window period.
anti-HBc IgG - past or chronic infection. It can distinguish past
infection from previous vaccine.
HBeAg - indicates active replication of virus and therefore
infectiveness.
Anti-Hbe - virus no longer replicating= no longer infectious .

HBV-DNA - indicates active replication of virus, more accurate


than HBeAg
HBsAg +ve

Repeat HBs Ag after 6 m to confirm,


if HBs Ag become -ve resolved
if HB sAg still +ve

HBe Ag -ve
HBe Ag +ve
ALT =n
=chr active
=carrier

observe biopsy & TTT


HBsAg +ve
TTT of HBV

+ve HBc Ig M +ve HBc IgG


=acute =chronic
HBsAg after 6 months

-ve +ve
=resolved =chronic
1. If acute symptomatic ttt
2. If HBsAg +ve after 6 mths, HBeAg +ve or -ve, HBeAb _ve,
PCR>105,
enzymes >2folds antiviral ttt.
3. If HBsAg +ve after 6 mths, HBeAg gray zone or normal enzymes
biopsy hepatitis activity index > 4 antiviral ttt.
4. If HBeAg-ve, HBeAb +ve, PCR+ve, normal enzymes Carrier
(no ttt).
5. Liver cirrhosis nucleozide analogue only.
Treatment
Interferon
- Peg INF 2a (high mw, long life)
- Mechanism; immune distruction of virally infected cells clear the
virus.
- Dose; 180 Ug/wk
- Duration; 6 mth for eAg +ve, 12 mths for eAg _ve (mutant form)
- Indications; chr hepatitis
- NB; not for normal biopsy or LC (lead to decompensated liver)
- Response rate is 30 to 40%.
- Side effects;
common; flu like S, N, V, wt loss, loss of hair, depression, mild
BM suppression.
less common; thyroiditis, BM suppression, seizures, retinopathy,
tinitus, teratogenic.
# to IFN therapy;
Asolute;
Poorly controlled medical condition, DM
Anaemia, thrombocytopenia.
Poor medication compliance.
Alcohol, IV drug abuse, Severe depression.

Relative;
Autoimmune dis (SLE, Rh, IBD, psoriasis)
Neuropathy.
Thyroiditis.
Nucleotide analogues;
Lamivudine, Adefovir, entecavir, tenofovir
Mechanism; reverse transcriptase inhibitorinhibit viral
replication, decrease enzymes but does not clear the virus.
Adv; oral, no hep decompensation, delay the progression of
LC & HCC.
Disadv; relapse , drug resistance mutant strains, no viral
clearance.
Duration; 1-3 yrs.

NB; there are 8 genotypes for HBV (AH).better response with A


&B.
HBe Ag-ve mutant forms are more drug resistant but less HCC,
common in Egypt.
Response to ttt;
1) virologic;
- Early response (12 wks)
- End of ttt response (at 24 or 48 wk)
- Sustained response (after 6 mths)
- Non responder (at the end)
- Breakthrough (reappear while on ttt)
- Relapse (-ve at the end but +ve during FU).
2) biochemical (normal ALT).
3) Histologic (>2 Hist. A ct. Ind improvement).
Predictors of good response;
Viral; genotype, titre.
Liver; minimal fibrosis, moderate enzymes.
Patient; female, Caucasian, lean, compliant, not IC, no alcohol, no other
CLD.
Prevention
Vaccination
- highly effective recombinant vaccines.
- given to those who are at increased risk ; health
care workers, neonates.
HBIG post exposure to hepatitis B. It is
particular efficacious within 48 hours, to
neonates whose mothers are HBsAg and
HBeAg positive.
Other measures - screening of blood donors,
blood and body fluid precautions, All pregnant
women .
Natural History of HCV
Incubation period: Average 6-7 wks (2-6 ms)
Chronic HCV----------Cr-------HCC
20-30 y 5-10 y
The rule of 20;
- 20% of pts with acute infection are
symptomatic.
- 20% of pts with acute infection clear the
virus.
- 20-40% with chronic HCV develop C after
20 yrs.
- Crr---20%--HCC
- 20%-55% of Chronic who receive IFN will
obtain a sustained rresponse.
Extrahepatic manifestation of HCV
1. PCT.
2. Cryo.
3. Leukocytic vasculitis.
4. MPGN.
5. Thyroiditis.
6. Sjogrens.
7. Lichen planus.
8. Vitiligo.
9. IGT (type II DM).
10. B cell lymphoma.
Laboratory Diagnosis
HCV antibody (by EIA or RIBA; more
sensitive) appear at least 4 weeks after
infection.
HCV-RNA ( in the acute phase), (monitoring
the response to antiviral therapy).
EIA detect anti-HCV by 4- 10 wks of
exposure, while PCR as early as 1-2
wks.
Diagnostic tests for HCV

Category ALT EIA RIBA HCV RNA

False +ve N + _ _
EIA
Resolved N + + _
infection
HCV carrier N + + +

If a low risk patients with normal LFTs has a +ve EIA Only
30-40% will be RIBA +ve.
However, in high risk individuals, PPV of ELISA> 95%.
HCV genotypes
6 genotypes.
Genotype 1 most common in US, low
response rates to TTT, 30% 1 year.
Genotype 2,370% response after 6 ms.
TTT guidelines for HCV
Pt ccc Recommendation

PCR +ve, ALT, liver INF+ ribavirin


biopsy with inflammation,
bridging necrosis
PCR +ve, ALT, INF+ ribavirin or do liver
biopsy
Normal ALT Observe.
Compensated C Observe, consider TTT
decompensated C Consider transplantation
Treatment

Interferon - may be considered for patients with


chronic active hepatitis. The response rate is
around 50% but 50% of responders will relapse
upon withdrawal of treatment.
Ribavirin - combination of interferon and ribavirin
is more effective than interferon . progression
even in absence of viral response; side effect;
teratogenic, hemolysis esp in renal
insufficiency..
Hepatitis D - Clinical Features

Coinfection with HBV


severe acute disease.
low risk of chronic infection.
Superinfection
usually develop chronic HDV infection.
high risk of severe chronic liver disease.
may present as an acute hepatitis.
Hepatitis E - Clinical Features

Incubation period: As HCV


Pregnant women
Illness severity: Increased with age
Chronic sequelae: None identified
Alcoholic Liver Disease, Alcoholic
hepatitis, and Cirrhosis

Diseases resulting from excessive alcohol


ingestion characterized by fatty liver (hepatic
steatosis), hepatitis, or cirrhosis .
Prognosis depends on degree of abstinence
and degree of complications
C/P; N/V, anorexia, abd pain, wt.
Malnutrition often an issue in these patients
TTT; abstinence, good nutrition.
Lab;
GGT.
MCV (macrocytosis).
AST/ALT > 2/1.
Ig A.
Mallory bodies; dense perinuclear esinophilic
fragments in hepatocytes (DD; PBC, wilson).
thrombocytopenia.
Blood alcohol
Bleeding tendency in alcoholic Cirrhosis;
1. synthesis of coagulation factors.
2. Plat. Function defect.
3. Thrombocytopenia ( hyperslenism, BM
suppression).
NAFLD
Spectrum; steatosis, NASH, cirrhosis.
Etiology;
- 1ry; obese, DM, hyperlipidemia.
- 2ry;
drugs; amiodarone, tamoxifen, steroid, tetracyclin
rapid wt loss, malnutrition, jej bypass, TPN.
Inv; U/S, MRI, AST/ALT<1,
ANA, ASMA +ve in 40%,hyperlipidemia in 20%.
TTT;
- orlistat, wt loss
- UDCA, antioxidents, Vit E, betaine
- Statin, gemfibrozil
- Thiazolidenidione, Metformin.
Autoimmune liver disease

Autoimmune Primary Biliary Cirrhosis


Hepatitis(AIH) (PBC)

Primary Sclerosing
Cholangitis (PSC)

Autoimmune Overlap

Cholangitis (AIC) Syndromes


The diagnosis requires: exclusion of major causes of liver
damage, including alcoholic, viral, drug- and toxin-induced,
hereditary metabolic, and NAFLD.
Autoimmune Hepatitis
Young women (10-20 yrs)
Typically present with malaise, artharalgia, tender
hepatomegaly, 2 ry amenorrhea,.
Patients may be asymptomatic
AIH may present as acute hepatitis, chronic hepatitis,
or well-established cirrhosis ,
Investigation. Type 1, ASMA ,ANA ,Anti-actin, Type 2, Anti-LKMA
nti-liver-kid., Type 3, Anti-Soluble liver- antigen , globulins,
Without therapy, most patients die within 10 years of
disease onset.
With TTT,10 yrs survival = 93%.
TTT=steroids + aza for pt with or without
Cirrhosis70% remission in 2 yrs.
Relapse after remission=50% at 6 ms, 80% at 3 yrs.
Primary Biliary Cirrhosis
Middle aged females
C/P; pruritis,Jaundice, steatorrhea( 2ry to cholestasis),
hyperpigmentation, xanthelasma.
PBC frequently is associated with other autoimmune
disorders , such as Sjgren syndrome, Hashimoto
thyroiditis, AIHA, arthritis.
Criteria for the diagnosis include:
(1) A cholestatic serum enzyme pattern, Alk. Phosph.
(2) Presence of AMAs antimitochondrial.
(3) Elevated serum IgM,
(4) bile duct lesion of mid-sized IHBD
(1),(2) & a compatible histology are regarded as
mandatory for the diagnosis of PBC.
Liver biopsy lymphocytic portal infiltration.
TTT; UDCA, colchicine, transplant.
Primary Sclerosing Cholangitis
Criteria for the diagnosis :
1- RUQ pain , jaundice , pruritis & wt loss
A cholestatic enzyme pattern;
2- histologically ., large bile duct (intra & extrahepatic)
stenosis & dilatations without prior bile duct surgery or
2ry SC ; mild to moderate portal infiltration.

3- concomitant IBD ( before, during, after UC mainly) in


70 % of the patients, detected by asympt Alk P.
but in case of UC, 3% have PSC.

4- presence of pANCAs in > 70% of patients.


MRCP, ERCP beading of intra & extrahepatic bile duct.

risk of cholangiocarcinoma, retroperitoneal fibrosis.

TTT; supportive (UDCA, colchicine), , transplantation.


Autoimmune Cholangitis
AIC (AMA ve PBC) shares many features with PBC
including :
1- F>M,
2- fatigue and pruritus,
3- a cholestatic serum enzyme pattern,
4- bile duct lesions (histology), &
5- a slowly progressive course leading to
fibrosis and cirrhosis of the liver.

Patients with AIC are by definition AMA -ve


& often present with serum ANA and/or ASMA.
Criteria AIH PBC PSC AIC
F:M 4:1 9:1 1:2 9:1
Predominant ALT, AST ALT, AST ALP, -GT ALP, -GT
liver test (7-10 times) (1-3 times)
ALP ALP (6-10
(1-3 times) times), -GT
Auto ANA, ASMA, AMA p ANCA ANA, ASMA
antibodies LKM, SLA, AMA-M2

Histology interface Florid bile duct Fibrosing bile Florid bile


hepatitis lesion duct lesion duct lesion
Diagnosis Hepatocellular AMA-M2, BD st./dilat. Cholestatic
enzyme cholestatic (choigraphy), AMA-ve, ANA
pattern, enzyme cholestatic or ASMA +ve,
pattern, enzyme histology
compatible pattern, compatible
histology IBD, p ANCA with PBC
Overlap Syndromes
Patients with overlap syndromes present with
both hepatitic and cholestatic biochemical and
histological features of AIH, PBC, and/or PSC,
and usually show a progressive course toward
liver cirrhosis .
AIH-PBC overlap syndromes ...reported in
almost 10% of adults with AIH or PBC.
AIH-PBC Overlap Syndrome
AIH-PSC Overlap Syndrome
AIH-AIC Overlap Syndrome
Coexistence of PBC and PSC
Coexistence of AIH and Chronic HCV
Wilsons Disease
AR.
Genetic defect in copper transport ( ATP7B
gene on ch 13) hepatic excretion Cu organ
deposition.
Presentations :
Hepatic, chronic hepatitis, macronodular C, mallory
bodies.
Neurological;(BG) flapping, chorea, dysarthria, tremors
parkinsonism
Kayser Fleicher ring
Renal; RTA
Haematological; coombs ve HA.
Endocrinal; hypoparathyroidism
pseudogout.
Wilsons Disease
Inv;
1. Low serum ceruloplasmin
2. High urinary copper.
3. Liver biopsy with orcein stain.
TTT;
- penicillamine for life + pyridoxine, oral Zn
- Transplantation
- Family screening with ceruloplasmin.
KF ring
Hemochromatosis
AR, mutation of HFE gene on chr 6.
Homozygous C282Y or hetero C282Y/ H63D.
Exessive Fe absorption & deposition in various organs;
- heart restrictive cardiomyopathy,
- pancreas DM,
- pituitary hypogonadism,
- joints pseudogout,
- skin bronzed colour.
risk of malignancy HCC.
Cause of death; HCC, restrictive cardiomyopathy.
Lab;
1. screening by TSAT > 60% in males & 50% in females.
2. Ferritin > 500
3. Fe, TIBC.
4. Hepatic iron conc (HIC), hepatic iron index (HII), MRI.
5. Liver biopsy staining with prussian blue.
6. Genetic study.
TTT; phlebotomy/1-2 wks for 2-3 yrs to Hb<10, ferritin<350,then every 3-4-m for
life. Desferroxamine.
Irreversible complications are arthropathy, hypogonadism, cirrhosis.
1 antitrypsin
Inhibit neutrophil elastase, its
absence emphysema & LC.
AR, Pi ZZ phenotype
Inv; PAS +ve globules in periportal
hepatocytes.
Ttt; hepatic transplantation, stop
smoking.
Drug induced liver disease
acetaminophen Acute hepatitis.
Estrogen Budd chiari.
Amoxiclave, chlorpromazine, Cholestasis.
erythro, OCP,
Amiodarone,allopurinol Hepatic granuloma.
OCP, Adenomas +/- intraperitoneal
rupture.
Cholelithiasis, cholestasis
HV thrombosis
Peliosis hepatis

Methyl dopa, INH, nitrofurantoin CAH.


MeThotreX, amiodarone, vit A Cryptogenic C.
Cocaine Massive ischemic necrosis
Anabolic steroids , OCP Peliosis hepatis
Cirrhosis
Etiology Diagnosis TTT

Alcoholic C. GGT, MCV, AST/ALT Abstinence

HBV serology INF-2b


Lamivudine
HCV serology INF
ribavirin
Hemochromatosis TSAT, Ferritin, hepatic Phlebotomy,
Fe index, HFE gene desferroxamine
AIH ASMA, anti LKM, SLA, Prednisone,
ANA azathioprine
PBC AMA ursodiol

wilson Serum& urine Cu, D-penicillamine, Zn


ceruloplasmin

1 antitrypsin 1 antitrypsin level Enzyme replacement,


transplantation
Fulminant Hepatic Failure
- Def; Rapid, severe acute liver injury with
encephalopathy
within 8 weeks in someone with a previously
normal liver.
- pathology; massive necrosis, severe fatty
degeneration.
- Causes; HEV in pregnancy, Reyes, IV
tetracycline, paracetamol toxicity, ecstasy,
halogenated anathesia, wilson, viral, Alcohol,
mushroom poising, shocked liver.
- C/P; enceph, hypoglycemia, Na, bleeding tendency,
renal failure.
- Ttt; as hep. enceph.
- Poor prognostic factors;
- Paracetamol; Ph<7.3, s creat>300, PT>100, grade3,4
enceph.
Reyes Syndrome
It is acute encephalopathy + fatty
degeneration of the liver
Pathogenesis: loss of mitochondrial function
. Disturbed FAO + carnitine def
Clinical picture: Acute fulminant hepatitis.
age 4-12 yrs
URTI or chicken pox
Mortality 50%
Ttt; supportive
The Child-Turcotte-Pugh Classification
Complications of LC;
1. Portal HTN ov, hypersplenism, ascites.
2. Ascites
3. HCC
4. Hep encephlopathy
5. HRS
6. Malnutrition
7. Coagulopathy
8. Endocrinal; amenorrha, testicular
atrophy.
Portal hypertension
Def; PV pressure > 12 mmhg.
Types
According to site of obstruction
Prehepatic eg PVT

Hepatic: presinusoidal eg CHF


sinusoidal eg Cir,
postsinusoidal eg VOD

Posthepatic eg Budd Chiari synd

Clinical Picture
Hematemesis
Melena
Splenomegaly
Hypersplenism
Dilated abdominal wall veins
Ascites
Encephalopathy
upper GIT bleeding;

1. Ulcers;
- high risk ulcers= active bleeding, visible vessel, adherent clot.
- Hospitalization for 3 days.
- PPI infusion
- endoscopy electrocoag, heater probe, injection of absolute
alcohol or 1: 10.000 epin.

2. Mallory Weiss; self limiting.

3. Esoph. Varices;
- octoreotide
-ligation better than injection sclerotherapy
- quinolone Ab
- chronic non selective BB
- decompressive surgery as distal splenorenal shunt in class A & B
lower rate of intervension than TIPS.
- TIPS; less bleeding, equal mortality but more encephalopathy
than endoscopy.

4. Others; gastric or esophageal erosions, malignancy


Management
A) Of an acute attack of hematemesis
1-Resuscitation
2-anti shock measures; IV cannula, line, Ryle, IV crystalloids,
colloids, Blood, FFP.
3- hemostatic drugs; Vit K, tranexamic acid, ethamsylate.
4- PPI infusion in PU.
5- Octreotide for esophageal varices.
6- anticoma measures; lactulose, metronidazole, enemas.
7- UGIE for band ligation or injection sclerotherapy + quinolone.
8- sengestaken-blackmore tube for massive uncontrolled
bleeding from OV.
9-if still uncontrolled TIPS for high risk,, emergency surgery
for low risk.
10- if massive bleeding, source unknown (obscure)
enteroscopy or angiography.

B) IN between the attacks;


1- PPI for PU
2- non selective BB; propranolol, quinolone for prophylaxis from
SBP, UGI Band ligation or injection sclerotherapy for OV.
Lower GI bleeding
SI bleeding
Causes;
1. Hemorroids
2. Anal fissure
3. Adolescent IBD, jej. Polyp
4. Adult diverticula
5. Elderly vascular ectasia, malignancy
6. Others; dysentry, ischemia, vasculitis, intussuception.
C/P;
- If facial, oral telangectasia HHT
- Acanthosis nigricans Malignancy
- Perioral pig spots PJS.
Investigation;
- UGIE
- Colonoscopy
- Push enteroscopy
- Videocapsule enteroscopy
- TC labelled RBC scan (during bleeding)
- Angiography (tumour, Vs malformation)
Occult bl in stool;
- benzidine, guaiac tests
False +ve; meat, NSAIDs
False ve; vit C, cauliflower
(peroxidase containing)
Iron darken the test.
PVT
Aetiology: intra abdominal infection
umbilical catheterization
hypercoagulable state
invasion by tumors
idiopathic

Clinical picture: splenomegaly


no hepatomegaly
normal LFTs
Veno occlusive disease
Thickening and fibrosis of small
hepatic venules and centrilobular
veins

Aetiology: * Herbs.
* toxins (aflatoxin).
* drugs . Azathiprine
* bone marrow tx
Budd Chiari Syndrome
Obstruction of the main hepatic veins
or IVC : Acute onset of abd. Pain&
ascites , jaundice with no LL oedema.
Risk factors;
1. Chemotherap
2. Irradiation
3. alkaloids
Hepatic Encephalopathy

Def; neuropsychiatric S that may complicate


CLD, fulminant LCF & portosystemic shunts.
Pathogenesis; protein in the colon bact
flora neurotoxins e.g. NH3, mercaptans,
false neurotransmitors as octopamine &
tyramine dt aromatic & branched chain aa,
GABA.
Role of ammonia;
1. Bind glutamic acid glutamine - CNS.

2. Bind ketoglutarate CNS energy.

Ttt by binding to lactic acid.


Precipitants of Hepatic Encephalopathy

Drugs
Benzodiazepines Increased Ammonia Production,
Narcotics
Absorption or Entry Into the Brain
Alcohol
Excess Dietary Intake of Protein
Dehydration GI Bleeding
Vomiting Infection
Diarrhea Electrolyte Disturbances (ie., hypokalemia)
Hemorrhage Constipation
Diuretics Metabolic alkalosis
Large volume paracentesis

Primary Hepatocellular Carcinoma


neurologic symptoms
Cognitive impairment

Inverted sleep rhythm

Neuromuscular disturbance

Altered consciousness
Stages of Hepatic Encephalopathy

Stage Symptoms

I Apathy Slow mentation


(Mild Confusion, agitation, irritability, sleep
disturbance, decreased attention)

II Easy arousable, disorientation, inappropriate


behavior, drowsiness
lethargy
III stupor Somnolent but difficult arousal by vigorous stimuli;
pain & voice, slurred speech, confused, aggressive

IV coma Light Coma; respond to pain only


Deep coma; no response
DD of flapping tremor;
1. Hepatic Encephalopathy.
2. Wilsons dis.
3. Uremia.
4. CO2 narcosis.

DD of irritable coma;
1. Hypoglycemia
2. Subdural hematoma
3. Acute alcohol intoxication
4. Delerium tremens; dt balcohol withdrawal, visual hallucinations,
paranoid psychosis, tremors, agitation, pyrexia, tachycardia,
sweating, dilated pupil within 3-1 wks, ttt; sedation that GABA as
BDZ.
5. Wernickes encephalopathy
6. Drug intoxication
7. Meningitis
8. wilson
Treatment of Hepatic Encephalopathy

1. Control precipitating factors.


2. Lactulose
3. Antibiotics
4. Enema/4 hr
5. Protein restriction
6. BCAA supplementation
7. Flumazenil (Anexate) BDZ receptor antagonist.
8. Ldopa or bromocriptine ; dopamine agonist; improve NM
transmission.
9. Zinc
10. Artificial liver support (hemoperfusion, artificial liver device.
11. Liver transplantation
Ascites
Definition
The presence of free fluid in the peritoneal cavity.
Differential diagnosis F F F.
Causes :
1. Portal HTN; SAAG> 1.1g/dl
Prehepatic eg PVT
Hepatic:
presinusoidal eg CHF, constrictive pericarditis, TI
sinusoidal eg Cirrhosis,metastasis, fibrosis, AFL.
postsinusoidal eg VOD
Posthepatic eg Budd Chiari synd
2- Hypoalbuminemia. SAAG< 1.1g/dl
- nephrotic, protein loosing entero, malnutrition.
3- Local causes; SAAG< 1.1g/dl
Diseased peritoneum;
malignant as P carcinomatosus, mesotholioma,
infections as TB, fungal, bact,others; FMF, vasculitis as HSP,
granulomatous, esinophilic. Normal peritoneum pancreatic,
chylous, myxedema, Meigs S.
Paracentesis
Transudates Exudates

Ascitic protein <3g >3g

Ascitic LDH <200 > 200


Modified Light criteria
Ascitic/serum LDH <0.6 > 0.6
Ascitic/serum protein <0.5 > 0.5

Examples CHF/ cirrhosis/ TB


nephrosis Fungal
Portal HTN SBP
Hypoalb. Cansers
Meigs S Budd Chiari S
Contrictive
pericarditis
Serum-Ascitic Albumin gradient ( SAAG ratio);

SAAG <1.1 SAAG >1.1


=abn cap. permeability = portal HTN mediated
Nephrotic S CHF/
Pancreatitis Budd Chiari
Peritoneal TB cirrhosis
Peritoneal carcinomatosis
Serositis, CT dis.

Causes of unilateral shifting dullness;


1. TB; lobulated type.
2. Dilated intestinal loop
3. Psoas abcess
4. Subcapsular splenic hematoma
Hepatic ascites
Mechanism of hepatic ascites;
- Hypoalb
- PHT
- Renal Na retension
- Lack of distruction of Est, aldosterone, ADH
- Complicated; PVT, SBP, HCC, HRS.
TTT of ascites
- Dietary Na restriction; 2 g/d
- Oral diuretics.
- Fluid restriction is not necessary unless serum sodium is <
120 mmol/L.
- A single 4-to-6 L abdominal paracentesis should be
performed in patients with tense ascites.
- Urine sodium excretion can assist in determining patient
compliance with diet and diuretic-resistance.
- Liver transplantation should be considered in eligible
patients with cirrhosis and ascites.
Refractory ascites;
1) Diuretic resistant= can not be mobilized despite Na restriction &
max dose of oral diuretics; 400mg spironolactone + 160 mg
frusemide.
2) Intractable ascites= can not be mobilized dt diuretic induced
complications.
Ttt;
1) Large volume parasentesis+ an albumin infusion of 8-10 g/ liter of
fluid removed
2) TIPS
3) Portosystemic shunt
4) Peritoneovenous shunt
5) Ascites ultrafiltration & reinfusion
6) Liver transplantation
7) Novel ttt; ANP, dopamine, docarbamine, Ve2 receptor antagonists
e.g. tolvaptan.
Sudden in ascites in previously stable ascites;
SBP.,Budd-chuari.,HCC., HRS.
Sudden deterioration of liver functions; e.g. sepsis, Hge, alcohol binge.
Hepatic hydrothorax;
- 70% RT dt opening of lymphatic channels through the diaphragm
- Inv; diagnostic paracentesis must be done to disclose SB empyema
(in 10%).
Ascitic fluid clues of specific causes of ascites
Causes of ascites Ascitic fluid clues
Peritoneal Carcinomatosis Cytology malignant cells

Tuberculous ascites WBC > 500/mm3


with lymphocyte predominance

Chylous ascites -TG in ascites > serum


(usually > 200mg/dL)
- milky, clear by ether
- sudan staining
- dt intestinal lymph e.g.TB,
filaria, nephrotic, ectasia,
Mucinous - dt pseudomyx perit,

Pancreatic ascites Amylase in ascites > serum


(often > 1000 U/L)
TIPS
Indications
1. Resistant, recurrent refractory
ascites.
2. Recurrent variceal bleeding.
contraindications of TIPS
1. HV thrombosis.
2. SBP
3. Mild to moderate ascites.
SBP
def; An infection of the peritoneal fluid in the absence of a
known or suspected intra-abdominal surgical source of the
infection.

Risk factors;
GI hemorrhage.
previous SBP.
ascitic fluid TP < 1gm/dL,
S. bilirubin > 3.2 mg/dL
low platelet count < 98,000 cells/mm.
fulminant hepatic failure.

diagnosis;
1. C/P; fever, abd pain, tenderness, ascites
2. a PNL in ascites 250 cells/mm (or WBCs >500/mm3).
3. a positive ascitic fluid culture (usually of a single organism;
enterobacteriaceae 63%, step pneum 15%, enterococci 6-
10%) but often culture ve.
of SBP
Treatment
empiric antibiotic therapy, e.g., 2g of
cefotaxime i.v. every 8 hours,or other 3rd G
ceph or Amoxacillin-clavulonate
or quinolone if not on quinolone prophylaxis
for 5 days
1.5g albumin/kg BW within 6 hours of
detection and 1g/kg on day 3.

Prophylaxis;
Oral norfloxacin 400mg 1x2
1ry or 2ry prevention for high risk patients for
short term (7d) as 1ry prophylaxis,
long term for 2ry prophylaxis
Variants of ascitic fluid infection
according to ascitic fluid characteristics
Category Ascitic fluid analysis

Count Organism(s)
SBP PMN 250/mm Single

Culture-negative neutrocytic PMN 250/mm Negative


ascites culture
Secondary bacterial peritonitis PMN 250/mm Multiple
Jaundice
Isolated unconjugated hyperbil (direct<15%)
- Hemolysis
- Ineffective erythropoiesis
- Drugs e.g. rifampin,
- Inherited e.g. gilbert, Cr Najar.
Hepatocellular (ALT, AST)
- viral, wilson, autoimmune, alcohol, drugs e.g. INH, acetaminophen,
toxins as vinyl chloride. Isolated conjugated Rotor, DJ.
Cholestatic (ALK P > 4 times)
- extrahepatic; U/S dilated bile ducts CT, ERCP. e.g. malignancy,
stones, stricture, PSC, parasitic as ascaris, AIDS cholangiopathy.
- Intrahepatic serology, biopsy.e.g.viral, alcoholic, PBC, PSC,
GVHD, infiltrative dis as TB, lymphoma, sarcoid, amyloid, drugs as
OCP, anabolic steroids (pure cholestasis), erythromycin (cholestatic
hepatitis).
Gilberts
1 ry cause of isolated indirect bili.

Usually caused by fasting states

Nicotinic acid test delay & high peak of

biliribin ( 3 hrs).
DD
Cholestasis in ICU shock liver, sepsis, TPN.
Jaundice after BM transplantation
venoocclusive dis, GVHD.

AIDS cholangiopathy;
- Picture of PSc. Cholangitis.
- Dt infection with CMV or cryptosporidia.
- ALK =800 but bil often normal.

GGT in alcohol, phenytoin,, pancreatitis, cholestasis.


Cholestasis; Alk P fractionate, 5nucleotidase, GGT.
DD of jaundice in 3rd trimester of pregnancy
1. Viral hepatitis (hep E).
2. Herpes hepatis.
3. HELPS.
4. Acute fatty liver of pregnancy
5. Intrahepatic cholestasis.

Pruritis gravidarum;
dt intrahepatic cholestasis.
Ttt;
- mild reassurance .
- Severe cholestyramine, ursodeoxycholic
acid.
HCC
Predisposing factors;
- LC esp hemochromatosis & alcoholic C.
- HBV, HCV
- Aflatoxin (fungal metabolite in food)
- Androgenic steroids & rarely OCP.
Fibrolamellar carcinoma; a microscopic variant consisting of large
polyclonal cells arranged in trabiculae separated by parallel bundles
of collagen, in young females, of better prognosis.
Inv; AFP>400 ng/ml, carboxyprothrombin, spiral CT.
TTT;
1. Surgery; resection, transplantation if < 3 cm, good liver function
2. Percutaneous ttt; if < 5cm or 3 <3 cm with no vascular or
extrahepatic invasion. e.g. ethanol, acetate injection, cryoprecipitate,
hot saline, laser photocoagulaton, radiofrequency ablation,
intralesional injection of chemo (cisplatinum).
3. Chemoembolisation; if not fit for surgery or percut but fit liver
functions.
4. Systemic; hormonal e.g. Tamoxifen or chemo e.g. capecitabine.
5. Radiotherapy
6. Targeted immunotherapy; anti AFP Ab.
Liver transplantation
Elevated liver enzymes after
transplantation;
Early few wks to ms
1. Allograft rejection.
2. Drug toxicity.
3. Art. Thrombosis.
After 1st yr recurrence of initial
dis
HRS
Post-cholecystectomy S;
Causes;
1. Oesophagitis, pancreatitis, radiculopathy
2. Functional BD (hepatic flexure S)
3. Stone in CBD, stricture CBD, FB granuloma
4. Sphincter iof odd dysfunction
Management;
1. Liver FTs
2. Abd U/S
3. UGI endoscopy.
4. Biliary manometry
5. Biliary scan
TTT;
1. If LFTs , abd U/S normal symptomatic pain releif.
2. If LFTs abnormal, U/SCBD dilatationERCP with
manometry or sphincterotomy.
Acute PANCREATITIS
Causes;
1. Gall stones 50%; 75% of pt with unexplained
pancreatitis has microlithiasis(microscopic stone
dis).
2. Alcoholism
3. Infections; Coxsackie, mumps, ECHO & hep
viruses
4. Tumours
5. Drugs; PD FAST VET (pentamidine, didanosine,
frusemide, azathioprine, steroids, sulfa, thiazide,
valproate, OCP, tetracyclin)
6. Hypertrigyceridemia
7. Hypercalcemia
8. Iatrogenic e.g. ERCP
9. Idiopathic.
C/P;
- Abd pain, N, V.
- MOF, shock
Cullen's sign (periumbilical discolouration)
Grey Turner's sign (flank discoloration)
Inv;
1. s. amylase 4 times,levels does not correlate with the
severity, macroamylasemia (bound to Ig), urinary
amylase/creat.
2. s. lipase; specific, remain high for long time (10d)

3. s trypsin
4. Leuckocytosis, AST, ALP, Ca.

5. Imaging; CT Scan

TTT;
1. Bowel rest
2. Analgesic; pethidine
3. Antibiotics; tienem, 3rd G ceph.

4. Correct fluid & electrolytes.


5. Somatostatin IV
6. Plat activating Fact antagonist (lexipafant)
poor prognostic indicators in acute pancreatitis:
During 48 hrs Ransons BASline BUNdle Sure Can Help
Out
- Base deficit > 4 meq/l
- BUN > 5 mmol/l
- Sequestration of fluid>6L
- Calcium < 2.0 (<8mg/dl)
- Hct >10%
- PaO2 <8 (60 mmHg)
Complications of acute pancreatitis;
1. Plegmon; a mass of inflamed pancreatic tissue.
2. Pseudocysts.
3. Hypocalcemia
4. Pancreatic abcess; 2-4 wks after
5. ARDS.
6. MOF.
7. Splenic or portal vein thrombosis varices--.
Hematemesis.
8. DM, exocrine pancreatic insufficiency.

Pseudocysts;
- Fluid collections.
- In 15 % of acute panc.
- Body & tail
- Do not have epithelial lining.
- resolve spontaneously within wks
- If > 5 cm, remain for > 6 wks drainage.
Causes of a raised amylase are:
acute/chronic pancreatitis
pancreatic cysts and carcinoma

perforated duodenal ulcer

ovarian carcinoma .

ectopic pregnancy
gallstones

salivary tumour
adenitis ,mumps
diabetic ketoacidosis
Chronic Pancreatitis
Causes;
1. Alcohol
2. Cystic fibrosis
3. Hemochromatois
4. Others; traumatic, autoimmune, Hypertrigyceridemia
5. Hypercalcemia
C/P;
abd. Pain.
Steatorrhoea, B12 deficiency, trypsin is required in the processing
of dietary B12 which enables absorption .
DM
Inv;
CT calcifications.
ERCP (of choice)..... Chain of lakes
Test for exocrine pancreatic functions;
- therapeutic test with pancreatic enzymes
- secretin test (most sens)
- fecal elastase, chemotrypsin
- serum trypsinogen.
TTT: analgesics, DM, steatorrhea.
Pancreatic cancer
Risk factors;
1. Smoking, advanced age, male gender, black
race.
2. Type I DM, chronic pancreatitis.
3. Familial pancreatitis
4. Industrial exposure to petroleum components
& leather tanneries.
Pathology; adeno, 80& in the head.
C/P; cachexia, obst jaundice with palpable
GB (courvoisier law), ascites, abd pain,
thrombophlebitis ( trousseau sign)
Inv; CT, MRI., ERCP.
TTT; whipple radical surgery or palliative
surgery, chemo, radio.
A 40 year old diabetic lady presents with a hot swollen
left leg. On examination has a temperature of 39C and
her leg is tender to compression. She was treated with
intravenous flucloxacillin and benzylpenicillin. However,
the erythema has spread even further after 3 days, she
is persistently hypotensive with a systolic BP of < 90
mmHg. Which of the following antibiotics should be
added?
A. Gentamicin
B. Tazocin
C. Chloramphenicol
D. Clindamycin
E. Amoxycillin
clindamycin. There is suspicion that this
lady may have Streptococcus A infection
with toxic shock syndrome. Clindamycin
has effects of reducing protein synthesis
and exotoxin production by the bacteria.
A 32 year old sailor is admitted to hospital with a
3 day history of fever, generalised
lymphadenopathy and a macular rash over the
trunk and legs. Which of the following diseases
may present in this way?
A. Infectious mononucleosis
B. Sarcoidosis
C. MumpsD.
Primary syphillis
E. Familial mediterranean fever
A 30 year old man presented with bloody
diarrhoea. This started 2 days ago. He returned
from a business trip to Egypt recently 1 week
ago. What is the most likely causative organism?
A.Cholera
B. E coli
C. Giardiasis
D. Shigella
E. Crytosporidiosis
Shigella. The common causes of bloody
diarrhoea include Salmonella , Shigella ,
Campylobacter and amoebiasis. E. coli
type E 0157 can also cause bloody
diarrhoea with Haemolytic uraemic
syndrome.
35 year old man has lymphopenia on his white
cell differential. He complains of headaches. A
CT scan of his brain showed a 5 cm ring
enhancing lesion in the frontal lobe. Which is the
likely infective organism?
A. Cryptosporidia
B. Toxoplasma gondii
C. Aspergillus
D. Cryptococcus neoformans
E. Mycobacterium avium intracellulare
toxoplasma gondii. The lymphopenia
suggests HIV infection. Ring enhancing
lesions in the brain suggest either cerebral
toxoplasmosis or lymphoma.
A 25 year old male man has urethral discharge.
Gram stain of the discharge sample shows gram
negative intracellular diplococci. The patient is
treated with cefotaxime as a 500mg single
intramuscular dose. A week later, the patient still
has urethral discharge. Which of the following
organisms is likely to be responsible?
A. Neisseria gonorrhoeae
B. Ureaplasma
C. Chlamydia trachomatis
D. Staphyloccocus aureus
E. Escherichia coli
chlamydia trachomatis. Although the diplococci
are likely to be gonorrhoea, this patient has
been treated. Recommended treatment options
for gonococcal infection are ceftriaxone 250mg
single im dose OR cefotaxime (Claforan) 500mg
single im dose for gonorrhoea. Persistent
discharge suggests another organism, in this
case likely chlamydia. Doxycycline would is
therefore recommended.
A 18 year old man was admitted to the emergency room
due to fever, headache, vomiting, irritability, and myalgia
that had begun 24 hours ago. There was no evidence of
previous infection in the upper airways. On examination,
the patient was lethargic, disoriented and hypotensive,
with petechiae in the legs and upper limbs, and he had
conjunctival suffusion. Cerebrospinal fluid (CSF) was
turbid, with 5300 cells/mm3 (97% neutrophils and 3%
monocytes)and protein was 0.9 (<0.5).
What is the treatment of choice?
A.Erythromycin
B. Gentamicin
C. Ceftriaxone
D. Metronidazole
E. Hydrocortisone
c) ceftriaxone. The patient has
meningococcal meningitis (suggested
by the purpuric rash). Until the
organism (Neisseria meningitidis) is
isolated and sensitivities tested, the
patient should be on a cephalosporin
or benzylpenicillin.
A 20 year old man has recently returned from holiday in Spain. He
has fever and a sorethroat. Examination reveals palpable
lymphadenopathy in the cervical and inguinal areas. Blood tests
show: Hb 11.0 g/dl
WCC 12 x 10^9/l
platelets 145 x 10^9/l
urea 7 mol/l
creatinine 80 mol/l
sodium 140 mmol/l
potassium 4.2 mmol/l
bilirubin 22 mol/l
AST 90 U/l
ALP 280 U/l
albumin 32 g/l
CRP 110 mg/l
What is likely diagnosis?
A. CMV infection
B. Typhoid fever
C. Infectious mononucleosis
D. Yellow fever
E. Schistosomiasis
Answer: c) infectious mononucleosis.
Ebstein Barr virus can cause a
pharyngitis, palpable lymphadenopathy,
thrombocytopenia and deranged liver
function tests. Heterophil antibodies will be
positive and there may also be
lymphocytosis.
30 year old lady presents with headache and
neck stiffness. Her temperature is 38.5 C, BP
100/65 and she has a petechial rash in the thigh.
CSF examination reveals gram negative
diplococci. Which is the best antibiotic therapy?
A.Gentamicin
B. Flucloxacillin
C. Cefuroxime
D. Ciprofloxacin
E. Benzylpenicillin
e) Benzylpenicillin. The diagnosis is
meningococcal meningitis (it would be
pneumococcal meningitis if gram positive
diplococci were seen). Intravenous
ceftriaxone or benzylpenicillin are
treatment of choice.
An 8 year old child develops fevers, myalgia, lethargy
and joint pains over 5 days, having been in contact with
another child with a similar illness. She has a
temperature of 39 C. On examination, she has a cheek
rash, synovitis of the hand and knee joints, and palpable
lymph nodes in the cervical area. Which is a likely
diagnosis?
A. Infectious mononucleosis
B. Lyme disease
C. Listeriosis
D. Leptospirosis
E. Parvovirus B19
A 25 year old lady returned from West Africa
(she had been there for 6 months). 6 weeks after
she returned to the UK and she complained of
fever. On examination, temperature was 39.6
C. There was a generalized petechial rash in the
lower part of the body. What is the most likely
diagnosis?
A. Acute HIV infection
B. Tyhoid fever
C. Lassa virus
D. Dengue fever
E. P falciparum malaria
34 year old patient has a CD4+ count of 80/mm.
He has had a generalised seizure recently. An
MRI scan is performed, it shows multiple 1 cm
white matter lesions. Which of these diagnoses
is likely?
A. Progressive multifocal
leukoencephalopathy
B. Demyelination
C. Calcified tubers
D. Behcet's disease
E. Systemic lupus erythematosus
A 75 year man has severe headaches and
photophobia. Examination reveals a temperature
of 40C and neck stiffness. Fundoscopy and CT
scan reveal no abnormalities. CSF examination
shows Gram positive diplococci. What is the
diagnosis?
A. Listeria meningitis
B. TB meningitis
C. Meningococcal meningitis
D. Pneumococcal meningitis
E. Mycoplasma infection
A 25 year old man presented to an emergency
department with a 1-day history of fever,
headache and myalgia. Two weeks before his
presentation, he had returned from a 10-day trip
to Costa Rica, where he had injured the sole of
his foot on coral. After injuring his foot, he had
swum in freshwater rivers. Thick and thin blood
films examined at the time for malaria parasites
were negative. What is the likely diagnosis?
A. Amoebiasis
B. Leishmaniasis
C. Schistosomiasis
D. Leptospirosis
E. Brucella abortius
40 year old diabetic lady presents with a hot
swollen left leg. On examination has a
temperature of 39C and her leg is tender to
compression. She was treated with intravenous
flucloxacillin and benzylpenicillin. However, the
erythema has spread even further after 3 days,
she is persistently hypotensive with a systolic BP
of < 90 mmHg. Which of the following antibiotics
should be added? A. Gentamicin B. TazocinC.
ChloramphenicolD. ClindamycinE.
Amoxycillin
An 75 year old man presents to hospital unwell with
diarrhea. He has a BP of 100/70, heart rate 110 and
Temp 38 C. A diastolic murmur is heard in aortic area.
His bloods show: Hb 9.0 g/dl
MCV 85 fl
WCC 13 x 10^9/l
platelets 270 x 10^9/l
urea 6 mol/l
creatinine 80 mol/l
sodium 140 mmol/l
potassium 3.8 mmol/l
ESR 80 mm/hr
CRP 220 mg/l
Which organism is likely to grow in the blood cultures?
A. Streptococcus mitis
B. Staphylococcus aureus
C. Streptococcus bovis
D. Escherichia coli
E. Brucella melitensis
There was an outbreak of Methicillin Resistant Staph
Aureus in the surgical ward. What is the most
appropriate measure?
A. Antibiotic treatment of the culture positive
patients .
B. Application of chlorhexidine to the walls and
floors of the ward
C. Closure of the ward to new admissions for one
month.
D. Regular handwashing by ward staff .
E. Screening of ward staff and exclusion of those
with positive cultures.
If there is an open wound then the patient
should be isolated into a separate room.
A 35 year old woman presented with a non-
healing genital ulcer. She had travelled through
Africa 3 years previously. On examination, An
indurated 1.5 cm ulcer was present on the inner
aspect of the left labia majoris.
Skin biopsy of the lesion revealed a granuloma
surrounding a schistosome egg. Schistosoma
haematobium eggs were detected in terminal
urine collected between midday and 2 pm. What
should she be treated with?
A. Quinine
B. Benzylpenicillin
C. Tetracycline
D. Flucloxacillin
E. Praziquantel
Which of the following is the commonest
world wide cause of traveller's diarrhoea?
A. E coli
B. Giardia
C. Shigella
D. Salmonella
E. Campylobacter
A 40 year old man presented 14 days after return from a
6-week field trip to Papua New Guinea. He had a six day
history of high fevers and rigors. On the day of
presentation, he had become vague and confused. He
had taken antimalarials as prophylaxis, but ceased when
he found that local people did not take them. His
temperature was 40oC, pulse rate 140 bpm, respiratory
rate 28 per minute, and blood pressure 100/60 mmHg.
He had dry mucous membranes, mild jaundice, pallor,
splenomegaly and generalised crackles in both lungs.
Full blood examination revealed: 6.5 g/dL
WCC 2.5 x 10^9/L
Platelet 10 x 10^9/L
bilirubin 60 mol/L(320 mol/L)
lactate dehydrogenase 489 U/L (100225 U/L) creatinine
250 umol/l What is the likely diagnosis?
A. Leishmaniasis
B. Tick bite fever
C. Endocarditis
D. Falciparum malaria
E. Viral haemorrhagic fever
A 42 year old man presented 10 days after returning from a 8-week
holiday in South-East Asia. He had an eight-day history of malaise,
chills, headache, sore throat and generalised rash. He had reported
many mosquito bites. He had fever, a macular rash and generalised
lymphadenopathy with mild splenomegaly, but no meningism and no
eschar present.
Full blood examination revealed lymphocytosis with numerous
atypical lymphocytes and thrombocytopenia. Blood cultures and
malaria films were negative. Liver function tests revealed marginally
elevated serum transaminase levels. Serological testing revealed
past infection with EpsteinBarr virus and cytomegalovirus and was
negative for Q fever, dengue, rubella, measles and rickettsial
infection. What is the most likely diagnosis?
A. HIV
B. Dengue fever
C. Syphilis
D. Lyme disease
E. Malaria
A 65 year old man had been on holiday to
Arizona in the united states 6 weeks ago. He
was brought to hospital with high fever, rigors,
malaise, and mild confusion. He had a
generalised, non-pruritic maculopapular rash,
predominantly on the trunk but also on the
extremities, including the palms and soles.
There was no history of animal or arthropod
exposure, but his house was on the edge of
forest. What is the most likely diagnosis?
A. Falciparum malaria
B. Rickettsial spotted fever
C. Tuberculosis
D. Allergic bronchopulmonary aspergillosis
E. Schistosomiasis
A 30 year old lady presents with headache and
neck stiffness. Her temperature is 38.5 C, BP
100/65 and she has a petechial rash in the thigh.
CSF examination reveals gram negative
diplococci. Which is the best antibiotic therapy?
A. Gentamicin
B. Flucloxacillin
C. Cefuroxime
D. Ciprofloxacin
E. Benzylpenicillin
Answer: e) Benzylpenicillin. The diagnosis
is meningococcal meningitis (it would be
pneumococcal meningitis if gram positive
diplococci were seen). Intravenous
ceftriaxone or benzylpenicillin are
treatment of choice.
Meningococcal Rash
A 40 year old man has been to Malaysia for 6
months. He has an erythematous, serpiginous,
pruritic, cutaneous eruption on the medial side of
the ankle. What is the diagnosis?
A. Lyme disease
B. Cutaneous larval migrans
C. Leishmaniasis
D. SarcoidosisE.
E. Tuberculosis
Answer: b) cutaneous larval migrans.
Cutaneous larva migrans is caused by the
penetration through intact skin of larval animal
hookworms (e.g. Ancylostoma braziliense).
Diagnosis is predominantly clinical.
Treatment is often necessary because of intense
pruritus, long duration (over a year) and
complications, such as impetigo and allergic
reactions. Therapy comprises ivermectin,
albendazole or thiabendazole.
20 year old man has recently returned from holiday in Spain. He has
fever and a sorethroat. Examination reveals palpable
lymphadenopathy in the cervical and inguinal areas. Blood tests
show: Hb 11.0 g/dl
WCC 12 x 10^9/l
platelets 145 x 10^9/l
urea 7 mol/l
creatinine 80 mol/l
sodium 140 mmol/l
potassium 4.2 mmol/l
bilirubin 22 mol/l
AST 90 U/l
ALP 280 U/l
albumin 32 g/l
CRP 110 mg/l
What is likely diagnosis?
A. CMV infection
B. Typhoid fever
C. Infectious mononucleosis
D. Yellow fever
E. Schistosomiasis
A 22 year old man had been to a Greek island on holiday 2 months
ago. He presented with a 2-week history of dry cough, diarrhoea,
lethargy, anorexia and fever. He had a past history of cutaneous
vasculitis of uncertain aetiology. He was not taking any regular
medication. On examination, the patients temperature was 37.5C,
but no other abnormalities were evident. Investigations showed that
he had pancytopenia, raised erythrocyte sedimentation rate and
hypergammaglobulinaemia.
A second bone marrow biopsy at this time revealed occasional
macrophages containing amastigotes. Further history revealed that
he had been bitten by sandflies. What is the diagnosis?
A. Schistosomiasis
B. Cutaneous leishmaniasis
C. Visceral leishmaniasis
D. Malaria
E. Dengue fever
30 year old man presented with bloody
diarrhoea. This started 2 days ago. He returned
from a business trip to Egypt recently 1 week
ago. What is the most likely causative organism?
A. Cholera
B. E coli
C. Giardiasis
D. Shigella
E. Crytosporidiosis
patient with HIV takes several different
drugs is concerned about changing facial
appearance. Which drug is most likely to
cause lipodystrophy?
A. Lamivudine (3TC)
B. Zidovudine (AZT)
C. Didanosine (DDI)
D. Nevirapine
E. Saquinavir
A 30 year old man had a tender painful leg
with fevers and a year later, progressively
develops a swelling in the left leg. Which
organism is likely to be responsible?
A. Loa loa
B. Wuchereria bancrofti
C. Schistosomiasis
D. Leishmaniasis
E. Toxoplasmosis
Wuchereria bancrofti. There is filarial
involvement of the lymphatics causing
lymphedema or 'elephantiasis'. The organisms
are commonly wuchereria bancrofti or brugia
malayi. In acute infection, organisms can be
detected on a blood film, and filarial serology
can be sent. Treatment is with
diethylcarbamazepine (DEC) or ivermectin.
Which of the following is the commonest
world wide cause of traveller's diarrhoea?
A. E coli
B. Giardia
C. Shigella
D. Salmonella
E. Campylobacter
E coli. Traveller's diarrhoea is an
extremely common occurrence, affecting
up to half of travellers to high risk areas
such as Africa, Asia and South America.
The commonest infective cause world-
wide is Escherichia coli. Other bacterial
causes include Shigella, Salmonella and
Campylobacter, all of which can cause
dysentery (diarrhoea with blood).
A 23 year old student has just returned from India having
been on a holiday. He was bitten by flies whilst he was
there. He has been lethargic for two months and has a
fever. Clinical examination reveals hepatosplenomegaly.
Ultrasound of the abdomen reveals lymphadenopathy.
One of the lymph nodes are biopsied (a smear shows
amastigotes within a macrophage). What is the
diagnosis?
A. Kala azar
B. Babesiosis
C. Schistosomiasis
D. Malaria
E. Amoebiasis
Infectious Diseases
Viral Infections
Lab.diagnosis of Viral Infections
Immunologic Studies
- Antibody titer ; a > 4 fold rise is considered evidence
of disease .
- Antigenic detection is used for certain viruse( HBsAg,
CMV, HIV)
Molecular Techniques;

- PCR (eg, HCV and herpesvirus)

Microscopic Methods

- search of specific cytopathic changes (eg,


multinucleated giant cells at the base of herpes
lesions, .
DNA Viruses
AHP
Adeno virus.

1.Acute pharyngitis.
2. Acute follicular conjuctivitis.
3.Acute GE.
Herpes viruses

Latent infections, giant cells by Tzanck smear.


HSV-I.
HSV-II.
Varicella (Chickenpox) & Herpes Zoster V.
1. EpsteinBarr Virus (Infectious Mononucleosis).
2. Cytomegalovirus.
3. Human Herpes viruses 6, 7, & 8 .
HSV-I.
Dormant in trigeminal ganglion

oral cavity; stomatitis, herpes labialis


(cold sores).
Esophagitis; in immunocompromised
Eye; Keratoconjunctivitis.
CNS; Encephalitis and Meningitis.
Skin; Erythema Multiforme .
Disseminated Infection.

TTT; Acyclovir 200 mg 5 times for 5 ds.


Herpes simplex encephalitis:.

There is more inflammation around


the temporal lobe, and the EEG
changes .
Acyclovir should be commenced upon
clinical suspicion as PCR result may
be a few days.
MENINGITIS
Meningococcal meningitis: Immunisation is available against strains A &C . (young
age)
Rifampicin ,ciprofloxacin and ceftriaxone can be used for prophylaxis.
Naisseria Meningitis:
Gram negative diplococci.
Presentation: fever, BP, DIC, palpable puprpura (purpura fulminans).
Treatment PenG; Dont wait for cultures if you suspect!
Prophylaxis for exposure (close contact): Options
Cipro 500 mg PO once
Rifampin 600 mg q12h x 4 dose
Ceftriaxone 250 mg IM x 1

Pneumococcal meningitis (adult) is caused by gram positive diplococci.IV penicillin


or ceftriaxone should be given. Dexamethasone is indicated in early infection to
prevent neurological sequelae.
Tuberculous meningitis: Symptoms are headache, vomiting, photophobia, and
fever. The duration of presenting symptoms may vary from 1 day to 9 months. CSF
typically shows elevated protein level, markedly low glucose, and lymphocytes.
Treatment of TBM includes (INH), (RIF), (PZA), and ethambutol (quadruple therapy).
Steroids are usually also indicated.
Listeria Monocytogenes
Small, gram +, motile rod (bacilli);
Causes aseptic meningitis/ septicaemia in neonates,
elderly , immuno-suppressed and pregnant women.
Associated with consumption of contaminated milk, ice
cream, undercooked hot dogs & pate.
Rx Amp + Gent.
Also sulfonamides.

Listeria meningitis is generally associated with


multiple cranial nerve deficits, particularly of the
VIth and VIIth nerves, as well as hemiparesis,
ataxia and respiratory abnormalities often leading
to respiratory arrest. High dose (eg 2g qds IV)
ampicillin & gentamycin are the treatment of
choice or sulfa.
Aseptic meningitis
Aseptic meningitis refers to patients who have clinical and laboratory
evidence for meningeal inflammation with negative routine bacterial
cultures. The most common cause is enterovirus.
Many causes, but usually viral.
Usually self-limited
CSF plenocytosis (monos), negative bacterial cultures.
Additional etiologies include other infections, (mycobacteria, fungi,
spirochetes), parameningeal infections, medications, and malignancy.
MENINGITIS VERSUS ENCEPHALITIS The presence or absence of
normal brain function is the important distinguishing feature between
encephalitis and meningitis.
Important Indications for Steroids in Meningitis
To control the inflammation in adults with acute bacterial meningitis
TB meningitis
Altered mental status or other signs of elevated intracranial pressure
Viral meningitis: A combination of lymphocytosis, normal or mildly
elevated protein and normal glucose suggests viral meningitis.
Common causes include enterovirus and mumps.
HSV-II.

genital ulcers; multiple, superficial, painful.


Latent in sacral ganglia.
Varicella (Chickenpox) &
Herpes Zoster (Shingles).

IP=1421 days,
Latent in dorsal root & cranial N ganglia .
face, scalp, and trunk, and later involves the extremities.
Rash: pruritic, centrifugal, papular, changing to vesicular, pustular, and
finally crusting (polymorphic) different stages of the eruption are usually
present simultaneously.
Complications;
1. Interstitial pneumonia is more common in adults
2. Encephalitis (1:1000),cerebilitis, Cerebellar ataxia (1:4000).
3. Reye's syndrome, Ramsy Hunt S.
4. In pregnancy; 1st 20 wksCongenital Varicella S.
laterH Zoster.
peripartumfatal.
Inv; EM, IF, serology.
Prophylaxis; ZIG IC, pregnancy, Newborn.
TTT; Acyclovir if; IC, pregnancy, within 72 hs for patients aged > 16 y.
Congenital Varicella S; lesions of an
extremity, growth retardation, microcephaly
microphthalmia, cataracts, chorioretinitis
and deafness.
Ramsy Hunt S; (geniculate ganglion) pain,
vesicles at EAM, facial palsy, loss of taste
from ant. 2/3 of tongue.
EpsteinBarr Virus
& Infectious Mononucleosis (IM)
Latent in B lymphocytes.
C/P;
IM should be suspected when an adolescent or young adult complains of;
fever, sore throat, malaise.
Cervical lymphadenopathy, splenomegaly.
Palatal petechiae,
a maculopapular rash in 90% after ampicillin or amoxicillin.

Complications:
- hepatitis, myocarditis,
- GBS, encephalitis,
- hemolytic anemia, thrombocytopenia,
- Lymphoproliferative disorder; Burkitt's lymphoma or nasopharyngeal
carcinoma, Oral hairy leukoplakia, B cell lymphomas among
immunodeficient patients.
Investigations
Hematologic findings
- lymphocytosis > 50% (most common).
- atypical lymphocytosis, > 10 % of total lymphocytes.
- DD of atypical lymphocytes ; toxoplasmosis, rubella, roseola, viral hepatitis,
mumps, CMV, acute HIV infection.
Heterophile agglutination tests
- detect Heterophil antibodies that agglutinate sheep red blood cells ( Paul-Bunnell
test), horse red blood cells ("Monospot" test). The sensitivity approach 85 % =
sensitive screening tests.
- false-positive heterophile tests; leukemia, lymphoma, SLE, HIV ,viral hepatitis.
- If the heterophile test is positive, no further testing is necessary if the clinical
scenario is compatible with typical IM.
- If the heterophile test is negative, but there is still a strong clinical suspicion, ask for
IgM and IgG VCA and EBNA antibodies.
EBV-specific antibodies
Viral capsid antigen& Nuclear antigen
- IgM and IgG antibodies directed against the Epstein-Barr viral capsid antigen
Nuclear antigen are usually present at the onset of illness & have high sensitivity
and specificity(97 and 94 %).
- IgM is a good marker of acute infection.
- IgG VCA & NA antibodies persist for life and are a marker of old EBV inf.
TTT; self limiting, steroid in Hemat & neuro.
Differential diagnosis:
- Streptococcal pharyngitis.
- CMV.
- acute HIV .
- toxoplasma infection.
- HHV-6, HBV.

The most important diagnosis to exclude is


primary HIV infection; this can be done with
both quantitative HIV RNA and HIV antibody
testing.
Cytomegalovirus
1.congenital infection
jaundice, HSM, thrombocytopenia,microcephaly,
periventricular CNS calcifications,mental retardation
motor disability & Hearing loss.
Cytomegalovirus
2. Disease in immunocompetent hosts
- asymptomatic.
- IMN like, but Monospot test ve.
- latent in endothelium & T lymphocytes.
3. Disease in immunocompromised hosts;
- retinitis ("pizza-pie" retinopathy).
- Gastrointestinal and hepatobiliary;
-Esophagitis, colitisgiant ulcers & bloody diarrhea, AIDS cholangiopathy.
- pneumonitis.
- Neurologic; polyradiculopathy, transverse myelitis,, and focal encephalitis.
- In transplanted liver hepatitis, which can mimic organ rejection.
- in renal transplant nephritis" = which can mimic organ rejection.
NB; CMV is not an oncovirus.
CMV
Cytomegalovirus
Inv;
- serology; Ig M.
- Histology; Owl eye inclusions.
- PCR
o TTT; in IC;
- ganciclovir, 5 mg/kg intravenously
every 12 hours for 1421 days.
- foscarnet, and cidofovir (nephrotoxic).
- Valganciclovir.
Which of the following is NOT a
potential treatment for cytomegalovirus
(CMV) infection?
A : Ganciclovir.

B : Foscarnet.

C : Cidofovir.

D : Valganciclovir.

E : Aciclovir.
HHV-6 (< 2 years)
children roseola infantum,, aseptic meningitis.
- In ICencephalitis and pneumonitis.
HHV- Diseases as HHV-6.

HHV-8

- Kaposi's sarcoma in AIDS patients.

- Castleman's disease (Fever, GLN, HSM).

- primary effusion lymphoma (body cavity


lymphoma).
TTT of all herpes group = ganciclovir.
Hepadna viruses

HBV.
HDV.
Papova Viruses
1. Human papilloma virus.
- Warts (genital, planter, flat)
- Cervical cancer (type 16, 18).
- Oral leukoplakia (16).
- SCC in IC.
- Condyloma accumunata.

2. Polyoma Viruses;
BK virus in IC tubulinterstial Nephritis &graft rejection .
JC virus PML;(memory changes , mood changes,visual)
MRI (perivent. White matter non enhancing lesions),
PCR , HIV Ab. Testing.
Progressive multifocal
leukoencephalopathy (PML)
is caused by chronic infection with JC virus, and
causes white matter lesions in the brain. A CD4+
of <100 predisposes to the condition .
JC virus invades oligodendrocytes, which
manufacture myelin, causing demyelination.
Hemiparesis, aphasia, cortical blindness, ataxia
and altered mental state may occur with an
insidious onset of dementia.
Anti-retroviral therapy including a protease inhibitor
is the main treatment for PML.
Parvovirus B19
by age 15 years about 50% of children have detectable IgG
In children, an exanthematous illness "fifth disease," erythema
infectiosum = slapped cheek appearance.
In ch H Anemia aplastic crisis and PRCA.

Middle-aged persons (especially women) symmetric


polyarthritis that mimics SLE and RA, involving the proximal
IP joints of the hands, wrists and knees.
In pregnancy , recurrent fetal loss .

Inv;

- IgM anti-parvovirus antibodies in serum


- PCR.
TTT; symptomatic.
Pox viruses
1.Smallpox (Variola ).
2.vaccinia ; vaccine for smallpox.
3.Monkey, Cow pox ( Zoonotic diseases).
4.Orf (contagious pustular dermatitis, or ecthyma contagiosa)
occupational disease acquired by contact with sheep & is
self limiting .
5.Molluscum contagiosum;
- transmitted sexually or by other close contact .
- Discrete, raised, solid, skin colored,1-2 mm in diameter,
central umblication, not surrounded by erythema.
- Kobner phenomenon.
- if giant or multiple suspect HIV.
TTT: curettage, cryo, liquid nitrogen , CO2 laser therapy,
trichloroacetate.
Human ORF
RNA Viruses
Poliomyelitis
At least 95% of infections are asymptomatic,
abortive poliomyelitis (minor illness), fever, headache, vomiting,
diarrhea, and sore throat.
nonparalytic poliomyelitis, signs of meningeal irritation.
paralytic poliomyelitis in 0.1% asymmetric, pure motor.
Prevention
- in the developed world , the inactive (Salk) parenteral vaccination
(IPV) is used for all four doses (at ages 2,4 ,6 and 18 months.

- In the developing world; Oral vaccines (OPV as sabin) ease of


administration, effective local gastrointestinal and circulating immunity,
The interval between oral polio vaccine doses should probably be longer
than 1 month. Intramuscular injections should be routinely avoided
during the month following oral poliomyelitis vaccination to prevent
provocation paralysis.
Coxachie,
Cutanuous & Oropharyngeal
Herpangina, Hand, foot & mouth dis. ( type A
Coxachie).
Neurological ( aseptic meningitis & encephalitis)

Cardiac ( Myocarditis, pericarditis, pleurisy with


type B Coxachie).
Muscle (Bornholm dis = intercostal myositis as
pleurisy with type B Coxachie)
ECHO, Entero 68-71- aseptic meningitis.
Coxsackie
(Hand, Foot and Mouth disease)
Influenza
Types; A,B ,C H1-5 & N1,2 are 2 glycoproteins on cell surface.
H (Hemagglutinin for viral attachement)
N (Neuraminidase for viral budding).
C/P; F, headaches, sore throat, limb aches, persistent dry cough.
Complications;
1) Strept pneumoniae, H influenza, staph areus.
2) Postinfectious encephalomyelitis, ADEM.
3) Viral pneumonia, pericarditis, myocarditis, myositis & GBS.
TTT;
- Bed rest, paracetamol.
- Antibiotics for patients with chronic bronchitis, cardiac & renal.
- Zanamivir, Oseltamivir ( neuraminidase inhibitor ) for pt > 65 yrs , IC, Used for
prophylaxis or infection within 2 days.
Prophylaxis;
1- inactivated influenza Vaccine.
2-Amantadine.
Given for pt > 65 yrs/year, CHD, CRF, chronic lung dis., DM.

Respiratory syncytial V.
- outbreaks of pneumonia in eldery & IC.
- TTT; ribavirin.
Measles= robeola
Prodrome of fever, cough, conjunctivitis, malaise, irritability,
photophobia, Koplik's spots.
Rash: red, maculopapular; onset 34 days after onset of prodrome;
begins on the face and proceeds "downward and outward," affecting
the palms and soles last.
Complications
1) Postinfectious encephalomyelitis, ADEM,SSPE .
2) Respiratory ; bronchopneumonia or bronchiolitis .
3) Gastroenteritis
Measles during pregnancy is not known to cause congenital
abnormalities of the fetus. However, it is associated with spontaneous
abortion and premature delivery.
Prevention;
- MMR at 1215 months and a second at age 46 years.
- IG within 5 ds of exposure. to; pregnant, child< 3y, IC.
o Inv; Ig M after 2 ds of rash.
TTT; Vit A mortality.
Mumps
Painful, swollen salivary glands, usually parotid.
commonly, orchitis (affecting fertility in around 5%
of bilateral cases), aseptic meningitis,
encephalomyelitis, pancreatitis, and oophoritis.
Mumps is an RNA virus. Its IP is 7 days.
Prevention over age 1 year, vaccines(eg, in MMR).
A second dose is recommended for children prior to
starting school.
Rubella= German measles(Toga viruses)
Prodrome; mild symptoms (fever, malaise, coryza) coinciding with
eruption.
Posterior cervical and postauricular lymphadenopathy 510 days
before rash.
Fine maculopapular rash of 3 days duration; face to trunk to
extremities.
Complications;
1.fetal infection during the first trimester congenital rubella $ in at
least 80% of fetuses. cataracts, microphthalmia, glaucoma, hearing
deficits, PDA.
2.Postinfectious Encephalopathy, 16 days after the rash.
Contact during the first trimester.
-immediate rubella antibody level should be obtained. If +ve=immunity.
- If veclinical observation and serologic follow-up & therapeutic
abortion.
Inv; Hemagglutination rubella antibody
Prevention; MMR not in pregnancy, at least 3 months before pregnancy.
Dengue( in Asia, Africa, America )
Ades Agypti.
C/P;
A) classic Dengue:
Fever, rash, back pain, LN .
severe musculoskeletal pain is the prominent feature.
B) Dengue Hgic fever :
Diagnosis; IgM Ab by Elisa.
TTT; supportive (IV fluids, bl. Transfusion ,
correct coagulopathy , no role for steroids).
Lassa fever

Subsahara west Africa (Nigeria)


CP; as dengue (Fever, rash, back pain,
LN , Hgic F).
Reservoir; rats (urine).
Diagnosis; Ig M, PCR.
TTT; Ribavirin
Yellow fever (Not in Asia)

Jungle YF;
Ades Africanus (in africa). (America).
Urban YF;
Ades Agypti.
C/P;
mild as influenza.
severe; 3 phases fever, flushing, conjuctival suffusion.
- remission.
- Hgic shock.
Diagnosis; clinical.
Prevention; live attenuated vaccine (17 d chicken embrio V.) protection >
10yrs.
TTT; supportive.
Rift Valley Fever
Africa (south, east).
Fever, hgic F, meningoencephalitis.
Diagnosis :serology- Ig M.

West nile F;
common, endemic in north America.
Transmitted by Mosquito.
Fever, headache,1% encephalitis.
Diagnosis; CSF & serum IgM.

Japanese encephalitis;
In east asia.
Wt loss, encephalitis
Diagnosis; Ig M by Elisa.
Vaccine; inactivated mouse brain V.
Hanta Virus
Europe, America.
Wild rodents by Aerosolized excreta.

Hgic fever, renal failure, pulmonary$

(pulmonary oedema, resp. failure).


Haemorrhagic fever
Causes:
Arbo viruses;
yellow F , Dengue, Rift valley fever, West nile, Japanese
encephalitis.
Other viruses; Hanta, Lassa, Ebola.
Rickettsia; Tuphus
Spirochetes; relapsing fever (borrelia).
C/P;
Fever, conj. Suffusion.
Liverhepatitis, jaundice, bleeding from all orifices, DIC.
Bloodpancytopenia (hgic shock), postural drop of BP.
Kid ATN, ARF.
Rabdoviruses (Rabies)
Urban Sylvian (wild)

Dogs, cats Bats

classic paralytic

Hyperreflexia, spasticity Ascending paralysis as GBS


Sympathetic overactivity, aero &
hydrophobia ,agitation,
hallucination, bezare
behaviour,convulsions,
Death within 2 wks
Rabies
virus travel along peripheral nerve to CNS.
Diagnosis;- Ag in salivary secretions
- Negri bodies in hippocampus & cerebellum postmortum.
IP; 1-2 months up to several yrs.
Management;
If animal captured watch it for 10 days.
If not1- Human diploid cell Vaccine.
2- wash wound with soap.
3- human rabies Ig.
4- antibiotics.
Retroviruses
Contain reverse transcriptase; RNADNA.

Lentivirus Oncovirus
HIV-1 HTLV-1
-Adult T cell lymphoma.
-Tropical spactic paraparesis

(a demyelinating disease that


causes spasticity and
weakness usually in middle-
aged women).
HTLV-2
HIV-2
- Myelopathy
- restricted to west Africa
Human immunodeficiency virus (HIV)
HIV is a retrovirus infecting T-helper cells
Transmission
Sexual (60-70%). Other causes: iv drug abuse (3%); infected blood
products; mother to child; accidental exposure (e.g. needle stick
injuries).
Clinical disease
Primary HIV
Early HIV
Most patients are asymptomatic
Symptoms: glandular-fever like syndrome of diffuse maculopapular
rash, fever, fatigue, night sweats and generalized
lymphadenopathy. Rarely, acute neurological symptoms (aseptic
meningitis, transverse myelitis, encephalitis). FBC may show
atypical lymphocytes.
Advanced HIV
Accompanied by immunosuppression or AIDS (if CD4 count <200
cells/mm3). Patients are at risk from opportunistic infections (e.g.
pneumococcal infection, TB, CMV, Pneumocystis carinii,
toxoplasmosis, and cryptosporidial diarrhoae) and AIDS-associated
malignancies (e.g. Kaposi's sarcoma, lymphoma).
Prevention
Prevention of transmission from mother to child ,risk can be dec. to <5%
by treatment with zidovudine given to the mother antenatally, during
delivery, and to the neonate for 1st 6wk.; elective LSCS; and advising
against breast-feeding
Antiviral drugs 3 groups:
Nucleoside analogues (e.g. zidovudine)
Non-nucleoside reverse transcriptase inhibitors (e.g. nevirapine)
Protease inhibitors (e.g. indinavir).
HAART (highly active antiretroviral therapy) is a combination of 3 drugs.
Adherence to therapy is essential to avoid resistance.
consideration for HAART
Patients who present with clinical manifestations of HIV(acute HIV $),
CD4 counts <350cells/mm3, or
viral loads > 30,000 copies.

Prophylaxis against opportunistic infection


Patients with low CD4 counts are started on prophylactic antibiotics:
<200 cells/mmPneumocystis carinii (co-trimoxazole)
<100 cells/mmtoxoplasmosis (co-trimoxazole)
<50 cells/mm Mycobacterium avium (azithromycin)& CMV
RECOMMENDATIONS FOR INDIVIDUAL PATHOGENS

Pneumocystis
TMP-SMX as first-line preventive therapy (one DS tablet
or one SS tablet daily)
intolerance or allergy to TMP-SMX, dapsone (100 mg
given once daily) (Grade 1A).
In patients who cannot tolerate TMP-SMX or dapsone,
aerosolized pentamidine
during the first trimester, pregnant women should be
treated with aerosolized pentamidine rather than
systemic prophylaxis
Toxoplasma
TMP-SMX as first-line preventive therapy (one double
strength tablet/ day)
who are allergic to TMP-SMX, dapsone plus
pyrimethamine plus leucovorin . Pyrimethamine is
teratogenic and should not be used during pregnancy.
Mycobacterium avium complex; macrolides for the primary prophylaxis
of MAC infection in patients with a CD4 count < 50 cells/microL (Grade
1A). E.g. weekly azithromycin rather than daily clarithromycin

Histoplasmosis; itraconazole (200 mg daily) for the prophylaxis of


histoplasmosis in patients with a CD4 count <100 cells/microL who are
at risk due to occupational exposure or who live in a community with a
histoplasma.

Candida; Do NOT use systemic therapy for the primary prophylaxis for
vaginal, oropharyngeal, or esophageal candidiasis due to cost and
potential for drug interactions (Grade 1A).

Cryptococcus; antifungal prophylaxis NOT be used routinely to prevent


cryptococcosis because of the relative infrequency of cryptococcal
disease, lack of survival benefits associated with prophylaxis,

Cytomegalovirus; Risk of CMV disease occurs mainly due to reactivation


of latent infection in the setting of advanced immunosuppression,
especially when CD4 counts are less than 50 cells/microL. However, we
do NOT suggest routine prophylaxis due to cost, risk of resistance, and
lack of proven survival benefit (Grade 2A).

Cryptosporidium, Do NOT administer prophylaxis for cryptosporidium


(Grade 2C).
Kaposi sarcoma
Caused by HHV-8.
Purple papules or plaques on skin or mucosa of any organ.
Metastasizes to lymph nodes:
Associated with AIDS or transplant patients commonly, skin or
pulmonary lesions.
Fatal.
Incidence decreased with HAART.
AIDS dermatology

Kaposi sarcoma.
Herpes Zoster.

Oral hairy leukoplakia; EBV in advanced


AIDS, premalignant .
Molluscum contagiosum

Bacillary angiomatosis.
Epstein Barr virus
Antiretrovrial therapy
is initiated in asymptomatic patients when the CD4 count
approaches 200. It is also initiated in patients with
symptomatic disease and higher CD4 counts and considered
in symptomatic seroconversion illness.

Nucleoside reverse transcriptase inhibitors (NRTIs) such


as (zidovudine) are associated with lactic acidosis, peripheral
neuropathy, pancreatitis, bone marrow suppression,
discolouration of the nails& proximal myopathy. Lamivudine
is the least likely agent to cause symptoms and is generall
well tolerated. DDI (Didanosine) causes pancreatitis.

Non-nucleoside reverse transcriptase inhibitors (NNRTIs)


inhibit reverse transcriptase non-competitively. Nevirapine
may cause Stevens Johnsons syndrome/ rash and deranged
LFTs.
Protease inhibitors, such as , saquinavir, ritonavir and
indinavir are associated with lipodystrophy, insulin resistance,
and disturbed fat and glucose metabolism
Indinavir crystallises in the renal tract stones
Bacterial infections
Impetigo

Crusted golden yellow erosions.


On nose & cheek, lips, chin.

Staph aureus, group A strept.


Erysipelas
Erythema + raised sharp demarcated borders,
group A strept( more commonly), & staph.
Shin of tibia & face.
Scarlet fever
Gp.A haemolytic streptococcus infection.
Incubation: 2-4 d.
Presentation: Fever, malaise, headache, tonsillitis, rash
(fine punctate erythema sparing face,palm & sole)& heal
by desquamation& pealing.
facial flushing with circumoral pallor.
strawberry tongue
Treatment: Penicillin V 250-500mg qds for 10d.
Complications: Rheumatic fever & acute
glomerulonephritis
Methicillin-resistant Staph.
aureus (MRSA)
MRSA acts in exactly the same way as any
other Staph. aureusit causes a range of
infections.
It is only different due to its multiple
resistance to antibiotics. Often contracted in
hospital.
Wash hands thoroughly with an appropriate
antibacterial preparation if they appear
soiled.
If hands appear clean, wash with an
alcoholic rub between each and every
patient contact.
2 Toxic shock syndrome
Staph aureus toxin mediated (TSST-1,
enterotoxin).
Fever, hypotension, generalised skin &
mm erythema ,vomiting, diarrhea,
confusion & MOF.
In menstrual & non menstrual patterns,
barrier contraception, puerpurium,
septic abortion, surgery, burns.
TTT; staph IV nafcillin or oxacillin.
strept pen. G + clinda IV
Streptococcus bovis usually enters the
bloodstream via the gastrointestinal tract.
Nearly all patients with St. bovis
endocarditis are older than 50 years, and
there is also an association with
malignancy of the GI tract.
Treatment is with penicillin or
vancomycin and gentamicin.
Haemophilus influenzae
Meningitis(60%); 8- 11% have permanent neurological sequelae;
Epiglottitis(15%)
Septicaemia(10%)
Osteomyelitis.
Septic arthritis
Pneumonia.
Pericarditis.
Management
Admit patients with severe infections. Organisms are often
penicillin-resistant and treatment is usually with iv cefotaxime.
Prevention
Vaccination is routinely offered to all children ,In addition, to all
unimmunized asplenic patients (preferably 2wk. prior to
splenectomy) and HIV +ve patients.
Diphtheria:

Corynebacterium diphtheriae causes


diphtheria. It is a gram positive rod. It may
present with sore throat, fever and
lymphadenopathy, heart failure or neurological
damage.
A greyish pseudomembrane can also form on
pharynx and conjunctivae. The illness is still
present in Eastern Europe and Russia.
Treatment is with diphtheria antitoxin, penicillin
or erythromycin.
Anthrax
is caused by a gram positive, aerobic, bacillus
called Bacillus anthracis. In humans, cutaneous
anthrax commonly causes a painless, black,
indurated eschar
Inhalational anthrax characteristically causes a
haemorrhagic mediastinitis. The prodrome includes
high fever and rapid deterioration follows.
Mortality of inhalational anthrax is 90% if untreated.
Treatment is with IV penicillin. Prophylaxis is with
ciprofloxacin.
Pseudomonas Aeruginosa
Common organism leading to infections
complicating burns
Cause of malignant otitis externa in diabetics.
when P.aeruginosa penetrates the epithelium
overlying the floor of the external auditory canal and
invades underlying soft tissue, cartilage, and bone.
Peudomonas aerugionosa and Staph aureus are
the most common causes of otitis externa among
patients in general.
May cause ecthyma gangrenosum in neutropenic
patients with bacteremia &common cause of
pneumonia and septicaemia.
In immunocompetent patients may cause UTI,
wound infections.
ecthyma gangrenosum
fish tank granuloma
fish tank granuloma

most commonly causes a localized cutaneous eruption, on


the fingers or hands of fishermen or seafood packers who
suffer minor trauma while handling contaminated shrimp,
crab, or fish. Handling fish.
Ethambutol plus rifampin appears more useful than
minocycline in treating cutaneous M marinum infection.
Granuloma and staining for acid-fast bacteria yielded positive
results in(63%).
Erysipeloid

Similar to fish tank granuloma. BUT


Erysipelothrix rhusiopathiae is gram-positive
bacillus.
Violaceous eythematous cellulitis to the hand.

TTT : .... penicillin V (500 mg/6 h) or


ciprofloxacin ,or erythromycin for 7days.
Occupational infectious diseases

Human ORF (virus)-------sheep handlers.


Sporotrichosis (fungal)-------------gardener

fish tank granuloma----- (MB marinum) fish


handler.
erysipeloid (Erysipelothrix rhusiopathiae) -----
--fishing
Clostridium Botulinum
(Botulism)
Botulism is caused by the neurotoxins of Clostridium
botulinum which interferes with presynaptic Ach
release, causing the 6Ds Dilated fixed pupil;
Diplopia; Dysphagia; Dyspnea; Descending paralysis.
It may be acquired by ingestion of toxin or more
commonly in the UK wound infection as a result of
intravenous drug use.
Therapy consists of approximately 10,000 IU of
antibodies against toxin types A, B, and E(antitoxin)
also supportive care (e.g. ventilation).
Tetanus
manifested by uncontrolled spasms, due to the
introduction of Clostridium tetani toxin into tissues.
Skin punctures, contaminated wounds with soil, dust,
neurotoxin (tetanospasmin) which causes severe
spasm---- painful muscle contraction and laryngeal
spasm which interfere with breathing .
The incubation period is typically between 1-2 weeks.
Vaccination with tetanus toxoid proved to be effective .
TTT:
Tetanus antitoxin.
Booster immunization for those who the last
immunization received was about 10 years or more.
supportive along with penicillin,
Pasteurella Multocida
Cat (>dog)bite is the #1 mode of transmission.

Pen G best, but Amox or Augmentin fine.

Capnocytophagia
Dog (>> cat) bites

Give Augmentin or Clindamycin

May cause bacteremia in immunosuppressed


patients, splenectomized patients, and alcoholics.
Cat scratch disease
Bartonella henselae, Benign self limiting.
1ry skin then tender LN

In IC-----Bacillary angiomatosis; vascular


papules or nodules, bartonella henselae&
quantana, transmitted by cats.
TTT: erythromycin or Doxycycline.
Cat scratch disease
Necrotizing fascitis
Anaerobe& aerobe.
DM & abd. Surgery.
C/P ---pain ,erythema,crepitus.
TTT: Debridement + pen G + clindamycine.
Hyperbaric oxygen.
Gas gangrene:
Clost. Perifringes
TTT: Debridement + pen G + clindamycine.
Hyperbaric oxygen.
Pseudomembranous colitis (Clostridium
difficile) common with clindamycine & is rare
with aminoglycosides, probably because they
have little activity against anaerobic gut flora.
Toxins mediated A or B.

TTT: metronidazole if not responding.......


Vancomycin.
Pertussis (whooping cough)
Caused by Bordetella pertussis.
Incubation: 7d.
Symptoms:
Catarrhal stage. symptoms and signs of URTI; lasts 1-2wk.
Coughing stage- increasingly severe and paroxysmal cough
with spasms of coughing followed by a whoop; associated
with vomiting& cyanosis. Lasts 4-6wk., then cough improves.
Examination: Chest is clear between coughing bouts.
FBC---lymphocytosis.
Complications: Pneumonia, bronchiectasis, convulsions,
subconjunctival haemorrhages.
Treatment with Erythromycin.
Prevention with DPT.
Enterobacteria

Examples include:
Salmonella
Shigella
Escherichia
Klebsiella
Enterobacter
Proteus
Yersinia
Some are normal gut commensals.
Others are pathogenic causing:
Diarrhea .
UTI . frequently E. coli; Proteus species
are associated with bladder stones.
Intra-abdominal infections. including
peritonitis and hepatobiliary infection.
Septicaemia.
Chest infection. Klebsiella may cause a
severe form of pneumonia.
Traveller's diarrhoea is an extremely common occurrence,
affecting up to half of travellers to high risk areas such as
Africa, Asia and South America. The commonest infective
cause world-wide is Escherichia coli.
Shigella, salmonella, campylobacter and amoebic
dysentery may be bloody. ( Yes Can Cause)
Cholera (profuse rice-water stools several litres/day)
and Giardia (explosive) diarrhoea are never bloody.
The common causes of bloody diarrhoea include
Salmonella , Shigella , Campylobacter and amoebiasis.
E. coli type H7: E 0157 can also cause bloody diarrhoea
with Haemolytic uraemic syndrome
The common causes of watery
diarrhoea

Giardia lamblia infection can present with abdominal


pains and diarrhoea or steatorrhoea. Duodenal aspirate +-
biopsy can confirm the diagnosis. Villous atrophy is
associated.
Bacillus cereus food poisoning is caused by toxin
release. There are two types of toxin, - the Diarrhoeal
(causing diarrhoea) and the Emetic (causing vomiting).
Symptoms are nausea, cramp like abdominal pains hours
after ingestion usually of rice and watery diarrhoea.
Cryptosporidium is water borne and is a protozoan
parasite. Swimming in hot tubes and pools, lakes are risk
factors. It can also be spread via uncooked food.
Treatment is conservative. Symptoms typically last for 1-2
weeks but longer in immunocompromised e.g. HIV.
Typhoid fever
Caused by Salmonella typhi and Salmonella paratyphi.
Spread: By the faeco-oral route.
Incubation: 3d-3wk..

Symptoms: Usually presents with malaise, fever,


headache, constipation (or diarrhoea), bruising, and/or
abdominal pain.
Examination: Pyrexia; relative bradycardia; rose-coloured
spots on the trunk (40%); splenomegaly; CNS signs
(coma, delirium, meningism).
Legionnaires Disease
Clinical: Lung/Liver/Lytes/Loose BMs
Lung Bilateral patchy infiltrates; relatively nonproductive cough;
Patients with community-acquired Legionnaires disease are much
more likely than patients with pneumonia from other causes to be
admitted to the ICU upon presentation.
Liver LFTs increase
Lytes Hyponatremia, hypophosphatemia
Loose BMs diarrhea.
Lymphopenia.

Weakness, malaise, high fever, cough.


The most common risk factors for Legionnaires disease are cigarette
smoking, chronic lung disase, advanced age and immunosuppression.
Laboratory Diagnosis:
b. Urine Antigen-1: Antigen in urine is detectable 3 days after the
onset of clinical disease, even if specific therapy has been started;
furthermore, urinary antigen persists for several weeks after antibiotic
therapy.
c. Ab with IFA (Indirect fluorescent Ab) test; > 1:256 or a 4-fold is
pos.
Treatment: Another clue to recognizing this disease is failure to
respond to -lactams (PCN or cephalosporins) or aminoglycosides.
Treatment options include erythro + rifampin; fluoroquinolone; or
azithromax.
Mycoplasma

Mycoplasma pneumoniae causes epidemics of


lower respiratory tract infection every 3-4y.
Spread by droplet infection.
Incubation :14 d.
Presentation :Dry, persistent cough
arthralgia.
CXR shows bilateral, patchy consolidation.
Infection is confirmed with serology.
Management :Erythromycin 500mg qds for
2wk. (alternative is tetracycline).
Mycomplasma Pneumoniae
(Walking Pneumonia)
Community-acquired pneumona, esp in young
patients
Cold agglutinin production + hemolytic anemia
Erythema multiforme, Stevens-Johnson syndrome
Neurological effects-Guilliain-Barre, cranial nerve
palsies, polio-like syndrome, aseptic meningitis
Mononeuritis multiplex
Erythromycin is the drug of choice in the young
patient with fever/dry cough/patchy bilateral
interstitial infiltrates, and relatively benign exam,
dont give PCN.
Chlamydial infection
3species:
C. trachomatis :Includes 15 serotypes. Causes
trachoma and inclusion conjunctivitis; sexually
transmitted diseases.
C. pneumoniae :community-acquired pneumonia,
especially in children and young adults. May be
clinically indistinguishable from pneumonia caused
by Mycoplasma .
Treat with tetracycline or erythromycin po for 2wk.
or azithromycin 500mg od for 3d.
C. psittaci :Infects many animals, but human
infection is closely related to contact with birds.
Treat as for C. pneumoniae)
Chlamydia Psittacosis
Most patients with psittacosis have a history of
contact with birds, e.g. poultry workers cleaning
out bird cages.
Clamydia psittaci infection of humans most
commonly presents as fever of abrupt onset,
severe headache, and dry cough , rigors, sweats,
and myalgias in a patient with a recent history of
bird exposure;
AST in nearly half of patients;
hyponatremia common.
Serology via the complement fixation test is
the test traditionally used to make the diagnosis.
Doxycycline and TNC are the drugs
Q fever
is a worldwide zoonosis & due to Coxiella
burnetii and In humans, is acquired via animal
contact or exposure results from inhalation of
contaminated aerosols . in farming communities
.
Clinical signs of Q fever are often extremely
mild or absent. Patients may be asymptomatic
or can present acutely with one of three clinical
presentations: A self-limited flu-like illness
Pneumonia. Hepatitis

Chronic infection most commonly involves the


heart as endocarditis. Aortic valve.& uvitis.
Treatment is with prolonged courses of
tetracyclines
Brucellosis
is caused by G ve coccobacilli which spread through untreated milk and
raw beef and may affect all systems.
B. abortus (cattle), B. suis (swine), B. melitensis (goats), B. canis (dogs)
are the different organisms.
Persons at risk Acquired via contact with animal tissues or after
ingestion of unpasteurized milk or cheese.
Farmers; dairymen; meat packers
Veterinary surgeons.
diagnosis
Cultures of blood or other sites, especially bone marrow or liver
biopsy specimens
Detection of brucella requires extended blood culture of up to 6 weeks.
. So, if you suspect it, ask the lab to hang on to the cultures for a longer
period.
CBC: Leucopenia is common.
Serology : Brucella agglutinins also helps confirm the diagnosis.
C/P; Spondylitis or osteomyelitis, endocarditis, pneumonia, liver granuloma
and jaundice and pyelonephritis.
Brucellosis
Brucellosis is a well-documented cause of fever of unknown
origin.
Virtually any organ system can be involved with brucellosis; most
commonly however:
Osteoarticular, especially sacroiliitis 20 to 30 percent.
Genitourinary, especially epididymoorchitis 2 to 40
percent of males.

Presentation
Fever, chills
Headache
Back pain, Arthralgias
Orchitis
Cough
Hepatomegaly/Splenomegaly,Endocriditis
Treatment: 2 accepted regimens:
Doxycoycline 100 mg PO twice daily for 6 wks plus
streptomycin 1 gram IM daily for the first 14 to 21 days.
Doxycycline 100 mg PO twice daily plus rifampin 600 to 900 mg
PO once daily for six weeks.
Neisseria Gonorrhea
Gram negative diplococci
Look out for the 20 y.o man or woman with fever +
swollen knee
>80% of women are asymptomatic; only 2% of men
asymptomatic.
Disseminated gonorrhea most likely to occur in
menstruating females;
2 phases
a) Bacteremic phase: tenosynovitis; skin lesions; joint
cultures negative
b)Nonbacteremic phase: monoarticular arthritis of
knee/wrist/ankle; joint culturemay be positive.
Remember, if a patient has a history of recurrent
infections, she probably has a deficiency in late
complement components (C5-8).
Patients with GC should also be empirically treated for
Chlamydia because of the high risk of co-occurrence.
Rickettsial Infections
Endemic typhus, epidemic typhus, Rocky Mt. spotted fever
All these infections have an insect vector.
They all yield a rash.
Rocky Mt. spotted fever (RMSF) erythematous and
hemorrhagic macules and papules begin peripherally
(wrists/forearms, ankles) and spread (to trunk, face). Fever,
H/A, myalgia. Rash begins on the 4th day of the fever.
Na+ is seen in half the cases.
transmitted by the American dog tick.
History of tick bite given in >80% of cases.
Doxycycline IV or PO BID x 7 days.
Typhus fever
FAHMRT.
Rash ---Measles like.

Multisystem affection.

Serology.
Doxycyclines.
Spirochetal infections

T. pallidum -------syphilis
Leptospira-------- Leptospirosis.

Borrelia.-------relapsing fever & Lyme dis.


Leptospirosis
Leptospirosis is a zoonosis caused by the spirochet,Leptospira interrogans.

Humans most often become infected after exposure to animal urine,


contaminated water or soil, or infected animal tissue through abraded skin,
mucous membranes or conjunctiva.
toxic phase: abrupt onset of fever, rigors, myalgias and Conjunctival suffus.
Immune phase: meningitis, myocarditis, arthritis,ARF,Jaundice.
NB .Conjunctival suffusion in a patient with a nonspecific febrile illness
should raise suspicion for the diagnosis of leptospirosis
Cultures of blood.
Serology.
TTT: penecillin G.
Lyme Disease:
Causes by the spirochete bacteria Borrelia Burgdorferi, which is
carried by the Ixodes dammini tick (the vector). It is seen
primarily in summer and fall.
Stage I
few days after the tick bite
Flu-like illness
ECM (Erythgema Chronicum Migrans; 60%)
Stage II
Weeks to months following bite.
10-15% develop neurologic abnormalities (e.g. Bells palsy)
and/or carditis)
Carditis and facial palsy are commonly reversible.
Stage III
Can become chronic
Arthritis develop in 30-50%
Usually oligo or monoarticular of large joints.
diagnosis with serology Lyme test
should be used only for confirmation if suspect the disease, and not
for screening, because of false positives and false negative.
Complications by system
Skin lesions (ECM).
Neurological- Cranial nerve palsies; peripheral
neuropathies; meningitis.
Cardiac- AV block; myopericarditis
Arthritis-large joints, asymmetric; often affects
the knee.
Treatment:
Early (ECM) Doxy or Amox x 2-3 wk;
Arthrisis Doxy or Amox duration unknown.
Carditis Ceftriaxone/Cefotaxime x 2-3 weeks.
Meningitis Ceftriaxone IV x 3 weeks
Pregnant women No doxycycline
Erythema chronicum migrans (ECM)
Rapid expanding ring with clearing middle
in trunk & axilla.
Relapsing fever
Borrelia Duttoni.
Rodents.

Tick born.

C/P----- (Haemorrahgic fever)

FAHMRT, Jaundice, HSM, Bleeding.


Diagnosis: microscopic demonstration of
spirochetes in blood during fever.
Serology.

TTT: Tetracyclines
Actinomyces Israelii
is a gram positive bacillus which behaves like a fungus, causing
actinomycetoma deep tissue infection which is granulomatous.
Nodules develop under the skin and erupt to discharge Sulfur
granules.
Anaerobic, gram +, branching, filamentous.
May present with
Paramandiblar infection with a chronic draining
sinus usually preceded by dental extraction.
Pulmonary abscess.
Rib destruction.
Chest wound infection.
Check anaerobic culture for diagnosis.
TTT: penecillin G.
Nocardia
Nocardia is another branching, filamentous gram
positive bacillus which may appear acid-fast.
It causes infection in immunosuppressed.
Causing fungal-like activity of mycetoma.
It is treated with co-trimoxazole.
Presents as chronic pneumonitis and lung abscess.
In patients with chronic pneumonia who develop
neurologic symptoms rule out nocardia brain abscess.
A 26-year-old female presents with a short
history of confusion, diarrhoea and
breathlessness. On examination she is
pyrexial at 38.5 C, pulse 120 bpm, Bp 80/60
and respiratory rate 26 breaths/min. She has
a faint blanching macular rash across the
trunk. She is disorientated without
meningism. Which of the following diagnoses
is most likely?
A : Salmonella enteritidis infection.
B : Pneumococcal meningitis.
C : Anaphylaxis.
D : Ecstasy overdose.
E : Toxic shock syndrome.
A 40-year old man from Saudi Arabia (in UK 4 days) presents
with painful episodes of trismus. He has never been vaccinated
against tetanus and sustained a slight cut to his foot while
walking barefoot in his garden in Saudi Arabia 3 days before
arriving in the UK. You suspect tetanus and he then has an
episode of generalised tetanus with respiratory arrest. Which of
the following is incorrect as part of his management?
A : Admit to side ward in intensive care unit.
B : IV gentamicin to kill any Clostridium tetani bacteria left in
wound.
C : Tetanus toxoid as he will not be immune to tetanus even after
recovery from infection.
D : Clean, debride and dress his foot lesion.
E : IM tetanus antitoxin.
Fungal infections

Systemic Subcutaneous Superficial


Candidiasis sporotrichosis Pityriasis versicolor

Histoplasmosis Subcut. Zygomycosis

Cryptococcus Mycetoma=madura Superf. Candidiasis


foot
Coccidiodomycosis Dermatophytosis

Aspergillosis

Blastomycosis

Zygomycosis=mucor
mycosis
Candidiasis
Most common fungal inf. In human, by candida Albicans.
Pt usually IC.
Candida infection commonly causes mucosal disease (eg. Vulvovaginitis).
Candidal oesophagitis.(small superficial erosions)
Cutaneous Candidiasis (intertriginous areas).
Endopthalmitis (causing blindness if not rapidly diagnosed) and IV line
related sepsis can also occur.
Hepatic candidiasis in neutropenic pt after BMT.
Chronic mucocutaneous candidiasis; rare, in children, +
endocrinopathies e.g. hypoadrenalism & hypoparath.(PAC).
The diagnosis can usually be made on blood cultures but the organisms
can be seen in the lesions on biopsy.
TTT; mild oral Nystatin.
severe Amphotericine, itraconazole IV.
Histoplasmosis As TB,
May occur in immunocompetent individuals.
causative agent, Histoplasma capsulatum, non encapsulated
fungus, transmitted by inhalation of spores in bird droppings.
found worldwide, particularly in North and Central America.
Histoplasmosis remains asymptomatic in most healthy
individuals following low level exposure.
Symptomatic infection usually produces self-limited
pulmonary illnesses, erythema nodosum, or pericarditis,
Individuals at the extremes of age or with underlying
immunosuppressive conditions can develop progressive
disseminated or diffuse pulmonary disease with cavity
formation.
Major clinical forms
Asymptomatic pulmonary Histoplasmosis; +ve skin test.
Acute diffuse pulmonary histoplasmosis- Following more
extensive exposure in areas with large amounts of bird droppings.
In such cases, antigen testing and cultures are often positive.
Chronic pulmonary histoplasmosis in Patients with underlying
lung disease productive cough, dyspnea, chest pain, fevers, and
sweats and have fibrotic apical infiltrates with cavitation.
Serology (complement fixation titers) is positive in nearly all cases.
Disseminated histoplasmosis as Disseminated TB; fevers, LN, HSM,
neutropenia, thrombocytopenia in infant, transplant, HIV.
Differential diagnosis sarcoidosis , tuberculosis , malignancy.
Diagnosis; demonstration of fungus histologically.
CFT after 3 wks.(serology).
TTT : Itraconazole, fluconazole and amphotericin B.
When to suspect pulmonary
histoplasmosis
1. Pneumonia with mediastinal or hilar lymphadenopathy.
2. Mediastinal, hilar masses, Pulmonary nodule suggestive
of malignancy
3. Cavitary lung disease suggestive of tuberculosis.
Shared findings with sarcoidosis include ;
1. diffuse pulmonary infiltrates, mediastinal lymphadenopathy,
2. erythema nodosum,
3. hepatic enzyme elevation, splenomegaly.
4. Serum angiotensin converting enzyme concentrations are
elevated,
5. noncaseating granulomas are a shared pathologic finding.
Coccidiomycosis
Coccidiomycosis manifests similar to histoplasmosis
and spreads to the lymph nodes.
In south America.
60% asymptomatic, 1%pulm cavities, fibrosis in IC.
Diagnosis;
1- Intradermal tests are diagnostic.
2- latex agglutination & precipitation test (Ig M).
3- CFT (Ig G).
Treatment is with Itraconazole, fluconazole &
amphotericin
Cryptococcus neoformans

Route; inhalation of fungal spores blood to the CNS, particularly in


patients with advanced AIDS.
Cryptococcal meningoencephalitis is the most frequently encountered
manifestation of cryptococcosis. Symptoms typically begin in an indolent
fashion, usually over a period of 1-2 weeks. The three most common
symptoms are fever,confusion, and headache. Stiff neck, photophobia,
papilloedema and vomiting are seen in 25- 30% of patients.
Pulmonary cryptococcosis (pl effusion, cavitation, fibrosis) is rare.
Diagnosis;
1. CSF +ve cryptococcal polysaccharideantigen Ag testing (90% sensitive).
2. Culture will almost always establish the diagnosis (100% sensitive).
3. The CSF WBC count is typically low (<50/microL) with a mononuclear
predominance and the protein and glucose conc. are only slightly abnormal.
TTT;
1. Induction therapy -- amphotericin B + flucytosine for 2 weeks.
2. Maintenance therapy fluconazole for at least 4 months.
DD of chronic meningitis
1. TB.
2. $

3.cryptococcal

4.sarcoidosis.
5. Behcet.

6.Malignant.

CP ----Meningism & inc. ICT


Aspergillosis

Major clinical forms


1. ABPA.

2. Invasive pulmonary aspergillosis.

3. Aspergilloma
ABPA
The major diagnostic features of classic ABPA
1. history of asthma.
2. Immediate skin test +ve to Aspergillus antigens. A
skin prick test should be the first step in an asthmatic
being evaluated for ABPA.
A negative prick skin test virtually excludes ABPA from
consideration.
1. Precipitating serum antibodies to A. fumigatus Serum.
2. total IgE concentration greater than 1000 ng/mL .
3. Peripheral blood eosinophilia >500/mm(3).
4. Lung infiltrates on chest x-ray or chest HRCT .
5. Central bronchiectasis on chest CT.
6. Elevated specific serum IgE and IgG to A. fumigatus.
Radiographic features.
CXR parenchymal infiltrates (usually involving the upper
lobes), atelectasis due to mucoid impaction, and a number of
findings characteristic of bronchiectasis . These include:
Migratory pulmonary infiltrate.
"Parallel Tram lines" due to the presence of ectatic bronchi.
Ring shadows due to mucus-filled bronchi.
TREATMENT aims to control episodes of acute
inflammation and to limit progressive lung injury.
Glucocorticoids are most commonly used,
increasing evidence of benefit from combined therapy with
itraconazole.
Aspergilloma
caused by a "ball" of fungal mycelia that can occur within
a cavity, usually within the parenchyma of the lung
Patients with pulmonary aspergilloma can be
asymptomatic. The most frequent symptom is
hemoptysis, in 75 % of patients . Less commonly, chest
pain, dyspnea, malaise, wheezing, or fever which may be
2ry to the underlying disease or bacterial superinfection
of the cavity.
CXR mass within a cavity surrounded by a radiolucent
crescent (crescent sign)
TREATMENT When required, surgery is the mainstay
of therapy.
Invasive pulmonary
aspergillosis
in the neutropenic (AML, CML, Hodgkin ) or
HIV-infected patient.
The major manifestation is fever that is
unresponsive to broad spectrum antibiotics.
TTT : voriconazole is superior to Amphotericin.

We recommend voriconazole as initial therapy


in patients with invasive aspergillosis (Grade
1B).
Pityriasis versicolor (also called
tinea versicolor) is a skin infection
caused by a fungus called Malassezia
furfur.
The treatment is topical selenium
sulphide or miconazole cream.
POTASSIUM HYDROXIDE PREP Potassium hydroxide (KOH) can be used
to microscopically identify fungus or yeast from epidermal skin scrapings. KOH
dissolves epidermal keratinocytes, allowing for easier demonstration and
identification of organisms. KOH prep is indicated to identify fungal infections
(eg, tinea pedis, manus, corporis, cruris, capitis; onychomycosis) and yeast
infections (eg, tinea versicolor, candidiasis).
WOOD'S LAMP EXAMINATION (BLACK LIGHT) A Wood's lamp
examination is used to aid in the diagnosis of tinea capitis, Certain specimens
will fluoresce when examined under a Wood's lamp.
FUNGAL CULTURE Fungal cultures are used to confirm the diagnosis of
fungal or yeast infection when the KOH test is negative,
Griseofulvin is not active against Candida
albicans or aspergillus. It is active against
trichophytons (tinea or ringworm) and other
dermatophytes. It is metabolised in liver (hence
caution in liver rather than renal failure).
Sporotrichosis

Ulceronodular dermatosis.
Sporotrhrix schenckii (soil fungus)

Gardner, farmer, florist.

+ chronic nodular lymphangitis & LN.


Sporotrichosis
Mucormycosis:
The hallmark of the disease is vascular
invasion with marked hgic necrosis.
Common in diabetics & DKA.

The commenst is Rhinocerebral.

Nasal stiffness, necrosis, black turbinates,


facial oedema.
Biopsy is confirmatory.

Ttt. Amphotericin B for 10-12 wks .

Pulm. , gastrointestinal, cut.,also occur.


Pneumocystis carinii (PCP)
May be classified as a protozoan or fungus. Causes
pneumonia in immunocompromised patients.
Presentation: Fever, breathlessness, tachypnoea, dry cough,
respiratory failure ( cyanosis).
Investigation: CXR----- normal or reticulonodular pattern or
ground glass appearance.
BAL & sputum culture may be diagnostic (sputum Induction).
Treatment : cotrimoxazole , clindamycine, IV pentamidine+
steroids 40 mg X 2 if O2 < 70.
Prevention: Prophylactic antibiotics are given to AIDS
patients with low CD4 counts< 200 continue till CD4 counts>
200 for 3 months.
Protozoal infections
Blood Protozoa Tissue Protozoa GIT Protozoa
Malaria Leishmania Giardia
Trypanosomiasis Toxoplasma Amebiasis
Babesiosis cryptosporidiosis
Microsporidiosis
Balantidiasis
MALARIA
IP=10-14 d.
Anopheles mosquito.
Plasmodium falciparum malignant subtertian malaria.
Plasmodium vivax and ovale benign tertian malaria(every 48.
Plasmodium malariae quartan malaria. (every 72 hours )
Stages;
1. Preerythrocytic stage (liverall).
2. erythrocytic stage; RBCs infected by micromerozoites
trophozoites schizontsmerozoites (release from ruptured
RBCs) gametes.
3. exo erythrocytic stage, only P. vivax and ovale remain in the liver
to cause relapse.
Plasmodium vivax and ovale only infect reticulocytes
falciparum infects erythrocytes of all stages (severe)

malariae infects mature erythrocytes.

NB; severe parasitemia= > 1% of RBCs are infected.

C/P; Fever, rigor, sweating, hemolytic anemia, splenomegaly.


People partially protected from severe malaria;
1. Repeated infection.

2. Certain Hb e.g. Hb AS (sickle trait), G6PD.


Severe Falciparum malaria
Parasitized RBCs;
1. are less flexible sequested in splenic post cap venules.
2. Develop knobs which bind adhesion molecules on cap
endothelium specially venules of brain, liver, spleen, kid .
C/P; (MOF)
1. Cerebral malaria is only caused by Plasmod. falciparum.
2. hepatitis, gastrointestinal symptoms(diarrhoea), spl.rupture.
3. GN/acute tubular necrosis and blackwater fever.
4. ARDS.
5. Severe anemia & DIC.
6. Hypoglycemia & met. Acidosis.
Diagnosis;
1. Thin & thick Bl film stained with giemsa, wright or leishman stain.2-3 Bl films
over 2-3 d.antimalarial should be stopped. Repeatedly ve exclude malaria.
2. Serological tests should not be relied on for malaria diagnosis bec. Ab tests
can remain +ve for months or years after infection.
TTT;
1. Chloroquine; 600 mg300mg after 6 hrs300 mg/24 hrs for 3 days.
2. Chloroquine resistant; Fansidar (sufadoxine 1.5 gm + pyrimethamine 75 mg)
3 tab as a single dose.
3. Chloroquine & Fansidar resistant; Quinine sulfate 600 mg 1x3x5 +
Fansidar 3 tab single dose.
4. Chl & Quinine resistant; mefloquine 20 mg/kg in 2 divided dose 8 hrs apart.
5. TTT of severe malaria Intravenous quinine is indicated in complicated
malaria e.g. cerebral malaria and hyperparasitaemia (>2%). Hypoglycaemia is
an important side effect of quinine therapy (causes insulin release)
6. For eradication & prevention of relapses (vivax & ovale); Chloroquine
followed by primaquine 7.5 mg/d for 14 d.
Chemoprophylaxis

1 wk before, during , 6 wks after.


Chloroquine sensitive 300 mg/wk.

Chloroquine resistant Mefloquine 250


mg/wk.(# with BB, neuropsychiatric).
Mefloquine resistant Doxycycline.
Trypanosomiasis
1) African Trypanosomiasis
- Sleeping sickness.
- Tryp gambiense & Rhodesience.
- Fever, LN, HSM, CNS.
- Diagnosis;
1. thick, thin bl film with giemsa stain.
2. Serology.
- TTT; suramin, eflornithin.
2) American Trypanosomiasis
= Chagas disease
- Blood sucking bugs Tryp. cruzi.
- Infection occur through rubbing bug over skin abrasions or
conjunctiva, bl transfusion & transplacental.
- Acute chagas firm reddish papule, fever, regional LN.the
parasite remain dormant in autonomic ganglia.
- Chronic chagas; (immune mediated tissue damage).
- Heart conduction abnnormalities, aneurysm formation &
cardiac dilatation.
- GIT mega oesophagus dysphagia & aspiration.
mega colon constipation.
Diagnosis;
1. Bl film in acute.
2. Serology in chronic.
TTT; acute benznidazole.
chronicsymptomatic.
Tissue protozoa Leishmaniasis
Leishmaniasis (Kala Azar) is spread by bites from female sandflies.
Multiply in macrophage & cells of RES then released into circulation after cell rupture.
Visceral leishmaniasis;=kala azar= black fever.
Main animal reservoir in Europe & asiadogs & Foxes. in Africarodents
- IP=1-2 months. - Leish donovani,
- Many infections are subclinical, but the classic presentation is with fever, weight loss,
hepatosplenomegaly, LN, pancytopenia and hypergammaglobulinaemia.
Diagnosis.
1- Demonstrating of parasite form aspirate from BM, spleen, LN. Splenic puncture is the
most sensitive means of obtaining a diagnosis (98%), but biopsy of the bone marrow
and liver is almost as good show Donovan bodies (amastigotes of Leishmania
donovani).
2- culture form these aspirate.
3- serology +ve in 95%.
4- pancytopenia, hypoalb, hypergamaglob.
5- Leishmanin skin test (CMI) -ve.
TTT,
1-Pentavalent antimony compounds IV I resistance is developing.
2- Amphotericin IV can achieve 98% long-term cure
3- pentamidine IV.
4- new oral drug; meltifasin.
This usually requires an invasive procedure
to obtain a tissue aspirate or biopsy. Both
staining and culture should generally be
performed because lesions can be
smear negative and culture positive or
vice versa . An alternative diagnostic
approach is to use immunologic based
tests, such as skin testing or serology.
Cutaneous leishmaniasis

- Old World (Africa, Mediterranean, Afghanistan)


and the New World (Central and South America).
- Cutaneous leishmaniasis can be caused by
several Leishmania; torpica, mexicano,
brasiliensis.
- Multiply in dermal macrophages.
Painless nodules->ulcerate with raised
bordercrustheal.
- Leishmanin skin test (CMI) + ve.
Treated with intra-lesional sodium stibogluconate
therapy. or ,( Na stibogluconate IV ) .
Toxoplasmosis
Intracellular protozoan.
Ingestion of contaminated food, undercooked meat
containing cysts, bl transfusion or transplacental.
C/P;
- Asymptomatic
- cervical LN; asymetrical, most pt have IMN like illness.
- Generalized LN.
- Myocarditis, hepatitis, uveitis, chorioretinitis.
- Neurological dis in IC (neck stiffness, headache, CSF;
high protein, CT; ring enhancing lesions).
- Cong toxoplasmosis (micro or hydroceph, MR, motor
imp, jaundice).
o TTT, pyremethamine & sulfadiazine,
o if pregnant spiramycin.
Toxoplasma choroidoretinitis
Babesiosis
(as malaria but in America)
As malaria in north America & europe.
Tick transmitted (Ixodes scapularis, Dammini).
Zoonotic disease in rodents & cattles & transmitted to man.
Babesia microti (rodents) & Babesia divergent (cattle).
In healthy individuals mild illness, recover spontaneously.
In asplenic pt or IC severe overwelming infection.
C/P; (as malaria) constitutional sympt, hemolytic anemia,
Hburia, renal failure, jaundice, HSM, abd pain, dark urine.
Lab; -thick & thin bl film.
-serology.
TTT; clindamycin +/- exchange transfusion.
Giardiasis
Common flagellate protozoan.
Most common parasitic infestation in travellers
returning to UK.
Path; villous atrophy +/- bact. Overgrowth.
C/P; watery diarrhea, distension (explosive
diarrhoea); no fever, steatorrhea.
IP (2wk.);
Stool microscopy may be +ve.
Duodenal aspirate---- flagellate protozoan.
Duodenal Biopsy.
If suspected, treat with metronidazole 2g single
doses daily for 3d.. Rapid response is diagnostic.
Amoebic dysentery

May begin years after infection.


C/P; Diarrhoea begins slowly, becoming profuse and
bloody fever malaise.
Chronic colitis liver abcess (through portal v),
ameboma of caecum, rectosigmoid.
Diagnosis is confirmed by microscopy of fresh stool.
(trophozoite)
Colonoscopy ---discrete flask shaped ulcers.
TTT; metronidazole 500 mg 1x3x5 ( x10-14 d in liver
abcess) followed by diloxanide furoate (luminal
amebeside).
Amoebic Liver abscess
Right lobe affected in 90% of cases.
Males in 80% of cases.

Single & unloculated abscess in 70% of cases.

Normal liver function tests.

aspiration -- chocolate syrup (anchovy sauce).

Pt with hepatic abscess usually do not have


diarrhea.
Serology : CFT is positive in 90% of cases.
Balantidiasis( as amoeba)
Balantidium coli.
Reservoir pigs.

TTT. Metro or tetra


Cryptosporidium
Protozoan causing diarrhoeal disease.
C. parvum causes most cases.
Route of inf. Direct person-to-person contact , or waterborne
transmission (faeces of cattle contaminating water supply &
swimming pools).
travellers to foreign countries, & IC patients are at high risk.
Incubation: 2-5d.
Symptoms: profuse watery diarrhea, abdominal cramp
nausea, anorexia, fever, and malaise.
Most common cause of watery diarrhea in HIV pt.
Diagnosis-- stool microscopy with modified acid fast stain.
Treatment is supportive. Usually symptoms last 1-2wk.
Immunocompromised patients develop profuse intractable
diarrhoea which is difficult to clear and may continue
intermittently for life.In HIVHAART)
Microsporidiosis

Diarrhea in HIV.
Diag; trichrome on fluorescent stain of
spores in stool.
TTT;Albendazole.
Helminthic infections
Neurocysticercosis is caused by Taenia solium (pork
tapeworm). This disease is found in South America and
Asia.
Neurocysticercosis typically is benign, and most lesions
resolve spontaneously within 2-3 months.
Diagnosis:
seizures (65-80%) due to localised inflammation that
accompanies their degeneration in the cerebral cortex
when calcified cysts occur.
Increased intracranial pressure (due to hydrocephalus,
which can occur in 15-25% of cases) causes headache,
nausea, and vomiting.
CT or MRI of the head may show granulomatous cysts.
ELISA of serum may confirm diag.
Albendazole is the recommended treatment.
Note: Tissue invasive helminth (worm)
infections may cause eosinophilia.
Protozoan parasites e.g. malaria,
amoebae do not.
Toxocariasis,
granuloma forms around the larvae which are spread by
dogs and cats. Fever, hepatomegaly and respiratory
symptoms constitute visceral larval migrans.
Treat with thiabendazole.

'Elephantiasis' affects the lymphatic system, causing


swollen legs. The organisms are filariae called Wuchereria
bancrofti.
can be detected on a blood film, and Oesinophilia.
filarial serology can be sent.
Treatment is with diethylcarbamazepine (DEC) or
ivermectin.
Echinococcus
is the cause of hydatid cysts. It is ingested
into the gut after handling of dogs (dog
tapeworm) and enters both portal and
pulmonary circulation. Hydatid cysts and
alveolar cysts form.
Treatment of choice is surgical excision plus
Albedazole and praziquantel with a high risk
of cystic rupture with metastatic spread of
organisms and anaphylaxis.
Schistosomiasis

Schistosoma haematobium infection is associated with


eosinophilia. The sexual replication stage may occur in
humans or other animal hosts.
Although most of the eggs are laid by the parasite in the
bladder (cystitis, granulomas, haematuria), The larvae
can travel to the lungs and cause pulmonary
hypertension.
some are deposited in the rectum and a rectal biopsy
may yield the diagnosis. The larvae can travel to the liver
and cause periportal fibrosis, portal hypertension.
Hepatomegaly is more typical of S mansoni and S
japonicum rather than haematobium.
Scabies

is spread by Sarcoptes scabei. It is


spread by contact only. Effective
agents are benzyl benzoate,
ivermectin and permethrin.
Genitourinary infections
Primary syphillis causes a chancre and inguinal
lymphadenopathy in the first weeks.(dark field examination)
Secondary syphillis (following few months) manifests as
generalised lymphadenopathy, maculopapular rash (affecting
hands and feet), fever, infectious papules around the perinanal
regions (condylomata lata) and snail track ulcers in the mucous
membranes of the mouth.
In tertiary syphillis (usually >2 years), gumma formation
(granulomas) Cardiovascular syphilis may lead to aortic
regurgitation as a result of a dilated aortic root and
neurological sequelae including general paralysis of the insane,
focal neurology (syphilis is the great imitator and dementia.
Successful therapy may be monitored by a fall in VDRL or rapid
plasma reagin (RPR) titre (if these were initially positive 75%
have positive VDRL in primary, 99% secondary)
There are two types of serologic tests for syphilis:

Nontreponemal tests such as


-Venereal Disease Research Laboratory (VDRL) test and -
Rapid Plasma Reagin (RPR) test.
specific treponemal tests such as
fluorescent treponemal antibody absorption (FTA-ABS) test
Treponema pallidum hemoagglutination assay (TPHA).
Microhemagglutination test for antibodies to Treponema pallidum
(MHA-TP)
The use of a single serologic test to diagnose syphilis is generally
inadequate because of the potential for false-positive results .
Thus, the usual testing algorithm is to screen with a nontreponemal
test such as the VDRL; a reactive specimen is then confirmed as a
true positive with a treponemal test such as the FTA-ABS.
TTT: Benzathine penecillin.
urethritis

Gonorrhoeae infection. Can cause penile discharge


and knee effusions with rash. The discharge and knee
aspirate may grow gram negative diplococci.
Treatment is with ceftriaxone IM single dose. Concurrent
treatment for chlamydia should be given for 3-6 weeks(oral
doxycycline 100 mg twice a day).

A patient presenting with urethral discharge may have


gonococcal or non-gonococcal urethritis (NGU). If there
are no gram negative diplococci seen on microscopy, it is
likely to be NGU.
The organisms causing NGU are as follows: Chlamydia
trachomatis 40% Ureaplasma urealyticum 20-40%
Trichomonas vaginalis (rare) < 2% Candidasis (rare) < 2%
Herpes simplex (rare) < 2%
Lymphogranuloma venereum is a sexually transmitted
disease caused by chlamydia trachomatis. It causes a
papule or ulcer that may occur on the penis, urethra or
cervix. Proctocolitis may also be present.
Confirmation of a diagnosis of LGV requires serological
tests or PCR on genitourinary specimens. Prolonged
treatment with doxycycline or roxithromycin for 3 weeks
is required for affected patients.
The differential diagnosis
of genital ulceration
chancroid, lymphogranuloma venereum , granuloma inguinale, also
herpes simplex virus infection and primary syphilis .
Multiple painful genital ulcers with localised lymphadenopathy may
suggest chancroid, which is caused by Haemophilus ducreyi.
Treponema pallidum infection causes (syphilis), which presents as a
solitary painless chancre.
Chlamydia trachomatis causes lymphogranuloma venereum,
which presents as a painless ulcerating papule and regional
lymphadenopathy.
Trichomaonas vaginalis is a protozoan. It is a very common
sexually transmitted infection in the developing world. Men are only mildly
symptomatic with urethritis and it is not a cause of genital ulceration. (
greenish discharge in females).
Herpes simplex virus infection
The standard assay for detecting
antibodies to HIV is an enzyme
immunoassay (EIA). (+ve within 3 month).
A confirmatory Western blot is performed
on EIA-positive specimens to exclude a
false positive test. (+ve within one month).
The diagnostic accuracy of serologic testing is
generally very high.
false negative the "window period" prior to
seroconversion HIV RNA testing in this
situation.
Rapid serologic tests offer several advantages
including fast turnaround times and
DDX of Fever + Purpura
(MERSA. Might also help you recall endocarditis &
sepsis; use as upside-down A)
M eningococcemia
E ndocarditis
R MSF
S epsis
ascultitis

Key Tick-Borne Disease (B aLERT on the exam!)


Babesiosis .
Lyme disease .
Erlichiosis.
RMSF .
Tularemia.
Nephrology
Urine analysis;
physical exam
Appearance; colour, odour.
Sp Gravity; n= 1003 1030
Chemical exam; pH, protein, glucose, ketones, bl,
urobilinogen.
Microscopic exam; crystals, casts, cells, organisms.
NB; nitrite test detect G ve Bact.
leukocyte esterase detect WBC (5-15/HPF)
RBC casts acute GN ( nephritic)
wbc cast interstitial N, pyelon.
tubular cell cast ATN.
Granular cell cast chronic GN, pyelo.
hyaline proteinuria
fatty Nephrotic
hematuria = rbcs > 3.
pyuria = wbcs > 4.
Red urine
Hematuria.
Hemoglobinuria.
Myoglobinuria.
Porphyrins.
Drugs; MD, L-dopa, desferoxamine,
metronidazol. Cyclophosphamidehgic
cystitis, Beetroot,.
Orange brown; direct bil, drugs;
phenazopyridine, nitrofurantoin, Fe, B1,
rifampicin, phenytoin.
Approach for hematuria
If heavy proteinuria >1 gm/24h , RBCs casts ,
dysmorphic RBCs >70% glomerular
serology, KFTs, U/S, renal biopsy.

If proteinuria <1 gm/24h, no RBCs casts,


dysmorphic RBCs <30% Non glomerular
urine cytology, spiral CT or IVP, cystoscopy,
or MRA.

In between manage according to clinical &


lab clues e.g.; PT, PTT, TB test, PCR for TB,
urine Ca.
Approach for proteinuria
If proteinuria >2 gm/24h, hypoalbuminemia,
dysmorphic RBCs glomerular serology, renal
biopsy.

if K, P, uricemia, glucosuria, aminoaciduria


tubular urine B2 microglobulin, heavy metal screen.

If proteinuria >2 gm/24h,disparity bet dipstick &


prot/creat, anemia , Ca , globulins overflow
(= plasma LMWP as MM, Hb, myogl urine
electrophoresis, BM biopsy, radiological survey.
Functional proteinuria
Causes
fever.
Exercise.
Congestive heart failure (renal ischemiaAgII).
Orthostatic proteinuria;
- young male
- Regress in 5-10 yrs.
- Diagnosed by early morning sample & by 12
hrs ambulatory & 12 hrs overnight.
Choice of imaging procedure
RAS.
- for screening duppler, MRA, CTA.
- to assess function ACEI renogram (renal
scan)
Chronic pyelonephritis & reflux nephropathy
IVP, DMSA scan.
Medullary sponge IVP.
Neoplasm US , CT, MRI for staging.
Dynamic renal scans are used to;

1) Measure GFR DTPA. A split GFR can


be obtained for each kidney, which is not
possible with the creatinine clearance
method.
2) Assess renal blood flow (RBF) in
patients with suspected renal artery stenosis
hippuran/ MAG3 + ACEI renogram.
3) differentiate between obstructive
versus non obstructive causes of
hydronephrosis MAG3 + Lasix renogram.
Lasix renogram
Non obstructive Obstructive
dilatation dilatation
Outflow +++ +++
delay
Parenchymal Normal delayed
transit time
(PTT) after
= immediate fall in time = .fall less
IVI of 40 mg
activity curve .does not fall
frusemide
2) Static renal scan

It consists of imaging of a radiotracer


(dimercaptosuccinic acid =DMSA) that is
taken up & retained by the renal proximal
tubular cells, providing a static image of
functioning nephrons.
Value,.
1. renal size, position, and axis.
2. focal renal parenchymal abnormalities,
e.g. scars appear as bites.
Renal biopsy
Indications;
1. Nephrotic S
2. Unexplained RF with normal Kid size.
3. Failure of recovery from ARF.
4. Asymptomatic prot., hematuria.

Contra indications;
1. Obese, oedema.
2. Uncontrolled HTN.
3. Bleeding tendency.
4. Shrunken kid.
5. Single kid except transplanted.
Complications;
1. Pain, hematoma, hematuria(in 20%, severe in 1-3%, need intervension
in 1;400)
2. AV aneurysm in in 20% but insignificant.
3. Infection
4. Mortality 0.1%.
Glomerulopathies
Normal Glomerulus (PAS)
Granular mesangial Linear Capillary

Granular
Clinical Presentations
* Asymptomatic proteinuria
* Nephrotic syndrome
* Nephritic syndrome
* Hypertension
* Hematuria which may be microscopic or macroscopic
* rapidly progressive renal failure
* chronic kidney disease.

Rapidly Progressive GN (or Crescentic)


- Pauci-immune ( ANCA associated Gn) =
- idiopathic cresentic.
- Microscopic Polyangiitis
- Wegener's Granulomatosis
- Churg-Strauss Syndrome.
- Immune complex e.g Post Streptococcal, IEC, lupus nephr.
- Goodpastures.
Hematuric Syndromes Proteinuric Syndromes Nephritic and Nephrotic
(Isolated Hematuria, (Isolated Proteinuria, Features
nephritic, or RPGN) Nephrotic)

-Mesangioproliferative GN -Minimal change disease -Membranoproliferative GN


(eg, IgA nephropathy)
-Focal and segmental -Mesangial proliferative GN
-Focal proliferative GN (eg, glomerulosclerosis(FSGS)
lupus nephritis WHO III, -Fibrillary glomerulopathies
infective endocarditis) -Membranous nephropathy
-Hereditary nephritis (Alport
-Diffuse proliferative GN -Diabetic glomerulosclerosi syndrome)
(eg, post- streptococcal GN,
lupus nephritis WHO IV)
-Amyloidosis.
-Crescentic GN ( severe IC,
pauci-immune nephritis, -Light-chain deposition
disease
anti-GBM nephritis)
What is this lesion encroaching the glomerular tuft.
What serious clinical condition is it associated with

Slide no 14

normal
Crescentic GN - (Trichrome Stain)
IgA Nephropathy (Berger dis)
one of the most common forms of GN worldwide.
male /female 2/1.
2nd and 3rd decades of life.
Causes, -1ry
-2 ry;, celiac, dermatitis herpetiformis, IBD, psoriasis,
alcoholic C
ankylosing spondylitis, mycosis fungoides, HIV.
- familial.
Pathogenesis; IgA deposited in the mesangium(polymeric , IgA)
C/P; children synpharyngetic macroscopic hematuria during
adults asymptomatic microscopic hematuria.
Prognosis; benign disease , progression to renal failure in 25
30% over 2025 years;
Recur in 50% posttransplant but good Graft survival.
IF: mesangial IgA and C3 IgG or IgM
IgA Nephropathy

<10 %
40 to 50 % 30 to 40 % Acute
hematuria CKD + proteinuria, or RPGN.

No Treatment progressive or severe disease. As RPGN


0.5 - 1g/day 1.0 -3.5 g/day
Monitoring ( s creat. >1.5 mg/dL,or rising ,
/6 -12 m. a GFR decline>15% /y,
nephrotic proteinuria,
marked proliferation without crescents

ACEI & ARBS Combined immunosuppressives ;


+ fish oil + statins. 6-months course
prednisone + cyclophosphamide for 2 yrs.
of steroids
What is this disease in a patient with abdominal
pain, hematuria, and renal impairment and a
palpable rash on thighs

What is the
Slide no 11
most common
glomerular
lesion on light
microscopy of
this patient
when a biopsy
is taken
Henoch-Schnlein purpura
distinguished clinically from IgA
nephropathy by;
- prominent systemic symptoms,
- a younger age (<20 years old),
- preceding infection, and
- abdominal complaints.
C/P; skin, arthritis, abd pain, Renal.
Ttt; arthralgias NSAIDs, severe abd pain,
renal steroids.
Poststreptococcal Glomerulonephritis
acute endocapillary proliferative glomerulonephritis
ages of 2 -14 years,
throat infections with particular strains of streptococci
(nephritogenic strains); After impetigo by 26 weeks
and 13 weeks after pharyngitis.
subepithelial "humps.
C/P; acute nephritic picture
C3 with normal levels of C4.
TTT; eradication of infection.
Postinfectious glomerulonephritis can occur in
patients with Subacute Bacterial Endocarditis ,
ventriculoatrial and ventriculoperitoneal shunts;
pulmonary, intra-abdominal, pelvic, or cutaneous
infections; and infected vascular prostheses.
ANCA associated GN
pauci-immune glomerulonephritis
C-ANCA= anti-proteinase 3 (PR3) in Wegener's
P-ANCA= anti-myeloperoxidase (MPO) more common
in microscopic polyangiitis, and Churg-Strauss
syndrome.
TTT;
Induction therapy usually includes some combination
of methylprednisolone, and cycloph.
plasmapheresis in case of pulm hge.
Maintenance, steroid tapering & give
cyclophosphamide for up to 2 years after remission.
1) Idiopathic Crescentic GN
Renal-limited glomerular capillaritis Pauci-immune
crescentic GN.
Both pANCA and cANCA +ve.
2) Microscopic Polyangiitis
Renal + systemic vasculitis.
3) Wegener's Granulomatosis
Renal + vasculitis + granulomas
nasal ulcers, sinus granuloma, hemoptysis
CXR...... nodules , cavities.
Biopsy of involved tissue small-vessel vasculitis and
noncaseating granulomas.
4) Churg-Strauss Syndrome
Renal + vasculitis + granulomas + eosinophilia.
Asthma, fleeting pulmonary infiltrates
May be associated with leukotriene receptor antagonists.
Antiglomerular Basement Membrane Disease

Autoantibodies directed against GBM


collagen IV.
focal or segmental necrosis with crescent.
IF; linear immunofluorescent staining for
IgG
TTT; 810 treatments of plasmapheresis
accompanied by oral prednisone and
cyclophosphamide in the first 2 weeks.
Goodpasture Syndrome:
Causes of pulmonary-renal S;
1. Microscopic Polyangiitis.
2. Wegener's Granulomatosis.
3. Good pasture $.
4. SLE.
5. Churg-Strauss Syndrome.
6. HSP, cryo.
Membranoproliferative GN=mesangiocapillary GN
Types;
Type I Disease (Most Common)
1ry.
2ry; SBE, SLE, HCV, cryo, HBV, solid malignancy.
Type II Disease (Dense Deposit Disease)
C 3 nephritic factor-associated
Partial lipodystrophy
Type III Disease
Idiopathic
Complement receptor deficiency

Pathology;
1. Subendothelial deposits
2. mesangioproliferative changes
3. mesangial interposition between the capillary BM and endothelial
cellsThickening of the GBM with a double contour .
4. lobular segmentation

Lab: low serum levels of C3 are typical.


TTT;
Long term alternate day steroids (prednisone 2
mg/kg) for one year, followed by slow tapering to
a maintenance dose of 20 mg every other day
for 3 to 10 years.
The role of aspirin and dipyridamole is unclear.

DD of MPGN;
cryoglobulinemia
Lupus nephritis class IV.
Nephrotic Syndrome
Heavy proteinuria> 3.5 gm/d/1.73 m2, minimal
hematuria, hypoalbuminemia,
hypercholesterolemia, edema, and no
hypertension.
TTT;
1. lipid-lowering agents
2. diuretics
3. inhibitors of the renin-angiotensin system can
lower urinary protein excretion.
4. +/- anticoagulants.
Minimal Change Disease
7090% of nephrotic syndrome in childhood but only 1015% of nephrotic
syndrome in adults.
Causes; 1 ry or 2ry to Hodgkin's disease, NSAIDs, IFN, IMN, lead.
Lab; selective proteinuria= LMWP.
Pathology;
LM, IF=nil
EM=effacement of the foot process.
TTT:
1. Prednisone 60 mg/m2/d for 4 wks then 40 mg/m2/d for 4 wks then gradual
tapering over 4 wks.
2. and other immunosuppressive drugs, such as cyclophosphamide,
chlorambucil, and mycophenolate mofetil, are saved for frequent relapsers,
steroid-dependent, or steroid-resistant patients.

Prognosis;
complete remission (<0.2 mg/24 h of proteinuria)
Relapses occur in 7075% of children after the first remission
steroid-dependent=relapse as their steroid dose is tapered.
steroid-resistant patients fail to respond to steroid therapy.
Membranous
Causes;
1 ry =70-80%.
2 ry;
- Malignancy; solid, NHL.
- Infection; HBV, HCV, P malari, S, leprosy.
- syst; SLE, MCTD, sickle.
-drugs; gold, penicillamine, captopril.
Patho;
LM& EM Thick BM, supepithelial deposits. IF; granular Ig G, C3
Prognosis;
40% spont remission
30% persistant proteinuria
30% progress to RF. need cytotoxic therapy
TTT
Alternate monthly for 6-12 months;
- pulse steroid then 0.5 mg/kg/d
- chlorambucil or oral cyclophosphamide.
Membranous GN :
Focal Segmental Glomerulosclerosis
segmental glomerular scars in some glomeruli.
1/3 of cases of nephrotic syndrome in adults and 1/2 of cases
of nephrotic syndrome in African Americans.
poor outcome in;
- Nephrotic range proteinuria,
- African-American race,
- renal insufficiency.
Causes;
1. 1ry
2. 2ry
- genetic (cong nephrotic S, steroid resistant, familial FSGS,
Alport, Nail & patella)
- infection; HIV ,Parvo B19 , Bilh.
- drugs; Heroin, lithium, pamidronate.
- Glom. HTN dt long standing nephron loss; sickle, single,
obesity, HTN, rejection, reflux.
PAS
perihilar
Path;
LM; FSGS.
EM; 1rydiffuse efface of foot process.
2 ry patchy efface of foot process.
IF; -ve except for Ig M & C3 trapped in sclerotic lesions
TTT
2ry as any nephrotic, TTT of the cause.
1ry steroid 1mg/kg/d for 3-4 months then
- if complete responsetaper after 1-2 wks over 3 months.
- if partial response (>50%)taper over 6-9 months.
- if little response add cyclosporin & switch to alternate day
then taper over 3 wks.
Hereditary nephropathies
ALPORT'S SYNDROME & Thin GBM
Transmisson;
1) 80%XL -------------------------- Alport, Thin GBM
= mutation in type IV collagen fibrils fragile GBM.
2) 15%AR, --------------------------- homozygous Alport , heterozygous Thin GBM
mutation in type IV collagen fibrils
3) 5% AD.
Clinical picture;
Alport:
Microscopic hematuria; begins at about 5 to 7 years of age.
Nephrotic proteinuria, HTN, ESRD late adolescence.
Extrarenal manifestations;
1- sensorineural deafness, start gradually in childhood, handicap
by 20 years of age.
2- Ocular abnormalities;lenticonus, Dot-and-fleck retinopathy;
not interfere with vision.
3- esophageal leimyomas .
Thin GBM= Benign familial hematuria
represent 25% of patients with microscopic hematuria.
Nail-Patella Syndrome
AD.
Clinical picture; appear at any age.
1) proteinuria to nephrotic syndrome.
2) nail dysplasia. (absent thumb nails)
3) Skeletal manifestations; absent patella, elbow dysplasia.
4) Eye manifestations; Heterochromia of the iris, cataracts.

FABRY'S DISEASE
XLR inborn error of glycosphingolipid metabolism.
defective activity of the lysosomal enzyme a-galactosidase A accumulation of
neutral glycosphingolipid in cellular lysosomes.
endothelial damage contributes to much of the pathology in Fabry's disease.
TTT= Enzyme replacement therapy.
C/P;
1) Renal Manifestations; progressive proteinuria & decline of renal function.
2) Skin symptoms; Anhidrosis, Angiokeratoma.
3) Visual symptoms; cornea verticillata,
4) Neurological symptoms; PN, Hearing deficit .
6) Other symptoms: coarse facial features.
diabetic nephropathy
Incidence
The risk for ESRD is 12 times as high in type 1
diabetes compared to type 2 diabetes.
About 80% type 1 will progress to proteinuria and
ESRD compared to only 20% of type 2.
Stages;
1. Renal hypertrophy & hyperfiltration.
2. Normoalbuminuria, but detectable glom lesions.
3. Microalbuminuria (30-300 mg/24h= 20-200Ug/min= U
alb/creat 0.03-0.3).
4. Overt proteinuria & azotemia.
5. ESRD.
Microalbuminuria
30-300 mg/24h
= 20-200 Ug/min
= U alb/creat 0.03-0.3.
Affects 25-30% of diabetics.
Progress in
- 80% of type 1 & 20% of type 2 without ttt.
Develop after 5-10 yrs of DM.
Screening for Microalbuminuria is recommended in
All type 2 diabetes at diagnosis.
Type 1 diabetes 5 years after diagnosis, at puberty.
Annually for all patient after.
Histopathology
LM;
- Thick BM.
- Mesangial expansion (diffuse, nodular=Kimmelstiel-Wilson lesion).
- Aff & eff arteriolar hyalinosis.
EM;
-fibrin cap (esinophilic focal thickening of a peripheral cap loop)&
- capsular drop (esinophilic focal thickening of bowmans capsule).
IF; psuedolinear deposition of alb & Ig G along BM.
Preventive Strategies in Diabetic Nephropathy.
1. Tight glycemic control (A1C <7%).
2. Aggressive control of hypertension.
Goal; DM <130/80
Diabetic nephropathy (>1g proteinuria/day) <125/75
3. Protein restriction.
4. Control of dyslipedemia.
5. Encourage smoking cessation.
6. Prevent sudden deterioration of kidney function.
7. Recent strategies in future.
Antihypertensives
1) ACEI;
Acute rise of serum creatinine of up to 30-35%, stabilize
after 2 months, may occur in proteinuric patients with
serum creatinine >1.4mg/dl.
greater increase of serum creatinine should raise the
possibility of RAS.
Albuminuria, serum creatinine and K should be
checked monthly till 2-3 months.
2) DHP CCBs (nefedipine- amlodipine):
may increase proteinuria and accerelate the
progression of diabetic nephropathy.
3) NDHP CCBs (deltiazem, verapamil) may reduce
proteinuria.
Lupus Nephritis
(WHO) classification & clinical presentation
INormal;-------------------Mild proteinuria
II Mesangial proliferation;---------------------asymptomatic hematuria or proteinuria
III Focal proliferative (FPGN); <50% of all glomeruli----active generalized SLE ----
--------------------mild-to-moderate renal disease
IV Diffuse proliferative (DPGN);-------Nephritic nephrotic + active generalized SLE
VMembranous;-----------nephrotic syndrome, usually without manifestations of
active SLE.
VI Advanced sclerosis , >90% of glomeruli------------significant renal insufficiency

Lab of lupus nephritis activity; +ve anti DNA,C3, C4 , +ve anti C1q(most
specific), antinucleosome Ab , urine (hematuria, RBCs casts).

Value of renal biopsy; poor correlation between C/P & histopath., associated
pathology e.g. drug induced AIN, TMA.
Characteristic lesions
LM;
Wire-loop lesion = massive subendothelial immune deposits
Hyaline thrombi = large intracapillary immune deposits
Fibrinoid necrosis = intimal immune and fibrin deposits
IF: full-house with IgG, IgA, IgM, C1q, C3, fibrin, and light chains.
EM:
All level dense deposits IImesangial;
III and IV mesangial,subendothelial,
Vsubepithelial
Fingerprint subtructure immune deposits ,
Tubuloreticular inclusions, present also in HIV, most diagnostic.
Therapy
Class I: no specific therapy.
Class II: if proteinuria > 1 g/dprednisone (20-40 mg/d)
for 1-3 months.
Classes III and IV;
Induction; pulse steroid +IV cyclophosphamide or MMF
Maintenance; prednisone 1 mg/kg/d gradual tapering to 5-10
mg/d for 2 years + azathioprine or MMF.
Class V: as 1ry membranous.

Renal affection in rheumatoid arthritis;


- Drug toxicity; NSAIDs AIN+nephrotic.
- 2ry amyloidosis.
Gammopathies
= diseases of globulins
=immunoglobulin overproduction

Alb.= 3.3- 4.7 g/dl.


1 glob.=0.1-0.4 g/dl.
2 glob.=0.3-0.9 g/dl.
2 glob.=0.7-1.5 g/dl.
glob.=0.5-1.4 g/dl.
Monoclonal Gammopathy
= Plasma cell dyscrasias
Polyclonal Gammopathy =immunoproliferative diseases
Due to proliferation of a single
Due to proliferation of Clone of Ig-forming cells that
many B cell clones produce homogenous excess of
e.g. CLD, Chr. light, heavy chains or complete IG
Inflammation, infection. molecule. (M-protein, where the
"M" stands for monoclonal).
MECHANISMS OF RENAL INJURY

I. Tubular precipitation; cast nephropathy (in MM)


II. Deposition; Amyloidosis, LCDD
III. Hyperviscosity; Waldenstrm macroglobulinemia
& Myeloma dt Igs.
IV. Glomerular reactions; MPGN, Pamidronate
induced FSGS
V. Tubular toxicity; ATN (NSAIDs, Iodinated
contrast), Fanconi syndrome.
DIAGNOSTIC APPROACH TO RENAL DYSFUNCTION IN PLASMA
CELL DYSCRASIA
I. Serum protein electrophoresis
- Monoclonal proteins will appear as a spike in the pattern
- Sensitivity (500-2000 mg/L)
- May not pick up small bands or bands outside of the gamma region
II. Urine protein electrophoresis
- Both serum and urine should be tested to increase detection to 95%
III. Immunofixation
- using Anti-serums to light chains.
- More sensitive than electrophoresis (detection limits 150-500 mg/L)
IV. Serum free light chains (FLC) assay
Most sensitive (detection limit of 0.5 mg/L)
Sensitivity is 99% when FLC is combined with serum and urine
immunofixation
V. Bone survey
VI. Bone marrow biopsy
VII. Abdominal pad of fat aspirate; 80% sensitive for AL amyloidosis
VIII. Renal biopsy
A. Should be performed on all cases if risk permits
B. Only way to distinguish between various kidney diseases
Multiple Myeloma
= CAST NEPHROPATHY

Precipitating factors
1. Volume depletion
2. Hypercalcemia
3. NSAIDs
4. Intravenous contrast
5. Infections
Pathogenesis
A. Increased tubular concentration of light chains enhanced by
decreased urine flow and furosemide.
B. Binding and co-aggregation with Tamm- Horsfall proteintubular
cast in the distal tubule then the proximal tubule
C/P;
Acute renal failure
10% to 15% present with ESRD
> 75% have subnephrotic range proteinuria
1. Mainly Bence-Jones proteinuria
2. Often dipstick negative
V. Treatment
A. Restore intravascular volume
B. Remove offending agents and nephrotoxic drugs
1. Hypercalcemia
i. Volume repletion
ii. Bisphosphonates in refractory cases.
C. Reduce light chain levels
1. Chemotherapy
i. Thalidomide plus dexamethasone,
or
ii. Bortezomib plus dexamethasone
D. Stem cell transplantation +/- kidney transplant is an option in
selected patients.
VI. Management of ESRD
A. Survival on dialysis is significantly decreased in patients with
dysproteinemia who reached ESRD.
1. Median survival was 2 -4 years for LCDD, & AL amyloidosis and 1
year for multiple myeloma.
What are these homogenous deposits with light microscopy
in this renal biopsy of a patient with long standing
rheumatoid arthritis who has recently developed nephrotic
syndrome

Slide no 12
AMYLOIDOSIS
TYPES OF AMYLOIDOSIS
1. AL amyloidosis
2. AA amyloidosis
3. Dialysis-related amyloidosis
4. Heritable amyloidoses e.g. heritable neuropathic
and/or cardiomyopathic amyloidosis due to deposition
of fibrils derived from transthyretin (also referred to as
prealbumin).
5. Organ-specific amyloid Amyloid deposition can be
isolated to a single organ, such as the skin, eye, heart,
pancreas, or genitourinary tract, resulting in specific
syndromes.
Renal Amyloidosis
- Congo red +ve.
- Biopsy of involved liver or kidney is diagnostic 90% , abdominal fat pad
aspirates are positive about 70%.
- SAP (serum amyloid P) scanning can identify the distribution of
amyloidSensitivity = 90 % & ; the specificity is 93 %.
1. AL amyloidosis, also called primary amyloidosis;
- 75% are of the lambda LC.
- 10% of these patients have overt myeloma.
- nephrotic syndrome is common, and about 20% of patients progress to
dialysis.
- treatment ;
1. melphalan+ dexamethazone, 4d courses/28 d for up to 9 courses.
2. autologous hematopoietic stem cell transplantation .
3. Thalidomide/dexamethazone.

2. AA amyloidosis is sometimes called secondary amyloidosis


- nephrotic syndrome, 4060% of patients progress to dialysis.
- It is due to deposition of -pleated sheets of serum amyloid A protein, an
acute phase reactant.
- 40% 2ry to rheumatoid arthritis, 10% have ankylosing spondylitis or psoriatic
arthritis, FMF
- treatment of the primary disease, Colchicine in FMF, Eprodisate.
Light Chain Deposition Disease

kappa light chains that do not form amyloid fibrils. Instead, they
self-aggregate and form granular deposits along the glomerular
capillary and mesangium, tubular basement membrane,
nephrotic syndrome , 70% of patients progress to dialysis.
not fibrillar and do not stain with Congo red,
IF + anti-lightchain antibody.
EM granular deposits.
Treatment = Melphalan/prednisone for 2 years as MM.
DD of nodular sclerosis;
Diabetic glomerulosclerosis
LCDD.
Amyloidosis.
Idiopathic.
Antiphospholipid antibody
Syndrome
Def & pathogenesis; auto Ab to phospholipids on endothelial cells
& LDL endothelial injury thrombotic microangiopathy.
Diagnostic criteria;
1 clinical (thrombosis arterial or recurrent venous/ recurrent miscarriage
before 34 wks of normal fetus)
+ 1 lab (anticardiolipin/ lupus anticoagulant)
C/P;
1. livedo reticularis, thrombosis; CNS, CVS, adrenal insufficiency,
lung, GIT, Kidney, ARF, TMA, HTN.
2. Catastrophic form => 3 organs simultaneously (Kid, lung, CVS).
Lab; PTT, +ve anticardiolipin/ lupus anticoagulant, false +ve
VDRL.
Ttt; APA +ve without previous thrombosis no ttt or aspirin.
APA +ve & previous clinical event long term warfarin, INR=2-3.
APA +ve & pregnancy heparin.
scleroderma
Renal failure in scleroderma;
- renal crises.
- TMA
- Membranous.
Path; as TMA but affect arcuate & interlobular arteries,
onion skin appearance, hypertrophy of JGA.
Marker of Scl renal crises; anti-RNA polymerase III
C/P; diffuse systemic scl + new onset severe HTN or
RPRF.
TTT; ACEI
Prognosis; continue ACEI after dialysis as 50% may
recover over 3-18 months.
Sickle cell;
Glomerular; hematuria , FSGS.
Tubular; DI , RTA , papillary necrosis.
Renal cell carcinoma.
Sarcoidosis; Ca stones, granulomas, TIN.
Sjogren $; Membranous, RTA.
Cryglobulinemia
Cryglobulins are proteins ((Ig & Ag) that precipitate on cooling to 4o
Precipitate in small cool Bvs in the peripheries complement activation
Raynaulds, vasculitis.
33% is called essential cryo (uknown cause, discovered to be mostly related to HCV)

Type I Type II Type III

Monoclonal Mixed Mixed


Mono Ig M against poly Ig polyclonal
G (RF activity)

causes Waldenstrom HCV - Autoimmune; SLE,


MM Plasma cell dyscrasia hepatobiliary, GN.
- Lymphoproliferative
- Chronic infection

C/P Hyperviscosity, Raynaulds, cutaneous ulcers, Purpura, arthralgia, LN,


HSM, peripheral neuropathy,
GN; HTN, acute nephritic, nephrotic less common.
Gn;
LM: Marked leukocytic infiltration, intraluminal deposits,
vasculitis with fibrinoid necrosis, TI infiltration.
IF; Ig M&G, C3.
Lab; C4, C1q,+ve cryo, RF, HCV Ab.
Poor prognostic factors;
1. Old age.
2. Recurrent purpura.
3. High s creat.
4. High cryo titre; Clinical and histologic activity does not always
correlate directly with detection of circulating cryoglobulins.
5. Low C3
TTT;
- steroids (pulse then 0.5 mg/kg for 6 months) + TTT of HCV.
- If severe; add cyclophosphamide,
- plasmapharesis& Rituximab.
Hypocomplementemic GN;
- Lupus nephritis
- Post infectious.
- MPGN
- Cryoglobulinemia.
HCV associated nephropathy;
1. MPGN +/- cryoglobulin
2. Membranous GN
3. FSGS
4. Prolif, TMA, fibrillary.
Thrombotic microangiopathies
(TMA)
Def; group of disorders ccc by fibrin deposition in the
lumen & wall of arterioles & glomerular cap.
Path;
LM; - intraluminal fibrin & plat.
- double contour of cap wall.
- fibrinoid necrosis of arterioles.
EM; tactoids of fibrin(subendothelial), swollen endo.
IF; no IC.
Lab; platelets, hemolytic anemia, shistocytes.
TTP more platelet.
HUS: more RBCs & cortical necrosis.
Thrombotic thrombocytopenic
purpura (TTP)
Pathogenesis;activity of VWF cleaving protein to < 5%
large VWF platelet adhesion dt;
- Familial ADAMTS13 mutation.
- Acquired Ig e.g. with ticlopedine,peripartum,sepsis,malig.
C/P; pentade;
fever, hemolytic anemia, thrombocytopenia, CNS, mild renal .
TTT; FFP, plasmapharesis,.
NB: platelet transfusion & antiplatelet e.g. aspirin are
contraindicated.
FU by LDH level.
Prognosis; mortality 90% if untreated, 90% survival if ttt.
Hemolytic Uremic Syndrome
(HUS)
Pathogenesis; endothelial injury;
1. By shiga toxins of Eoli O157 H7 (Diarreal form
D+HUS).
2. Complement mediated in familial form dt factor H.
3. Complement mediated in the acquired form by a
triggering factor e.g.pregnancy, malignancy, drugs.
C/P; triad of hemolytic anenia, thrombocytopenia,
severe renal failure.
Ttt; supportive, recombinant factor H.
Poor prognostic factors; old, pregnant, D-HUS,
shigella, pneumococcal, HTN, cortical necrosis,
marked leuckocytosis, anuria, persistent proteinuria.
Uses of plasmapharesis in renal dis;
1. Good pasture.
2. ANCA +ve dis.
3. Idiopathic cresentic Gn.
4. Cryo.
5. MM with hyperviscosity.
6. TTP & HUS.
Tubulointerstitial diseases
Acute interstitial nephritis
Causes;
1. Drugs; antibiotics, diuretics, NSAIDs.PPI.
2. Infections e.g.bact, viral as CMV, Hanta V, fungal.
3. Immune dis; SLE, Sjogren, sarcoid, cryo, acute transplant
rejection.
4. Idiopathic.
C/P; Fever, rash, arthralgias +/- flank pain, hematuria.
Lab; Serum.., KFTs, & electrolytes.
Urine.....RBCs , WBCs, White cell casts, eosinophiluria,
Eosinophilia, tubular defects, proteinuria<1g.
TTT; stoppage of offending agent high dose steroid for 2-3
wks for drug induced & systemic dis. not for infection.
Chronic interstitial nephritis
Causes;
1. Drugs; analgesics, lithium, heavy metals as lead, calcinurin inhibitors.
2. Infections e.g. EBV.
3. Immune dis; SLE, Sjogren, sarcoid, cryo, chronic transplant rejection.
4. Obstructive uropathy.

CP.....> CKD,HTN, PCT defects, marked acidosis, hypo or hyperkalemia.

Lead; renal, blue line in gum, gout, wrist & foot drop.
diagnosis; lead mobilization test.
TTT; chelation with EDTA or oral succimer.

Lithium; DI; ttt; stop it , give amiloride , thiazide ,


endomethacin,carbamazepine
RTA, CIN, FSGS, ARF in acute intoxication, hypothyroidism, goiter,
hypercalcemia.
Effect of NSAIDS
Vasomotor ARF.
Acute interstitial nephritis + nephrotic S.
Acute papillary necrosis.
Nacl retension.
K in low RAA as DM, ACEI.
CKD; either NSAIDS- induced CKD.
NSAIDS- induced CKD analgesic nephropathy
Mixture; aspirin,
paracetamol, caffeine ( 2-3)
Kg).
=
age> 60 40-50
less Anemia out of proportion
Does not occur with low Diagnosed by non-contrast
dose aspirin CT papillary necrosis &
medullary calcification, small
kidneys, irregular contour.
RENAL FANCONI SYNDROME

generalized dysfunction of the PCT.


phosphate, glucose, amino acid, and bicarbonate
wasting by the proximal tubule.
C/P;
in children is usually rickets and impaired growth.
In adults, osteomalacia and osteoporosis.

+ polyuria, renal salt wasting, hypokalemia, acidosis,


hypercalciuria, and LMW proteinuria.
Papillary necrosis
Age > 60yrs except sickle.
Due to medullary blood flow.
Causes;
DM
UT obstruction.
Analgesic nephropathy.
Sickle cell anemia.
Graft rejection
Pyelonephritis, TB.
Hyperviscosity syndromes.
C/P; hematuria , necroturia , loin pain, UTI, sepsis, renal failure.
Diagnosis; IVP (best). signs;calyceal irregularities, sinus tract,
ring sign, clubbing, filling defects.
prevention; ACEI are protective.
TTT; control B sugar, avoid analgesics & drugs that Bl flow as
thiazides, BB, fluids,
Vesicoureteric reflux
Not inherited.
Most common cause of ESRD in children.
Child with UTI 30% VUR.
NB;
Most common inherited cause of ESRD ADPKD.
Most common inherited cause of ESRD in
childrennephronophthisis (as medullary cystic dis).
Reflux nephropathy=VUR + CIN.
Grades; I ureter
II ureter & pelvis ( without dilatation).
III pelvic dilatation, preserved forniceal angles.
IV blunting of forniceal angles.
V clubbing of forniceal angles.
Diagnosis;
- early micturating cystography, dynamic renal scan.
- late U/S, DMSA scan.
Screening for VUR in:
: 1st attack of UTI at any age.
: 2nd attack or- 1st with family history,
- abn. Voiding,
- HTN,
- poor growth,
TTT;
Grade I nothing.
Grade II, III till puberty antibiotics TMP-SMX or nitrofurantoin,
cephalexin.
Grade IV, V Long-term antibiotics or surgery.

NB: Surgical correction of VUR in children have failed to show significant


benefit in terms of renal function & progressive scarring. Surgical
correction is reserved for the child who, in a 2- to 4-year period, appears
to be not responding to medical therapy.
Cystic diseases of the kidneys
1. Hereditary;
- ADPKD.
- ARPKD.
- Tuberous sclerosis
- Von Hippel-Lindau disease (VHL)
- Juvenile nephronophthisis and medullary cystic disease.
2. Acquired renal cysts;
- Medullary Sponge Kidney.
- acquired cystic disease.
- Simple cysts.
cystic Kidney Disease
Inherited Cystic Mode of Renal Extra-Renal
Kidney Disease Inheritance Abnormalities Abnormalities
ADPKD AD Cortical and Cerebral aneurysms; liver
medullary cysts cysts, othera

Nephronophthisis AR Small fibrotic Retinitis pigmentosa


kidneys; medullary
cysts
Medullary cystic AD Small fibrotic None
kidney disease kidneys; medullary
cysts
Tuberous sclerosis AD Renal cysts; Adenoma sebaceum;
angiomyolipomas; CNS hamartomas
Von Hippel-Lindau AD Renal cysts; renal Retinal angiomas; CNS
disease cell carcinoma hemangioblastomas;
pheochromocytomas
Adult Polycystic disease
Commonest inherited kidney disease.
Accounts for 5-10% of (ESRD).
Clinical Picture
1. Renal disease; Abdominal pain , hematuria, Palpable kidneys,
Recurrent UTI,stones, Hypertension, Renal Failure .
2. Extra renal disease; Cerebral Aneurysm (Berrys) (5-10%),
Hepatic, Pancreatic cysts, Mitral Valve Prolapse
(26%),diverticulosis.
Diagnostic radiological criteria
If +ve Family history
< 30 years ----- 2 cysts in at least one kidneys.
30 - 59 years ----- 2 cysts in each kidney.
> 60 ---4 cysts in each kidney.
If -ve Family history 5 cysts in each kidney.
TTT;
1. ACEI, ARBs
2. If Hge; analgesics, rest, hydration.
3. If infected; bl culture not urine,sutrim, cipro, vanco not ceph nor
aminogly.
APKD
Tuberous sclerosis
AD, hamartin (tumor suppressor)multiple hamartomas.
Clinical Picture;
CNS epilepsy in 80%, Mental retardation.
SkinFacial adenoma sebaceum, Shagreen patches
(lower back), Ash leaf (Hypomelanotic macules).
Periungual fibromas.
Renal (60%) cysts, Angiomyolipomas.
Retinal hamartoma (50%), is almost always
asymptomatic.
Liver ( 40% ), angiomyolipomas and cysts.
Heart (rhabdomyoma).
Lung (lymphangiomyomatosis; affects females)
Acquired renal cysts
Medullary Sponge Kidney
- dilatation of collecting ducts,
Benign course.
Calcify nephrocalcinosis, Nephrolithiasis and UTI.
Defective concentrating ability & RTA.
Diagnosed by IVPradial, linear striations in the papillae
or cystic collections of contrast .
Dialysis Cysts= acquired cystic disease.
the size of the kidneys is usually not markedly increased,
Precancerous.
Simple cysts
- Fluid filled, may enlarge to 10 cm.
- Single or multiple.
- Common 2% of people< 50y, up to 20% > 70yrs.
Acquired cystic disease of the
kidney
Acquired cystic disease of the
kidney
Vascular renal diseases
Renovascular HTN
Clues;
- abrupt onset, accelerated HTN.
- recurrent flash pulmonary edema.
- Deterioration in renal function with BP reduction
and/or ACE inhibitor therapy.
- Generalized atherosclerosis obliterans.
- asymmetrical kidneys.
Causes;
- RAS (narrowing > 50%)
- vasculitis
- TMA.
Atherosclerotic renal artery disease
(ASO-RAD)
5th to 7th decades of life.
70-80% ostial stenosis.
Tend to progress, renal impairement.
TTT; anti HTN, lipid lowering,antiplat.
interfere if; > 60% stenosis.
progressiveKFTs
evidence of salvageability;
- normal Kid size
- good function in renal scan
- RI<80
TTT percutaneous angioplasty +stent .
if failed surgical revascularization
114
Classification of renal artery stenosis:
1. Atherosclerotic renal artery disease (60 - 80%)
2. Fibrous dysplasias (20 - 40%).

FMD
Female, 30-50 yrs.
Slow progression.
Renal functions preserved.
1% of HTN.
Most frequent is medial dysplasia with multiple contiguous
stenosis string of beads.
TTT; ACEI, percutaneous angioplasty (curable, low
restenosis), if failed surgical revascularization
Hypertensive Nephrosclerosis
27% of ESRD patients.
risk factors for progression to ESRD include age,
sex, race, smoking, hypercholesterolemia,
duration of hypertension, and preexisting renal
injury.
Kidney biopsies (not needed); arteriolosclerosis,
chronic nephrosclerosis, and interstitial fibrosis
in the absence of immune deposits .
TTT; Treating hypertension <130/80 mmHg if
there is preexisting diabetes or kidney disease,
most patients begin therapy with two drugs,
classically a thiazide diuretic and an ACE
inhibitor.
Renal vein thrombosis
Etiology; History of nephrotic syndrome or
pulmonary embolism.
C/P; Flank pain.
Urine analysis; Mild proteinuria Occasional
hematuria.
Renal venogram or MR venogram are
diagnostic.
Atheroembolic disease
Def; separation of cholesterol crystals from atheromatous
plaques to small renal arteries.
Etiology; Vascular disease; classically occurs within days
weeks of manipulation of the aorta or other large vessels as
coronary angio, or in the setting of anticoagulation.
Clinical picture; Retinal plaques, palpable purpura, livedo
reticularis, slow deterioration of renal function.
Lab; Eosinophilia, Hypocomplementemia, Eosinophiluria
Diagnosis; Skin or renal biopsy ccc clefts, concentric intimal
fibrosis, FSGS.
TTT; stop anticoagulation, low dose steroids.
Urinary tracts
Urinary Tract Infection
The infecting organism
E coli =80-95%
Staph. Sapro. = 5-10%
Enterococci (Strept. fecalis) =5-14%
Others; proteus = 2-3%,Klebsiella = 2-3%, Pseudomonas, candida esp in
complicated cases.
Uncomplicated UTI= healthy young woman.
1. Uncomplicated cystitis;
- C/P; dysuria, frequency, suprapubic pain.
- diagnosis; history, urine analysis pus cells>4/HPF.
- TTT; 3 daysTMP/SMX or Quinolones or 5d Nitrofurantoin.
- if recurrent long term small dose postcoital or 3 times/wks at bed time.

2. Uncomplicated pyelonephritis;
- C/P; fever, loin pain.
- diagnosis; urine culture; bact count >104
- TTT; TMP/SMX or Quinolones (IV if vomiting) till culture sensitivity for a total of 14
days.
Complicated UTI;
- In structural or functional abn e.g. anatomic
abnormality, Instrumentation, Medical condition; as
Pregnant, Diabetic CKD,
transplant,Nosocomial,Childhood UTI, symp.> 7d,
Drugs; antibiotics, immunosup.
- Elderly, men & children.
diagnosis; bact count >105.
TTT; 7 days for lower & 14 days for upper UTI with broader
spectrum Ab covering pseudomonas e.g.
Piperazin/tazobactam, Cefepime, Imipenem, Meronem,.
if recurrentlong term suppressive therapy ie full dose
then dose when culture ve.
Catheter-Associated UTI

Asymptomatic patient
no therapy is indicated. (as relapse is very
common).
In symptomatic patient
antibiotics is based on the Gram's stain of urine
or the antimicrobial sensitivity patterns.
Prophylactic antimicrobial therapy
In case the time of catheterization is clearly
limited (e.g., in gynecologic , vascular surgery,
kidney transplantation).
UTI in Men
In men > 50 years with UTI,
- Intensive therapy for at least 4 to 6 weeks
up to 12 weeks is recommended due to deep
tissue invasion of the prostate & the kidneys
even in the absence of overt signs of
infection at these sites.
Treat Relapse
(1) long-term antimicrobial suppression
(2) surgical removal of the infected prostate.
Asymptomatic bacteriuria in Pregnancy
Screening for asymptomatic bacteriuria by
urine culture at the first prenatal visit (12-16
wks pregnancy) is mandatory (for fear of
pyelonephritis that develop by the end of the
second trimester & so may lead to premature
delivery).
If +ve repeat to confirm then
treat with 5- 7 days of nitrofurantoin (100 mg twice daily) ,
amoxacillin (500 mg PO three times daily )
cephalexin (500mg twice daily).
FU culture 1 week after and then monthly until the completion of the
pregnancy.
Fungal Infection of the Urinary Tract
Candida AlbicansFluconazole, itraconazole or 5 flurocytosine.
Treat if:
1.symptomatic,
2.Asymptomatic only if neutropenia, or urinary tract manipulation or
repeated culture counts > 10.000.
otherwise rapid recurrence is common, selection of resistant
Candida, and clinical outcomes do not appear to be improved .
Regimens:
1. Catheter-associated candidal UTI,
removal of the preceding catheter,
insertion of a three-way catheter, and
infusion of an amphotericin rinse for a period of 3 to 5 days
.
2. Without catheter, fluconazole, 200 mg/day for 10 to 14
days, (insertion of a catheter for an amphotericin rinse carry risk
of bacteriuria).
Success is increased if such contributing factors as hyperglycemia,
corticosteroid use, and antibacterial therapy can be eliminated.
Etiology of Urolithiasis
Anatomical causes
- ureteropelvic junction (UPJ) obstruction,
- Horseshoe or ectopic kidney
- vesicoureteral reflux,
- calyceal diverticula
- medullary sponge kidney
Metabolic causes
- low urinary volume,
- hypercalcuria (25%40%),
- hyperoxaluria (10%50%),
- hyperuricosuria(8%30%) and
- hypocitraturia (5%30%)
Major Causes of Renal Stones
Stone Type and Etiology Diagnosis Treatment
Causes

Calcium stones 85% , Alkali supplements +


Idiopathic hypercalciuria Hereditary(?) Normocalcemia, unexplained diet; thiazide
50% hypercalciuria ( > 300 mg /
24 hrs ).
1ry hyperPTH Neoplasia Unexplained hypercalcemia Surgery
Distal RTA Hereditary Hyperchloremic acidosis, Alkali replacement
minimum urine pH >5.5
hyperoxaluria High oxalate or low Urine oxalate >45 mg per 24 h Low oxalate diet
calcium diet Cholestyramine
Bowel surgery pyridoxine
Hereditary

Hypocitraturia Hereditary (?), diet Urine citrate <320 mg per 24 h Alkali supplements
%Occurrence Etiology Diagnosis Treatment
Uric acid stones 5-10%
Hyperuricosuria Hereditary Clinical diagnosis Alkali and allopurinol
50% Gout Intestinal, habit Uric acid stones, if urine uric acid >1000
50 % Idiopathic Urine uric acid mg/d
Dehydration >750 mg /d (women),
>800 mg /d (men)
Cystine stones 1% Hereditary Stone type; elevated Massive fluids, alkali ,
cystine excretion D-penicillamine
Struvite stones Infection Stone type Antimicrobial agents
5-10%
Acetohydroxamic acid
judicious surgery

Dietary modification
Increase fluid intake to maintain urine output of 2-3 l/day:
Decrease intake of animal protein
Restrict salt intake
Normal calcium intake.
Decrease dietary oxalate;
RENAL STONES
Calcium oxalate stones are the commonest kind of stones.
Calcium phosphate stones are the second commonest and
associated with 1ry hyperpara, d RTA, CAI (alkaline urine).
Uric acid stones (5% of all stones) are associated with high
purine metabolism, chronic diarrhoea, gout.
cystine stones associated with amino aciduria; a disorder of
proximal tubular cells. (COAL cystine, ornithine, arginine,
lysine) .
Proteus splits urea into ammonia, causing alkaline urine
struvite stones (magnesium ammonium phosphate).
Radiopaque stones are: Calcium oxalate, calcium phosphate,
triple phosphate, cystine stones.
Radiolucent stones are: Uric acid, xanthine stones.
comprehensive metabolic evaluation
serum calcium, bicarbonate, creatinine, chloride,
potassium, magnesium, phosphate, and uric acid.
24 hour urine collections for: volume, pH, calcium,
oxalate, citrate, uric acid, phosphate, sodium,
potassium, and creatinine.
Na nitroprusside test & 24 hour measurement of cystine
Intact PTH and 1,25 dihydroxycholecalcifirol in
hypercalcaemic patients.
Indicated for;
1. Patients with multiple stones at first presentation,
2. Patients with family history of urinary stones.
3. Patients with recurrent urinary stone.

Noncontrast helical Computed Tomography (CT)


91% sensitive and 98% specific in detecting urolithiasis
Beverage type risk

Coffee and tea Decreased

Alcohol Decreased

Milk Decreased

Lemon juice Decreased

Grapefruit juice Increased

Cranberry juice Increased


Carbonated
Increased
beverages
Cola Increased
Promotors & Inhibitors of stone formation
Dietary factor Proposed mechanism(s)

Promotors
Oxalate Increased urinary oxalate excretion
Sodium Increased urinary calcium excretion
Animal protein Increased urinary calcium and uric acid excretion; reduced urinary
citrate excretion
Vitamin C Increased oxalate generation and excretion
Inhibitors
Dietary calcium Binding of dietary oxalate in gut
Potassium Increased urinary citrate excretion; reduced urinary calcium excretion

Phytate Inhibition of calcium oxalate crystal formation


Magnesium Reduced dietary oxalate absorption; inhibition of calcium oxalate crystal
formation
Urinary citrate oppose crystal formation by thermodynamic and kinetic mechanisms

Vitamin B6 Vitamin B6 deficiency may increase oxalate production and oxaluria


Hyperoxaluria
urinary excretion of oxalate in excess of 45 mg/day.
Primary (Inherited) Hyperoxaluria.
Rare AR , excessive oxalate production and systemic deposition of
calcium oxalate tissue damage e.g. heart, bone, retina, kidneys
Nephrocalcinosis.
Secondary (Enteric) Hyperoxaluria.
- Reduced availability of free calcium to bind intestinal oxalate in malabs.
- The colon absorbs unbound oxalate.
- increased colonic permeability in IBD.
- Contributing factors include a low urinary citrate concentration,
decreased urine volumes, and a low urinary pH, all due to diarrhea and
consequent loss of fluid and bicarbonate in the stool.
NB; Renal stones are primarily composed of calcium oxalate when the ileum is
involved (e.g., ileocolonic Crohns disease), and uric acid when patients
have copious diarrhea or small bowel ostomies.
Retroperitoneal fibrosis
Definition; ureter embedded in dense
fibrous tissue
Causes;
1. Idiopathic (, 40th-50th)
2. Trauma, surgery, radiation.
3. Inflammation; infection, granuloma, autoimmune (sclerosing
cholangitis)
4. Neoplastic; lymphoma, Cx, bladder.
5. Drugs; methysergide, bromocriptine, ergotametrine ,MD,
hydralazine, BB.
C/P; insidious onset of dull aching pain.
IVP & U/Smedial indrawing of ureter at junction of middle
& lower part.
CT; periaortic mass.
ESR very high.
TTT; surgical releave of ureter, steroid for idiopathic type.
Acute Renal Failure
Def; Acute renal failure (ARF) is characterized by a rapid decline in glomerular
filtration rate (GFR) over hours to days.
1. Prerenal (60-70%)
- ESF volume; renal loss e.g,diuretics, extrarenal e.g. diarrhea, burns
- effective bl volume; COP , VD , oedematous states.
- intrarenal VC; cyclosporin, NSAIDs, ACEI, HRS.
2. Intrinsic renal (25-40%);
- ATN; ischemic, toxic e.g. G-ve sepsis/ Endogenous as rhabdomyolysis, hemolysis,
cast neph, tumour lysis / exogenous toxins; aminoglycosides, amphotericin,
cisplatinum,cyclosporin.
- tubular obstruction; Endogenous: myeloma proteins, uric acid (tumor lysis syndrome).
Exogenous: acyclovir, gancyclovir, methotrexate, indinavir, ethylene glycol.
- AIN.
- vascular; TMA e.g. DIC, malignant hypertension, preeclampsia.
- glomerular; acute nephritis.
3. Postrenal ARF (Obstruction)
I. Ureteric (bilateral, or unilateral in the case of one kidney): calculi, blood clots,
sloughed papillae, cancer, external compression (e.g., retroperitoneal fibrosis)
II. Bladder neck: neurogenic bladder, prostatic hypertrophy, calculi, blood clots.
III. Urethra: stricture or congenital valves
Prerenal ARF;
- High BUN/CR ratio>20.
- FENa < 1 %
-UNa <10 mmol/L
- high urine osmolarity > 500 mosm/l.
- SG >1.018.

Renal (Acute tubular necrosis);


- Muddy brown granular or tubular epithelial cell casts
- FENa > 1 %
- UNa > 20 mmol/L
- low urine osmolarity < 500 mosm/l.
- SG <1.015
Diagnosis cause serum urine ttt

Rhabdomyolysis Physical Increased U/A positive for Fluid therapy


(Myoglobinuria) Metabolic & myoglobin, heme but no forced alkaline
elect. CPK,creatinine, RBCs diuresis with
Drugs. P, K, uric acid, HCO3,
infections high AG MA, mannitol in
Ca. dextrose
BUN/creat<10
Hemolysis: Fever, Pink plasma, Pink, heme- As Myoglob.
recent blood other evidence Increased LDH positive urine
transfusion, of transfusion without
G6PD, PNH, reaction hematuria,
cold Ab hemosiderinuria
Tumor lysis recent Hyperuricemia, Urate crystals alkalinization
chemotherapy increased LDH urine uric of urine,
acid/u creat>1 allopurinol,
uricase,
fuboxostat
Hepatorenal syndrome
It is a functional impairement in kid functions 2ry to
intrarenal VC in LC with S. creat > 1.5 mg/dl or GFR < 40
ml/min in the absence of any other cause of kid
dysfunction.
Major criteria for diagnosis;
1. LCF with portal HTN.
2. S creat > 1.5 mg/dl or GFR < 40 ml/min
3. No improvement after stopping diuretics & fluid chalenge
of 1.5 L.
4. Absence of shock, infection, nephrotoxins.
5. Proteinuria <0.5 gm/dl, US normal, no obst or
parenchymal change.
Minor criteria;
1. S Na < 130 meq/l
2. Urine Na< 10 meq/l
3. Urine/P osm>1
4. Urine volume<1 l.
Contrast nephropathy
Non oliguric ATN acute rise of serum creatinine 24-48 hrs
after administration of IV contrast, peak = 3-5 days, baseline =
7-10 days.
Risk factors; DM, CKD, MM, ACEI, NSAIDS, prerenal failure,
high dose.
Pathophysiology; VC & tubular toxicity.
Prevention;
1. Use of low osmolality, non ionic contrast agent e.g.
gadopentate dimeglumine (ultravest).
2. Least dose.
3. IV infusion of NS 1-2 hrs before to 24 hrs after at a rate of 1
ml/kg/hr.
4. Acetylcysteine 600 mg sachet/12 hr 2 days before.
5. +/- theo 2 ds before, nefidipine 10 mg subluigual before.
6. # mannitol, frusemide, dopamine, ANP.
TTT; fluid chart, electrolytes, HD.
NB; Gadolinium in MRInephrogenic systemic fibrosis.
Chronic Kidney Disease
Classification of Chronic Kidney Disease (CKD)

Stage GFR, mL/min per 1.73 m2


1 > 90b
2 8960
3 5930
4 2915
5 < 15

bWith demonstrated kidney damage (e.g., persistent proteinuria,


abnormal urine sediment, abnormal blood and urine chemistry,
abnormal imaging studies).
Cockcroft-Gault equation; GFR e= (140-age) x BW/s. creat x 72 X
0.85 for women
Clinical and Laboratory Manifestations of CKD and Uremia

Fluid and electrolyte Neuromuscular disturbances Dermatologic disturbances


disturbances Fatigue, Sleep disorders Pallor, Hyperpigmentation
Volume expansion Headache, Impaired mentation Pruritus Ecchymoses
Na, K,P Lethargy, Asterixis Nephrogenic fibrosing
Muscular irritability PN, dermopathy ,Uremic frost
Endocrine-metabolic Myoclonus, Myopathy
disturbances Restless legs syndrome Gastrointestinal
Secondary PTH Seizures, Coma disturbances
Adynamic bone dis. Muscle cramps Anorexia, Nausea and
Vitamin Ddeficient Dialysis disequilibrium vomiting, Peptic ulcer
osteomalacia Gastrointestinal bleeding
Carbohydrate resistance Cardiovascular and Idiopathic ascites.
Hyperuricemia pulmonary disturbances
Hypertriglyceridemia. Arterial hypertension CHF Hematologic and
Decreased HDL Pericarditis,Hypertrophic or immunologic disturbances
Protein-energy malnutrition dilated cardiomyopathy , Anemia
Impaired growth Accelerated atherosclerosis Bleeding diathesis
Infertility and sexual dysfunction Hypotension and arrhythmias Increased susceptibility to
Amenorrhea Vascular calcification( infection
Leukopenia
Thromboathenia
Treatment
* Slowing the Progression of CKD
1. Protein Restriction, between 0.60 and 0.75 g/kg per
day.
2. Reducing Intraglomerular Hypertension and
Proteinuria; ACE inhibitors and ARBs, target blood
pressure in proteinuric CKD patients=125/75 mmHg.
3. Slowing Progression of Diabetic Renal Disease;
hemoglobin A1C should be < 7%.
4. Managing Other Complications of Chronic Kidney
Disease.
5. Medication Dose Adjustment.
6. Preparation for Renal Replacement Therapy.

* Renal replacement therapy.


Dialysis in the Treatment of Renal Failure

Hemodialysis
Hemodialysis relies on the principles of solute
diffusion across a semipermeable membrane.
Movement of metabolic waste products takes
place down a concentration gradient from the
circulation into the dialysate.
Complications during Hemodialysis;
1. Hypotension.
2. Muscle cramps.
3. Anaphylactoid reactions to the dialyzer.
4. Disequilibrium S.
Peritoneal Dialysis
In peritoneal dialysis, 1.53 L of a dextrose-
containing solution is infused into the peritoneal
cavity and allowed to dwell for a set period of
time, usually 24 h.
As with hemodialysis, toxic materials are
removed through a combination of ultrafiltration
and down a concentration gradient.
The major complications of peritoneal dialysis
are peritonitis, catheter-associated infections,
weight gain and other metabolic disturbances,
and residual uremia.
Anemia of CKD

Develops when the GFR < 60 mL/min, symptomatic


only when GFR<30 ml/min due to increase 2,3DPG &
LVH.
normocytic and normochromic.
due to;
1. reduced renal erythropoietin production (reduction in
functioning renal mass) and,
2. shortened red cell survival (60-90d vs. 120d).
3. Hemolysis, bl loss during HD.
4. Bleeding tendency & Fe deficiency.

Anemia may be a risk factor for progression of CKD.


TTT
1. Correct Fe deficiency
if ferritin <200 ng/ml & TSAT<20%.
Target ferritin 200-500 ng/ml & TSAT=20- 50%.
For predialysis oral 200mg elemental Fe/d or IV 200mg/1-3
months.
for HDIVI, 100mg for 10 sessions then /wk.
Side effects;
1. free Fe reaction; N,V,BP, back pain.
2. Anaphylaxis (Fe dextran due to anti-dextran Ab).
Contraindications;
1. Active inflammation.
2. Fe overload.
2) Erythropoietin
When Hb of < 11 g/dL .
Target Hb= 11 to 12 g/dL , NOT above 13 g/dL (adverse CVS
effects, increase risk for hypertension ) .
EPO , , Darbipoitin.
Dose= 80-120 U/Kg/wk SC, dose by 30-50% iv.
SC rather than IV ; stable level, more biologically active but PRCA
with EPO.
dose by 25% when target reached.
Benefit;
1. regression of left ventricular hypertrophy.
2. aerobic capacity,
3. cognitive and
4. sexual function.
Side effects
Hypertension
Headache
PRCA; due to neutralizing anti-erythropoietin antibodies
Causes of EPO resistance;
1. Iron deficiency (most common )
2. Bone disease due to 2ry hyperparathyroidism.
3. Occult malignancy
4. vitamin B12 and folic acid deficiency.
5. Multiple myeloma/myelofibrosis/myelodysplastic
syndrome.
6. Chronic inflammation.
7. Aluminum toxicity.
8. Hemoglobinopathies.
9. ACEI or ARBs.
10.pure red cell aplasia with neutralizing anti-erythropoietin
antibodies esp. with SC EPO .
11.HIV infection.
Renal Osteodystrophy
A systemic disorder of mineral and bone
metabolism due to CKD manifested by either
one or a combination of the following:
- Abnormalities of calcium, phosphorus, PTH,
or vitamin D metabolism
- Abnormalities in bone turnover,
mineralization, volume, linear growth, or
strength
-Vascular or other soft tissue calcification
Renal Osteodystrophy
A systemic disorder of mineral and bone metabolism due to CKD manifested by
either one or a combination of the following:
- Abnormalities of calcium, phosphorus, PTH, or vitamin D metabolism
- Abnormalities in bone turnover, mineralization, volume, linear growth, or strength
-Vascular or other soft tissue calcification
Pathogenesis;

PO4 Bone
Vit D PTH Normal pulsatile action+ osteoblast
Ca Continuous high
downregulate osteoblast receptors
unopposed osteoclastCa/P efflux
Classification;
1. High turnover disease (2 ry PTH > 300 pg/ml or osteitis fibrosa).
2. low turnover (adynamic) bone disease ( PTH<150 pg/ml ) due to Ca load=
overtreated.
3. Osteomalacia ( defective mineralization) due to Alm. Toxicity, Vit D, metabolic
acidosis, P.
4. Mixed uremic osteodystrophy; 2 ry PTH + Osteomalacia
C/P;
1. Bony aches, reccurent pathological fractures.
2. Soft tissue calcifications; vascular, valvular, skin.
calcifications in atherosclerotic plaques,
arteriosclerosis, calciphylaxis.
Diagnosis;
1. Intact PTH= full length PTH (1-84)(active) + PTH
fragment (7-84)( inactive accumulate in RF).
2. Bone specific ALK P (osteoblastic act).
3. Bone biopsy.
4. X rays.
5. DEXA.
6. Desferoxamine test for Alm tox.

N C
1 7 84
PTH
TTT
Best= sevelamer + Vit D analogue.
1. P;
- dietary restriction
- Ca based P binders; Ca carbonate, acetate.
- if Ca AL OH, sevelamer, lanthanum carbonate.
2. PTH;
- Vit D (calcitriol).
- Vit D analogues (1 , paricalcitol)
- calcimimetics (cinacalcet= CaSR agonist)
- PTH dectomy if;
PTH > 800 + Ca or P despite medical TTT.
calciphylaxis
severe sympt.
3. Transplantation.
This is a patient with end stage renal disease, with
severe uncontrolled hyperparathyroidism. What is
the skin lesion

Slide no 9
Transplantation
Tissue Typing;
1. ABO(O) blood groups
2. human leukocyte antigen (HLA) class I
(A, B, C) or class II (DR) antigens
3. Punnel of reactive antibodies (PRA)
4. cross-match of recipient serum with
donor T lymphocytes
Hyperacute Accelerated Acute Chronic
acute

onset immediate 5 days 5 d-4 m > 4m

Mechanism Humoral cellular & cellular cellular &


humoral humoral
Effector preformed Ab memory T T cytotoxic T helper & B
cells lymphocytes
Pathology Intravascular Interstitial Interstitial Chronic
thrombosis tissue tissue interstitial
mononuclear mononuclear nephritis &
& neutrophyl infiltration fibrosis
infiltration &
vasculitis
Immunosuppressive Treatment
1. Induction therapy with Antibodies to
Lymphocytes;
Depleting Ab e.g.
- ATGAM, OKT3
- Thymoglobulin is the most common
agent currently in use.
- Alemtuzumab
non- Depleting Ab e.g.
- anti CD25 (IL2 receptors); Basiliximab &
Daclizumab.
- Belatacept (costimulatory pathway blockade)
2. Maintenance Immunosuppressive Drugs
Agent Mechanisms Side Effects
Glucocorticoids Binds heat shock proteins. Hypertension, glucose intolerance,
Blocks transcription of IL-1,-2,-3,- dyslipidemia, osteoporosis
6, TNF and IFN

Cyclosporine CsA calcineurin --- block IL-2 Nephrotoxicity, HTN, dyslipidemia,


production; however, stimulates glucose intolerance, hirsutism/
TGF production hyperplasia of gums

Tacrolimus (FK506) Similar to CsA, but


hirsutism/hyperplasia of gums
unusual, and diabetes more likely
Azathioprine inhibit purine synthesis Marrow suppression (WBC > RBC
> platelets)
Mycophenolate Inhibits purine synthesis Diarrhea/cramps; dose-related
mofetil (MMF) liver and marrow suppression is
uncommon
Sirolimus blocks p70 S6 kinase in the IL-2 Hyperlipidemia, thrombocytopenia
receptor pathway for proliferation
The Most Common Opportunistic Infections in the Renal Transplant Recipient

Peritransplant (<1 month) Late (>6 months)


Wound infections Aspergillus
Herpes virus Nocardia
Oral candidiasis BK virus (polyoma)
Urinary tract infection Herpes zoster
Hepatitis B
Early (16 months) Hepatitis C
Pneumocystis carinii
Cytomegalovirus
Legionella
Listeria
Hepatitis B
Hepatitis C
BK virus
BK virus nephropathy & ureteral stenosis.
Urothelial carcinoma,vasculopathy.
Biopsy; patchy interstitial infiltration, IF; Ab to
simian v. 40.
Urine cytology +ve for decoy cells(tubular cells
appear malignant due to viral inclusions.
PCR.
TTT; reduce IS, leflunamide, cidofovir.
Malignancy
The incidence of tumors in patients on
immunosuppressive therapy is 56%, or approximately
100 times greater than that in the general population of
the same age range.
The most common lesions are cancer of the skin and lips
and carcinoma in situ of the cervix, as well as
lymphomas such as non-Hodgkin's lymphoma.
The risks are increased in proportion to the total
immunosuppressive load administered and time elapsed
since transplantation.
Surveillance for skin and cervical cancers is necessary.
Slide no 2
This skin lesion
appeared 2 months
after a successful
renal transplant
What is the most
likely diagnosis
Rate of relapse & graft loss

Glomerular disease recurrence graft loss


FSGS 15% 50%
Membranous 10% 50%
Ig A 50% 15%
MPGN 50%
Alport anti-GBM
Pregnancy related kidney diseases
ARF in pregnancy;
1st trimester; hyperemesis, septic abortion.
2nd trimester; TTP (any time).
3rd trimester; acute fatty liver, HELLP, post renal dt
enlarged fibroid, stones, Gravid uterus,
Polyhydramnios.
Post partum; HUS, bil cortical necrosis.
TTP HELLP Acute fatty HUS Cortical
liver necosis
timing 2nd, 3rd trimester 3rd trimester 3rd trimester postpartum postpartum
(more common) Hge e.g.
abruptio
placentae
C/p Fever -Mild renal -ANV abd pain -Severe Oliguria
neuro failure -Hepatic renal failure Hematuria
-Proteinuria encephalopathy - Poor renal Flank pain
-HTN - hypoglycemia prognosis
-Jaundice
- DIC
-Severe renal
failure
Lab -plat, Hb -liver enz -liver enz Hypo or
-uncong. bilirubin -uric acid -PT, PTT, plat hyper echoic
-shistocytes - -fibrinogen areas in US
Hypocalcuria -congugated bil
-PT, PTT, - hypoglycemia
plat

ttt plasmapharesis termination termination Supportive dialysis


dialysis
Hypertensive disorders in pregnancy;
1.Pre-eclampsia (27%); de novo, after 20th week
gestation, resolve within 3 months + proteinuria.
2.Chronic HTN (23%); before pregnancy, before
20th week gestation, does not resolve post partum
(essential 19% or 2ry 4%)
3.Pre-eclampsia superimposed upon underlying
HTN(7%)
4.Gestational HTN (43%); mild, de novo, after 20th
week gestation, resolve within 3 months without
maternal organ dysfunction.
NB; preeclampsiaafter 20 wks, liver
enzymes, normal Complement.
Lupus activity complement.
Neurology
Dementia
Definition; acquired progressive loss of cognitive
function.
Most common cause is Alzheimers dis., then lewy
body 20%, multi-infarct 20%.
Classification;
1. Ant. Dementia;
fronto temporal lobe lesion,
antisocial personality & behavior.
e.g. pick, metabolic.
2. Post. Dementia;
Parietotemporal.
Memory, conc, orientation
e.g. Alzheimers.
3. Subcortical;
lewy body, multi-infarct, Parkinson, PSP, normal
pressure hydrocephalus, Huntington, wilson, MS, HIV.
Pick disease;
atrophy of frontal & or temporal lobe, pick bodies,
language, behavioral, frontal lobe features.
Alzheimers dis;

20% familial (AD),


loss of recent memory,,

Histopath. senile plaques, neurofibrillary tangles,


Biochemical. loss of cholinergic neurons.

mutation of presenilin 2 on chro 1,

presenilin 1 on chro 14 .
Associated with Down S, .

TTT; Donepezil (cholinesterase inhibitor).


if aggressive valproate.
if severe; stop Donpezil & give Melatonine
Lewy body Dementia;
Intermittent dementia within 2 month of
Parkinsonism
Visual hallusinations, paranoid delusions
Marked sensitivity to neuroleptics (may be fatal, so
antipsychotic are #).
Intranuclear inclusions in the cortex.

Atherosclerotic= Vascular =Multiinfarct;


Stepwise fashion.
Marked rigidity, shuffling gait & pseudobulbar palsy
+ risk factors
Normal pressure hydrocephalus;
Dementia + gait abnormalities (wide based) + urinary
incontinence (unaware, indifferent).
No headache, no papilledema.
Defect in CSF absorption, may be post meningitic or
trauma.
CT= hydrocephalus, no cerebral atrophy..
CSF= normal pressure (intermittently high).
TTT; ventriculoperitoneal shunt.
Parkinson plus syndromes;
1. Progressive supranuclear palsy; (PSP)
Parkinson + ophthalmoplegia (vertical gaze defect) + freq.
falls Dementia (late), pseudobulbar palsy.
2. Multiple system atrophy;
Parkinson + autonomic distur (aware) + cerebellar & .
Early age (50 y), median survival=6-9 yrs.
Ttt; dopaminomimetic, good response but lead to
orthostatic hypotension.
Huntington Chorea;
- AD, Chr 4 , CAG trinucleotide repeat.
Anticipation refers to the increase in severity with
subsequent generations.
- Ach, GABA in caudate.
- in contrast to parkinson; dopamine is normal.
- age; 30-40 yrs.- triad; Chorea + dementia +
(antisocial behavior) +opthalmoplegia. + FH
- genetic testing.
- no specific ttt.
phenothiazines (haloperidol) for symptomatic control.
( haloperidol)
Progressive multifocal leukoencephalopathy
(PML) is caused by chronic infection with JC virus,
and causes white matter lesions in the brain. A
CD4+ of <100 predisposes to the condition .

JC virus invades oligodendrocytes, which


manufacture myelin, causing demyelination.

Hemiparesis, aphasia, ataxia and altered mental


state with an insidious onset of dementia.

Anti-retroviral therapy including a protease inhibitor


is the main treatment for PML with HIV.
Progressive Multifocal leuckoencephalopathy;
P M L esions
CSF protein Male white matter
PCR memory hypodense
Papova virus (JC) mood areas esp.
motor occip cortex
MRI
Chronic Subdural haemorrhage
Bleeding is from bridging veins between cortex and venous
sinuses, resulting in accumulation of blood between dura and
arachnoid.
Risk factors age, alcohol, fallstrauma (may be trivial).
epilepsy, anticoagulant therapy.
Presentation
Often insidious and history may go back several weeks:
Fluctuation of conscious level (35%).
Sleepiness ,Headache ,Personality change.
Unsteadiness on feet
Slowly evolving hemiparesis.
Sympoms/signs of inc.ICP.
dementia.
Differential diagnosis Stroke , cerebral tumour,
TTT: If suspected, admit as a medical emergency for further
investigation. Evacuation of clot is possible even in very
elderly patients and results in full recovery.
Creutzfeldt-Jacob dis;
Def; prion dis, caused by abn prion protein which accumulate in the brain
which is thought to be an infectious agent, resistant to heat& autoclaving.
C/P; vague sensory manifestations. then Dementia, ataxia, myoclonic jerks.
CSF; normal, mild in protein.
Normal CT brain. Dis duration 1/2 yr -2 yrs
EEG; biphasic high-amplitude Sharpe waves.
Most common is sporadic (85%), other forms are familial, new variant.

new variant (BSE) sporadic

age Young (30 yr) Old (60 yr)

C/P Ataxia cerebellar signs The same.


(motor, ) sensory , occipital
blindness,Psyhatric dementia
Personality, behaviour,
ECG Not specific specific; periodic biphasic
Sharpe waves in deep brain
e.g. thalamus

MRI high signal in T2. bilateral High signal in T2 in putamen


& caudate
posterior thalamic ( pulvinar)
Prion diseases
Variant (20-30 yrs,)BSE
From introduction of infected brain tissue to another hostthe abnormal
prion protein trigger the conversion of the normal protein into the
atypical isoform accumulation of amyloid plaques spongiform
change of the brain.
Transmission;
1. Dural or corneal grafts, neurosurgical
instruments, human derived hormones as GH.
2. Food from infected cattle with bovine
spongiform encephalopathy.
Pathology; gliosis, small vaculation, plaques
containing protease resistant PrP .
Differentiation of depression
from dementia
depression dementia

Onset,course Acute, rapid, short Gradual, slow, long


duration
C/O of memory loss yes No
O/E Detailed history giving, Vague history
poor effort at testing, Good
pick on faults, Pleased
global memory loss, Recent

+ve sympt of depression


Transient global amnesia;
-sudden confusion & amnesia.
-Antegrade & retrograde amnesia for recent up to
years memories.
-Self identification is preserved.
-dt transient vascular insuff of both hippocampi.
Demylenating diseases
1. MS.
2. Diffuse sclerosis.
3. ADEM.
4. Neuromylitis optica.
5. Central pontine mylenolysis.
Multiple sclerosis
Def; a cell-mediated autoimmune dis against myelin.
Subtypes;
1. Relapsing/ remitting; (80-85%), acute attacks for 4-8 wks, relapse
rate= 0.8/y.
2. 2 ry progressive; after 10 y of Relapsing/ remitting, progressive with
less prominent relapses.
3. 1ry progressive; (10-20%).
Aetiology unknown.
C/P; mainly UMNL, LMNL is rare dt rare affection of ant horn cells.
Optic neuritis;
- in isolated ON 40-60% MS.
- painful loss of vision.
- early affected colour vision.
- ttt; pulse Methyl Prednisolone.
Good prognostic factors;
1. Young.
2. Female
3. Relapsing/ remitting.
4. Sensory.
Diagnosis; disseminated lesions in time & place in brain
& SC
.MRI; demyelinated plaques, gadolinium-enhancing lesions,
affection of corpus callosum is pathognomonic.
if free relapse rate is 16%, if >3 plaques, relapse rate is
50%.
.Delayed VEP.
.CSF; oligoclonal bands in 50%.
DD of oligoclonal bands in CSF=IG synthesis in the CSF;
Neurosarcoid, SLE, CNS lymphoma ,ADEM , GBS ,Subarachnoid Hge,
Internuclear opthalmoplegia: The medial
longitudinal fasciculus (MLF) connects the sixth
nerve to the third & fourth nerves of opposite site.
There is nystagmus in the abducting eye
(mnemonic NAB) and limited movement in the
adducting eye on the same side. Upward gaze is
not affected.
Internuclear opthalmoplegia can be caused by:
Multiple Sclerosis ,stroke (basilar artery) post circ.
Encephalitis, lyme disease, phenothiazines, TCA.
TTT of MS;
1. During acute attack pulse steroid for 3-5 ds.
2. To prevent relapses INF.
SC
prevent activated T lymphocytes from crossing BBB.
# in pregnancy.
criteria for INF therapy; &Glatiramer
- Relapsing/ remitting or 2 ry progressive
- 2 disabling relapses in the previous 2 yrs.
- - able to walk at least 100 m (+/-assistance)
- Effects; frequency, severity of relapses, delay progression to disability,.
brain lesions on MRI.
Side effects; flu-like, teratogenic, neutralizing Ab.
3. Glatiramer.
4. IVIG.
5. Symtomatic;
- fatigueamantadine, methylphenidate.
- bladder dysfunction anticholinergic, catheter.
- spacticity baclofen, dantrolene, BDZ.
- neuralgic pain carbamazipine, gabapentine.
End stage of MS; dementia, quadriplegia,
blindness, recurrent respiratory & UTI.
Life expectancy = 20-30 yrs.

Death dt; renal failure, bronchopneumonia.

NB; baclofen;
- site of action is the spinal cord.
- -- synaptic transmission.
Diffuse sclerosis; severe form of MS.
ADEM (acute disseminated encephalomylitis);
- autoimmune,
- 2 wks after viral (EB,CMV, exanthems, mumps, influenza,)
- mycoplasma
- vaccine (rabies)
- monophasic i.e. no relapses.
- disseminated neurological symp, rare sensory.
- MRI; white matter lesions, CSF; oligoclonal bands.
- ttt; short course of high dose steroid, plasmapharesis.
Neuromylitis optica
- Transverse myelitis + optic neuritis.
- 1ry or 2ry to SLE
- ADEM
- TB.
- Recovery is poor
- Monophasic or relapsing.
Central Pontine Myelinolysis
Due to rapid correction of Hyponatremia
Possibly due to disturbance of blood brain barrier and damage by
cytokines
Causes;
- rapid correction of Na.
- Alcohol, Liver cirrhosis.
-Malnutrition, Malignancy, hyperemesis gravidarum.

MRI shows change in pons, but not diagnostic.


Fatal, No effective therapy.
Central pontine mylenolysis
C/P;
Pons -, quadriplagia.
Pontocerebellar cerebellar.
Vasomotorhypotension
Consciousness is usually impaired.
Presentation

Dysarthria-Dysphagia ,nuclei 4,5,6;


Pseudobulbar
Paraparesis or quadriparesis
Lethargy and coma
Seizures-Nystagmus
Ataxia
emotional lability-akinetic mutism
gait disturbance -myoclonus
Behavioral disturbances-
Fatal, no ttt.
Dysmylenating diseases
= leucodystrophies
(reading only
Adrenoleukodystrophy Metachromatic leukodystrophy

X-linked AR
MR, OA, paraparesis MR, OA, paraparesis
Adrenal insufficiency PN as HMSN type 1
- Confirmed by serum VLCFA --urinary sulphatides
(very long chain FA) biopsy; metachromatic inclusions
Ttt; BMT Ttt; BMT
Causes of polyneuropathy
Inherited Refsum's syndrome,Charcot-Marie-Tooth syndrome (mostly
motor),Porphyria
Inflammatory Guillain-Barr syndrome (mostly motor),Chronic
inflammatory demyelinating polyneuropathy (CIDP)
Metabolic DM (mainly sensory),Renal failure (mainly sensory),
Hypothyroidism
Vasculitis PAN ,Rheumatoid arthritis,Wegener's granulomatosis.
Malignancy Paraneoplastic syndromes (esp small cell lung cancer)
Paraproteinaemias e.g. multiple myeloma, amyloidosis
Infection Leprosy (mainly sensory) Syphilis,Lyme disease,HIV.
Vitamin deficiency Lack of B1, B6, B12 (e.g. alcoholic)
Toxins Lead (mostly motor),Arsenic
Drugs Alcohol,Cisplatin,Isoniazid,Vincristine,Nitrofurantoin.
Mononeuropathy
Lesions of individual peripheral (including cranial) nerves
.Causes : trauma, compression, DM, lepros, HZV,PAN, DM .
Common mononeuropathies
Median nerve C5-T1 Inability to flex the terminal phalanx of the thumb and loss of
sensation over lateral 3 fingers and palm. Wasting of the thenar eminence .Common
causes :trauma (especially wrist lacerations), carpal tunnel syndrome
Ulnar nerve C8-T1 Weakness and wasting of interossei muscles (weakness of
abduction of fingers) and wasting of hypothenar eminence, sensory loss over medial 1
fingers and ulnar side of the hand. Flexion of 4th and 5th fingers is weak .Common
causes :trauma or compression at the elbow, trauma at the wrist.
Radial nerve C5-T1 Sensory loss is variable but always includes the dorsal aspect of
the root of the thumb. This nerve opens the fist .Common causes :compression against
the humerus, trauma.
Sciatic nerve L4-S2 Weakness of hamstrings and all muscles below the knee (foot
drop), loss of sensation below the knee laterally .Common causes :back injury, pelvic
tumours.
Common peroneal nerve L4-S2 Inability to dorsiflex the foot (foot drop), evert the foot,
extend the toes. Sensory loss over dorsum of the foot .Common causes :trauma.
Tibial nerve S1-3 Inability to stand on tiptoe, invert the foot, or flex toes. Sensory loss
over sole.
Carpal tunnel syndrome
Pain in the radial 3 digits of the hand numbness, pins
and needles, and thenar wasting. Due to compression of
the median nerve as it passes under the flexor retinaculum.
Worse at night. Symptoms are improved by shaking the
wrist.Associations: Pregnancy , hypothyroidism, obesity
,acromegaly , amyloidosis and Rh arthritis.
Investigations.
Phalen's test. hyperflexion of wrist for 1min. triggers
symptoms
Tinnel's test. tapping over the carpal tunnel causes
paraesthesiae
Nerve conduction studies.
Management.
steroid injection, US waves , Laser therapy.
surgery to divide the flexor retinaculum is curative in
mild/moderate disease.
Causes of mononeuritis multiplex
DM.
Amyloidosis.
Sarcoidosis.
Leprosy.
Vasculitis. ( PAN).
Carcinomatosis.
C/P of polyneuropathy
Motor :
LMNL( hypotonia ,hyporeflexia , marked wasting )
Weakness Distal > proximal , LL>UL , Bilateral &
symmetrical,
Foot & Wrist drop .high steppage gait.
Sensory :
Superficial:Glove&stock hyposthesia& parasthesia
Deep :Sensory ataxia(dec sense of position , sense
of movement, sense of vibration)
Autonomic: skin , GIT,GU .
Management of Autonomic neuropathy
Treat any underlying cause.
Advice:
stand slowly
raise head of the bed at night
eat little and often
avoid alcohol intake.
Fludrocortisone (100mg/d., increasing prn) may
help those most severely affected.
Mainly Motor Polyneuropathy
GB$.
Diphteria .
Porpheria.
Lead poisoning.
CIDP.
Mainly Sensory Polyneuropathy
DM.
Uremic N.
Amyloidosis.
Vitamin deficiency.
Carcinomatosis.
Leprosy.
Acute polyneuropathy
GB$.
Lead poisoning .
Alcohol toxicity.

Polyneuropathy with bilateral facial paralysis.


Sarcoidosis.
GB$.
Bilateral Bell s palsy.
Bilateral acoustic neuroma.
Lyme disease.

Polyneuropathy with proximal weakness.


Diabetic amytrophy (weakness,wasting & pain of thigh muscles & pelvic
girdles.) for few months, no effective ttt
GB$
CIDP.
Guillain Barre syndrome
is acute post-infectious polyneuropathy preceded by
diarrhoea (e.g. campylobacter) and chest infections (e.g.
mycoplasma) in two thirds of cases.
Demyelination & mainly motor.
Weakness begins in the legs and ascends to involve the
trunk, upper limbs and finally the bulbar muscles.
Symmetrical involvement being typical & asymmetry is
present in only 9% of patients.
Autonomic dysfunction and hyporeflexia are associated.
Bulbar involvement occurs in 50%,
Studies have shown that plasma exchange and IVIg
are equally effective in treating people within two to four
weeks of onset of GBS
The dissociation between a high CSF protein and a lack
of cellular response with normal glucose in a person with
an acute or subacute polyneuropathy is diagnostic of
Guillain Barr Syndrome
Miller Fisher Syndrome
In the, Miller Fisher Syndrome a variant of GBS
there is external ophthalmoplegia, ataxia and
areflexia. Clinical symptoms usually improve
within 2-3 weeks.
It usually follows antecedent infections.
Serum IgG antibody to the ganglioside GQib is
present in more than 95% of patients. It is highly
specific for the syndrome.
Pellagra: A triad of dementia, diarrhoea and dermatitis. Niacin
(nicotinamide or nicotinic acid) deficiency causes pellagra only if
tryptophan, an amino acid, is also deficient. Encephalopathy usually
follows. It is ccc by confusion, disorientation,, and memory loss.
Pyramidal tract.
Peripheral nerve.
Subacute Combined Degeneration of the cord: Vitamin
B12 deficiency causes PN, posterior column cord damage (Loss of
vibration sense and proprioception), spasticity, extensor plantars and
neuropsychiatric symptoms.
Pyramidal tract.
Peripheral nerve.
posterior column.
HMSN 1. is the most common form of hereditary
neuropathy. Severely and uniformly slowed (NCVs) and
primary hypertrophic myelin pathology.
Motor symptoms predominate over sensory symptoms.
Often, patients report loss of balance, muscle weakness,
and foot deformities. Onset in the first decade of life.
HMSN 2. on the other hand, represents the
nondemyelinating neuronal type with relatively normal
NCVs and primary axonal pathology. weakness often is
less marked and onset of this neuropathy is delayed.
Peripheral nerves are not enlarged.
Patients experience sensory loss in the distal
extremities, and foot deformities (ie, pes cavus) .
Neurofibromatosis.
NF1 (NF gene 1 defect) is found on chromosome 17
Lisch nodules (pigmented spots) of the iris are present in more than 90% of
patients with neurofibromatosis type 1. The diagnosis is suggested by six
or more caf au lait spots. Although the condition is autosomal dominant,
almost half of all cases are new mutations.
NF2 gene defect is on chromosome 22. Bilateral acoustic neuromas are
a hallmark feature of neurofibromatosis type 2

Tuberous sclerosis. This is an inherited (autosomaLD)


hamartomatous condition facial angiomas (adenoma sebaceum) skin
changes such as shagreen patches and ashleaf macules, subungual
fibromas ,kidney angiomyolipomas, cardiac rhabdomyomas,,.

von Hippel Lindau. disease is associated with cerebellar


haemiangioblastomas and retinal angiomas and polycystic liver or kidneys.
Ectopic erythropoietin secretion by the haemangioblastomas.RCC
Slide no 6
This female patient sufferred convuslions in childhood.
She experienced several attacks of hematuria recenly.
Proteinuria was less thn 1gm/24hrs and her RFTs
were normal.
1. What is her condition.
2. What is the most likely lesion of her CT scan
Myasthenia gravis
It is a neuromuscular disorder and therefore does not cause any lower
motor neuron signs such as fasciculations, wasting, and loss of
reflexes. Pupils are always normal. diurnal variation.
Myasthenia gravis is well known to be associated with other autoimmune
disease such as pernicious anaemia, thyroid disease and rheumatoid arth.
is commoner in females (it is an autoimmune disease). The commonest
features include ptosis, diplopia and ophthalmoplegia.
Electromyography. Repetitive stimulation in myasthenia gravis leads to a
decrement of evoked muscle action potentials
in myasthenia gravis the tendon reflexes are characteristically normal.
The list of drugs which may make myasthenia worse are: propranolol
Aminoglycosides, Quinidine, Procainamide, Magnesium, Lithium .
In myasthenia gravis, there are positive anti-Ach antibodies.
MYASTHENIA

Pyridostigmine is an anticholinesterase which reduces


acetylcholine breakdown and hence improve symptoms
of fatiguability in myasthenia gravis.
Thymectomy improves the prognosis in patients below
40 years and in those with positive receptor antibodies.
Following thymectomy, 60 % of patients will improve.
Thymectomy is usually recommended because of the
risk of malignancy.
In Lambert Eaton myasthenic syndrome, 60%
of cases are paraneoplastic (small cell lung ca is
most associated 50% of the cases ). The clinical
features are proximal weakness, loss of tendon
reflexes and autonomic dysfunction is common
(e.g. dry mouth).. reflexes normalize with repetitive
muscle contraction
Ptosis and ophthalmoplegia are rare.
the condition improves by repetitive stimulation.
It results when IgG autoantibodies blockade the
voltage-gated calcium channels of peripheral
cholinergic nerves.
DD of purely motor disease
Myopathy.
Myasthenia graves.
Motor neuron disease.
Extrapyramydal.

DD of mixed UMNL & LMNL.


MS.
MND.( Motor neuron disease).
Cervical cord compression (mylopathy).
Causes of Myopathies
Inherited : Duchenne's muscular dystrophy Becker's muscular dystrophy
Facioscapulohumeral dystrophy
Inflammatory : Polymyositis & dermatomyositis.
Chronic inflammatory demyelinating polyneuropathy (CIDP).
Metabolic : DM ,Renal failure, hypoK , hyperk .
Endocrinal : hypothyroidism ,hyperthyroidism , cushing ,addison dis.
acromegaly.
Vasculitis: Rheumatoid arthritis,Wegener's granulomatosis
Malignancy : Paraneoplastic syndromes (especially small cell lung
cancer)
Infection: coxsachi virus.
Vitamin deficiency: Lack of vit. D.
Toxins: Alcohol
Drugs Alcohol, Cortisone, penecillamine, thiazides, carbexenolone.
Lovastatin , Cocaine, Colchicine.
Classification
1)Muscular 2)Channelopathies 3)Congenital 4)Metabolic
dystrophies 1)Cl -->Myotonias Glycogen storage
1-X-linked 2)Na-->Hyperkalemic PP Acid maltase
Duchenne 3)Ca-->hypokalemic PP MP
Baker Lipid storage
2-Autosomal CPT
Facioscapulohumeral Mitochondrial
Limb girdle Kearn Sayre
Myotonia dystrophica MERRF
MELAS

5)Endocrinal Toxic Inflammatory


Addison disease, Alcohol (acute or chronic) Polymyositis
licorice
Cushing disease Drugs Inclusion body
Hypothyroidism Steroids Sarcoidosis
(CK may be mildly Lovastatin connective tissue
elevated) Infectious
Cocaine
Hyperthyroidism Trichinosis Coxsackie A and B
Colchicine
(CK may be normal) Cysticercosis (T solium) HIV
Toxoplasmosis Influenza
Hyperparathyroidism
Lyme disease
acromegaly Staphylococcus aureus
C/P of Myopathies
Motor :
LMNL( hypotonia ,hyporeflexia , wasting )
Weakness proximal > Distal , Bilateral &
symmetrical , Waddling gait .
NO Sensory affection:
NO autonomic affection.
NO fasiculations.
NO cranial nerve.
NO sphencteric affection.
Investig.
CPK, EMG, Ms biopsy.
MUSCLE DISEASES
Mutation in the dystrophin gene causes deficiency of dystrophin in
Duchenne's muscular dystrophy. XL R Patients develop progressive
upper and lower limb weakness with pseudohypertrophy of calves and
quadriceps.
Becker's muscular dystrophy. XL R Later presentation & milder form.
Facioscapulohumeral dystrophy (FSHD) is one of the most common
types of muscular dystrophy. It is of autosomal dominant inheritance.
Onset is usually age 20 years. Initial weakness is seen in facial muscles,
starting in the orbicularis oculi, orbicularis oris, and zygomaticus. Shoulder
weakness is the presenting symptom in more than 82% of patients.
Winging of the scapula is the most characteristic sign. Creatine kinase
levels are raised.
Mitochondrial diseases
All mitochondria of the fetus are derived
from the ovum (Maternally Derived).
Mutations in mitochondria accumulate in
cells and can be transferred to the offsprings
Examples :
MELAS : myopathy, encephalopathy, lactic acidosis &
stroke like episodes. Lactic acidosis may be provoked by
exercise & induce nausea, Vomiting,& coma. Present in
early childhood
MERRF: Present in adults, with proximal muscle
weakness ataxia & seizures
Kearn Sayre $: present in childhood with ptosis,
ophthalmoplegia, retinitis pigmentosa, epilepsy, cardiac
conduction defect,& proximal weakness. DM, hypothy.
MITOCHONDRIAL MYOPATHIES
Chronic progressive external ophthalmoplegia (CPEO) is a disorder
characterized by slowly progressive paralysis of the extraocular muscles.
Patients usually experience bilateral, symmetrical, progressive ptosis,
followed by ophthalmoparesis months to years later.
Leber's Hereditary Optic Neuropathy is a rare condition cause loss of
central vision. It usually affects men in the late 20s or early 30s,.
MELAS (myopathy, encephalopathy, lactic acidosis and stroke like
episodes). Lactic acidosis is a very important feature of this disorder, as
measured by a high lactate to pyruvate ratio. Patients have a proximal
myopathy, seizures and stroke like episodes.
MERRF (myoclonic epilepsy, ragged red fibres). The most characteristic
symptom of MERRF syndrome is myoclonic seizures that are usually
sudden, brief, jerking, spasms that can affect the limbs or the entire
body. Ataxia as well As Dysarthria, optic atrophy, short stature, hearing
loss, dementia and nystagmus may also occur.
Acid maltase deficiency
Mc cardle disease.
Acid maltase deficiency typically presents with
insidious onset of proximal myopathy and early
respiratory muscle weakness. Respiratory failure.
Inclusion body myositis:
Old male > 50 yrs.
In this condition, dysphagia and respiratory
involvement can also occur.
Muscle biopsy shows intracellular inclusions
(amyloid precursor protein, ubiquitins) and
inflammatory infiltrates.
Clinical suspicion for -IBM should be very high
when the pattern of weakness affects both P& D.

inclusion body myositis is the commonest in >50 years old


Neuroleptic malignant syndrome
is the most likely diagnosis. Its major features
are: rigidity, altered mental state, autonomic
dysfunction, fever, and high creatinine kinase.
It is usually caused by potent neuroleptics.
The treatment of choice is dantrolene and
bromocriptine.
Withdrawal of neuroleptic is mandatory.
Rhabdomyolysis and acute renal failure are
potential complications.
Seizure
A Seizure= paroxysmal event dt abnormal excessive
discharge from an aggregate of CNS neurons,
prevalence 5% in the population.
Epilepsy= a clinical syndrome of recurrent Seizures
(2 or more unprovoked Seizure) dt a chronic
underlying pathology, prev=0.5%.

NB; risk of recurrence of 1st seizure = 35% at 2 yrs, so


do not give antiepileptics except for high risk
patients ( neurologic insult, abn EEG).
Causes of seizures
1. Idiopathic.
2. Traumatic.
3. Infection.
4. Br tumor.
5. Alcohol withdrawal.
6. Illicit drug use.
7. Metabolic; uremia, LCF, electrolytes, hypoglycemia,
hypoxia).
8. Drugs; Alkylating (chlorambucil, busulfan),
immunomodulators ( cyclosporin, Tacrolimus, OKT3, ),
antimalarial , antibiotics ( B lactam, quinolones, INH,
acyclovoir, gancyclovoir), antidepressant, antipsychotic,
withdrawal,(alcohol, barb, BDZ), abuse (Amphetamine,
cocaine), radiocontrast, theophylline, flumazenil.
DD;
- TIA.
- Syncope.
- Sleep disorders (narcolepsy/ cataplexy).
- Movement disorders (tics, myoclonus,
choreoathetosis).
Types of epilepsy
Partial Seizure=
within discrete area of the brain.
A) simple =fully conscious.
B) complex = impaired consciousness.
Generalized =
from both cerebral hemisphere simultaneously without detectable
focal onset.
1) Absence s = lapses of consciousness without loss of posture
control, no post ectal confusion.
2) Tonic clonic.
3) Myoclonic.
4) Atonic.
5) partial with 2 ry generalization.
Generalized Epilepsy
Absence seizures;
- Infant , child.
- Blinking of eye, myoclonic jerks of arms
- Never acquired.
- May develop generalized S in adult.
Myoclonic. (jevenile epilepsy);
- Commonest of idiopathic Generalized.
- Chr 6.
- Occur within 1 hr of waking up.
- Ppting factors; alcohol, menses, sleep deprivation.
- Ttt : Na valproate, lamotrigine, topiramate.
- Life long ttt to prevent relapses.
Atonic = Akinetic = sudden loss of tone.

NB: status epilepticus= fit > 30 min or recurrent without regaining consciousness in
bet.
Partial Seizure
Simple ( e.g. motor, sensory, Autonomic, psychic)
- Followed by todds paralysis
- If in the frontal lobe conjugate deviation of eye to
other side.
Complex;
- typically medial temporal,
- Aura (e.g. dj vu, smell, abd sens)
- Autonomic, psychic (dj vu), motor (grimacing),
sensory (vertigo)
- Followed by loss of consc.
Antiepileptic drugs
1. Phenytoin (all except absence).
2. Valproate (generalized).
3. Carbamazepine (partial).
4. Lamotrigine, Topiramate (generalized)
5. Gabapentine (partial)
6. Clonazepam (Myoclonic).
7. Ethusuximide ( Absence).
8. Vigabatrine, levitiraceticam.
Tonic clonic Phenytoin, Valproate, Lamotrigine, Topiramate.
Absence Valproate, Ethusuximide.
Myoclonic Valproate, Clonazepam.
Partial Carbamazepine, Gabapentine

NB; in Myoclonic, Valproate is most effective but use Clonazepam,


Lamotrigine, Topiramate better for fear of hepatotoxicity.
- Begin with low dose.
- Try monotherapy, if failed another mono, if failed
combination.
Phenytoin Valproate Carbamazepine

uses Tonic clonic All generalized -Partial

all except -5thneuralgia


absence -Diab neurop

-Migraine

-DI

Drug Enz inducer Not Enz inducer


interaction level of; phenobarbital If uses with phenytoin
-OAC metab dose of phenytoin
-ACP

--cyclosporine

-Other antiepil
Phenytoin Valproate Carbamazepine
Side effects
CNS depression
hepatotoxic Hepatitis, Fatty, Reye S
granuloma
GIT upset , pancreatitis
Blood Megaloblastic an plat pancytopenias
LN, gum thrombathenia
skin Lupus, Hirsutism Gynecomastia Toxic epidermal necrolysis
Serum sickness Rash, alopecia
endocrine DM, DI Wt gain, PCO SIADH
hypercholestrolemia
Bone
(osteomalacia)
Neuro PN, enceph Ataxia Headache, diploplia
Chorea, Nystag tremors
Cereb ataxia,
teratogenic Fetal hydantoin Neural tube Toxic epidermal necrolysis
(cleft, cong ht) defect
Gabapentine;
- used as add on drug
- in partial, general.
- not enz inducer.
- Dose adj in renal failure.

Lamotrigine rash TEN.

Vigabatrine irreversible constriction of visual field.

Levitiraceticam
- oral
- Safe, fewer side effects
- used for partial seizures.
- No effect on liver enz
- Dose adj in renal failure.

Phenobarbitone 1st line in infants.

Paraldehyde may be given IVI or rectally or IM in extreme circumstances


because it cause tissue damage & slughing.
Epilepsy & driving;
- After 1 seizure or lost consc not allowed for 1 year.
- For vocational licence free for 10 yrs of
1- attacks
2- medications
3- liability (e.g. structural br lesion).

Epilepsy & pregnancy;


- Continue antiepileptics as effect of epilepsy & hypoxia is more
dangerous.
- Folic acid supplementation.
Sleep disorders
Narcolepsy;
is a clinical syndrome of;
1. daytime sleepiness,
2. cataplexy, emotionally-triggered, transient muscle weakness. Most
episodes are triggered by strong, generally positive emotions such as
laughter, joking, or excitement, result from loss of the neuropeptides
orexin-A and orexin-B. TTT; a REM sleep-suppressing medication
(eg, venlafaxine, clomipramine, fluoxetine) .
3. hypnagogic hallucinations, are vivid, often frightening hallucinations
that occur just as the patient is falling asleep or upon awakening.
4. sleep paralysis; is a complete inability to move for one or two minutes
immediately after awakening.

Only about 1/3 of patients will have all four symptoms; thus, the
diagnosis of narcolepsy should be considered even in patients with
sleepiness alone.
The HLA association is DQB1,
Diagnostic testing for narcolepsy includes an overnight
polysomnogram (PSG) followed by a Multiple Sleep Latency Test
(MSLT).
In Narcolepsy, REM sleep occur at the onset of nocturnal sleep.
Daytime attacks also consist of periods of REM sleep occurring out
of context.
Secondary Narcolepsy can occur with rare lesions of the posterior
hypothalamus and midbrain. Tumors, vascular malformations,
strokes, Prader-Willi syndrome, Niemann-Pick disease type C, and
paraneoplastic syndromes. All patients with secondary narcolepsy
have obvious neurologic deficits, with cognitive, motor, and/or eye
movement abnormalities .
TTT of narcolepsy;
1. Creating a sleep time table & alcohol intake.
2. CNS stimulant as modafinil, methylphenidate or amphetamines.
3. Clomipramine is a tricyclic antidepressant
Hyperkinetic movement disorders
Tics; brief, repeated, stereotyped ms contractions that
are often suppresible.
Myoclonus; sudden, brief, shock like, ms twitches that
interfere with normal movement & are not suppressible.
- Types;
1. Normal while falling asleep.
2. Jevenile myoclonic epilepsy, Variant CJD.
3. Static myoclonic encephalopathy (hypoxia after cardiac
arrest).
4. Drugs as amitryptiline, TCA.
5. Metabolic (LCF, RF).
- TTT; clonazipam, valproate.
Fasciculations = visible contraction of a group of ms
fibres.
Causes; anxiety ,fatigue, (Ca), MND.
Fibrillation; invisible contraction of individual ms fibrils.
Chorea; rapid, semiperpuseful, dancelike, non-patterned, involuntary mov,
proximal > distal.
causes;
1. Hereditary; Huntington.
2. Metabolic; wilson, abetalipoproteinemia, thyrotoxicosis.
3. Inflammatory; Rh fever (Sydenhams), SLE, APA.
4. Drugs; phenytoin, L-dopa, anticholinergics (TTT of parkinson) Alcohol,
cocaine, amphetamine. (illicit drugs) OCP, pregnancy.().
5. Toxins; Cu, Mercury, Mg.
- TTT; neuroleptics; Haloperidol, tetrabenazene.
Sydenhams Chorea;
+ hypotonia. + pendular or sustained reflexes.
It is mostly observed in young girls and presents as involuntary
movements of face, tongue and upper extremities. emotional
instability 3% of acute rheumatic fever
ttt; rest, diazepam, Haloperidol.
Wilsons Disease
Definition
Its a neurodegenerative disease of copper metabolism
characterized by accumulation of copper in different
tissues mainly in the liver, brain , kidney & cornea due to
inability of the liver to excrete copper in bile which is the
main pathway for copper elimination.
NeuroPsychiatric:
are the most common initial presentation in adults.
Kayser-Fleisher rings are almost always present when the
patient has neurological symptoms
Irritability, psychosis, mania, abnormal behavior
Personality changes, depression.
Parkinsonian symp, rigidity, bradykinesia , tremors,
dysarthria. Dystonia, incoordination.
Investigations:
No one test is completely reliable
Low serum copper level (may be normal)
Low serum ceruloplasmin (Normal in 20%)
Increase 24 hrs urinary copper
MRI Brain: bilateral degeneration in basal
ganglia with caudate atrophy.
Liver biopsy: increase in hepatic copper by
quantitative essay
Neuroacanthocytosis
This AR syndrome = features of acanthocytosis
(i.e.spiked RBCs), chorea & Personality
changes ,Progressive cognitive impairment&
normal betalipoproteins.
It begins in adult life 20-50 yrs old and more
common in men. course is progressive.
Deg. of Basal ganglia is a feature. caudate atrophy.
hypobetalipoproteinemia:
it results from homozygous or heterozygous mutation in APO B gene
with impaired APO B synthesis and decreased LDL cholesterol level.
ABL is an AR condition while FHBL shows codominance.
GIT symptoms: malabsorption,diarrhea and distension.
Neurologic symptoms:
Slow intellectual development.
Peripheral neuropathy. Absent deep tendon reflexes.
Intension tremors, ataxia. Chorea.
Ophthalmic:
visual acuity.
retinitis pigmentosa.

CBC: Acanthocytosis + thrombocytopenia


low fasting cholesterol levels
LFTs And PT due to vitamin K def.
ttt; IM vit E.
Hemiballismus; violent form of chorea, wild large
amplitude mov of one side of the body or 1 limb only,
due to contralateral subthalamic hge or
infarction.ttt; tetrabenazine.

Athetosis; slow distal snake-like involuntary mov


mainly in the hand, in putamin .

Dystonia = twisting mov & abn posture dt


contraction of both agonists & antagonists e.g.
torticollis, writer cramp. ttt; haloperidol.
Myotonia dystrophica.
AD,Presents at 3rd decade.
Myotonic facies,Frontal baldness
Ptosis, cataract, mild cognitive impairment.
Wasting of face, sternomastois, arm, legs, reflexes.
Myotonia (delayed muscular relaxation after contraction).
Endocrine; DM, gynecomastia, testicular atrophy.
Cardiomyopathy, ECG changes.
Diagnosis; myotonic discharge on EMG.
Ttt; phenytoin for the myotonic phenomena.
Myotonia
dystrophica
Tremors; rhythmic oscillation of a body part
dt intermittent ms contraction.

TREMORS

Coarse Fine
Alcohol.
Senile.
Thyrotoxicosis.
Anxiety.
Hysterical.
Drugs; theoph, B2 agonist,.

STATIC KINETIC amiodarone, valproate


cyclosporin
-Parkinson (4-8 hz) =INTENSION
-LCF
-CRF
Benign essential.
-RF Cerebellar.
-Wilson MS.
Benign essential Tremors
-most common tremor.
-Elderly.
-AD.
-Coarse (8 Hz) tremors& UL mainly.
-Postural & appear immediately on mov (but cerebellar
tremors appear after 1st part of vol mov)
-Titubition( MS also has titubition & intension tremors).
--by anxiety.
- by alcohol.
-ttt; BB.
Parkinsonism ( dopamine in substantia nigra)
Refers to a triad of;
1. Resting tremors (4-6 Hz), pill rolling, by stress,
by sleep & mov.
2. Bradykinesia; difficult starting walking, short
steppage gait, micrographia.
3. Rigidity. (flexors); gorilla like attitude.

Loss of emotional & associated mov; mask face,


monotonus speech, loss of arm swinging, can not
resist retropulsion &propulsion,pers glabellar reflex.
NB; No sensory, no fasciculation, no weakness,

no sphincteric disturbance.
Akinetic rigid syndromes (Parkinsonism)
Cause History

1-Idiopathic asymetrical tremors, good response


to L-dopa
2-Heredofamilial (wilson). Age<40

3-Traumatic (boxers)

4- Inflammatory; postencephalitic, syphilis. Sudden onset

5-Vascular; atherosclerotic (multiinfarct dementia)

6- Toxic; drugs (reserpine, PZN, valproate)


antidopamenrgic, M ( C Monoxide, Mn).

7- Parkinson plus+ Lack of tremors

* Progressive supranuclear Palsy ophthalmoplegia, peudobalbar palsy


(falls as 1st symp)
* Multiple System Atrophy autonomic,
* Lewy body dementia. Intermittent consciousness , visual
hallucin, early dementia.
Ttt of Parkinsonism
1. Anticholinergics for tremors e.g. benzhexol; side
effect(atropin like action); urine retension, hallucination,
tachycardia, dry mouth.
2. Dopamine agonist for bradykinesia (ropirinole, pramipexole,
cabergoline, pergolide)(cause galactorea).side effects; less
effective ,GIT upset, VC, arrythmias, # PVD, IsHD, mental.
3. Levodopa (Sinemet=Ldopa +carbidopa which prevent
peripheral decarboxilation). side effect; on & off , wear off,
psychosis, chorea, GIT, tachy, # in narrow angle glaucoma.
4. MAO B inhibitor (Selegiline); inhibit dopamine catabolism in
the brain but not neuroprotective, may mortality.
5. COMT(catechol o methyl transferase) inhibitor (Tolcapone,
Entacapone)
6. Presynaptic Dopamine uptake inhibitor (Amantadine)
7. Surgery.
(Sinemet=Ldopa +carbidopa
on & off phenomenon
- Fluctuation of drug response with prolonged
therapy= chorea, immobility.
- Ttt;- shortening the interval between doses , dose,
restrict dietary protein, give L dopa 1 hr before
meals , selegiline , Dopamine agonist , drug
holidays. Apomorphine(facelitate the effect of
sinemet)

wear off; end of dose phenomena;


ttt; controlled release prep, frequency, or dose.
Drug induced neurological symptoms
1. Drug induced extra symptoms;e.g Dystonia
e.g. antipsychotic e.g., antiemetic.(anti dopaminergic drugs)
ttt; stop the drug, give anticholinergic for tremors e.g. Benztropine or
benzhexson procyclidine.( akeniton tab 1x1, parkinol)
2. Tardif(=delay) dyskinesia;
- Antipsychotic as chlorpromazine(largectil) & Haloperidol, unusual with
newer agents(=atypical antipsycotic) as olanzapine, clozapine(clozapex).
- After 6 months of the drug intake.
- 50 % irreversible, progressive.
- C/P; orofacial dyskinesia e.g. chewing, lip smacking.
- ttt; stop & give tetrabenazine(used in TTT of hemibalismus)
- valproate.
- baclofen 10, 25 mg 1x2, diazepam.
3. Malignant neuroleptic S;
- C/P; altered mental state, rigidity, autonomic dysf, fever, high CPK
- Cause; potent neuroliptics e.g. haloperidol.
- ttt; dantrolene, bromocriptine.
- Complications; Rabdomyolysis, ARF.
DD of hypotonia.
Chorea , Cerebellar disease , Shock stage
of ., Post.Col , Polyneuropath , myopathies.

DD of hypertonia

Spasticity Rigidity
() (extra )
Clasp knife Lead pipe or
FL of UL cogwheel (interrupted by tremors)
EXT of LL flexors
Ataxias
= incoordination = loss of orchestration & fluidity of mov.

Ataxias

hysterical

Motor
=cerebellar Sensory(pathway Vestibular
of deep sensation e.g. minieres dis
1. PN Labyrinthutus
2. Tabes dorsalis. Acoustic neuroma
3. PC.
4. Medial lemniscus (Br stem)
5. Thalamic S
Vestibular ataxia; post. circulation
- + vertigo, Nystagmus.
- Stamping gait.
- Rombergism.
- Drop attacks Drop attacks are attributed to a sudden loss
of tone mediated by vestibulospinal reflexes. Unlike
presyncope and seizures, there is no faintness or loss of
consciousness with drop attacks. Patients with drop attacks
of vestibular origin often have a sensation of being pushed or
pulled to the ground
- Sensory ataxia
- Rombergism
- Stamping gait.
- Deep sensory loss.
- No crainial n, no dysarthria or nystagmus.
Cerebellar Ataxia
Causes;
Hereditary;
- Friedriech ataxia.
- Ataxia telangectasia
- Vit E def; spinocerebellar deg dt abetalipoproteinemia.
- Acquired ;
1. Cong; Arnauld chiari malformtion.
2. Vascular; hge, infarction.
3. Demyelinating dis; MS.
4. Infective; encephalitis , AIDS, polio, coxsachi, lyme, HZV.
5. Metabolic; Na, B1, B12, vitE .
6. Toxic; Alcohol , phenytoin , valproate, lithium , mercury.
7. Neoplastic ; paraneoplastic , tumors ,2ries.
Friedriech
- AR, frataxin gene, trinucleotide repeat GAA.(as CAG in Huntington chorea)
- Age = 5-15 yrs.
- Ataxia + PC, PN, .i.e(ataxia +SCD)
- Optic atrophy, deafness.
- Cardiomyopathy, DM.
- skeletal=; pes cavus, kyphoscoliosis.
- Inv; ECG, vit E, DNA testing.

Ataxia telangiectasia.
-AR, childhood.
-Ataxia,Telangiectasia; face, conjunctiva.
-Recurrent infection. -Malignancy e.g. leuk, lymph.
Inv; Ig A & other Igs, FP.
Ataxia with vit E def;
-AR, defect of -tocopherol transferase gene.
- As Friedriech + hypobeta lipoproteinimia fat soluble vit & retinopathy dt vit A def.
- ttt; oral or IM vit E.
Cerebrotendinous xanthomatosis.
-inherited.
-Accumulation of cholesterol in tissues (brain, PN, tendons)
-C/P; early dementia, ataxia, large tendon xanthomas.
-ttt; chenodeoxycholic acid.
Paraneoplastic syndrome
severe ataxia, dysarthria, vertigo, nystagmus.
-May preceed tumors esp small cell lung C, ovarian ,lymphoma
-50-75% antineuronal Ab; anti Yo (# cerebellar perkinjie), anti Hu (#
wider range of neurons).

. Anti Yo antibodies & Anti Hu antibodies are


found in around half of all patients with
paraneoplastic cerebellar degeneration.
Associated with small cell carcinoma, ovarian
tumours and Hodgkins lymphoma. It is
usually associated with sensory neuropathy.
Benign paroxysmal positional vertigo
is probably the most commonly recognized cause of
vertigo. commonly attributed to calcium debris within the
posterior semicircular canal
Classically, patients describe a brief spinning sensation
brought on when turning in bed or tilting the head .
The dizziness is quite brief, usually seconds.
Ear pain, hearing loss, and tinnitus are absent.

The diagnosis of BPPV is generally made by history.


The Hallpike maneuver can support its presence, but is
positive in only 50 to 80 %
The natural history of BPPV is one of repeated, brief
vertiginous episodes that continue for weeks or months.
TTT: Vestibular sedatives. (betahistidine).
Vestibular neuritis or neuronitis
is believed to be a viral or postviral inflammatory disorder,
affecting the vestibular portion of the 8th cr.N.
Vestibular neuritis is characterized by the rapid onset of
severe, persistent vertigo, nausea, vomiting, and gait
instability.
A diagnosis of vestibular neuritis is largely based on
clinical information. There are no specific diagnostic tests.
However, the clinical features of cerebellar hemorrhage or
infarction may be similar and brain imaging is therefore
often required to rule this out .
Recovery may be hastened with corticosteroid therapy.
Herpes zoster oticus (Ramsay Hunt syndrome)
is believed to represent activation of latent herpes zoster infection of
the geniculate ganglion.
In addition to acute vertigo and/or hearing loss, ipsilateral facial
paralysis, ear pain, and vesicles in the auditory canal and auricle
are typical features.TTT by acyclover

Meniere's disease
is a peripheral vestibular disorder attributed to excess endolymphatic
fluid pressure, which causes episodic inner ear dysfunction.
Affected patients present with spontaneous episodic vertigo lasting
for minutes to hours, usually associated with unilateral tinnitus,
hearing loss, and ear fullness.
Headache.
Acute recurrent headache .
Episodic migraine.
Cluster headache.
Trigeminal neuralgia. Intense stabbing pain lasting seconds in trigeminal
nerve distribution.
Glaucoma .Red eye, haloes, visual acuity, pupil abnormality.
Subacute headache.
Temporal (giant cell) arteritis>50y., scalp tenderness, ESR, rarely visual.
Chronic headache.
Tension type headache : Band around the head, stress, low mood.
Cervicogenic headache : Unilateral or bilateral, band from neck to
forehead, scalp tenderness.
Medication overuse headache: Rebound headache on stopping analgesics.
intracranial pressure: Worse on waking/sneezing, neurological signs,BL Pr.
Headache
Causes
1. Extracranial; eye, sinuses, ear, dental, TMJ,
cx osteoarthritis.
2. Vascular; migraine(VC followed by VD),
cluster, HTN, occlusion.
3. Intracranial; artritis, pituitary, ICT.

4. Tension; common tension H, conversion


reaction, depression.
Headache.
The skill lies in deciding which headaches are benign, requiring no intervention, and which require action.
History.
When did the headaches start? New or recently changed headache.
How often do they happen?
Do they have any pattern (e.g. constant, episodic, daily)?
How long do they last?
Nature and quality of the pain.
Site and spread of pain.
Associated symptoms e.g. nausea/vomiting, visual dist., photophobia, neuro S.
Predisposing and/or trigger factors. and/or relieving factors.
Family history
Examination.
BP.
Brief neurological examination including fundi and visual acuity.
Palpation of the temporal region/sinuses for tenderness.
Examination of the neck.(cervical spine lesion)
Investigation.
Often not needed. Consider ESR if temporal arteritis is suspected.
CT scan if....
Migraine. 10-24 hrs
Cause,Disturbance of cerebral blood flow under the influence of 5-HT.
basilar migraine (affect the post. Circulation) visual chaos (e.g. zig-zag lines,
jumbling of print, dots); hemianopia; hemiparesis; dysphasia;; dysarthria; ataxia.
Classical migraine :Aura lasting 10-30min. followed by unilateral throbbing
Pulsating headache nausea or vomiting photophobia.
Episodic migraine (common migraine): Unilateral throbbing headache nausea
or vomiting photophobia but without aura often premenstrual.
Trigger factors
have a trigger for their migraine.
.Psychological factors :Stress ; anxiety/depression; extreme emotions e.g. anger .
Food factors :Lack of food; specific foods e.g. chocolate, cheese; alcohol.
Sleep :; changes in sleep patterns (e.g. late nights, weekend lie-in, shift work,
holidays); long-distance travel.
Environmental factors :Loud noise; bright/flickering lights; strong perfume;
Management
Management of an acute attack
Advise to rest in a quiet, dark place and sleep if possible.
Analgesia ibuprofen 400mg, or soluble paracetamol 1g at 1st signs of an attack
antiemetic e.g. domperidone 20mg. consider 5HT 1agonists e.g.
sumatriptan(sumagrain,emegrain) 50-100mg po, stops 70-85% attacksstart with
lowest dose and as needed; do not give if ergotamine(migranil,no migrain) taken
<24h. Previously.
Management of chronic migraine

Reassure about the benign nature of migraine.


Instruct patients about management of an acute attack.
Ask the patient to keep a diary to identify possible trigger factors, assess
headache frequency, severity, and response to treatment.
Avoid trigger factors where possible. Give advice on relaxation techniques
and stress management to all patients.
Stop the OC pill if migraine starts or worsens when the pill is started.
Prophylaxis.
Consider if >4 attacks/mo. or very severe attacks. Try a drug for 2mo. If
effective, continue for 4-6mo. then dec. dose slowly before stopping.
B.blockers :e.g. atenolol 25-100mg bd. Patients who do not respond to
oneB-blocker may respond to another.
Sodium valproate(depakin) Start at 300mg bd.
Tricyclic antidepressants e.g. amitriptyline 10-75mg once. Start at
low dose and inc.
Pizotifen(antihistamenic drug).
Cluster headaches
Clusters of extremely painful headaches focused around 1 eye with
associated autonomic symptoms (drooping eyelid, red watery eye, runny
or blocked nose). May occur at any age.
6:1 male to female.More common in smokers.Pain lasts up to 1-2 hr. and
occurs once every day for 4-12wk. then disappears for 1-2y. Recurrences
affect the same side.
Management
Acute attack:
%100 oxygen at a rate of 7-12 l/min(to releif the VD).
5HT agonists e.g. sumatriptan (6mg s/cut) stops 75% attacks
within 15min.
Prophylaxis : Consider if attacks are frequent, last >3wk. or cannot be
treated effectively .Options:
Verapamil stops 66% attacks(BBin migrain).
Lithium.
Methysergide(ergometrin).effective but use limited by side-
effects. Only used if other drugs are contraindicated, not tolerated,
or ineffective.
Benign intracranial hypertension=psudotumer cerebri)
is most common in women between the ages of 20 and 50 yrs.
BIH is likely due to high pressure caused by the buildup or poor
absorption of CSF in the subarachnoid space.
Symptoms include headache, nausea, vomiting, and pulsating
intracranial noises, closely mimic symptoms of brain tumors.
Papilloedema.
Obesity ,pregnancy, Behcet $ , SLE , Hypopara.
Vitamin A, prednisolone , minocycline(used in TTT of acne), OCP
,Nitrofurantoin(uvamin retard) and cyclosporine.
Diagnosed by CT & fundus examination.
TTT:
Wt reduction.
Acetazolamide(Diamox):carbonic anhydrase inhibitor is used to treat
BIH.
VP shunt.
Repeated Lumbar puncture.
Subarachnoid haemorrhage
Spontaneous bleeding into the subarachnoid space.Frequently fatal, 85% of IC aneurysms are
located in Ant. Circuation.mostly ant. Cumminicat.
Causes:
No cause (15%).
Rupture of congenital berry aneurysm (70%).
Arterio-venous malformation (15%).
Bleeding disorder., cocaine & amphetaine.
Mycotic aneurysm 2 to endocarditis (rare).
Risk factors
Smoking, alcohol, BP, lack of oestrogen (less common pre-menopause).
Associations (Berry aneurysm)polycystic kidneys(do screening by MRA if there is +ve FH of IC
hge , coarctation of the aorta, Ehlers-Danlos syndrome, fibromuscular dysplasia RAS, PAN.
Complications: rebleeding , focal ischemic injury dt cerebral vasospsm, hydrocephalus, SIADH
Presentation
Typically presents as a sudden devastating headache often occipital.
Vomiting and collapse with loss of consciousness fitting focal neurology follow.
Examination
May be nothing to find initially. Neck stiffness takes 6h& Focal neurology
Papilloedema.
Retinal and other intraocular hemorrhages.
Inv. CT & Lumbar puncture(xanthochromia ie RBCs in the CSF).
TTT(nimodepin) (nimotop which is Cch Bs act selectively on the cerebral BV)
If suspected, admit immediately as a medical emergency.
Trigeminal neuralgia
Paroxysms of intense lansinating , burning electric shock type pain
lasting seconds to minutes in the trigeminal (V) nerve distribution. 96%
unilateral. Mandibular/maxillary > ophthalmic division. Between attacks,
there are no symptoms.
Frequency of attacks is variable up to hundreds of attacks/d. Pain is
often provoked by movement of the face (talking, eating, laughing) or by
touching the skin (shaving, washing). Can occur at any age but more
common >50y.
Causes: DM,HZV, MS.
Management :
Spontaneous remission may occur.
Carbamazepine100-400mg tds po dec.frequency and intensity of attacks
treatment with lamotrigine, phenytoin, or gabapentin, or clonazepam.
Thermocoagulation.
surgical intervention.
paraparesis

Flaccid
1. PN e.g. GBS, CIDP
Spastic paraplegia 2. Myopathies
3. radiculomyelitis

With sensory level Without


1. 5 syst dis (MS, pellaga, SCD, friedriech, MND)
1. Trans. Myelitis
2. Occlusion of unpaired ACA, Parasagital meningioma
2. Cord compression 3. Familial, tropical (HTL2)
3. Ant spinal art occ
paraparesis
Paraparesis with sensory level.
1.Vascular.
2.Traumatic.
3.Inflammatory.
4.compression.
Extramedulary lesion. root pain
Intramedulary lesion. Sphincteric dysf.
Paraparesis without sensory level.
Transverse Myelitis
Causes;
1. Infection;
- viral; exanthema, EBV, influenza.
- Bact; pyogenic.
2. Immune;
- collagen; SLE, sjogren.
- demylinating; MS, ADEM.
3. Iatrogenic; post vaccine, radiotherapy.
C/P;
- mainly lower thoracic.
- Acute or sub acute + fever + back pain + sensory loss with
level, sphincteric dis, flaccid paraparesis then spastic.
Inv; MRI; cord swelling , plaques in MS.
CSF; normal protein, oligoclonal bands in MS.
Ttt; Pulse steroids then full dose.
Epidural abscess
Spinal epidural abscess is a rare disease,
The infection can originate via contiguous spread from infections of skin and soft
tissues or as a complication of spinal surgery . Other cases of epidural abscess arise
from a remote site via the bloodstream.
Diabetes, alcoholism, and HIV infection are risk factors.
The most common pathogen is Staphylococcus aureus, 2/3 of cases
The classic clinical triad consists of fever, spinal pain, and neurologic deficits..
MRI is the preferred test and is highly sensitive for this diagnosis.
Blood cultures and/or aspirate of abscess contents to identify the etiologic organism.
Surgical decompression and drainage with systemic antibiotic therapy is the treatm.
Epidural metastasis
Usually >50 years ,Subacute, may worsen abruptly,Segmental
cord syndrome,MRI.
Gliomas
Young adults ,Slowly progressive
Central Intramedullary cord tumor syndrome
MRI with gadolinium enhancement .
Syringomyelia
Children, young adults , Slowly progressive, Central
intramedullary cord lesion.
is chronic disorder characterised by the presence of glial-
lined cavities situated in the central part of the spinal cord.
Recognised causes include Chiari type I malformation,
central cord tumours, basal arachnoiditis and trauma.
There may be sensory loss, wasting of the small hand
muscles, uni- or bilateral Horners syndrome, abnormalities
of sweating, development of Charcots joints and Chiari I
malformation (due to arachnoiditis).
MRI
Ant. Spinal artery occlusion.
Infarction Usually >60 years, Abrupt onset
Anterior cord syndrome
intact PC, intact deep sensation.
MRI .

Dissociated sensory loss;


= preserved touch, lost P & T.Dt;
- Brown Sequard S
- ant sp art occlusion.
- lat medullary S.
- early syringomyelia.
Motor neuron disease:
UMN signs include muscle spasticity,
weakness especially in the extensors of the
upper limb and flexors of the lower limb muscles,
pseudobulbar palsy.
LMN features include muscle wasting and
fasciculation, depressed reflexes and bulbar
palsy.
Onset of the disease is usually in mid to late
adult life with the incidence increasing with age.
ALS: Usually >60 years ,Slowly progressive ,
Pure motor syndrome,
Hereditary spastic paraparesis
often classified based upon spasticity occurs with
or without other neurologic abnormalities {mental
retardation; deafness or retinopathy; ataxia and
dementia}. (complicated Vs simple HSP).
Bulbar & Pseudobulbar palsy(bulb=medulla)
Bulbar palsy Pseudobulbar palsy

LMNL of 9-12 cr ns Bilateral UMNLof 9-12 cr ns


(nuclear of infranuclear) (supranuclear)
Causes 1- Br stem; -MND

- infarction, - MS
- syringobulbia - Bil Stroke

- MND - Severe head injury

- 2- Jugular foramen;
- nasopharyngeal carcinoma
- Fracture base.
C/P dysarthria (spastic Bulbar palsy)
Hoarseness Spastic dysarthria
Nasal regurgitation Hoarseness
Dysphagia Nasal regurgitation
Lost reflexes Dysphagia
Exaggerated jaw reflex
Preserved palatal
pharyngeal reflexes
Quadriparesis
1. Cx myelopathy above C4 (sens level)
2. MND (U & L)
3. MS (mainly upper)
4. Bilateral stoke (upper & sens)
5. PN ( peripheral, sens)
6. Myopathy ( proximal)
Dermatomes
1. insular cortex
2. lenticular nucleus (globus
palladus)
3. lenticular nucleus (putamen)
4. corpus callosum (genu)
5. caudate nucleus (head)
6. internal capsule -- anterior
limb
7. internal capsule -- posterior
limb
8. thalamus
9. internal capsule --
retrolenticular portion
10.optic radiations
Lesions of the frontal lobe include
1. Difficulties with task sequencing and executive
skills.
2. Contralateral motor.
3. Expressive aphasia .
4. Primitive reflexes , Loss of bladder control.
5. Preseveration=(repeatedly asking the same
question or performing the same task).
6. Anosmia and changes in personality.
Epilepsy; 50% of pts presented with status
epilepticus. without history of epilepsy, have
frontal lobe tumors.
Lesions of the parietal lobe include
Cortical sensory loss or inattention.
Apraxias, astereognosis (unable to recognise an object by feeling it)
Visual field defects (typically homonymous inferior quadrantanopia).
Alcalculia (inability to perform mental arithmetic).
Lesions of the temporal lobe cause;
Visual field defects (typically homonymous superior quadrantanopia),
Wernike's (receptive) aphasia,
Auditory agnosia.
Psycomotor seizures.
Hallucinations; auditory, visual, gustatory. Lip smacking, grimacing.
Lesions of the Occipital lobe;
Occipital lobe lesions include cortical blindness (blindness due to damage
to the visual cortex and may present as Antony syndrome where there is
blindness but the patient is unaware or denies blindness visual agnosia
(seeing but not percieving objects - in agnosia the objects are seen and
followed but cannot be named).
homonymous hemianopia ( macular sparing)
Transient ischaemic attack
History is as for stroke but recovery takes place within 24h. of initial
symptoms.
Patients with a history of TIA have 20% risk of stroke in the following
month, with highest risk in the first 72h.

Investigations.
CT or MRI scan to confirm diagnosis.
Carotid dopplers.
Echocardiogram (if recent MI, CCF/LVF, or murmur.(
ECG, CXR. blood, FBC, ESR, U&E, Cr, lipids, glucose, PT & PTT

Management of TIA.
Once all symptoms have stopped, start aspirin 50-300mg od.
Start treatment for risk factors e.g. advise to stop smoking, start
antihypertensives .
L M ACA
ACA

Face

UL

LL
sensation Anterior circulation
LGG to optic R
Posterior circulation

PCA cortical B.C.HH with macular sparing.


capsular B.Thalamic S (C.hemiplegia, hyperpathia, choreoathetosis)
PICA or vertebral Lat. Medullary S.
BS branches( crossed H)Midbrain ipsi 3, 4, C. hemiplegia
Pons ----------------- ipsil 6, 7+ C. hemiplegia.
Medulla -----------------ipsil 9,12+C. hemiplegia
Vertebrobasilar insufficiency
1. Cranial nerve palsies. (Diplopia, dysarthria,
dysphagia, hiccup).
2. Gait ataxia.
3. dizziness, nausea, vomiting,
4. Motor deficit ,Coma.
5. Bilateral signs suggest basilar artery
disease.
6. Drop attacks vertebrobasilar
insufficiency.
Acute stroke syndromes
Anterior cerebral artery :
Motor and/or sensory deficit (LL >> face, UL)
C. hemiplegia +7,12+LL cortical sens. Loss
personality, primitive reflexes,(Grasp, suckling reflexes), urine incontinence.
Middle cerebral artery:
Dominant hemisphere: aphasia, motor and sensory deficit (face, UL>LL)
homonymous hemianopia.
Apraxia, alexia, anosognosia.
C. hemiplegia UL>LL+7,12 +hemihypothesia.
ICA as MCA + ipsilateral blindness (ophth. A.).
Posterior cerebral artery.
Homonymous hemianopia,; visual hallucinations,
(thalamus) sensory loss, choreoathetosis, spontaneous pain;
( midbrain) III nerve palsy, paresis of vertical eye movement, motor deficit.
Bilateral PCA cortical blindness ( Antony S).
hemiplegia

No cranial N Other crossed + 7, 12


Dissociated cranial N
sens. loss
Capsular MCA

main stem MCA


Br.Stem
Brown sequard S br. Of post. circ.
main stem ACA
L

Frontal pathways

Corticobulbar T

Corticospinal T

Retrolentiform . Spinothalamic T

LGG to optic R
sublentiform
PCA

MCA ACA
Lacunar infarctions <1.5 cm2
Small penetrating vessels.
1- Pure motor hemiparesis. (classic lacunar syndromes)
2- Pure sensory deficit .
3- Hemiparesis & ataxia.
4- Dysarthria/clumsy hand.

hemiplegia without sensory, visual, or cognitive


abnormalities (pure motor stroke) favors lacunar infarction
or a small ICH.
Abnormalities of language, the presence of motor and
sensory signs on the same side of the body suggest
anterior circulation disease.
Vertigo, loss of consciousness, diplopia, deafness, crossed
symptoms suggest posterior circulation dis.
The sudden onset of impaired consciousness in the
absence of focal neurologic signs is characteristic of SAH.
Lacunar infarction

LT thalamic lacunar Infarction


Stroke in young;
- Cardiac embolization.

- Vasculitis.

- Familial hypercholestrolemia.

- Hypercoagulable state.

- Patent foramen oval.

- Vertebral or carotid A dissection.

- Cocaine.

Superior sagital sinus thrombosis;

Pregnant, OCP , dehdration in children ,


- Biparietal or bifrontal hgic infarction.

- Multiple scattered lesions.

- Prominent epileptic fits.


Internal carotid artery dissection
Headache, including neck and facial pain,
is commonly ipsilateral to the dissected artery.
usually precedes a cerebral ischemic event, unlike a headache of
stroke, which follows the ischemic event.
Transient episodic blindness, or amaurosis fugax.
a partial Horner syndrome (Ptosis with miosis)is painful .
Neck swelling, isolated ipsilateral neck pain 25%.
Focal weakness.

An acute Horner's syndrome associated with pain of the neck or face suspect
carotid ar diss.
....>An axial MRI of the neck and MRA will detect most internal carotid artery
dissections .
However, conventional angiography remains the gold standard.
Vertebral artery dissection
Patient history may include the following:
Ipsilateral facial dysesthesia (pain and numbness)- Most common symptom.
Dysarthria or hoarseness (cranial nerves IX and X). Dysphagia (CN IX &X).
Vertigo,Disequilibrium ,Unilateral hearing loss. Diplopia.
Nausea and vomiting, Hiccup.

Examination.
Depending upon which areas of the brain stem or cerebellum are ischemic:
Limb or truncal ataxia. .Nystagmus.
Ipsilateral Horner syndrome in 1/3 of patients (ie,
impairment of descending sympathetic tract)
Contralateral impairment of pain and thermal sensation in
the extremities (ie, spinothalamic tract)
Lateral medullary syndrome.
lateral medullary syndrome
8

sympathetic
5

9, 10

Ipsilateral 5,8,9,10.
Horner.
cerebellar.
Ipsilateral loss of pain & temp of face.
Contralateral loss of pain & temp of the trunk.
(spinothal tract)
lateral medullary syndrome
Vestibulocerebellar symptoms and signs.
Sensory symptoms and signs.
Pain or unpleasant feelings in the face are
sometimes the earliest and most prominent
feature of the lateral medullary syndrome.
Contralateral loss of pain & temp of the trunk.
Bulbar muscle weakness .
Respiratory dysfunction.
Autonomic dysfunction The anatomic basis
is thought to be involvement of the dorsal motor
nucleus of the vagus nerve.
Horners S
Central (B.S., S.C.)
=1st order neuron. Br. Stem.
e.g.syringobulbia, pontine glioma.
Loss of sweating from face, upper chest.

Peripheral preganglionic (ant root of C8-T3,


sympathetic chain) =2nd order neuron. Root
of Neck
e.g. pancoast tumour, cervical rib, LN,
thyroidectomy.
wasting of small ms of hand (T1 root).
Loss of sweating from face only.

Peripheral postganglionic (internal carotid


art) =3rd order neuron. I C art.
e.g. ICA dissection, cavernous sinus $, orbital
apex S.
+ bulbar S if at jugular foramen.
Loss of sweating from forehead only.

NB: 1% Hydroxyamphetamine or pholedrine eye


drops releases stored norepinephrine from
the postganglionic adrenergic nerve endings.
One hour after instillation, a normal pupil and
a first or second-order Horner's pupil will
dilate, whereas a third-order Horner's pupil
will not dilate.
Pancoast S;
Triad; Horner + waisting of small ms of the hand+
pain in axilla.
Causes; apical Br C, subclavian Art aneurysm.
Radiological evaluation
Non contrast CT.
MRI is more sensitive than CT for the early brain infarction , lacunar &
br. Stem infarction.
Small subarachnoid hemorrhages can be missed by either CT or MRI.
Lumbar puncture may be needed to make the diagnosis of
subarachnoid hemorrhages .
Cardiac studies Transthoracic echocardiogram may
miss a clot in the atrial appendage & a patent foramen ovale.
TEE.
Vascular studies
Anterior circulation duplex ultrasound of the neck and transcranial
Doppler (TCD) of the intracranial arteries,MRA of the neck and head
arteries.
Posterior circulation, duplex of the origins of the vertebral arteries. CTA
and MRA of the neck vertebral arteries do not adequately show the
origins of the vertebral arteries
Initial assessment of acute stroke :
1.Classification of stroke.
Transient brain ischemia.
87%.
Ischemia:(Thrombosis,Embolism,Syst.hypoperf.)

Intracerebral hemorrhage. 10%.

Subarachnoid hemorrhage. 3%.

2. vital signs.

3. Decide if thrombolytic therapy is warranted


(first 3-4.5 hrs).
4. Immediate laboratory studies

Serum glucose.

CBC ,Cardiac enzymes. Electrolytes,bl. urea,


creat. PT,INR, PTT.O2 sat. Lipid profile.
ACUTE THERAPY
Thrombolytic therapy: administered within three
hours of the onset of symptoms significantly
reduces long-term disability despite risk of bleeding.
Before Thrombolytic therapy is started, SBP 180 mmHg
and DBP is 110 mmHg.

Antithrombotic therapy;
Guidelines issued in 2007 anticoagulation is not
recommended for the treatment of patients with acute
ischemic stroke because of limited efficacy and an increased
risk of bleeding complications.

We do not recommend the routine use of unfractionated or


low molecular weight heparin in patients with AF presenting
with an acute ischemic stroke except after 2 weeks .
We recommend early anticoagulation for :
1. Cardioembolism from intracardiac thrombus associated with
significant valvular disease, severe congestive heart failure,
or prosthetic heart .
2. Large artery atherosclerotic stenosis with intraluminal
thrombus.
3. Dissection of a cervical or intracranial large artery.
Among patients with large infarcts, the initiation of warfarin
therapy should be delayed for two weeks because of the
potential risk of hemorrhagic transformation. A similar delay is
recommended in patients with poorly controlled hypertension.

Full-dose anticoagulation should not be used for patients with


a large infarction , uncontrolled hypertension , or other
bleeding conditions.
For secondary prevention of stroke
Noncardioembolic stroke antiplatelet agent (Grade 1A).
clopidogrel (75 mg daily) as monotherapy, or the combination
of aspirin plus ER-DP, rather than aspirin alone (Grade 2A)
50 to 100 mg daily (Grade 1B).
For patients having carotid endarterectomy, we recommend
aspirin (81 to 325 mg daily) started before surgery and
continued indefinitely in the absence of a contraindication
(Grade 1A).
Lipid lowering therapy initiation.
Blood pressure reduction, once the acute phase of ischemic
stroke has passed.
Smoking cessation along with management of obesity,
diabetes, and metabolic syndrome.
Cardioembolic stroke oral anticoagulation .
PREVENTION OF MEDICAL COMPLICATIONS. Aspiration
pneumonia UTI, DVT, Dehydration, Malnutrition, Pressure
sores, Orthopedic complications and contractures.
ICH

target BP = 160/90 mmHg using intermittent or


continuous IV medication. lowering by 15 % during the
first 24 hours. may be delayed until 24 hours after stroke
onset.
NOT be treated acutely unless SBP >220 mmHg or DBP
>120 mmHg, or the patient has active ischemic coronary
disease, heart failure, aortic dissection, hypertensive
encephalopathy.

Labetalol, nicardipine, enalapril, hydralazine, and


nitroglycerine are useful intravenous agents.
Cranial nerves
1. Olfactory N;
- Pathway; bipolar olf cells cribriform plate olf bulb->
temporal lobe.
- olfactory halucination Temporal lobe epilepsy.

2. Optic nerve;
- tested by;
1- visual acuity
2- visual field
3- light reflex
4- pupil size,
5- colour vision, fundus.
Optic neuritis central or paracentral scotoma
Papilledema--. Enlargement of blind spot
The pathway of the pupillary light reflex consists of:
Superior quandrantinopia is seen in damage to the temporal
part of optic radiation lesions.
Inferior quandrantinopia is seen in damage to the parietal
part of optic radiation lesions
Bitemporal hemianopia is seen in optic chiasmic lesions.

Contralateral homonymous hemianopia, is seen in any


lesion from optic chiasma till occipital cortex.
Contralateral homonymous hemianopia with macular
sparing is seen in .....Occipital cortex .
Optic neuropathy causes a central scotoma.
optic tract lesion incongruous homonymous hemianopia.
occipital lobe lesion congruous homonymous hemianopia

Causes of Central Scotoma optic neuritis & Papillitis.


The pathway of the pupillary light reflex consists of:
light reflex;
- Absent direct & consensual optic n lesion.
- Absent direct only or consensual only occulomotor n lesion.
Pupil size;
Causes of Miosis;
- Pontine Hge.
- Horners S.
- Drugs; opiates, pilocarpine,; organophosphates.
- Argyll Robertson pupil; bil small irregular pupil, does not react to light but
can accommodate, in DM, MS, Syphilis, autonomic neuropathy.
Causes of Mydriasis;
- Adies pupil; 70% in females, (+hyporeflexia esp ankle =Holmes Adies S)
- 3rd n palsy.
- Trauma; rupture sphincter pupillary.
- Drugs; antidepressant, amphetamine, CO, ethylene glycol.
3. Cr n 3,4,6; (eye movement)
Tested by;
- ocular motility (H)
- nystagmus
- ptosis
- accommodation reflex;
Causes of bilateral ophthalmoplegia;
1. Graves dis
2. Myathenia gravis, myositis.
3. GBS(Miller-Fisher S)Ataxia , ophthalmoplegia ,areflexia.
4. Basal meningitis as TB
5. Wernicks encephalopathy.
6. Midbrain tumour or infarction.
Causes of Internuclear ophthalmoplegia;
1- MS (commonest, may be bilateral)
2-other br stem lesions; vascular, trauma.
3- SLE, Miller-Fisher S, wernickes enceph.
4-drug overdose; barbiturate, phenytoin, amytriptyline.
Causes of impaired vertical conjugate gase;
1. Progressive supranuclear palsy (impaired down dev).
2. Parinaud S; dt midbrain lesion upgaze defect, does not react to
light but can accommodate.
3. Graves dis.
4. Myathenia gravis
3rd n palsy;
- C/P; ptosis, eye deviated down (SO4) & out (LR6),
dilated fixed pupil.
- Types; n n
3rd n palsy
ptosis
Central Peripheral
(at the nucleus) (nerve)
incomplete Complete
Bil ptosis unil ptosis

Medical= vascular
Surgical
e.g. DM, atherosclerosis
Dilated fixed pupil
- Causes; Pupillary sparing

1. DM, arteriosclerosis (pupillary sparing).


2. Orbital apex dis as graves, cellulitis, granuloma (+ 2, 3,4
n affection).
3. Cavernous sinus (+ 4,5,6 n affection).
4. Posterior communicating art aneurysm.
5. Uncal herniation; dt IC tension.
4th (trochlear) nerve palsy
Causes as 3rd. trauma (30%), DM (30%), tumour,
idiopathic. + whiplash injury dt dorsal brainstem exit
difficult looking down & in vertical diploplia.
Superior oblique paralysed. Diplopia ocular torticollis
(holding head tilted), eye cannot look down and in.
6th (abducens) nerve palsy Causes: as 3rd + IC
tension (false localizing sign) tumour, trauma to base of
skull, vascular. medial deviation of the eye Diplopia.
NB;
- constrictor pupillae supplied by parasympathetic through
3rd n.
- Dilator pupillae, sympathetic, through OPTIC N.
DISORDERS OF CONJUGATE GAZE
Tonic conjugate deviation to paralyzed
side Frontal Area 8. the patient will recover,
Tonic conjugate deviation to normal side
Pontine centre. Recovery is unusual.

(Internuclear ophth.)
Medial longitudinal fasciculus; Each medial longitudinal
fasciculus connects the 3 & 4TH nerve nuclei of one side to the
contralateral 6 -nerve nucleus & pontine centre.
- Interruption of one MLB prevents adduction of the eye on the
side of the lesion (Internuclear ophth.) & nystagmus in the
contralateral abducting eye..
Trigeminal nerve

(deep sens)

Motor nucleus

(touch)

(P&Temp)
Causes of 5th cr. Nerve lesion.
Nuclear or infranuclear ( LMNL).
Central. (Nuclear) Brain stem lesion(
infarction ,MS, syringobulbia ).
Peripheral.( infranuclear)
cerebellopontine angle lesions.(acoustic neuroma).
Apex of petrous bone. (petrositis...5th& 6th ns).
Cavernous sinus.(CST ,ICA aneurysm , pit. tums extension)
Peripheral branches. ( DM, HZV)
petrous osteitis (Gradenigo S) but is now more likely the result
of a meningioma or nasopharyngeal carcinoma of the petrous
apex{5 (maxil),6}.
The cavernous sinus thrombosis consists of variable
involvement of: oculomotor, trochlear, abducens, trigeminal
(ophthalmic and maxillary division) and oculo-sympathetic
nerves. Classic presentations are abrupt onset of unilateral
periorbital edema, headache, and (proptosis).
Ptosis, chemosis, cranial nelve palsies (III, IV, V, VI). Sixth
nerve palsy & Sensory deficits of the ophthalmic and
maxillary are common. impaired corneal reflex may be noted.
Papilledema, retinal hemorrhages, Fever, tachycardia
and sepsis may be present.
The cause is usually from a spreading infection in the nose,
sinuses, ears, or teeth. Staphylococcus aureus and
Streptococcus are often the associated bacteria.

The superior orbital fissure syndrome is similar to the


cavernous 3,4,5,6,
Facial n
1. M; facial expression,
stapedius.
2. Parasympath to lacrimal,
submandibular, subligual.
3. taste of ant 2/3 of tongue.
4. Somatic sensation from the
external auditory canal and
pinna.

NB; lesion proximal to internal


auditory meatus Motor&
taste sensation affected.
- lesion distal to internal
auditory meatus(e.g. in the
auditory canal as Bells
palsy)motor affection only
(taste is spared).
ANATOMY OF THE FACIAL N. The facial nerve is a mixed nerve, containing Fibers for
motor output to the facial muscles.
Parasympathetic fibers to the lacrimal, submandibular, and sublingual salivary glands.
Afferent fibers for taste from the anterior two thirds of the tongue .
Somatic afferents from the external auditory canal and pinna.
Causes
LMN :
Nuclear : Br. Stem ( infl., vas. , neoplastic ,
demylinating,deg.).
Infranuclear : Bell's palsy, otitis media , skull
fracture, foreceps injury, cerebellopontine angle
tumours, parotid tumour,HZV(Ramsay Hunts),GB$
UMN : stroke, tumour.
UMN lesion lower 1/2 face affected only; since upper facial structures
receive bilateral innervations, the emotional movement is preserved .

LMN lesion such as Bell's Palsy,all one side of face affected, absence of
both voluntary and spontaneous emotional movement .

C/P; facial weakness and droop. Ask to raise eyebrows, show teeth, puff
out cheeks, inability to close the eye, disappearance of the nasolabial fold,
and the mouth drawn to the non affected side. Decreased tearing,
hyperacusis, and/or loss of taste sensation on the anterior two-thirds of the
tongue which is used more as an indicator of severity than anatomical
diagnosis.
Bell's palsy

Facial palsy without other signs. Unknown cause?? viral. Peak age: 10-40y.
Presentation.
Usually sudden onset of unilateral facial paralysis; may be preceded by pain
around the ear. Other possible symptoms: facial numbness; noise
tolerance;+/- disturbed taste on the anterior part of the tongue.
Management
~70% recover completely; 13% have insignificant sequelae; the remainder
have permanent deficit. 85% improve in < 3wk.
reassure.many neurologists advocate the use of oral prednisolone(1mg/kg/d.
max 80mg/d.)and aciclovir (800mg 5x/d.for 5d. (though,no evidence base)
Bulbar & Pseudobulbar palsy(bulb=medulla)
Bulbar palsy Pseudobulbar palsy

LMNL of 9-12 cr ns Bilateral UMNLof 9-12 cr ns


(nuclear of infranuclear) (supranuclear)
Causes 1- Br stem; -MND

- infarction, - MS
- syringobulbia - Bil Stroke

- MND - Severe head injury

- Polio
2- Jugular foramen;
- nasopharyngeal carcinoma
- Fracture base.
C/P dysarthria (spastic Bulbar palsy)
Hoarseness Spastic dysarthria
Nasal regurgitation Hoarseness
Dysphagia Nasal regurgitation
Lost reflexes Dysphagia
Exaggerated jaw reflex
Preserved palatal 7
pharyngeal reflexes
Encephalitis
Herpes simplex encephalitis .
is associated with high signal in one or both temporal
lobes (limbic encephalitis).
Seizures are commonly present in HSE.

Herpes Simplex Virus type 1 is the causative virus

PCR for herpes simplex virus on CSF is specific test.


There is more inflammation around the temporal
lobe, and the EEG changes .
Acyclovir should be commenced upon clinical
suspicion as PCR result may take a few days.
MENINGITIS
Meningococcal meningitis: (meningogoccus = gram negative
diplococci). Immunisation is available against strains A&C however
strain B is the most often implicated in meningococcal meningitis in
the UK. Rifampicin , ciprofloxacin and ceftriaxone can be used for
prophylaxis. (young age)

Pneumococcal meningitis (adult) is caused by gram positive


diplococci Nerve deafness is likely, paralysis can occur and mortality
rate is high among the elderly. IV penicillin or ceftriaxone should be
given. Vancomycin and rifampicin can also be used. Dexamethasone
is indicated in early infection to prevent neurological sequelae.

Tuberculous meningitis: Symptoms are headache, vomiting,


photophobia, and fever. The duration of presenting symptoms may
vary from 1 day to 9 months (insidious onset). CSF typically shows
elevated protein level, markedly low glucose, and lymphocytes.
Treatment o (INH), (RIF), (PZA), and ethambutol (quadruple therapy).
Steroids are usually also indicated.
Viral meningitis: A combination of lymphocytosis,
normal or mildly elevated protein and normal glucose
suggests viral meningitis. Common causes include
enterovirus and mumps.
Listeria meningitis ( gram +ve bacilli, cheese, pate,
elderly, pregnant, immunocomprmised) is generally
associated with multiple cranial nerve deficits, particularly
of the VIth and VIIth nerves,(br. Stem involvement) as
well as hemiparesis, ataxia and respiratory abnormalities
TTT. High dose (eg 2g qds IV) ampicillin & gentamycin
are the treatment of choice. Or sulphoneamides
(cephalosporines are ineffective).
Aseptic meningitis
Aseptic meningitis refers to patients who have clinical and
laboratory evidence for meningeal inflammation with negative
routine bacterial cultures. The most common cause is
enterovirus.
Many causes, but usually viral. Other infections, medications & malignancy
Usually self-limited
CSF pleocytosis (monos), negative bacterial cultures.
MENINGITIS VERSUS ENCEPHALITIS The presence or
absence of normal brain function is the important
distinguishing feature between encephalitis and meningitis.
Important Indications for Steroids in Meningitis
To control the inflammation in acute bacterial meningitis

TB meningitis

Altered mental status or o signs of ICT.


Cryptococcus neoformans

Route; inhalation of fungal spores blood to the CNS, particularly in


patients with advanced AIDS.
Cryptococcal meningoencephalitis is the most frequently encountered
manifestation of cryptococcosis. Symptoms typically begin in an indolent
fashion, usually over a period of 1-2 weeks. The three most common
symptoms are fever , confusion, and headache.
Stiff neck, photophobia and papilloedema are seen in 25- 30% of patients.
Pulmonary cryptococcosis (pl effusion, cavitation, fibrosis) is rare.
Diagnosis;
1. CSF +ve cryptococcal polysaccharide antigen Ag testing (90% sensitive).
Culture will almost always establish the diagnosis (100% sensitive).
2. The CSF WBC count is typically low (<50/microL) with a mononuclear
predominance and the protein and glucose conc. are only slightly abnormal.
TTT;
1. Induction therapy -- amphotericin B + flucytosine for 2 weeks.
2. Maintenance therapy fluconazole for at least 4 months.
DD of chronic meningitis
Infectious.
1. TB.
2. $
3.cryptococcal
Non-infectious.
4.sarcoidosis.
5. Behcet.
6.Malignant.

CP ----Meningism & inc. ICT


DD..of acute meningitis.
Toxoplasmosis
- Neurological dis in Immunocompromized (neck
stiffness, headache, CSF; high protein).
- CT.....> ring enhancing lesions).
o TTT, pyremethamine & sulfadiazine,
if pregnant spiramycin.
Tetanus
manifested by uncontrolled spasms, due to the introduction
of Clostridium tetani toxin into tissues.
Skin punctures, contaminated wounds with soil, dust,
neurotoxin (tetanospasmin) which causes severe spasm----
painful muscle contraction and laryngeal spasm.
The toxin tetanospasmin doesn't cross the BBB.
absence of a wound does not exclude tetanus.
The incubation period is typically between 1-2 weeks.
Vaccination with tetanus toxoid is effective .
TTT:
Tetanus antitoxin.
Booster immunization to whom has been injured & received
the last immunization 10 years or more.
supportive along with penicillin, is still the most effective
treatment.
Clostridium Botulinum (Botulism)
is a paralytic disease caused by the neurotoxins of
Clostridium botulinum which interferes with presynaptic
Ach release, causing the 6Ds Dilated fixed pupil;
Diplopia; Dysphagia; Dry tongue; Descending paralysis.
It may be acquired by ingestion of toxin or more
commonly in the UK wound infection as a result of
intravenous drug use. associated with trauma, surgery,
subcutaneous heroin injection
typically produces a descending paralysis which
starts with diplopia or blurred vision due to cr.nr.
progresses to involve muscles of the neck, arms, and
legs.. The botulinum toxin assay confirm the diagnosis

Therapy consists of approximately 10,000 IU of


antibodies against toxin types A, B, and E to
neutralize serum toxin concentrations, and also
supportive care (e.g. ventilation).(antitoxin)
Lyme disease.
Coma definition
Coma is defined as a sleeplike state with total absence of
awareness of self and the environment, even after vigorous
external stimulation.
Coma results from one of two
pathophysiologic mechanisms: a diffuse
insult to both cerebral hemispheres or a
focal lesion involving the ascending
reticular activating system (ARAS) located
in the upper pons & midbrain.
Metabolic coma
CH.CH.:
1. Reactive pupils.
2. Absence of focal neurologic signs.
3. No evidence of increased intracranial pressure.
AW

AR
coma
Structural
metabolic
or
surgical or
medical
Decorticate rigidity (flexion &
adduction of arms, extension of legs
supratentorial lesion.
Decerebrate rigidity (extention of neck,
back, arms, internal rotated flexed
wrists) pontine.
Vegetative form (awake coma)
global damage of CC.
. Glasgow Coma Scale
Eye opening (E) Score
Spontaneous 4
To speech 3
To pain 2
No response 1
Best motor response (M)
Obeys 6
Localizes 5
Withdraws 4
Abnormal flexion posturing 3
Extension posturing 2
No response 1
Verbal response (V)
Oriented 5
Confused conversation 4
Inappropriate words 3
Incomprehensible sounds 2
No response 1
Total score = E + M + V _______
Range of possible scores = 3-15
A score of 13 to 15 indicates mild coma. A score between 9 and 12 points to
moderate coma, and a score of 8 or less indicates severe coma.
Weakness of small ms of the
hand;

- T1 root e.g. cervical rib, pancoat,


sub art aneurysm. all small ms
of the hand.

- lower trunk of Br plexus injury


all small ms of the hand + long
flex & ext of fingers (C8).

- ulnar n all small ms of the


hand except 2 lateral lumbricals &
thenar group (T1 through median
n).
Peroneal neuropathy usually presents with acute foot
drop. The foot and ankle weakness on neurological
examination is restricted to ankle and toe
dorsiflexion and ankle eversion. Ankle reflex (Tibial
nerve mediated) and knee reflex (Femoral nerve
mediated) are intact. Sensory involvement may
include the lower two thirds of the lateral leg and
dorsum of foot.
Retinal vein occlusion.
, there is rapid visual loss in one eye, the fundus looks like a stormy sunset
(haemorrhages and engorged veins). Branch retinal vein occlusion causes partial visual
loss. Causes :glaucoma, arteriosclerosis, BP, polycythaemia, hypercholesterolaemia,
homocystein.
Retinal artery occlusion.
Usually due to thromboembolism .Signs :Sudden visual loss in 1 eye (counting fingers or
light perception), afferent pupil defect . Retina appears white cherry red spot at the
macular.
Treatment
If the patient presents <1h. after onsetapplying then releasing firm eyeball pressure can
sometimes dislodge an embolus into one of the smaller branches and thus preserve some
vision. Refer for confirmation of diagnosis. There is no reliable treatment
Vitreous haemorrhage.
Signs :sudden in vision, loss of red reflex, difficulty visualizing the retina .Risk factors :
DM with new vessel formation, bleeding disorders, retinal detachment, central retinal vein
occlusion, trauma.
Management
Treat the cause (e.g. photocoagulation of new vessels). Most bleeds disperse
spontaneously in <3mo. Vitrectomy may be required to remove persisting blood.
Optic neuritis.
Presentation :loss of vision (over hours or days); painful eye movements; colour
discrimination (red desaturation); optic disc swelling (papilloedema). Recovery usually
occurs over 2-6wk. but ~50% develop multiple sclerosis) .
Treatment is with high-dose steroids.
Other causes of sudden loss of vision. Migraine ,Stroke , Amaurosis fugax , Temporal
arteritis .
Papilloedema.
Swollen optic disc with blurred disc margins. Can be due
to ICP , malignant hypertension , cavernous sinus
thrombosis or optic neuritis.
Optic atrophy.
Signs: gradual visual loss; pale optic disc.
Causes: glaucoma, MS, ischaemia (e.g. retinal artery
occlusion), retinal damage (choroiditis, retinitis
pigmentosa), toxic (tobacco ambylopia,methanol, arsenic)
Background retinopathy
Microaneurysms (dots), microhaemorrhages
(small blots), hard exudates not affecting the
macula
Venous abnormalities, large blot
haemorrhages, cotton wool spots (small
infarcts) Leakage in macular region
macular oedema,
Proliferative retinopathy though complications
cause visual loss. New vessel formation
either at the disc (NVD) or elsewhere.
passmedicine.com Page 1 of 1

passmedicine.com
Reference ranges Return to session summary

Acute coronary syndrome: prognostic factors

sqweqwesf erwrewfsdfs adasd dhe


The 2006 Global Registry of Acute Coronary Events (GRACE) study has been used
to derive regression models to predict death in hospital and death after discharge in
patients with acute coronary syndrome

Poor prognostic factors

age
development (or history) of heart failure
peripheral vascular disease
reduced systolic blood pressure
Killip class*
initial serum creatinine concentration
elevated initial cardiac markers
cardiac arrest on admission
ST segment deviation

*Killip class - system used to stratify risk post myocardial infarction

Killip class Features 30 day mortality


I No clinical signs heart failure 6%
II Lung crackles, S3 17%
III Frank pulmonary oedema 38%
IV Cardiogenic shock 81%
Comment on these notes

mhtml:file://D:\books\last resources\mrcp\MRCP\MRCP\PassTest Note 2011\Cardiology\... 8/13/2014


psychiatry
Anxiety disorders
Generalized anxiety disorder GAD
Panic disorder
Phobic Disorders
Post-traumatic stress disorder (PTSD)
Mixed anxiety and depression
Somatization disorder
Obsessive-compulsive disorder (OCD)
Generalized anxiety disorder
(GAD)
Long-term condition, fluctuating in severity
and nature, often beginning In adolescence.
Lifetime prevalence 5%.
Clinical features
Psychological Physical
Apprehension Dry mouth
Irritability Tremor
Dizziness Headache
Poor concentration Parasthesiae
Worrying thoughts Tinnitus
Insomnia. Frequent or loose motions
Nightmares. Chest discomfort
Depression. Difficulty breathing/ hyperventilation
Obsessions. Palpitations
Frequency or urgency of micturition
Erectile dysfunction
Menstrual problems
Associations
- Other psychiatric illness :Panic attacks; depression; and alcohol
dependence.
- Physical illness :Thyrotoxicosis; hypoglycemia; chronic fatigue
syndrome; stroke; Cushing's disease; phaeochromocytoma.

Management.
1. pharmacological; SSRIs(e.g. paroxetine 20 - 40mg, for 12
wk, if effective continue for 6mo.), BDZ e.g. diazepam 2 -
5mg tds prn not >2-4wk, BB , TCA , buspirone (5-HT1A
partial agonist)
2. Psychological = CBT.
3. General measures :Check TFTs. Avoid caffeine.
4. Refer for specialist if significant symptoms despite
treatment with 2 interventions (CBT, medication) or if
considering initiation of venlafaxine.
Panic disorder (non situational)
Panic attacks are very common, but panic disorder is uncommon.
Intense feeling of apprehension or impending disaster. Anxiety builds up
quickly and unexpectedly without a recognizable trigger and
patients often present with any combination of these physical symptoms:
Shortness of breath, Choking.
Palpitations, Chest pain
Sweating
Dizziness
Nausea or abdominal pain
Numbness or tingling sensations
Flushes or chills,tremors
Fear of dying
Panic attack: 4 symptoms listed above in 1 attack.
Panic disorder: > 4 attacks in 4wk. or 1 attack followed by a persistent
fear of having another. TTT; SSRIs (paroxetine and citalopram 10mg). For
6mo., then discontinue slowly over 4-6mo.TCA (imipramine, clomipramine).
Don't use BDZless good outcome in the long term.
CBT.
Phobias (Situational but out of proportion).
Same symptoms as GAD but limited to certain situations & fear out of
proportion to the situation .
2 main features:
1. Avoidance of the circumstances that provoke anxiety.
2. Anticipatory anxiety when there is prospect of encountering these
circumstances.
Simple phobia.
anxiety in the presence of a specific object/situation e.g. flying, enclosed
spaces, spiders.
Social phobia.
fear and avoidance of social situations (e.g. meeting people in authority,
using a telephone, speaking in front of a group).
Agoraphobia.
fear in crowds, away from home, or in situations from which escape is
difficult.
TTT;
1. Behavioural therapy e.g. exposure, desensitization.
2. Drug treatment :SSRIs (citalopram and paroxetine are licensed); TCAs
(imipramine and clomipramine are commonly used).
Post-traumatic stress disorder (PTSD)
is a delayed and/or protracted response to an
exceptionally stressful event e.g. major accident, fire,
military combat.
3 main symptom clusters:
1. Intrusive recollections of thoughts, nightmares,
flashbacks;
2. Avoidance behaviour.
3. anxiety and irritability, insomnia, poor concentration.
TTT;
CBT,
SSRIs reserve for those with continuing symptoms
despite trauma-focused psychological therapy or who
have refused therapy for those with continuing
symptoms.
Obsessive-compulsive disorder
(OCD)
Features
Obsessional thinking; recurrent persistent
thoughts causing anxiety or distress.
Compulsive behaviour; repetitive behaviours,
rituals done to prevent the anxiety.
Treatment ; patient education, SSRI or
clomipramine, and CBT.
Unexplained physical symptoms
1. Somatoform disorders; (Somatization).
>2y history of multiple symptoms with no adequate physical explanation.
persistent requests for medical investigations & -ve results.
Reassurance have little effects.
Pt refuses the possibility of psychological cause.
2. Hypochondrial disorder.
Persistent belief of the presence of one serious illness e.g. cancer or AIDS.
Management; acknowledging that the symptoms exist and taking the
symptoms seriously, offering any necessary examination and investigations,
enquiring about psychosocial problems, and explaining the link between
symptoms and stress.
Treat comorbid psychiatric problems (e.g. depression, anxiety, panic).

Senario: A professional man aged 50, previously well, cannot rid himself of
the conviction that he has bowel cancer despite reassurance that no
disease has been discovered with full investigations.
Conversion disorder;
= intolerable psychic stress is
unconsciously converted to physical
symptoms with loss of physical function.
1ry gain= reduction of the stress.
2ry gain= attention of others.
TTT; psychotherapy.
Munchausen syndrome (factitious disorder)
Intentional production of physical or psychological symptoms to
assume the sick role ( hospital admission, willing to receive invasive
procedure).
Can be difficult to detect.
Differs from malingering as there is no external reward (e.g.
financial), no convincing secondary gain (occasionally said to be
due to attention seeking behaviour). may be harmed.
Associated with personality disorder.

Common presentations;
1. Physical :Dermatitis artefacta, bruising disorders, diarrhoea of
unknown cause, neurological symptoms e.g. pseudoparalysis or
pseudofits (neurologica diabolica), abdominal pain
(laparotomophilia migrans), chest pain
2. Psychological :psychosis, factitious bereavement, factitious
overdose.

TTT; psychotherapy, Often difficult to detect and treat.


Malingering
Intentional production of physical or psychological symptoms to
assume the sick role for a known external purpose.
Malingering is not considered mental illness or psychopathology,

Common motivating factors


1. Avoidance of going to jail or release from jail.
2. Avoidance of work.
3. Avoidance of family responsibility
4. Desire to obtain narcotics.
5. Desire to be awarded money .
6. Need for attention

.
Forms::
1. Pure malingering; the individual falsifies all symptoms.
2. Partial malingering; the individual has symptoms but exaggerates
the impact they have upon daily functioning.
Management
Difficult. As doctors we tend to believe our patients.
Exclude causes for the presenting symptoms through
careful history/examination.
Avoid prescribing drugs for symptoms and unnecessary
referrals as these might perpetuate symptoms.
Avoid certifying the patient as unfit to work or perform
activities; if the patient is unhappy about this, suggest a
second opinion..
Differential diagnosis
1. True medical or psychiatric illness yet to be diagnosed
2. Somatization disorder (unconsciously)
3. Factitious disorder/Munchausen syndrome (severe
personality disorder, consciously, but no gain)
4. Malingering (consciously, with gain)
Deliberate self-harm (DSH)
Self-harm is often aimed at changing a situation (e.g. to get
a boyfriend back) a sign of emotional distress, or may be a
failed genuine suicide attempt.
90% DSH is due to self-poisoning and it accounts for 20% of
admissions to general medical wards.
Paracetamol or aspirin are the most common drugs used.
Management ;
1. Assess the situation and admit to A&E as necessary.
2. Ask about present circumstances: Ask the patient to explain
his feelings and reasons for the act of DSH/attempted
suicide.
3. Assess psychiatric state: Depression, agitation, early
schizophrenia
4. Refer for psychiatric assessment
Assessment of suicide risk
Ask about suicidal ideas and plans. It is a common
misconception that asking about suicide can plant the
idea into a patient's head. Evidence is to the contrary.

Risk factors for suicide


1. male> female; 3:1.
2. Age 40-60 y.
3. Living alone.
4. Divorced > widowed > single > married.
5. Unemployment.
6. Chronic physical illness.
7. Past psychiatric history.
8. Recent admission to psychiatric hospital
9. History of suicide attempt/ self-harm.
10. Alcohol/drug misuse.
suicide Non-fatal Self-harm

sex

age 40-60 <35

Physical illness + -
(handicap, pain,
terminal illness)
Psychiatric Depression 60%, Depression 10%
illness Alcohol 20%
Premorbid good Antisocial,
Personality borderline
setting Planed , alone. Impulsive, with
others
precipitants Guilt , hopeless. situational
Mood (affective) disorders
Depression
Risk factors ( Depression is commoner in women and in urban areas.
1. Children < 14 yrs living at home, &Adolescents.
2. Lack of employment outside the home.
3. lack of a confidant.
C/P;
1. Mood; lowered, worse in am (diurnal variation)
2. Thought & speech; slow, poor attention, pessimistic, self
esteem, self harm.
3. Behavior, insomnia (early morning awakening), loss of libido,
appetite, wt, enjoyment (anhedonia).
4. Psychotic symptoms; mood congruent, delusion of guilt,
auditory hallucinations.
Bipolar disorder or manic depression
Consists of episodes when the patient has mania (bipolar I) or hypomania but no
manic episodes (bipolar II) against a background of depression. Lifetime prevalence
[approximate, equals]1%.

In dysthymic disorder, the patients mood is chronic depression with never a manic or
hypomanic episode, for at least two years.

Sodium valproate and carbamazepine are efficacious as first line treatment in the
prophylaxis of manic and depressive episodes in bipolar I disorder. Lithium may be
used if these anticonvulsants are ineffective. However, in the initial stages of manic
episodes, the addition of drugs with potent sedative effects are often required e.g.
clonazepam, lorazepam and haloperidol. These drugs can be tapered and then
discontinued as soon as the initial phase of the manic episode has subsided and the
effects of the anticonvulsants or lithium are seen clinically.

Increased risk of suicide in depression if


social isolation- lives alone
elderly males
terminal medical illness.
recent bereavement
unemployed
alcohol intake
Causes /comorbidity
1. Psychiatric disorders e.g. anxiety disorders,
alcohol abuse, substance abuse, eating
disorders.
2. Physical disorders e.g. parkinson, MS,
dementia, endocrine disease (thyroid
disorders, Addison's disease ,
hypercalcaemia), rheumatoid arthritis, SLE,
cancer, AIDS and other chronic infections,
3. Drugs ;B blockers , anticonvulsants,
corticosteroids , OCP , antipsychotic drugs,
drugs used for parkinson (e.g. Levodopa).
Diagnosis
depressed mood and/or lack of interest or pleasure,
which must be disabling to the patient .+ 5
symptoms from following list present most of the
time for 2wk:.
Change in appetite or weight (Poor appetite or
overeating)
Insomnia or hypersomnia.
Fatigue or loss of energy.
Poor concentration or difficulty making decisions.
Low self-esteem.
Sense of worthlessness or guilt.
Recurrent thoughts of death or suicide.
Feelings of hopelessness.
TTT;
pharmacological; The suicide risk can increase early in antidepressant therapy.
- antideppressants; Major groups are:
Selective serotonin re-uptake inhibitors (SSRIs) e.g. fluoxetine 20mg is usually 1st
choice as less likely to be discontinued due to side-effects. Gl side-effects including dyspepsia
are common
.Withdrawal of SSRIs; headache, nausea, paraesthesia, dizziness, and anxiety.

venlafaxine
Tricyclic antidepressants (TCAs)e.g. lopramine 70mg od/bd/tds, titrate dose up.
Common side-effects include drowsiness, dry mouth, blurred vision, constipation, urinary
retention, and sweating.

Monoamine oxidase inhibitors (MAOIs) e.g. phenelzine 15mg tds.(drug ineraction).


MAOIs should not be started until at least 1-2wk. after a tricyclic has been stopped. Other
antidepressants should not be started for 2wk. after treatment with MAOIs has been stopped
.Withdrawal of MAOIs; nausea, vomiting, anorexia, headache, chills, insomnia, anxiety/panic,
and restlessness.
In patients with recurrent relapses, antidepressants should be continued indefinitely.
- ECT; Severe depression (melancholia) is the most frequent indication for ECT. Patients
experience sadness and despair,, lose appetite and weight, sl ,and often think of suicide.
hazardous effects of ECT: partial loss of memory events over weeks following ECT, cardiac
arrhythmia and minor musculoskeletal trauma.
Contraindications; ICT, CVA, cardiopulmonary dis.

Psychological = cognitive behavioral therapy.


Social; for adaptation for the patient & relatives.
Mania;
usually part of manic depressive (bipolar);
Bipolar disorder is equally common in men and
women. Symptoms are evident before age 30 years,
Hypomania is a mild form of mania, and can
occur in bipolar disorder.
Olanzepine or benzodiazepines are more
effective during episodes of hypomania
C/P;
1. Mood; elated
2. Thought & speech; fast, Pressure of speech and
flight of ideas (quick succession of thoughts) poor
attention, optimistic, grandiose.
3. Behavior; over activity, insomnia, loss of sexual
inhibition, appetite, wt.
4. Psychotic symptoms; mood congruent. Auditory H
TTT;
During acute episode; Acute management
Treatment in hospital is usually required for the 1st episode or acute relapses. If
unwilling to accept voluntary admission, use compulsory admission .
use chlorpromazine 50-100mg po or 50mg im
- antipsychotics; clozapine, Olanzepine or BDZ are more effective during episodes
of hypomania.
- lithium, (plasma level=1mmol/l)
- ECT.
Long term;
- depot antipsychotics,
- anticonvulsants; Valproate, carbamazipine, lamotrigine,
- lithium (plasma level=0.5 mmol/l) is the drug of choice.
Check levels weekly until the dose is constant for 4wk., then monthly for 6mo., then
every 3mo.
Check plasma creatinine and TFTs every 6mo..
Toxic effects :blurred vision, D&V, K+, drowsiness, ataxia, coarse tremor, dysarthria,
fits, psychosis, coma, and shock.
Grief reaction
Normal bereavement can manifest as intense
symptoms that subside slowly .
Grief reaction symptoms of depression & thoughts
about dying . Symptoms last for up to 6 months.
suicidal thoughts 4 weeks after bereavement.
feeling that the dead person is still present 1 month after
their death.
absence from work after bereavement usually no more
than 2 weeks .
Feelings of hopelessness, guilt and worthlessness may
signify depression rather than grief reaction.
Depression in elderly;
C/P; physical symptoms more than
depressed mood, insomnia, agitation,
cognitive impairement (can present with
features suggestive of dementia)
. DD; dementia.
Differentiation of depression
from dementia
depression dementia

Onset,course Acute, rapid, short Gradual, slow, long


duration
C/O of memory loss yes no
O/E Detailed history giving, Vague history
poor effort at testing, Good
pick on faults, Pleased
global memory loss, Recent
variable concentration Consistently poor
+ve sympt of depression Cortical neuro deficit;
apraxias,
Post partum psychotic disorders
1. PP blues;
- tears
- 3-4 days after delivery.
- primigravada.
- TTT; reassurance.
- good prognosis.

2. PP depression;
- depression
- 2 weeks after delivery.
- young age.
- marital conflicts.

3. Puerperal psychosis
- psychosis
- 2 weeks after delivery, usually acute onset.
- is more common primigravada, older age, CS.
- The prognosis is good,
- recurrence rate = 14-20%.
- past history or FH of psychosis.
- Treatment is with antipsychotics (be cautious of breast feeding) or ECT.
Puerperal psychosis

A usually begins after the second week of


the puerperium
B often takes the form of schizophrenia.
C recurrence of perperal psychosis in
subsequent pregnancies is 15 - 20 % .
D the onset is usually acute .
E the prognosis is usually good.
Schizophrenia
Definition; disorder ccc by disturbance of
- thought; loss of insight
- mood; incongruous
- perception; delusions
- personality.
Epidemiology; prevalence= 1%, early onset in men, genetic
predisposition.
Symptoms;
1. +ve Symptoms; delusion, hallucinations.
2. -ve Symptoms; flat affect, motor (catatonia), speech, self care.
Diagnostic criteria;(1st rank Symptoms)
(ATPD) (aim to pass definitely)
1. Auditory hallucinations; 3rd person (hearing 2 persons speaking about
the patient).
2. Passivity of Thoughts; his Thoughts are under external control;
withdrawal, insertion, broadcasting.
3. Passivity of body experiences; his actions, feelings are under external
control.
4. Delusional perception; self referential delusions
The 4 main subtypes of schizophrenia are;
1. catatonic, describes patients whose clinical presentation is
dominated by profound changes in motor activity, negativism.
2. paranoid, describes patients who have a prominent preoccupation
with a specific delusional symptoms.
3. disorganized, in which disorganized speech and behavior are
accompanied by a superficial or silly affect.
4. residual. negative symptomatology exists in the absence of
delusions, hallucinations, or motor disturbance.

schizoaffective disorder is used for those who manifest symptoms of


schizophrenia and independent periods of mood disturbance.
Good prognostic criteria;
- Old age, good premorbid, married.
- acute onset, +ve precipitant , +ve symptoms.
- Prominent mood disorder, FH of mood disorder.

Bad prognostic criteria;


- young, male, single, low IQ, abn premorbid personality.
- Insiduous onset, -ve precipitant, -ve symptoms.
- Neurological signs, FH of Schizophrenia.

TTT;
1. pharmacological;
A) Atypical antipsychotic; as clozapine, olansapine, resperidone
less side effects as extra, cardiac, more effective for ve
symptoms.
B) traditional antipsychotic; chlorpromazine, haloperidol, trifluoperazine
tardive dyskinesia, cardiac, more effective for +ve symptoms.
electrocompulsive therapy (ECT) for catatonic stupor.
2. Psychological = cognitive behavioral therapy.
3. Social; for adaptation for the patient & relatives.
Acute schizophrenia
Typically presents in young people with +ve
symptoms (delusions, hallucinations, and/or
thought disorder) delusions of reference,
blunt affect, auditory hallucinations, Olfactory
or gustatory hallucinations, persecutory
delusions, suspiciousness, hostility, social
withdrawal and thought echo .
The patient lacks insight.
Organic Psychiatry
organic brain disorder can mimic other functional mental disorders.
Features that raise the possibility of organic disorder;
1. Visual perceptual abnormalities e.g. hallucinations,
2. Cognitive deficit.
3. Neurological signs.
4. Fluctuating symptoms.

Acute confusional states (delirium)

Delirium or acute confusional state is a transient global disorder of


cognition. Delirium is defined as a transient, usually reversible, cause of
cerebral dysfunction and manifests clinically with a wide range of
neuropsychiatric abnormalities.
Common condition seen in general practice, particularly amongst elderly patients due
to break of the BBB.
Presentation
Global cognitive deficit with onset over hours/days.
Fluctuating conscious level typically worse at night/late afternoon
Impaired memory on recovery, amnesia of the events is usual
Disorientation in time and place
Odd behaviour may be underactive, drowsy, and/or withdrawn or hyperactive and
agitated
Possible causes
Infection ; particularly UTI, pneumonia. Rarely encephalitis, meningitis.
Drugs; opiates, sedatives, L-dopa, anticonvulsants, recreational drugs,
Alcohol or drug withdrawal.
Metabolic; hypoglycaemia, uraemia, liver failure, hypercalcaemia.
Hypoxia e.g. severe pneumonia, exacerbation of COPD, cardiac failure.
Cardiovascular; Ml, stroke, TIA.
Intracranial lesion; SOL, ICP, head injury (especially subdural haematoma).
Thyroid disease.
Carcinomatosis.
Epilepsy; temporal lobe epilepsy, post-ictal state.
Differential diagnosis
Dementia; longer history and lack of fluctuations in conscious level.
Primary mental illness e.g. schizophrenia
Physical diseases assiciated with psychiatric
disorder; (depression, dementia)
1. Neurologic; Parkinson, Huntingtons chorea, syphilis,
epilepsy, MS, wilsons, prion, br tumour, MG, MND.
2. Endocrine; cushing, addison, TH, pheochromocytoma.
3. Other systemic diseases; SLE, B12 def, porphyria,
paraneoplastic.

4)
Chronic alcohol abuse
Chronic alcohol abuse is associated with
parotid gland enlargement
gynaecomastia c-and loss of body hair and testicular atrophy
particularly common in alcoholic liver disease
dementia
macrocytosis
atrial fibrillation
hypertriglycerdiaemia
neurological features
intoxication, coma, delirium tremens, alcoholic fits, alcoholic
amblyopia, myopathy, painful peripheral neuropathy, severe
rhabdomyolsis, subdural haematoma
cerebellar degeneration& ataxia
Wernicke-Korsakoff syndrome,
central pontine myelinolysis
Other notable effects -cardiomyopathy, hypoglycaemia,Chronic
pancreatitis,
Alcohol withdrawal
Delirium Tremens
cocaine
Cocaine is also known as "ice."
Through central effects cocaine induces
sweating, pyrexia and also adrenergic
mediated hypertension.
It may also be responsible for
coronary(chest pain) and cerebral artery
spasm causing infarction .
The classic withdrawal syndrome for heroin appears within four to 12 hours,
peaks at 48 to 72 hours, and subsides.
There is often a period of several hours before frank withdrawal symptoms
begin, during which the addict becomes agitated and anxious.
Characteristic withdrawal symptoms include aching muscles and joints,
dysphoria, insomnia, agitation, diarrhoea, shivering, yawning, and fatigue.
More objective measures include
Tachycardia
Hypertension
Lacrimation
Rhinorrhoea
Dilated pupils, and
'Insomnia (with increase in REM sleep)
Opiate withdrawal is not usually life threatening
Methadone is a synthetic orally effective opiate with a longer half life than
heroin (24 to 36 h, making it suitable for daily administration. It is the
mainstay of treatment for heroin dependency in the Western world.
Methadone may be used as substitute opiate drug, prescribed long term with
the aim of achieving stable (non-injecting) opiate dependence (methadone
maintenance) or it may be prescribed in the short term to aid withdrawal.
symptomatic relief may be given with clonidine and benzodiazepines. Opiate
withdrawal is not usually considered to be life threatening.
Cannabis abuse is associated with an amotivational state.
Methamphetamine is a sympathomimetic with a variety of stimulant,
anorexiant, euphoric, and hallucinogenic effects.

Clinicians should consider the diagnosis of methamphetamine intoxication


in any diaphoretic patient with hypertension, tachycardia, severe agitation,
and psychosis.

Agitation, tachycardia, hypertension, and psychosis are among the most


frequent findings.

Patients with methamphetamine intoxication range from the virtually


asymptomatic to those in sympathomimetic crisis with imminent
cardiovascular collapse. Methamphetamine can cause a host of
respiratory, cardiovascular, psychiatric .
The differential diagnosis includes a poisonings and medical conditions
with characteristics of the sympathomimetic signs of adrenergic excess.
We suggest severely intoxicated patients be treated immediately with
parenteral benzodiazepines (Grade 2B). initial doses include lorazepam
4 mg IV or diazepam 5 to 10 mg IV. These doses can be repeated every
eight to ten minutes based on patient response;
IV administration is strongly preferred; i (IM) injection may be used initially
when IV access is unavailable.
Atypical antipsychotic agents are used as adjuvant therapy.
Amphetamine dependence drug induced Schizophrenia.
Dementia
Definition; global deterioration of higher mental functioning 2ry to progressive
neurodegenerative disease.

C/P;(fully conscious)
1. Episodic memory loss
2. Deterioration of self care skills
3. Apraxia
4. Temporal & topographic disorientation
5. Personality changes.

Causes;
1. Alzheimers dis;
2. Pick disease;
3. Lewy body;
4. Vascular =Multiinfarct;
5. Normal pressure hydrocephalus;
6. Progressive supranuclear palsy; (PSP)
7. Multiple system atrophy
8. Huntington Chorea;
9. Progressive multifocal leukoencephalopathy
10. Creutzfeldt-Jacob dis;
11. Chronic Subdural hematoma.
Alzheimers dis Multiinfarct
onset gradual acute
course Slowly progressive stepwise
cognitive diffuse patchy
impairment
insight - +
personality Lost early preserved
Focal neuro sign, - +
seizures
Previous TIA, stroke, - +
HTN, IsHD
Time of death Early 2-5 ys Later 4-5 yrs
Sleep disorders
Narcolepsy;
is a clinical syndrome of;
1. daytime sleepiness,
2. cataplexy, emotionally-triggered, transient muscle weakness. Most
episodes are triggered by strong, generally positive emotions such as
laughter, joking, or excitement, result from loss of the neuropeptides
orexin-A and orexin-B. TTT; a REM sleep-suppressing medication (eg,
venlafaxine, clomipramine, fluoxetine) .
3. hypnagogic hallucinations, are vivid, often frightening hallucinations
that occur just as the patient is falling asleep or upon awakening.
These are not due to psychiatric disease, but probably result from a
mixture of REM sleep dreaming and wakefulness.
4. sleep paralysis; is a complete inability to move for one or two minutes
immediately after awakening.

Only about 1/3 of patients will have all four symptoms; thus, the
diagnosis of narcolepsy should be considered even in patients with
sleepiness alone.
Diagnostic testing for narcolepsy includes an overnight
polysomnogram (PSG) followed by a Multiple Sleep
Latency Test (MSLT).
In Narcolepsy, REM sleep occur at the onset of
nocturnal sleep. Daytime attacks also consist of periods
of REM sleep ocuring out of context.
Secondary Narcolepsy can occur with rare lesions of the
posterior hypothalamus and midbrain. Tumors, vascular
malformations, strokes, Prader-Willi syndrome,, and
paraneoplastic syndromes. All patients with secondary
narcolepsy have obvious neurologic deficits, with
cognitive, motor, and/or eye movement abnormalities .
TTT of narcolepsy;
1. Creating a sleep time table & alcohol intake.
2. CNS stimulant as modafinil, methylphenidate or
amphetamines.
TTT of Hyperkinetic attention deficit
methylphenidate.
TTT of restless leg S correction of anemia,
bromocriptine.
Eating disorders
Target groups for screening for eating disorders;
1. Young women with low BMI compared with age.
2. Patients consulting with weight concerns who are not
overweight.
3. Women with menstrual disturbances or amenorrhoea.
4. Patients with symptoms/signs of starvation, sensitivity to cold,
constipation, low BP, bradycardia, hypothermia.
5. Patients with physical signs of repeated vomiting, pitted teeth ,
dental caries, general weakness, cardiac arrythmias.
Screen target populations with simple screening questions
1. Do you worry excessively about your weight?
2. Do you think you have an eating problem?
Anorexia nervosa
Prevalence 0.02- 0.04%. >. Usually begins in
adolescence. Peak prevalence at 16-17y .
Diagnostic criteria:
1. Self induced loss of wt (>15% BW) avoid eating,
vomiting, exercise or laxative abuse.
2. nervosa psychopathology= overvalued idea that fatness
is a dreadful state. Disturbed experience of body weight
or shape.
3. 2ry endocrinal disturbance; delayed puberty,
Amenorrhea, loss of sexual interest, 2ry sexual ccc.
4. Other features; Depression and social withdrawal are
common.
Prognosis; poor, mortality 20%, full recovery only 20%.
Treatment involves psychotherapy, and possible
admission for refeeding.
Features of anorexia nervosa are lanugo hair
(fine hair on the back, arms and face)
Metabolic effects in anorexia nervosa
metabolic alkalosis, elevated plasma amylase,
hypercholesterolaemia, hypoglycaemia, high
cortisol, hyponatraemia and hypokalaemia,
impaired glucose tolerance, beta-hydroxybutyric
acid increase, protein deficiency, zinc deficiency.
QT
Ht size
Relative lymphocytosis

Neuro; seizures, PN, AN, EEG changes


Bulimia nervosa
Prevalence 1-2%. Mainly aged 16-40y.
Diagnostic criteria :
1. Recurrent episodes of binge eating, far beyond
normally accepted amounts of food.
2. Inappropriate compensatory behaviour to prevent
weight gain e.g. vomiting; use of laxatives,
diuretics, and/or appetite suppressants.
3. nervosa psychopathology.
4. Normal menses and normal weight. If low BMI,
classified as anorexia.

TTT; psychotherapy, Antidepressant medication;


fluoxetine 60mg od is the drug of choice.
Some Memory disorders
Frontal lobe damage poor memory, perseveration (repeating
behavior), personality change, disinhibition, euphoria, apathy,
aphasia, reduced activity,, inability to plan ahead.

Transient Global Amnesia or TGA


- sudden confusion & Antegrade & retrograde amnesia for recent up
to years memories..
- The attack lasts for hours, sometimes a whole day.
- preceded by physical or emotional stress, for example hot baths,
cold swims and personal dramas.
- Despite patchy memory loss about the recent past, distant
memories remain.
- Self identification is preserved.
- The lost memories come back, but events that took place during
TGA are never recalled.
The underlying cause is proposed to be vascular insufficiency to the
memory areas - hippocampi.

Korsakoffs psychosis is typically associated with short term


memory loss and then confabulation by patient when he is unable to
accurately describe something.
chronic fatigue syndrome
Diagnostic criteria are :
1) severe chronic fatigue of six months or longer duration
2) Have four or more of the following symptoms:
- substantial impairment in short-term memory
- poor concentration
- sore throat
- tender lymph nodes
- muscle pain
- multi-joint pain without swelling redness
- headaches of a new type pattern severity
- unrefreshing sleep
- post-exertional malaise lasting more than 24 hours

Treatment; CBT, Low dose antidepressants.


IGT, DKA
Antidepressants

Indications;
1. Depression
2. Anxiety disorders; panic, phobic
3. Obsessional illness.
Antidepressants
SIADH

BP

BP
(Selective serot & NE reuptake inhibitors)

(Reversible inhibitor of monoamine oxidase A)

Has both NE & 5HT activiry, used if SSRI is ineffective or poorly tolerated
Benzodiazepines
Indications;
for short term relief of;
1. severe anxiety( only for 2-4 wks),
2. agitation in acute psychosis or mania.
3. Disabling insomnia.

BDZ toxicity hypersalivation.


BDZ withdrawal syndrome insomnia, perspiration, anxiety, tremors,
tinnitus, appetite, wt &perceptional disturbance auditory
hallucinations only.
BDZ suppress REM sleep, so BDZ withdrawal REM sleep rebound
night mares, insomnia.
So to avoid BDZ withdrawal syndrome change to equivalent dose of
diazepam, 1/8 of the dose/ 2wks.
Lithium
ACE Inhibitors & Angiotensin II Receptor Blockers&
Calcium Channel Blockers
(Nondihydropyridine)&NSAIDs : May increase the
serum concentration of Lithium.
Carbamazepine, Phenytoin, TCA, SSRI, Methyldopa,
MAO Inhibitors enhance the toxic effect of Lithium.
Thiazide Diuretics: May decrease the excretion of
Lithium.
Amphetamines: Lithium may diminish the stimulatory
effect of Amphetamines.
Sodium Bicarbonate, Sodium Chloride, Theophylline
Derivatives : May increase the excretion of Lithium.
Lithium
Cardiovascular: Cardiac arrhythmia, hypotension, sinus node dysfunction, flattened
or inverted T waves (reversible), edema, bradycardia, syncope

Central nervous system: Dizziness, vertigo, slurred speech, blackout spells,


seizure, sedation, restlessness, confusion, psychomotor retardation, stupor, coma,
dystonia,, lethargy, headache, pseudotumor cerebri.

Dermatologic: Dry or thinning of hair, folliculitis, alopecia, exacerbation of psoriasis.

Endocrine & metabolic: Euthyroid goiter and/or hypothyroidism, hyperthyroidism,


hyperglycemia, diabetes insipidus

Gastrointestinal: Polydipsia, anorexia, nausea, vomiting, diarrhea, xerostomia,


metallic taste, weight gain, salivary gland swelling.

Genitourinary: Incontinence, polyuria, glycosuria, oliguria, albuminuria.

Hematologic: Leukocytosis.

Neuromuscular & skeletal: Tremor, muscle hyperirritability, ataxia, choreoathetoid


movements, hyperactive deep tendon reflexes, myasthenia gravis (rare)

Ocular: Nystagmus, blurred vision, transient scotoma


Rheumatology
* SYNOVIAL FLUID ANALYSIS
FEATURE Normal NONINFLAMMATORY INFLAMMATORY SEPTIC
(eg osteoarthritis) (eg RA, crystal-induced,
spondyloarthropathies,
connective tissue dz, HOA)

WBCs/m <200 200- 2000 2000 -50.000 >100,000


m3
Polymalgia Rheumatica
Diagnostic Criteria for PMR
Age >50.
ESR>40mm/h.
Neck/bilateral shoulder / pelvic girdle AM stiffness.
Symptomatic episodes > 1h in duration.
Clinical history >1 month duration.
Response to low-dose (15mg/day) steroids.
Remember, PMR patients complain more of stiffness than pain.
Minor criteria include:
Weight loss; fever; night sweats,Synovitis.
Symmetrical proximal muscle tenderness( no weakness).
Alk phos / GGT.
Normocytic anemia.
Normal CPK, EMG, muscle biopsy.
50% of all PMR patients (TA symptoms) will be found to have giant-cell
arteritis on TA biopsy.
fibromyalgia
Diagnosis of exclusion.
Female > male .
Pain must be present for > 3m.
Multiple tender points at least 11points .
Pts are poorly able to localize the pain.
Nervousness is frequent concomitant.
TTT: Aerobic exercise.
TCA. ( Tricyclic antidepressant).
DD OF MORNING STIFFNESS/
PAIN WORSE IN AM
Rheumatoid arthritis.
Difficulty doing up buttons
Ankylosing Spondylitis.
LBP + stiffness radiating to buttocks
and thighs
polymaylgia Rheumatica (PMR)
Difficulty getting out of bed
Fibromyalgia
Osteoarthritis

Bouchards nodes are proximal interphlangeal


nodes.
Heberden's nodes are distal interphalangeal
nodes.
X ray features of osteoarthritis are: joint space
narrowing ,osteophytes , subchondral
sclerosis & subchondral cysts.
Rheumatoid arthritis
Seven diagnostic criteria for
Rheumatoid arthritis
Morning stiffness.
Arthritis of 3 joint area.
Arthritis of the hand joint.
Arthritis symmetric.
Rheumatoid nodules.
Serum Rheumatoid factor.
Radiographic changes.
* In order to have RA, you must have 4 of
the 7 criteria present for at least 6 weeks.
Symptoms and signs

Predominantly peripheral joints are affected


symmetrical joint pain, effusions, soft tissue
swelling, early morning stiffness. Progression
to joint destruction and deformity.
Nodes and nodules in rheumatic
diseases

1. Heberden's nodes DIP osteophytes in 1ryOA.


2. Bouchard's nodes PIP osteophytes in 1ry OA.
3. Rheumatoid nodules.
4. Gouty tophi (elbow, ears, heels, PIPs, DIPs).
5. Xanthomata.
Investigations

Check FBC (normochromic normocytic ),


ESR, and/or CRP, platelets &WCC are
increased.
Rheumatoid factor and anti-CCP antibodies
are +ve in the majority 60- 70%. A minority
have a +ve ANA titre.
X-rays normal, periarticular osteporosis or
soft tissue swelling in the early stages;
Later---- loss of joint space, erosions, and
joint destruction.
RHEUMATOID ARTHRITIS OSTEOARTHRITIS

ETIOLOGY Autoimmune disease Degenerative disease


PATTERN of Morning stiffness Worse after effort or as
symptoms activity progresses,
relived by rest
Predominant PIP, MCP, wrists (symmetric) CMC, PIP, DIP
HAND joint
involvement
LAB +RF, ESR and C-reactive Normal RF, ESR, CRP
protein
XRAY findings 1. Periarticular osteopenia. 1. Osteophyte formation
2. Marginal bony erosions. 2. Subchondral sclerosis
3.Symmetrical narrowing of (no periarticular
the joint space. osteopenia)
3. Joint space narrowing.
DD of symmetrical polyarthropathy
Rheumatoid Arthritis.
Viral arthropathy.
Psoriatic polyarthropathy.
DMARDs (Disease- modifying
antirheumatoid drugs):
2 Key Points:
Do not wait to start patients on DMARDs until
they have failed multiple courses of NSAIDs
but, as soon as the diagnosis of RA is
confirmed.
Methotrexate is the DMARD of choice in
patients with severe disease. Start therapy
with 7.5mg weekly and raise the dosage at 1-
intervals until peak efficacy is achieved.
* DMARD Therapy for RA ("GOLD PILE SCAM")

DMARD Potential Toxicities Requiring F/U Monitoring Studies

Gold IM and PO Myelosouppression, proteinuria CBC and urine dipstick for protein

Penicillamine Myelosuppression, proteinuria CBC and urine dipstick for protein

Infliximab (Remicade) Flu-like sx, auto-Abs; for patients not responding to None
methotrexate; given IV

Leflunomide (Avara) Thrombocytopenia, hepatotxicity, diarrehea CBC and AST

Etanercept (Enbrel) (a TNF blocker) Reactions at site of SQ injection, flu-like sx None

Sulfasalzine Myelosuppression CBC, AST, creatinine

Cyclophosphamide Myelosuppression, myeloproliferative disorders, CBC, urinalysis, and urine cytology


malignancy, hemorrhagic cystistis

Cyclosporine Renal insufficiency, anemia, HTN, hirsutism creatinine, CBC, K+, LFTs

Azathioprine Myelosuppression, hepatotoxicity, CBC


lymphoproliferative disorders

Antimalarials (Hydroxycholorquine) Macular damage Yearly fundus exams

Methotrexate Myelosuppression, hepatic fibrosis, cirrhosis, CBC, AST, albumin


pulmonary infiltrates or fibrosis

Minocycline Photosensitivity, skin discoloration, GI upset, drug- None


induced hepatitis, dizziness
Juvenile chronic arthritis
is rheumatoid factor negative.
Different classifications are systemic, pauciarticular
and polyarticular.
Commonest type is Stills disease.( Systemic
onset juvenile R.A ). Peaks of disease are about 5
years and 15 years of age.
Typical diagnostic criteria include High spiking
fevers; arthralgias/ macular rash, hepatomegaly,
splenomegaly, lymphadenopathy, serositis
(pleuritis, pericarditis), seronegativity (-veRF; -ve
ANA); leucocytosis. Thrombocytosis.
Very high serum ferritin.
Diagnosis of Felty's Syndrome
Neutropenia caused by hyperslenism and
antineutrophil antibodies in a patient with long term
rheumatoid arthritis.
Triad of Neutropenia; Seropositive RA;
Splenomegaly /hyperslenism.
Frequent concomitants
Serious infections
Anemia, thrombocytopenia.
Leg ulcers
LN, Hepatomegaly
Sjogren's synd.
Weight loss.
Still's Disease Felty's Syndrome

WBC--inc WBC---dec

Fever No fever necessary

Seronegative RA Seropositive RA

Splenomegaly splenomegaly
Systemic lupus erythematosus
9:1.and Asians. Onset 15-40y. Presentation is variable multisystem
involvement must be demonstrated to make a diagnosis:
Joints (95%)arthritis, arthralgia, myalgia, tenosynovitis
Skin (80%)photosensitivity, facial butterfly rash, vasculitis, hair loss,
urticaria, discoid lesions
Lungs (50%)pleurisy, pneumonitis, effusion, fibrosing alveolitis
Kidney (50%)proteinuria, BP, glomerulonephritis, renal failure
Heart (40%)-pericarditis, endocarditis
CNS (15%)depression, psychosis, infarction, fits, cranial nerve
lesions.
Blood; anaemia, thrombocytopoenia, splenomegaly
Diagnostic criteria for SLE
4 of 11 for 6 weeks;
1. butterfly rash
2. discoid rash.
3. Photosensitivity
4. Oral ulcer
5. Arthritis
6. Serositis
7. Nephrological; lupus nephrotis
8. Neurological
9. Hematologic; lymphopenia, thrombocytopenia.
10. Immunological; anti DNA
11. ANA
Treatment;
Skin only chloroquine.
JOINT only; NSAIDs.
Other extraarticular; steroid & other
immunosuppressants.
Drug-induced lupus
Occurs with minocycline, isoniazid,
hydralazine, procainamide,BB,
chlorpromazine, sulfasalazine, losartan, and
anti-convulsants. Remits slowly when drug is
stopped but may need steroid treatment to
settle.
No nephrological, no neurological,
DNA ve
Anti histone Ab +ve.
TYPES OF ANTINUCLEAR ANTIBODIES
(ANA)
Following detection of a high titer of ANAs (e.g. 1:160), various
subtypes are determined. Examples include:
Anti-dsDNA (double-stranded DNA) specific for SLE
Anti- histones Drug induced lupus
Anti-chromatin= anti-nucleosome antibodies.
Anti-ENA (Extractable nuclear antigen)=non-DNA = RNP
Anti-Ro (SS-A)
Anti-La (SS-B)
Anti-Sm (Smith antigen)
Anti-U1RNP (nuclear ribonucleoproteins)
Anti Scl-70 (topoisomerase I)
Anti-Jo
Antinucleolar20% of systemic sclerosis .
Anti-centromere
Antibodies to nuclear pore proteins as anti-gp- APA.
Anti-ENA (Extractable nuclear
antigen)

Ro/SSA Sjgren's syndrome , 40% SLE, ANA -ve


SLE, subacute cutaneous LE & neonatal LE.
La/SSB 1ry Sjogren.
Anti- Smith (Sm) Ab is very specific for SLE, but
only in 25 % .
Antibodies to U1-RNP 100% of MCTD, 30-40 % of
SLE (+ anti- Smith) & small proportion of patients with
localized (linear) scleroderma.
Antibodies to topoisomerase I (Scl-70), RNA
polymerases I and III systemic sclerosis related
disorders.
Anti-Jo1 PM especially with IPF.
Chromatin= histones + DNA.
Anti-chromatin= anti-nucleosome antibodies.
DD of sponylitis

1. Sero ve Spondyloarthoropathies.
2. infections. ( TB , Brucella ).
Spondyloarthoropathies
A- HLA-B27 Rheumatic Diseases:
1. Ankylosing Spoondylitits (<90% are HLA-B27).
2. Reiter's Syndrome or
3. Reactive arthritis (<80%).
4. Enteropthic spondylitis (75%).
5. Psoriatic Spondylitis (50%).
B- Characteristis:
1- Sacroiliac joint involvement.
2- Peripheral arthritis (usually asymmetric and oligarticular).
3- Seronnegativity (absence of RF or other autonatibodies).
4- Association with HLA-B27.
5- Relatively early age of onset (<40).
6- Enthesopathy.
7- Anterior uveitis.
1- Radiographic findings:
a) Erosions.
b) Syndesmophytes.
c) Bamboo spine.
D) fusion.
2- Enthesopathic involvement
a) Plantar fasciitis.
b) Achilles tendonitis.
3- Iritis (an important clue in spondyloarthropathies).
ankylosing spondylitis
Schober's test

Schober's test assesses the amount of lumbar flexion. In this test a mark
is made at the level of the posterior iliac spine on the vertebral column,
i.e. approximately at the level of L5. The examiner then places one
finger 5cm below this mark and another finger at about 10cm above this
mark. The patient is then instructed to touch his toes. If the increase in
distance between the two fingers on the patients spine is less than 5cm
then this is indicative of a limitation of lumbar flexion.
TTT;
1. Symptoms modifying; NSAIDS
2. Disease modifying; infliximab,
methotrexate.
Reactive arthritis Often asymmetrical aseptic arthritis
Occurs 2-6wk. after bacterial infection elsewhere e.g.
gastroenteritis (salmonella, campylobacter), GU infection
(chlamydia, gonorrhoea).
Management: NSAIDs, physiotherapy, and steroid joint
injections. Recovery usually occurs within months. A minority
develop chronic arthritis requiring disease-modifying drugs.
Reiter's syndrome: Polyarthropathy, urethritis, conjuctivitis.
Affects men with HLA B27 genotype. Commonly follows GU or
bowel infection. Joint and eye changes are often severe.
characteristic mucocutaneous changes
oral ulcers , circinate balanitis , keratoderma blenorragicum.
Enteropathic spondylarthropathy
Oligoarticular or polyarticular arthritis linked to inflammatory
bowel disease. Includes sacroiliitis, plantar fasciitis,
inflammatory spinal pains, and other enthsitides (insertional
ligament/tendon inflammation). Arthritis may evolve and
relapse/remit independently of bowel disease. NSAIDs may
help joint pain but aggravate bowel disease.
circinate
balanitis
Psoriatic arthritis
Psoriatic arthritis
Inflammatory arthritis associated with psoriasis. RhF -ve.
Presentation variable. Disease modifying drugs (e.g.
methotrexate) may improve both skin and musculoskeletal
symptoms .

presents as several forms:


Symmetrical polyarthritis.
Arthritis involving DIP joints.
Asymmetrical oligoarthritis including dactylitis (70%)
Ankylosing Spondylitis type.
Arthritis mutilans.
Among the differentials of ank. Spond. Is DISH.
(Diffuse Idiopathic Skeletal Hyperostosis,
a form of 1ry osteoarthritis).

a) Patients with DISH are often obese and 60% have diabetes;
b) 'stiffness' at the spine' yet relatively well-preserved spinal motion.
c) Criteria for DISH'
'Flowing' ossification along the anterolateral aspects of 4 contiguous verterbral
bodies with preservation of disk height.
Absence of SI joint involvement.
Intervertebral disk spaces are preserved.
Marked calcification and ossification of paraspinous ligaments occur
in DISH. Ligamentous calcification and ossification in the anterior spinal
ligaments give the appearance of "flowing wax" on the anterior vertebral
bodies. However, a radioucency may be seen between the newly
deposited bone and the vertebtral body,
Differentiating DISH from the marginal osteophytes in spondylosis.
Diffuse idiopathic skeletal hyperostosis =DISH
vasculitis
Group of diseases in which tissue ischemia and necrosis occur as a consequence of
inflammation of blood vessels, either as a primary event or secondary to a
systemic disease .1.Autoimmune dis. 2. Infection 3.Malignancy 4. drugs .

Large vessel vasculitis: chronic granulomatous reaction


affecting the aorta and major branches.
Medium vessel vasculitis: necrotizing arteritis affecting
visceral arteries renal artery leading to renal infarction .
Small vessel vasculitis: capillaries, venules, arterioles
(glomeruli)leading to glomerulonephritis ..
Clinical Presentation

Constitutional symptoms.
Skin: purpura,ulcers , livedo reticularis , nail bed
infarcts ,digital gangrene.
Eye :episcleritis ,ulcers and vision loss.
Lung: dyspnea, hemoptysis.
Cardiac: angina, myocardial infarction , heart
failure.
GIT: abdominal pain, mesenteric
ischemia,malabsorpt.
Kidney: hematuria, proteinuria, acute/chronic
renal failure
CNS: Mononeuritis multiplex, sensorymotor
neuropathy , convulsions, hemiplegia.
Names and definitions of vasculitides
Large vessel vasculitis
Giant cell (temporal) arteritis
Granulomatous arteritis of the aorta and its major branches,
with a predilection for the extra cranial branches of the
carotid artery. Often involves the temporal artery. Usually
occurs in patients older than 50 and often is associated with
polymyalgia rheumatica.
Presentation (classical) of Temporal Arteritis Headache,
scalp tenderness; jaw claudication; sudden visual loss.
50% of TA patients have symptoms of PMR.
Flase-negative biopsies may occur 2o to presence of "skip lesions" (if
a 2cm biopsy is taken the false-neg rate is >5%; therefore a 3-5cm
segment biopsy is recommended).
Takayasu's arteritis

Granulomatous inflammation of the aorta and its


major branches. Usually occurs in patients
younger than 50.
Middle aged women
0f patients presents with initial systemic
illness of malaise, fever, night sweats, weight
loss, arthralgia, and fatigue then subside with
more gradual vascular changes.
The other half of patients with Takayasu's
arteritis present with only late vascular
changes, without an antecedent systemic
illness.

It is also known as "Pulseless disease" because pulses on the upper


extremities, such as the wrist pulse, may not be able to be felt.
Medium sized vessel vasculitis

Polyarteritis nodosa.
(classic polyarteritis nodosa)
Necrotizing inflammation of medium-sized without
glomerulonephritis or vasculitis in arterioles, capillaries, or
venules.

Kawasaki disease .
Arteritis involving medium sized , and associated with
mucocutaneous lymph node syndrome.
Fever of 5 days' duration associated with at least 4 of the
following 5 change.

Bilateral nonsuppurative conjunctivitis

One or more changes of the mucous membranes of the upper


respiratory tract, including pharyngeal injection, dry fissured lips,
injected lips, and "strawberry" tongue

One or more changes of the extremities, including peripheral


erythema, peripheral edema, periungual desquamation.

Polymorphous rash, primarily truncal

Cervical lymphadenopathy >1.5 cm in diameter


its most serious effect is on the heart where it can cause severe coronary artery
aneurysms in untreated children.
Differential diagnosis, including scarlet fever, toxic shock
syndrome, juvenile idiopathic arthritis.
There is no specific laboratory test for this condition
Blood tests
CBC reveal normocytic anemia and eventually thrombocytosis
ESR will be elevated
CRP will be elevated
Other optional tests
Electrocardiogram may show evidence of ventricular
dysfunction or, occasionally, arrhythmia due to myocarditis .
TTT:
1.Intravenous immunoglobulin (IVIG) is the standard treatment
for Kawasaki disease[
2.Salicylate therapy, particularly aspirin, remains an important
part of the treatment
Polyarteritis nodosa
Necrotizing vasculitis causing microaneurysms
in the medium sized arteries.
HBsAg
Constitutional symptoms: Fever , abdominal
pain,weight loss, arthralgia.
Cardiac: coronary arteritis , hypertension , heart
failure ,pericarditis.
CNS: mononeuritis multiplex, sensorymotor
polyneuropathy. seizures, hemiplegia.
GIT: abdominal pain, malabsorption
Skin : purpura, urticaria, infarcts, livedoreticularis.
Blood: eosinophilia ,anaemia , ESR , CRP.
Kidney: hypertension, hematuria, proteinuria,
renal failure, intrarenal aneurysms.
Testicular pain.
ANCA is ve.
Diagnosis by nerve biopsy of affected nerve .
If affected tissue not accessible ....visceral Angio.
Small vessel vasculitis
ANCA Positive:
Wegener's granulomatosis
Granulomatous inflammation involving the respiratory tract, and
necrotizing vasculitis (Necrotizing glomerulonephritis is common).
Churg Strauss syndrome
Eosinophil-rich and granulomatous inflammation, necrotizing vasculitis
involving the respiratory tract, with asthma and eosinophilia.
Microscopic polyangiitis (microscopic polyarteritis)
Necrotizing vasculitis, with no immune deposits, affecting small vessels .
Necrotizing glomerulonephritis is very common. Pulmonary capillaritis
often occurs.
Small vessel vasculitis
ANCA Negative:
Henoch Schnlein Purpura.
Vasculitis, with IgA-dominant immune deposits. Typically
involves skin, gut, and glomeruli, and arthralgia or arthritis.
Essential cryoglobulinaemic vasculitis.
Vasculitis, with cryoglobulin immune deposits, affecting small
vessels, and associated with cryoglobulins in serum. Skin
and glomeruli are often involved.
Cutaneous leucocytoclastic angiitis .
Isolated cutaneous leucocytoclastic angiitis without systemic
vasculitis glomerulonephritis.
What is ANCA?
Autoantibodies against intracellular
neutrophil antigens.
C-ANCA cytoplasmic anti PR3.
P-ANCA perinuclear anti MPO.
vasculitis with a predilection for renal involovement?
Small vessel vasculitis.
Medium vessel vasculitis : PAN
How does vasculitis affect the kidney?
Urine analysis shows RBCs ,red cell casts and proteinuria
common but usually < 3 gm.
Pathology
The shared glomerular lesion of the pauci-immune small-
vessel vasculitides is a necrotizing glomerulonephritis with
crescent formation.
Early lesions have segmental fibrinoid necrosis with or
without an adjacent small crescent.
Severe lesions have global necrosis with large
circumferential crescents.
Renal arteritis affecting the interlobular arteries.
Wegner granulomatosis
Necrotizing granuloma in the
respiratory tract(nasal ulcers,
epistaxis , sinus involvement,
otitis media, oral ulcers
,proptosis).
hemoptysis
Rapidly progressive
glomerulonephritis.
C-ANCA +ve in 70%
Wegener Granulomatosis
Clinical Presentation
upper respiratory tract
Lung involvement in 80%; transient infiltrates or nodular densities may be seen
on (CXR)
Histopathology
(1) vasculitis of small vessels,
(2) granulomatous changes,
(3) focal necrosis
LM :segmental necrotizing.
crescents

IF: Pauci-immune GN;


crescents;
tubulointerstitial granulomas

EM: No immune deposits are seen


Laboratory Diagnosis
cANCA is specific and sensitive
20% to 30% have pANCA
Segmental fibrinoid necrosis with nuclear debris and
glomerular basement membrane disruption, typical
of early stage of pauci-immune crescentic
glomerulonephritis
Well-developed cellular crescent with collapse of
small amount of remaining glomerular tuft with
segmental fibrinoid necrosis and extracapillary
fibrin and necrosis
Churg-Strauss Vasculitis
Small vessel vasculitis .
eosinophilic pulmonary infiltrates
Allergic rhinitis.
Nasal polyposis.
Asthma
Eosinophilia
Necrotizing inflammation
Almost no kidney affection
glomerulonephritis is usually (but not always) mild.
Associated with leukotriene antagonist therapy,
P-ANCA Usually in 60%. (MPO) positive. +ve
Microscopic polyangitis
Small vessel vasculitis
Absence of asthma ,eosinophilia,necrotizing
inflammation.
P-ANCA is +ve in 50%
Kidney : Rapidly progressive glomeruloneph.
Treatment for ANCA-associated disease
combination of glucocorticoids (1 mg/kg/d) and IV or
oral cyclophosphamide (2 mg/kg/d) for 1 year after
remission.
methylprednisolone at 7 mg/kg/d IV for 3 days, then
oral prednisone at 1 mg/kg/d.
plasmapheresis in who RPGN or pulm Hge.
Sulfamethoxazole/trimethaprim may diminish relapse
rates.
Course and Prognosis
90% 1-year mortality if left untreated, 90% remission
if treated.
Relapse occurs in 25% to 50% of patients followed
up for 3 to 5 years
Henoch-Schnlein purpura
systemic vasculitis mostly seen in children. It is a multisystem disorder involving
the skin, joints ,gastrointestinal and renal tracts.

Aetiology is unknown, but the syndrome is often preceded by infections


such as Group A beta hemolytic streptococcal

Pathology is due to intravascular deposition of IgA immune complexes.

Patients often present with a skin(palpable purpuric eruption purpuric rash usually
involving the buttocks and lower limbs, arthralgia and severe colicky
abdominal pain and tenderness caused by vasculitis-induced thrombosis in
the gut.
Renal involvement commonly presents as microscopic haematuria and
proteinuria Hematuria 50%/asymptomatic hematuria proteinuria 40%, nephrotic
syndrome ,acute and chronic renal failure.
Similar to IgA nephropathy , but crescents, are more frequent
Immunoflorescence :IgA mesangial deposition.
Course.
Overall good renal outcome; complete recovery
Therapy.
Symptomatic for mild cases.
For severe nephritis, treatment is steroids+/-cyclophosphamide.
Can complement levels help diagnosing vasculitis?

Exclude infection.
Low in SLE ,Cryoglobulinemia .
Normal in ANCA +ve vasculitis.
essential mixed cryoglobulinemia
essential mixed cryoglobulinemia was
appropriate. complexes in which one
component, usually IgM, exhibits antibody
activity against IgG (i.e., mixed
cryoglobulins).
The diagnosis of mixed cryoglobulinemia is
typically made from the history, skin
purpura, low complement levels, and
demonstration of circulating cryoglobulins.
CLINICAL PRESENTATION of Mixed
Skin manifestations CGs
Palpable purpura (80% in mixed types)
Ischemic necrosis ( 020% in mixed types)
Livedoid vasculitis (1% in type I, 14% in type III)
Cold-induced urticaria (15% in type I, 10% in type III)
Acrocyanosis
Nailfold capillary abnormalities
Musculoskeletal manifestations
Arthralgias (5% of type I, 20-58% of mixed)
Frank arthritis and progressive joint deformity (distinctly rare)

such as arthralgias and myalgias are common in the mixed


CGs, but frank arthritis or myositis are rare to uncommon.,
especially affecting the metacarpophalangeal, proximal
phalangeal, knees, and ankles,
Neuropathy
Sensorimotor neuropathy affects a high percentage of mixed, but clinically
significant neuropathy is uncommon., Visual disturbances
Renal manifestations
Membranoproliferative glomerulonephritis described in all types (more
common in type II)
Intraluminal cryoglobulin deposition
Hypertension
Nephrotic-range proteinuria with resultant edema
Pulmonary manifestations
Dyspnea
Cough
Pleurisy
Pleural effusions
Gastrointestinal manifestations
Abdominal pain (2-22%)
Hemorrhage
Hepatomegaly or signs of cirrhosis (ie, palmar erythema, abdominal wall
collateral vessels, spider angiomata)
abnormal liver function tests
Splenomegaly
lymphadenopathy
Cryoglobulinemic Glomerulonephritis can be
differentiated from idiopathic MPGN by the following
findings
.
(1) Intracapillary thrombi the presence of large
deposits filling the capillary lumen.
(2) Massive infiltration of capillary spaces by
monocytes and polymorphonuclear cells (the
exudative component ).
(3) possibly some vasculitis in small and medium-
sized renal arteries .(25% to 33%).
The combination of vasculitis and glomerular thrombi
should suggest cryoglobulinemic nephropathy, but
proliferative lupus nephritis with associated vasculitis
is also in the differential diagnosis.
Aggressive therapy in idiopathic mixed
cryoglobulinemia reserved for patients with acute
severe disease (manifested by progressive renal
failure, distal necroses requiring amputation, or
advanced neuropathy).
plasmapheresis (remove the circulation
cryoglobulins) with steroids (1000 mg of
intravenous methylprednisolone daily for three
days , followed by oral prednisone) and
cyclophosphamide.
Rituximab is a promising alternative treatment
option for severe cryoglobulinemic vasculitis and
nephritis.
( Hughes Syndrome),

Anti-Phospholipid Syndrome
disorder characterized by multiple
different antibodies that are associated
with both arterial and venous thrombo.

.
There are three primary classes of
antibodies associated with the
antiphospholipid antibody syndrome:
1) anticardiolipin antibodies.
2) the lupus anticoagulant .
3) antibodies directed against beta-2-
glycoprotein 1.
Sensitivity and Specificity for APS
APL Test Sensitivity Specificity
ACL 80-90% Low
LA Moderate High
2-GP I 74% 85%
APhL 91% 99%
Harris. Lupus, 1998.
A patient must meet at least
one clinical and one
laboratory criterion for a
diagnosis of APS
Thrombosis
Venous and arterial circulations
Embolic risk high
Venous > Arterial
Occurs in any system or organ with any part of
vascular tree involvement possible(Large-
small)
Single or multiple vessel involvement
Recurrent events common
Catastrophic anti-phospholipid
syndrome (CAPS)
This syndrome, termed "catastrophic
antiphospholipid syndrome," is defined by the clinical
involvement of at least three different organ systems
over a period of days or weeks with histopathological
evidence of multiple occlusions of large or small
vessels.
steroids or aggressive immunosuppression is not
used unless recurrent thrombotic or ischemic events
despite high intensity warfarin therapy.
High dose steroid is reserved for treatment of
underlying active lupus and not for lab
antiphospholipid AB.
Rituximab
Is a humanized mouse monoclonal antibody
against the B cell antigen CD20 that results in
the prolonged depletion of B cells.
CD20 is relatively selectively expressed on pre-
B and mature B cells, but not plasma cells or T
cells, and may selectively treat antibody-
mediated disorders with less immune
suppression than other treatment regimens.
Vasculitis mimickers
Subacute bacterial endocarditis.
Left atrial myxoma
HIV
Paraneoplastic syndrome
Cholesterol embolization
Cocaine and amphetamine use.
Treatment
Combined treatment with steroids and
cyclophosphamide induces remission in 75% of ptns
at 3months and 90% at 6 months.
Pulse steroid therapy 7mg/kg for 3 days(IV methyl
prednisone)
IV cyclophosphamide therapy 0.5g/m2 monthly
reaching 1g/m2 based on the leucocyte count.
Role of plasma exchange is controversial ,beneficial
in dialysis dependant renal failure patients and those
with pulmonary hge .
Crystal induced Arthropathy
A 48-year-old gentleman presented with arthritis of
rigt ankle for one day.
.. S.uric acid : 7.8

.. Synovial fluid : inflammatory with MSU crystals

.. 24-h urine uric acid : 1200 mg.

The most likely cause.

1- hyperparathyroidism 2- hypothyroidism
3- MPD 4-Lactic acidosis
5- thiazide therapy
A 52-year-old lady , diagnosed to have gouty arthritis
two years back with synovitis right MTP1., subsided with
local steroid injection.

.. No further attacks since then

..s.Uric acid ranging from 6.8 8.2


..s.cr : o.8 , No renal stones
.. No tophi.

.. She has to start Allopurinol


** YES ** NO
A 58-year-old gentleman , first attack of gout in R
knee persistent in the last 6 days

.. The drug to be least effective at this stage :

1- NSAID
3- Colchicin 4- local steroids
5- Oral PDN
A 52-year-old gentleman upon the first attack of
gouty arthritis in L MTP1, started Colchicin for the
last 2 weeks. Inflammation subsided

.. S.uric acid now is 13 , supposed to start Allopurinol

.. You will combine Colchicin until :


1) 3 months 2) 6 months
3) s.U.A < 6 4)s.U.A < 7
5) 6-months after s.U.A < 6
A 48-year-old lady accidentally discovered to have
s.UA of 8 mg%

.. 24-h urine uric acid was found 1200 mg

.. Best way of management :


1) Start Colchicin
2) Observation with regular monitoring
3) Start Prebencid
4) Start Allopurinol
5) Start Allopurinol & Colchicin
A 54-year-old male , diagnosed as gouty arthritis,
first attack.

.. S.UA : 8
.. S.creatinin : 1.8
.. C.cr : 35 ml/min

Best way of managemeent :


1) Colchicine 0.5 mg OD
2) Colchicine 0.5 mg BD
3) Avoid Colchicine
A 52-year-old lady, four weeks after first attack of
gouty arthritis, lab. Investigations revealed :

.. S.UA : 10
.. S.cr : 1.9
.. Ccr : 40ml/min

Best way of management :


1) Start Allopurinol 50mg/d,increase to 100 after 2w
2) ,, ,, 100mg/d ,, 200 ,,
3) ,, ,, 300 mg/d
4) ,, ,, 100mg/d, build up to 300mg
5) No need for Allopurinol.
Hyperuicemia
Gout negatively birefringent crystals
Hyperuicemia is either 2ry to overproduction (> 800mg
/ d) 10% or underexcretion (<600mg / 24hrs) 90% .
Causes of underexcretion . HARD to ..
HTN.
Hypothyroidism.
Hyperparathyroidism.
Acidosis.
Renal insuffeciency.
Drugs( thiazides duiretics, Alcohol)
Causes of overproduction
Tissue damage . (Rhabdomy, tumor lysis,
hemolytic process, exercise).
Proliferative disorders.( mylo& lymphoprolif,
psoriasis)
Gout is associated with :
Hypertriglyceridaemia, diabetes, obesity.
serum hyperuicemia is neither necessary nor
sufficient to make the diagnosis of gout.
A synovial fluid aspirate is necessary for definitive
diagnosis. Always remember, for definitive
diagnosis, to send the synovial fluid for crystal
analysis, gram stain.
Alcohol/diet/diuretics are the most common
cause of acute gout in the outpatient
population.
Gout is very uncommon among
premenopausal women.
When to treat asymptomatic hyperuricemia?
** Three specific circumstances warrant at least consideration
for institution of hypouricemic drugs :

1) Persistent s.urate >13mg/dl in men and >10 mg/dl in women

2) Excretion of U.uric acid >1100 mg/d is associated with 50%


risk of uric acid calculi.

3) Patient about to receive radio or chemotherapy that is likely


to result in extensive tumour lysis
Renal disease is a limitation for both
allopurinol & colchicine. With allopurinol,
there is a consensus that you need to start
with a lower dose and only gradually
increase it.

Uricosurics are not effective if the serum


urate is over about 12 mg/dL, Uricosurics
also increase the risk of nephrolithiasis.

colchicine works only if you use it fairly


early in the attack.
Benzbromarone Benzbromarone, a
uricosuric agent that reduces serum uric acid
levels in transplant recipients with
hyperuricemia and gout.
Benzbromarone is an effective uricosuric,
even in patients with creatinine clearances
as low as 20 mL/min
1. Treat acute flares with antiinflammatory
drugs; NSAIDs, colchicine or steroids.
2. Initiate urate lowering drug (Allopurinol)
weeks after any acute flare.
3. Choose effective & safe drug e.g. allop
allergy, renal disease.
4. Protect against flare from urate lowering drug
by adding anti-inflamatory for 6 months after
achieving the target.
5. Continue life style modification
6. Monitor/ 2-4 wks till target uric acid< 6 mg/dl.
7. Then Monitor every 6-12 months.
Pseudogout
Inflammatory arthritis due to deposition of pyrophosphate crystals.
Chondrocalcinosis may be seen on X-ray (calcification of articular
cartilage). Knee, wrist, and shoulder are most commonly affected.

causes are:
Haemochromatosis.
wilson's disease .
Hypothyroidism.
hyperparathyroidism.
hypomagnesaemia.
hypophosphataemia
Amyloidosis.
Aging.DM
Chronic renal failure on dialysis.
Acute attacks can be triggered by intercurrent illness and metabolic
disturbance. Attacks are less severe than gout and may be difficult to
differentiate from other types of arthritis. Presence of joint crystals
confirms diagnosis.
Pseudogout positively birefringent crystals .
Pseudogout
(calcium pyrophosphate "CPPD")
Can mimic and coexist with gout.
similar to gout, surgery, trauma, and alcohol
may precipitate.
Treatment options are similar to gout, except
for allopurinol.
Pseudo gout, is Positively birifringent under
polarized light microscopy.
A 68-year old man presented with sudden severe
pain and swelling in the left knee. Synovial fluid
analysis shows abundant calcium
pyrophosphate dihydrate (CPPD) crystals.
Which of the following tests is NOT appropriate
for further assessment of this patient illness?
A : Creatinine kinase
B : Serum calcium
C : Thyroid function test
D : Serum ferritin level
E : Hb AIc.
Crystal Analysis: Gout Versus Pseudogout
Feature Gout Pseudogout
Crystal Monosodium urate Calcium
pyrophosphate
Color under Yellow Blue
polarized light
Shape of crystal Needle Rhomboid
Birefringence/ Negative Positive (clockwise)
rotation under (counterclockwise)
polarized light
direction
PseudoPseudogout
( calcific tendinitis)
Pain on adduction more with active movement.
Characterized by deposition of hydroxy appetite
crystals within rotator cuff muscle near humeral
attachment most commonly involve supraspinatus
tendon.
Severe shoulder pain localized mainly to humorous
diffusely around antrolateral shoulder.
Palpation or compression around the greater tubercle
of the humerus causes tenderness.

X ray : linear calcific density in supraspinatus tendon.


Ttt: intralesional steroids & physiotherapy.
70 years old lady complaining of
gradually progressive bilateral
lower limb oedema
Paget disease of bone
Paget disease of bone
Paget disease of bone
What are the manifestations?

What are the serious complications?

What are the biochemical abnormalities?

What is the treatment?


Paget's disease of bone
Accelerated, disorganized bone remodelling due to abnormal osteoclast activity.
Affects up to 1:10 of the elderly but only a minority are symptomatic.
Presentation.
Skeletal Pain ,deformity & fracture.
Deformity bowing of weight-bearing bones, especially tibia, femur, Frontal bossing.
High COP failure.
Complications
Pathological fractures; O Arthropathy of adjacent joints.
cranial nerve compression., neurological symptoms e.g. deafness,
bone sarcoma (10% of those affected >10y.).
Investigation.
X-ray Distinctive changes;
Alk phos--->very high.
normal Ca2+, PO4, PTH,
bone scan...diffuse density.
Urinary hydroxyproline.
Management
pain and long-term complications with bisphosphonates (e.g. risedronate 30mg/d. for
2mo.).
Paget disease of bone
Pain,deformities,fractures.
Hyperdynamic circulation
Marked increase of alkaline Phosphatase
Bone scan
Bone scan
The radioactive tracer is evenly distributed among the
Normal: bones. No areas of abnormally high or low
accumulation are seen.

The tracer may accumulate in certain areas of the


Abnormal: bone, indicating one or more hot spots. Hot spots may
be caused by a fracture that is healing, bone cancer, a
bone infection (osteomyelitis), arthritis, or a disease of
abnormal bone metabolism (such as Paget's disease).
Bone scintigraphy

Normal bone scan. Bone metastasis


Bone metastasis Paget disease of bone Pulmonary osteodystrophy
Hypertrophic osteoarthropathy (or
hypertrophic pulmonary osteoarthropathy
when there is mesothelioma or bronchogenic
carcinoma associated)
X rays show periosteal reaction at the ends
of the radius and ulnar bones suggestive of
periostitis.
Osteomalacia
Osteomalacia is characterized by a low serum
calcium and phosphate with elevated serum alkaline
phosphatase. Osteomalacia may be caused by
deficiency of vitamin D or phosphate deficiency.
Skeletal x-rays
osteomalacia present with the pathognomonic Looser
zones are linear areas of low density surrounded by
sclerotic borders,
better observed in the pubic ramus, upper femoral bone,
and ribs.
Treatment is with a daily dose of calciferol (20-25
micrograms).
Vitamin D deficiency (osteomalacia)
Nutritional deficiency
Impaired absorption ; Small bowel diseases, such as celiac
disease, gastric bypass, steatorrhea, and pancreatic diseases.
Hepatic disease
impaired 25-hydroxylation of vitamin D,
decreased bile salts with malabsorption of vitamin D,
decreased synthesis of vitamin Dbinding protein, or other factors.
vitamin D dysfunction.
Anticonvulsants

Renal failure
decreased conversion of 25-hydroxyvitamin D to its active form 1,25-
dihydroxyvitamin D. This results in an increase in PTH.
Inherited conditions
vitamin D dependent rickets (type I) or 1-alpha-hydroxylase deficiency .
vitamin D dependent rickets (type II )--- Hereditary vitamin D resistance rickets.
X-linked hypophosphataemic Vit
D resistant rickets.
serum phospate is low and urine
phosphate is high due to inappropriate
renal phosphate wasting.
Serum parathyroid levels are usually
slightly elevated. Clinically, growth
retardation that causes very severe
rickets, especially in affected males.
Treatment is with oral phosphate and high
dose activated Vitamin D.
Bone Disease Serum Serum Alkaline
calcium phosphorus phosphatase

osteoporosis

osteomalacia

Paget disease

Primary
hyperparathyroidism

Renal
osteodystrophy

X-linked
hypophosphat
aemic Vit D
resistant
rickets.
Behcet's Syndrome
Immune mediated vasculitis, venulitis.
Equal sex affection but more severe in males ,Eastern Mediterranean and Asia .
C/P;
Painful Genital ulcers.
Painful Oral ulcers (98%).
Ocular disease ( 80%); more common in HLA B5 men; eg uveitis. iritis, retinal
vessel occlusions and optic neuritis can be found. Hypopyon uveitis
Skin; E.nodosum ( 80%); (F<M; associated with non-deforming arthritis), pyoderma
gang, Thrombophlebitits (30%),
Thrombophilia.
CNS disease (30%); Aseptic meningitis. Meningoencephalitis, TIA-like episodes,
headache, papilledema, Cranial nerve pseudobulbar palsies, Ataxia, dementia.
Colitis (30%).
Associated with HLA-B5.
May lead to perforation.
Clinically overlaps with IBD.
Arthritis ; intermittent, self-limited, not deforming and localized to the knees and
ankles. Spondylits, sacroilitis; When present, linked to HLA-B27.
Inv; A positive pathergy test refers to skin injury by needle prick leads to a papule or
pustule formation in 48 hours
TTT; colchicine, prednizone, NSAIDs, thalidomide.
Behcets
Sjgren syndrome
Sjgren syndrome (SS) is a systemic chronic inflammatory disorder
characterized by lymphocytic infiltrates in exocrine glands.
C/P; sicca symptoms; xerophthalmia (dry eyes), xerostomia (dry mouth),
dyspareunia fatigue and parotid gland enlargement.
extraglandular features ; arthralgia, arthritis, Raynaud phenomenon,
myalgia, pancreatitis, leukopenia, anemia,LN, neuropathy, vasculitis,
RTA, and later lymphoma (suspected if low C4, cryo, persistent parotid
enlarg, purpura & leukopenia).
Primary Sjgren syndrome occurs in the absence of another underlying
rheumatic disorder, whereas secondary Sjgren syndrome is
associated with another underlying rheumatic disease, such as SLE, RA,
or scleroderma.
Inv;
ANA 90%, Anti RO & Anti La in 1ry SJ,
ESR, CRP, RF, hypergammaglobulinema in 80%.
Schirmer's test
Management
artificial tears , cool drinks, artificial saliva sprays or sugar-free gum.
pilocarpine
NSAIDs, hydroxycholoquine for arthritis.
Dermatomyositis
C/P;
proximal muscle weakness, tenderness.
Skin; Raynaulds, Gottron's papules , heliotrope rash around the eye, calcinosis,
periungual telangiectasia, mechanics hand, v shaped, shawl sign.
Others;
Dysphagia.
cardiomyopathy,
IPF.
The condition is associated with carcinoma of the breast, lung, ovary and bowel in
old age only.

Inv :
elevation of muscle enzymes; CPK, LDH, AST.
EMG; myopathic pattern (high frequency, low amplitude).
ESR, CRP in 5 % only.
Autoantibodies;
- ANA
- anti-jo-1(IPF, mechanics hand, Raynaulds)
- anti M2 (classic)
- anti SRP (severe rapidly progressive) severe form with cardiac affection.
muscle biopsy; inflammatory changes .

TTT; steroids rapid CPK but power takes weeks+/- methotrexate, cyclophosphamide.
Scleroderma
Inv;
ANA in 90%,
RF in 30%
Anti-topoisomerase (= anti Scl 70, specific, in 30%) in diffuse
scleroderma.
Anticentromere in 50-90% of limited & 10% of diffuse sclero.
Anti-RNA polymerase III in renal crises.
Patterns;
1. Limited systemic; CREST S (calcinosis, raynaulds, esophageal
dysmotility, scleroderma, telangiectasia), tight skin limited to face,
neck, distal extremities. cause of death; pulmonary HTN.
2. Diffuse systemic; cause of death; renal crises, worse prognosis.
3. Localized to the skin; morphea (plaques), coup de sabre (linear).

TTT; NSAIDs, PPI, antibiotics for bact overgrowth, VD, PG, D-


penicillamine, ACEI for renal crises.
Overlap Syndromes
E.g. mixed CT disease; SLE, Sclero, PM.
+ve anti RNP.
Osteoporosis

**A disease characterised by low bone mass


and microarchitectural deterioration of bone
tissue, with a consequent increase in bone
fragility and susceptibility to fracture .

**Bone mineral density (BMD) T-score < 2.5


What are the major risk factors for osteoporosis?

Non-modifiable Modifiable

1) Age. 1) low ca. intake


2) Race ( Caucasian,Asian). 2) low vit-D intake
3) Gender (female). 3) estrogen deficiency
4) Early menopause(<45) 4) sedentary life
5) Slender built. 5) smoking
6) Positive family history. 6) alcohol(>2drinks/d)
7) caffeine(>2serving/d
8) medications:
(steroids-thyroxine)

Other conditions with low bone mass

** hyperparathyroidism ** Celiac d.
** hyperthyroidism ** Liver d.
** hypogonadism ** I.B.D
** cushing s. ** myeloma
** osteomalacia ** Renal failure
** Drugs :
.. Steroids thyroxine cyc,A - heparin
How to interpret BMD H

1)T-score : comparison of patients bone mass


to that of young normal subject(age 30)

2)Z-score : comparison of patients bone mass to


that of age-matched subject
indications for BMD:
**

1)Women aged 65 years and older .


2)Postmenopausal women< 65 years with risk factors for osteoporosis
3)Men aged 70 years and older .
4)Adults with fragility fracture .
5)Adults with a disease or associated with low bone mass or bone loss.
6)Adults taking medication associated with bone loss.
7)Anyone being considered for pharmacological osteoporosis therapy .
Pharmacologic therapy of osteoporosis
Ca, vit D
Drugs that decrease bone resorption;
1. HRT (Euterine cancer& breast cancer).
2. SERM (selective Estrogen receptor modulators)= agonist on bone &
antagonist on breast osteoporosis, br cancer & IsHD. Side effects;
flushes, DVT.
- Raloxifen (Evista)used in postmenopausal osteoporosis.
3. STEAR= selective tissue estrogenic activity regulator e.g. Tibolon (Levial
)
4. Bisphosphonate (oral alendronate)
5. Calcitonin ; anabolic bone effect, Side effects; nasal congestion, rash.

Drugs that increase bone formation;


1. Synthetic PTH (Teripatide).
2. Strontium ranelate (Protelos) dual mechanism , incr bone formation &
decrease bone resorption.
3. Others; GH, GHRH, IGF1, anabolic steroids, statins.
Ca, vit D;
- ca alone; small effect on BMD, no antifracture effect.
- Vit D alone; unknown effect in pt without vit D def.
- Ca + Vit D; may BMD, fracture.
Raloxifen (Evista ) 60 mg once daily
- TTT & prevention postmenopausal osteoporosis.
- adv; vertebral fracture,but not non vert fracture,
br cancer &
IsHD.
- Side effects; increase risk of DVT.
not effective against non vert fracture.
Calcitonin; nasal spray, SC; vertebral fracture
( weak evidence) , small BMD.
HRT
Thromboembolism, breast cancer, TG,
risk of fracture, cancer colon, HDL coronary.
Contraindications;
- Absolute;
E dependant tumours; breast, endometrium.
Thromboembolic disorder.
Abn vaginal bleeding.

- Relative;
FH of br cancer,
GB disease.
Endometriosis
Alendronate (oral daily 10 mg or weekly 70 mg)
- mech; taken up by osteoclast apoptosis.
- adv; effective against all fractures, long term benefit (10 yrs).
- disadv; poor upper GIT tolerance.
- contraindications; hypocalcemia, osteomalacia, renal failure, bed
ridden, serious oesophageal dis.
- side effects; hypocalcemia, PTH, skin rash
oral; esophagitis, ulceration.
IV; flu like, arthralgia, myalgia, fever, leukopenia, eye inflammation
etidronate; phosphate, osteomalacia.

Teriparatide; rh PTH
- daily SC, 1st anabolic, all fractures, marked & rapid BMD.
- disadv; daily SC, use restricted to high risk pts, osteosarcoma in
toxicologic studies.
- contraindication; bone dis; paget dis, Hx of bone metastasis or cancer,
growing child, bone, hypercalcemia, pregnant, lactation.
The following subjects are candidates for
BMD except :

1) A lady of 68 years
2) An obese lady at age of 52(menopause)
3) A man of 74 years age
4) A lady 34-year age with 1ry
hyperparathyroidism
A 54-year-old lady advised to do BMD for
persistent spine pain, Which revealed :

** T-score
.. L.spine : -2.6
.. F.neck : -1.7

** All are possible except :


1)Osteoporosis
2)Hyperparathyroidism
3)I.B.D
4)Hypothyroidism
5)Cushing d.
A 52-year-old lady, menopause for 3 years
on longterm steroids for her uncontrolled
chronic asthma

** BMD done for her revealed :


T-score L.spine : -2.0
F.neck : -1.2

** suggested treatment :
1)Calcium + vitamin-D
2)Calcium + vitamin-D + Alendronate
3)Calcium + vitamin-D + Alendronate + Calcitonin
4) No TTT
A 62-year-old lady presented with severe
back pain , she lost about 2 inches of her
height. Plain XR revealed farcture of T12
and wedged L3. She is immobile for
severe knee OA and on steroids for ILD
** BMD advised revealing :
T-score L.spine : -4.2
F.neck : -2.8
** Best TTT is :
1) Ca + vit-D + Alendronate
2) Ca + vit-D + Calcitonin
3) Ca + vit-D + Teriparatide (PTH)
4) Ca + vit-D + Alendronate + Calcitonin
A 58-year-old lady admitted for
fracture neck-femur
** BMD revealed :
T-score L.spine : -1.6
F.neck : -2.8

** The drug being least effective is :


1)Alendronate
2) HRT
3) PTH
4) Raloxifene ( SERM)
Gonoccocal arthritis
is a common cause of septic arthritis in which the organism cannot be cultured on
routine culture media.
Occur in disseminated gonococcal infection.
C/P;
sexually active individuals, ; (4:1), during menstruation and pregnancy.
fever, chills, skin rashes( pustular) , polyarthralgias, and tenosynovitis persistent
monoarthritis or oligoarthritis,
Inv;
The mean synovial fluid WBCs = 50,000 cells/mm3.
Cultures of synovial fluid tend to be positive in < 50 % of cases of gonococcal arthritis
The joint aspirate should be cultured for N. gonorrhoeae when the history is
suggestive. ( chocolate agar or Thayer-Martin medium)
cultures from clinically appropriate sites (eg, the pharynx, urethra, cervix, rectum, and
skin lesions).
Blood cultures are often positive in patients presenting with tenosynovitis and skin
lesions alone,
PCR
screening test for HIV and a syphilis.
TTT; , the initial therapy of choice is ceftriaxone (1 gm either IV or IM) or another 3rd
G cephalosporin.
Fluoroquinolones .
Relapsing polychondritis
Associated with autoimmune disease in 30%.
Leads to fever, arthralgias, episcleritis, swollen
floppy ears.
Nasal Septum collapse (the other 'saddle nose'
deformity, i.e. not just Wegener's).
laryngeal disease hoarseness; respiratory
obstruction.
Tracheobroncial degeneration recurrent
infections,
AI/MVP/ Aneurysm in 10%.
Causes of avascular necrosis
SLE .
vasculitis & any hypercoagulable state.
long term steroids .
sickle cell disease.
Hip involvement.
MRI is the best diagnostic tool.
Causes of Charcots joints:
diabetic neuropathy,
syphillis
syringomyelia
leprosy
Causes of Iritis:
Behcet's disease
Reiter's syndrome
ankylosing spondylitis
sardoidosis
Eosinophilic fasciitis is a disorder
characterized by peripheral eosinophilia and
fasciitis. Swelling and progressive induration of
the skin associated with aching of the
extremities and occasional morning stiffness
develop over a period of weeks. The distribution
most often is in the upper extremity, proximal
and distal to the elbow, and in the lower
extremity, proximal and distal to the knee. Onset
may be acute following some sort of strenuous
exercise, or it may be subacute. The diagnosis is
confirmed by deep biopsy from skin to muscle.
Eosinophilic fascitis

Eosinophilic fascitis
Rheumatologic Manifestations of
sickle cell disease

Gout.
Sickle lower extremity arthralgias (knees and
ankles most common), myalgias, synovitis.
2ry hemochromatotic (Fe overload 2o to
frequent transfusions).
Septic arthritis (or osteomyelitis), esp due to
Salmonella, particularly if hyposplenic.
Aseptic (avascular) necrosis.
Familial Mediterranean fever
AD
is an inherited condition characterized by recurrent
episodes of painful inflammation in the abdomen, chest,
or joints, fevers and rash.
The first episode usually occurs by the age of 20 years,
but in some cases, later in life.
FMF primarily affects populations originating from the
Mediterranean region.
A buildup of amyloid. AA amyloidosis commonly involves
the kidneys, spleen and GI tract.
Colchicine given prophylactically in FMF offers some
protection against the development of amyloidosis in most
patients.
De Quervain's tenosynovitis
is inflammation of the abductor pollicis
longus and extensor pollicis brevis.
Finkelstein's test is positive. This is
performed with the thumb flexed across the
palm of the hand, asking the patient to
move the wrist into flexion and ulnar
deviation. This stresses the tendons of
abductor pollicis longus and extensor
pollicis brevis and reproduces the pain of
de Quervain's tenosynovitis.
A 15-year-old boy presented with arthralgia, skin
rash and haematuria. Renal biopsy showed
focal
necrotising glomerulonephritis with diffuse
mesangial IgA deposits. What is the most likely
diagnosis?
A : Systemic lupus erythematosus (SLE)
B : HenochScholein purpura
C : Juvenile rheumatoid arthritis
D : Post-streptococcal glomerulonephritis
E : Goodpasture's syndrome
24. A 64-year-old man presents to A&E with a 2-
day history of increasing pain and swelling of
his left knee. He denies a history of trauma. On
examination, the knee is hot, red, swollen and
extremely tender. Which of the following
investigations is most important?
A : Plain radiograph of the knee
B : Blood cultures
C : C-reactive protein (CRP)
D : Joint aspiration
E : Plasma uric acid level.
64-year-old man with congestive heart failure presents to the emergency room
complaining of acute onset of severe pain in his right foot. The pain began during
the nightand awoke him from a deep sleep. He reports the pain to be so severe
that he could not wear a shoe or sock to the hospital.His current medications are
furosemide, 40 mg twicedaily, carvedilol, 6.25 mg twice daily, candesartan, 8 mg
once daily, and aspirin, 325 mg once daily. On examination, he is febrile to
38.5C. The first toe of the right foot is erythematous and exquisitely tender to
touch. There is significant swelling and effusion of the first metatarsophalangeal
joint on the right. No other joints are affected. Which of the following findings
would be expected on arthrocentesis?

A. Glucose level of <25 mg/dL


B. Positive Gram stain
C. Presence of strongly negatively birefringent needleshaped crystals under polarized
light microscopy
D. Presence of weakly positively birefringent rhomboidal crystals under polarized light
microscopy
E. White blood cell (WBC) count >100,000/L
A 36-year-old woman presents with deteriorating
nocturnal paraesthesia affecting both
hands. It improves during the morning. Which of
the following tests is least likely to be helpful in
establishing the cause?
A : Thyroid function test
B : Rheumatoid factor
C : Tinels test
D : MRI scan hands
E : Urinary hCG.
A 25-year-old woman with a history of 3-second
trimester fetal losses is planning a fourth pregnancy.
She has evidence of the primary anti-phospholipid
syndrome (strongly positive cardiolipin antibody,
positive lupus anticoagulant but no evidence of lupus).
Which of the following treatment regimens offer her
the best chance of having a successful pregnancy?
A : Steroids alone
B : Steroids combined with low-dose aspirin
C : Low-dose aspirin alone
D : Low-dose aspirin combined with low molecular
weight heparin
E : Intravenous immunoglobulin.
Serum biochemistry of a 60 year old man revealed
calcium of 1.98 mmol/l and phosphate of 0.55
mmol/l with an alkaline phosphatase of 450 IU/l.
Which among the following most suits with the
above serum biochemistry?
1) Osteoporosis
2) Osteomalacia
3) Pagets Disease
4) Secondary Hyperparathyroidism
5) Renal failure
A 36-year old woman is referred with a 1-year history of
muscle pain, tiredness and sleep disturbance. She
denies fever, weight loss and arthralgia. Examination
reveals tenderness over her occiput, trapezius and
lumbar area. Her blood results show a normal ESR,
CRP, FBC, a weakly positive ANA 1:80 and normal
complement. Which is the most likely diagnosis?
A : Polymyositis
B : System lupus erythematous (SLE)
C : Sjogrens syndrome
D : Polymyalgia rheumatica
E : Fibromyalgia.
A 58-year-old female presents complaining of right shoulder pain. She does not
recall any prior injury butnotes that she feels that the shoulder has been
getting progressively more stiff over the last several months. She previously
had several episodes of bursitis of the right shoulder that were treated
successfully with NSAIDs and steroid injections. The patients past medical
history is also significant for diabetes mellitus, for which she takes metformin
and glyburide. On physical examination, the right shoulder is not warm or red
but is tender to touch. Passive and active range of motion is limited in flexion,
extension, and abduction. A right shoulder radiogram shows osteopenia
without evidence of joint erosion or osteophytes.
What is the most likely diagnosis?
A. Adhesive capsulitis
B. Avascular necrosis
C. Bicipital tendinitis
D. Osteoarthritis
E. Rotator cuff tear
A 45-year-old woman is admitted with a spiking temperature
and sweats. She has been unwell for the last 3 weeks with
flitting arthralgia and lethargy. There is a rash over her
trunk which is most prevalent in the mornings. Blood
cultures are sterile. Her recent transthoracic
echocardiogram is normal. ESR 56mm/hour. Her ferritin is
elevated at 6000(g/l. Autoimmune screen is negative.
1) bacterial endocarditis
2) systemic lupus erythematosus
3) rheumatoid arthritis
4) adult onset Stills disease
5) meningitis
A 60-year-old accountant complains of recurrent
attacks of exquisite pain and swelling in
the left big toe. Which of the following conditions is
NOT likely to be associated with this disorder?
A : Chronic alcoholism
B : Obesity
C : Rheumatoid arthritis
D : Diabetes mellitus
E : Diuretic therapy.
A 68-year-old woman with longstanding congestive cardiac
failure (ejection fraction 20%) presents with a hot,
swollen right knee. The following results are obtained:
FBC normal, Urea 11mM, Creatinine 196ul. Synovial
fluid: many monosodium urate crystals seen on
microscopy, culture sterile. What is the best treatment for
her acute arthritis?
A : Allopurinol
B : Colchicine 0.5mg every 2-4 hours
C : Indomethacin 50mg tds
D : Co-codamol 30/500 every 6 hours
E : Intra-articular corticosteroids
A 34-year-old man presents with severe low back
pain, which has forced him to stop work as a bus
driver. He has had back pain on and off for many
years, on occasion with right-sided sciatica. The
pain used to be helped by rest, but is now present
more or less all the time and is stopping him from
sleeping properly. The most likely diagnosis is:
A : mechanical back pain.
B : ankylosing spondylitis.
C : myeloma.
D : osteoporosis.
E : osteoarthritis.
Red flag symptoms, requiring urgent investigation
to exclude sinister pathology, include:
age >55 or <18 years
progressive pain
night pain
systemic symptoms
progressive neurological deficit
past history of malignancy or
immunosuppression
recent trauma.
A 50 year old man presents with a 6 week history of
general malaise and a 2 day history of a right foot
drop, a left ulnar nerve palsy and a widespread
purpuric rash. He complains of arthralgia but has
no clinical evidence of inflammatory joint disease.
Echocardiogram is normal, blood cultures are
negative, ESR 100 mm/hr, ANCA negative, ANA
negative, rheumatoid factor strongly positive, C3
0.8 g/l (0.75 - 1.6), C4 0.02 g/l (0.14 - 0.5).
Dipstick urinalysis shows blood ++ but no protein.
1) ANA negative SLE
2) Cryoglobulinaemia
3) Infective endocarditis
4) Polyarthritis nordosa
5) Rheumatoid arthritis
A patient with primary Sj grens syndrome that was
diagnosed 6 years ago and treated with tear
replacement for symptomatic relief notes continued
parotid swelling for the last 3 months. She has also
noted enlarging
posterior cervical lymph nodes. Evaluation shows
leukopenia and low C4 complement levels. What is the
most likely diagnosis?
A. Chronic pancreatitis
B. Secondary Sj grens syndrome
C. HIV infection
D. Lymphoma
E. Amyloidosis
A 64-year-old man with coronary artery disease an atrial fibrillation is
referred for evaluation of fevers, arthralgias, pleuritis, and malar
rash. The symptoms have developed over the past 6 months. The
pleuritis has responded to steroid therapy, but prednisone has been
unableto be tapered off due to recurrence of symptoms at daily
steroid doses <15 mg of prednisone. His medications include
aspirin, procainamide, lovastatin, prednisone, and carvedilol. At this
stage antibodies directed against which of the following proteins is
most likely to be positive?
A. Cardiolipin.
B. Double-strand DNA.
C. Histone.
D. Ribonucleoprotein (RNP).
E. Ribosomal .
A 69-year-old woman with a 5-year history of intermittent left knee pain owing
to osteoarthritis presents to your office 5 days after vacationing in Greece,
where she developed increased knee pain and swelling. She says that she had
been walking and climbing more than usual on this vacation. She and some of
the vacationers in her group developed watery diarrhea around the same time,
but there were no chills or fever. She had been treated with naproxen until 2
years ago, when she developed atrial fibrillation. She now takes
acetaminophen as needed for knee pain and takes warfarin for the atrial
fibrillation. Physical examination shows a temperature of 38 C (100 F). There
is a large effusion in the left knee, with increased warmth but no redness.
There is moderate tenderness and pain on motion, and she walks with a limp,
which she did not have before. The other joints are unchanged.
Laboratory studies
Hemoglobin 13.7 g/dL Hematocrit 42%
Leukocyte count 10,000/L Neutrophils 80%
INR 2.1 Left knee radiograph reveals changes of osteoarthritis and an
effusion.
Which of the following would you do next?
(A) Obtain stool culture and start treatment with ciprofloxacin.
(B) Reduce warfarin dose and start treatment with celecoxib.
(C) Order MRI of left knee.
(D) Perform arthrocentesis.
(E) Admit to hospital and start treatment with intra- venous ceftriaxone.
A 28-year-old woman presented with fatigue and extreme
tiredness. Physical examination
revealed facial skin rash and tenderness across the small
joints of the hands. She was concerned
that she might have systemic lupus erythematosus (SLE).
Which of the following tests when
NEGATIVE will virtually exclude the diagnosis of SLE?
A : Antinuclear antibody (ANA)
B : Anti-double stranded DNA (Ads-DNA)
C : Anti-Sm antibodies
D : Anti-histone antibodies
E : Anti-Ro/SSA antibodies.
35. A 60-year-old-man complained of pain in both wrist
evolving over 8 weeks. He noted swelling around that area
but denied stiffness. On examination there was swelling
and tenderness just proximal to the wrist joints without
limitation of movement. There was also prominent finger
clubbing. Radiographs revealed periosteal reaction over
the lower end of the radius and ulnar.
Each of the following disorders could be the cause behind
this patient complain EXCEPT?
A : Mesothelioma
B : Bronchiectesis
C : Diabetes mellitus
D : Crohn's disease
E : Whipple's disease.
THANK YOU
Rheumatology
* SYNOVIAL FLUID ANALYSIS
FEATURE Normal NONINFLAMMATORY INFLAMMATORY SEPTIC
(eg osteoarthritis) (eg RA, crystal-induced,
spondyloarthropathies,
connective tissue dz, HOA)

WBCs/m <200 200- 2000 2000 -50.000 >100,000


m3
Polymalgia Rheumatica
Diagnostic Criteria for PMR
Age >50.
ESR>40mm/h.
Neck/bilateral shoulder / pelvic girdle AM stiffness.
Symptomatic episodes > 1h in duration.
Clinical history >1 month duration.
Response to low-dose (15mg/day) steroids.
Remember, PMR patients complain more of stiffness than pain.
Minor criteria include:
Weight loss; fever; night sweats,Synovitis.
Symmetrical proximal muscle tenderness( no weakness).
Alk phos / GGT.
Normocytic anemia.
Normal CPK, EMG, muscle biopsy.
50% of all PMR patients (TA symptoms) will be found to have giant-cell
arteritis on TA biopsy.
fibromyalgia
Diagnosis of exclusion.
Female > male .
Pain must be present for > 3m.
Multiple tender points at least 11points .
Pts are poorly able to localize the pain.
Nervousness is frequent concomitant.
TTT: Aerobic exercise.
TCA. ( Tricyclic antidepressant).
DD OF MORNING STIFFNESS/
PAIN WORSE IN AM
Rheumatoid arthritis.
Difficulty doing up buttons
Ankylosing Spondylitis.
LBP + stiffness radiating to buttocks
and thighs
polymaylgia Rheumatica (PMR)
Difficulty getting out of bed
Fibromyalgia
Osteoarthritis

Bouchards nodes are proximal interphlangeal


nodes.
Heberden's nodes are distal interphalangeal
nodes.
X ray features of osteoarthritis are: joint space
narrowing ,osteophytes , subchondral
sclerosis & subchondral cysts.
Rheumatoid arthritis
Seven diagnostic criteria for
Rheumatoid arthritis
Morning stiffness.
Arthritis of 3 joint area.
Arthritis of the hand joint.
Arthritis symmetric.
Rheumatoid nodules.
Serum Rheumatoid factor.
Radiographic changes.
* In order to have RA, you must have 4 of
the 7 criteria present for at least 6 weeks.
Symptoms and signs

Predominantly peripheral joints are affected


symmetrical joint pain, effusions, soft tissue
swelling, early morning stiffness. Progression
to joint destruction and deformity.
Nodes and nodules in rheumatic
diseases

1. Heberden's nodes DIP osteophytes in 1ryOA.


2. Bouchard's nodes PIP osteophytes in 1ry OA.
3. Rheumatoid nodules.
4. Gouty tophi (elbow, ears, heels, PIPs, DIPs).
5. Xanthomata.
Investigations

Check FBC (normochromic normocytic ),


ESR, and/or CRP, platelets &WCC are
increased.
Rheumatoid factor and anti-CCP antibodies
are +ve in the majority 60- 70%. A minority
have a +ve ANA titre.
X-rays normal, periarticular osteporosis or
soft tissue swelling in the early stages;
Later---- loss of joint space, erosions, and
joint destruction.
RHEUMATOID ARTHRITIS OSTEOARTHRITIS

ETIOLOGY Autoimmune disease Degenerative disease


PATTERN of Morning stiffness Worse after effort or as
symptoms activity progresses,
relived by rest
Predominant PIP, MCP, wrists (symmetric) CMC, PIP, DIP
HAND joint
involvement
LAB +RF, ESR and C-reactive Normal RF, ESR, CRP
protein
XRAY findings 1. Periarticular osteopenia. 1. Osteophyte formation
2. Marginal bony erosions. 2. Subchondral sclerosis
3.Symmetrical narrowing of (no periarticular
the joint space. osteopenia)
3. Joint space narrowing.
DD of symmetrical polyarthropathy
Rheumatoid Arthritis.
Viral arthropathy.
Psoriatic polyarthropathy.
DMARDs (Disease- modifying
antirheumatoid drugs):
2 Key Points:
Do not wait to start patients on DMARDs until
they have failed multiple courses of NSAIDs
but, as soon as the diagnosis of RA is
confirmed.
Methotrexate is the DMARD of choice in
patients with severe disease. Start therapy
with 7.5mg weekly and raise the dosage at 1-
intervals until peak efficacy is achieved.
* DMARD Therapy for RA ("GOLD PILE SCAM")

DMARD Potential Toxicities Requiring F/U Monitoring Studies

Gold IM and PO Myelosouppression, proteinuria CBC and urine dipstick for protein

Penicillamine Myelosuppression, proteinuria CBC and urine dipstick for protein

Infliximab (Remicade) Flu-like sx, auto-Abs; for patients not responding to None
methotrexate; given IV

Leflunomide (Avara) Thrombocytopenia, hepatotxicity, diarrehea CBC and AST

Etanercept (Enbrel) (a TNF blocker) Reactions at site of SQ injection, flu-like sx None

Sulfasalzine Myelosuppression CBC, AST, creatinine

Cyclophosphamide Myelosuppression, myeloproliferative disorders, CBC, urinalysis, and urine cytology


malignancy, hemorrhagic cystistis

Cyclosporine Renal insufficiency, anemia, HTN, hirsutism creatinine, CBC, K+, LFTs

Azathioprine Myelosuppression, hepatotoxicity, CBC


lymphoproliferative disorders

Antimalarials (Hydroxycholorquine) Macular damage Yearly fundus exams

Methotrexate Myelosuppression, hepatic fibrosis, cirrhosis, CBC, AST, albumin


pulmonary infiltrates or fibrosis

Minocycline Photosensitivity, skin discoloration, GI upset, drug- None


induced hepatitis, dizziness
Juvenile chronic arthritis
is rheumatoid factor negative.
Different classifications are systemic, pauciarticular
and polyarticular.
Commonest type is Stills disease.( Systemic
onset juvenile R.A ). Peaks of disease are about 5
years and 15 years of age.
Typical diagnostic criteria include High spiking
fevers; arthralgias/ macular rash, hepatomegaly,
splenomegaly, lymphadenopathy, serositis
(pleuritis, pericarditis), seronegativity (-veRF; -ve
ANA); leucocytosis. Thrombocytosis.
Very high serum ferritin.
Diagnosis of Felty's Syndrome
Neutropenia caused by hyperslenism and
antineutrophil antibodies in a patient with long term
rheumatoid arthritis.
Triad of Neutropenia; Seropositive RA;
Splenomegaly /hyperslenism.
Frequent concomitants
Serious infections
Anemia, thrombocytopenia.
Leg ulcers
LN, Hepatomegaly
Sjogren's synd.
Weight loss.
Still's Disease Felty's Syndrome

WBC--inc WBC---dec

Fever No fever necessary

Seronegative RA Seropositive RA

Splenomegaly splenomegaly
Systemic lupus erythematosus
9:1.and Asians. Onset 15-40y. Presentation is variable multisystem
involvement must be demonstrated to make a diagnosis:
Joints (95%)arthritis, arthralgia, myalgia, tenosynovitis
Skin (80%)photosensitivity, facial butterfly rash, vasculitis, hair loss,
urticaria, discoid lesions
Lungs (50%)pleurisy, pneumonitis, effusion, fibrosing alveolitis
Kidney (50%)proteinuria, BP, glomerulonephritis, renal failure
Heart (40%)-pericarditis, endocarditis
CNS (15%)depression, psychosis, infarction, fits, cranial nerve
lesions.
Blood; anaemia, thrombocytopoenia, splenomegaly
Diagnostic criteria for SLE
4 of 11 for 6 weeks;
1. butterfly rash
2. discoid rash.
3. Photosensitivity
4. Oral ulcer
5. Arthritis
6. Serositis
7. Nephrological; lupus nephrotis
8. Neurological
9. Hematologic; lymphopenia, thrombocytopenia.
10. Immunological; anti DNA
11. ANA
Treatment;
Skin only chloroquine.
JOINT only; NSAIDs.
Other extraarticular; steroid & other
immunosuppressants.
Drug-induced lupus
Occurs with minocycline, isoniazid,
hydralazine, procainamide,BB,
chlorpromazine, sulfasalazine, losartan, and
anti-convulsants. Remits slowly when drug is
stopped but may need steroid treatment to
settle.
No nephrological, no neurological,
DNA ve
Anti histone Ab +ve.
TYPES OF ANTINUCLEAR ANTIBODIES
(ANA)
Following detection of a high titer of ANAs (e.g. 1:160), various
subtypes are determined. Examples include:
Anti-dsDNA (double-stranded DNA) specific for SLE
Anti- histones Drug induced lupus
Anti-chromatin= anti-nucleosome antibodies.
Anti-ENA (Extractable nuclear antigen)=non-DNA = RNP
Anti-Ro (SS-A)
Anti-La (SS-B)
Anti-Sm (Smith antigen)
Anti-U1RNP (nuclear ribonucleoproteins)
Anti Scl-70 (topoisomerase I)
Anti-Jo
Antinucleolar20% of systemic sclerosis .
Anti-centromere
Antibodies to nuclear pore proteins as anti-gp- APA.
Anti-ENA (Extractable nuclear
antigen)

Ro/SSA Sjgren's syndrome , 40% SLE, ANA -ve


SLE, subacute cutaneous LE & neonatal LE.
La/SSB 1ry Sjogren.
Anti- Smith (Sm) Ab is very specific for SLE, but
only in 25 % .
Antibodies to U1-RNP 100% of MCTD, 30-40 % of
SLE (+ anti- Smith) & small proportion of patients with
localized (linear) scleroderma.
Antibodies to topoisomerase I (Scl-70), RNA
polymerases I and III systemic sclerosis related
disorders.
Anti-Jo1 PM especially with IPF.
Chromatin= histones + DNA.
Anti-chromatin= anti-nucleosome antibodies.
DD of sponylitis

1. Sero ve Spondyloarthoropathies.
2. infections. ( TB , Brucella ).
Spondyloarthoropathies
A- HLA-B27 Rheumatic Diseases:
1. Ankylosing Spoondylitits (<90% are HLA-B27).
2. Reiter's Syndrome or
3. Reactive arthritis (<80%).
4. Enteropthic spondylitis (75%).
5. Psoriatic Spondylitis (50%).
B- Characteristis:
1- Sacroiliac joint involvement.
2- Peripheral arthritis (usually asymmetric and oligarticular).
3- Seronnegativity (absence of RF or other autonatibodies).
4- Association with HLA-B27.
5- Relatively early age of onset (<40).
6- Enthesopathy.
7- Anterior uveitis.
1- Radiographic findings:
a) Erosions.
b) Syndesmophytes.
c) Bamboo spine.
D) fusion.
2- Enthesopathic involvement
a) Plantar fasciitis.
b) Achilles tendonitis.
3- Iritis (an important clue in spondyloarthropathies).
ankylosing spondylitis
Schober's test

Schober's test assesses the amount of lumbar flexion. In this test a mark
is made at the level of the posterior iliac spine on the vertebral column,
i.e. approximately at the level of L5. The examiner then places one
finger 5cm below this mark and another finger at about 10cm above this
mark. The patient is then instructed to touch his toes. If the increase in
distance between the two fingers on the patients spine is less than 5cm
then this is indicative of a limitation of lumbar flexion.
TTT;
1. Symptoms modifying; NSAIDS
2. Disease modifying; infliximab,
methotrexate.
Reactive arthritis Often asymmetrical aseptic arthritis
Occurs 2-6wk. after bacterial infection elsewhere e.g.
gastroenteritis (salmonella, campylobacter), GU infection
(chlamydia, gonorrhoea).
Management: NSAIDs, physiotherapy, and steroid joint
injections. Recovery usually occurs within months. A minority
develop chronic arthritis requiring disease-modifying drugs.
Reiter's syndrome: Polyarthropathy, urethritis, conjuctivitis.
Affects men with HLA B27 genotype. Commonly follows GU or
bowel infection. Joint and eye changes are often severe.
characteristic mucocutaneous changes
oral ulcers , circinate balanitis , keratoderma blenorragicum.
Enteropathic spondylarthropathy
Oligoarticular or polyarticular arthritis linked to inflammatory
bowel disease. Includes sacroiliitis, plantar fasciitis,
inflammatory spinal pains, and other enthsitides (insertional
ligament/tendon inflammation). Arthritis may evolve and
relapse/remit independently of bowel disease. NSAIDs may
help joint pain but aggravate bowel disease.
circinate
balanitis
Psoriatic arthritis
Psoriatic arthritis
Inflammatory arthritis associated with psoriasis. RhF -ve.
Presentation variable. Disease modifying drugs (e.g.
methotrexate) may improve both skin and musculoskeletal
symptoms .

presents as several forms:


Symmetrical polyarthritis.
Arthritis involving DIP joints.
Asymmetrical oligoarthritis including dactylitis (70%)
Ankylosing Spondylitis type.
Arthritis mutilans.
Among the differentials of ank. Spond. Is DISH.
(Diffuse Idiopathic Skeletal Hyperostosis,
a form of 1ry osteoarthritis).

a) Patients with DISH are often obese and 60% have diabetes;
b) 'stiffness' at the spine' yet relatively well-preserved spinal motion.
c) Criteria for DISH'
'Flowing' ossification along the anterolateral aspects of 4 contiguous verterbral
bodies with preservation of disk height.
Absence of SI joint involvement.
Intervertebral disk spaces are preserved.
Marked calcification and ossification of paraspinous ligaments occur
in DISH. Ligamentous calcification and ossification in the anterior spinal
ligaments give the appearance of "flowing wax" on the anterior vertebral
bodies. However, a radioucency may be seen between the newly
deposited bone and the vertebtral body,
Differentiating DISH from the marginal osteophytes in spondylosis.
Diffuse idiopathic skeletal hyperostosis =DISH
vasculitis
Group of diseases in which tissue ischemia and necrosis occur as a consequence of
inflammation of blood vessels, either as a primary event or secondary to a
systemic disease .1.Autoimmune dis. 2. Infection 3.Malignancy 4. drugs .

Large vessel vasculitis: chronic granulomatous reaction


affecting the aorta and major branches.
Medium vessel vasculitis: necrotizing arteritis affecting
visceral arteries renal artery leading to renal infarction .
Small vessel vasculitis: capillaries, venules, arterioles
(glomeruli)leading to glomerulonephritis ..
Clinical Presentation

Constitutional symptoms.
Skin: purpura,ulcers , livedo reticularis , nail bed
infarcts ,digital gangrene.
Eye :episcleritis ,ulcers and vision loss.
Lung: dyspnea, hemoptysis.
Cardiac: angina, myocardial infarction , heart failure.
GIT: abdominal pain, mesenteric
ischemia,malabsorpt.
Kidney: hematuria, proteinuria, acute/chronic renal
failure
CNS: Mononeuritis multiplex, sensorymotor
neuropathy , convulsions, hemiplegia.
Names and definitions of vasculitides
Large vessel vasculitis
Giant cell (temporal) arteritis
Granulomatous arteritis of the aorta and its major branches,
with a predilection for the extra cranial branches of the
carotid artery. Often involves the temporal artery. Usually
occurs in patients older than 50 and often is associated with
polymyalgia rheumatica.
Presentation (classical) of Temporal Arteritis Headache,
scalp tenderness; jaw claudication; sudden visual loss.
50% of TA patients have symptoms of PMR.
Flase-negative biopsies may occur 2o to presence of "skip lesions" (if
a 2cm biopsy is taken the false-neg rate is >5%; therefore a 3-5cm
segment biopsy is recommended).
Takayasu's arteritis

Granulomatous inflammation of the aorta and its


major branches. Usually occurs in patients
younger than 50.
Middle aged women
0f patients presents with initial systemic
illness of malaise, fever, night sweats, weight
loss, arthralgia, and fatigue then subside with
more gradual vascular changes.
The other half of patients with Takayasu's
arteritis present with only late vascular
changes, without an antecedent systemic
illness.

It is also known as "Pulseless disease" because pulses on the upper


extremities, such as the wrist pulse, may not be able to be felt.
Medium sized vessel vasculitis

Polyarteritis nodosa.
(classic polyarteritis nodosa)
Necrotizing inflammation of medium-sized without
glomerulonephritis or vasculitis in arterioles, capillaries, or
venules.

Kawasaki disease .
Arteritis involving medium sized , and associated with
mucocutaneous lymph node syndrome.
Fever of 5 days' duration associated with at least 4 of the
following 5 change.

Bilateral nonsuppurative conjunctivitis

One or more changes of the mucous membranes of the upper


respiratory tract, including pharyngeal injection, dry fissured lips,
injected lips, and "strawberry" tongue

One or more changes of the extremities, including peripheral


erythema, peripheral edema, periungual desquamation.

Polymorphous rash, primarily truncal

Cervical lymphadenopathy >1.5 cm in diameter


its most serious effect is on the heart where it can cause severe coronary artery
aneurysms in untreated children.
Differential diagnosis, including scarlet fever, toxic shock
syndrome, juvenile idiopathic arthritis.
There is no specific laboratory test for this condition
Blood tests
CBC reveal normocytic anemia and eventually thrombocytosis
ESR will be elevated
CRP will be elevated
Other optional tests
Electrocardiogram may show evidence of ventricular
dysfunction or, occasionally, arrhythmia due to myocarditis .
TTT:
1.Intravenous immunoglobulin (IVIG) is the standard treatment
for Kawasaki disease[
2.Salicylate therapy, particularly aspirin, remains an important
part of the treatment
Polyarteritis nodosa
Necrotizing vasculitis causing microaneurysms
in the medium sized arteries.
HBsAg
Constitutional symptoms: Fever , abdominal
pain,weight loss, arthralgia.
Cardiac: coronary arteritis , hypertension , heart
failure ,pericarditis.
CNS: mononeuritis multiplex, sensorymotor
polyneuropathy. seizures, hemiplegia.
GIT: abdominal pain, malabsorption
Skin : purpura, urticaria, infarcts, livedoreticularis.
Blood: eosinophilia ,anaemia , ESR , CRP.
Kidney: hypertension, hematuria, proteinuria,
renal failure, intrarenal aneurysms.
Testicular pain.
ANCA is ve.
Diagnosis by nerve biopsy of affected nerve .
If affected tissue not accessible ....visceral Angio.
Small vessel vasculitis
ANCA Positive:
Wegener's granulomatosis
Granulomatous inflammation involving the respiratory tract, and
necrotizing vasculitis (Necrotizing glomerulonephritis is common).
Churg Strauss syndrome
Eosinophil-rich and granulomatous inflammation, necrotizing vasculitis
involving the respiratory tract, with asthma and eosinophilia.
Microscopic polyangiitis (microscopic polyarteritis)
Necrotizing vasculitis, with no immune deposits, affecting small vessels .
Necrotizing glomerulonephritis is very common. Pulmonary capillaritis
often occurs.
Small vessel vasculitis
ANCA Negative:
Henoch Schnlein Purpura.
Vasculitis, with IgA-dominant immune deposits. Typically
involves skin, gut, and glomeruli, and arthralgia or arthritis.
Essential cryoglobulinaemic vasculitis.
Vasculitis, with cryoglobulin immune deposits, affecting small
vessels, and associated with cryoglobulins in serum. Skin
and glomeruli are often involved.
Cutaneous leucocytoclastic angiitis .
Isolated cutaneous leucocytoclastic angiitis without systemic
vasculitis glomerulonephritis.
What is ANCA?
Autoantibodies against intracellular
neutrophil antigens.
C-ANCA cytoplasmic anti PR3.
P-ANCA perinuclear anti MPO.
vasculitis with a predilection for renal involovement?
Small vessel vasculitis.
Medium vessel vasculitis : PAN
How does vasculitis affect the kidney?
Urine analysis shows RBCs ,red cell casts and proteinuria
common but usually < 3 gm.
Pathology
The shared glomerular lesion of the pauci-immune small-
vessel vasculitides is a necrotizing glomerulonephritis with
crescent formation.
Early lesions have segmental fibrinoid necrosis with or
without an adjacent small crescent.
Severe lesions have global necrosis with large
circumferential crescents.
Renal arteritis affecting the interlobular arteries.
Wegner granulomatosis
Necrotizing granuloma in the
respiratory tract(nasal ulcers,
epistaxis , sinus involvement,
otitis media, oral ulcers
,proptosis).
hemoptysis
Rapidly progressive
glomerulonephritis.
C-ANCA +ve in 70%
Wegener Granulomatosis
Clinical Presentation
upper respiratory tract
Lung involvement in 80%; transient infiltrates or nodular densities may be seen
on (CXR)
Histopathology
(1) vasculitis of small vessels,
(2) granulomatous changes,
(3) focal necrosis
LM :segmental necrotizing.
crescents

IF: Pauci-immune GN;


crescents;
tubulointerstitial granulomas

EM: No immune deposits are seen


Laboratory Diagnosis
cANCA is specific and sensitive
20% to 30% have pANCA
Segmental fibrinoid necrosis with nuclear debris and
glomerular basement membrane disruption, typical
of early stage of pauci-immune crescentic
glomerulonephritis
Well-developed cellular crescent with collapse of
small amount of remaining glomerular tuft with
segmental fibrinoid necrosis and extracapillary
fibrin and necrosis
Churg-Strauss Vasculitis
Small vessel vasculitis .
eosinophilic pulmonary infiltrates
Allergic rhinitis.
Nasal polyposis.
Asthma
Eosinophilia
Necrotizing inflammation
Almost no kidney affection
glomerulonephritis is usually (but not always) mild.
Associated with leukotriene antagonist therapy,
P-ANCA Usually in 60%. (MPO) positive. +ve
Microscopic polyangitis
Small vessel vasculitis
Absence of asthma ,eosinophilia,necrotizing
inflammation.
P-ANCA is +ve in 50%
Kidney : Rapidly progressive glomeruloneph.
Treatment for ANCA-associated disease
combination of glucocorticoids (1 mg/kg/d) and IV or
oral cyclophosphamide (2 mg/kg/d) for 1 year after
remission.
methylprednisolone at 7 mg/kg/d IV for 3 days, then
oral prednisone at 1 mg/kg/d.
plasmapheresis in who RPGN or pulm Hge.
Sulfamethoxazole/trimethaprim may diminish relapse
rates.
Course and Prognosis
90% 1-year mortality if left untreated, 90% remission
if treated.
Relapse occurs in 25% to 50% of patients followed
up for 3 to 5 years
Henoch-Schnlein purpura
systemic vasculitis mostly seen in children. It is a multisystem disorder involving
the skin, joints ,gastrointestinal and renal tracts.

Aetiology is unknown, but the syndrome is often preceded by infections


such as Group A beta hemolytic streptococcal

Pathology is due to intravascular deposition of IgA immune complexes.

Patients often present with a skin(palpable purpuric eruption purpuric rash usually
involving the buttocks and lower limbs, arthralgia and severe colicky
abdominal pain and tenderness caused by vasculitis-induced thrombosis in
the gut.
Renal involvement commonly presents as microscopic haematuria and
proteinuria Hematuria 50%/asymptomatic hematuria proteinuria 40%, nephrotic
syndrome ,acute and chronic renal failure.
Similar to IgA nephropathy , but crescents, are more frequent
Immunoflorescence :IgA mesangial deposition.
Course.
Overall good renal outcome; complete recovery
Therapy.
Symptomatic for mild cases.
For severe nephritis, treatment is steroids+/-cyclophosphamide.
Can complement levels help diagnosing vasculitis?

Exclude infection.
Low in SLE ,Cryoglobulinemia .
Normal in ANCA +ve vasculitis.
essential mixed cryoglobulinemia
essential mixed cryoglobulinemia was
appropriate. complexes in which one
component, usually IgM, exhibits antibody
activity against IgG (i.e., mixed
cryoglobulins).
The diagnosis of mixed cryoglobulinemia is
typically made from the history, skin
purpura, low complement levels, and
demonstration of circulating cryoglobulins.
CLINICAL PRESENTATION of Mixed
Skin manifestations CGs
Palpable purpura (80% in mixed types)
Ischemic necrosis ( 020% in mixed types)
Livedoid vasculitis (1% in type I, 14% in type III)
Cold-induced urticaria (15% in type I, 10% in type III)
Acrocyanosis
Nailfold capillary abnormalities
Musculoskeletal manifestations
Arthralgias (5% of type I, 20-58% of mixed)
Frank arthritis and progressive joint deformity (distinctly rare)

such as arthralgias and myalgias are common in the mixed


CGs, but frank arthritis or myositis are rare to uncommon.,
especially affecting the metacarpophalangeal, proximal
phalangeal, knees, and ankles,
Neuropathy
Sensorimotor neuropathy affects a high percentage of mixed, but clinically
significant neuropathy is uncommon., Visual disturbances
Renal manifestations
Membranoproliferative glomerulonephritis described in all types (more
common in type II)
Intraluminal cryoglobulin deposition
Hypertension
Nephrotic-range proteinuria with resultant edema
Pulmonary manifestations
Dyspnea
Cough
Pleurisy
Pleural effusions
Gastrointestinal manifestations
Abdominal pain (2-22%)
Hemorrhage
Hepatomegaly or signs of cirrhosis (ie, palmar erythema, abdominal wall
collateral vessels, spider angiomata)
abnormal liver function tests
Splenomegaly
lymphadenopathy
Cryoglobulinemic Glomerulonephritis can be
differentiated from idiopathic MPGN by the following
findings
.
(1) Intracapillary thrombi the presence of large
deposits filling the capillary lumen.
(2) Massive infiltration of capillary spaces by
monocytes and polymorphonuclear cells (the
exudative component ).
(3) possibly some vasculitis in small and medium-
sized renal arteries .(25% to 33%).
The combination of vasculitis and glomerular thrombi
should suggest cryoglobulinemic nephropathy, but
proliferative lupus nephritis with associated vasculitis
is also in the differential diagnosis.
Aggressive therapy in idiopathic mixed
cryoglobulinemia reserved for patients with acute
severe disease (manifested by progressive renal
failure, distal necroses requiring amputation, or
advanced neuropathy).
plasmapheresis (remove the circulation
cryoglobulins) with steroids (1000 mg of
intravenous methylprednisolone daily for three
days , followed by oral prednisone) and
cyclophosphamide.
Rituximab is a promising alternative treatment
option for severe cryoglobulinemic vasculitis and
nephritis.
( Hughes Syndrome),

Anti-Phospholipid Syndrome
disorder characterized by multiple
different antibodies that are associated
with both arterial and venous thrombo.

.
There are three primary classes of
antibodies associated with the
antiphospholipid antibody syndrome:
1) anticardiolipin antibodies.
2) the lupus anticoagulant .
3) antibodies directed against beta-2-
glycoprotein 1.
Sensitivity and Specificity for APS
APL Test Sensitivity Specificity
ACL 80-90% Low
LA Moderate High
2-GP I 74% 85%
APhL 91% 99%
Harris. Lupus, 1998.
A patient must meet at least
one clinical and one
laboratory criterion for a
diagnosis of APS
Thrombosis
Venous and arterial circulations
Embolic risk high
Venous > Arterial
Occurs in any system or organ with any part of
vascular tree involvement possible(Large-
small)
Single or multiple vessel involvement
Recurrent events common
Catastrophic anti-phospholipid
syndrome (CAPS)
This syndrome, termed "catastrophic
antiphospholipid syndrome," is defined by the clinical
involvement of at least three different organ systems
over a period of days or weeks with histopathological
evidence of multiple occlusions of large or small
vessels.
steroids or aggressive immunosuppression is not
used unless recurrent thrombotic or ischemic events
despite high intensity warfarin therapy.
High dose steroid is reserved for treatment of
underlying active lupus and not for lab
antiphospholipid AB.
Rituximab
Is a humanized mouse monoclonal antibody
against the B cell antigen CD20 that results in
the prolonged depletion of B cells.
CD20 is relatively selectively expressed on pre-
B and mature B cells, but not plasma cells or T
cells, and may selectively treat antibody-
mediated disorders with less immune
suppression than other treatment regimens.
Vasculitis mimickers
Subacute bacterial endocarditis.
Left atrial myxoma
HIV
Paraneoplastic syndrome
Cholesterol embolization
Cocaine and amphetamine use.
Treatment
Combined treatment with steroids and
cyclophosphamide induces remission in 75% of ptns
at 3months and 90% at 6 months.
Pulse steroid therapy 7mg/kg for 3 days(IV methyl
prednisone)
IV cyclophosphamide therapy 0.5g/m2 monthly
reaching 1g/m2 based on the leucocyte count.
Role of plasma exchange is controversial ,beneficial
in dialysis dependant renal failure patients and those
with pulmonary hge .
Crystal induced Arthropathy
A 48-year-old gentleman presented with arthritis of
rigt ankle for one day.
.. S.uric acid : 7.8

.. Synovial fluid : inflammatory with MSU crystals

.. 24-h urine uric acid : 1200 mg.

The most likely cause.

1- hyperparathyroidism 2- hypothyroidism
3- MPD 4-Lactic acidosis
5- thiazide therapy
A 52-year-old lady , diagnosed to have gouty arthritis
two years back with synovitis right MTP1., subsided with
local steroid injection.

.. No further attacks since then

..s.Uric acid ranging from 6.8 8.2


..s.cr : o.8 , No renal stones
.. No tophi.

.. She has to start Allopurinol


** YES ** NO
A 58-year-old gentleman , first attack of gout in R
knee persistent in the last 6 days

.. The drug to be least effective at this stage :

1- NSAID
3- Colchicin 4- local steroids
5- Oral PDN
A 52-year-old gentleman upon the first attack of
gouty arthritis in L MTP1, started Colchicin for the
last 2 weeks. Inflammation subsided

.. S.uric acid now is 13 , supposed to start Allopurinol

.. You will combine Colchicin until :


1) 3 months 2) 6 months
3) s.U.A < 6 4)s.U.A < 7
5) 6-months after s.U.A < 6
A 48-year-old lady accidentally discovered to have
s.UA of 8 mg%

.. 24-h urine uric acid was found 1200 mg

.. Best way of management :


1) Start Colchicin
2) Observation with regular monitoring
3) Start Prebencid
4) Start Allopurinol
5) Start Allopurinol & Colchicin
A 54-year-old male , diagnosed as gouty arthritis,
first attack.

.. S.UA : 8
.. S.creatinin : 1.8
.. C.cr : 35 ml/min

Best way of managemeent :


1) Colchicine 0.5 mg OD
2) Colchicine 0.5 mg BD
3) Avoid Colchicine
A 52-year-old lady, four weeks after first attack of
gouty arthritis, lab. Investigations revealed :

.. S.UA : 10
.. S.cr : 1.9
.. Ccr : 40ml/min

Best way of management :


1) Start Allopurinol 50mg/d,increase to 100 after 2w
2) ,, ,, 100mg/d ,, 200 ,,
3) ,, ,, 300 mg/d
4) ,, ,, 100mg/d, build up to 300mg
5) No need for Allopurinol.
Hyperuicemia
Gout negatively birefringent crystals
Hyperuicemia is either 2ry to overproduction (> 800mg
/ d) 10% or underexcretion (<600mg / 24hrs) 90% .
Causes of underexcretion . HARD to ..
HTN.
Hypothyroidism.
Hyperparathyroidism.
Acidosis.
Renal insuffeciency.
Drugs( thiazides duiretics, Alcohol)
Causes of overproduction
Tissue damage . (Rhabdomy, tumor lysis,
hemolytic process, exercise).
Proliferative disorders.( mylo& lymphoprolif,
psoriasis)
Gout is associated with :
Hypertriglyceridaemia, diabetes, obesity.
serum hyperuicemia is neither necessary nor
sufficient to make the diagnosis of gout.
A synovial fluid aspirate is necessary for definitive
diagnosis. Always remember, for definitive
diagnosis, to send the synovial fluid for crystal
analysis, gram stain.
Alcohol/diet/diuretics are the most common
cause of acute gout in the outpatient
population.
Gout is very uncommon among
premenopausal women.
When to treat asymptomatic hyperuricemia?
** Three specific circumstances warrant at least consideration
for institution of hypouricemic drugs :

1) Persistent s.urate >13mg/dl in men and >10 mg/dl in women

2) Excretion of U.uric acid >1100 mg/d is associated with 50%


risk of uric acid calculi.

3) Patient about to receive radio or chemotherapy that is likely


to result in extensive tumour lysis
Renal disease is a limitation for both
allopurinol & colchicine. With allopurinol,
there is a consensus that you need to start
with a lower dose and only gradually
increase it.

Uricosurics are not effective if the serum


urate is over about 12 mg/dL, Uricosurics
also increase the risk of nephrolithiasis.

colchicine works only if you use it fairly


early in the attack.
Benzbromarone Benzbromarone, a
uricosuric agent that reduces serum uric acid
levels in transplant recipients with
hyperuricemia and gout.
Benzbromarone is an effective uricosuric,
even in patients with creatinine clearances
as low as 20 mL/min
1. Treat acute flares with antiinflammatory
drugs; NSAIDs, colchicine or steroids.
2. Initiate urate lowering drug (Allopurinol)
weeks after any acute flare.
3. Choose effective & safe drug e.g. allop
allergy, renal disease.
4. Protect against flare from urate lowering drug
by adding anti-inflamatory for 6 months after
achieving the target.
5. Continue life style modification
6. Monitor/ 2-4 wks till target uric acid< 6 mg/dl.
7. Then Monitor every 6-12 months.
Pseudogout
Inflammatory arthritis due to deposition of pyrophosphate crystals.
Chondrocalcinosis may be seen on X-ray (calcification of articular
cartilage). Knee, wrist, and shoulder are most commonly affected.

causes are:
Haemochromatosis.
wilson's disease .
Hypothyroidism.
hyperparathyroidism.
hypomagnesaemia.
hypophosphataemia
Amyloidosis.
Aging.DM
Chronic renal failure on dialysis.
Acute attacks can be triggered by intercurrent illness and metabolic
disturbance. Attacks are less severe than gout and may be difficult to
differentiate from other types of arthritis. Presence of joint crystals
confirms diagnosis.
Pseudogout positively birefringent crystals .
Pseudogout
(calcium pyrophosphate "CPPD")
Can mimic and coexist with gout.
similar to gout, surgery, trauma, and alcohol
may precipitate.
Treatment options are similar to gout, except
for allopurinol.
Pseudo gout, is Positively birifringent under
polarized light microscopy.
A 68-year old man presented with sudden severe
pain and swelling in the left knee. Synovial fluid
analysis shows abundant calcium
pyrophosphate dihydrate (CPPD) crystals.
Which of the following tests is NOT appropriate
for further assessment of this patient illness?
A : Creatinine kinase
B : Serum calcium
C : Thyroid function test
D : Serum ferritin level
E : Hb AIc.
Crystal Analysis: Gout Versus Pseudogout
Feature Gout Pseudogout
Crystal Monosodium urate Calcium
pyrophosphate
Color under Yellow Blue
polarized light
Shape of crystal Needle Rhomboid
Birefringence/ Negative Positive (clockwise)
rotation under (counterclockwise)
polarized light
direction
PseudoPseudogout
( calcific tendinitis)
Pain on adduction more with active movement.
Characterized by deposition of hydroxy appetite
crystals within rotator cuff muscle near humeral
attachment most commonly involve supraspinatus
tendon.
Severe shoulder pain localized mainly to humorous
diffusely around antrolateral shoulder.
Palpation or compression around the greater tubercle
of the humerus causes tenderness.

X ray : linear calcific density in supraspinatus tendon.


Ttt: intralesional steroids & physiotherapy.
70 years old lady complaining of
gradually progressive bilateral
lower limb oedema
Paget disease of bone
Paget disease of bone
Paget disease of bone
What are the manifestations?

What are the serious complications?

What are the biochemical abnormalities?

What is the treatment?


Paget's disease of bone
Accelerated, disorganized bone remodelling due to abnormal osteoclast activity.
Affects up to 1:10 of the elderly but only a minority are symptomatic.
Presentation.
Skeletal Pain ,deformity & fracture.
Deformity bowing of weight-bearing bones, especially tibia, femur, Frontal bossing.
High COP failure.
Complications
Pathological fractures; O Arthropathy of adjacent joints.
cranial nerve compression., neurological symptoms e.g. deafness,
bone sarcoma (10% of those affected >10y.).
Investigation.
X-ray Distinctive changes;
Alk phos--->very high.
normal Ca2+, PO4, PTH,
bone scan...diffuse density.
Urinary hydroxyproline.
Management
pain and long-term complications with bisphosphonates (e.g. risedronate 30mg/d. for
2mo.).
Paget disease of bone
Pain,deformities,fractures.
Hyperdynamic circulation
Marked increase of alkaline Phosphatase
Bone scan
Bone scan
The radioactive tracer is evenly distributed among the
Normal: bones. No areas of abnormally high or low
accumulation are seen.

The tracer may accumulate in certain areas of the


Abnormal: bone, indicating one or more hot spots. Hot spots may
be caused by a fracture that is healing, bone cancer, a
bone infection (osteomyelitis), arthritis, or a disease of
abnormal bone metabolism (such as Paget's disease).
Bone scintigraphy

Normal bone scan. Bone metastasis


Bone metastasis Paget disease of bone Pulmonary osteodystrophy
Hypertrophic osteoarthropathy (or
hypertrophic pulmonary osteoarthropathy
when there is mesothelioma or bronchogenic
carcinoma associated)
X rays show periosteal reaction at the ends
of the radius and ulnar bones suggestive of
periostitis.
Osteomalacia
Osteomalacia is characterized by a low serum
calcium and phosphate with elevated serum alkaline
phosphatase. Osteomalacia may be caused by
deficiency of vitamin D or phosphate deficiency.
Skeletal x-rays
osteomalacia present with the pathognomonic Looser
zones are linear areas of low density surrounded by
sclerotic borders,
better observed in the pubic ramus, upper femoral bone,
and ribs.
Treatment is with a daily dose of calciferol (20-25
micrograms).
Vitamin D deficiency (osteomalacia)
Nutritional deficiency
Impaired absorption ; Small bowel diseases, such as celiac
disease, gastric bypass, steatorrhea, and pancreatic diseases.
Hepatic disease
impaired 25-hydroxylation of vitamin D,
decreased bile salts with malabsorption of vitamin D,
decreased synthesis of vitamin Dbinding protein, or other factors.
vitamin D dysfunction.
Anticonvulsants

Renal failure
decreased conversion of 25-hydroxyvitamin D to its active form 1,25-
dihydroxyvitamin D. This results in an increase in PTH.
Inherited conditions
vitamin D dependent rickets (type I) or 1-alpha-hydroxylase deficiency .
vitamin D dependent rickets (type II )--- Hereditary vitamin D resistance rickets.
X-linked hypophosphataemic Vit
D resistant rickets.
serum phospate is low and urine
phosphate is high due to inappropriate
renal phosphate wasting.
Serum parathyroid levels are usually
slightly elevated. Clinically, growth
retardation that causes very severe
rickets, especially in affected males.
Treatment is with oral phosphate and high
dose activated Vitamin D.
Bone Disease Serum Serum Alkaline
calcium phosphorus phosphatase

osteoporosis n n n

osteomalacia

Paget disease n n

Primary
hyperparathyroidism

Renal
osteodystrophy

X-linked n
hypophosphat
aemic Vit D
resistant
Behcet's Syndrome
Immune mediated vasculitis, venulitis.
Equal sex affection but more severe in males ,Eastern Mediterranean and Asia .
C/P;
Painful Genital ulcers.
Painful Oral ulcers (98%).
Ocular disease ( 80%); more common in HLA B5 men; eg uveitis. iritis, retinal
vessel occlusions and optic neuritis can be found. Hypopyon uveitis
Skin; E.nodosum ( 80%); (F<M; associated with non-deforming arthritis), pyoderma
gang, Thrombophlebitits (30%),
Thrombophilia.
CNS disease (30%); Aseptic meningitis. Meningoencephalitis, TIA-like episodes,
headache, papilledema, Cranial nerve pseudobulbar palsies, Ataxia, dementia.
Colitis (30%).
Associated with HLA-B5.
May lead to perforation.
Clinically overlaps with IBD.
Arthritis ; intermittent, self-limited, not deforming and localized to the knees and
ankles. Spondylits, sacroilitis; When present, linked to HLA-B27.
Inv; A positive pathergy test refers to skin injury by needle prick leads to a papule or
pustule formation in 48 hours
TTT; colchicine, prednizone, NSAIDs, thalidomide.
Behcets
Sjgren syndrome
Sjgren syndrome (SS) is a systemic chronic inflammatory disorder
characterized by lymphocytic infiltrates in exocrine glands.
C/P; sicca symptoms; xerophthalmia (dry eyes), xerostomia (dry mouth),
dyspareunia fatigue and parotid gland enlargement.
extraglandular features ; arthralgia, arthritis, Raynaud phenomenon,
myalgia, pancreatitis, leukopenia, anemia,LN, neuropathy, vasculitis,
RTA, and later lymphoma (suspected if low C4, cryo, persistent parotid
enlarg, purpura & leukopenia).
Primary Sjgren syndrome occurs in the absence of another underlying
rheumatic disorder, whereas secondary Sjgren syndrome is
associated with another underlying rheumatic disease, such as SLE, RA,
or scleroderma.
Inv;
ANA 90%, Anti RO & Anti La in 1ry SJ,
ESR, CRP, RF, hypergammaglobulinema in 80%.
Schirmer's test
Management
artificial tears , cool drinks, artificial saliva sprays or sugar-free gum.
pilocarpine
NSAIDs, hydroxycholoquine for arthritis.
Dermatomyositis
C/P;
proximal muscle weakness, tenderness.
Skin; Raynaulds, Gottron's papules , heliotrope rash around the eye, calcinosis,
periungual telangiectasia, mechanics hand, v shaped, shawl sign.
Others;
Dysphagia.
cardiomyopathy,
IPF.
The condition is associated with carcinoma of the breast, lung, ovary and bowel in
old age only.

Inv :
elevation of muscle enzymes; CPK, LDH, AST.
EMG; myopathic pattern (high frequency, low amplitude).
ESR, CRP in 5 % only.
Autoantibodies;
- ANA
- anti-jo-1(IPF, mechanics hand, Raynaulds)
- anti M2 (classic)
- anti SRP (severe rapidly progressive) severe form with cardiac affection.
muscle biopsy; inflammatory changes .

TTT; steroids rapid CPK but power takes weeks+/- methotrexate, cyclophosphamide.
Scleroderma
Inv;
ANA in 90%,
RF in 30%
Anti-topoisomerase (= anti Scl 70, specific, in 30%) in diffuse
scleroderma.
Anticentromere in 50-90% of limited & 10% of diffuse sclero.
Anti-RNA polymerase III in renal crises.
Patterns;
1. Limited systemic; CREST S (calcinosis, raynaulds, esophageal
dysmotility, scleroderma, telangiectasia), tight skin limited to face,
neck, distal extremities. cause of death; pulmonary HTN.
2. Diffuse systemic; cause of death; renal crises, worse prognosis.
3. Localized to the skin; morphea (plaques), coup de sabre (linear).

TTT; NSAIDs, PPI, antibiotics for bact overgrowth, VD, PG, D-


penicillamine, ACEI for renal crises.
Overlap Syndromes
E.g. mixed CT disease; SLE, Sclero, PM.
+ve anti RNP.
Osteoporosis

**A disease characterised by low bone mass


and microarchitectural deterioration of bone
tissue, with a consequent increase in bone
fragility and susceptibility to fracture .

**Bone mineral density (BMD) T-score < 2.5


What are the major risk factors for osteoporosis?

Non-modifiable Modifiable

1) Age. 1) low ca. intake


2) Race ( Caucasian,Asian). 2) low vit-D intake
3) Gender (female). 3) estrogen deficiency
4) Early menopause(<45) 4) sedentary life
5) Slender built. 5) smoking
6) Positive family history. 6) alcohol(>2drinks/d)
7) caffeine(>2serving/d
8) medications:
(steroids-thyroxine)

Other conditions with low bone mass

** hyperparathyroidism ** Celiac d.
** hyperthyroidism ** Liver d.
** hypogonadism ** I.B.D
** cushing s. ** myeloma
** osteomalacia ** Renal failure
** Drugs :
.. Steroids thyroxine cyc,A - heparin
How to interpret BMD H

1)T-score : comparison of patients bone mass


to that of young normal subject(age 30)

2)Z-score : comparison of patients bone mass to


that of age-matched subject
indications for BMD:
**

1)Women aged 65 years and older .


2)Postmenopausal women< 65 years with risk factors for osteoporosis
3)Men aged 70 years and older .
4)Adults with fragility fracture .
5)Adults with a disease or associated with low bone mass or bone loss.
6)Adults taking medication associated with bone loss.
7)Anyone being considered for pharmacological osteoporosis therapy .
Pharmacologic therapy of osteoporosis
Ca, vit D
Drugs that decrease bone resorption;
1. HRT (Euterine cancer& breast cancer).
2. SERM (selective Estrogen receptor modulators)= agonist on bone &
antagonist on breast osteoporosis, br cancer & IsHD. Side effects;
flushes, DVT.
- Raloxifen (Evista)used in postmenopausal osteoporosis.
3. STEAR= selective tissue estrogenic activity regulator e.g. Tibolon (Levial
)
4. Bisphosphonate (oral alendronate)
5. Calcitonin ; anabolic bone effect, Side effects; nasal congestion, rash.

Drugs that increase bone formation;


1. Synthetic PTH (Teripatide).
2. Strontium ranelate (Protelos) dual mechanism , incr bone formation &
decrease bone resorption.
3. Others; GH, GHRH, IGF1, anabolic steroids, statins.
Ca, vit D;
- ca alone; small effect on BMD, no antifracture effect.
- Vit D alone; unknown effect in pt without vit D def.
- Ca + Vit D; may BMD, fracture.
Raloxifen (Evista ) 60 mg once daily
- TTT & prevention postmenopausal osteoporosis.
- adv; vertebral fracture,but not non vert fracture,
br cancer &
IsHD.
- Side effects; increase risk of DVT.
not effective against non vert fracture.
Calcitonin; nasal spray, SC; vertebral fracture
( weak evidence) , small BMD.
HRT
Thromboembolism, breast cancer, TG,
risk of fracture, cancer colon, HDL coronary.
Contraindications;
- Absolute;
E dependant tumours; breast, endometrium.
Thromboembolic disorder.
Abn vaginal bleeding.

- Relative;
FH of br cancer,
GB disease.
Endometriosis
Alendronate (oral daily 10 mg or weekly 70 mg)
- mech; taken up by osteoclast apoptosis.
- adv; effective against all fractures, long term benefit (10 yrs).
- disadv; poor upper GIT tolerance.
- contraindications; hypocalcemia, osteomalacia, renal failure, bed
ridden, serious oesophageal dis.
- side effects; hypocalcemia, PTH, skin rash
oral; esophagitis, ulceration.
IV; flu like, arthralgia, myalgia, fever, leukopenia, eye inflammation
etidronate; phosphate, osteomalacia.

Teriparatide; rh PTH
- daily SC, 1st anabolic, all fractures, marked & rapid BMD.
- disadv; daily SC, use restricted to high risk pts, osteosarcoma in
toxicologic studies.
- contraindication; bone dis; paget dis, Hx of bone metastasis or cancer,
growing child, bone, hypercalcemia, pregnant, lactation.
The following subjects are candidates for
BMD except :

1) A lady of 68 years
2) An obese lady at age of 52(menopause)
3) A man of 74 years age
4) A lady 34-year age with 1ry
hyperparathyroidism
A 54-year-old lady advised to do BMD for
persistent spine pain, Which revealed :

** T-score
.. L.spine : -2.6
.. F.neck : -1.7

** All are possible except :


1)Osteoporosis
2)Hyperparathyroidism
3)I.B.D
4)Hypothyroidism
5)Cushing d.
A 52-year-old lady, menopause for 3 years
on longterm steroids for her uncontrolled
chronic asthma

** BMD done for her revealed :


T-score L.spine : -2.0
F.neck : -1.2

** suggested treatment :
1)Calcium + vitamin-D
2)Calcium + vitamin-D + Alendronate
3)Calcium + vitamin-D + Alendronate + Calcitonin
4) No TTT
A 62-year-old lady presented with severe
back pain , she lost about 2 inches of her
height. Plain XR revealed farcture of T12
and wedged L3. She is immobile for
severe knee OA and on steroids for ILD
** BMD advised revealing :
T-score L.spine : -4.2
F.neck : -2.8
** Best TTT is :
1) Ca + vit-D + Alendronate
2) Ca + vit-D + Calcitonin
3) Ca + vit-D + Teriparatide (PTH)
4) Ca + vit-D + Alendronate + Calcitonin
A 58-year-old lady admitted for
fracture neck-femur
** BMD revealed :
T-score L.spine : -1.6
F.neck : -2.8

** The drug being least effective is :


1)Alendronate
2) HRT
3) PTH
4) Raloxifene ( SERM)
Gonoccocal arthritis
is a common cause of septic arthritis in which the organism cannot be cultured on
routine culture media.
Occur in disseminated gonococcal infection.
C/P;
sexually active individuals, ; (4:1), during menstruation and pregnancy.
fever, chills, skin rashes( pustular) , polyarthralgias, and tenosynovitis persistent
monoarthritis or oligoarthritis,
Inv;
The mean synovial fluid WBCs = 50,000 cells/mm3.
Cultures of synovial fluid tend to be positive in < 50 % of cases of gonococcal arthritis
The joint aspirate should be cultured for N. gonorrhoeae when the history is
suggestive. ( chocolate agar or Thayer-Martin medium)
cultures from clinically appropriate sites (eg, the pharynx, urethra, cervix, rectum, and
skin lesions).
Blood cultures are often positive in patients presenting with tenosynovitis and skin
lesions alone,
PCR
screening test for HIV and a syphilis.
TTT; , the initial therapy of choice is ceftriaxone (1 gm either IV or IM) or another 3rd
G cephalosporin.
Fluoroquinolones .
Relapsing polychondritis
Associated with autoimmune disease in 30%.
Leads to fever, arthralgias, episcleritis, swollen
floppy ears.
Nasal Septum collapse (the other 'saddle nose'
deformity, i.e. not just Wegener's).
laryngeal disease hoarseness; respiratory
obstruction.
Tracheobroncial degeneration recurrent
infections,
AI/MVP/ Aneurysm in 10%.
Causes of avascular necrosis
SLE .
vasculitis & any hypercoagulable state.
long term steroids .
sickle cell disease.
Hip involvement.
MRI is the best diagnostic tool.
Causes of Charcots joints:
diabetic neuropathy,
syphillis
syringomyelia
leprosy
Causes of Iritis:
Behcet's disease
Reiter's syndrome
ankylosing spondylitis
sardoidosis
Eosinophilic fasciitis is a disorder
characterized by peripheral eosinophilia and
fasciitis. Swelling and progressive induration of
the skin associated with aching of the
extremities and occasional morning stiffness
develop over a period of weeks. The distribution
most often is in the upper extremity, proximal
and distal to the elbow, and in the lower
extremity, proximal and distal to the knee. Onset
may be acute following some sort of strenuous
exercise, or it may be subacute. The diagnosis is
confirmed by deep biopsy from skin to muscle.
Eosinophilic fascitis

Eosinophilic fascitis
Rheumatologic Manifestations of
sickle cell disease

Gout.
Sickle lower extremity arthralgias (knees and
ankles most common), myalgias, synovitis.
2ry hemochromatotic (Fe overload 2o to
frequent transfusions).
Septic arthritis (or osteomyelitis), esp due to
Salmonella, particularly if hyposplenic.
Aseptic (avascular) necrosis.
Familial Mediterranean fever
AD
is an inherited condition characterized by recurrent
episodes of painful inflammation in the abdomen, chest,
or joints, fevers and rash.
The first episode usually occurs by the age of 20 years,
but in some cases, later in life.
FMF primarily affects populations originating from the
Mediterranean region.
A buildup of amyloid. AA amyloidosis commonly involves
the kidneys, spleen and GI tract.
Colchicine given prophylactically in FMF offers some
protection against the development of amyloidosis in most
patients.
De Quervain's tenosynovitis
is inflammation of the abductor pollicis
longus and extensor pollicis brevis.
Finkelstein's test is positive. This is
performed with the thumb flexed across the
palm of the hand, asking the patient to
move the wrist into flexion and ulnar
deviation. This stresses the tendons of
abductor pollicis longus and extensor
pollicis brevis and reproduces the pain of
de Quervain's tenosynovitis.
A 15-year-old boy presented with arthralgia, skin
rash and haematuria. Renal biopsy showed
focal
necrotising glomerulonephritis with diffuse
mesangial IgA deposits. What is the most likely
diagnosis?
A : Systemic lupus erythematosus (SLE)
B : HenochScholein purpura
C : Juvenile rheumatoid arthritis
D : Post-streptococcal glomerulonephritis
E : Goodpasture's syndrome
24. A 64-year-old man presents to A&E with a 2-
day history of increasing pain and swelling of
his left knee. He denies a history of trauma. On
examination, the knee is hot, red, swollen and
extremely tender. Which of the following
investigations is most important?
A : Plain radiograph of the knee
B : Blood cultures
C : C-reactive protein (CRP)
D : Joint aspiration
E : Plasma uric acid level.
64-year-old man with congestive heart failure presents to the emergency room
complaining of acute onset of severe pain in his right foot. The pain began during
the nightand awoke him from a deep sleep. He reports the pain to be so severe
that he could not wear a shoe or sock to the hospital.His current medications are
furosemide, 40 mg twicedaily, carvedilol, 6.25 mg twice daily, candesartan, 8 mg
once daily, and aspirin, 325 mg once daily. On examination, he is febrile to
38.5C. The first toe of the right foot is erythematous and exquisitely tender to
touch. There is significant swelling and effusion of the first metatarsophalangeal
joint on the right. No other joints are affected. Which of the following findings
would be expected on arthrocentesis?

A. Glucose level of <25 mg/dL


B. Positive Gram stain
C. Presence of strongly negatively birefringent needleshaped crystals under polarized
light microscopy
D. Presence of weakly positively birefringent rhomboidal crystals under polarized light
microscopy
E. White blood cell (WBC) count >100,000/L
A 36-year-old woman presents with deteriorating
nocturnal paraesthesia affecting both
hands. It improves during the morning. Which of
the following tests is least likely to be helpful in
establishing the cause?
A : Thyroid function test
B : Rheumatoid factor
C : Tinels test
D : MRI scan hands
E : Urinary hCG.
A 25-year-old woman with a history of 3-second
trimester fetal losses is planning a fourth pregnancy.
She has evidence of the primary anti-phospholipid
syndrome (strongly positive cardiolipin antibody,
positive lupus anticoagulant but no evidence of lupus).
Which of the following treatment regimens offer her
the best chance of having a successful pregnancy?
A : Steroids alone
B : Steroids combined with low-dose aspirin
C : Low-dose aspirin alone
D : Low-dose aspirin combined with low molecular
weight heparin
E : Intravenous immunoglobulin.
Serum biochemistry of a 60 year old man revealed
calcium of 1.98 mmol/l and phosphate of 0.55
mmol/l with an alkaline phosphatase of 450 IU/l.
Which among the following most suits with the
above serum biochemistry?
1) Osteoporosis
2) Osteomalacia
3) Pagets Disease
4) Secondary Hyperparathyroidism
5) Renal failure
A 36-year old woman is referred with a 1-year history of
muscle pain, tiredness and sleep disturbance. She
denies fever, weight loss and arthralgia. Examination
reveals tenderness over her occiput, trapezius and
lumbar area. Her blood results show a normal ESR,
CRP, FBC, a weakly positive ANA 1:80 and normal
complement. Which is the most likely diagnosis?
A : Polymyositis
B : System lupus erythematous (SLE)
C : Sjogrens syndrome
D : Polymyalgia rheumatica
E : Fibromyalgia.
A 58-year-old female presents complaining of right shoulder pain. She does not
recall any prior injury butnotes that she feels that the shoulder has been
getting progressively more stiff over the last several months. She previously
had several episodes of bursitis of the right shoulder that were treated
successfully with NSAIDs and steroid injections. The patients past medical
history is also significant for diabetes mellitus, for which she takes metformin
and glyburide. On physical examination, the right shoulder is not warm or red
but is tender to touch. Passive and active range of motion is limited in flexion,
extension, and abduction. A right shoulder radiogram shows osteopenia
without evidence of joint erosion or osteophytes.
What is the most likely diagnosis?
A. Adhesive capsulitis
B. Avascular necrosis
C. Bicipital tendinitis
D. Osteoarthritis
E. Rotator cuff tear
A 45-year-old woman is admitted with a spiking temperature
and sweats. She has been unwell for the last 3 weeks with
flitting arthralgia and lethargy. There is a rash over her
trunk which is most prevalent in the mornings. Blood
cultures are sterile. Her recent transthoracic
echocardiogram is normal. ESR 56mm/hour. Her ferritin is
elevated at 6000(g/l. Autoimmune screen is negative.
1) bacterial endocarditis
2) systemic lupus erythematosus
3) rheumatoid arthritis
4) adult onset Stills disease
5) meningitis
A 60-year-old accountant complains of recurrent
attacks of exquisite pain and swelling in
the left big toe. Which of the following conditions is
NOT likely to be associated with this disorder?
A : Chronic alcoholism
B : Obesity
C : Rheumatoid arthritis
D : Diabetes mellitus
E : Diuretic therapy.
A 68-year-old woman with longstanding congestive cardiac
failure (ejection fraction 20%) presents with a hot,
swollen right knee. The following results are obtained:
FBC normal, Urea 11mM, Creatinine 196ul. Synovial
fluid: many monosodium urate crystals seen on
microscopy, culture sterile. What is the best treatment for
her acute arthritis?
A : Allopurinol
B : Colchicine 0.5mg every 2-4 hours
C : Indomethacin 50mg tds
D : Co-codamol 30/500 every 6 hours
E : Intra-articular corticosteroids
A 34-year-old man presents with severe low back
pain, which has forced him to stop work as a bus
driver. He has had back pain on and off for many
years, on occasion with right-sided sciatica. The
pain used to be helped by rest, but is now present
more or less all the time and is stopping him from
sleeping properly. The most likely diagnosis is:
A : mechanical back pain.
B : ankylosing spondylitis.
C : myeloma.
D : osteoporosis.
E : osteoarthritis.
Red flag symptoms, requiring urgent investigation
to exclude sinister pathology, include:
age >55 or <18 years
progressive pain
night pain
systemic symptoms
progressive neurological deficit
past history of malignancy or
immunosuppression
recent trauma.
A 50 year old man presents with a 6 week history of
general malaise and a 2 day history of a right foot
drop, a left ulnar nerve palsy and a widespread
purpuric rash. He complains of arthralgia but has
no clinical evidence of inflammatory joint disease.
Echocardiogram is normal, blood cultures are
negative, ESR 100 mm/hr, ANCA negative, ANA
negative, rheumatoid factor strongly positive, C3
0.8 g/l (0.75 - 1.6), C4 0.02 g/l (0.14 - 0.5).
Dipstick urinalysis shows blood ++ but no protein.
1) ANA negative SLE
2) Cryoglobulinaemia
3) Infective endocarditis
4) Polyarthritis nordosa
5) Rheumatoid arthritis
A patient with primary Sj grens syndrome that was
diagnosed 6 years ago and treated with tear
replacement for symptomatic relief notes continued
parotid swelling for the last 3 months. She has also
noted enlarging
posterior cervical lymph nodes. Evaluation shows
leukopenia and low C4 complement levels. What is the
most likely diagnosis?
A. Chronic pancreatitis
B. Secondary Sj grens syndrome
C. HIV infection
D. Lymphoma
E. Amyloidosis
A 64-year-old man with coronary artery disease an atrial fibrillation is
referred for evaluation of fevers, arthralgias, pleuritis, and malar
rash. The symptoms have developed over the past 6 months. The
pleuritis has responded to steroid therapy, but prednisone has been
unableto be tapered off due to recurrence of symptoms at daily
steroid doses <15 mg of prednisone. His medications include
aspirin, procainamide, lovastatin, prednisone, and carvedilol. At this
stage antibodies directed against which of the following proteins is
most likely to be positive?
A. Cardiolipin.
B. Double-strand DNA.
C. Histone.
D. Ribonucleoprotein (RNP).
E. Ribosomal .
A 69-year-old woman with a 5-year history of intermittent left knee pain owing
to osteoarthritis presents to your office 5 days after vacationing in Greece,
where she developed increased knee pain and swelling. She says that she had
been walking and climbing more than usual on this vacation. She and some of
the vacationers in her group developed watery diarrhea around the same time,
but there were no chills or fever. She had been treated with naproxen until 2
years ago, when she developed atrial fibrillation. She now takes
acetaminophen as needed for knee pain and takes warfarin for the atrial
fibrillation. Physical examination shows a temperature of 38 C (100 F). There
is a large effusion in the left knee, with increased warmth but no redness.
There is moderate tenderness and pain on motion, and she walks with a limp,
which she did not have before. The other joints are unchanged.
Laboratory studies
Hemoglobin 13.7 g/dL Hematocrit 42%
Leukocyte count 10,000/L Neutrophils 80%
INR 2.1 Left knee radiograph reveals changes of osteoarthritis and an
effusion.
Which of the following would you do next?
(A) Obtain stool culture and start treatment with ciprofloxacin.
(B) Reduce warfarin dose and start treatment with celecoxib.
(C) Order MRI of left knee.
(D) Perform arthrocentesis.
(E) Admit to hospital and start treatment with intra- venous ceftriaxone.
A 28-year-old woman presented with fatigue and extreme
tiredness. Physical examination
revealed facial skin rash and tenderness across the small
joints of the hands. She was concerned
that she might have systemic lupus erythematosus (SLE).
Which of the following tests when
NEGATIVE will virtually exclude the diagnosis of SLE?
A : Antinuclear antibody (ANA)
B : Anti-double stranded DNA (Ads-DNA)
C : Anti-Sm antibodies
D : Anti-histone antibodies
E : Anti-Ro/SSA antibodies.
35. A 60-year-old-man complained of pain in both wrist
evolving over 8 weeks. He noted swelling around that area
but denied stiffness. On examination there was swelling
and tenderness just proximal to the wrist joints without
limitation of movement. There was also prominent finger
clubbing. Radiographs revealed periosteal reaction over
the lower end of the radius and ulnar.
Each of the following disorders could be the cause behind
this patient complain EXCEPT?
A : Mesothelioma
B : Bronchiectesis
C : Diabetes mellitus
D : Crohn's disease
E : Whipple's disease.
THANK YOU
A 35-year-old nurse developed Raynaud's
phenomenon six months ago . The antinuclear antibody
(ANA) test was positive at 1:1000 with a speckled
staining pattern. She came to the out-patient
department
complaining of excessive tiredness and lethargy. The
creatine phosphokinase (CPK) was three times higher
than the upper normal limits, and the hand joints
radiographs showed no erosions. Which one of the
following tests would you request to establish the
diagnosis?
A : Anti-U1 RNP
B : Anti-ds DNA
C : Anti-Scl 70
D : Anti-centromere antibodies
E : Anti-PM-Scl
Serum biochemistry of a 60 year old man revealed
calcium of 1.98 mmol/l and phosphate of 0.55
mmol/l with an alkaline phosphatase of 450 IU/l.
Which among the following most suits with the
above serum biochemistry?
1) Osteoporosis
2) Osteomalacia
3) Pagets Disease
4) Secondary Hyperparathyroidism
5) Renal failure
A 25-year-old woman with a history of 3-second
trimester fetal losses is planning a fourth pregnancy.
She has evidence of the primary anti-phospholipid
syndrome (strongly positive cardiolipin antibody,
positive lupus anticoagulant but no evidence of lupus).
Which of the
following treatment regimens offer her the best chance
of having a successful pregnancy?
A : Steroids alone
B : Steroids combined with low-dose aspirin
C : Low-dose aspirin alone
D : Low-dose aspirin combined with low molecular
weight heparin
E : Intravenous immunoglobulin.
A 58-year-old female presents complaining of right shoulder pain.
She does not recall any prior injury bu tnotes that she feels that
the shoulder has been getting progressively more stiff over the
last several months. She previously had several episodes of
bursitis of the right shoulder that were treated successfully with
NSAIDs and steroid injections. The patients past medical
history is also significant for diabetes mellitus, for which she
takes metformin and glyburide. On physical examination, the
right shoulder is not warm or red but is tender to touch. Passive
and active range of motion is limited in flexion, extension, and
abduction. A right shoulder radiogram shows osteopenia
without evidence of joint erosion or osteophytes.
What is the most likely diagnosis?
A. Adhesive capsulitis
B. Avascular necrosis
C. Bicipital tendinitis
D. Osteoarthritis
E. Rotator cuff tear
A 70-year-old female has a 17-year history of
rheumatoid arthritis. She presents with
recurrent attacks of red congested eyes with
a sensation of grittiness. The most likely
cause of her red eyes is likely to be:
A : scleritis
B : episcleritis
C : keratitis
D : keratoconjunctivitis sicca
E : choroiditis.
24. A 64-year-old man presents to A&E with a 2-day
history of increasing pain and swelling of his left
knee. He denies a history of trauma. On
examination, the knee is hot, red, swollen and
extremely tender. Which of the following investigations
is most important?
A : Plain radiograph of the knee
B : Blood cultures
C : C-reactive protein (CRP)
D : Joint aspiration
E : Plasma uric acid level.
A 72-year-old man with longstanding rheumatoid arthritis
(RA) presents with a 3-day history of numbness and
clumsiness of both hands and difficulty walking more
than a few yards because of weakness and
unsteadiness of his legs. Which of the following causes
of his deterioration seems most likely?
A : Carpal tunnel syndrome
B : Peripheral neuropathy
C : Secondary osteoarthritis
D : Systemic vasculitis
E : Cervical myelopathy.
A 15-year-old boy presented with arthralgia, skin
rash and haematuria. Renal biopsy showed
focal necrotising glomerulonephritis with diffuse
mesangial IgA deposits. What is the most likely
diagnosis?
A : Systemic lupus erythematosus (SLE)
B : HenochScholein purpura
C : Juvenile rheumatoid arthritis
D : Post-streptococcal glomerulonephritis
E : Goodpasture's syndrome
A 70-year-old diabetic had problems with decreased mobility
of his back. The problem had progressively worsened over
the last seven years. His lumbar spine was remarkable for
loss of the normal lordosis and decreased range of motion
in all planes. He had no pain on percussion over the spine
or sacroiliac joints. He was otherwise in good health, and
the erythrocyte sedimentation rate (ESR) was normal
The most probable diagnosis is:
A : Ankylosing spondylitis (AS)
B : Alcaptonuria
C : Osteoporosis
D : Spondylolisthesis
E : Diffuse idiopathic skeletal hyperostosis (DISH)
A 30-year-old teacher presents with a 6-month
history of swelling and pain involving the distal
interphalangeal joints of the hands. The ESR is
65 mm in the first hour. What is the most likely
diagnosis?
A : Generalised osteoarthritis
B : Rheumatoid arthritis
C : Psoriatic arthritis
D : Systemic lupus erythematosus (SLE)
E : Gout.
A 28-year-old woman presented with fatigue and
extreme tiredness. Physical examination revealed
facial skin rash and tenderness across the small
joints of the hands. She was concerned that she
might have systemic lupus erythematosus (SLE).
Which of the following tests when -ve will virtually
exclude the diagnosis of SLE?
A : Antinuclear antibody (ANA)
B : Anti-double stranded DNA (Ads-DNA)
C : Anti-Sm antibodies
D : Anti-histone antibodies
E : Anti-Ro/SSA antibodies.
An 8-year-old has been admitted for a third time
with haemolytic-uraemic syndrome and is found
to have a low C3. Which of the following is the
most likely diagnosis?
A : Lupus
B : Gram-negative septicaemia
C : Factor H deficiency
D : Factor I deficiency
E : C3 deficiency.
A 69-year-old woman with a 5-year history of intermittent left knee pain owing
to osteoarthritis presents to your office 5 days after vacationing in Greece,
where she developed increased knee pain and swelling. She says that she had
been walking and climbing more than usual on this vacation. She and some of
the vacationers in her group developed watery diarrhea around the same time,
but there were no chills or fever. She had been treated with naproxen until 2
years ago, when she developed atrial fibrillation. She now takes
acetaminophen as needed for knee pain and takes warfarin for the atrial
fibrillation. Physical examination shows a temperature of 38 C (100 F). There
is a large effusion in the left knee, with increased warmth but no redness.
There is moderate tenderness and pain on motion, The other joints are
unchanged.
Laboratory studies
Hemoglobin 13.7 g/dL Hematocrit 42%
Leukocyte count 10,000/L Neutrophils 80%
INR 2.1 Left knee radiograph reveals changes of osteoarthritis and an
effusion.
Which of the following would you do next?
(A) Obtain stool culture and start treatment with ciprofloxacin.
(B) Reduce warfarin dose and start treatment with celecoxib.
(C) Order MRI of left knee.
(D) Perform arthrocentesis.
(E) Admit to hospital and start treatment with intra- venous ceftriaxone.
13-year-old boy was hospitalised because of a purpuric rash involving the
legs (see image) associated with abdominal pain and fever (38C). The
patient also complained of arthralgiainvolving the knees and ankles.
Urinalysis showed proteinuria with microscopic haematuria. A biopsy of
the purpuric lesion revealed leucocytoclastic vasculitis in the small
vessels. Which of the following statements is true about this boy's
illness?

A.The urine abnormality and fever is most probably related to


a recent urinary tract infection
B : In such cases blood cultures are often positive for
Pseudomonas aerogenosa
C : Renal biopsy typically shows mesangial IgA deposition
D : Bilateral small kidneys are a constant feature on ultrasound
of the kidneys
E : Untreated, up to 50% will develop chronic renal failure.
A 67-year-old woman was referred to a specialist
rheumatology clinic. She had poorly controlled
rheumatoid arthritis. For the last fifteen years she
had been treated with various disease-modifying
drugs including sulphasalazine and methotrexate.
Blocking the biologic function of which of the
following molecules will provide significant anti-
inflammatory effect in RA?
A : Interleukin-10 (IL-10)
B : Interleukin-4 (IL-4)
C : Interleukin-1 (IL-1)
D : Soluble TNF receptors
E : Transforming growth factor-beta (TGF-beta).
A 32-year-old man presents with a 4-month
history of back pain. The pain is worse in the
morning and after sitting watching TV. Plain
radiograph of the spine/pelvis shows
evidence of sacroiliitis. Each of the following
clinical features might be identified on clinical
examination of this patient EXCEPT?
A : Keratoderma blenorhegica
B : Rheumatoid nodule
C : Onycholysis
D : Uveitis
E : Urethritis.
A 36-year-old woman presents with deteriorating
nocturnal paraesthesia affecting both hands. It
improves during the morning. Which of the
following tests is least likely to be helpful in
establishing the cause?
A : Thyroid function test
B : Rheumatoid factor
C : Tinels test
D : MRI scan hands
E : Urinary hCG.
35. A 60-year-old-man complained of pain in both wrist
evolving over 8 weeks. He noted swelling around that area
but denied stiffness. On examination there was swelling
and tenderness just proximal to the wrist joints without
limitation of movement. There was also prominent finger
clubbing. Radiographs revealed periosteal reaction over
the lower end of the radius and ulnar.
Each of the following disorders could be the cause behind
this patient complain EXCEPT?
A : Mesothelioma
B : Bronchiectesis
C : Diabetes mellitus
D : Crohn's disease
E : Whipple's disease.
Comment : Hypertrophic osteoarthropathy (HOA) is a
syndrome characterized by proliferative
changes in the skin and skeleton. Proliferative periostitis
of the radius and fibula and digital clubbing 97%).
Secondary HOA was initially described in association
with chronic suppurative infection and malignancy of the
lung and pleura. Therefore it used to be called
hypertrophic pulmonary osteoarthropathy (HPOA).
Pleural causes include pleural fibroma and
mesothelioma. Pulmonary causes include bronchogenic
carcinoma, pulmonary tuberculosis; pulmonary
abscesses, bronchiectasis, emphysema; and
Pneumocystis carinii infection in patients with AIDS,
Hodgkins disease, metastases, or cystic fibrosis.
Cyanotic heart disease with a right-to-left shunt is the
only cardiac cause described.
Abdominal causes include liver cirrhosis, ulcerative
colitis, Crohn disease, Whipple disease,.
A 36-year old woman is referred with a 1-year history of
muscle pain, tiredness and sleep disturbance. She
denies fever, weight loss and arthralgia. Examination
reveals tenderness over her occiput, trapezius and
lumbar area. Her blood results show a normal ESR,
CRP, FBC, a weakly positive ANA 1:80 and normal
complement. Which is the most likely diagnosis?
A : Polymyositis
B : System lupus erythematous (SLE)
C : Sjogrens syndrome
D : Polymyalgia rheumatica
E : Fibromyalgia.
A 40-year-old woman presents with a 6-month
history of a purpuric rash on her legs,
nonspecific joint pains and vague ill health. Initial
investigations reveal a positive rheumatoid
factor. The GP makes a tentative diagnosis of
rheumatoid arthritis and refers her to hospital.
Results of further investigations are as follows:
Urinalysis: red cell casts, protein+, ANA 1/80,
anti-DNA negative, anti-ENA negative, serum C3
1.02 g/l (NR 0.75-1.65), C4 <0.02 g/l (0.20-0.65),
CRP <5mg/l (NR <5), creatinine 145 (NR 50-
140). what is the most likely diagnosis?
A 60-year-old accountant complains of recurrent
attacks of exquisite pain and swelling in the left big
toe. Which of the following conditions is NOT likely to
be associated with this disorder?
A : Chronic alcoholism
B : Obesity
C : Rheumatoid arthritis
D : Diabetes mellitus
E : Diuretic therapy.
A 34-year-old man presents with severe low back
pain, which has forced him to stop work as a bus
driver. He has had back pain on and off for many
years, on occasion with right-sided sciatica. The
pain used to be helped by rest, but is now present
more or less all the time and is stopping him from
sleeping properly. The most likely diagnosis is:
A : mechanical back pain.
B : ankylosing spondylitis.
C : myeloma.
D : osteoporosis.
E : osteoarthritis.
Red flag symptoms, requiring urgent investigation to
exclude sinister pathology, include:
age >55 or <18 years
progressive pain
night pain
systemic symptoms
progressive neurological deficit
past history of malignancy or immunosuppression
recent trauma.
A:
Which one of the following
musculoskeletal disorders is NOT
associated with diabetes mellitus?

A : Dupuytren's contracture
B : Chondrocalcinosis
C : Scleroderma
D : Carpal tunnel syndrome
E : Frozen shoulder.
A 68-year old man presented with sudden
severe pain and swelling in the left knee.
Synovial fluid analysis shows abundant
calcium pyrophosphate dihydrate (CPPD)
crystals. Which of the following tests is NOT
appropriate for further assessment of this
patient illness?
A : Creatinine kinase
B : Serum calcium
C : Thyroid function test
D : Serum ferritin level
E : Hb AIc.
Which one of the following antibodies, when present in high
titre, is NOT regarded as diagnostic of the disease
indicated?
A : Anti-Sm antibodies in systemic lupus erythematosus
(SLE)
B : Ads-DNA antibodies in systemic lupus erythematosus
(SLE)
C : Anti-centromere antibodies in limited systemic sclerosis
D : Rheumatoid factor in rheumatoid arthritis (RA)
E : Anti-Scl 70 antibodies in diffuse systemic sclerosis
Which one of the following rheumatological
disorders is NOT associated with increased
incidence of malignancy?
A : Dermatomyositis
B : Polymyalgia rheumatica
C : Sjogren's syndrome
D : Diffuse systemic sclerosis
E : Hypertrophic pulmonary osteoarthropathy
A 40-year-old woman suddenly develops severe
Raynauds that is troublesome in the summer as
well as the winter. She starts to develop swelling
of the fingers and feels tired. She also develops
reflux oesophagitis and has difficulty swallowing.
She has noticed that she has become more
breathless. On examination, she has skin
thickening affecting her hands, face and trunk.
Which of the following is most likely to be positive?
A : Ds DNA antibody
B : Ro antibody
C : Anticentromere antibody
D : Jo-1 antibody
E : Scl-70 antibody
A 77-year-old man presents with persistent head ache
and progressive deafness. On examination the
patient has frontal bossing of the forehead and
conductive deafness, more severe in the right ear.
His serum alkaline phosphatase is significantly
raised at 870 u/L. Which of the following statements
is most accurate about this disease?
A : It usually affects a single bone.
B : The skull is the most commonly affected bone.
C : Bone pain is the most common presenting feature.
D : Hearing loss is often due to involvement of the
middle ear ossicles leading to conductive
deafness.
E : Bone pain is typically increased with rest and on
weight bearing.
A 50 year old man presents with a 6 week history of
general malaise and a 2 day history of a right foot
drop, a left ulnar nerve palsy and a widespread
purpuric rash. He complains of arthralgia but has
no clinical evidence of inflammatory joint disease.
Echocardiogram is normal, blood cultures are
negative, ESR 100 mm/hr, ANCA negative, ANA
negative, rheumatoid factor strongly positive, C3
0.8 g/l (0.75 - 1.6), C4 0.02 g/l (0.14 - 0.5).
Dipstick urinalysis shows blood ++ but no protein.
1) ANA negative SLE
2) Cryoglobulinaemia
3) Infective endocarditis
4) Polyarthritis nordosa
5) Rheumatoid arthritis
A patient with psoriatic arthritis has active joints
and troublesome plaque psoriasis. Which of
the following will improve both the joint and
skin problems?
A : Sulphasalazine
B : Hydroxychloroquine
C : Gold
D : Methotrexate
E : Penicillamine
A 45-year-old woman is admitted with a spiking temperature
and sweats. She has been unwell for the last 3 weeks with
flitting arthralgia and lethargy. There is a rash over her
trunk which is most prevalent in the mornings. Blood
cultures are sterile. Her recent transthoracic
echocardiogram is normal. ESR 56mm/hour. Her ferritin is
elevated at 6000(g/l. Autoimmune screen is negative.
1) bacterial endocarditis
2) systemic lupus erythematosus
3) rheumatoid arthritis
4) adult onset Stills disease
5) meningitis
Which of the following has the greatest specificty
for Wegener's granulomatosis?
1) pANCA and positive antibodies to
myeloperoxidase
2) atypical ANCA and positive antibodies to
myeloperoxidase
3) cANCA and positive antibodies to
myeloperoxidase
4) cANCA and positive antibodies to proteinase 3
5) cANCA and positive antibodies to lactoferrin
A 29-year-old woman has a 3-year history of arthralgia
and Raynaud's phenomenon. A year ago she had a
miscarriage at 29 weeks, complicated by a deep vein
thrombosis. Investigations show:Hb11.2Wcc 4.3
Platelets 145,ANA positive1/160,DNAnegative,ENA
Ro positive, Anticardiolipin antibodies present at
moderate titre, lupus anticoagulant present. Which
statement is true?
A : She has primary antiphospholipid syndrome.
B : She should be anticoagulated for a further 6
months.
C : She has active lupus.
D : A high C reaction protein (CRP) and erythrocyte
sedimentation rate (ESR) would be consistent with
active lupus.
E : Hydroxychloroquine may improve her arthralgia
A 34-year-old man is admitted to the hospital for evaluation and
treatment of renal failure and an abnormal CT of the chest. For
the past 2 months, he has had fatigue, malaise, and
intermittent fevers to as high as 38.2C. About 3 weeks ago,
he sought treatment from his primary provider for sinus pain
and congestion with a purulent and bloody nasal discharge.
He was treated for 2 weeks with ampicillinsulbactam, but his
symptoms have only minimally improved.When he returned to
his physician, a basic metabolic panel was performed which
showed a creatinine of 2.8mg/dL. A urinalysis showed 1+
protein with 25 red blood cells per high-power field. Red blood
cell casts were present. His chest CT is shown below. Which
of the following tests
would be most likely to be positive in this individual
A. Antiglomerular basement membrane antibodies
B. Antiproteinase-3 antibodies
C. High titers of antibodies to antistreptolysin O
D. Perinuclear antineutrophil cytoplasmic antibodies
E. Positive blood cultures for Staphylococcus aureus
56-year-old patient presented with 6-month
history of excessive tiredness, stiffness and
pain across the upper and lower limbs. On
examination he has proximal muscle weakness.
The creatinine kinase was 10 times the upper
limits of normal. The skin lesions shown in the
image are most likely to be:

tendon xanthoma
Gottron's papules
rheumatoid nodules
gouty tophi.
A 75-year-old retired farmer has been seen in the outpatient clinic. He has
been generally unwell for the last month with fever and weight loss,
examination reveals this rash (see image). While in the clinic he starts
to cough up blood and becomes acutely breathless. He is admitted and
initial investigations reveal a serum creatinine of 170 micromol/l, a
positive myeloperoxidase (MPO) ELISA, negative proteinase 3 (PR3)
ELISA, negative GBM ELISA, a negative ANA and normal complement
levels. Red cell casts are seen on microscopy of his urine. Which two of
the following
diagnoses are most compatible with the clinical picture?
A : Sjogrens syndrome
B : Goodpastures disease
C : Lupus
D : Henoch schonlein purpura
E : Cryoglobulinaemic vasculitis
F : Polyarteritis nodosa
G : Anti-phospholipid syndrome
H : Wegener's granulomatosis
I : Microscopic polyangiitis
J : Takayasus arteritis
Which one of the following drugs does NOT
cause proximal muscle weakness?
A : Corticosteroid
B : D-penicillamine
C : Methotrexate
D : Chloroquine
E : Colchicine
A 42-year-old man is evaluated because of right renal colic
and microhematuria. He has had three previous episodes
of calcium nephrolithiasis. Diagnostic studies show a 4-
mm calcified stone in the middle right ureter.
Which of the following laboratory test results is not a risk
factor for calcium nephrolithiasis?
(A) Urinary calcium of 315 mg/24 h
(B) Serum uric acid of 10.5 mg/dL
(C) Urinary citrate of 100 mg/24 h (normal 300-700
mg/24 h)
(D) Urinary oxalate of 72 mg/24 h (normal < 40 mg/24 h)
(E) Serum calcium of 11 mg/dL and serum phosphorus of
2.1 mg/dL
A 50 year old woman presents with dry eyes, a dry mouth, an
erythematous rash and polyarthralgia. Investigations: ANA
strongly positive (1:1600), anti-Ro/SSA antibodies strongly
positive, rheumatoid factor positive, IgG markedly elevated at
45 g/l (normal - <15 g/l), IgM and IgA levels are normal and
the kappa/lambda ratio is normal. What is the most likely
diagnosis?
1) Hyperviscosity syndrome
2) Myeloma associated vasculitis
3) Primary Sjogren's Syndrome
4) Rheumatoid arthritis with secondary Sjogren's Syndrome
5) Systemic Lupus Erythematosus
Serum biochemistry of a 60 year old man revealed
calcium of 1.98 mmol/l and phosphate of 0.55
mmol/l with an alkaline phosphatase of 450
IU/l.Which among the following mosts suits with
the above serum biochemistry?
1) Osteoporosis
2) Osteomalacia
3) Pagets Disease
4) Secondary Hyperparathyroidism
5) Renal failure
A 31-year-old woman presents to your clinic complaining of
painful arthritis that is worse in the mornings
when she wakes up. She was recently evaluated by an
ophthalmologist for uveitis in her right eye. A recent
laboratory report shows an erythrocyte sedimentation rate of
48 mm/h. Which of the following will be helpful in
distinguishing relapsing polychondritis from rheumatoid
arthritis (RA)?
A. Arthritis associated with RA is nonerosive.
B. Eye inflammation is absent in relapsing polychondritis.
C. Relapsing polychondritis will not present with vasculitis.
D. Relapsing polychondritis will present with high-titer
rheumatoid factor.
E. The arthritis of relapsing polychondritis is asymmetric
A 42-year-old obese male presents to your office with complaints of
paresthesias in the right hand that are worst in the fourth and fifth
fingers. Symptoms have been present intermittently for the last 4
months. He has no other past medical history and takes no
medications. The examination is significant for an intact neurologic
examination of the right upper extremity but mild wasting of the
intrinsic muscles on inspection of the right hand.
Laboratories show a normal white blood cell count, hemoglobin, and
sedimentation rate. Electrolytes and creatinine and liver function tests
are normal except for a serum glucose of 148 mg/dL. What is the most
likely etiology of this patients symptoms?
A. Diabetes mellitus
B. Cholesterol emboli
C. Churg-Strauss disease
D. Cervical spondylosis
E. Neurogenic thoracic outlet syndrome
A 54-year-old man is admitted for persistent lower abdominaland groin pain that began 7
months previously.Two months before his present admission, he required
exploratorylaparoscopy for acute abdominal pain and presumedcholecystitis. This
revealed necrotic omental tissueand pericholecystitis necessitating omentectomy and
cholecystectomy.However, the pain continued unchanged. Hecurrently describes it
as periumbilical and radiating into hisgroin and legs. It becomes worse with eating.
The patient has
also had episodic severe testicular pain, bowel urgency, nausea,vomiting, and diuresis.
He has lost ~22.7 kg over thepreceding 6 months. His past medical history is
significantof hypertension that has recently become difficult to control.Medications on
admission include aspirin, hydrochlorothiazide,hydromorphone, lansoprazole,
metoprolol,and quinapril. On physical examination, the patient appearscomfortable.
His blood pressure is 170/100 mmHg,his heart rate is 88 beats/min, and he is
afebrile. He hasnormal first and second heart sounds without murmurs,and an S4 is
present. There are no carotid, renal, abdominal,or femoral bruits.His lungs are clear
to auscultation. Bowel sounds are normal.Abdominal palpation demonstrates minimal
diffusetenderness without rebound or guarding. No masses arepresent, and the stool
is negative for occult blood. Duringthe examination, the patient develops Raynaud's
phenomenonin his right hand that persists for several minutes. Hisneurologic
examination is intact. Admission laboratorystudies reveal an erythrocyte
sedimentation rate of 72 mm/h,
a BUN of 17 mg/dL, and a creatinine of 0.8 mg/dL. The patienthas no proteinuria or
hematuria. Tests for antinuclearantibodies, anti-double-stranded-DNA antibodies,
and antineutrophilcytoplasmic antibodies are negative. Liverfunction tests are
abnormal with an AST of 89 IU/L and anALT of 112 IU/L. Hepatitis B surface antigen
and e antigenare positive. Mesenteric angiography demonstrates smallbeaded
aneurysms of the superior and inferior mesentericveins. What is the most likely
diagnosis?
A. Hepatocellular carcinoma
B. Ischemic colitis
C. Microscopic polyangiitis
D. Mixed cryoglobulinemia
E. Polyarteritis nodosa
A 64-year-old African-American male is evaluated inthe hospital for
congestive heart failure, renal failure, and
polyneuropathy. Physical examination on admission was notable for these
findings and raised waxy papules in the axilla and inguinal region.
Admission laboratories showeda BUN of 90 mg/dL and a creatinine of
6.3 mg/dL. Total protein was 9.0 g/dL, with an albumin of 3.2 g/dL.
Hematocrit was 24%, and white blood cell and platelet countswere
normal. Urinalysis was remarkable for 3+ proteinuriabut no cellular casts.
Further evaluation included an echocardiogram with a thickened left
ventricle and preserved systolic function. Which of the following tests
ismost likely to diagnose the underlying condition?
A. Bone marrow biopsy
B. Electromyogram (EMG) with nerve conduction studies
C. Fat pad biopsy
D. Right heart catheterization
E. Renal ultrasound
A 66-year-old woman with a history of rheumatoid arthritis and
frequent pseudogout attacks in her left knee presents with night
sweats and a 2-day history of left knee pain. On physical
examination, her temperature is 38.6C, heart rate is 110
beats/min, blood pressure is 104/78 mmHg, and oxygen
saturation is 97% on room air. Her left knee is swollen, red,
painful, and warm. With 5 offlexion or extension, she develops
extreme pain. She hasevidence of chronic joint deformity in her
hands, knees,and spine. Peripheral white blood cell (WBC)
count is16,700 cells/L with 95% neutrophils. A diagnostic tapof
her left knee reveals 168,300 WBCs per microliter,99%
neutrophils, and diffuse needle-shaped birefringentcrystals
present. Gram stain shows rare gram-positivecocci in clusters.
Management includes all of the following except
A. blood cultures
B. glucocorticoids
C. needle aspiration of joint fluid
D. orthopedic surgery consult
E. vancomycin
A 41-year-old female presents to your clinic with 3 weeks of weakness, lethargy. and
depressed mood. She notes increasing difficulty with climbing steps, rising fro a chair,
and combing her hair. She has no difficulty buttoning
her blouse or writing. The patient also notes some dyspnea on exertion and orthopnea.
She denies rash, joint aches, or constitutional symptoms. She is on no medications,
and the past medical history is otherwise uninformative.The family history is notable
only for coronarartery disease. The physical examination is notable for an elevated
jugular venous pressure, an S3, and some bibasilar crackles. The neurologic
examination shows some marked proximal muscle weakness in the deltoids and
biceps and the hip flexors. Distal muscle strength is normal. Sensoryexamination and
reflexes are normal. Laboratories are unremarkable except for a negative antinuclear
antibody screen and a creatinine kinase of 3200 IU/L. You suspect a diagnosis of
polymyositis. All the following clinical conditions may occur in polymyositis except
A. an increased incidence of malignancy
B. interstitial lung disease
C. dilated cardiomyopathy
D. dysphagia
E. Raynauds phenomenon
64-year-old man with congestive heart failure presents to the emergency room
complaining of acute onset of severe pain in his right foot. The pain began during
the nightand awoke him from a deep sleep. He reports the pain to be so severe
that he could not wear a shoe or sock to the hospital.His current medications are
furosemide, 40 mg twicedaily, carvedilol, 6.25 mg twice daily, candesartan, 8 mg
once daily, and aspirin, 325 mg once daily. On examination, he isfebrile to 38.5C.
The first toe of the right foot is erythematous and exquisitely tender to touch.
There is significant swelling and effusion of the first metatarsophalangeal joint on
the right. No other joints are affected. Which of the following findings would be
expected on arthrocentesis?

A. Glucose level of <25 mg/dL


B. Positive Gram stain
C. Presence of strongly negatively birefringent needleshaped crystals under polarized
light microscopy
D. Presence of weakly positively birefringent rhomboidal crystals under polarized light
microscopy
E. White blood cell (WBC) count >100,000/L
A 58-year-old female presents complaining of right shoulder pain. She does not
recall any prior injury butnotes that she feels that the shoulder has been
getting progressively more stiff over the last several months. She previously
had several episodes of bursitis of the right shoulder that were treated
successfully with NSAIDs and steroid injections. The patients past medical
history isalso significant for diabetes mellitus, for which she takes metformin
and glyburide. On physical examination, the right shoulder is not warm or red
but is tender to touch. Passive and active range of motion is limited in flexion,
extension, and abduction. A right shoulder radiogram shows osteopenia
without evidence of joint erosion or osteophytes.
What is the most likely diagnosis?
A. Adhesive capsulitis
B. Avascular necrosis
C. Bicipital tendinitis
D. Osteoarthritis
E. Rotator cuff tear
A patient with end-stage renal disease on hemodialysis presents
to your office wit hand pain and you diagnose carpal tunnel
syndrome. A serum thyroid-stimulating hormone level is
normal. You also note bilateral knee effusions,
which the patient states have been there for many
months.Suspecting an amyloid deposition disease, you
perform a fat pad biopsy. Which protein do you expect to find
on immunohistochemical staining?
A. 2-Microglobulin
B. Fibrinogen -chain
C. Immunoglobulin light chain
D. Serum amyloid A
E. Transthyretin
A 48-year-old male has a long-standing history of ankylosing
spondylitis. His most recent spinal film shows straightening
of the lumbar spine, loss of lordosis, and squaring of the
vertebral bodies. He currently is limited by pain with
ambulation that is not improved with nonsteroidal anti-
inflammatory medications. Which of the following treatments
has been shown to improve symptoms the best at this stage
of the illness?
A. Celecoxib
B. Etanercept
C. Prednisone
D. Sulfasalazine
E. Thalidomide
A 64-year-old man with coronary artery disease an atrial fibrillation is
referred for evaluation of fevers, arthralgias, pleuritis, and malar
rash. The symptoms have developed over the past 6 months. The
pleuritis has responded to steroid therapy, but prednisone has been
unableto be tapered off due to recurrence of symptoms at daily
steroid doses <15 mg of prednisone. His medications include
aspirin, procainamide, lovastatin, prednisone, and carvedilol. At this
stage antibodies directed against which of the following proteins is
most likely to be positive?
A. Cardiolipin
B. Double-strand DNA
C. Histone
D. Ribonucleoprotein (RNP)
E. Ribosomal P
A 68-year-old woman with longstanding congestive cardiac
failure (ejection fraction 20%) presents with a hot,
swollen right knee. The following results are obtained:
FBC normal, Urea 11mM, Creatinine 196ul. Synovial
fluid: many monosodium urate crystals seen on
microscopy, culture sterile. What is the best treatment for
her acute arthritis?
A : Allopurinol
B : Colchicine 0.5mg every 2-4 hours
C : Indomethacin 50mg tds
D : Co-codamol 30/500 every 6 hours
E : Intra-articular corticosteroids
A 68-year-old woman with longstanding congestive
with a 3-month history of fatigue. She has read
about lupus on the world wide web and asks her
GP to carry out appropriate blood test(s) to rule
out lupus. Which single blood test is best suited
to screen for lupus in this situation?
A : Antinuclear antibody
B : Serum Immunoglobulin levels
C : Anti-cardiolipin antibodies
D : Anti-DNA antibodies
E : Anti-ENA antibodies.
A 25-year-old woman with a history of 3-second
trimester fetal losses is planning a fourth pregnancy.
She has evidence of the primary anti-phospholipid
syndrome strongly positive cardiolipin antibody,
positive lupus anticoagulant but no evidence of lupus.
Which of the
following treatment regimens offer her the best chance
of having a successful pregnancy?
A : Steroids alone
B : Steroids combined with low-dose aspirin
C : Low-dose aspirin alone
D : Low-dose aspirin combined with low molecular
weight heparin
E : Intravenous immunoglobulin.
A 78-year-old man presents with a 2-day history of severe
headache localising to his right temple. He also feels tired and
lethargic. He denies any eye problems and pain on chewing.
The biopsy of his right temporal artery is shown (see image).
Which of the following statements fit best?

A : This is a normal temporal artery biopsy.


B : This man should be treated with 60 mg prednisolone.
C : There is no need to consider bone protection.
D : 50% of patients with this condition develop permanent
visual loss.
E : An ESR of less than 40mm/hr excludes Giant Cell Arteritis
(GC
A 72-year-old woman presents to the emergency room for an episode of vision loss in her right
eye. The vision loss came on abruptly and is described as a curtain falling across her
visual field. She immediately called her daughter and upon arrival to the emergency room
40 min later, her vision had returned to normal. Recently she also has been experiencing
dull throbbing headaches for which she is taking acetaminophen, with limited relief. She
has a past medical history of hypercholesterolemia and coronary artery disease,
undergoing angioplasty and stenting of the right coronary artery 8 years previously. She
does not smoke currently but has a 40-pack-year history of tobacco, quitting only after her
diagnosis of coronary artery disease. On review of systems, the patient recalls pain in her
scalp with combing her hair, particularly on the right side, and occasional pain with
chewing food. She has also recently noticed stiffness and pain in her hips, making it
difficult to stand from seated position. On examination, she has 20/30 visual acuity in the
left eye, and 20/100 visual acuity in the right eye. Funduscopic examination suggests
anterior ischemic optic neuropathy. There are no carotid bruits present, but palpation of
the temporal arteryis painful. The neurologic examination is otherwise normal. The
erythrocyte sedimentation rate (ESR) is 102 mm/h. The hemoglobin is 7.9 g/dL, and
hematocrit is 25.5%. A head CT shows no acute ischemic event. Which of the following is
the next most important step in the management of this patient?

A. Initiate treatment with indomethacin, 75 mg twice daily.


B. Initiate treatment with prednisone, 60 mg daily.
C. Initiate treatment with unfractionated heparin
D. Perform magnetic resonance angiography of the brain.
E. Perform a temporal artery biopsy.
A patient presents with 3 weeks of pain in the
lower back. All the following are risk factors
for serious causes of spine pathology except
A. age more than 50 years
B. urinary incontinence
C. duration of pain more than 2 weeks
D. bed rest without relief
E. history of intravenous drug use
A 43-year-old male presents to your office complaining of weakness in the right hand for
2 days. He reports that he had been in excellent health until 2 monthsago, when he
was diagnosed with hypertension. Since that diagnosis, he has lost 20 lb
unintentionally and complains of frequent headaches and abdominal pain that is
worse after eating. He previously was an injection drug user but now is maintained on
methadone. His only medications are hydrochlorothiazide 25 mg/d, methadone70
mg/d, and lisinopril 5 mg/d. On physical examination,the patient appears well
developed and without distress. Blood pressure is 148/94. He is not tachycardic. The
examination is otherwise notable only for the inabilityto extend the right wrist and
fingers against gravity.Laboratory studies show an erythrocyte sedimentation rate
(ESR) of 88 mm/h, an aspartate aminotransferase (AST) of 154 IU/L, and an alanine
aminotransferase(ALT) of 176 IU/L. Which of the following tests is most useful in
establishing a diagnosis?
A. Hepatitis B surface antigen
B. Hepatitis C viral load
C. Anticytoplasmic neutrophil antibodies
D. Mesenteric angiography
E. Radial nerve biopsy
36. A 23-year-old woman was diagnosed with systemic lupus
erythematosus based upon the presence of polyarthritis,
malar rash with photosensitivity, and oral ulcerations.
Antibodies to double-stranded DNA, Smith protein, and
antinuclear antibodies were present in high titers. A
urinalysis is normal.. What is the best choice for initial
therapy in this individual?
A. Hydroxychloroquine, 200400 mg daily
B. Methotrexate, 15 mg weekly
C. Physical therapy only
D. Prednisone, 1 mg/kg daily
E. Quinacrine, 100 mg three times daily
A 25-year-old female presents with a complaint of painful mouth ulcerations.
She describes these lesions as shallow ulcers that last for 1 or 2 weeks.
The ulcers have been appearing for the last 6 months. For the last 2 days,
the patient has had a painful red eye. She has had no genital ulcerations,
arthritis, skin rashes, or photosensitivity. On physical examination, the
patient appears well developedand in no distress. She has a temperature
of 37.6C (99.7F), heart rate of 86, blood pressure of 126/72, and
respiratory rate of 16. Examination of the oral mucosa reveals two shallow
ulcers with a yellow base on the buccal mucosa. The ophthalmologic
examination is consistent with anterior uveitis. The cardiopulmonary
examination is normal. She has no arthritis, but medially on the right thigh
there is a palpable cord in the saphenous vein. Laboratory studies reveal
an erythrocyte sedimentation rate of 68 s. White blood cell count is
10,230/mm3 with a differential of 68% polymorphonuclear cells, 28%
lymphocytes, and 4% monocytes. The antinuclear antibody and anti-
dsDNA antibody are negative. C3 and C4 are normal.
What is the most likely diagnosis?
A. Behets syndrome.
B. Systemic lupus erythematosus.
C. Discoid lupus erythematosus.
D. Sj grens syndrome.
E. Cicatricial pemphigoid.
What is the best initial treatment for this patient?

A. Topical glucocorticoids including ophthalmic


prednisolone
B. Systemic glucocorticoids and azathioprine
C. Thalidomide
D. Colchicine
E. Intralesional interferon
A 63-year-old white female is admitted to the hospital complaining of
hemoptysis and shortness of breath.She had been well until 3 months ago,
when she noted vague symptoms of fatigue and a 10-lb unintentional
weight loss. Past medical history is notable only for osteoporosis.Her
current symptoms began on the day of presentation with the expectoration
of >200 mL of red blood in the emergency department. On physical
examination ,the patient is in marked respiratory distress with a respiratory
rate of 44 breaths per minute. Oxygen saturationis78% on room air and
88% on non re breather mask. Pulse is 120 beats/min, with a blood
pressure of 170/110. There are diffuse crackles throughout both lung fields,
and thecardiac examination is significant only for a regular tachycardia.
There are no rashes or joint swellings. Laboratory studies reveal a
hemoglobin of 10.2 mg/dL with amean corpuscular volume (MCV) of 88
m3 (fL). The white blood cell count is 9760/mm3. Blood urea nitrogen
(BUN) is 78 mg/dL, and creatinine is 3.2 mg/dL. The urinalysis shows 1+
proteinuria, moderate hemoglobin, 25to 35 red blood cells (RBC) per high-
power field, and occasiona lRBC casts. Chest computed tomography (CT)
shows diffuse alveolar infiltrates consistent with alveolar hemorrhage. The
antimyeloperoxidase titer is positive at126 U/mL (normal <1.4 U/mL). What
is the most likelydiagnosis?
A. Goodpastures disease
B. Wegeners granulomatosis
C. Microscopic polyangiitis
D. Polyarteritis nodosa
E. Cryoglobulinemia
A 42-year-old man presents to your clinic complaining of left
shoulder soreness that has been bothering him for 8
months. He experiences intermittent pain that is
worse at night. Active abduction of his left arm over his
head causes extreme pain. He describes his pain as a dull
ache in his shoulder. He cannot identify a specific trauma
that led to his pain but notes that he lifts weights and plays
sports on a regular basis. On physical examination, he has
tenderness over the lateral aspect of the humeral head and
pain with arm abduction. Which of the following is the most
likely cause of his symptoms?
Acromioclavicular arthritis
B. Bicipital tendonitis
C. Inflammation of the infraspinatus tendon
D. Inflammation of the supraspinatus tendon
E. Subluxation of the left humeral head
A 60-year-old woman with a history of Sj grens
syndrome diagnosed 20 years ago presents to her primary
care doctor complaining of facial swelling. Her xerostomia
dry eye symptoms have not changed. She is known to
be positive for rheumatoid factor in addition to Ro and
La antibodies but is not thought to have rheumatoid arthritis.
She previously had cutaneous vasculitis requiring
treatment with prednisone, but she has been off steroids
for 5 years without evidence of recurrence. She is currently
using artificial tears and cevimeline, 30 mg three
times daily. On physical examination, her right parotid
gland is enlarged. It is not tender, but is firm and hard to
touch. It is noted that the right parotid gland was similarly
enlarged on a visit 3 months ago. She denies systemic
illness or any new symptoms. What is the most
likely diagnosis?
A. Adenoid cystic carcinoma
B. B cell lymphoma
C. Impacted sialolith
D. Mumps
E. Recurrent vasculiti
66. An 84-year-old man is seen by his primary care provider, and chronic =
kidney disease. He is taking pravastatin, aspirin, furosemide, metolazone,
lisinopril,and metoprolol XL. His baseline creatinine is 2.4 mg/dL, and uric acid
level 9.3 mg/dL. His most recent with symptoms of acute gouty arthritis in the
first great toe and ankle on the left. He has a prior history ofgout presenting
similarly. His past medical history is significant for myelodysplasia, congestive
heart failure ,hypercholesterolemia complete blood count results are white
blood cell count 2880/L, hemoglobin 8.2 g/dL, hematocrit 26.2%, and
platelet 68,000/ L. Which of the following medication regimens are most
appropriate for the treatment of this patient?

A. Allopurinol, 100 mg once daily


B. Colchicine, 1 mg IV once, then 0.5 mg IV every 6 h until improvement
C. Indomethacin, 25 mg three times daily
D. Prednisone, 40 mg once daily
E. Probenecid, 250 mg twice daily
A patient with primary Sj grens syndrome that was
diagnosed 6 years ago and treated with tear
replacement for symptomatic relief notes continued
parotid swelling for the last 3 months. She has also
noted enlarging
posterior cervical lymph nodes. Evaluation shows
leukopenia and low C4 complement levels. What is the
most likely diagnosis?
A. Chronic pancreatitis
B. Secondary Sj grens syndrome
C. HIV infection
D. Lymphoma
E. Amyloidosis
A 23-year-old man seeks evaluation for low back pain. He states that when
he first awakens there is a dull aching pain in his lower lumbar and
gluteal region. When he first noticed the pain about 6 months ago, he
thought the pain might be related to his mattress, but it has worsened
even after buying a new mattress. Most mornings, it takes about 4560
min to loosen up after he has awakened, but the pain will recur if he is
idle. He is currently in law school and finds it increasingly difficult to
remain in classes because of back pain. When he exercises, the pain
lessens. There are occasional nights that the pain will awaken him from
sleep, and he will have to move around and stretch his back to improve
the pain. On physical examination, there is pain with palpation at the
iliac crests, ischial tuberosities, greater trochanters, and heels. With
maximal inspiration, the chest expands 4 cm, and there is decreased
flexion of the lumbar spine. A radiograph of the pelvis shows erosions
and sclerosis of the sacroiliac joints bilaterally. Which of the following
tests is most likely to be positive in this individual?
A. Alkaline phosphatase
B. Antibodies directed against cyclic citrullinated peptides
(CCP)
C. Antinuclear antibodies
D. HLA-B27
E. Rheumatoid factor
A 46-year-old woman is referred to your clinic by her primary
care physician. She describes fatigue and diffuse muscle
aches that have been worsening over a period of 6 months.
She also has not been sleeping well. Her primary doctor
evaluated her and sent screening laboratory tests, which
returned with a positive rheumatoid factor. She has read
about rheumatoid arthritis on the Internet and is very
concerned that she has the disease based on her
symptoms and her positive test. Which of the following is
true in regard to diagnosing rheumatoid arthritis (RA)?
A. 10% of healthy individuals will test positive for antibodies to
cyclic citrullinated polypeptides (anti-CCP).
B. B. Erythrocyte sedimentation rate (ESR) is elevated in 70%
of patients with active disease.
C. In early disease, rheumatoid factor is more accurate than
anti-CCP.
D. Fewer than one-third of unselected patients with positive
rheumatoid factor will have RA.
E. Radiographs should be performed in this patient to help with
diagnosis.
A 53-year-old woman presents to your clinic complaining of
fatigue and generalized pain that have worsened over 2
years. She also describes irritability and poor sleep and is
concerned that she is depressed. She reveals that she was
recently separated from her husband and has been stressed
at work. Which of the following elements of her presentation
meet American College of Rheumatology criteria for
fibromyalgia?
A. Diffuse chronic pain and abnormal sleep
B. Diffuse pain without other etiology and evidence of major
depression
C. Major depression, life stressor, chronic pain, and female
gender
D. Major depression and pain on palpation at 6 of 18 tender
point sites
E. Widespread chronic pain and pain on palpation at 11 of
18 tender point sites
What is the most common extraarticular manifestation

of ankylosing spondylitis ?

A. Anterior uveitis
B. Aortic regurgitation
C. Cataracts
D. Inflammatory bowel disease
E. Third-degree heart block
A 52-year-old female has poorly controlled rheumatoid arthritis on prednisone 5
mg daily and etanercept 50 mg weekly by subcutaneous injection. Despite this,
she has ongoing symptoms with severe pain in the wrists, hands, feet, and
ankles. She also has destructive arthritis causing swan-neck and boutonnire
deformities in the hands as well as plantar subluxation of the metatarsal heads
that prevents ambulation. She has subcutaneous nodules on the extensor
surfaces of the arms. She presents to the emergency room complaining of
fevers and dysuria. On physical examination temperature is 39.1C (102.3F).
Heart rate is 112, and blood pressure is 122/76. The examination is
unremarkable except for right costovertebral angle tenderness and
splenomegaly. Laboratory studies at the time of presentation reveal a white
blood cell count of 2300/mm3 with 15% polymorphonuclear cells, 75%
lymphocytes, 8% monocytes, and 2% eosinophils. She is also anemic with a
hemoglobin of 9.2 mg/dL and a hematocrit of 28.7%. The mean corpuscular
volume is 88 fL. The platelet count is 132,000/mm3. A peripheralblood smear
shows normocytic anemia without anisocytosis or poikilocytosis. She is found
to have Escherichiacoli bacteremia related to a urinary tract infection. She is
treated with ceftriaxone and does well. However, she remains anemic and
neutropenic. The patient undergoes a bone marrow biopsy that shows
hypercellularity with a lack of mature neutrophils. What is the most likely
diagnosis?
A. Acute myelogenous leukemia
B. B cell lymphoma
C. Disseminated Mycobacterium tuberculosis infection
D. Feltys syndrome
E. Idiosyncratic reaction to etanercept
A 42-year-old female presents to the physician with 3 months of
worsening dyspnea on exertion, malaise, and weakness. She reports
that the symptoms have worsened gradually and are associated with
low-grade fever, anorexia,
and an 8-lb weight loss. She has trouble climbingstairs because of leg
weakness and shortness of breath. Recently she has noticed that her
arms tire while she is brushing her teeth or combing her hair. Her
mother also commented that the patient seems to have difficulty rising
from the couch. Her writing is normal, and she has no sensory
symptoms. Physical examination is notable for a temperature of
37.8C (100F), bilateral lung crackles and diminished strength in the
deltoids, quadriceps, and psoas muscles. Laboratory studies are
notable for an elevated creatine kinase. Chest radiography shows
bilateral interstitial infiltrates, and lung volumes are reduced to70% of
the predicted values. Which of the following autoantibodies is most
likely to be present in this patient?
A. Antiglomerular basement membrane antibody
B. Antihistone antibody
C. Anti-Jo-1 antibody
D. Antimicrosomal antibody
E. Antineutrophil cytoplasmic antibody (
A 42-year-old Turkish man presents to his physician complaining of recurring
ulcers in the mouth and on his penis. He states that the ulcers are painful
and last for about 2 weeks before spontaneously resolving. In addition, he
intermittently gets skin lesions that he describes as painful nodules on his
lower extremities. You suspectthat he has Behets syndrome. A pathergy
test is performed.
What response would you expect after injecting
0.3 mL of sterile saline under the skin?
A. Development of 10 mm of induration with overlying
erythema after 72 h
B. Development of a 2- to 3-mm papule at the site of
insertion in 23 days
C. Development of granulomatous inflammation 46
weeks after the injection
D. Development of an urticarial reaction within 15 min
E. No reaction
A 35-year-old female presents to her primary care doctor complaining of
diffuse body and joint pain. When asked to describe which of her joints are
most affected, she answers, All of them. There is no associated stiffness,
redness, or swelling of the joints. No Raynauds phenomenon has been
appreciated. Occasionally she notes numbness in the fingers and toes. The
patient complains of chronic pain and poor sleep quality that she feels is
due to her pain. She previously was seen in the clinic for chronic
headaches that were felt to be tension-related. She has tried taking over-
the-counter ibuprofen twice daily without relief of pain. She has no other
medical problems. On physical examination, the patient appears
comfortable. Her joints exhibit full range of motion without evidence of
inflammatory arthritis. She does have pain with palpation at bilateral
suboccipital muscle insertions, at C5, at the lateral epicondyle, in the upper
outer quadrant of the buttock, at the medial fat pad of the knee proximal to
the joint line, and unilaterally on the second right rib. The erythrocyte
sedimentation rate is 12 s. Antinuclear antibodies are positive at a titer of
1:40 in a speckled pattern. The patient is HLA-B27-positive. Rheumatoid
factor is negative. Radiograms of the cervical spine, hips, and elbows are
normal. What is the most likely diagnosis?
A. Ankylosing spondylitis
B. Disseminated gonococcal infection
C. Fibromyalgia
D. Rheumatoid arthritis
E. Systemic lupus erythematosus
A 45-year-old African-American woman with systemic lupus erythematosus
(SLE) presents to the emergency room with complaints of headache
and fatigue. Her prior manifestations of SLE have been arthralgias,
hemolytic anemia, malar rash, and mouth ulcers, and she is known to
have high titers of antibodies to double stranded DNA. She currently is
taking prednisone, 5 mg daily, and hydroxychloroquine, 200 mg daily.
On presentation, she is found to have a blood pressure of 190/110
mmHg with a heart rate of 98 beats/min. A urinalysis shows 25 red
blood cells (RBCs) per high-power field with 2+ proteinuria. No RBC
casts are identified. Her blood urea nitrogen is 88 mg/dL, and creatinine
is 2.6mg/dL (baseline 0.8 mg/dL). She has not previously hadrenal
disease related to SLE and is not taking nonsteroidalanti-inflammatory
drugs. She denies any recent illness, decreased oral intake, or diarrhea.
What is the most appropriatenext step in the management of this
patient
A. Initiate cyclophosphamide, 500 mg/m2 body surfacearea IV, and plan to
repeat monthly for 36 months.
B. Initiate hemodialysis.
C. Initiate high-dose steroid therapy (IV methylprednisolone, 1000 mg daily
for 3 doses, followed by oral prednisone, 1 mg/kg daily) and
mycophenolate mofetil, 2 g daily.
D. Initiate plasmapheresis.
E. Withhold all therapy until renal biopsy is performe
A 32-year-old pregnant woman presents to clinic with
right thumb and wrist pain that has worsened over
several weeks. She has pain when she pinches her
thumb against her other fingers. On physical examination
she has mild swelling and tenderness over the radial
styloid process, and pain is elicited when she places her
thumb inher palm and grasps it with her fingers. A Phalen
maneuver is negative.

Which condition is most likely?


A. Carpal tunnel syndrome
B. DeQuervains tenosynovitis
C. Gouty arthritis of the first metacarpophalangeal joint
D. Palmar fasciitis
E. Rheumatoid arthritis
Which of the following is true concerning a 68 year
old male with type 2 diabetes diagnosed with
type IV renal tubal acidosis?
1) Aminoaciduria would be expected.
2) Fludrocortisone treatment is effective
3) Increased Glomerular filtration rate is
expected.
4) Increased urinary bicarbonate would be
expected.
5) Normal renal handling of K+ and H+
Factors predisposing to urinary stone formation
includes all of the following EXCEPT
Increased consumption of Calcium rich foods
Increased consumption of oxalate rich foods
Hypocitraturia
Water deprivation
All are indications of renal biopsy except:
Failure to recover from acute renal failure
Diagnosis of systemic diseases with
renal involvement
Nephrotic syndrome
Uncontrolled hypertension
statistics
Types of studies
Experimental;
- Individuals are divided into groups; 1 group taking a new drug & the other group take
old drug or placebo (control gr).
- Which is either;
1. Double blind (doct & pt).
2. Single blind (pt).
3. Unblinded.
Crossover;
- Each pt receive ttt & placebo, one following the other.
- Use small no of patients.
- Used in chronic dis & ttt that lead to temporary relief.
Observational;
Only observe what happens without intervention.
1. Cohort prospective; follow up study.
e.g. take 1 group of smokers & non smokers & f.u. who will get the cancer.
2. Case control; search in the past
e.g. take cancer pt & search who was smoker in the past. (healthy control).
3. Cross sectional; e.g. present pt on pills, how much have endometrial cancer?
Types of studies
Cohort (prospective) study; one group with an exposure of interest is selected and
compared over time with another cohort without that exposure.
to study short period disease e.g. smokers & follow them, use incidence.
Disadv; expensive, liable to losses, drop out.
Adv;
1. ideal to study disease causal relationship & temporal relationship between disease
& risk exposure,
2. recall bias.
3. To see multiple outcomes.

Cross sectional study; use prevalence to study the prevalence of disease. also called
prevalence studies, look at the number of cases of a disease at a particular point
in time. They are not useful for investigating rare diseases or exposures.
Case control (retrospective) study;
Adv;
1. To study rare diseases
2. Suitable for dis of long latent period
3. Less costly , easy, rapid.
Disadv;
1. More prone to bias.
2. Difficult to prove temporal relationship between dis & exposure (risk)
Meta analysis; when you combine data of 2 or more large
previous studies to conclude new database regard the
point of research. Advantage; rapid, cheap.

Sequential trial; data are analyzed after each participant


individual results become available, so the trial is
continued until clear benefit is seen in one of comparing
groups. Advantage ;shorter than fixed length trial, used
when outcome of interest is known relatively quickly.

factorial trials test two or more treatments simultaneously.


Ecological study; (Geographical studies)
does not give data on individuals but give
idea about average values on groups of
people . (use prevalence or incidence) e.g.
incidence of Ca colon in certain population.
Screening test;
1. Should identify pts who require further
investigations or ttt.
2. Should be safe, acceptable.
Q; A researcher is trying to design a study to find
out the cause (or causes) of a rare disease,
about which very little is known. What study
design is most likely to be appropriate?
A : Geographical
B : Cross-sectional
C : Cohort
D : Intervention
E : Case control.
Concerning cohort studies, which one of the following
statements is true?
A : They can only be used to compare two groups
with one another.
B : They are particularly useful with rare outcomes.
C : Cohort studies are retrospective.
D : They are better than other study designs for
measuring prevalence of a disease in a population.
E : They are better than other study types for
measuring the incidence of a disease in a
population.
Regarding case-control studies, which one of the
following statements is FALSE?
A : They are good for investigating rare diseases.
B : They may be un-interpretable if controls are
selected poorly.
C : They are good for identifying rare causes of
disease.
D : They can examine multiple risk-factors for a
single disease.
E : They compare exposures of interest in cases
and controls.
Case-control studies compare exposures of interest in
cases and controls. Two of their great strengths is that
they can be used with rare diseases (because cases are
pre-selected), and can examine multiple risk-factors
(exposures). They are not good at identifying rare
exposures.
If the question is whether or not a rare exposure causes
a disease then the appropriate design is a cohort study,
where one group with the particular exposure of interest
is compared with a control group without that exposure.
The greatest difficulty in designing case-control studies
is selection of an appropriate control group, and poor
control selection often makes otherwise well-conducted
studies uninterpretable.
Regarding crossover trial design, which one of the
following statements is true?
A : It can be used to compare treatments for an
acute infection.
B : It is a good method for comparing analgesics in
arthritis.
C : It cannot be double-blinded.
D : It cannot be randomized.
E : Tends to need more patients than are required
with other trial designs to get adequate statistical
power.
B:
The principle of a crossover design is that a
patient has one drug or treatment, then a
washout period, and then another drug, and the
effect is compared between the two in a single
individual.
For this reason it is a good study design for
treatment of chronic conditions, but not
appropriate for acute conditions. It is just as
easy (or difficult) to randomize and double-blind
as for other study designs. Because each
person is acting as their own control, it is usually
possible to use smaller numbers to get the same
power.
Interpreting Randomized controlled trials of
therapeutic interventions (= experimental studies)
Types of errors;
1. Confounding;
in which a measured effect attributed to a particular variable is in fact dt an
unmeasured co- variable.
e.g. age is a Confounding factor in a study of effect of folic acid supp. During
pregnancy on neural tube defects.
i.e. older women will have risk of neural tube defect irrespective of folic acid supp.
Avoided by matching individuals in the groups acc. To potential confounders.
2. Bias;
-Systemic differences bet groups that distorts the comparisons between those groups
- = non randomized sampling.
- Bias means a flaw in study design that leads to a built-in likelihood that the
wrong result may be obtained.
- E.g. TB in Cairo.
- Avoided by random sampling & choose a reasonable study.
3. Sampling error;
- Arise because not all the population is examined but only a sample is taken.
- So the bigger the size of the sample, the smaller the sample error.
Types of data;
1. Qualitative data; proportion in the population
2. Quantitative data; mean, mode, median &
standard deviation.
Mean= is the arithmetic average.(sum/ n)
Median= is the middle value.(1st + last /2)
Mode= is the value that occurs most often.

mode Distribution;
median
mean
80
100

70 90

60 80

70
50
East 60
40 East
50 Line 2
Line 3
30 40

20 30

20
10
10
0
1st Qtr 2nd Qtr 3rd Qtr 4th Qtr 5th 0
1st Qtr 2nd Qtr 3rd Qtr 4th Qtr 5th

Skewed distribution Normal distribution


= asymmetrical
Mean
Mean= X = (sum of all observed values)
(n of sample size)
It is a measure of the overall magnitude of the observations.

Standard deviation
- It is a measure of the variability of the values
-is a measure of the scatter of observations about the mean
- is the square of the variance SD= variance .
- If the values are normally distributed, then:
Approximately 68% of the values lie within +/- 1 SD of the mean.
Approximately 95% of the values lie within +/- 2 SD of the mean.
i.e. x +/- 2 SD should include about 95% of the observation.
Mode is the value that has the largest
frequency distribution (most frequent value).
The median is the value above & below which
of the values lie. ()
In the normal distribution the mean, mode,
median all have the same value ( so we use
the mean)
but in the Skewed distribution, we prefer the
median.
Q: If data are skewed, then they should be
summarized in the form:
A : mean and standard deviation.
B : median and range
C : mean and range
D : median and standard deviation
E : mean and 95% confidence intervals.
B:
Comment : Skewed data should always be
summarized using the median and range.
Standard deviation is based on the mean,
which is not appropriate for skewed data.
How do you decide if data are skewed?
Plot them out and look at them, or find the
median and calculate the mean: if these
are more than slightly different, then the
data is skewed.
Q: If a characteristic is normally distributed in a
population;
A this means that most of the population is
composed of normal individuals
B there will be equal numbers who have more
or less of the characteristic than the mean
C the median value will be greater than the
mean
D ten percent of individuals will be beyond two
standard deviations from the mean .
answer true = b
a- nonsense
b- i.e. the values will be symmetrical about
the mean
c- median =mean in normal distribution
d-about 5%
mode = mean
The standard error is a measure of how
precisely the sample mean approximates
the population mean.
standard error = standard deviation
n
as n , standard error .
Confidence intervals
The interval (mean +/- 2 st error) is an approximate 95%
confidence interval for the pop. mean i.e. we are 95%
confident that the true mean lies inside this interval.
The interval (mean +/- 1.64 st error) is a 90% confidence
interval.
NB;
SD gives a measure of the spread of the data values.
S error is a measure of how precisely the sample mean
approximates the pop. Mean.
E.g. FEV1 in 100 students
Mean = 4.5 liter
SD = 0.5 liters.
- The interval where 95% of values lie is = mean +/- 2
st dev. =4.5 +/- 1 =3.5- 5.5
- The 95% confidence interval for pop mean= mean +/-
2 SE = 4.5 +/- 2 (0.5/ 100) = 4.5 +/- 2 (0.05)= 4.5
+/- 0.1= 4.4- 4.6 liters.
NB:
the larger the sample size, the narrower
the confidence interval.
An outcome which varies widely in the pop
will produce a wider confidence interval.
Correlation & regression
It is the relationship between 2 variables in the same individual (X & Y).
r= correlation coefficient, r is never <-1 or >1, if r= +/- 1=perfect+/-=all
pairs lie on the line, values near 1 or -1 show significant correlation.
b= regression coefficient= measure the average increase in y/ unit
increase in X= slope of the line.

0<r<1 -1<r<0
B +ve B -ve

+ve correlation -ve correlation

R=0
B=0 R=0
B=0

No association Non linear association


Q: The following statements are correct:
A A correlation coefficient (r) of 0.04 is almost
certainly significant.
B The values of chi-squared range from 0 to +1.
C The standard error of the mean is always less
than the standard deviation.
D A p value of 0.001 for the difference between
two sample means is more significant than one
of 0.01
E A confidence interval is the mean +/- 2 standard
deviations.
answer true = d
a- values near 1 or -1 show significant
correlation.
b-probability (p) ranges from 0 to +1,
c- standard error of the mean = standard
deviation / square root of n.
e-for a normal distribution a confidence
interval for a mean can be given by +/-
1.96 standard errors of the mean.
How large was the difference between the intervention &
the control groups in Randomized control trials

Control event rate (CER) = absolute risk.


= risk of outcome (e.g. mortality) in the control group
Experimental event rate (EER) = risk of outcome (e.g. mortality) in the Experimental gr
.relative risk of avoiding the event= 100-EER.
Absolute risk reduction (ARR) = CER-EER
Relative risk reduction (RRR) = CER-EER/CER
Number needed to treat (NNT)= 1/ARR

If 9.4% of patients given aspirin after myocardial infarction die (EER), compared
with 11.8% of those not given aspirin (CER),
then the absolute risk reduction (ARR) produced by aspirin is 11.8 9.4 = 2.4%,
the relative risk reduction (RRR) when taking aspirin is 2.4/11.8 = 0.2 (20%), and
the Number needed to treat (NNT) is 1 divided by 0.024 = 42, meaning that 42
patients with myocardial infarction must be treated with aspirin to prevent one
death.
the relative risk of avoiding the event (e.g.mortality) in
experimental group = 100- EER= 100- 9.4=90.6%.
The odds of avoiding the event (mortality) in experimental
group = RR of avoiding / EER = 90.6/9.4=

the relative risk of avoiding the event (e.g.mortality) in


the control group =100- 11.8= 88.2% .
The odds of avoiding the event (e.g.mortality) in the
control group =RR of avoiding /CER= 88.2 / 11.8 = .

Odd ratio (OR)= odds of E/C

NB: In general, ORs & RRs tend to be rather similar when


the CER < 20%, above this point, the figures divert (OR
tends to be higher).
Q: A placebo-controlled study randomized 10 000 patients
undergoing surgery for hip fracture to 160 mg aspirin /
day, started preoperatively and continued for 35 days.
About 1.5% of the patients allocated aspirin had DVT or
pulmonary embolism (PE), compared with about 2.5%
allocated placebo. Which one of the following statements
about this trial is true?
A : Aspirin produced a 1.5% absolute risk reduction in
DVT/PE.
B : Aspirin produced a 40% absolute risk reduction in
DVT/PE.
C : Aspirin produced a 1% proportional risk reduction in
DVT/PE.
D : Aspirin produced a 40% proportional risk reduction in
DVT/PE.
E : The Number Needed to Treat (NNT) to prevent one
DVT/PE is 100/1.5 = 67.
D:
Comment : In this study aspirin reduces
the risk of DVT/PE from 2.5% to 1.5%: this
is an absolute risk reduction of 1% and a
proportional (or relative) risk reduction of
1/2.5 = 40%.
The NNT to prevent one DVT/PE is
1/absolute risk reduction = 1/0.01 = 100.
Q: In a study of patients with myocardial infarction, the
death rate of those given aspirin is 8%,compared with
10% in those not given aspirin. This means that:
A : the relative risk of death after myocardial infarction is
1.25 in those given aspirin
B : the relative risk reduction produced by aspirin is 2%
C : the number needed to treat with aspirin to prevent one
death is 8
D : the number needed to treat with aspirin to prevent one
death is 10
E : the absolute risk reduction produced by aspirin is 2%.
Absolute risk reduction (or increase) = (Risk in
group 1) minus (Risk in group 2), which is 2% in
this example.
Relative risk reduction is the difference of
outcome in one group compared to another =
(Risk in group 1) divided by (Risk in group 2). In
this case aspirin reduced relative risk by 20%.
The Number Needed to Treat = 1 divided by
(Absolute Risk Reduction), which is 1/0.02 or 50
in this example.
Probability (P) = Prevalence= % = diseased/population.

To convert probability to odd;


Odd= P/1-P.
e.g. 20% probability= 20/1-0.2 or 20/100-20= 20/80= 1/4= 0.25.

To convert odd to probability:


P= O/1+O. e.g. if odd = 1:3, prob= 1/1+3=1/4=25%.

Means that for every 3 healthy there is 1 diseased so


there is 1 dis among total of 4 so the diseased are
25%.
P value is the probability of no difference (similarity) (null
hypothesis).
The smaller the P value, the bigger the difference. ( the more
significant the difference) less likely of null hypothesis is true.
If P value >0.05 the results would be found by chance in more
than 1:20.
The conventional cut-off for significance is P=0.05, or a 1-in-20
chance. Hence if 20 trials were conducted, you would expect to
get one that was positive by chance alone.

null hypothesis= non significant difference.


Type I error; reject the null hypothesis while it is actually true.
Help to determine sample size, In practice this means that the
study claims to find a difference that does not really exist.

Type II error; acceptance of the null hypothesis while it is false.


Power of the study= 1- Type II error
Sensitivity is the probability that a test will be positive
when a patient has the condition.
Sensitivity= true +ve /all diseased.

Specificity is the probability that a test will be negative


when a patient does not have the condition.
Specificity = true -ve /all non diseased (healthy).

Positive predictive value of a test (PPP)= post test


probability of a +ve test = true+ve/ all +ves.

negative predictive value of a test (NPP)= post test


probability of a -ve test = true-ve/ all -ves.

Diseased non
New test +ve True +ve False +ve
New test -ve False -ve True -ve
Q: The specificity of a test is defined as follows:

A : The number of true negatives detected by the test divided by the


number of all true negatives& false positive in the population tested.

B : The number of true positives detected by the test divided by the


number of all true positives in the population tested.

C : The number of true positives detected by the test divided by the total
number of true positives in the population tested.

D : The number of true negatives detected by the test divided by the total
number of true negatives in the population tested.

E : The number of true negatives detected by the test divided by the total
number of true positives in the population tested.
Likelihood ratio of a +ve test (LR+ve)= Sensitivity /1-specificity.
Likelihood ratio of a -ve test (LR-ve)= 1-Sensitivity /specificity
Pretest probability= prevalence in the population= diseased/
total no.
Pretest odds= convert probability above to odds (Odd= P/1-P).
Postest odds= Pretest odds X LR+ve
Postest probability= convert above odds to probability (Postest
P= O/1+O).
Accuracy= all true results/ all results = probability of correct
results.
reliability is the ability of a test to produce the same result
when repeated under identical conditions.
The validity of the test is defined as the relevance of the test to
the activities being treated.
Efficacy= the effect of something under ideal or lab conditions.
A clinical investigation examined the effectiveness of a
new test in diagnosing Pancreatic carcinoma. The
sensitivity was reported as 70%. Which one of the
following statements is correct?
1 )70% of people will be correctly classified as having
the disease
2 )70% of people with an abnormal test result will have
the disease
3 )70% of people with a + test result will not have the
disease
4 )70% of people with the disease will have an abnormal
test result
5 )70% of people with the disease will have a - test result
Q: The statistical reviewer of a paper states
that they are concerned that the findings
are biased. In statistical terms, bias
means:
A : There is a flaw in study design that leads to a built-
in likelihood that the wrong result may be obtained.
B : There is a flaw in statistical analysis leading to a
likelihood that the wrong result may be obtained.
C : There is reason to believe that the authors wanted
to obtain the result that the study showed.
D : Both study design and statistical analysis are
flawed, leading to a likelihood that the wrong result
may be obtained.
E : The study is not of sufficient statistical power to
exclude the missing of a significant effect.
A
Bias means a flaw in study design that leads to a
built-in likelihood that the wrong result may be
obtained. It cannot be controlled for at the
analysis stage. It can be extremely difficult to
design studies without potential bias, particularly
when there are complex interactions between
exposures under study. Techniques such as
restriction and stratification are commonly used
to reduce potential for bias.
A 95% confidence interval means:
A : 95% of the data fall within the confidence
interval.
B : there is a 95% chance that two groups are
different
C : that p=0.05
D : there is a 95% chance that the true value
falls within the confidence interval
E : there is a 95% chance that the finding is
clinically significant.
D:
Comment : The 95% confidence intervals (95% CI) around a value
are the range within which there is a 95% chance that the true value
lies.
Similarly, the 95% CIs around a difference are the range in which
there is a 95% chance that the true difference lies.
If the means of two groups have overlapping 95% CIs, then the two
groups are not statistically significantly different. If the 95% CI of the
difference between two groups overlaps zero, then the difference
between the two groups in not statistically significant.
Statistical and clinical significance should not be confused. A very
large study can generate very narrow 95% CIs (or very small p
values) for very small differences, which may be of no clinical
significance at all.
By contrast, a small study may fail to show a statistically significant
effect even if the effect is both large and clinically important.
Parametric tests means that the data will be of normal distribution.
1. Paired test or student T test is used if the members of the groups are
well matched = paired. E.g. each diseased pt is matched with
healthy individual of same age & sex.
2. Unpaired T test is used to compare the average values of 2
independent groups e.g. pts with & without disease/ treated vs.
placebo.
When you compare independent or diff groups, use unpaired t test.

Non- Parametric tests means that the data will be of Skewed


distribution.
1. Wilcoxon or Mann-Whitney U-test
2. Chi-square test Used to compare % or proportion between 2
groups.

In correlation coeffecient (r) r takes values between -1 & 1. the


closer it is to zero, the less linear association.
Chi-squared tests are used to compare 2 (%) or 2
proportions= used to test the difference between 2 nominal
variables (count data).
The larger the value X2 the smaller the p value, the more
significant the test.
Chi 2= X2= (observed-expected)2/expected to be compared
with standard critical or standard table for determined degree
of freedom.
degree of freedom= (raws-1)x(columns-1).
It assume that all table cells have expected freedum >1.
It assume that 80% of table cells have expected freedum >5.
Regarding the description and comparison of two groups
of data, which one of the following statements is true?
A : Categorical data should be described as percentages
and compared using a Students t-test.
B : Normally distributed continuous data should be
described as median and range and compared using a Chi-
squared test.
C : Skewed continuous data should be described as
median and range and compared using a Wilcoxon rank-
sum test.
D : Normally distributed continuous data should be
described as mean and standard deviation and compared
using a Chi-squared test.
E : Skewed continuous data should be described as mean
and standard deviation and compared using a Students t-
test.
C
Comment : Categorical variables are not continuous, e.g.
drug / placebo, dead / alive. They should be described
as percentages or proportions and compared with a Chi-
squared test.
Normally distributed continuous data should be
described as mean and standard deviation and
compared with a Students t-test.
Skewed continuous data should be described as median
and range and compared using a test such as the
Wilcoxon rank-sum test or the Mann-Whitney U-test.
researcher compared the mean scores of
nausea on a rating scale between standard
therapy & a new drug in the treatment of
chemotherapy induced nausea.
Which one of the following is the appropriate
statistical test?
1 )Chi-square test
2 )Paired T-test
3 )Life table analysis (log rank test)
4 )Pearson correlation
5 )Unpaired T-test
Answer 5:
The two-sample unpaired t test is used to test
null hypothesis in two populations
corresponding to two random samples are
equal.
For a paired t test, the data are dependent, i.e.
there is a one-to-one correspondence between
values in the two samples. For example,the
same subject measured before & after a
process change, & the same subject
measured at different times.
Q: To compare two groups of categorical
data, e.g. dead / alive by drug / placebo,
the correct test is:
A : Students t-test
B : Analysis of variance (ANOVA)
C : Wilcoxon rank-sum
D : chi-squared
E : p value
D:
Comment : Chi-squared tests (and variants there of) are
widely used to compare percentages or proportions of
categorical data. From the chi-squared statistic a p value
is read off a statistical table to give the degree of
significance. Traditionally a p value of less than 0.05,
indicating a less than 5% probability that a result has
arisen by chance, is taken (arbitrarily) as indicating that
chance alone is not responsible for the difference
between groups.
Normally distributed data can be compared with a
Students t-test (with correction for multiple comparisons
when appropriate).
Skewed continuous data can be compared with a
Wilcoxon rank-sum test or a Mann-Whitney U-test.
In a trial of a new drug the following results were obtained:-
treatment group 44 improved 16 not improved, placebo
group 36 improved 26 not improved.
A the results so obviously show the benefit of treatment
that statistical analysis is not required.
B the data could be evaluated using the chi-squared test
C Pearson's coefficient of linear regression would be an
appropriate significance test
D the numbers are too small to draw any conclusions
E a Student t-test could be used
answer true = b
a-Nothing is ever that obvious surely.
b-This data would be ideal for a chi-squared test.
c-nonsense there is no linear regression to plot
e-We are comparing proportions not means.
Q: Concerning the statistical power of studies, which one of
the following statements is FALSE?
A : International journals do not publish studies that are
underpowered.
B : A power calculation must always be performed before
conducting randomized clinical trials.
C : A type II error occurs if it is claimed two treatments are
the same when the study is not large enough to detect
equivalence.
D : A type I error is where the null hypothesis is falsely
rejected.
E : The smaller the difference you want to detect, the larger
a study must be.
A:
Comment : It is only ethical to conduct a clinical trial if it is capable of
detecting a meaningful difference between two treatments to guide
future practice. If a trial is underpowered it cannot detect a
statistically significant difference.
It is therefore mandatory to do a proper power calculation before
exposing patients to a clinical trial. The rather daunting formal
definition of a type I error means that a study falsely (but not
deliberately) appears to find a difference between two groups which
has actually arisen by chance alone. The conventional cut-off of p
<0.05 will arise by chance alone one time in twenty.
As many thousands of studies are published every month, type I
errors are not rare.
A type II error is formally where the null hypothesis is falsely
accepted. To claim two treatments are equivalent requires huge
numbers, and most studies are underpowered (too small) to reliably
rule out a small difference between one treatment and another.
Most published studies are small enough that type I and type II
errors are a real possibility, and examples are published in good
journals every week.
Q: A report of a clinical trial of a new analgesic
states "In a comparison between the new drug
and a placebo a higher proportion of patients
taking the new drug obtained relief from pain
(p<0.05)". It follows that:-
A the trial was well designed
B amongst 100 patients treated with the drug five
would be expected to have a placebo response
C the result may have occurred by chance alone
on less than one in 20 occasions
D the probable error of the observations is +/- 5%
answer true = c
Q: Regarding a randomized trial in which a new treatment for
Clostridium difficile diarrhea is compared with an established
treatment. A reviewer states that they are concerned that
there might be type 2 statistical error. What does this mean?
A : That the method of statistical analysis used is inappropriate.
B : That the study has shown a difference between the
treatments that is statistically significant but which is unlikely
to be clinically significant
C : That the study claims to find a difference that does not
really exist, ie. the result is a statisticalfluke
D : That the data is skewed (not normally distributed) and
analysis should have used non-parametric rather than
parametric statistical techniques
E : That the study claims that there is no difference between
the treatments, when in reality the trial was just too small to
detect a difference.
E:
The null hypothesis is always that there is no difference
between groups under study. A type 1 error occurs when
the null hypothesis is falsely rejected. In practice this
means that the study claims to find a difference that
does not really exist,. A type 2 error occurs when the null
hypothesis is falsely accepted. This means that it is
claimed that there is no difference between two groups,
when in reality the study is simply too small to detect a
difference. This type of error can be avoided by making
explicit power calculations before embarking on any
study. This will answer the question if I am studying an
outcome that occurs in (say) 20% of a conventionally
treated group and want to show a (say) halving in the
rate of this outcome, then how many patients do I need
to study?
A type 1 statistical error in a clinical trial means that:
A : patients were not allocated into groups with an
appropriate randomisation method
B : the null hypothesis is falsely accepted
C : the null hypothesis is falsely rejected
D : the statistical analysis was incomplete or incorrect
E : the statement of the hypothesis to be tested was
incomplete or flawed.
C:
Comment : A type 1 error is formally
defined as being where the null hypothesis
(which is that there is no difference
between the groups) was falsely rejected.
In practice this means that the study
claims to find a difference that does not
really exist.
Molecular medicine
Genome= complete complement of coding
genes.
Transcriptome= complete complement of
expressed m RNA.
Proteome= protein translated from m RNA.
2 organism may have the same n of genes but
diff proteins dt;
1. Variation in gene expression (temporal,
spatial).
2. Post transcription (diff. exons, splicing).
3. Post translation (glycosylation, sialydation).
Human genome = 30.000 gene.
Each cell express 16.000 gene.
Housekeeping genes =genes expressed in
all cells to provide basic function for cell
survival (constitutive).

Microarray analysis of transcriptome


identify the expressed genes.e.g.
Gene
3
5
RNA polymerase ex in ex in
binding site

Transforming factor TATA ATG


binding sites box Translation
Translation
termination code
Starting site

Promotor elements
Exons= segment of the gene transcripted into m RNA then translated into
proteins.
Introns= segment of the gene transcripted then removed by splicing (not
translated).
Promotor elements = binding sites for initiation of transcription complex at 5.
TF can activate any gene that has a TATA box.
TATA box=
- a promotor element.
- at 25-30 base pairs from the start of transcription.
- anchor to RNA polymerase II.
Enhancers=
- present at 5 or 3.
- not obligatory for initiation.
- but gene expression.
TF;
- basal = constitutive - Housekeeping genes.
- inducible = temporal, spatial expression of genes for tissue phenotype.
Application of TF;
1- many cong malformation are dt inherited mutation of TF.
2- can be oncogenic e.g. CMyC, P53.
3- steroids affect TF.
cyclins= are proteins key regulators of cell
cycles.
telomere = DNA sequence at the end of
each chromosome become progressively
shorter with each cell division when it is
reduced to a critical length, the cell is not
capable of dividing. The enzyme
telomerase lengthen it.
Haplotype patterns= group of genes or alleles
carried on the same chr, closely linked, travel
together during meiosis, inherited as a unit.
Gene mutations;
- mismatch = change in the nucleotide.
- inversion= nucleotide base removed, reverse
directed & reinserted.
- point mutation; single base pair substitution.
Analysis of the proteome is better as it
detects changes at the protein level, not
reflected at transcriptome level dt Post
translation processing.(bioinformatics).
Direct DNA testing = identify abn within
specific gene.
- PCR.
- restrictive enzyme digestion.
- southern blot.
- sequencing.
Indirect DNA testing = unknown gene by
tracking DNA markers in different
members of the family (linkage analysis).
southern blot (lab procedure); electrophoresis of DNA
fragment through gel solid memb as nitrocellulose+
labelled probe visualised under x ray film.
Northern blot is a mean to detect RNA (uracil instead of
thymine in m RNA).
Somatic cell hybridization;
- method for gene maping.
- using 2 diff species, chr from 1 species is selectively lost
resulting in clones of certain chr of the another species.
FISH; fluorescence in situ hybridization, labeled probes are
hybridized to chromosomes, and the hybridized probes are
detected with fluorochromes. visualised under florescent
microscope, This technique is a rapid and sensitive means of
detecting recurring numerical and structural abnormalities. for
microdeletions & trisomy.
SSCP (single strand conformation polymorphism analysis); is
a technique for detecting variation in DNA sequence by
running single stranded DNA fragments through a non
denaturating gel.

PCR
Def; it is an amplification reaction in which a small amount of target DNA
(template) is amplified to produce enough amount to perform analysis.
PCR is powerful (need only one copy).
Uses=
1. Detect viral or bact DNA.
2. Detect mutations.
Stages;
1. Mix the specimen with 2 primers & Taq polymerase (thermostable DNA
polymerase).
2. Heat & cool primer anneal to template.
3. Heat ( 72 c) polymerization.
4. Repeat & analyse.
Multiplex PCR; the use of more than 2 primers if more than one gene is to
be identified.
Nested PCR; when the sequence to be amplified need special definition e.g.
resemble others.
rt PCR (reverse transcriptase ); instead of DNA template, we take the m
RNA (expressed genes) & transform it into DNA by reverse transcriptase
enzyme (retroviral) as the m RNA is unstable.
uses; - detection of expressed genes in tumor cells.
- research; function of certain dis gene in diff tissues.
Preparation of monoclonal Ab;
For specific protein detection.
1. Inject Ag into an animal.
2. Hybridize its splenic cells + Myeloma cells (no
longer produce its Ab) Ab to this Ag.
3. Select most specific Ab tissue culture.
Uses;
- diagnosis (scan) & TTT of cancer (drugs as
majic bullet & radiotherapy).
- Transplantation & immunomodulation (OKT3).
Receptors
A) Cell membrane surface receptors;
1- ligand gated Ion channel;
e.g. neurotransmitter open ion channel
- Nicotinic Na.
- GABA, Glycine Cl.
2- receptors with protein tyrosine kinase (phosphorylation of
tyrosine residue of receptors cascade of cytoplasmic prot).
e.g. insulin, PDGF, prolactin, IGF1, MQ CSF, NGF, EGF.
3- G protein coupled ( is a protein that bind to guanine
nucleotide). e.g. muscarinic, adrenergic.
Dis associated with G protein abn e.g. cholera, Albright HOD,
MeCune Albright S, pit adenoma.
NB; prot kinases add phosphate group to serine, threonine,
tyrosine residue (# phosphatase)
B) Nuclear H;
- no 2 ry mess.
- intracytoplasmic receptors.
- the complex travel to the nucleus & bind
to hormone responsive elements.
Molecular pathogenesis of cancer
1. Somatic evolution of Ca; escape from strictly regulated mechanism
that control the growth of somatic cells.
2. Oncogenes; geneprotein cancer (=loss of growth control)
protoncogene=Oncogene that is normally present in human cells i.e.
C-onc e.g. C-Myc.
Mutations resulting in tumors;
RAS = G protein cell growth 1/3 of tumours if mutated.
Mutations of protein kinasesTF (Fos & Jun)+ Myc tumor.
In Burkitt lymphoma C Myc is transposed to Ig heavy chain locus
on chr 14 in lymphocytes its expression & cellular mitosis.
Philadelphia chr bcr + abl (9;22) fusion protein tumor growth.
3. Tumor suppressor gene;
P53= normally function to inhibit cell cycle & abn growth, +
apoptosis, if mutated tumor (most common cause of tumours).
Le- Fraumeni S; AD dis, ccc by cancer breast, sarcoma, brain dt
inactivation of P53.
P27 Tumor suppressor gene through down regulation of cell cycle
(cyclin dependent kinase inhibitor), if downregulated sporadic
Cancer colon.
Apoptosis
Def; is the morphological changes that accompagny
the programmed cell death e.g. cell shrinkage,
compaction of chromatin, nuclear & cytoplasmic
apoptotic bodies phagocytosed by MQ, laddering of
DNA on electrophoresis gel by activation of
intracellular nucleases.
programmed cell death= naturally occurring cell
death dt activation of a set of genes in response to
ext signals e.g. from neighbour or extracellular
matrix.
Apoptosis;
- non-inflammatory process.
- no proteolytic enzymes.
- no free radicals.
- no damage of neighbouring cells.
E.g.
apopt of finger web, selection of neurons ( normal apopt in embrio).
- apopt of excess or autoreactive T lymphocytes ( normal apopt in adult).
- neurodegenerative dis, HIV (dis).
- insufficient apoptosis e.g. cancer, autoimmune dis, viral dis.
Factors that + apoptosis; P53, P27, Fas or CD95 (receptor for TNF),
withdrawal of GF.
Factors that - apoptosis; bcl2 (survival signals), B catenin accumulation
adenoma.
Apoptosis occur through proteases called caspases (e.g. ICE= IL-1B
converting enzyme) that + endonuleases.
Caspases = cysteine aspartate specific proteases.
P53 +

Fas, CD 95 + _
Signals for cell death bcl2
+
TF
+

caspases

endonucleases
Cancer resists apoptosis by;
- Mutation of P53.
- Causing apoptosis of cytotoxic T cells
(TNF like + Fas).
- Over-expression of Bcl2.
Nitric oxide
(NO)
Endothelial derived relaxation factor
Produced from L arginine by oxidation of Nitrogen NO + citrulin.
C GMP (2ry mess) in neighboring cells.
Produced in;
1. Constitutive..Vascular end & Nervous system VD, sm
hyperplasia, plat agg. new memory.
2. Inducible.. in MQ, PNL, plat, hepatocytecytotoxic.
Clinical application;
- So used as nitrates or inhaled NO in pulm HTN.
- endothelial dysf in DM, HTN, smokers & hypercholestrolemia is dt
loss of NO bioavailability.
- NO in atherosclerosis, HTN dt CRF, HRS, Alzeheimer.
- NO in septic shock, ARDS, acute inflammation.
Endothelin I (VC)
ET-1 (end, sm, coronary, GIT).
Related to dis;
- HTN, HRS, ARF, CHF, Raynaulds. (VC)
- VC following subarachnoid Hge.
ET1 receptor blockers & CEI used as anti HTN.
Pro-inflammatory cytokines

Il-1, TNF, TGF-B, Heat shock protein, free radicals.


IL-1; Involved in;
- Rh arthritis IL-1 + collagenase, phospholipase,
cyclooxygenese (facilitator of damage).
- Atherosclerosis; endo uptake of LDL IL-1
PDGF.
- Septic shock IL1 NO, PG, PAF VD.
- Infection, acute graft rejection IL1 T & B
lymph.
TNF; + GM- CSF, + PG.
MQ, esinoph, NK.
T lymph .
dis associated; Rheumatoid, MS, MOF. So,
neutralizing Ab = anti- TNF are used.
not used in cancer highly toxic, + tumor
growth.
TGF-
action;
- tissue repair.
- extracellular matrix.
- fibrosis.
tissue injury + plat release of TGF-B
chemotaxis monocytes (+ fibroblast GF,
TNF, IL-1)
involved in glomerulosclerosis, hep fibrosis,
pulm fibrosis, bleomycin lung.
NB;
HSPs (Heat shock proteins)
heat, chemicals, free radicals damage of
intracellular proteins HSP cell
resistance to stress through;
- prot folding & unfolding.
- degrad of prot ( by ubiquitination).
dis associated; if mutated cataract, motor
neuron deg.
bact HSP + immune syst.
Free radicals
Any molecule with 1or more unpaired electron (more
reactive); peroxide, super, hydroxyl, NO.
NB; hydroxyl is the most reactive.
action;
- lysosomes.
- lipid peroxidation of memb.
- mutations (by attaching purines & pyrimidine).
Diseases athero, cancer, neurodeg ( MND).
Free radical scavengers;
- tocopherol (Vit E).
- ascorbate (Vit C).
- glutathione.
- Beta carotene.
- Flavenoids.
Adhesion Molecules
Def; Molecules that interact as receptors & ligand.
4 groups;
- Ig ( CD2, CD3, NCA (neural cell adh), ICAM (intercellular)
bind to LFA ( lymph funct ass)to recruit lymphocytes.
- integrin ( cell to matrix) Integrins are surface receptors by
which cells are attached to extracellular matrix..
- Cadherins (Nerve & Muscle ).
- selectins ( leukocytes to endoth in inflam, over expressed
in autoimmune viral hepatitis, organ rejection).
Clinical application;
1. leuk adhesions deficiency recc bact sepsis.
2. integrin IIb IIIa ( plat receptor to fibrinogen)
deficiency Glansman thrombathenia.
Ab (abciximab) antithrombotic in coronary Ht.
Stem cells
progenitor cells.
present in certain tissues e.g. BM, embrionic.
embryonic totipotent (any tissue).
BM Bl. cells only, can be recruited by Ag
sorting with CD34 Ab & undergo
transdifferentiation to non hematologic cells.
47. A 24-year-old man presented with agitation and central chest
pain. He had been taking cocaine earlier in the day. On examination,
he had a pulse of 100 beats per minute and a blood pressure of
170/110 mmHg.
Which complication is most likely?

A hypercalcaemia
B hyperkalaemia
C hyperthermia
D hyponatraemia
E hypothermia

C
Drugs affected by acetylator status;
DHIPS
(Dapson, hydralazine, INH, Procainamide sulphonamides)
Slow acetylators liable to adverse effect of the drug itself
e.g. drug induced lupus.
INH induced PN.
Fast acetylator liable to adverse effect of the drug metabolites e.g. INH
induced hepatitis & drug resistance.
Fast acetylation is a trait which is autosomal dominant inherited.

The difference in speed of acetylation is due to the amount (or activity) of the
enzyme N-acetyltransferase available.
Drug induced lupus
Drug list;
- Anti HTN; MD, BB, clonidine, Hydralazine
- antiarrhythmics; procainamide, flecainide
- antibiotics; sulfa, minocyclin, INH.
- Neuro; chlorpromazine, haloperidol, lithium, phenytoin.

C/P; arthralgia, butterfly rash, pleurisy, equal sex.


NB;- no renal involvement - no CNS.
- Drug induced lupus occur with minocyclin not oxytetracyclin.
- associated with slow acetylators & HLA DR4.
- subacute cut lupus can occur with thiazide.
Investigation;
- ESR, CRP, hypergammaglob,
- +ve ANA, +ve antihistone
- But anti DNA & RF are ve.
TTT; Remits slowly when drug is stopped but may need steroid
treatment to settle.
Rate of drug metabolism
Drugs metabilized by :
Zero order kinetics;
i.e. drugs whose metabolism is constant/
unit time, so dose plasma cons.
(liable to drug intoxication). E.g. phenytoin,
hydralazine, lithium, theophilline, alcohol,
fluoxetin.
1st order kinetic;
i.e. drugs whose metabolism is proportinal
to drug conc. i.e. dose metabolism
constant serum level (linear conc).
Drugs which are highly bound to plasma
protein ;
- sulphonylureas,
- oral anticoagulants,
- salicylates,
- propranolol, ACEI, clofibrate.
Enzyme inducers;
i.e. induce the synthesis of hepatic enzymes (cytochrome P450)
responsible for drug metabolism over days Treatment failure.
List; PC BRAS + griseofulvin
(Phenytoin, Carbamazipine, Barbiturates, Rifampicin, chronic Alcohol,
sulphonylureas, griseofulvin).
NB: no antibiotics!

Enzyme inhibitors;
i.e. inhibit the synthesis of hepatic enzymes
List; OAAK DEVICES (Omeprazole,Allopurinol, Amiodarone ,
ketoconazole, Disulfiram, Erythromycin, Valproate, INH, Cimetidine,
cipro, acute ethanol, Sulphonamides)
+ Quinupristin & dalfopristin, simvastatin, fibrates.

NB: many antibiotics!

Drugs that are affected;


Warfarin, OCP, steroids, phenytoin, carbamazipine, theophillins,
cyclosporin, azathioprine.
B lactam antibiotics
Inhibit bact cell wall., bactericidal
1. Penicillins
2. Cephalosporins
3. Monobactams= Astreonam (aerobic G-ve
bacilli, so used instead of garamycin for abd
sepsis.
4. Carbapenems.
- Imepinem + cilastatin (Tienem)
- Meropenem
Penicillins
Benzyl pen (Pen G); contain B lactam ring.
Pen V (oral, acid resistant.)
Ampicillin, amoxacillin (bread spectrum)
Antipseudomonas; Carbinicillin, piperacillin, ticarcillin, Azlocillin.
Antistaph
1- B lactamase inhibitors;
- Clavulonic acid + amoxacillin (Augmentin)
- Sulbactam + ampicillin (Unasyn)
- Tazobactam + piperacillin
2- B lactamase resistant
- Mecicillin, oxacillin, cloxacillin

Mechanism of action; bind to peptidases involved in cell wall synthesis(-) cell wall
synthesis.

MRSA; staph that retain peptidase activity even in the presence of mecicillin.Ttt;
vancomycin, teicoplanin

Side effects; diarrhea


ampicillin + IMN 90% rash.
co-amoxaclav cholestatic jaundice
Cephalosporins
B lactam antibiotics
Active # G+ve & G-ve except enterococci, listeria &
anaerobes.
CLASSIFICATION
1. First generation; (cefazolin) most G +ve cocci except
enterococci, MRSA, or penicillin-resistant
pneumococci
2. Second generation less active against G +ve cocci
but are more active against certain G ve
bacilli (cefuroxime)
3. Third generation more active against G-ve bacilli, and
are highly active against Enterobacteriaceae.
(cefotaxime, ceftriaxone, and ceftizoxime,
cefoperazone and ceftazidime (antipseudo)
4. Fourth generation (cefepime) better penetration
through the outer membrane of G-ve bacteria.
NB;
All of the cephalosporins except cefoperazone and ceftriaxone
require dose modification in the presence of severe renal failure.
Biliary concentrations are particularly high for cefazolin,
cefoperazone, and ceftriaxone.

The third generation achieve much more reliable CSF levels in


patients with meningeal irritation. Cefotaxime, ceftriaxone, and
ceftazidime are approved for the treatment of bacterial meningitis.

Fatal reactions due to calcium-ceftriaxone precipitates in the lungs


and kidneys of neonates have been reported. The FDA has issued
an advisory that IV ceftriaxone and IV calcium-containing solutions
should not be co-administered to neonates, children or adults within
48 hours of one another.
Carbapenems.
- Imepenem, Meropenem
- B lactam antibiotics
- The most broad spectrum Ab, Antibacterial activity
includes resistant G-ve bacilli (Pseudomonas aeruginosa
and Enterobacter sp), G +ve (MRSA and Streptococcus
sp) and anaerobes.
- Inactivated by the kid, so administered in combination with
cilastatin (Imepenem) seizures if given in meningitis esp
in renal impairment.
Side effects;
Central nervous system: Seizure
Dermatologic: Rash
Gastrointestinal: Nausea , diarrhea , vomiting
Genitourinary: Oliguria/anuria
Local: Phlebitis/thrombophlebitis , pain at I.M. injection
site.
Side effects of B lactams
allergic reactions
Serum sickness
drug-induced lupus,
Skin; morbilliform rash is the most common. Erythema multiforme , Stevens-
Johnson syndrome. Exfoliative dermatitis. Toxic epidermal necrolysis ..
Hypersensitivity angiitis - palpable purpura. photosensitivity reactions.
Neurologic reactions high-dose penicillin therapy (>20 million units per day),
are the most common to cause encephalopathy. Imipenem in renal disease.
Pulmonary eosinophilia,
GIT;
- Diarrhea, esp ampicillin or amoxicillin. Clostridium difficile colitis, esp with
ampicillin .
- hypersensitivity hepatitis. Ceftriaxone may cause biliary sludge.
- Suppression of gut flora vitamin K deficiency Hypoprothrombinemia
Renal reactions
- Glomerulonephritis with hypersensitivity angiitis or serum sickness.
methicillin allergic interstitial nephritis
- ticarcillin (disodium salt), sodium overload and hypokalemic alkalosis .
Hematologic reactions immune-mediated destruction of PNL,with
eosinophilia, hemolytic anemia, immune thrombocytopenia. Platelet dysfunction
may be caused by high doses of ticarcillin.
Macrolides
(-) protein synthesis
Spiramycin Toxoplasma
Erythromycin
Clarithromycin.
Azithromycin.
Uses; legionella, Mycoplasma, chlamidia, coxiella,
Campylobacter, bordetella pertussus.
Side effects;
- prokinetic V, D, abd pain.
- Cholestatic jaundice
- QT torsade de point
enzyme inhibitor, so toxicity of theophillin, digoxin,
carbamazepine, cyclosporin.
fluoroquinolones
MECHANISMS OF ACTION;
(-) DNA gyrase , Nucleic acid synthesis
bactericidal
wide spectrum of activity.

Spectrum;
- against aerobic G-ve bacilli, particularly Enterobacteriaceae,
Haemophilus , Pseudomonas and G-ve cocci as Neisseria spp. ,Moraxella
& staphylococci
- Levofloxacin and moxifloxacin, however, have greater potency against
gram-positive cocci, anaerobic bacteria & TB, atypical pneumonias;

PHARMACOKINETICS
oral bioavailability, 70 95%.
No dose adjustment of moxifloxacin in the presence of renal failure.
Otherwise The dose in patients with creatinine clearances below 50
mL/min.
Clearance by hemodialysis is low
Should be avoided in pregnancy, lactation & children < 18 yrs old.
Side effects;

- GIT; mild ANV, less frequent diarrhea.


- CNS; headache and dizziness, seizure with theophylline and NSAIDs
- Rash and other allergic manifestations, photosensitivity
- Arthropathy, Tendinitis and tendon rupture
- QT prolongation with moxi
-Hypoglycemia and hyperglycemia levofloxacin, gatifloxacin was
withdrawn from the market in the United States and Canada in June
2006.
- Hematologic Leukopenia and eosinophilia in<1%
- Drug interactions;
IV ciprofloxacin, precipitates with aminophylline, amoxicillin
Enzyme inhibitor esp cipro theoph.
Aminoglycosides
(-) protein synthesis
Against G-ve
e.g. Amikacin, Gentamicin, Kanamycin, Neomycin, Streptomycin,
Tobramycin,
Amikacin is more potent than gentamycin & Tobramycin .
Aminoglycosides should only be used for treatment of serious
infections because of their potential toxicity and antimicrobial
spectrum.
Adverse Effects;
- irreversible, cumulative ototoxicity, the vestibular system (manifest
as dizziness or vertigo).
- Reversible nephrotoxicity may occur and acute renal failure has
been reported, often in association with the use of other nephrotoxic
drugs ,
- neuromuscular-blocking action and respiratory depression and
muscular paralysis
(-) protein synthesis
Tetracyclins
Its efficacy by antacids, Fe therapy & dairy products.
Used for;
1. Acute bacterial exacerbations of chronic bronchitis, mycoplasma.
2. brucellosis (with streptomycin), leptospirosis,
3. rickettsial infections,
4. chlamydial infections (doxycycline), granuloma inguinale,
5. Borrelia (Lyme disease and relapsing fever; doxycycline), syphilis,
6. Vibrio cholera , plague, Vibrio vulnificus,
7. Mycobacterium marinum (minocycline),
8. actinomycosis in the penicillin-allergic patient
Side effects;
1. Gastrointestinal distress.
2. Esophageal ulceration; Doxycycline (take in A.M. with fluids)
3. Outdated tetracyclins fanconi S.
4. Enhance established or incipient renal failure.
Doxycyclin (vibramycin) is the safest in renal failure but may cause photosensitivity.

Chloramphenicol
(-) protein synthesis
Potentiate the action of anticoagulant, phenytoin, hypoglycemics.
Aplastic an.
sulphonamides
Competitively inhibit enzymes in folic acid biosynthesis (-) cell
metabolism.
Uses;
1. Nocardial infections,
2. leprosy (dapsone, a sulfone),
3. toxoplasmosis (sulfadiazine)
4. Trimethoprim-sulfamethoxazole (Cotrimoxazole) is used in
pneumocystis carinii, listeria, UTI.

Side effects;
1. Allergic reactions; Rashes ; erythema multiforme, Stevens-Johnson
syndrome, toxic epidermal necrolysis.
2. Hematologic reactions; Uncommon; include agranulocytosis and
granulocytopenia , hemolytic and megaloblastic anemia,
thrombocytopenia.
3. Renal insufficiency, Crystalluria with sulfadiazine therapy
Fusidic acid
Active against penicillinase producing
staph aureus (narrow spectrum)
Well absorbed orally
Used in osteomyelitis, endocarditis, staph
septicemia
Safe , can be used in pregnancy
Occasional hepatotoxicity.
Linezolid (Zyvox)
Bacteriostatic
(--) protein synthesis.
Active against G+ve resistant org.;
- penicillin resist. Strept.
- mecithillin resist. Staph (MRSA)
- Vancomycin resist enterococc. (VRE).
Side effects;
- GIT: ANVD, gastritis, pancreatitis.
- Bl; leucopenia, pancytopenia.
- CNS; dizziness, bluring, tinnitus
- Skin;rashes.
- drug interaction as MAOI; serotonergic & adrenergic drugs.
Glycopeptides
Vancomycin, Teicoplanin (Targocid).
Uses;
- MRSA
- penicillin resist strep pneum (Meningitis)
- oral in Cl difficile.
Side effects;
- oto/ nephrotoxicity.
- if given rapidly release of histamine (red man S)
- if extravasation necrosis.
NB; Teicoplanin
- less nephrotoxic & once daily dose.
- but many side effects as linizolid (GIT, Skin, CNS, Blood)

Quinupristin & dalfopristin (synercid)


(-) protein synthesis
For G +ve mainly esp resistant strept pneum. & staph aureus.
Only given through central line.
Enz. Inhibitors.
Side effects; arthralgia, myalgia, thrombophlebitis.
Metronidazole, tinidazole (fasigyn)
Nucleic acid synthesis
Used for;
- anaerobics esp bacteroids.
- ameba, Giardia
- trichomonas vaginalis.
Drug interaction;
- disulfiram like action
- anticoagulant.
Toxicity;
- Metallic taste
- PN after prolonged use.

Clindamycin
(-) protein synthesis
Used for anaerobic infections; e.g. type 1 necrotizing faschitis.
Side effect; diarrhea, pseudomembranous colitis by clostridium
difficile.
Antimicrobials
Antistaph Antibiotics;
Mecicillin, oxacillin, Amoxc/clavulonate, Unasyn.
- Erythromycin,
- Vancomycin
- 2nd & 3rd g cephalosporins
- Cotrimoxasole, quinolones
- clinda, fusidic acid

Antipseudomonas;
- Carbinicillin, piperacillin, ticarcillin
- Aminoglycoside (Tobramycin),
- 3rd G ceph (ceftazidime),
- quinolones,
- Tienam

Anaerobics;
- Metronidazole,
- Clindamycin
- Tienam
Antifungal
Mucous membrane;
- clotrimazole, econazole, Nystatin (not absorbed).
Skin;
- terbinafin, griseofulvin.
Deep;
- Candida Albicans fluconazole.
- other candida, aspergillosis amphotericin B.
- CSF cryptococcal meningitisfluconazole, amphotericin B, 5 flucytosine.

Side effects;
amphotericin B; nephrotoxicity, K, Mg, neurotoxicity.
Ketoconazole; hepatotoxic, enzyme inhibitor, cortisone & androgens (medical
adrenalectomy).
5 flucytosine mild side effects, BM suppression.
Griseofulvin enz inducer, Stevens Johnson S, headache, neurotoxicity.
Antiviral
Nucleosides analogues (-) DNA polymerase.
Acyclovir HSV,
purine analogue,
so highly viral specific & very low toxicity.
Gancyclovir 1st line for CMV.
significant side effects; BM, gonadal toxicity (-)
spermatogenesis.
Foscarnet 2nd line for ttt of CMV,
severe side effects esp renal damage.
Cidofovir for CMV in HIV patients, nephrotoxic.
Lamivudine, Adefovir, entecavir HBV.
Zidovudin, didanosine, lamivudine, adefovir HIV.
Amantadine proph & TTT of influenza A (not B),
parkinsonism (- dopamine uptake).
side effects; (CNS); insomnia, dizziness & headache.
Neuraminidase inhibitors ;
- Zanamivir (inhalor),
- Oseltamivir (Tamiflu) in influenza A & B, less side effects, given oral.
Ribavirin ; HCV, Lassa fever, inhalation in children with RSV.
INF- B, C, some malignancy (RCC, Kaposi).
Migraine
Precipitating factors- chocolate, alcohol, smoking, stress,
type A personality, bright light, hunger, OCP.
TTT;
- During the attack;(in order)
- paracetamol
-codeine + anti-emetic
- ergotamine (# in preg, renal, PVD, CVS)
- sumatriptan (5HT1 agonist)
- Prophylactic (if attacks are frequent; > 2/months)
- propranolol
- amitriptyline (TCA)
- Methysergide (retroperitoneal fibrosis)
Sumatriptan is a 5HT1 agonist and may
be useful in the treatment of acute
migraine attacks. is available in injectable,
intranasal, and oral formulations.
Ergotamine tartrate is also effective in
acute migraine. Propanolol, valproate,
NSAIDs,
amitriptyline, pizotifen and gabapentin are
effective as prophylactic drugs in migraine
Benign intracranial Hypertension
(BIH)
Causes;
- obesity, behcet, pregnancy
- Addison, PTH, PCO
- Drugs;
- OCP e.g. Dianette.
- steroid use or withdrawal.
- systemic tetracyclin, vit A, retinoic acid
- nitrofurantoin, nalidexic acid.
- lithium.
TTT;
- wt loss, VP shunt, repeated lumbar puncture
- acetazolamide.
- glucocorticoids; 30-60 mg/d.
Antiepileptic drugs

Phenytoin Valproate Carbamazepine

uses Tonic clonic All generalized -Partial


all except -5th neuralgia
absence -Diab neurop
Therap level=10- -Migraine
20 Ug/ml -DI
Drug Enz inducer Enz inhibitor Enz inducer
interaction level of; If uses with phenytoin
-Oral anticoag. dose of phenytoin
-OCP
--cyclosporine
-Other antiepil
Phenytoin Valproate Carbamazepine
Side effects
CNS depression
hepatotoxic Hepatitis, Fatty, Reye S
granuloma
GIT upset , pancreatitis
Blood Megaloblastic an plat pancytopenias
LN, gum thrombathenia
skin Lupus, Hirsutism Gynecomastia Toxic epidermal necrolysis
Serum sickness Rash, alopecia
endocrine DM Wt gain, PCO SIADH
Bone
(osteomalacia)
Neuro PN, enceph Ataxia Headache, diploplia
Chorea, Nystag tremors
Cereb ataxia,
teratogenic Neural tube defect Neural tube Toxic epidermal necrolysis
(cleft, cong ht) defect
Gabapentine;
- used as add on drug
- in partial, general.
- not enz inducer.
-CNS side effects.
- Dose adj in renal failure.

Lamotrigine rash TEN.

Vigabatrine irreversible constriction of visual field.

Levitiraceticam
- oral
- Safe, fewer side effects
- used for partial seizures.
- No effect on liver enz
- Dose adj in renal failure.

Phenobarbitone 1st line in infants.

Paraldehyde may be given IVI or rectally or IM in extreme circumstances


because it cause tissue damage & slughing.
Drug induced neurological symptoms
1. Drug induced extra symptoms;e.g Dystonia
e.g. antipsychotic e.g., antiemetic.
ttt; stop the drug, give anticholinergic for tremors e.g. Benztropine or
procyclidine.( akeniton tab 1x1, parkinol)
2. Tardive dyskinesia;
- Antipsychotic as chlorpromazine & Haloperidol, unusual with newer
agents as olanzapine, clozapine.
- After 6 months of the drug intake.
- 50 % irreversible, progressive.
- C/P; orofacial dyskinesia e.g. chewing, lip smacking.
- ttt; stop & give - tetrabenazine
- reserpine, valproate.
- baclofen 10, 25 mg 1x2, diazepam.
3. Malignant neuroleptic S;
- C/P; altered mental state, rigidity, autonomic dysf, fever, high CPK
- Cause; potent neuroleptics e.g. haloperidol.
- ttt; dantrolene, bromocriptine.
- Complications; Rabdomyolysis, ARF.
Neuroleptic malignant syndrome is characterized by fever, muscular
rigidity, labile blood pressure, altered mental status, decreased conscious
level and autonomic dysfunction. Although potent neuroleptics (eg,
haloperidol, fluphenazine) are more frequently associated with NMS, all
antipsychotic agents may precipitate the syndrome. For example, these
agents are prochlorperazine, promethazine , clozapine , risperidone .
Dystonic reactions usually subsides within 24 hours following cessation of
treatment and can be treated with procyclidine 5-10 mg i.m.
They are well-recognized with dopamine receptor antagonists
(neuroleptics). Phenothiazines, prochloperazine, haloperidol and
metclopramide are examples of drugs which can cause dystonic reactions.
Oculogyric Crisis is one of the acute dystonic reactions. It is the most
common of the ocular dystonic reactions (which include blepharospasm,
periorbital twitches, and protracted staring episodes).
Priapism as a side effect is associated with phenothiazines
(chlorpromazine), haloperidol, trazodone and alpha blockers (prazosin).
Drug induced peripheral neuropathy;
- amiodarone
- INH, DDI, metronidazole, nitrofurantoin
- Gold, cisplatin, vincristin, vinblastin.

Drug induced Myopathy;


- statins, fibrates
- Corticosteroids, cocaine, colchicine, chloroquine
- Zidovudine

Drugs provoking Myasthenia


Aminoglycosides.
B. Blocker.
Quinidine.
Procainamide.
penicllamine.
Phenytoin.
Lithium.
Antidepressants
TCA overdose
Mech; anti-cholinergic (dilated pupil, confusion, tachycardia), blocking
hypotension.
E.g. procyclidine used to treat parkinson induced by neuroleptics.
ECG QT
TTT;
- IV Bicarb reduce risk of seisure & arrhysth.
- pacing
- activated charcoal, emesis, gastric lavage
- fluid & electrolytes
- diazepam for convulsion

Mortality is high; 20%

Quinidine poisoning;
As TCA (anticholenergic, blocking, QT )+ irreversible blindness.
Common side effects of Selective Serotonin
Reuptake Inhibitors (SSRIs) are nausea,
vomiting, diarrhoea, appetite and weight loss,
sexual dysfunction and deranged liver function
tests.
Similarly, the common side effects of fluoxetine
are :anxiety or nervousness; decreased
appetite; diarrhoea; drowsiness; headache;
increased sweating; nausea; tiredness or
weakness; trembling or shaking; insomnia.
Lithium
-uses;
1- prophylaxis in bipolar affective disorder
2- acute manic episode.
2- augment action of antidepressant in recurrent, resistant
depression.

- It has a narrow therapeutic window (0.6-1.2mmol/l).toxic > 2 mmol/l.


- Take Blood sample for therapeutic monitoring 12 hr after the last
dose
- Toxicity increase when renal excretion decrease as with RI, ACEI,
NSAIDs, thiazide, loop diuretics, tetracyclin, phenytoin, ciclosporin.
- Serum level with antacids, theoph, acetazolamide.
Lithium Toxicity;
- neuro; lethargy ,tremor ,confusion ,seizure.
- GIT; N, diarrhea.
- renal; DI, RTA, ARF, CIN.
- goiter, hypothyroidism
- hypercalcemia.
- TTT:
- gastric lavage
- Replete volume/forced diuresis
- HD;
- level > 4 mEq/L
- Level > 2.5, severe CNS symptoms
Antipsychotics

Indication of antipsychotics
-Schizophrenia
-Mania
-Paranoid psychosis
-Psychotic depression
Atypical antipsychotics Examples Side effects

Highly selective blockade sulpiride hyperprolactenimia


of D2 & 5 HT2A
Olanzapine - Wt gain
Resperidone - N, dyspepsia
clozapine - Bl dyscrasia;
Neutropenia
Agranulocytosis
-Myocarditis,
cardiomyopathy
-Hyperglycemia, DKA

Atypical antipsychotics are the 1st line ttt of schizo (+ve symptoms)
dt less side effects.
Clozapine is used for severe tardive dyskinesia
sexual dysfunction
Cholestatic jaundice
Clozapine induced agranulocytosis
occurs in about 1% to 10% of patient who
take clozapine. Patients who have
experienced agranulocytosis with prior
treatment of clozapine should not receive
clozapine again.
Quetiapine is indicated for the
management of the manifestations of
schizophrenia. The commonest side
effects (>5%) are excessive sedation,
dizziness, dry mouth, postural
hypotension, and elevated ALT.
-interferon is a long term treatment (as
opposed to steroids for acute relapses)
which is of benefit only in the relapsing
remitting form (about 40% of MS patients
have this form), and slows progression of
disability and reduces demyelinating
lesions.
Chemotherapy
Chemotherapy
1. Alkylating agents
2. Antimetabolites
3. Antibiotics
4. Plant alkaloids
5. Hormonal therapy
6. Molecular targeted therapy
7. Biological therapy
- immune therapy
- gene therapy
Cell cycle:

G1 phase ; enzymes for the production of RNA and protein, are


produced.
S phase ; DNA is synthesized by replication of the existing strand.
G2 phase ; RNA and protein are synthesized.
M phase mitosis occur:
Cell division results in 2 daughter cells, each of which can
enter its own G1 phase
become an inactive resting (G0) or
die (cell loss fraction).
Mechanism of action
phase non-specific: act at any phase or when cells are
resting;
- alkylating agents:
- antibiotics: actinomycin D,

phase specific: act only in a specific phase of the cell


cycle & are only active when the cells are not resting.
E.g. plant product
vinca alkaloid: vincristine, vinblastine (M phase)
etoposide (G2 phase)
taxol (G2 phase)
antibiotic: bleomycin (G2 phase)
antimetabolite:(s phase)
Alkylating agents
Mech;
- bind covalently to bases of DNA cross linkage
Apoptosis.
- cell cycle non-specific.
- used for slowly growing cancers e.g.CML, cll, MM,
lymphomas.
- side effects; 2ry neoplasms as leukemia, sterility.
e.g. cyclophosphamide (endoxan) hemorrhagic cystitis,
sterility, BM suppression.
chlorambucil (leucoran)
Melphalan
busulfan (myleran) lung fibrosis.
cisplatinum Nausea, PN, deafness, nephrotoxic,Mg
dose in renal.
procarbazine
mustine
Antimetabolites
Mech;
- compounds that structurally simulate purine & pyrimidine precursors
interfere with P & P synthesis.
- S phase specific.
- used for rapidly growing cancers; AML, ALL.
- side effects; stomatitis, diarrhea, BM suppression, no 2nd neoplasms.
E.g.
- methotrexate;
- inhibit dihydrofolate reductase reduced folate thymine.
- side effects; BM & mucosa, crystalluria, dose in renal, hepatic fibrosis,
pneumonitis, CNS dysf. given with leucovorin formyltetrahydrofolate,
- Cytosine arabinoside (Arac) # pyrimidine ARDS, BM, mucositis, ataxia.
- 5 fluorouracil # pyrimidine. BM, mucositis, DVT.
- Capecitabine; in HCC, 5FU prodrug, intratumoural metabolism Hand
foot s, diarrhea.
- Azathiprine & 6 Mercaptopurine # purine BM, liver, Nausea,
metabolized by xanthine oxidase so the dose with allopurinol.
- Mycophenolate Mofetyl
- Hydroxyurea (-)ribonucleotide reductase in myeloproliferative states
Azathioprine
A purine analogue ,
rapidly converted to 6-mercaptopurine
inhibits DNA and RNA synthesis, and thus cell proliferation
Prevents early stages of activated T & B-cell proliferation. (Non-selective
immunosuppression)
Metabolised with xanthine oxidase which is inhibited by allopurinol give
Quarter dose or Better , not to be used in combination with allopurinol.
NB; dont begin aza during pregnancy but if pregnant was on it, continue,
dont stop.
Approximately 1 in 300 Caucasians have thiopurine methyl transferase
(TPMT) deficiency. TPMT is the enzyme that metabolises 6-mercaptopurine
and its deficiency results in high risk of azathioprine toxicity.
Azathioprine is used when steroid withdrawal causes recurrent relapse.
Bone marrow suppression (low white cell count) and pancreatitis are side
effects of azathioprine
Principal side-effect is
Bone-marrow suppression (Reversible)Monitoring by CBC ,platelets,
Reduce if WCC<4, stop if <3, reintroduce at a lower dose when >3
Hepatotoxicity (Reversible) ( reduce dose if hepatic dysfunction present )
Pancreatitis
Antibiotics
Mech;
- bind directly to DNA & generate free radicals.
- e.g.bleomycin; Pulm fibrosis, in renal.
Adriamycin ); () Topoisomerase ( enzymes
responsible for DNA unwind for duplication). BM,
cardiac, mucositis.
Mitomycin HUS, CHF, Pulm fibrosis, BM, in
renal.
Plant alkaloids;
Mech;
- cell cycle specific (M phase).
- mitotic spindle inhibitors.
e.g.
- vinblastine plat, HTN, raynauds, PN.
- vincristine (Oncovin) SIADH, paralytic ileus, urine
retension ,plat, jaw pain, PN.
- Docetaxel; (G2 specific) stabilize microtubules against
depolymerization. The "stabilized" microtubules are not
able to undergo the normal dynamic changes necessary
for cell cycle completion.
broad spectrum against solid tumoursPN, arrhysthmias,
fluid retension, BM, alopecia, mucositis, hypersensitivity
reaction so premedicate with steroids, anti H1.
Molecular targeted therapy
Mech; products of oncogenes & tumour suppressor
genes, regulators of cell death pathway, mediators
of cell immortality e.g. telomerase, proteases,
angiogenic factors.
e.g.;
1- proteozome inhibitor bortizomib in MM (-)
proteozome which P53 P53 apoptosis;
postural hypotension, hypoNa.
2- Imatinib (Gleevec) in CML abn. Tyrosine
kinase produced by bcr-abl gene, induce phil ve
haematopoisis
3- Tretinoin teratogenic, pseudotumour cerebri.
Hormonal therapy;
- High dose steroids;
- Tamoxifen, aromatase inhibitors partial Estrogen
antagonists.
- Prostate; DES LH test.
LHRH analogue as leuprolide tonic LH.
- Adrenal ketoconazole, aminoglutithemide,
mitotane.
- Islet cell somatostatin
- Prolactin Bromocriptine.
Tamoxifen
partial Estrogen antagonists; antagonist on
breast, agonist on endometrium
Uses; breast cancer
Side effect; endometrial hyperplasia and
polyp formation Endometriosis, uterine
fibroids and endometrial cancers .
Aromatase inhibitors
aromatase is a cytochrome P450 enzyme, convert
testosterone into estrogen in the adipose and breast
tissue.
E.g. Anastrozole (Arimidix), Letrozole (Femara),
Exemestane (Aromasin).
in post-menopausal women, aromatization is
responsible for the majority of circulating estrogen.
Uses; post-menopausal patients with hormone-
dependent breast cancer.
Antiandrogens
E.g. Cyproterone Acetate, Flutamide and hydroxyflutamide.
Uses;
- Female
Hirsutism
PCO
precocious puberty
acne vulgaris
- Male: advanced cancer prostate.

Enzyme Inhibitors; Finasteride (Testosterone 5 reductase


inhibitor); the enzyme responsible for peripheral conversion of
testosterone into DHT. Used in BPH & hirsutism.
Acute complication of Cancer chemotherapy
1) Myelosuppression;
febrile neutropenia.
Def; fever > 38.5 in a neutropenic pt with neopl involving BM or
receiving cytotoxic agents.
Ttt;
1) imperical antibiotics acc. To C/P e.g. broad spectrum B lactam
with antipseud e.g. Ceftazidime +/- vanco +/-metro or imipinem
(anaerobes or abd) + antifungal if persistts > 7 days.
2) G-CSFs; 1-3 days after chemo.till wbc =10.000
3) Platelet transfusion till plat >10.000
4) Anemia Packed RBCs, epo. till Hb>8
2) Nausea;
- 5HT3 blocker; ondansetron (zofran)
- Dexamethazone
- Antidopaminergic phenothiazine e.g. prochlorperazine.
- Haloperidol
- Lorazepam (Ativan)
- Metoclopramidem(primperan)
- Antihistaminics; diphenhydramine.
3) Diarrhea
- lopiramide
- octreotide
4) Mucositis
- topical anathetics
5) Infertility

Pain killers in Cancer patients;


- We can replace morphin by diamorphin 1/3 dose in 24 hr (nausea, constipation
but less addiction).
- Breakthrough pain is treated by oromorphe 1/6 daily dose of morphin (oral
morphin)
- Morphin undergo extensive 1st pass metabolism.
MORPHINE EQUIVALENT DOSES
100 mg of MST has the equivalent dose of 25 ug/hr
of fentanyl over a day.
A 24 hour diamorphine dose should be 1/3 of the 24
hour morphine dose.
The equivalent dose of oral oxycodone 0.833 is to
diamorphine SC 1.3 (about 2/3).
Nephrology
Drugs in renal failure
Drugs causing systemic toxicity in RF
(GFR<10ml/min);
- Digoxin; cardiac arrhys, Ht block.
- Penicillins, cephalosporins;.... encephalopathy
- Lithium;... cardiac arrhys, seizures.
- Tienem;..... seizures.
- Ribavirin;....... hemolysis.
Nephrotoxic drugs
Prerenal ARF;
- ACEI, NSAIDS esp in edematous states.
- calcineurin inhibitors, amphotericin, contrast, cocaine.
- diuretics
Acute tubular necrosis
radiocontrast, calcineurin inhibitors, aminoglycosides, cisplatin,
amphotericin B, ethylene glycol.
Interstitial nephritis
B- lactams, sulfonamides, quinolones, rifampin, vanco,
tetracyclin, NSAIDs, diuretics
Intratubular obstruction
acyclovir, gancyclovir, methotrexate, indinavir
Nephrotic;
Membranous; gold, peniccilamine, captopril.
Minimal change; NSAIDs, INF
FSGS; INF, pamidronate, heroin.
Drugs associated with retroperitoneal
fibrosis include:
- methysergide,
- beta-adrenergic blockers,
- lysergic acid diethylamide (LSD),
- methyldopa
- pergolide,Bromocriptine

TTT; Azathioprine is used to treat


retroperitoneal fibrosis.
Drug induced SIADH
Drugs that ADH release from hypothalamus;
- Nicotine, pheno (chloropromaine) , TCA
- Carbamazipine, Clozapine, SSRI, ecstasy.
- Chlorpropamide, clofibrate,.
- Cyclophosphamide, vincristine.
- omeprazole.
Immunosuppressive therapy used in renal
transplantation
Glucocorticoids.
Small-molecule drugs;
1. Immunophillin- binding drugs;
- CNI, cyclosporin, tacrolimus
_ TOR-I; sirolimus
2. Antiproliferative
- inhibitors of nucleotide synthesis;
Purine; MMF. Azathioprine
Pyrimidine; leflunamide, Gemcitabine.
Protein drugs
1. Depleting Ab; Polyclonal, anti CD3, anti CD52, anti
CD20.
2. Non- Depleting Ab; anti CD 25, CTLA4 IG.
Agent Mechanisms Side Effects
Glucocorticoids Binds heat shock Hypertension, glucose intolerance,
proteins. Blocks dyslipidemia, osteoporosis
transcription of IL-1,-2,-
3,-6, TNF and IFN
Cyclosporine CsA CNI block IL-2 PN, Nephrotoxicity, K, Mg, HTN,
production; stimulates dyslipidemia, glucose int., hirsutism/
TGF production hyperplasia of gums, uric acid
Tacrolimus (FK506) CNI ; block IL-2 Similar to CsA, but less HTN,
production; stimulate dyslipidemia, no hirsutism/hyperplasia
TGF production of gums, but tremors & diabetes more
likely
Azathioprine Hepatic metabolites Marrow suppression (WBC > RBC >
inhibit purine synthesis platelets)
Mycophenolate Inhibits purine synthesis Diarrhea/cramps; dose-related liver
mofetil (MMF) via inosine and marrow suppression is
monophosphate uncommon
dehydrogenase
Sirolimus blocks IL-2 receptor Hyperlipidemia, thrombocytopenia
pathway for proliferation impair wound healing.
CNI
Mechanisms of action
CNIs inhibit the activity of a complex of phosphatases called calcineurin.
Calcineurin.... phosphorylates a family of proteins termed nuclear factor-
activating T cells (NFAT), thus allowing its entrance into the nucleus, where
NFAT encodes interleukin-2 (IL-2) and other cytokines .
We measure the trough (lowest therapeutic concentration) levels obtained
before the dose.
Drug interactions
Drugs That CNIs Blood Concentrations:
1. enzyme inhibitors e.g. Ketoconazole, metoclopramide, omeprazole.
2. Calcium Channel Blockers, as Diltiazem, Verapamil, but not amlodipine
or nifedipine
3. Macrolide Antibiotics.

Drugs That CNIs Blood Concentrations:


1. enzyme inducers e.g. Anticonvulsants, rifampin,
2. Magnesium & aluminum-hydroxide

NB; post-transplant lymphoproliferative disorder [PTLD] result from the


degree of immunosuppression, not specifically from the use of CNI.
steroids
Name Glucocorticoid Mineralocorticoid Duration of
potency potency action
Hydrocortisone 1 1 8
(cortisol)/
Cortisone acetate
Prednisone/ 4-5 0.8 16-36
Prednisolone
Dexamethasone 30 0 36-54
steroids
Corticosteroids act on cytosolic rather than cell membrane receptors.
Uses;
1. suppress various allergic, inflammatory, and autoimmune disorders.
2. post-transplantion immunosuppressants
NB; Prednisone is safe to use in pregnancy.

Side-effects
1. Immunosuppression
2. Hyperglycemia due to increased gluconeogenesis, insulin resistance.
3. Increased skin fragility, easy bruising
4. Reduced bone density (osteoporosis, osteonecrosis)
5. central obesity
6. Muscle breakdown , weakness;
7. salt and water retention , hypertension ,potassium depletion, and metabolic
alkalosis
8. Endocrine System; ACTH, TSH, FSH, Anovulation, irregularity of
menstrual periods, Growth failure, pubertal delay, Adrenal insufficiency
9. Gastrointestinal System; Increased gastric acid secretion
Depleting Non-depleting

Definition Antibodies that destroy T cells, B cells, monoclonal antibodies that reduce
or both. responsiveness without lymphopenia

class 1.Anti- CD3 (OKT3) -anti CD25 (Basiliximab & Daclizumab)


2.polyclonal anti-lymphocyte sera - CTLA-4-Ig fusion protein (Abatacept &
(ATGAM, Thymoglobulin) Belatacept).
3.anti- CD52.(alemtuzumab)
4. anti-CD20 (Rituximab). (B cell
depleting).
Advantag -long-lasting immunosuppression. - risks of infection and PTLD
es - lowest rates of acute rejection. -Target protein are present only in
immune cells less systemic toxicity.
-No long-lasting effects
Disadvant - cytokine release syndrome. - may not facilitate recovery from
ages -higher rates of CMV, EBV, and ischemic injury.
other viral infections. - ineffective for the treatment of acute
- Recovery from immune depletion rejection.
takes months to years and may
never be complete in older adults.
Rheumatology
TTT of rheumatoid arthritis
DMARDs (Disease- modifying antirheumatoid drugs):

2 Key Points:
start DMARDs as soon as the diagnosis of RA
is confirmed.
Methotrexate is the DMARD of choice in
patients with severe disease with relative
rapidity of action. Start therapy with 7.5mg
weekly and raise the dosage at 1- or 2-month
intervals until peak efficacy is achieved.
Methotrexate overdose;
C/P; NV, rash, GIT bleeding
The most common dose-related toxic effects of methotrexate are on the bone
marrow and gastrointestinal tractoral ulceration . Megaloblastic anaemia, usually with
marked macrocytosis
Hepatic periportal fibrosis and cirrhosis may develop without obvious signs of
hepatotoxicity,
interstitial pneumonitis. This is rare but a serious complication. Chest radiography
reveals a diffuse interstitial or mixed interstitial and alveolar infiltrate, with a
predilection for the lower lung fields.
Neurotoxicity may be seen: leukoencephalopathy, paresis, demyelination are
associated particularly with intrathecal use
Carcinogenicity ,There are reports of lymphomas associated with low-dose
methotrexate therapy for rheumatic disorders
crystal formation.1 These crystals can cause intrarenal obstruction and are a factor in
the development of acute renal failure
Methotrexate: Binding of methotrexate to dihydrofolate reductase reduces
nucleotide synthesis. Folinic acid rescue is usually given after methotrexate within
24 hours to reduce myelosuppression effects.
TTT; Ca folinate (Leucovorin).
* DMARD Therapy for RA ("GOLD PILE SCAM")
DMARD Potential Toxicities Requiring F/U Monitoring Studies
Gold IM and PO Myelosouppression, proteinuria CBC & urine dipstick for protein
Penicillamine Myelosuppression, proteinuria CBC & urine dipstick for protein
Infliximab (Remicade) Flu-like sx, auto-Abs; None
for patients not responding to methotrexate;
given IV
Leflunomide (Avara) Thrombocytopenia, hepatotxicity, diarrehea CBC and AST
Etanercept (anti TNF) Reactions at site of SQ injection, flu-like sx None
Sulfasalzine Myelosuppression CBC, AST, creatinine
Cyclophosphamide Myelosuppression, malignancy, hemorrhagic CBC, urinalysis, and urine
cystitis cytology
Cyclosporine Renal insufficiency, anemia, HTN, hirsutism creatinine, CBC, K+, LFTs
Azathioprine Myelosuppression,hepatotoxicity, CBC
lymphoproliferative disorders.
Antimalarials Macular damage Yearly fundus exams
(Hydroxycholorquine)
Methotrexate Myelosuppression, hepatic fibrosis, cirrhosis, CBC, AST, albumin
pulmonary infiltrates or fibrosis
Minocycline Photosensitivity, skin discoloration, GI upset, None
drug-induced lupus, dizziness
LEFLUNOMIDE (Arava).

Blocks pyrimidine synthesis in lymphocytes.


inhibits the proliferation of both T and B cell lines and non-immune cell lines
Uses;
-Rh. A.
- Inhibits antidonor antibody synthesis Prevent and treat acute rejection,
reverse established chronic rejection.
- has antiviral activity against herpes, CMV and polyoma virus
Well Tolerated, Not nephrotoxic
Common adverse effects seen with leflunomide are
hypertension, gastrointestinal disturbances, Hepatotoxicity , pancreatitis ,
weight loss,
Neuro., headache, dizziness, asthenia, paraesthesia, anxiety, peripheral
neuropathy.
Skin , alopecia, eczema, and dry skin , erythema multiforme ,SJS, TEN,
Hypersensitivity reactions ,synovitis .
Bl. leucopenia, anaemia, mild thrombocytopenia, eosinophilia, pancytopenia
& agranulocytosis.
Penicillamine;
Uses; DMARD Therapy for RA
Chelate cysteine in cystinuria
Chelate Cu in Wilsons
scleroderma.
Side effects; reversible loss of taste
membranous GN proteinuria.
thrombocytopenia, neutropenia
Myasthenia, drug induced lupus, SJS,
myositis.
Common side effects of gold are mouth ulceration,
leucopenia, proteinuria and skin rashes.

Hydroxychloroquine can cause renal toxicity.


Sulphasalazine causes nausea and vomiting,
leucopenia,megaloblastic an., oligospemia and deranged
liver function.
Etanercept is a dimeric fusion protein consisting of the extracellular
ligand-binding portion of the tumor necrosis factor receptor (TNFR)
linked to a human IgG1. The receptor binds to TNF, it is not a
monoclonal antibody. It is licensed for juvenile rheumatoid arthritis and
juvenile idiopathic arthritis. Side effects are septic arthritis,
demyelination, pancytopenia, aplastic anaemia and congestive heart
failure. .
Infliximab is a monoclonal anti-TNF antibody. ADVERSE
REACTIONS SIGNIFICANT
Central nervous system : Headache (18%) Fatigue , fever
Cardiovascular: Hypertension (7%)
Gastrointestinal : Nausea (21%), diarrhea , abdominal
pain, Dyspepsia
Hepatic : ALT increased (risk increased with concomitant
methotrexate)
Respiratory : Upper respiratory tract infection (32%),
sinusitis , pharyngitis , TB flare. Bronchitis , rhinitis, dyspnea (6%)
Miscellaneous: Development of antinuclear antibodies,
infection , development of antibodies to DNA, infusion
reactions. Moniliasis,Arthralgia , back pain, Rash , pruritus
symptoms modifying drugs :
Important points about COX-2 Inhibitors
Lower incidence of significant GI toxicity than
NSAIDs.
No effect on platelet aggregation.
Similar renal and hepatic effects as NSAIDs.
Contraindicated in patients with urticaria,
asthma, or other allergic-type reactions to ASA.
Can be dosed once daily for osteoarthritis.
Can be dosed without regard to food intake.
Celecoxib (Celebrex) is contraindicated in patient
with allergic-type reactions to sulfonamides.
Treatment of asymptomatic hyperuricemia;

1) Persistent s.urate >13mg/dl in men and >10 mg/dl in


women

2) Excretion of U.uric acid >1100 mg/d , Dose should be


adjusted to keep U.uric acid <800mg/d

3) Patient about to receive radio or chemotherapy that is


likely to result in extensive tumour cytolysis
Treatment of gout;
1. Treat acute flare with anti-inflammatory drugs.
2. Start urate lowing drugs weeks after the acute attack & continue till
the patient is attack free, no tophi & s urate < 6 mg/dl for 6 months.
Urate lowering drugs include;
1. Allopurinol;
- xanthine oxidase inhibitor inhibit synthesis of uric acid.
- use lower dose in renal impairment.
- interfere with metabolism of azathioprine, cyclophosphamide.
- hypersensitivity reactions.
- enzyme inhibitor.

2. Uricosuric drugs; can lead to nephrolithiasis.


not effective if serum urate is more than 2 mg/dl.

3. Uricase (Urate oxidase) IV degrade urate into allontoin, used in


tumour lysis.
HRT
Thromboembolism, breast cancer, TG,
risk of fracture, cancer colon, HDL
coronary.
Contraindications;
- Absolute;
E dependant tumours; breast, endometrium.
Thromboembolic disorder.
Abn vaginal bleeding.
- Relative;
FH of br cancer, hyper TG,
CLD, GB disease.
Pharmacologic therapy of
osteoporosis
Ca, vit D
Drugs that decrease bone resorption;
1. HRT (Euterine cancer& breast cancer).
2. SERM (selective Estrogen receptor modulators)= agonist on bone & antagonist
on breast osteoporosis, br cancer & IsHD. Side effects; flushes, DVT.
- Raloxifen (Evista)used in postmenopausal osteoporosis.
- Tamoxifen used in br cancer.
3. STEAR= selective tissue estrogenic activity regulator e.g. Tibolon (Levial )
4. Bisphosphonate (oral alendronate)
5. Calcitonin ; anabolic bone effect, Side effects; nasal congestion, rash.

Drugs that increase bone formation;


1. Synthetic PTH (Teripatide).
2. Strontium (Protelos)
3. Na floride
4. Others; GH, GHRH, IGF1, anabolic steroids, statins.
Ca, vit D;
- ca alone; small effect on BMD, no antifracture
effect.
- Vit D alone; unknown effect in pt without vit D
def.
- Ca + Vit D; may BMD, fracture.
Raloxifen (Evista )
- used in postmenopausal osteoporosis.
- adv; osteoporosis, vertebral fracture,but
not non vert fracture, br cancer & IsHD.
- Side effects; flushes, DVT.
Calcitonin; nasal spray, SC; vertebral
fracture, small BMD.
Alendronate (oral daily 10 mg or weekly 70 mg)
- mech; taken up by osteoclast apoptosis.
- adv; effective against all fractures, long term benefit (10 yrs).
- disadv; poor upper GIT tolerance.
- contraindications; hypocalcemia, osteomalacia, renal failure, bed
ridden, serious oesophageal dis.
- side effects; hypocalcemia, PTH, skin rash
oral; esophagitis, ulceration.
IV; flu like, arthralgia, myalgia, fever, leukopenia, eye inflammation
etidronate; phosphate, osteomalacia.

Teriparatide;
- daily SC, 1st anabolic, all fractures, marked & rapid BMD.
- disadv; daily SC, use restricted to high risk pts, osteosarcoma in
toxicologic studies.
- contraindication; bone dis; paget dis, Hx of bone metastasis or cancer,
growing child, hypercalcemia, pregnant, lactation.
Endocrine
Drug induced gynecomastia;
- E like action; Estrogen, digoxin, spironolactone, stilbestrol.
- Antiandrogen; cimetidine, cyproterone acetate, LHRH.
- Others; verapamil, Nifedipine

Drug induced hypothyroidism;


- amiodarone,
- Antithyroiid; carbimazole, propylthiouracil
- Lithium
- Iodine.

Carbimazole;
- Inhibit a peroxidase which catalize all phases of Th synthesis.
- Takes 6 wks to TH level.
- Agranulocytosis may occur during the 1st 4 months
- Side effects of antithyroid drugs (carbimazole & propylthyiouracil);
common= MP rash, lymphopenia.
rare=agranulocytosis, aplastic anemia, hepatitis, fever, arthralgia,
vasculitis, LN, goiter, cholestasis.
Drug induced gum hyperplasia; cyclosporin,
phenytoin, nefidipine.
Drugs causing galactorrhoea are:
- Oral contraceptive pills
- Phenothiazines such as
- Metoclopramide

- TTT : Bromocriptine

Drugs producing hypercalcemia include:


lithium
Thiazides
Antihyperglycemic Agents
Major Sites of Action

-Glucosidase inhibitors Glitazones

Plasma glucose
Carbohydrate absorption
+Glucose uptake

GI tract Muscle/Fat
Glucose production

Metformin Liver +

Insulin secretion
+
Secretagogues Pancreas
Metformin a biguanide. Its mode of action
is
delayed uptake of glucose from the GIT,
increased peripheral glucose utilization,
inhibition hepatic glucose production .
In metformin overdose, main symptoms
of toxicity include gastrointestinal upset &
severe lactic acidosis. Hypoglycaemia is
not often seen in metformin overdose.
Insulin glargine is a long-acting insulin
analogue, there is a smooth, prolonged
absorption profile with no peaks. As such, it is a
long-acting agent, suitable for providing a basal
level of insulin which mimics the normal
physiological state.
Its smooth profile reduces the risk of
hypoglycaemia, and when given at night,
provides good control of the fasting blood
glucose.
GIT
Drug induced diarrhea;
- Mg containing antacids, laxatives.
- Antibiotics.
- NSAIDs.
- Chemotherapy.

constipation;
- Ca & Al antacids, Fe, anticholinergics, opiates.
- Antidepressants, antiparkinson, anticonvulsants.
- Anti HTN; CaCBs, diuretics.
Hepatotoxic drugs
Hepatitis (ALT); cholestasis (Alk P );

INH metabolites, Rifampicin, Chlorpromazine.


HMG COA reduct inhib. Coamoxaclav.
MD Erythromycin.
Paracetamol overdose. Flucloxacillin.
Amiodarone. Sulphonylurea.
Phenytoin. OCP (E).
Valproate.
Pyrazinamide.
Nitrofurantoin.
Drug induced specific liver disease

Amiodarone, allopurinol Hepatic granuloma.


OCP, Adenomas +/- intraperitoneal rupture.
Cholelithiasis, cholestasis
Budd chiari.
Peliosis hepatis

methotrexate, amiodarone, vit A Cirr..or fibrosis


Cocaine Massive ischemic necrosis
TTT of IBD
depends upon both the extent of colonic involvement and the severity of the
disease process at presentation.
1. for ulcerative proctitis = topical 5-aminosalicylate (5-ASA) suppositories or
steroid foams.
2. Patients with left-sided colitis often respond to topical enemas.
3. Pan colitis ( extends beyond the splenic flexure).

A) mild to moderate;
- oral 5-ASA or sulfasalazine + topical 5-ASA or steroid enemas .
- add oral prednisone (40 to 60 mg/day) in:
1- more severe symptoms and
2- failed oral 5-ASA and topical therapy. ```

B) Severe active disease


- pulse IV steroid,
- IV cyclosporine
- surgery.
These agents often require 3-6 weeks to exert their maximal benefit.
Once remission is achieved, the drug dose can be tapered to maintenance levels of
sulfasalazine (2 g/day), mesalasine (1.2 to 2.4 g/day), or olsalazine (1 g/day) .
Antiinflammatory drugs in CD, UC;

1. 5ASA;
- e.g. - sulfasalazine (sulfapyridine + 5 ASA).
Side effect; oligospermia, allergic, idiosyncrasy,
agranulocytosis, rash, folate def.
- Olsalazine; 2 5-ASA.
- Enteric coated; mesalazine (Pentasa, Asacol)2-4g.
Mesalazine can cause neutropenia. & is given in
the acute attacks in Crohn's disease.

2. Steroids;
for induction only not maintenance.
Infliximab;
chimeric monoclonal antibody to TNF.
Used in;
1- refractory IBD, not fit for surgery.
2-perianal
3-RA, Psoriasis.
Dose; IV 5 mg/kg at 0, 2, 6 wks then /8 wks.
Side effects;
- Ab to infliximabe resistance & reaction so given with
methotrexate + hydrocortisone before the dose.
- lymphoma.
- allergic reactions, serum sickness
- infection esp TB
- demylenating disorders.
Drugs causing acute pancreatitis
PD FAST VET
pentamidine, didanosine,
frusemide, azathioprine, steroids, sulfa, thiazide,
valproate, Estrogen (OCP), tetracyclin

Orlistat is an inhibitor of gastrointestinal lipases,


leading to reduced fat absorption. It is licensed for
patients with BMI > 28 with associated risk factors,
a weight management programme should be in
place. Use is not recommended for more than 2
years.
Dermatology
Drug induced photosensetivity

Thiazide, tetracyclin
retinoic A.
Cipro, Griseo, amiodarone.
Retinoic A
Teratogenicity.
Hepatotoxicity.
hyperTG.
Benign increase ICT.
Dryness of mucous membrane.Alopecia.
Common drugs which can precipitate a
porphyria attack are:
Barbiturates
Sulfonamides
chloroquine
steroids
Hematology
Drug induced Megaloblastic anemia;
antifolate drugs; Alcohol, phenytoin, DHF reductase inhibitors
(methotrexate, trimethoprine, pyrimethamine), sulphasalazine.
drugs DNA synthesis (Arac, 6MP, hydroxyurea)
Drug induced hemolytic anemia;
- Immune mediated; Penicillin, Methyldopa ,INH
- Direct; Dapson, lead, benzene.
- Ribavirin in RF.
- In G6PD;
1-antimalarials; primaqine, Dapson.
2- ab, nitrofurantoin. sulfa
3- analgesic; phenazopyridine.
Others; naphtalene, methylene blue.
Drug induced Aplastic anemia; chemo, chloramph, phenylbutazone,,
carbamazepine, benzene.
Drug-induced neutropenia and agranulocytosis; the second most common
cause of neutropenia, including clozapine, the thionamides (antithyroid
drugs), and sulfasalazine .
REACTIVE THROMBOCYTOSIS; Vincristine, Epinephrine, steroids
Drug induced thrombocytopenia
Heparin
Valproic acid
Gold salts
Quinine and quinidine
Trimethoprim-sulfamethoxazole &other sulfonamides
Interferons
Penicillin
Ranitidine
Glycoprotein IIb/IIIa inhibitors (eg, abciximab)
Diuretics.
Drug induced TTP; cyclosporin,
ticlopedine, Mitomycin C,
PRINCIPLES OF (Fischer,
Hemodialysis international, 2007)
COAGULATION

- -
citrate -
anti Th III
LMWH + heparin &
- indirect thrombin
Direct thrombin inhibitors
inhibitors
Antithrombotic drugs

Antiplatelets Anticoagulants Fibrinolytics

1) Aspirin
2) anti ADP receptors Oral
Coumarin derivatives IV
(clopidogril [plavix],
- strep., urokinase,
ticlopidine)
alteplase
3) GPIIbIIIa antagonists
(Abceximab,
Tirofibam [Agrastat])
4) Anti Phosphodiesterase; Indirect
Dipyridamol [Aggrenox] Direct
thrombin inhibitors
thrombin inhibitors
Heparin
LMWH e.g. enoxaparine Lepirudin (Thrombexx)
Heparinoid e.g. Danaparoid Argatroban
Fondaparinux [Arixtra] Binalirudin
(Harrison's
PRINCIPLES OF INTERNAL MEDICINE, 2008)
Ticlopidine;
- blocks ADP receptorsreducing platelet aggregation.
- uses; 2ry stroke prevention.
unstable angina
post-coronary stent insertion.
- side effects; agranulocytosis, TTP, diarrhea, cholestasis, BM aplasia.
Clopidogril (Plavix);
- blocks ADP receptorsreducing platelet aggregation.
- less neutropenia than ticlopidine.
Abceximab;
- irreversible bind GPIIb IIIa receptors (-) plat aggregation.
- used in; unstable angina
NSTMI
post-coronary stent insertion.
Drugs which decrease mortality in MI;
1) Aspirin (not dypyridamol, does not rec in 1 yr).
2) thrombolytics/ angioplasty.
3) BB (FU with PR < 0.24, HR >45, SBP > 100)3 doses of 5 mg
metoprolol IV > 2 min interval then 100 mg 1x2 forever esp
compensated CHF.
4) ACEI esp. CHF, EF< 40%, cardiomegaly.
5) Statin ( mortality & recurrence) even with average
cholesterol prior to discharge.
NB: - nitrate morbidity not mortality.
- LV function is the prognosticator post MI.
- DHP CCBs inc.CVS risk after MI.
- In type II DM, must be kept on insulin infusion for 24 hs then SC
for 3 months mortality.
TTT of HT failure;
1) VD;
NB; ACEI mortality & hospitalization.
of no benefit in RVF.
- obstructive valve lesion.
- in HCM.
2) Digoxin- renin, sympathetic activity, hospitalization but not
mortality.
- used in Syst HF, S3, severe CHF.
- AVB, asymptomatic HF, acute MI.

3) Diuretics e.g. spironolactone mortality & hospitalization &


symptoms. used in severe CHF.

4) BB ( Meto, Biso, Carvi) mortality & hospitalization & symptoms.


not used in volume overloaded patients.
Antiarrhysmic drugs:
Class effect site use
Class I A Quinidine (- )Na channels A WPW+ AF
Disopyramide V
procainamide
acc
B Lignocaine (- )Na channels V VT
Phenytoin
Mexiletin
C Flecainide (- )Na channels A AF
propafenone V
acc
Class II Propranolol BB nodes Stress, thyroid,
Atenolol sinus, cong long
QT, VT
Class III Amiodarone (- )Na, Ca, K channels A AF, A flutter
Sotalol plateau acc AVRT+ wpw
Ibutilide V VT
Class IV verapamil (- )Ca channels AV node SVT, MAT

Others Adenosine K opener, no(-) inotrop. AV node SVT, diag of


digoxin flutter
Adenosine
Is a purine nucleotide.
Acts via adenosine receptors activate K channels
Short acting.
IV, 6-12 mg
Uses; (diag diff bet VT & SVT, therapeutic SVT)
Enhanced by dipyridamole& inhibited by theophyllin.
Side effect; chest pain, bronchospasm, flushing,
hypotension.
NB; no prophylactic use of lignocaine after MI mortality.
Amiodarone
Mech; prolong the refractory period QT, it is the least ve inotropic
antiarrhysmic.
Uses; prolong life span in pt with recurrent VT, HOCM.
39% of its wt is iodine, structurally related to thyroid Hormone.
Side effects.
most common after oral neurotoxicity,
most common after IV hypotension.
Lung toxicity (1-10%) IPF, ARDS, BOOP, pl effusion, radiodense
plaques, bil patchy infiltrate diagnosed by CT, ttt; stop the drug & give
steroids.
others; hepatitis, thyroiditis, optic neuritis, corneal opacities,
photosensitivity,
PR, QRS, QT esp with 1a.
in Torsade de points.
Drug interaction; digoxin & anticoag level
Effect of Amiodarone on thyroid hormone;
- initial TH;
* dt inhibitory effect of iodine on T4 release (wolf-Chaikoff )
* then dt;
* iodine loadtype1 (jod basedow), ttt with carbimazole
* thyroiditis (type2)CRP, ttt with steroids.
Digoxin;
- mech; (-) AVN.
+ve inotropic effect.
- Used in; Ht failure + AF or AF only.
- Kinetics; 85% renal elimination, 25% bound to PP, narrow therapeutic
window; Dose
- Contraindicated in;
AVB, WPW, MAT, prior to elective cardioversion, constrictive
pericarditis, acute myocarditis, HCM.
- Drug interaction; digoxin level with amiodarone (displace for tissue
binding sites) & CaCB & quinidine (interfere with renal tubular excretion).
- ECG; reversed stick ST depression is an indication of digitalis therapy
not toxicity, but pulsus bigemini is an early sign of digitalis toxicity.
1. Arrhysmogenic;
- most common is Ht block.
- characteristic arrhys is non paroxismal atrial tachy. with varing block.
2. Other symptoms; anorexia, nausea, vomiting, diarrhea, yellow vision,
gynecomastia, yellow vision.
3. Precipitating factors; K, Mg, Ca, aging, hypoxia, hypothyroidism,
renal failure.
4. TTT; digibind (Ab to digoxin its clearance).
Flecainide;
- Class 1c for ttt of Ventricular arrhys., .
- medical cardioversion for atrial aarhys.
- Avoided in ; post MI, LV dysfunction
- Side effects; vertigo, visual disturbance.

Nicorandil;
- Antianginal (possesses nitrate component)
- K channel opener arterial & venous VD
- Side effects; headache, flushing, dizziness, hypotension,
tachycardia, oral ulcers.
Drugs that prolong QT;
1. antiarrhyth (Ia, III),
2. antibiotics ( macrolids, quinolones,
quinine, sutrim)
3. antifungal,
4. pentamidine
Antihypertensives
Drug Class Examples Other Indications Contraindications/Cautions

Diuretics
Thiazides Hydrochlorothiazide Diabetes, dyslipidemia, hyperuricemia, gout,
hypokalemia
Chlorthalidone
Loop diuretics Furosemide CHF, renal failure Diabetes, dyslipidemia, hyperuricemia, gout,
hypokalemia
Ethacrynic acid
Aldosterone Spironolactone CHF, primary Renal failure, hyperkalemia
antagonists aldosteronism
Eplerenone
K +retaining Amiloride Renal failure, hyperkalemia

Triamterene
Beta blockers Asthma, COPD, 2nd or 3rd degree heart block, sick-sinus
syndrome
Cardioselective Atenolol Angina, CHF, post-MI,
sinus tachycardia,
Metoprolol
ventricular
Nonselective Propranolol tachyarrhythmias
Propranolol LA
Combined Labetalol ?Post-MI, CHF
alpha/beta
Carvedilol
Alpha
antagonists
Selective Prazosin Prostatism
Doxazosin
Terazosin
Nonselective Phenoxybenzamine Pheochromocytoma
Drug Class Examples Other Indications Contraindications/Cautions
Sympatholytics
Central Clonidine

Clonidine patch

Methyldopa

Reserpine

Guanfacine

ACE inhibitors Captopril Post-MI, CHF, nephropathy Renal failure, bilateral renal artery stenosis,
pregnancy, hyperkalemia
Lisinopril

Ramipril

Angiotensin II Losartan CHF, diabetic nephropathy, Renal failure, bilateral renal artery stenosis,
antagonists ACE inhibitor cough pregnancy, hyperkalemia
Valsartan

Candesartan

Calcium Heart failure, 2d or 3d degree heart block


antagonists
Dihydropyridines Nifedipine (long acting( Angina

Nondihydropyridin Verapamil (long acting( Post-MI, supraventricular


es tachycardias, angina

Diltiazem(long-acting(

Direct Hydralazine Severe coronary artery disease


vasodilators
Minoxidil
Thiazides block Na+ and Cl- reabsorption in the distal tubule. The
distal convoluted tubule accounts for 5% of total sodium chloride
reabsorption.
Thiazides elevate LDL cholesterol, reduce urinary calcium excretion
and can cause impotence,thrombocytopaenia, hypokalaemia and
hyperuricaemia.
Hypercalciuria can be treated with thiazides.
Thiazides can cause

Frusemide acts on the thick portion of the ascending loop of Henle.


It inhibits Na+ and Cl- reabsorption there via Na+, K+, -ATPase-
dependent pump. Owing to the large NaCl absorptive capacity of the
loop of Henle, agents that act at this site produce a diuretic effect
much greater than that seen with other diuretic groups.
Control of systemic hypertension.

Target =
140/90 in HTN alone
130/80 mm Hg in patients with DM or CKD &
proteinuria < 1 g/24 hrs.
125/75 mm Hg in patients with CKD & proteinuria > 1
g/24 hrs.
ACE inhibitors/ARBs appear to be more protective
than other antihypertensive drugs in patients excreting
> 500 to 1000 mg of protein per day .
ACE inhibitors/ARBs may not be of greater benefit
than other antihypertensive agents in patients without
proteinuria and only mildly reduced kidney function.
ACEI & ARBs
Target; > 60 % reduction, optimally to total protein excretion < 1 g/24
hrs.
No limitations;
- Even with normal BP, antiproteinuric measures should be applied
guided by BP.
- not limited by the initial level of GFR.
Limitations for ACEI;
Hyperkalemia, pregnancy, bilateral RAS
angioedema, cough.
Follow up ; with BP, S. creat., S. K;
at day 4 then
weekly up to 3 months then
monthly if high risk or every 3 months if low risk, stable dose.
Initial rise of s. creat up to 30% that stabilize in 2 months.
Stop if rise of s. creat > 30 % or s. K > 5.5 meq/dl.
Benefit of RAAS blockade
Control BP
( Syst BP by 10-15 mmHg & diastolic BP by 5-10 mmHg).
Antiproteinuric effect
(by 40%, Combination therapy with ACE inhibitors and
ARBs additive antiproteinuric effect; 18 to 25 % greater
reduction in proteinuria) due to;
- GFR.
- alter mesangial function.
Renoprotective;
(reduce the progression from 1 ml/min/month to 0.3
ml/min/month)
- bradykinin & other antifibrotic agents.
- TGF-B.
- Ang II mediated free Oxygen radicals.
ARBs
- No cough (do not inhibit bradykinin)
- Less hyperkalemia.

Verapamil is contraindicated in;


- Digoxin toxicity
- SSS, WPW, AVB
- With BB
- Hypotension, Ht failure.
Medicati
ons of
Hyperli
pidemia
HMG Co A reductases (Statins)
Mech;
- (-) HMGCo A reductase (rate limiting
enzyme in cholesterol synthesis).
- LDL receptors LDL
- HDL
Side effects;
- fatigue, myalgia, myopathy, rabdomyolysis.
- Stop statin if CPK > 10 times, or AST >3
times.
Ezetimibe acts by prevent cholesterol
absorption from the small intestine.
Typically it reduces LDL-cholesterol by
approximately 20%, triglycerides by up to
5% and raises HDL-cholesterol by
approximately 5%.
Ezetimibe is currently licensed for use in
combination with a statin in patients who
fail to reach desired lipid profiles or as
monotherapy in patients intolerant to a
statin.
Poisonings & Intoxications

Hemodialysis;
- Alcohol (methanol,ethylene glycol, alcohol)
- Lithium,
- salicylate,
- theophylline,
- metformin
- barbiturate (phenobarbital).

Not dialysable;
- Highly protein bound; digoxin, phenytoin
- Large volume of distribution as amiodarone, organoph.
Methanol
- solvents, varnish ,de-icing solutions
N/V, abdominal pain (pancreatitis),
CNS symptoms, seizures, coma, blurred vision (papilledema),
increase osmolar gap, metabolic acidosis (increase anion gap).
Treatment:
- empty stomach contents
- Ethanol/fomepizole (inhibit ETOH DeH)
- Hemodialysis: severe metabolic acidosis, very high osmolar gap,
methanol
& visual changes (disc hyperemia).
Ethylene Glycol
- Antifreeze ,solvents
- not protein bound
- 30-60 ml is lethal
- NB; Cause Na & osmolality & osm gap+ high AG met acidosis.
- EG glycolic acid glyoxylic acid oxalic acid.
- Stage 1 (<12 hrs): n/v, seizure, hyporeflexia, coma, high osmolar gap,
hypertension, noncardiogenic pulmonary oedema, metabolic acidosis.
- Stage 2 (>24 hrs): flank pain, oliguria, renal failure, ca oxalate crystals.
- Treatment:
- Empty stomach contents
- Ethanol/fomepizole (competitive inhibition of alcohol dehydrogenase)
- Thiamine 100 mg IM qid, B6 50 mg IM qid ; (glyoxalate glycine)
- Hemodialysis, (severe metabolic acidosis, very high osmolar gap)
Salicylate
- C/P: headache, tinnitus, confusion, sweating, n/v,
- hyperventilation, resp alkalosis then metabolic acidosis, cardiovascular
collapse, hypoglycemia, PT.
Treatment;
- Gastric emptying , activated charcoal even 4 hr after dt delayed
gastric empting.
- Hemodialysis:
seizure, coma
severe metabolic acidosis + renal failure
Theophylline
Toxic level > 15 mg/L
C/P:
GIT , N/V, abdominal pain, diarrhea,
CNS , tremors, agitation, confusion, seizure,
CVS , arrhythmia
Pptating factors ,K dt vomiting.
Treatment:
- Gastric lavage, activated charcoal
- - Extracorporeal therapy:
Hemoperfusion better than HD
Ectasy (Amphetamine)

Features of acute (ecstasy) toxicity include


agitation, tachycardia, hypertension, dilated
pupils, sweating, hyperthermia,(sympathetic)
(DIC) , rhabdomyolysis and acute renal failure.
TTT; forced acid diuresis with NH4 Cl.
Paracetamol overdose
Self poisoning 10-15 gm = 20-30 tab.
Patho; In normal condition, paracetamol is conjugated with glucuronic acid
in overdose it is conjugated with glutathione&bind to liver macromolecules> necrosis.
C/P; NV, later hepatic (peak after 12 days) & renal failure.
Lab; measure serum level after 4 hr of ingestion.
INR most sensitive for liver damage.
hyoglycemia> severe damage.
Poor prognostic criteria;
- INR >3
- raise s creat
- Ph <7.3 more than 24 hr after (lactic acidosis dt hepatic lactate metabolism).
Pt at higher risk for hepatic necrosis;
- taking enz inducers; phenytoin, chronic alcohol.
- malnurished, HIV.
TTT;
- 1 st hr activated charcoal
- 2 hr oral ipecac & gastric lavage
- then wait for Bl level after 4 hrs ; if high N acetylcysteine IV, given earlier for
high risk patients
Organophosphate poisoning
mech; cholinesterase inhibitor Ach.
C/P; parasymp; miotic pupil, bradycardia,
salivation ,ANVD. GI ulceration.
TTT; pralidoxime.
CO poisoning
Bind to Hb Hb O2 carrying capacity tissue anoxia.
Hx; a family found in the morning in their bed dead, pink in colour.
C/O; mild headacke, NV, dyspnea.
severe; hypertonia, fever, rabdomyolysis, ARF, confusion,
coma.
Lab; PO2, normal O2 sat.
Level of carboxy Hb;
non smoker; <3%
Smoker; 5-6%
TTT; 100% O2 by mask, hyperbaric O2
Headache is the most common symptom (90%) followed by
nausea & vomitting, vertigo, alteration in consciousness and
weakness. The cherry red skin colour occurs when COHb
concentration exceeds 20% but it is rarely seen in life. Pulse
oximetry gives falsely high oxygen saturation
Met-Hb
Causes; either by a genetic defect in red cell
metabolism or acquired by Dapsone, water
contaminated with nitrate, local anathetic prilocaine,
antimalarials, sulphonamides, aniline dyes in dyed
blankets, and cleaning solution, NO gases
C/P=cyanosis, without resp sympt, dizziness,
headache, coma.
Invest; ABG shows normal PO2=n, but Standard
pulse oximeters give spuriously low readings in the
presence of excess methaemoglobin.
TTT= if <20% no ttt, methelene blue is indicated
in any patient with (mental changes, tachycardia,
dyspnoea, chest pain).
Lead poisoning;

- Anemia (MCV, basophilic stippling, hemolytic


in acute toxicity)
- Encephalopathy, demylinating PN.
- Proximal RTA, abd pain, constipation
- Blue line on gums.
Chronic cocaine use can result in erectile
dysfunction, ejaculatory dysfunction,
excitement, emotional instability, restlessness,
irritability, tremors, twitching of small muscles
Severe anxiety and paranoid hallucinations,
mania, and psychosis .
Specific Antidote Therapy

- Acetaminophen (N-acetyl cysteine)


- Atropine (physostigmine)
- Nitrites, Met Hb (methylene blue)
- Digoxin (Fab)
- Carbon monoxide (oxygen)
- Organophosphates (atropine)
- Cyanide (amyl nitrite)
- Methtrexate (leucovorin)
Methanol, ethylene glycol ethanol (reduce their toxic
metabolites.
- B blocker glucagon
PREGNANCY
Trimethoprim is a folate antagonist and can increases the risk of
neural tube defects. There is relative contraindication for ciprofloxacin
in pregnancy due to the possible teratogenic effect. Augmentin,
cefaclor, nitrofurantoin and metronidazole are safe in pregnancy.
Carbimazole crosses the placenta and can cause nail/finger
abnormalities.
Propylthiouracil is commonly used in pregnancy instead of
carbimazole due to the risks of neonatal hypothyroidism with
carbimazole.
Warfarin in the first trimester can cause fetal hypoplasia of the nose
and limbs. After this period warfarin is associated with neurological
damage mental retardation, microcephaly, optic atrophy and
blindness. There is an option to convert from heparin to warfarin in
the third trimester, but the patient will have to be re-converted back to
heparin before delivery.
questions
mode of action of docetaxel
prevent microtuble
drug causes of cholastatic picture
flucloxacillin,other option was parcetamol,tramdol
s/e of progesteron :
option inculde:nausea,breast pain,headache
nephrogenic DI asking about drug causing it and answer was:lithum
drug causes constipation and option were:metformin,glagazid and other???
mention hyop glycemia and hypotension and hyponatremia,which is best to give
hydrocortison
pt is not controled on glgazid and has renal impairment
extenide,other were metformin
drug causes of gynecomastia:option amidaron,pheothiazine...?!!!
excessof cortisol where will it go?
bind 2 albumin
bind to fat
others.....
mechansim of alloprinol
machansim of imatinib
vomiting from ca what other you add to ondansetron:dexamethone,metochropromide
vt what is contra indicating:verapamil
Bosentan mode of action
dog bite: coamoxiclave
pt with ethenol poisining and asking about the mechansim by which inhibation
of alchol dehydrogens is done by fomepizole
which drug can be givin with finsteride
doxazin
nitrate
nicorandil
ACEinhibitor
drug which cause pancytopenia/aplastic an
trimethoprin
drug lead to LN and wt gain??
phenytoin
the prognosis 26 hr after paracetamol poisoning?
Inotropic support for patient with TSS (from tampons) -
dobutamine/noradrenaline/adrenaline
Patient with SBP, treatment? - vanco/cipro/3d gen cephalosporin
Pt on MTX, has been taking 40mg instead of 10mg per week. Now has ulcers,
signs/symptoms of MTX overdose, treatment? - folinic acid
51) Pt had TURP done for BPH, now has urge incontinence and nocturia,
treatment? - amitryptyline/can't remember options
Oral treatment of S.aureus ulcer, with a list of resistance/sensitivities -
chlramphenical/vancomycin/ciprofloxacin (to test if you know that certain drugs
do not have oral formulations)
Pt with AF on amiodarone and developed hyperthyroidism, stopped
amiodarone, started carbimazole to no effect, isotope scan shows low uptake,
treatment? - prednisolone
Drug induced lupus - minocycline
Pt on dapsone, with methhaemoglobinemia, treatment? - methylene blue
Sickle cell pt on morphine for bone infarction, overdosed, reversed with naloxone, screams
in pain, treatment? - IM diclofenac/IM haloperidol/IM pethidine
8 soon starting chemo patient having agitation / suicidal thoughts ? steroids psychosis
patient on lithium need to start HTN medication
. patient on bendro still having high bp had side effects of fluid
retention / gum bleeding /lehtargy / with other HTN medication what next ? beta blocker
.inr decrease after starting tb medicine ? rifam
clopigogrel adp inhibitor
obese lady failed sulphonylurea low gfr what next ? exenatide
high cholesterol with raised tsh what treatment ? thyroxine
46 . od of 80 tab thyroxine 24 hr later treatment ? beta blocker
cellulitis showing MRSA what should be added along with vanco
73. cipro causing tendon damage
withdrwal symptoms give chlordiazepoxide
124. wernicke / delerium tremens
125. phenytoin levels not decreaing in renal failure patient why

126. acetylar status which drug hydralazine


patient on bendrofluthiazide having low k whats the mechanism
patient agitated / mettalic taste after stopping her pshychiatric medications
raised cholestrol ,ldl,triglycerides tx ....... atrorva
Subacute IE. treatment? Benpen + gent
2. litium toxicity. precipitant.
3. chronic CML, (?not candodate for imintab). a-interferon
4. Discoid lupus on steroids. next treatment. hydroxychloroquine.
jaw stiffness with multiple injected sites with discharging sinus tx? metronidazole /vac
Carbamazepine autoinduction
16. Respiratory depression in an overdose--Diazepam ??
Rate control in AF in a heart failure patient already on Digoxin---Amiodarone
Hypertension in Pregnancy -- Methyldopa
Beta Blocker Toxicity with very low blood sugar and bradycardia non-responsive to
atropine -- Give Glucagon
Pt took cocaine, what else can be found -- Ischaemic Cardiac pain
54. Coronary Vasospam--give Calcium Channel Blocker
Patient with SIADH -- Fluoxetine
1. Lithium toxicity ---Concomittant use of ACE-Inhibitor
64. Rheumatooid Arthritis patient alread on Diclofenac Sodium,what should be started
next Methotrexate
2. Lithium toxicity ---Concomittant use of ACE-Inhibitor --/increas toxicity
3. recently had chemotherapy now has neuropathy ? cause /vincristine
4. carbamazemine autoinduction
3. valporate hair loss
4. cyclosporin excessive hair
5.patient suffered peripheral neuropathy , had chemo whic medication to stop ? vincristine
patient on 5 Anti HTN medications
develops ankle edema amlodipine/
doxazocin / minoxidil
Patient on lithium HTN medication
made levels high ACE
CML treatment Imatinib
Dermatology
Scabies
Molluscum contagiosum caused by a DNA pox virus.
The lesions are Discrete, solid, small, skin coloured
papules -central umbilication, not surrounded by
erythema & they may be spread following scratching
to other sites. TTT: curetage, cryo, liquid nitrogen,
trichloroacetate.

(DD chicken pox), Chickenpox lesions in the early


stages may be mistaken as molluscum. However,
presence of associated macules with later vesicles &
pustules help to differentiate them. These lesions also
affect mucus membranes, usually disappear within
a few weeks, while molluscum can persist up to a
year.
Sister Mary Josephs
Nodule
Psoriasis is associated with a dermopathy &
arthropathy which may be range from mild
distal Interphalangeal joint involvement & nail
pitting to severe Arthritis Mutilans.
A Koebner Phenomenon refers to outbreak of
a skin eruption following minor trauma which
is a feature of psoriasis.
Psoriatic arthropathy may be associated with
an anterior uveitis.
Chloroquine may produce a severe attack of
psoriasis
Psoriasis
Psoriasis
T cell.
Prevalence 0.5-4.5%.

Types;

1. Plaque 80% most common.


2. Pustular
3. Guttate.
4. Erythrodermic.
5. Nail psoriasis; onycholysis, pitting, subungual hyperkeratosis.

Associated signs;
1. Koebner.
2. Auspitz.
3. Pitting.
Types;
Plaque psoriasis
Guttate psoriasis
Nail psoriasis
Pustular psoriasis
Erythrodermic psoriasis
Psoriasis
Associated; arthropathy, gout, IBD
Exacerbating factors;
1. Trauma,stress
2. Infection; strept., HIV
3. Endocrine; improve during pregnancy,
increase during post partum period.
4. Drugs; BB, lithium, antimalarials, steroid
withdrawal , NSAIDs, INF.
Psoriasis TTT;
Topical;
tar, dithranol, vit D, weak steroids for face
genitalia & flexures.

Systemic
retinoids (not monotherapy, used with UV),
methotrexate, ciclosporin, aza, hydroxyurea, MMF.
UV radiation; PUVA, UVB narrow band

Biologic; efalizumab, Alefacept, Etanercept,


infleximab.
Exacerbating factors

Several drugs are associated with worsening


psoriasis. The most common offenders are beta
blockers, lithium, and antimalarial drugs;ACEI ,
NSAIDs, and terbinafine .

Infections, both bacterial and viral, also may be


associated with worsening of psoriasis.
poststreptococcal flares of guttate psoriasis and the
onset or worsening of psoriasis in association with
HIV infection are well recognized.

The prevalence of psoriasis is increased among


patients who abuse alcohol .
Urticaria is a common & usually responds very
well to systemic antihistamines as first line ttt.
Oral steroids .... in severe cases .
topical steroids/ topical antihistamines have no
effect.
Type Description Mechanism Clinical features

I Anaphylactic, Antigen exposure causes Anaphylaxis


immediate-type release of vasoactive Angioedema
Immediate reaction substances such as
hypersensitivity Bronchospasm
(30-60 min) histamine, prostaglandins,
and leukotrienes from mast Urticaria (hives)
Accelerated reaction cells or basophils. This
response is usually but not
(1-72 hrs
always IgE-dependent.

II Antibody-dependent An antigen or hapten that is Hemolytic anemia


cytotoxicity intimately associated with a Interstitial nephritis
cell binds to antibody, leading
to cell or tissue injury

III Immune complex disease Damage is caused by Serum sickness


formation or deposition of
antigen-antibody complexes
in vessels or tissue

IV Cell-mediated or delayed Antigen exposure sensitizes T Contact dermatitis


hypersensitivity cells, which then mediate
tissue injury
A patient an allergy bee stings likely
develop a type I reaction a bee sting
therefore anaphylactic shock.
Bullous disease DIFFERENTIAL
DIAGNOSIS
Cong; epidermolysis bullosa.
Immune; pemphigus V, pemphigoid,
dermatitis herpitiform, herpes gestationis.
Drugs; barbiturate.
Others; SSSS, TEN, diabetic bullae.
Nicolesky sign
Pemphigus
Autoimmune bullous diseases
Bullous pemphigoid Pemphigous vulgaris

benign May be life threatening

IgG at BM IgG at intercellular


substance
Tense bullae Flaccid bullae

subepidermal intraepidermal

TTT; less more


immunosuppressive
Potential causes of erythema
multiforme include:
INFECTIONS:
viruses: herpes simplex 1 2, EBV, enteroviruses.
bacteria: Group A Streptococcus , mycoplasma
pneumoniae.
other: TB , histoplasma, coccidioides.
NEOPLASIA:
leukaemia
lymphoma.
ANTIBIOTICS:
penicillins, sulphonamides, isoniazid, tetracycline.
ANTICONVULSANTS:
Phenytoin , phenobarbitone, carbamazepine , NSAIDs.
Steven Johnson S
SJS & TEN mucocut. drug induced or
idiopathic erythema then desquamation with
prominent bullae but no target lesion.
Erythema multiformSJS (<10% BSA epidermal
detachment) TEN (>30%).
Drugs; sulfa, allopurinol, carbamazepine,
phenylbutazone , piroxicam.
Mortality 5% in SJS.
TEN
TEN is a severe mucocutaneous exfoliative disease
of an uncertain pathogenesis & a high mortality rate.
It is difficult to say whether it is another variant of
Stevens-Johnson Syndrome. treatment of both are
similar. It is often idiopathic but may be associated
with viral infections, leukaemia, lymphoma & drugs -
in particular Sulphonamides , anticonvulsants &
allopurinol.
Usually caused by drugs, toxic epidermal necrolysis
begins with widespread erythema; confluent vesicular
and necrotic areas of the arm and trunk then lead to
peeling away of the skin.
TTT ; IVIG ,antibiotics , mortality 30%.
Contact dermatitis
asteatotic eczema
asteatotic eczema which a common
problem improve just by plain emollients.
dry, pruritic rash affecting upper back
shins.
Seborrheic Dermatitis and Otitis
Externa
Hereditary angioneurotic
edema.
2 ry to C1 esterase deficiency.
Non painfull, nonpruritic, non erythematous.
No urticaria.
Inherited FH , adolescent , AD.
Symptoms:
Mococutaneous .:lips, eyes, penis,Glottal& laryngeal oedema.
Abd pain ( intestinal wall).
mild trauma flare.
TTT: attenuated androgens or danazol hepatic synthesis
of C1-INH & C4.
Acquired; excessive utilization in malignancy or anti C1-
INH Ab or drugs; ACEI, opiates, NSAIDS.
Pytriasis Rosea
Begin as a herald patch,on the trunk, after
1-2 wks generalized exanthematous
eruption, resolve spontaneous after 6 wks.
Spring & fall.
May be due to papova virus.
Most patients do not require therapy. For
patients with mild itching treatment with
moderate potency topical corticosteroids.
Acne rosacea
A chronic acneform disorder due to increased
reactivity to heat flushing & telangectasia.
It begins as erythema (flushing and redness) &
telangiectasia, red domed papule and pustules on
the central face mainly the forehead, the chin and
the lower half of the nose & progress to a red
lobulated nose (rhinophyma) .
Rosacea
Ppt factors; alcohol, caffeine, spicy, hot.
No comedones.
40-50 years old.
F>M except rhinophyma.
TTT: topical metronidazole or topical antibiotics, oral
TCN, PO isotretinoin.(steroid contraindicated worse)
Complication in males; rhinophyma=bullous red nose.
e.g. Rhinophyma
"end stage" of sebaceous
overgrowth and scarring from
poorly controlled acne rosacea .
Older age.
Differential diagnosis of flushing
Autonomic-mediated
Thermoregulatory flushing
Fever
Exercise
Heat exposure (environmental or ingestion)
Menopause
Emotional flushing
Neurologic
CNS tumor.
Cluster headache, Migraine.
Multiple sclerosis
Trigeminal neuralgia .

Vasodilator-mediated
Rosacea .
Medications (ie, calcium channel blockers, nicotinic acid)
Food ingestion .
Alcohol .
Serotonin syndrome .
Anaphylaxis .
Carcinoid syndrome .
Systemic mastocytosis .
Pheochromocytoma .
Medullary thyroid carcinoma .
Hyperthyroidism..
Osler-Weber-Rendu syndrome=
hereditary hemorrhagic telangiectasia
AD
triad of; telangiectasia, recurrent epistaxis, and a positive
family history for the disorder. presenting at any age.
multiorgan arteriovenous malformations (AVMs).
C/P; asymptomatic or have multiple organ involvement,
- left-to-right shunting, high-output cardiac failure, CV
stroke.
- Most commonly, telangiectases involve the mucous
membranes, skin, conjunctiva, retina, GI tract, lungs &
brain.
MRI scanning is the best noninvasive test to delineate the
extent of pulmonary and CNS AVMs.
Angiography is used to map the exact extent of the
vascular lesions.
Treatment controlling the bleeding.
Drug eruptions

Urticaria.
Morbilliform .
Fixed drug eruption.
Erythema multiforme.
Stevens-Johnson Syndrome.
Toxic epidermal necrolysis .
Angioedema/anaphylaxis.
Hypersensitivity syndrome .
Fixed drug eruption
SCC
Actinic keratosis
Actinic keratosis (also
called solar keratosis) is a
premalignant condition of
thick, scaly, or crusty
patches of skin.
Basal cell carcinoma
The most common type of skin cancer.
Rolled in borders.
Nodular or superficial type.
Locally malignant.
Risk factors:
Sun exposure.
Arsenic exposure.
BCC
Malignant melanoma
Most important prognostic factor is tumour thickness.
5 signs of malignant melanoma:
Asymmetry.
Border is irregular.
Color mottled.
Diameter > 6 mm
Enlargement.
Risk factors;
FH of melanoma.
cong. Melanocytic nevi.
white skin, red hair, freckling, severe sunburns.
Superficial spreading.
Nodular.
Lentigo.

Kaposi sarcoma
Kaposi Sarcoma
Vascular neoplasia any organ, mc. Violaceous .
HHV-8 in HIV
TTT radiotherapy+/-chemo + antiretroviral ttt.
Cutaneous T call lymphoma
= Mycosis fungoids.
Malignant helper cell (CD4).
HTLV 1.
Severe pruritis, sun exposed areas, plaques, nodules,
LN, large ant. Mediastinal mass.
Mimic psoriasis (SCALY).
EXTENSIVE INFILTRATION OF THE FACE
LEONINE.
Esinophilia, high LDH.
Male: female 27:1.
Ttt; Puva, topical nitrogen mustard, total body irradiation,
topical & systemic retinoid.
Internal malignancy
Acanthosis nigricans. Tripe palm
Pruritis.
Dermatomyositis.
Erythema gyratum repens.
Acquired hypertrichosis lanuginosa.
Necrolytic migratory erythema.
Migratory thrombophlebitis.
Acquired ichthyosis.
Pyoderma gangrenosa.
Migratory thrombophlebitis
(underlying malignancy)
Tripe palms
cutaneous
paraneoplastic AN,
(90%) of the cases are
associated with internal
neoplasm. (gastric
adenocarcinoma,
bladder, bronchus,
rectum).
release of epidermal
velvety thickening of palms, with
growth factors (EGF) by accentuation of the normal
dermatoglyphic ridges and sulci
the tumor. (exaggerated finger prints) .
ICHTHYOSIS

Non Hodgkin lymphoma


Erythema gyratum repens
Associated with cancer in 80% of cases
Most commonly associated with Lung Cancer
& breast
Onset may precedes cancer diagnosis by 9
months
Resolves within 1 week of cancer treatment
Erythema nodosum characterised by painful, indurated, shiny,
red, hot, elevated nodules 1-3cm diameter particularly on
shins. There may be associated fever, malaise, arthralgia.
Over days become violaceous, then dull purple then fade like a
large bruise without residual ulceration or scar.

Causes include:
INFECTIONS:
bacteria: strep pharyngitis ,Streptococci, leptospirosis, psittacosis,
yersinia. viruses: EBV.
OTHER:
TB, histoplasmosis, coccidiodomycosis.
DRUGS:
sulphonamides, oral contraceptive pill.
SYSTEMIC DISEASES:
SLE, vasculitis, regional enteritis, ulcerative colitis, Behet Syndrome.
Lofgrens S. sarcoidosis. Hodgkin. Pregnancy
Idiopathic 25%.
Dermatologic manifestations of
HCV infection
Lichen planus .
Acral necrolytic erythema.
sicca syndrome (Sjogren Syndrome).
Leukocytoclastic reactions .
Cryoglobulinemia .
Antiphospholipid syndrome.
Vitiligo .
Porphyria cutanea tarda.
Pruritus .
Acral necrolytic erythema
PCT
Porphyria cutanea tarda (PCT) is the most common of the
porphyrias in North America and Europe. Exposure of
patients with PCT to sunlight results in increased skin fragility,
vesicles, bullae, hypertrichosis, hyperpigmentation and
scarring in a photodistribution.
Adult
c/o fragile skin,bullae, vesicle on dorsum of hand.
Hyperpigmentation, hypertrichosis, bullae, crusts, scars on
face &hand.
Confirmed by orange red fluoresence under woods lamp.
Cause: ethanol, estrogen, Fe, HCV, HIV.
Gradual onset.
Increase plasma Fe in 50%.
TTT; stop offending agent, phlebotomy, chloroquine .
Porphyria cutanea tarda
sicca syndrome
(Sjogren Syndrome)
vitiligo
vitiligo is associated with numerous autoimmune
conditions including in order of frequency,
autoimmune hypothyroidism,
pernicious anemia,
alopecia areata.
Addison's disease.
Gastrointestinal
Liver cirrhosis: White nails, Terrys nail,
Muehrcke's lines, palmar erythema.
Underlying cause;
Dupuytrens contracture------ Alcoholic L. C.
PCT------HCV
Clubbing: primary biliary cirrhosis
Inflammatory B.D Clubbing.
Pyoderma gangrenosum.
Celiac disease--------- Dermatitis herpetiform
Migratory thrombophlebitis------- Malignancy
Malabsorption:
Acrodermatitis Enteropathica (Zinc deficiency)
koilonychia, Long. striation, brittle----Fe def.
Colonic Bilh. polyposis-------- Clubbing.
palmar erythema
Idiopathic
Cirrhosis
alcohol
Pregnancy
Connective tissue
disorders:
Particularly rheumatoid
arthritis ,SLE.
Thyrotoxicosis
Polycythaemia.
Dupuytren's
contracture
Dupuytren's contracture is
thickening of the palmar fascia
which may eventually pull the
fingers into the palm .
Dupuytren's disease is
alcoholic liver cirrhosis
familial,
vascular disease,
diabetes .
Hypoalbuminemia
Dermatitis herpetiformis
occurring in up to 24 % of patients with celiac d.
characterized by an itchy papular vesicular
eruption usually located symmetrically on the
extensor surfaces of the elbows, knees,
buttocks, and occasionally within the mouth.
The predominant symptoms are itching and
burning .
Similar to celiac disease, anti-tTG antibodies
are elevated in patients with dermatitis
herpetiformis .
TTT : medications such as dapsone,
complete resolution of the skin lesions occur
with gluten withdrawal. .
Dermatitis herpitiformis

dermatitis herpetiformis. DH is one of the immunobullous


conditions characteristically by very intensely pruritic vesicles.
It is not usually responsive to topical steroids, but would
respond well to dapsone. It associated with gluten sensitivity
coeliac disease .
pyoderma gangrenosum.
Peutz-Jeghers S
AD.
GI hamartomatous polyp , (low malignant
ptential 2%).
MC pigmentation (blue/black macules
mouth, lips, buccal mucosa.
New onset DM, Diarrhea ,DVT of lt Lower limb.
Deep vein thrombosis .
N N anemia , Weight loss. (WARD)
glossitis, angular cheilitis, stomatitis, and
blepharitis.
Necrolytic migratory erythema NME ,
characteristically begins as erythematous
papules or plaques involving the face,
perineum, and extremities . the lesions
enlarge and coalesce. The affected areas
are often pruritic and painful.
NME, the characteristic skin lesion of the
glucagonoma syndrome, is often the clue
which leads to the correct diagnosis.
Black hairy tongue.
Black tongue
In rheumatological diseases
Nails: Pitting / Oncholysis (Psoriasis)
splinter Hge------------Vasculitis
Nail fold erythema & telangectasia----- DM
infarcts-------------vasculitis
Joint swelling (DIP, PIP, MCP, Wrist) :
Heberden's nodes/ Bouchard's nodes------OA
rheumatoid nodules.
Gouty tophi
Deformity : Ulnar deviation, Z-thumb, Swan-neck, Boutonnire
(RA,PA);
squaring of hand (OA)
Wasting of small muscles of hand.
Skin lesions: Gottron's papules-----------Dermato M
Pyoderma gangrenosum-----Behset
Palm: Dupytrens, palmar erythema, DeQuervain tenosynovitis,
vasculitis, keratoderma blennorragica (Reiter)
elbows psoriatic plaques, rheumatoid nodules, gouty tophi.
lupus vasculitis
Palpable purpura
1. Vasculitis
Hypersensitivity (HSP)
Septic
Cryo.
Wegner, PAN, Kawasaki.
Discoid Lupus
plaque on nose with follicular plugging-
scarring
red scaly plaques on her cheeks, forehead
sides of neck. On close inspection of lesions
there was plugging of some hair follicles .
Lesions are discrete plaques, often
erythematous, covered by scale & These
lesions are typically occur on face, scalp,
behind ears on neck. They can exist in areas
exposed to sun. Central atrophic scarring is
characteristic.
Scleroderma
Systemic sclerosis (finger plup ulcer & scars)
Systemic sclerosis - Calcinosis
Finger tips & pulp
Raynaud's phenomenon
Acrogangrene - Raynaud's
Rheumatoid deformities

Swan-neck

Boutonnir
e

Rheumatoid nodules.
Elbow
Rheumatoi
d nodule
overlying
the
olecranon
of the right
arm
Livedo reticularis
a web-like pattern of
reddish-blue
discoloration,
mostly involving the
legs, occurs in
autoimmune
vasculitis, especially
in SLE and
antiphospholipid
syndrome, &
cholesterol emboli.

mechanics hand
mechanic's hand seems to be a
distinctive feature of the
Polymysitis.

The anti-synthetase antibodies


e.g. anti-Jo-1 (-tRNA synthetases ) characterized by scaling and hyperkeratosis
predicts interstitial lung of the distal skin pad and lateral aspect of
pathology, which is its most the fingers.
relevant internal manifestation. PM-SCL autoantibody positive scleroderma
& are specific for a distinctive with dermatomyositis (scleromyositis)
syndrome characterized by
myositis
non-erosive arthritis,
interstitial lung disease,
skin changes (Mechanic's hand
in 70% of patients, Raynaud's
phenomenon)
fever.
Dermatomyositis
Nail fold erythema & telangectasia
1. rheumatoid arthritis,
2. SLE,
3. dermatomyositis,
4. scleroderma,

irregular, twisted, and


dilated vessels giving an
injected, erythematous
appearance
Strawberry tongue Kawasaki
disease
Signs of inflammation
e.g. Relapsing polychondritis
Pseudoxanthoma elasticum
-Multiple, discrete as well as
coalescent, skin colored to
yellowish, indurated,
papular lesions ("plucked
chicken skin")
-occur in flexural regions ,
most commonly the neck
and axillae.
--characterized by
dermal,
ocular (angioid streaks )and
vascular lesions
(gastrointestinal bleeding ,
premature atherosclerosis
and hypertension).
Gouty tophi
Gouty tophi of the index finger
Suspicious skin lesions on the pinna
e.g. Gouty tophi
Nodal osteoarthritis
(Heberden's nodes).
Nodules

rheumatoid nodules

Gouty tophi
1. Keratoderma blenorrhagicum: vesicles
& crust in palm & soles in Reiter S +
conjunctivitis, urethritis, arthritis, circinate
balanitis.
Diabetes Mellitus
Acanthosis nigricans
Diffuse velvety thickening &
hyperpigmentation of the flexural skin.
5 types;
1. Hereditary benign.
2. Benign; insulin resistant states (DM,
acromegaly, cushing, Addison.
3. Pseudo; obesity.
4. Drug induced; OCP, nicotinic acid.
5. Malignant; Gastric adenocarcinomas.
Diabetic dermopathy
Granuloma
annulare
pale, shiny rings and
nodules usually seen
on the hands in DM.
Granuloma annulare
Granuloma annulare.
Diabetes:

Fungal nail infections

granuloma annulare

Diabetic cheiroarthropathy
Dupuytren's contracture
Necrobiosis L D
familial partial lipodystrophy
The lipodystrophic syndromes are a
heterogeneous group of congenital or
acquired disorders characterized by either
complete or partial lack of adipose tissue
(lipoatrophy) . The extent of fat loss correlates
with the severity of the metabolic
abnormalities. Clinically, patients with severe
lipodystrophy have severe insulin resistance
and a group of unique features, such as
MPGN type 2, severe hyperlipidemia,
progressive liver disease, and increased
metabolic rate .
Endocrinal disorders
Acromegaly --------------Spade hand, Carpal tunnel S
Graves' disease: finger-clubbing (thyroid acropachy),
thyrotoxicosis of any cause: Palmar erythema, fine
tremor, warm, sweaty.
hypothyroidism Dryness & thickness & yellow pig. due
to carotenemia.
Addison's Palmar pigmentation, brown nails --------
increased ACTH with ; vitiligo.
Cushing: cigarette paper like shining of the skin,
ecchymosis.
diabetes: Fungal nail infections, Diabetic
cheiroarthropathy, Dupuytren's contracture, carpal
tunnel syndrome, granuloma annulare, Vitiligo with type
1 diabetes, Acanthosis nigricans in type II .
tendon xanthomata in familial hypercholesterolaemia.
eruptive xanthomata in hypertriglyceridaemia.
Pretibial Myxedema

Pretibial myxedema is characteristically associated


with Graves disease.Patients with pretibial
myxedema may be hypothyroid , hyperthyroid or
euthyroid when this skin disorder occurs. Pretibial
myxedema occurs in about 3-5% of patients with
Graves disease .The majority of patients have
associated exophthalmos. Clinically.sharply
demarcated,edematous,indurated plaques over the
pretibial areas .
How is pretibial myxedema treated?
-. Treatment of the thyroid disease doesnot
affect the cutaneous findings
-some cases may resolve spontaneously
-many cases respond to potent topical
corticosteroids or better to intralesional
corticosteroids
-More extensive cases may be treated with
oral systemic corticosteroids
These are the shins of a 73 year old female who
is admitted as a consequence of increasing
confusion and inability to look after herself.
Erythema Abe Igni
The diagnosis is erythema ab igne and
this is due to sitting too close to a fire. It
frequently occurs on the front of the shins
or lower back, the latter especially
associated with the use of a hot water
bottle. In this patient case the confusion
and coldness with erythema ab igne
suggest a diagnosis of hypothyroidism.
Albright Hereditary
Osteodystrophy
Albright Hereditary
Osteodystrophy is characterised
by short stature, round face, short
neck, short 4th & 5 th ,metacarbals
and obesity.
May be associated with
pseudohypoparathyroidism
Brittle nails,

Alopecia
Dry skin
Coarse hair
What is the diagnosis of the patient
whose hand on the right and suffers
hyponatremia

Slide no 10

Normal hand
DD of Hyperpigmentation
Addison's disease
Cigarette paper like skin
(Cushing syndrome)
Xanthomas
Xanthomas are firm, raised waxy-
appearing papules (bumps) which may
occur on the trunk, arms, and legs. The
lesions may be skin-colored, pink or even
yellow. The presence of this type of skin
lesion may be associated with abnormal
levels of lipids (fats) in the blood.
Xanthomas
xanthomas may be the first sign of one of the hyperlipoproteinemias.
1. xanthelasma are xanthomas of eyelids that may or may not be
associated with hyperlipidemia (Familial hypercholestrolemia=type 2a).
2. tendon xanthomas
extensor tendons of fingers, patella, elbows, Achilles tendon (one of the
most common sites); diffuse infiltration of tendon by lipid
hypercholesterolemia; Types II and III
3. tuberous xanthomas
lipid deposits in the dermis and subcutis; papuler, nodular or plaques;
extensor surfaces of large joints, heels (type III)
4. Eruptivesmall yellow papules; buttocks, posterior thighs
, usuallyincrease in serum triglyceride levels (type 1)
5. Plane xanthomas--flat yellow plaques on palms, face,
seen in biliary cirrhosis, familial type III.
NB: xanthelasma may not be associated with hyperlipidemia
but eruptive xanthomas, tendon xanthomas, and tuberous
xanthomas are signs of significant hyperlipidemia; these patients
require careful evaluation and prompt treatment
1ry hyperlipidemia

combined hypertrigliceridemia hypercholestrolemia

Type IIb Type IIa


Type 4
(xanthelasma & tendon)

Type III Type I


(tuberous, palmar) Eruptive, pancreatitis, IHD
lipemia retinal
xanthelasma
Swellings
Tendon xanthoma
familial
hypercholesterolae
mia
Achilles tendon,
tibial tuberosities
and the extensor
tendons of the
hands, over the
knuckles ,.
felt as hard
nodules along the
length of the
tendon and quite
easy to miss.
Tendon xanthoma
Tendon xanthoma
tuberous
xanthomas
Eruptiv
e
Plane
xanthomas-
Cutaneous manifestations in
infectious diseases
2 Toxic shock syndrome
Staph aureus toxin mediated (TSST-1,
enterotoxin).
Fever, hypotension, generalized skin &
mm erythema, diarrhea, MOF.
In menstrual & non menstrual patterns,
barrier contraception, puerpurium, septic
abortion, surgery, burns, insect bite.
TTT; staph IV nafcillin or oxacillin.
strept pen. G + clinda IV
Toxic shock syndrome (streptococcal
toxins ----- injury to the skin).
3 Ampicillin + EBV or CMV
Morbilliform rash, viral like exanthematous
drug eruption , mimic measles.
5- erysipelas
Cellulitis + raised sharp demarcated
borders, group A strept (more commonly),
& staph. Less commonly
Shin of tibia & face.
7- Scarlet fever
Desquamation & stawberry tongue.
Group A strep.
DD; Kawasaki.
Norwegian scabies
Scabies is a highly contagious disease caused
by mite, Sarcoptes Scabiei. Infestation is
common, found worldwide, affects people of
all races & low social classes
. Infestation is easily spread with sexual
partners , household members. Infestation
may also occur by sharing clothing, towels,
bedding. Scabies is characterised by papular-
like irritations, burrows rash of skin, especially
webbing between fingers; skin folds on wrist,
elbow.
treatment includes Permethrin cream , topical
Benzyl Benzoate & malathion, oral ivermectin
in resistant cases. Antihistamines , calamine
lotion may be used to alleviate itching.
Tinea capitus
is a dermatophyte infection of the scalp
caused by trichophyton tonsurans, occasionally by microsporum
canis.
scaly circular plaque (ringworm) with broken infected hairs
(exclamation mark hairs).
pruritis.
a severe inflammatory response produces an elevated boggy
granulomatous mass (kerion), studded with sterile pustules.
Fever, regional LN, scarring alopecia.
The crusted patches fluoresce dull green under Wood's light.
Microscopic examination of a KOH preparation shows tiny spores of
fungi .
Oral griseofulvin 2-3 months is required, Ketoconazole in
resistant cases.
Pytriasis versicolor
Chronic asymtomatic scaling rash.
Pityriasis versicolor (also called tinea versicolor)
a skin lesion cause by a fungus called
Malassezia furfur.
Well demarcated scaling patch with variable
hyperpigmentation on the trunk.
Diagnosis; KOH prep. Hyfal form of
pytriosporum oval.
TTT; antifungal as ketoconazole or Oral
Itraconazol or topical azole creams, topical
selenium sulfide.
Hypopigmented areas
Vitiligo
(hypo, hyper thyroidism, addisson or,
SLE, Alopecia areata)
Pityriasis versicolor.
Tuberculoid leprosy.
Ash leaf of tuberous sclerosis.
Morphea.
Occupational infectious
diseases

Human ORF (virus)-------sheep


handlers
fish tank granuloma (MB marinum)-------
fishing
Sporotrichosis (fungal)-------------
gardener
Human ORF
fish tank granuloma
fish tank granuloma
Sporotrichosis
Ulceronodular dermatosis.
Sporotrhrix schenckii (soil fungus)
Gardner, farmer, florist.
+ chronic nodular lymphangitis & LN.
Sporotrichosis
4- Impetigo
Crusted golden yellow erosions.
On nose & cheek, lips, chin.
Staph aureus, group A strept.
ECM
Erythema chronicum migrans
Rapid expanding ring with clearing middle
in trunk & axilla.
Lyme disease.
Borrelia burgdorferi transmitted by tick
bite.
Prodome; malaise, stiff neck, arthralgia.
Coxsackie
(Hand, Foot and Mouth disease)
Cat scratch disease
Cat scratch disease
Bartonella henselae, as bacillary
angiomatosis.
Benign self limiting.
1ry skin then tender LN

Bacillary angiomatosis; vascular papules


or nodules, bartonella henselae&
quantana, transmitted by cats & ticks,
TTT: erythromycin or Doxycycline.
Scrofuloderma

Extension into the skin from underlying mycobacterial infection, most


commonly of cervical lymph nodes .
firm, painless, subcutaneous nodules that gradually enlarge and
suppurate and then form ulcers and sinus tracts in overlying skin.
Typical ulcers have undermined edges and a floor of granulation
tissue.
GVHD
Desquamation

Causes
Kawasaki
Contact dermatitis
Toxic epidermal necrolysis
GVHD
Streptococcal infection /pharyngitis /Scarlet fever/ Toxic shock
syndrome (streptococcal toxins ----- injury to the skin).
AIDS dermatology
Kaposi sarcoma.
Herpes Zoster.
Oral hairy leukoplakia; EBV in advanced
AIDS.
DD; Oral leukoplakia; HPV, smoking,
smokless tobaco, alcohol, syphilis, can be
invaded by candida, premalignant.
Molluscum contagiosum
Bacillary angiomatosis.
Renal
Slide no 6
This female patient sufferred convuslions in childhood.
She experienced several attacks of hematuria recenly.
Proteinuria was less thn 1gm/24hrs and her RFTs
were normal.
1. What is her condition.
2. What is the most likely lesion of her CT scan
Tuberous sclerosis
Periungual fibroma
Angiokeratoma
This is a patient with end stage renal disease, with
severe uncontrolled hyperparathyroidism. What is
the skin lesion

Slide no 9
Image quiz
NSF
Nephrogenic systemic fibrosis (NSF) or Nephrogenic
fibrosing dermopathy is a rare and serious syndrome that
involves fibrosis of skin, joints, eyes, and internal organs.
Its cause is not fully understood, but it seems to be
associated with exposure to gadolinium (which is
frequently used as a contrast substance for MRIs) in
patients with severe kidney failure.
Four of the five FDA-approved gadolinium contrast
agents have been principally implicated in NSF, including
Omniscan, Multihance , Magnevist, and OptiMARK.
No cases were reported by use of ProHance (the fifth
one).
What is this disease in a patient with abdominal
pain, hematuria, and renal impairment and a
palpable rash on thighs

What is the
Slide no 11
most common
glomerular
lesion on light
microscopy of
this patient
when a biopsy
is taken
Chest disease
Respiratory failure: cyanosis, Flapping
tremors, warm.
Clubbing------non small cell lung cancer,
Mesothelioma, Interstitial lung disease,
Suppurative lung disease, Pulmonary
hypertension
COPD: Brown nails due to nicotine staining.
Sarcoidosis--- dactylitis, digital cysts,
nodules
Wegners granulomatosis---nodules
Yellow nails ------bronckiectasis, pleural
effusion
Tar
staining
Sarcoidosis
Sarcoidosis
affecting the ear
lobule , with a
prominent purple
color. Lymphomas
can produce very
similar lesions at this
site.
Lupus pernio
Lupus pernio
Lupus pernio, a
chronic variant of
sarcoidosis which
typically affects the
nose and central
face, is usually very
purple, and may be
destructive locally.
This type of lesion
may also become
verrucous, but can
respond well to low-
dose methotrexate.
Cardiovascular cases
Clubbing---- Congenital cyanotic heart disease, SBE,
Atrial myxoma
Signs of infective endocarditis: Oslers nodes,
Janeway spots, splinter hemorrhage.
CHF -------- Red lunnula, Pitting oedema, cold,
cyanosed or pale.
AI ----- Capillary pulsations & WHP , arachnodactyly
in Marfan
IsHD ---------Beau lines, Mees lines, tendon
xanthomas.
Vascular diseases; Burger's dis, Raynaulds
Cholesterol emboli; livedo reticularis
Signs of infective
endocarditis
Splinter hemorrhages
longitudinal red or
brown thin lines,
beneath the nail plate.
when capillaries
within the epidermal
ridges leak.
1. local trauma,
2. fungal infection,
3. endocarditis.
Janeway lesion in infective
endocarditis
Oslers nodes
Burger's dis
venous ulcerations are the most common type
of ulcer affecting lower extremities. The
probable underlying cause of venous
congestion, which may promote ulceration,
venous insufficiency. The treatment of venous
ulceration are control of oedema, treating any
infection, compression. However, compressive
dressings should be applied if arterial
circulation is impaired. It is thus important to
identify any arterial disease, ankle-brachial
pressure index is a simple way of doing this.
One may then progress to lower limb
arteriogram if indicated.
Acrodermatitis Enteropathica
Zinc deficiency can lead to acrodermatitis
which presents with perioral dermatitis
with acral involvement & sometimes
alopecia.

The diarrhea bowel resection suggest zinc


deficiency
1ry systemic amyloidosis
Pink ,Pinch ,purpura, in upper eye lid
following straining due to deposition of
amyloid in bl vs.
Papules
RACCONS EYES
NF
NF1; lish nodules; pigmented hamartomas
of Iris. chr 17, AD.
NF2; bilateral acoustic neuromas , chr 22,
AD.
Both; >6 caf au lait patches &
neurofibromas.
individual must have at least two of following
features. Some people NF1 have only two, while
others can have several of these features:

Six more cafe-au-lait spots, coffee-colored birthmarks,


each measuring over an inch in adults (1/4 inch in
children).
Two of more neurofibromas or a plexiform neurofibroma.
Freckles under arm & in groin region.
A tumor of nerve the eye called an optic glioma.
Two or more spots on iris of eye called Lisch
nodules.
A problem of one of bones such as bowing of a leg,
without a fracture.
A parent, brother, sister, child with NF1( FH).
Bone metastasis Paget disease of bone Pulmonary osteodystrophy
Non scaring alopecia
Androgenic baldness.
A. areata.

Endocrine; hypopituitarism, hypo & hyper


thyroidism, hypo para., pregnancy.

Drugs; retinoids, anticoagulants,antimitotics,OCP,


carbimazole, thiouracil, Lithium, valproate.
Fe def.
Chronic illness.
Scaring alopecia

Bacterial;TB, S.
Physical injury; burns, radiotherapy.
Fungal (favus, kerion).
Discoid lupus.
Licken planus.
Scleroderma; morphea.
Cicatricial pemphigoid.
DD of pruritis
Nail examination

distal matrix ------- deeper layers of the nail plate-----ridging


or splitting.
proximal matrix ---------- superficial layers---- superficial nail
problems (e.g., pitting).
Hypoplasia
(Nail-Patella Syndrome)
brittle nails
Low levels of zinc
iron deficiency
thyroid problems
aging.
Pitting

punctate depressions in the nail plate.


caused by defective layering of the superficial nail plate by the proximal nail
matrix.
Psoriasis(10-50 %), Reiter's syndrome, alopecia areata
Longitudinal Linear Lesions
Longitudinal striations

accentuated ridges in the nail surface as a


normal part of the aging process.
If nails become thin and lusterless , the
condition may be referred to as trachyonychia;
in patients with vitiligo.
Koilonychia
Clubbing

CLUBBING is a thickening of the soft tissue beneath the


proximal nail plate that results in sponginess of the
proximal plate.
caused by hypervascularity and the opening of anastomotic
channels in the nailbed.
may result from megakaryocytes and platelet clumps that
have escaped filtration in the pulmonary bed and have
entered the systemic circulation. Platelets then may release
platelet-derived growth factor at the nail bed, causing
periosteal changes.
Degrees of Clubbing

1st degree: obliteration of


the normal diamond-
shaped space at the
proximal end of the nail
when the distal phalanges
are opposed (Schamroth
window sign).
2nd degree: Parrot peak
3rd degree: drum stick
4th degree: HOA (X ray
finding)
Causes of clubbing
familial .
secondary :
Lung disease:
Suppurative lung disease: lung abscess, empyema, bronchiectasis, cystic fibrosis
Interstitial lung disease esp. cryptogenic fibrosing alveolitis, asbestosis.
non small cell bronkogenic carcinoma, Mesothelioma (HOA )
Pulmonary hypertension

Heart disease:
Congenital cyanotic heart disease
Subacute bacterial endocarditis
Atrial myxoma

Gastrointestinal and hepatobiliary:


Crohn's disease and ulcerative colitis
Cirrhosis, especially in primary biliary cirrhosis

Skin disease - Pachydermoperiostosis, palmoplantar keratoderma.

Malignancies - Thyroid, thymus, Hodgkin, CML, POEMS syndrome with plasma cell dyscrasia

Endocrinal - Acromegaly, thyroid acropachy, pregnancy, severe secondary hyperparathyroidism

Others: Vascular anomalies of the affected arm (unilateral clubbing)


Transverse Linear Lesions
Beaus line
They develop in response
to many diseases, such as
chronic paronychia
zinc deficiency,
syphilis,
uncontrolled DM,
myocarditis,
PVD,
chemotherapy.
Mees lines

In patients with Mees' lines, the nail bed is normal, but the nail
itself is microscopically fragmented, probably because of the
disruption of normal growth at the nail matrix during the insult.
The width of the lines varies and, because the defect is in the
nail itself, the line moves distally with time.
Onycolysis
Systemic diseases
Anemia (iron deficient) ,
Bronchiectasis , Pleural effusion ,
Hyperthyroidism , Pellagra ,
peripheral ischemia , Neuritis
Leprosy , Syphilis ,
SLE , Scleroderma , Psoriatic arthritis
Dermatologic diseases
Psoriasis , Lichen planus , PCT, Pemphigus, _
Mechanical
Chemical Drug induced
Microbial: Dermatophytosis, Candida, Bacteria
(Pseudomonas),Virus (HS)
Color changes
Leukonychia
Totalis,
Partialis; Terrys nails, Muehrcke's lines , True' Leukonychia.

Half-and-half nails
Yellow nail syndrome
brown nails
Blue nails; cyanosis
Leukonychia Totalis
Terrys nails

80% of patients severe liver disease, usually cirrhosis.


25 % of hospitalized patients with varied diseases
The condition is thought to be caused by a decrease in vascularity and an
increase in connective tissue in the nail bed .
Pairs of transverse white lines that extend all the way across the nail.
hypoalbuminemic states ( less than 2 g per dL) and disappear when the protein
level normalizes.
abnormality of the vascular nail bed
disappear while the nail is depressed
does not move with nail growth.
These characteristics distinguish Muehrcke's lines from Mees' lines.
'True' Leukonychia.
Children and active adults
commonly have one or more
white lines or spots on one or
more nails; this condition is
known as leukonychia .
These lines and spots are
nonuniform, appear in
different places on different
nails, do not span the nail, and
are of no significance. They are
thought to result from random
minor trauma to the proximal
nail bed.
They should not be confused with
Mees' and Muehrcke's lines,
which parallel the lunula
across the entire nail bed and
occur in more than one nail.
Half-and-half nails
in patients with
chronic renal failure,
increased melanin
production may cause
the distal part of the
nail bed to turn brown.
In patients with severe
renal disease, the
proximal portion of
the nail bed can turn
white, obliterating the
lunula and giving a
half-brown, half-white
appearance, also called
half-and-half nails
Yellow nail syndrome

Patients note that nail growth slows and appears to


stop. The nail plate may become excessively curved
Changes in the color of the lunula
Wilson's disease------- blue (azure lunula).
Silver poisoning will turn the nail itself a blue-
gray color.
Heart failure ---------- red,
tetracycline ------------ yellow.
Excessive fluoride ingestion can turn nails
brown or black.
Diabetes Mellitus
Diagnosis of DM
normal IFG IGT DM Goal
of ttt in DM
Fasting <110 mg/dL 110-125 mg/dL >126 mg/dL 80-120
<6 m mol/ L 6-7 m mol/ L >7 mmol/ L
RBS <140 140-199 >200 <180
or 2h PP <7.8 >11.1
after a
75-g
OGTT
Bed time 7.8-11.1 100-140

IGT Predicts increased risk of diabetes and cardiovascular disease)


Classification of DM
Type 1 (10%) -cell destructioncomplete lack of insulin

Type 2 (85%) -cell dysfunction and insulin resistance

Gestational -cell dysfunction and insulin


resistance during pregnancy, 50 %DM in
10yrs.

Other specific types

American Diabetes Association: Position Statement, Diagnosis and Classification of Diabetes Mellitus.
Diabetes Care 32:S62-S67, 2009
VI. Other specific types
C. Diseases of the exocrine
A. Genetic defects of b-cell function pancreas
1. Chromosome 12, HNF-1 (MODY3) 1. Pancreatitis
2. Chromosome 7, glucokinase (MODY2)2. Trauma/pancreatectomy
3. Chromosome 20, HNF-4 (MODY1) 3. Cystic fibrosis
5. Hemochromatosis
4. Chromosome 13, insulin promoter
factor-1 (IPF-1; MODY4) D. Endocrinopathies
5. Chromosome 17, HNF-1 (MODY5) 1. Acromegaly
6. Chromosome 2, NeuroD1 (MODY6) 2. Cushings syndrome
7. Mitochondrial DNA. 3. Glucagonoma
4. Pheochromocytoma
5. Hyperthyroidism
B. Genetic defects in insulin action 6. Somatostatinoma
1. Type A insulin resistance 8. Others
2. Lipodystrophy S
VI. Other specific types
E. Drug- or chemical-induced G. Uncommon forms of
1.Calcinurin inhibitors ( cyclo, Tac) immune-mediated diabetes
2. Glucocorticoids 1. Antiinsulin receptor antibodies
3. Nicotinic acid
4. Pentamidine H. Other genetic syndromes
6. Diazoxide sometimes associated with
7. -blocker diabetes
8. Thiazides 1. Downs syndrome
9. Interferon 2. Klinefelters syndrome
10.phenytoin, clozapine 3. Turners syndrome
13. somatostanin 4. Friedreichs ataxia
6. Myotonic dystrophy
F. Infections 7. Laurence moon beadle
1. Congenital rubella 8. Prader willi.
2. Cytomegalovirus
3. coxsakie
Maturity onset diabetes of youth
(MODY)
Type 2 DM in patients under the age of 25 ys.
AD & strong family history.
Single gene defect defect insulin secretion.
Treated by oral hypoglycemics.

Latent autoimmune DM of the adult (adult, not obese,


autoimmune, require insulin).
NB; so diagnosis of new onset DM may require
assessment of;
- C peptide, if low need insulin,
- & auto Ab (ICA, GAD) need insulin.
Type 1 type2

genetics Both parents=10-20% 70-100%


Identical twins 30-50% 90%
Father DM=6%
Mother DM 3%
risk On chr 6, HLA DR3/4, DQ8 No HLA, K channel
subunit KCNJ11
protective HLA DR 2

autoab 60-90 % ICA, GAD ----

prevalence 1/1000 1/100

age <30 yrs > 30 yrs

Associated Other autoimmune, wt loss, Metabolic S, obesity,


ketosis, insulin def acanthosis, amyloid
deposit in Islet cells
ttt Insulin all
Gestational Diabetes
Hyperglycemia during pregnancy usually resolves
after birth
Complicates ~4% of all pregnancies.
High risk of perinatal morbidity and mortality
High risk of later type 2 diabetes in both mother and
baby
Diabetic mother high risk of fetal malformation.
Screening for GDM at 24-28 wks if >35 yrs, obese, FH.
Oral HypoGlycemics are #.
Glucose cross the placenta macrosomia, but
insulin does not cross.
Who Should Be Tested for Diabetes?
Consider if One or More of the Following
Apply
1. Symptoms suggesting diabetes: weight loss,
hunger, urinary frequency, blurred vision
2. Age >45 (>30 if patient has other risk factors)
3. family history of diabetes
4. Prior gestational diabetes
5. Women with polycystic ovarian syndrome (PCOS)
Management of DM
1- patient education; self monitoring, insulin administration,
hypos, foot care.
2-nutrition
3-glycemic control
glycemic control
Assessment of glycemic control;
1- serial measurement of Blood glucose (SMBG)( 3 times
preprandial, 1 at bedtime)
2- Hb A1C (assess nocturnal & postprandial), reflects BG in the
last 3 months.
A1C 6% = mean BG 135 mg/dl, 7.8 mmol/l .
1% 35 mg/dl
7% 170 mg/dl, 9.5 mmol/l
8% 205 mg/dl. 11.5mmol/l.
3-fructosamine (glycated Alb); used in hemoglobinopathies,
reflects BG in the last 2-3 weeks.

NB; Hb A1C, normal SMBG nocturnal & postprandial,


SMBG, normal Hb A1C intercurrent infection or
illness, hemolysis.
-Glucosidase inhibitor
(Acarbose)
Pioglitazone,
Repaglinide rosiglitazone
nateglinide
Antihyperglycemic Agents
Major Sites of Action

-Glucosidase inhibitors Glitazones

Plasma glucose
Carbohydrate absorption +Glucose uptake

GI tract Muscle/Fat
Glucose production

Metformin Liver +

Insulin secretion
+
Secretagogues Pancreas
Insulin Secretagogues
Sulfonylureas
Mechanism of action Increase basal and/or postprandial
insulin secretion
Efficacy depends upon Functioning -cells
Dosing Sulfonylureas: 1 or 2 times daily

Side effects Hypoglycemia, Weight gain, allergy (rare)


CVS mortality, cholestasis,
agranulocytosis, drug interaction ( Hajat wehsha).
Contraindications; Renal, hepatic, old
Selected Sufonylureas
Dosing Information
Recommended Usual
Generic Name Dose Strengths Dose Range Maximal
Effect

Glimepiride 1, 2,3 or 4 mg 18 mg 4 mg qd
(Amaryl)

Glibenclamide 1.25, 2.5, or 5 mg 1.2520 mg 5 or 10 mg bid


(Doanil)

Gliclazide MR 30 mg 30120 mg 60 or 120 mg

Gliclazide 40 or 80 mg 40360 mg 80 or 160 mg bid

(Diamicron MR & Diamicron)


Insulin Secretagogues
Repaglinide (Novonorm 0.5, 1, 2 mg)
repaglide 1 mg
Mechanism of action Increase postprandial insulin
secretion ( fast phase of insulin release)
Efficacy depends upon Functioning -cells
Dosing Repaglinide: (Novonorm 0.5, 1, 2 mg)
3 or 4 times daily with meals
Side effects Weight gain, allergy (rare)
Main risk Hypoglycemia
Biguanides
Metformin (Glucophage, Cidophage, 500,
850, 1000mg)
Primary mechanism Decreases hepatic glucose
of action production (insulin
resistance at the liver)
Efficacy depends upon Presence of insulin
Dosing 2 or 3 times daily
adv; wt loss
Side effects Diarrhea, nausea
Main risk Lactic acidosis
Contraindications; s creat >1.5, 1.4 , radiocontrast, serious
illness, acidosis, CHF. 1-2 days after surgery.
Glitazones (TZDs)
Pioglitazone (Glusitin, Actos 15, 30, 45 mg)
Rosiglitazone (Avandia 2, 4, 8 mg)

Mechanism of action Enhance tissue response to insulin, +


PPAR- Glut 4 (insulin
resistance at ms & adipose tissue
Efficacy depends upon Presence of insulin and resistance
to its action
Dosing Once daily
Side effects Edema, weight gain, anemia, macular
edema, fractures, resumption of ovulation
Main risk Congestive heart failure
Contraindications; CHF, hepatic
-Glucosidase Inhibitors
Acarbose (Glucobay 50, 100mg)
Mechanism of action slow glucose absorption.
Efficacy depends upon Postprandial hyperglycemia
Dosing Acarbose (Glucobay 50, 100mg)
3 times daily, with meals
Side effects Flatulence
Main risk Liver enzyme elevation (rare)
GLP-1-based therapies
GLP-1 insulin release, glucagon
glycogenolysis & gluconeogenesis, gastric
empting ( postprandial hyperglycemia but GIT
upset).
1. GLP-1 agonist (Incretin analogues)(SC).

1. Dipeptidyl peptidase IV inhibitors (DPP-IV I )


catabolism of endogenous GLP-1 (Incretin
enhancers ) (oral).
GLP-1 agonist
(Exenatide, Liraglutide)
Mechanism of action GLP-1analogue insulin release, glucagon,
gastric empting, appetite, B cell mass, as adjuvant in combination in
type2.
Efficacy depends upon Functioning -cells
Power Decrease A1C 0.5%1%
Dosing Exenatide sc injection twice daily, Liraglutide once daily.
Adv; wt loss, CVS risk, HTN
Disadv; injection, not used with insulin, hypoglycemia with secretagogues.
Side effects GIT upset

Contraindications; renal, agents which GI motility.


DPP-IV INHIBITORS
(Sitagliptin, Vildagliptin)
Mechanism of action endogenous GLP-1 insulin release,
glucagon, gastric empting. as monotherapy or in
combination in type2.
Efficacy depends upon Functioning -cells
Dosing Sitagliptin (Januvia) 25 mg, 50 mg, 100 mg once daily
Adv; no hypoglycemia
Disadv; no appetite, no wt loss, no CVS risk, dose in CKD.
Side effects GIT upset
Amylin agonist
(pramlintide)
Mechanism of action Amylin is a 37-amino acid peptide co-secreted with insulin
glucagon, gastric empting, appetite. Amylin is deficient in type 1
diabetes and relatively deficient in type 2 diabetes.
Adv; wt loss, postprandial hyperglycemia,decrease insulin dose by 20% used
in CKD, moderate dose if GFR<20ml/min
Disadv; injection, only as adjuvant with insulin in type 1 & 2. cannot be mixed in a
syringe with insulin. Mild to moderate nausea is the most commonly reported
side effect and generally dissipates by four weeks.
Dosing it requires injections with each meal in the setting of type 1 diabetes, and
at least twice daily in type 2 diabetes.
Side effect Contraindications; agents which GI motility.
Reasons for insulin use in type 2
diabetes
1. Hyperglycemia despite maximum dose of
oral agents; A1c levels >7%, FPG > 7 mmol/l)
2. FPG > 11.1 mmol/l from the start
3. Decompensation due to inter current
events: Infection, acute injury .
6. Perioperative
7. Pregnancy
8. Renal or hepatic diseases.
9. Allergy to oral agents
10. Latent autoimmune diabetes in adult.
Current Insulin Preparations
Onset Peak Duration
Short acting
Lispro/aspart <15 min 1 hr 3 hr
Regular 0.5-1 hr 2-3 hr 3-6 hr
Long acting
NPH 2-4 hr 7-8 hr 10-12 hr
Insulin glargine 1-2 hr Flat 24 hr
Insulin detemir 1-2 hr Flat 12-20 hr
Combinations
NPH/ Regular 0.5-1 hr dual; 2-3 10-12
(70/30, 50/50) 7-8
Protamine Lispro/Lispro <15 min dual; 1 hr 12
(75/25, 50/50) several hrs

Lispro as Apidra , aspart as novorapid , glargine =lantos, not mixed.


Twice-daily regimen of insulin
or
Multiple daily injections.

(0.5 -1.0 IU / Kg /d)


Twice-daily Insulin Regimen (Split-
Mixed and Pre-Mixed Regimens)
Total insulin dose

2/3 1/3
Morning dose : Evening dose:
2/3 NPH 1/2 NPH
1/3 Regular (or insulin 1/2 Regular (or insulin
lispro/aspart) lispro/aspart)
Physiologic Insulin Replacement
Insulin therapy should closely mimic normal insulin physiology
Ideally, insulin therapy should address:
Basal needs: Intermediate-/long-acting insulins (NPH, detemir, glargine)
Bolus needs: Rapid-/short-acting insulins given with meals (lispro, aspart, glulisine, regular insulin)

Meal Meal Meal

Expected insulin changes during the day for individuals without diabetes

Normal insulin physiology is matched by multiple insulin injections


This image cannot currently be display ed.

Onset Peak Duration


Humulin R, Mixtard 0.5-1 hr 2-3 hr 3-6 hr
Onset Peak Duration
Humalog, Novorapid, Apidra <15 min 1 hr 3 hr
Insulin Lispro
Insulin lispro is an insulin analog that The (proline),
(lysine) amino acid sequence of the insulin molecule is
reversed to be lysine-proline resulting in a rapid
absorption, within 15 minutes. Because insulin lispro
can be injected just before (or after) the meal versus
waiting 30 minutes with regular insulin, patients may
find it provides them with more flexibility and
convenience & has a reduced risk of hypoglycemia
compared to regular insulin.
Insulin Aspart
Insulin aspart is a human insulin analog approved
2000. The amino acid proline is substituted with aspartic
acid resulting in a rapid onset of activity. Insulin aspart
should be injected 5-10 minutes before the meal.
Onset Peak Duration
Humulin N, Insulatard 2-4 hr 6-8 hr 10-12 hr
Onset Peak Duration
Lantus 1-2 hr Flat/Predictable 24 hr
Insulin therapy perioperatively;
1. Continuous basal insulin is given (either IVI 1 unit /h
or 0.02 U/kg or SC long lasting insulin (25-50%) of
the usual dose .
2. +/- IVI glucose.
3. Oral HypoG # as insulin & Glucose requirement are
changing rapidly, dangerous specially in fasting.
4. Sliding scale upon rise of BG should be avoided.
5. TPN insulin need even in non-DM IVI of
insulin.
Diabetes:

Fungal nail infections

granuloma annulare

Diabetic cheiroarthropathy
Dupuytren's contracture
Granuloma
annulare
pale, shiny rings and
nodules usually seen
on the hands in DM.
Granuloma annulare.
Ketoacidosis
Hyperglycemic Hyperosmolar Coma
Clinical Impact of Diabetes Mellitus
Diabetes

A 2- to 4- The leading The leading The leading


fold cause of cause of cause of
increase in new cases new cases nontraumatic
lower
cardio- of end of blindness
extremity
vascular stage renal in working-
amputations
mortality disease aged adults
Lessons from UKPDS:
Better control means fewer
EVERY 1% complications REDUCED
reduction in HBA1C
RISK*
Deaths from diabetes

Heart attacks

1%
Microvascular complications

Peripheral vascular disorders

*p<0.0001
UKPDS 35. BMJ 2000; 321: 405-12
Hypoglycemia is a serum glucose
< 60 mg/dL (3.3 mmol/L)
Hypoglycemia
Hypoglycemia: Etiology:
Liver disease
Alcohol intake
Adrenal failure, hypopituatarism,.
Factitious hypoglycemia.
Insulin producing tumours & B-cell hyperplasia( insulinoma
& others)
Hypoglycemias in diabetics : Risk
Factors
Patient related: Drug related:
o Elderly o Insulin treatment
o Pattern of insulin release
o Poly-pharmacy. o Type of SU and presence of
o Initiation of treatment. active metabolites.
o Drug interaction e.g. ACEI and BB
o Tight BG control.
o Neuropathy.
o Non compliance.
o Infection.
o Irregular/missed meals or
exercise
CAUSES OF HYPOGLYCEMIA IN
DIABETICS
Management related
Overdosages
Insufficient caloric intake.
Site of injection, local massage, hot baths or sauna.
Sudden exercise.
Alcohol intake with SU.
Severe renal & hepatic dysfunction.
Adrenal failure, hypothyroidism. Hypopituitarism.
Factitious hypoglycemia.
Nocturnal hypoglycemia
Often identified by the patients partner, or
family members by :
Restlessness ,Sweating ,Convulsions.
The patients may have :
Morning headache. Chronic fatigue.
Poor sleep quality. Nightmares.

More common with long-acting insulins that have a distinct


peak four to eight hours after injection (such as NPH
insulin, 70/30 insulin )

More likely when the evening dose of long-acting insulin is


taken before the evening meal rather than at bedtime
Always Carry Rapid-Acting Carbs
Hypoglycemia Treatment

Severe Low
Unconscious /
Unresponsive
Seizure

Uncooperative
A 25-year-old female presented with six months
history of depression, irritability and painful sensory
symptoms in her legs. Over the last four weeks she
presents a broad base ataxic gait.
An MRI brain showed bilateral posterior thalamic
nuclei (pulvinar region) high signals.
Which of the following is the most likely diagnosis?
Herpes simplex encephalitis.
Multiple system atrophy.
New variant CJD.
Sporadic CJD .
Wilson disease
New variant Creutzfeldt-Jakob disease (CJD) usually
presents in a young person, in their twenties or thirties.
In the majority of the cases the first symptoms are
psychiatric and painful sensory symptoms in the lower
limbs.
Ataxia and involuntary movements (for example,
myoclonus) usually appear at an interval of about six
months after the initial symptoms.
MRI brain shows bilateral pulvinar (posterior thalamic
nuclei) high signals.
EEG is usually normal in new variant CJD.
A 35 year old woman has been admitted to hospital for
investigation of progressive weakness in her legs. For the
past 5 years. The patient's mother has similar difficulties with
weakness and sensory problems. Examination revealed
power of 3/5 distally in the upper and lower limbs with a glove
and stocking pattern sensory loss to pain and touch.
What is the most likely diagnosis?
A. subacute combined degeneration of spinal cord.
B. Friedrich's ataxia
C. Chronic inflammatory demyelinating polyneuropathy .
D. Multiple sclerosis .
E. Hereditary sensory & motor neuropathy.
Answer: e) hereditary sensori motor neuropathy. In view of the family history, this
patient is most likely to have hereditary sensori motor neuropathy type I (Charcot
Marie Tooth disease). HMSN 1 is the most common form of hereditary neuropathy.
Severely and uniformly slowed nerve conduction velocities (NCVs) and primary
hypertrophic myelin pathology with prominent onion bulbs and secondary axonal
changes are the hallmarks of the disease. Motor symptoms predominate over
sensory symptoms. Often, patients report loss of balance, muscle weakness, and foot
deformities. Onset in the first decade of life is typical, but disease develops in some
patients in young or mid adulthood.
HMSN 2, on the other hand, represents the nondemyelinating neuronal type with
relatively normal NCVs and primary axonal pathology. Although nerves are not
enlarged in the neuronal form, weakness often is less marked and onset of this
neuropathy is delayed. Peripheral nerves are not enlarged clinically, and weakness of
feet and leg muscles predominates; hands are less severely affected than the legs.
Patients experience sensory loss in the distal extremities, and foot deformities (ie,
pes cavus) tend to be less marked than those of HMSN 1.

A 40 year old man presents with an


uncomfortable sensation in his face, comes
to the casualty department. He is unable to
lift his eyebrows and also has bilateral facial
weakness. Which one of the following is
most likely to cause bilateral lower motor
neuron weakness?
A. Cerebrovascular disease.
B. Pontine haemorrhage.
C. Lyme disease.
D. Multiple sclerosis.
E. Motor neuron disease
Answer: c) lyme disease.

Causes of Upper Motor Neuron facial weakness


are:
CVA
pontine haemorrhage
Multiple Sclerosis
motor neuron disease

Lyme disease usually causes lower motor


neuron facial weakness.
A 47-year-old personal assistant attends the outpatient
clinic with a six-month history of malaise and joint pains.
She has had to quit her job. She is also complaining of
difficulty sleeping at night and urinary frequency. The
investigations show: full blood count normal, U&E
normal, LFTs normal, ESR normal, CRP normal, Thyroid
function normal Autoimmune screen normal What is the
most likely diagnosis?
1. fibromyalgia
2. Depression
3. rheumatoid arthritis
4. Vasculitis
5. systemic lupus erythematosus
True 1. Fibromyalgia is said to occur in
between 1 and 2% of the general population
and has a female to male ratio of 9:1,
presenting most often in the 3050 year age
group. It presents with musculoskeletal pain
that often has multiple trigger points and
may be poorly defined. Self-management
strategies, reassurance and tricyclic anti
depressants for sleep disturbance are the
mainstays of therapy. Aerobic exercises
such as swimming are recommended.
Symptoms wax and wane for many years.
33- A teenage girl presents with Guillain-Barre
syndrome. Her weakness continues to worsen
after admission to hospital. Which of the
following should be used to monitor her?
1) arterial blood gases
2) chest expansion size
3) FEV1/FVC ratio
4) PEFR
5) vital capacity
A 46-year-old male presents passing 4-5 litres
of urine per day, after commencing a new drug.
S.sodium 142 mmol/l, Plasma osmolality 295
mosmol/l (275-290), Urine osmolality 280
mosmol/l (350-1000).What drug was
prescribed?
1 )Carbamazepine
2 )Chlorpropamide.
3 )Fluoxetine .
4 )Furosemide .
5 )Lithium.
A 21-year-old Nigerian woman was referred by her general
practitioner with a progressive history of polydipsia and polyuria
of 6 months duration. She has a history of sickle cell disease
and had been admitted on two previous occasions to hospital
with chest pain precipitated by crisis. Fasting blood glucose is
4.5 mmol/l. She is not on any medication.
A water deprivation test was performed. Water deprivation
phase plasma osmolality298 mosm/kg (278300 mosm/kg)
urine osmolality300 mosm/kg
DDAVP Phase plasma osmolality295 mosm/kg,urine
osmolality325 mosm/kg. What diagnosis would best fit the
clinical picture and investigation result?
1. Sickle cell nephropathy
2. Cranial diabetes insipidus
3. Psychogenic polydipsia
4. Nephrogenic diabetes insipidus
5. Lithium-induced nephrotoxicity
True 4. The disease is characterised by the inability to concentrate urine
due to the lack of response of renal tubules to antidiuretic hormone
(ADH). Normally the kidneys concentrate urine by increasing water
resorption by the collecting duct in the presence of ADH; this
mechanism helps to maintain plasma osmolality and extracellular
volume. Without ADH, a large amount of dilute urine is excreted.
The water deprivation test after an overnight fast assesses the kidneys
urine-concentrating ability and response to ADH. The result shows
abnormally low (< 750 mosm/kg) urine osmolality, which increases
only slightly after exogenous ADH (1-deamino-8-D-arginine-
vasopressin (DDAVP), vasopressin). Nephrogenic diabetes insipidus
(NDI) is X-linked recessive. Homozygous affected people (all males)
are completely unresponsive to ADH while heterozygous females
show normal or slight impairment. Acquired NDI occurs in disorders
that disrupt the medulla or distal nephron and impair concentrating
ability. Causes are: sickle cell nephropathy, polycystic kidney disease,
pyelonephritis, amyloidosis, certain nephrotoxins such as lithium and
demeclocycline.
27- In Psoriatic arthropathy all true except
A is usually associated with psoriatic nail
change.
B may cause sacro-iliitis .
C can occurs in the absence of psoriasis of the
skin.
D is associated with uveitis.
E responds to chloroquine, which is the
treatment of choice.
A 25 year-old lady recently diagnosed with rheumatoid
arthritis. She has developed weakness, double vision
and tiredness. Examination reveals bilateral weakness of
eye abduction, bilateral ptosis, slightly reduced proximal
motor power in the limbs, normal reflexes and sensation.
What is the diagnosis?
1) Chronic progressive external opthalmoplegia.
2) Guillain-Barre syndrome.
3) Multiple sclerosis.
4) Myasthenia gravis.
5) Polymyositis
The answer is 4
Myasthenia gravis is well known to be
associated with other autoimmune
diseases such as pernicious anaemia,
thyroid disease and rheumatoid arthritis. In
Guillain-Barre syndrome you will expect
absent reflexes. Polymyositis does not
usually cause ptosis or ophthalmoplegia.
A 70 year man has been short of breath for 1 year. An
electrocardiogram shows T wave inversion and q
waves in the anterolateral leads at rest. He has
cardiomegaly on the chest X ray. Clinical examination
shows a third heart sound, a soft systolic murmur in
the mitral area and also bilateral inspiratory
crepitations in his lungs.
Which of the following is he at risk of?
A. Deep vein thrombosis
B. Pulmonary embolus
C. Systemic arterial embolus from mural thrombus
D. Venous thrombosis due to mural thrombus
E. Coronary artery thrombus due to mural
thrombus
Answer: c) systemic arterial embolus from
mural thrombus. This man has features
indicating that he has an enlarged left
ventricle from previous anterior myocardial
infarction in the LAD artery territory. Poor
LV function also causes his symptoms of
heart failure. He is at risk of developing
mural thrombus with embolus to the
arterial circulation.
A 12 year old girl has a diagnosis of the 21
hydroxylase deficiency form of congenital
adrenal hyperplasia. Which one of these
features is likely to be present?
A. High cortisol .
B. Ambiguous genitalia .
C. Alopecia .
D. Precocious puberty.
E. Hypertension.
Answer: b) ambiguous genitalia. In congenital
adrenal hyperplasia, hirsutism, ambiguous
genitalia and normal puberty (precocious
puberty in boys) are seen in girls. There is also
cortisol deficiency and replacement with
dexamethasone is appropriate. Hypertension
typically occurs in the rarer forms but not 21
hydroxylase deficiency which is the commonest
form of CAH.

21 Hydroxylase deficiency
A 63 year old male is admitted with acute onset
unsteadiness of gait, dizziness and dysphagia.
Examination revealed a right-sided Horner's syndrome,
nystagmus, loss of pain and temperature sensation on
the left side of the trunk and in the left arm and leg, and
gait ataxia.
What is the most likely diagnosis?
1) leaking posterior communicating artery aneurysm
2) left sided acoustic neuroma
3) posterior inferior cerebellar artery occlusion
4) right sided pontine infarct
5) spontaneous left sided cerebellar haemorrhage
The answer is 3
This is Wallenberg's syndrome/ lateral
medullary syndrome and is due to
occlusion of the posterior inferior
cerebellar artery.
A 50 year old lady has polyuria. Her serum
calcium is 2.85 mmol/l and phosphate is 0.9
mmol/l. Which of the following could cause
this?
A. Vitamin D deficiency
B. Loop diuretics
C. Lithium
D. Diltiazem
E. Bisphosphonates
Answer: c) lithium. Lithium, thiazides (not
loop) diuretics and vitamin A or D
intoxication can cause hypercalcaemia.
.A 45 year old patient presents with proximal
muscle weakness, particularly in the lower
limbs. She has a heliotropic rash around
the eyes and also Gottrons papules.
Which one of the following antibodies is
most strongly associated?
A.La
B. Ro
C. Jo-1
A.D. SCL-70
E. Anti DsDNA
Answer: C) Jo-1. The diagnosis is
dermatomyositis. Anti Jo-1 antibody is
associated with acute onset myositis, particularly
dermatomyositis. The limb girdle or proximal
muscles are most severely affected in both
polymyositis and dermatomyositis.

Gottron's papules
A 35 year old lady has grey pigmentation of her skin and
hypotension. Her early morning cortisol is 45 mol/l
and her sodium is 127 mmol/l. Which of the following is
the best replacement regimen?
A. Hydrocortisone 10mg mane, 5mg mid day and 5
mg evening and also fludrocortisone 100 g mane
B. Hydrocortisone 10mg mane, fludrocortisone 100 g
mane
C. Prednisolone 20 mg mane
D. Hydrocortisone 40mg mane and fludrocortisone 300
g mane
E. Hydrocortisone 20mg mane, 10 mg mid day
Answer: a) hydrocortisone 10mg mane,
5mg mid day and 5 mg evening and also
fludrocortisone 100 g mane. She has
Addison's disease and needs both
glucocorticoid (hydrocortisone) and
mineralocorticoid (fludrocortisone)
replacement. Steroid replacement is
usually given 10/5/5 mg or 10/5 mg,
although this is adjusted with cortisol day
curves. Fludrocortisone 100 g mane
should be adequate.
A patient has been in atrial fibrillation for at least
several months. He is brought in for DC
cardioversion because of troublesome
palpitations and breathlessness. His current
medications are digoxin and warfarin. Following
successful cardioversion, which changes
should be made?
A. Discontinue warfarin and digoxin
B. Continue warfarin but not digoxin till
outpatient review
C. Continue digoxin but not warfarin
D. Continue both digoxin and warfarin
E. Continue warfarin for life
Answer: b) continue warfarin but not digoxin till
outpatient review. For at least 4 weeks following
DC cardioversion, the patient is still at risk of
thromboembolism. If the patient remains in sinus
rhythm after 6-8 weeks then warfarin can be
discontinued unless there is evidence of
paroxysmal AF. Digoxin should be stopped once
sinus rhythm is achieved since its role is only for
rate control. It does not help in maintenance of
sinus rhythm.
A 20-year-old man presents with a history of intermittent fever for 1
month, accompanied by chest pain, which is worse on lying down.
He has recently also noticed pain in the joints of his hands and feet.
He is febrile, with a temperature of 39oC. His pulse is 98/min and his
blood pressure is 110/70 mmHg. The positive findings on
examination are enlarged anterior cervical lymph nodes, pericardial
rub, mild hepatomegaly and synovitis of the proximal
interphalangeal (PIP) and wrist joints. Investigations show the
following:Hb 10.5 g/dl, WBC 20 x 10 9/l, P82%, PLT540 x 109/l,
ESR110 mm/1st hr, CRP 246 mg/l, ALT69 IU/l, ALK-P 246 IU/l,
Ferritin 4567 g/l (normal range 14 200 /l), CXR cardiomegaly, 2-
D echocardiogram pericardial effusion MSU and blood culture
negative, ANA, RF, ANCA all negative, CT abdomen
hepatosplenomegaly. Which of the following is most likely
diagnosis?
1. SLE
2. Mixed connective tissue disease (MCTD)
3. Undifferentiated connective tissue disease (UCTD)
4. RA
5. Adult-onset Stills disease
True 5. Clinical features of adult-onset Stills disease
include arthralgia or arthritis; a high fever (> 39oC),
especially in the afternoon and evening; fleeting
salmon-pink maculopapular rash which is most
prominent with fever; serositis; anaemia; elevated
acute phase reactants; and evidence of
reticuloendothelial activation. In between the febrile
episodes the patient feels rather well.

The laboratory investigations commonly reveal


leucocytosis, thrombocytosis, raised serum ferritin
and an elevated erythrocyte sedimentation rate and
C-reactive protein. Autoantibodies are negative.
A 70-year-old woman referred by a a breast lump.
She was asymptomatic but her investigations
reveal:Corrected calcium2.72 (2.2 -
2.6)Phosphate0.80 (0.8-1.4)Alkaline
phosphatase110 U/L (20 - 95)PTH
concentration5.1 pmol/L (0.9-5.4)
What is likely diagnosis?
1 )bony metastases
2 )chronic vitamin D excess
3 )ectopic PTH related peptide (PTHrp) secretion
4 )multiple myeloma
5 )primary hyperparathyroidism
A 19-year-old male presents with concerns regarding his
pubertal development. On examination he 1.8 m tall, thin
little pubic & axillary hair. Both testes are approximately 5
mls in volume (Normal 15mls). No other abnormalities are
encountered. Investigations reveal: LH 3.3 mu/l (3-10) FSH
5.5 mu/l (3-10) Testosterone 5.5 nmol/l (9-30) Which of
following is the most likely diagnosis?
1 )Anorexia nervosa .
2 )Craniopharyngioma .
3 )Kallmann syndrome .
4 )Klinefelters syndrome .
5 )Primary testicular failure.
A 74-year-old man has had increasingly severe, throbbing headaches
for several months, centered on the right. There is a palpable tender
cord-like area over his right temple. His heart rate is regular with no
murmurs, gallops, or rubs. Pulses are equal and full in all
extremities, BP is 110/85 mmHg. A biopsy of this lesion is obtained,
and histologic examination reveals a muscular artery with lumenal
narrowing and medial inflammation with lymphocytes, macrophages,
and occasional giant cells. He improves with a course of high-dose
corticosteroid therapy. Which of the following laboratory test findings
is most likely to be present with this disease?
1) Anti-double stranded DNA titer of 1:1024
2) Erythrocyte sedimentation rate of 110 mm/hr
3) HDL cholesterol of 0.6 mmol/L
4) pANCA titer of 1:160
5) Rheumatoid factor titer of 80 IU/mL
A 21-year-old man has recently returned from a
holiday in the Far East. He gives a history of
symmetrical joint pain in his knees and ankles
and feet and also complains of sore eyes. His
ESR is raised. A synovial fluid aspirate is sterile
but with a high neutrophil count.What is the
most likely diagnosis?
1. Ankylosing spondylitis
2. Septic arthritis
3. Reactive arthropathy
4. Rheumatoid arthritis
5. Stills disease
True 3. The history of recent travel to the
Far East raises the possibility of
venereal disease. The lower limb
symmetrical arthropathy is
characteristic of reactive arthropathy. A
history should be taken of
urethritis/dysuria and conjunctivitis to
complete the Reiters syndrome triad.
A 32-year-old woman presents a four month history of amenorrhoea.
She takes no specific therapy. She has two children her husband has
a vasectomy. Examination reveals an obese individual but no other
abnormality.
Investigations reveal:Serum oestradiol 100 pmol/L(NR 130 - 500) S.
LH 2.1 mU/L(NR 3.0 - 6.6) S. FSH 2.2 mU/L(NR 3.3 10) S. prolactin
800 mU/L(NR 50 - 300)Serum testosterone2.1 pmol/L(NR < 3.0)
Which investigation
1 )Insulin tolerance test
2 )Pregnancy test
3 )17 hydroxy-progesterone
4 )Urine free cortisol concentration
5 )Magnetic resonance imaging (MRI) of pituitary .
A 75-year-old man presents with 12 months history of
cognitive impairment, parkinsonism, intermittent
confusion and generalised myoclonus. He was started
on 62.5 tds of sinemet. In the following 2 months he was
started experiencing visual hallucinations. The most
likely diagnosis is:
1) Idiopathic Parkinson's disease
2) Alzheimer's disease
3) Diffuse Lewy body disease
4) Multiple system atrophy
5) Progressive supranuclear palsy
A 70-year-old man from Lancashire has noted increasing
back & leg pain over several years. X-rays reveal bony
sclerosis of the sacroiliac, lower vertebral, & upper tibial
regions with cortical thickening, but without mass effect
or significant bony destruction. He also says his hat
does not fit him anymore. He has greater difficulty
hearing on the left. He has orthopnea & pedal edema.
Blood tests reveal an elevated serum alkaline
phosphatase. The most likely pathologic process that
explains these findings is?
1 )Decreased bone mass
2 )Metastatic adenocarcinoma
3 )Paget's disease of bone
4 )Renal failure & renal osteodystrophy
5 )Vitamin D deficiency
Comments: true 3.

This man has Paget's disease with high output


cardiac failure & sensorineural deafness.
Renal osteodystrophy leads to lesions of osteitis
fibrosa cystica admixed with osteomalacia,
which are focal in nature. Metastatic disease in
bone produces focal lesions.
A 17 year presents tingling & muscle cramps. There
is no other past history of note. Investigations reveal
Creatinine68 micromol/L (50-100)calcium1.76 (2.2-
2.6)albumin38 g/L (37-49)
Which one of following investigations is likely to
confirm diagnosis?
1 )Alkaline phosphatase concentration
2 )CT brain scanning
3 )PTH concentration
4 )Urine calcium concentration
5 )Vitamin D concentration
An otherwise healthy middle-aged man without
prior medical history has had increasing back
pain & right hip pain over the past 10 years.
The pain is worse at the end of the day. He
has bony enlargement of the distal
interphalangeal joints. A radiograph of the spine
reveals the presence of prominent osteophytes
involving vertebral bodies. There is sclerosis
& narrowing of the joint space at the right
acetabulum seen on a radiograph of the pelvis.
Which of the following pathologic processes is
likely to be taking place in this patient?
1 )Gout
2 )Lyme disease
3 )Osteoarthritis
4 )Osteomyelitis
5 )Rheumatoid arthritis
Comments: true 3.
Degenerative osteoarthritis is a common & progressive
condition that becomes more frequent symptomatic
with aging. There is erosion & loss of articular
cartilage. Rheumatoid arthritis typically involves small
joints of the hands & feet, there is a destructive
pannus that leads to marked joint deformity. A gouty
arthritis is more likely to be accompanied by swelling,
deformity & joint destruction. The pain is related to
usage. Osteomyelitis represents an ongoing infection
that produces marked bone deformity, not just joint
narrowing. Lyme disease produces a chronic arthritis,
but it is typically preceded by a deer tick bite & a skin
lesion. It is much less common than osteoarthritis.
A 62-year-old man attends for review. He has
evidence of arthritis affecting the shoulder, elbow,
radiocarpal, and knee joints bilaterally, with pain
also in the left first metatarso-phalangeal joint. X-
ray of the left knee reveals calcification within the
hyaline cartilage. Aspiration of the left knee
reveals calcium pyrophosphate crystals. Serum
uric acid is slightly above the upper limit of normal.
Other medical history of note is ulcerative colitis.
What is the most likely diagnosis in this case?
1. Gout
2. Pseudogout
3. Primary osteoarthritis
4. Rheumatoid arthritis
5. Sero-negative arthritis
True 2. The clinical picture of joint involvement, coupled with
evidence of chondrocalcinosis and pyrophosphate crystals is
strongly suggestive of a diagnosis of pseudogout. The slightly
elevated uric acid and history of ulcerative colitis are red-
herrings in this case. Management involves use of simple
painkillers and non-steroidal anti-inflammatory agents coupled
with physiotherapy where appropriate. Associated conditions
include haemochromatosis and Wilsons disease, where there
are other symptoms and signs it may be worth screening for
these concomitant illnesses. Pseudogout has an equal
male:female ratio, with frequency increasing with increasing
age; it is said to affect around 5% of subjects above the age of
30 years.
35- A 35-year-old man returned from a two-week holiday
complaining of pain in the loins and painful swollen knees. On
examination he was afebrile and had significant bilateral knee
effusions. Mild penile erythema was also noted. Laboratory
investigations showed.
Hb 15.6 g/dL
WBC 16.2 x 109/l
Neutrophils 14.1 x 109/l
ESR 65 mm/h
Rheumatoid factor 10 IU/L
Urinalysis No cells, casts or bacteria seen
What is the most likely diagnosis?
1) Arthritis due to Neisseria gonorrhoeae infection.
2) Lymphogranuloma venereum.
3) Reactive arthritis.
4) Reitter's syndrome.
5) Rheumatoid arthritis.
A 70-year-old man presents with weight loss, lower limb
weakness and dry mouth. He has been a heavy smoker.
On examination, he looks cachectic; he has proximal
lower limb weakness, areflexia (reflexes normalise with
repetitive muscle contraction). There is no wasting or
fasciculations. Sensory examination is normal. Which of
the following blood test is the most likely to confirm the
diagnosis?
1) Acetylcholine receptors
2) Voltage gated calcium channels antibodies
3) Anti GM1 antibody
4) Antinuclear antibody
5) Anti Ro/La antibodies
The answer is 2
The most likely diagnosis is Lambert-Eaton syndrome. It
results when IgG autoantibodies blockade the voltage-
gated calcium channels of peripheral cholinergic nerve
territory. 50% of the cases are associated with small cell
lung carcinoma. Proximal lower limb weakness is the
most consistent neurological feature. Ptosis and
ophthalmoplegia are rare. Autonomic dysfunction is
common (e.g. dry mouth). The reflexes are depressed or
absent but normalise with repetitive muscle contraction.
A 42-year-old man reviewed in outpatients has a 6-
month history of increasing shortness of breath on
exertion and feelings of lightheadedness when
digging in his garden. His general practitioner
organised an open access echocardiogram which
showed a septal thickness of 26mm and a left
ventricular outflow gradient of 85mmHg. Which of
the following is NOT a risk factor for sudden death
in patients with hypertrophic cardiomyopathy?
A : Unexplained syncope
B : Sudden death from HOCM in 2 or more first
degree relatives <40years of age
C : Family history of sudden death
D : left ventricular wall thickness of >30mm
E : Hypertension.
Major risk factors for hypertrophic
cardiomyopathy are:
Cardiac arrest (ventricular fibrillation)
Spontaneous sustained ventricular tachycardia
Family history of sudden death.
Unexplained syncope
Left ventricualr wall thickness>30mm
Abnormal blood pressure on exercise(failure to
rise from baseline by 25mmHg)
Non sustained Ventricular tachycardia
61- 54 year old female is admitted with
progressive weakness following a trivial flu-
like illness. Which of the following would
exclude Guillain-Barre Syndrome as the
diagnosis?
1) Autonomic dysfunction.
2) Elevated protein on CSF examination.
3) Areflexia.
4) Ophthalmoplegia.
5) Sensory level below D1.
The answer is 5
GBS is a post-infectious acute polyneuritis typified
by elevated CSF protein with few cells and often
normal glucose. There is a profound weakness
associated with areflexia and peripheral sensory
neuropathy. Ophthalmoplegia is associated in
particular with the Miller-Fisher variant.
However, a sensory level is NOT a feature and
would suggest cervical myelopathy. Muscle
wasting is typical with prolonged illness.
Autonomic disease may also feature.
A 68 year-old presents with a 4 month history of weight loss,
headaches & had recently developed double vision. 6 years
previously she underwent a right mastectomy for breast carcinoma
& remains on treatment with Tamoxifen.
Examination shows tenderness over the temporal region & a left
sixth nerve palsy.
Her chest X-ray was reported as normal, but she had an ESR of 100
mm/hr , her Hb was 10.8 g/dl. Which of following statements
correct?
1 )An isotope bone scan should be performed
2 )An urgent CT brain scan required
3 )She should be treated prednisolone immediately
4 )She should have a lumbar puncture
5 )She should be given Diamorphine
An 18 year old man presented with a history of a sudden
onset of a frontal headache and photophobia. He had
neck stiffness and a temperature of 38C.
Which one of the following findings would suggest a
diagnosis of subarachnoid haemorrhage rather than
bacterial meningitis?
1) a blood neutrophil leucocytosis
2) a family history of polycystic renal disease
3) a fluctuating conscious level
4) a history of diabetes mellitus
5) a history of opiate abuse
The answer is 2
Fluctuating level of consiousness can occur
in both meningitis and subarachnoid
haemorrhage (SAH). Hypertension is a
risk factor for SAH, but not diabetes.
Opiate abuse does not increase the risk
for SAH. Cerebral aneurysm are
associated with polystic kidney disease.
An 80-year-old woman has a three month history of progressive
numbness and unsteadiness of her gait. On examination, there is a
mild spastic paraparesis, with brisk knee reflexes, ankle reflexes are
present with reinforcement, extensor plantars, sensory loss in the
legs with a sensory level at T10, impaired joint position sense in the
toes, and loss of vibration sense below the iliac crests.
Investigations were as follows:-
haemoglobin 12.0 g/dl
MCV 99 fl
What is the most likely diagnosis?
1) anterior spinal artery occlusion
2) dorsal meningioma
3) multiple sclerosis
4) subacute combined degeneration of the cord
5) tabes dorsalis
A 46-year-old male presents passing 4-5 litres
of urine per day, after commencing a new drug.
S.sodium 142 mmol/l, Plasma osmolality 295
mosmol/l (275-290), Urine osmolality 280
mosmol/l (350-1000).What drug was
prescribed?
1 )Carbamazepine
2 )Chlorpropamide.
3 )Fluoxetine .
4 )Furosemide .
5 )Lithium.
A 50-year-old man presented with a six-week history of
general malaise and a 2 day history of a right foot drop,
a left ulnar nerve palsy and a widespread purpuric rash.
He complained of arthralgia but had no clinical evidence
of inflammatory joint disease. Investigations revealed:
ESR100 mm/hr, ANCA negative, ANA negative,
Rheumatoid factor strongly positive, C3 0.8 g/L (NR
0.75 - 1.6), C4 0.02 g/L (NR 0.14 - 0.5), Urine dipstick
Blood ++, no protein. An echocardiogram was normal
and two sets of blood cultures were negative.What is the
most likely diagnosis?
1 )ANA negative SLE
2 )Cryoglobulinaemia
3 )Infective endocarditis
4 )Polyarteritis nodosa
5 )Rheumatoid arthritis
The answer is 2
The presence of a sensory loss at T10
indicates a thoracic mylopathy. Subacute
combined degeneration of the cord is
unlikely as Hb and MCV are normal.
Anterior spinal artery occlusion is unlikely
as the history is progessive.
63- A 45-year-old man has a history of progressive
weakness for 5 weeks. He had particular difficulty getting
out of the bath. On examination there was severe truncal
and proximal limb weakness, without wasting or
fasciculation. Tendon reflexes, plantar and sensation
were all normal. The vital capacity was 1.8L. What is the
most likely diagnosis?
1) cervical myelitis
2) Guillain-Barre syndrome
3) polio
4) polymyositis
5) syringiobulbia
The answer is 4
The presentation of myopathy is
characterised by priximal weakness with
normal reflexes and sensation and the
absence of fasciculations. Polymyositis is
the commonest cause of inflammatory
muscle disease in < 50 years old
(inclusion body myositis is the commonest
in >50 years old).
A 17-year-old male is investigated for short stature. He has a previous
diagnosis of slipped femoral epiphysis diagnosed at the age of 11. He
presented with this disorder at this age with pain in the hip and limp. This
was treated by the orthopaedic surgeons with nonsurgical containment of
the femoral head in the acetabulum using casts.

On examination he is on the 12th centile for height, a BMI of 30 and has


normal pubertal development. His blood pressure is 108/70 mmHg and he
has a pulse of 90 beats per minute. No abnormalities are noted on
examination of the chest, heart and abdomen.
Investigations reveal:
Haemoglobin 12.8 g/dL (13.0-18.0) White cell count 5.4 x109/L (4-11
x109) Platelets 143 x109/L (150-400 x109) Serum Na 133 mmol/L (137-
144)Serum K 4.2 mmol/L (3.5-4.9) Serum Creatinine 96 mol/L (60-110)
Serum Calcium 2.02 mmol/L (2.2-2.6) Serum Phosphate 1.8 mmol/L (0.8-
1.4) PTH 15.8 pmol/L (0.9-5.4).

Which of the following is the most likely explanation of this boy's presentation?
Coeliac disease.
Primary hypoadrenalism .
Pseudohypoparathyroidism.
Renal osteodystrophy.
Vitamin D resistant osteomalacia.
This young boy has short stature and previous
history of slipped femoral epiphysis.
The investigations reveal a hypocalcaemia and
a hyperphosphataemia suggesting a
hypoparathyroidism (both calcium and
phosphate would be expected to be low in
vitamin D deficiency and hypophosphataemia in
vitamin D resistant rickets) yet the raised
parathyroid hormone (PTH) concentration is
elevated indicating pseudohypoparathyroidism.
A 55-year-old man who had received
haemodialysis for many years presents with
deteriorating discomfort in both shoulders. Past
medical history included bilateral carpal tunnel
decompression. His Investigations reveal:
haemoglobin10 g/dl, ESR 30 mm/1st hr (1-10),
C-reactive protein 12mg/L (1-10), Urate 0.58
(less than 0.45)What is the most likely
diagnosis?
1 )B2 microglobulin amyloidosis
2 )Gout
3 )Pseudogout
4 )Polymyalgia rheumatica
5 )Osteoarthritis
Comments: true 1.
The features of shoulder pain associated with a
past history of carpal tunnel syndrome in a
patient receiving haemodialysis suggests a
diagnosis of b2 microglobulin amyloidosis.
Amyloid deposits composed of b2-microglobulin
as a major constituent protein are mainly
localized in joints with periarticular bone that
lead to destructive arthropathy which tends to
develop 5- 10 years after the initiation of
dialysis. Death from amyloidosis of gut & heart
may occur after 20 years of dialysis.
The dual X-ray bone absorptiometry (DXA)
scan results of a 60-year-old man with RA
being treated with prednisolone 7.5 mg per
day and methotrexate 10 mg per week
show T scores of -1.6 at spine and - 1.8 at
hips. Which of the following would you
add?
1.Calcium and vitamin D
2.Parathyroid hormone
3.Alendronate
4.Strontium
5.No change in therapy
True 3. A T score of less than -2.5 at hip or spine is
indicative of osteoporosis. However, in patients on
steroid therapy, a score of -1.5 is taken as a cut-off
value to start osteoporosis therapy. Use of
bisphosphonates (alendronate and risedronate) has
been shown to be associated with a reduced risk of
fractures. Bisphosphonates are most commonly
used first-line agents for the treatment of
osteoporosis. Calcium and vitamin D alone are not
sufficient in reducing the risk of fractures, although
they do offer some benefit. They may be given
concomitantly with bisphosphonates. Strontium and
parathyroid hormone are used in case of intolerance
or lack of response to bisphosphonates.
A 16 year old girl presented with a three week history of headache and
horizontal diplopia on far right lateral gaze. On two separate
occasions she noted dimmed vision whilst bending forwards. Over
the last year she had gained 12 kilograms in weight. On
examination, her weight was 95 kg, and height 162cms.
Neurological examination revealed bilateral papilloedema and a
partial right sixth cranial nerve palsy. What is the most likely
diagnosis?
1) Benign intracranial hypertension.
2) Multiple sclerosis.
3) Pituitary tumour
4) Superior sagittal vein thrombosis.
5) Thyroid eye disease.
The answer is 1
This patient is markedly obese with a BMI of 36 and the
history suggestive of BIH. Vision may be affected with
enlargement of the blind spot and the visual obscuration
with movements that provoke a rise in ICP (eg bending)
is typical of BIH. Dysthyroid eye disease would not
present like this and is more commonly associated with
Hyperthyroidism. The papilloedema would argue against
MS. A bitemporal hemianopia or a visual field defect
would be expected with a pituitary tumour. Venous sinus
thrombosis is a possibility but would be expected to
produce deteriorating symptoms.
5.A 30-year-old lady with a history of intravenous drug use is
admitted feeling generally unwell. She has a fever and is
complaining of arthralgia and a photosensitive rash. On
examination she has a soft systolic murmur at the lower left
sternal edge.
Investigations reveal: Electrocardiogram Sinus tachycardia,
Haemoglobin10.2 g/dl,White cell count 12.2 109/l,
Platelets,474 109/ l , Sodium 132 mmol/l, Potassium 4.6
mmol/l , Urea15.2 mmol/l,Creatinine 145 mol/l,Urine
microscopy=Red cell casts,Erythrocyte sedimentation rate 32
mm/h (020),C-reactive protein 64 mg/l (<10), Complement
C3--- 20 mg/dl (65190), Complement C4 --- 3 mg/dl (1550),
Renal biopsy=Focal segmental proliferative glomerulonephritis
What are the next appropriate investigations?
1. Antinuclear antibodies.
2. Rheumatoid factor.
3. Echocardiography.
4. Anticardiolipin antibodies.
5. Chest x-ray.
6. One of the following features is
consistent with a diagnosis of
polymyalgia rheumatica:

A proximal muscle pain & stiffness.


B an increased creatine kinase activity .
C proximal muscle weakness.
D an abnormal EMG.
E a macrocytic anaemia .
Which is true of herpes simplex
encephalitis?
1) brain MRI is characteristically normal
2) fits are uncommon
3) genital herpes is usually present
4) temporal lobe involvement is common
5) viral identification using polymerase chain
reaction on CSF is non-specific
The answer is 4
Herpes simplex encephalitis (HSE) is associated
with high signal in one or both temporal lobes
(limbic encephalitis). Seizures are commonly
present in HSE. Herpes Simplex Virus type 1 is
the causative virus (Not type 2 which is
associated with genital herpes). PCR for herpes
simplex virus on CSF is highly specific test.
A patient who has rheumatic mitral stenosis is
considered for percutaneous mitral
valvuloplasty. Which of the following would
contraindicate this procedure?
A. Dilated left atrium
B. Atrial fibrillation
C. Aortic regurgitation
D. Heavy calcification of the mitral valve
E. Long history of mitral stenosis
Answer: d) heavy calcification of the mitral
valve. The contraindications towards
valvuloplasty are heavy MV calcification,
thrombus in the left atrial appendage on
transoesophageal echocardiography and
severe mitral regurgitation. These patients
are indicated for mitral valve surgery
instead.
A 35 year old woman is admitted with a blood
pressure of 230/120. She has a sinus tachycardia
of HR 160 with intermittent runs of non sustained
ventricular tachycardia. 24 hour urine shows
increased Adrenaline of 720 (<80 nmol/24 hours)
and Noradrenaline 2300 (<780 nmol/24 hours).
Which one of the following medications would be
most useful?
A. Intravenous labetalol.
B. Intravenous amiodarone.
C. Intravenous phentolamine.
D. Intravenous diltiazem .
E. Oral flecainide.
Answer: c) intravenous phentolamine. In the
management of acute hypertensive crisis of
phaeochromocytoma as in this case, IV administration of
sodium nitroprusside, nitroglycerine, or phentolamine
can be used. Preoperatively, phenoxybenzamine
preoperative adrenergic-blockade of a1 and a2 receptors
with phenoxybenzamine (10-30 mg twice daily), or a1
receptors with prazosin (starting with 1 to 2 mg three
times daily. Beta blockers can be useful for arrhythmias,
but should not be commenced before alpha blockers
because b-blockade alone can cause marked
hypertension.
A 50 year old man presents with breathlessness.
His chest XR shows cardiomegaly. Which of the
following in the history might elucidate a cause
of cardiomyopathy?
A. Inferior T wave inversion
B. Systolic murmur in the mitral area
C. History of diabetes and a tanned
complexion
D. Family history of hyperlipidaemia
E. Family history of myocardial infarction
Answer: c) history of diabetes and a
tanned complexion. A history of diabetes
and bronze / tanned pigmentation suggest
haemochromatosis. Liver function tests
and iron studies would help to confirm the
diagnosis.
A 68 year old man with bone pain has the
following blood results:
calcium 2.27 (2.25-2.7) mmol/l
phosphate 1.3 (0.8-1.45) mmol/L
ALP 335 (20-120) U/l
What is the likely diagnosis?
A. Multiple myeloma
B. Pagets disease
C. Osteoporosis
D. Osteomalacia
E. Primary hyperparathyroidism
Answer: B) Pagets disease In pagets
disease, increased plasma alkaline
phosphatase reflects osteoblastic activity. .
Parathyroid hormone secretion usually
maintains a normal level of calcium ions in
the serum. Concentration of phosphate in
the serum may be normal or slightly
elevated.
A 21 year old female with epilepsy is well controlled on
sodium valproate 600mg bd and had been taking oral
contraceptives for three years. She presented to her
general practitioner 12 weeks pregnant.
Which of the following is correct?
1) An alternative anticonvulsant should be used in place of
sodium valproate
2) Interaction of sodium valproate with the oral
contraceptive increased the risk of pregnancy
3) The dose of sodium valproate should be increased
4) There is an increased risk of a neural tube defect in her
fetus
5) She is at increased risk of anaemia in pregnancy
The answer is 4
There is an increased risk of neural tube defects
associated with anti-convulsants during
pregnancy. However, the risks associated with
treatment are outweighed by the benefits in
preventing seizures, so the drug should be
continued. The risks may be minimised through
use of folate supplements. Sodium valproate is
not an enzyme inducer and would not speed up
metabolism of the pill.
A 35 year old woman is suspected of having
systemic sclerosis following presentation with
Raynaud's phenomenon. Which one of the
following is an associated feature?
A. Lax skin on the hands.
B. Non blanching purpura .
C. Perioral puckering.
D. Liver nodules.
E. Mesothelioma .
Answer: c) perioral tethering. Systemic
sclerosis is usually associated with dry
mouth and dry eyes, raynaud's
phenomenon, telangiectasia and perioral
tethering.

Perioral tethering in systemic sclerosis


A 60 year old woman has had difficulty walking due to
unsteadiness. She has lost 5 kg in weight over the past 6
months. On examination, she has horizontal nystagmus.
Her speech is slurred. There is incoordination of the
upper limbs evident on past pointing. Her gait is ataxic.
General examination reveals a palpable breast lump.
Which of the following is most likely to yield the
diagnosis?
A. Anti Yo antibodies
B. Anti Hu antibodies
C. Anti GAD antibodies
D. MRI of the brain
E. CT of the head
Answer: a) Anti Yo antibodies. This lady
has a cerebellar syndrome which may be
related to a breast malignancy. Anti Yo
antibodies are associated with a cerebellar
syndrome due to either lung, breast or
ovarian carcinoma.
Regarding pseudotumours cerebri (benign
hypercranial hypertension) which is true?
1) A mildly increased CSF cell count is typical.
2) May be caused by prolonged steriod therapy.
3) Is occasionally associated with focal
neurological signs.
4) Frequently presents with ataxia.
5) Is distinguished from hydrocephalus by the
absence of suture separation.
The answer is 2
Pseudotumour cerebri is a clinical syndrome that mimics brain tumours, and is characterised by raised
intracranial pressure with normal CSF cell count and protein content, normal ventricular size,
anatomy and position. Causes:
Metabolic disorders: galactosaemia, hypoparathyroidism, pseudohyperparathyroidism,
hypophosphatasia, steroid therapy, hypervitaminosis A, vitamin A deficiency, Addison's Disease,
obesity, menarche, oral contraceptives, pregnancy.
Infections: Roseola infantum, chronic otitis media, mastoiditis, Guillain Barr Syndrome. Drugs:
Nalidixic acid, tetracycline.
Haematological disorders: Polycythemia, haemolytic and iron deficiency anaemia, Wiskott Aldrich
Syndrome.
Destruction of intracranial drainage by venous thrombosis: Lateral sinus or posterior saggital sinus
thrombosis, head injury, obstruction of the superior vena cava. It usually presents with headache
and vomiting, though this is rarely as bad as that associated with posterior fossa tumour.
Diplopia is common due to 6th nerve palsy. Children are alert with no systemic upset. A bulging
fontanelle, cracked pot sounds, or separation of the cranial sutures may be present. Papilloedema
with an enlarged blind spot is the most consistent sign beyond infancy. Focal and neurological
signs indicate a process other than pseudotumour cerebri. It may be complicated by optic atrophy
and blindness. Most can be treated conservatively with monitoring of visual acuity. For others,
multiple lumbar punctures may be necessary to reduce intracranial pressure. Very rarely are
shunts required.
Which of the following features is
characteristic of myasthenia gravis?
1) Diplopia
2) Equal sex incidence
3) Fasciculation
4) Lid lag
5) Loss of pupillary reflexes
The answer is 1
Myasthenia gravis is commoner in females (it is an
autoimmune disease). The commonest features
include ptosis, diplopia and ophthalmoplegia. It
is a neuromuscular disorder and therefore does
not cause any lower motor neuron signs such as
fasciculations, wasting, and loss of reflexes.
Pupils are always normal. Lid lag is a feature of
thyroid eye disease.
A 68-year-old man presents with progressive
visual impairment. On examination there is an
incongruous homonymous hemianopia. The
most likely anatomical site of the neurological
lesion is at:
1) optic nerve
2) optic tract
3) chiasma
4) optic radiation
5) occipital lobe
The answer is 2
Optic neuropathy causes a central scotoma,
an optic tract lesion an incongruous
homonymous hemianopia, a chiasmal
lesion a bitemporal hemianopia, an optic
radiation and occipital lobe lesion a
congruous homonymous hemianopia.
A 25-year-old female presents with 2 days history of
diplopia and unsteadiness. 2 weeks ago she suffered an
upper respiratory tract infection. On examination there is
complete opthalmoplegia, areflexia and gait ataxia.
Which of the following blood tests is the most likely to
confirm the underlying diagnosis?
1) Acetylcholine receptors antibodies
2) Anti GM1 antibodies
3) Anti GQib antibodies
4) Anti Hu antibodies
5) Anti purkinje cell antibodies
The answer is 3
The most likely diagnosis is Miller Fisher syndrome (variant of Guillain Barre
syndrome). It consists of complete or partial ophthalmoplegia, areflexia and
ataxia. It usually follows antecedent infections. Serum IgG antibody to the
ganglioside GQib is present in more than 95% of patients. It is highly
specific for the syndrome.
Elevated levels of antibodies to the glycolipid ganglioside-monosialic acid (GM1
antibodies) have been shown, in some instances, to be associated with
certain neurological disorders: lower motor neuron syndromes, amyotrophic
lateral sclerosis, multiple sclerosis, other multifocal neuropathies, and
systemic lupus erythematosus (SLE) with central nervous system
involvement.
Neuronal Nuclear (Hu) Antibodies (NNA) are found in a number of
paraneoplastic syndromes, including subacute sensory neuronopathy,
paraneoplastic encephalomyelitis and paraneoplastic cerebellar
degeneration and are associated with small cell lung carcinoma.
Purkinje cell cytoplasmic antibodies are useful for identifying individuals with
subacute cerebellar degeneration or peripheral neuropathy due to a remote
(autoimmune) effect of gynecologic or breast carcinoma.
A 17-year-old man has been diagnosed with schizophrenia
4 weeks ago. He was started on haloperidol. Two weeks
later he was found confused and drowsy. On
examination he was pyrexial (40.7 C), rigid with blood
pressure of 200/100. Which of the following treatment
will you initiate?
1) phenytoin
2) diazepam
3) cefuroxime
4) acyclovir
5) dantrolene
The answer is 5
Neuroleptic malignant syndrome is the most likely
diagnosis. Its major features are: rigidity, altered
mental state, autonomic dysfunction, fever, and
high creatinine kinase. It is usually caused by
potent neuroleptics. The treatment of choice is
dantrolene and bromocriptine. Withdrawal of
neuroleptic treatment is mandatory.
Rhabdomyolysis and acute renal failure are
potential complications.
A 60-year-old woman presents with a short history of sinus
congestion, epistaxis and joint pains in her hands. She
becomes increasingly short of breath and develops
haemoptysis precipitating admission to hospital. On direct
questioning she admits to passing less urine in the
preceding few days. On examination, Heart sounds are
normal with no murmurs. She has bibasilar fine crackles on
auscultation of the lungs but her JVP is not elevated and she
has no pedal oedema. A chest X-ray (CXR) shows bilateral
air space shadowing throughout both lung fields. Urinalysis
demonstrates red blood cells and red blood cell casts. Her
haemoglobin (Hb) is 6 g/dl and her urea is 45 mmol/l, with a
creatinine of 800 mol/. What is the most likely diagnosis?
1. IgA nephropathy
2. Goodpasture's syndrome
3. Antineutrophil cytoplasmic antibody (ANCA)-positive
vasculitis
4. Cryoglobulinemia
5. Uric acid nephropathy
True 3. This is a classic history for Wegener's granulomatosis.
She would need treatment with dialysis; immunosuppression
and most centres would advocate plasma exchange with a
creatinine above 400 mol/l. None of the other diagnoses would
classically cause sinus or joint involvement, and pulmonary
haemorrhage is typically seen only in Goodpasture's syndrome
and ANCA-positive vasculitis. Note (LVF) can lead to pink
sputum and may be confused for pulmonary haemorrhage. To
confirm the diagnosis of pulmonary haemorrhage, a raised
potassium channel opening (K+-CO) would be seen on lung
function tests. The low haemoglobin (Hb) in the setting of acute
renal failure and the disproportionate degree of abnormality
seen on chest X-ray (CXR) compared to clinical findings
suggest pulmonary haemorrhage rather than infection or fluid
to explain the CXR findings.
26-A 25-year-old man presents with right-sided
facial weakness and swelling. There is a right
LMN facial nerve palsy but no other
abnormalities on examination. Lumbar puncture
findings are: Opening pressure18 cm, CSF
Protein 0.9 g/l Glucose3.5 mmol/l, Microscopy
85 lymphocytes. What is the most likely
diagnosis?
A. Lyme disease
B. Multiple sclerosis (MS)
C. Guillain Barre syndrome (GBS)
D. Neurosarcoidosis
E. Ramsey Hunt syndrome
An 81-year-old female presents with bilaterally
painful knees. There was no history of
gastrointestinal diseases. On examination she
had crepitus but had a full range of movement of
both knees. Which one of the following is an
appropriate initial treatment for her painful
knees?
1 )Dihydrocodeine
2 )Naproxen
3 )Paracetamol
4 )Celecoxib
5 )Topical Diclofenac
Comments: true 3.
This woman has osteoarthritis (OA) of both knees. The
principle goal of systemic therapy is to provide an
effective pain relief with the least associated toxicity.
Paracetamol is the initial therapy recommended for the
treatment of OA of the hip & knee. Studies have shown
that short-term & long-term efficacy of paracetamol is
comparable to that of ibuprofen & naproxen in people
with knee osteoarthritis. Specific COX-2 inhibitors such
as celecoxib have clinical benefit similar to that of
traditional NSAIDS, but less GI toxicity although issues
remain regarding their cardiovascualr risk. They may be
used in patients with GI intolerance of traditional
NSAIDs.
A 50 year-old male epileptic presents with paraesthesia of
hands and feet. He also has unsteadiness when walking.
On examination he has a peripheral sensory neuropathy
and palpable lymph nodes in his neck and axillae.
Which of the following drugs is the most likely cause of
these features?
1) Carbamazepine.
2) Clonazepam
3) Lamotrigine.
4) Phenytoin.
5) Sodium valproate.
The answer is 4
Phenytoin is well known to cause
neurological side effects such as
peripheral sensory neuropathy and
cerebellar ataxia. Other side effects
include gingival hypertrophy,
lymphadenopathy hypocalcaemia,
hirsutism.
18. A 45-year-old male attends for an insurance & is
in good health. Examination was normal but
investigations reveal that he has a serum urate
concentration of 0.55 mmol/l (NR 0.25-0.45). Which of
the following is the most appropriate management of
this patient?
1 )Lifestyle advice.
2 )Start Allopurinol .
3 )Start Colchicine.
4 )Start Diclofenac.
5 )Start Prednisolone.

.
A 30-year-old woman presented with a deep vein
thrombosis. Her previous history included
investigation for infertility. Investigations
revealed: Haemoglobin 12.8 g/dl (12.5-16.5),
White cell count 3.6 x 109/L (4-11), Platelet
count 35 x 109/L (150-400). Select one of
following investigations which are most likely to
be abnormal?
1 )Antiphospholipid antibodies.
2 )Homocystine concentration
3 )Platelet function test
4 )Protein C concentration.
5 )Indium-labelled white cell scan.
A 30 year old of average height and weight presents with
polyuria and thirst. He has a blood glucose of 15 mmol/l.
There is no ketonuria and his older sister is diabetic
How should he be treated?
A. Start metformin
B. A fasting blood glucose should be sent before
treatment
C. Subcutaneous insulin should be started
D. Commence on gliclazide and reassessment with
BM monitoring at home
E. Dietary advice, review in a month with repeat
glucose without any treatment
Answer: d) commence on gliclazide and
reassessment with BM monitoring at
home. He is a type 1 diabetic but there are
no features of ketonuria or acidosis. He
may have some residual islet cell
function and hence sulphonylureas may
help to stimulate insulin production.
A 62-year-old man presented with difficult walking. He had a past
history of diabetes mellitus and cervical spondylosis, which had
required surgical decompression eight years previously. He drank
40 units of alcohol weekly. On examination there was fasciculation,
wasting and weakness in the left deltoid and biceps, with weakness
in the shoulder girdle muscles bilaterally. There was fasciculation in
the glutei and quadriceps bilaterally, weakness of hip flexion and
foot dorsiflexion, brisk reflexes in upper and lower limbs, and
extensor plantar responses. There was no sensory impairment.
What is the diagnosis?
1) alcoholic myopathy
2) diabetic amyotrophy
3) motor neurone disease
4) recurrent cervical cord compression
5) syringomyelia
The answer is 3
There are signs of lower (wasting, fasciculations)
and upper (brisk reflexes, extensor plantar
response) motor neuron involvement in the
presence of normal sensation. Motor neuron
disease is the commonest cause of such
presentation. Alcoholic myopathy and diabetic
amyotrophy do not share upper motor neuron
signs. Syringomyelia presents with sensory
symptoms and signs (spinothalamic). You
expect sensory involvement with cervical cord
compression.
78 years old man presents with an acute onset
of severe pain & swelling of the left wrist which
had developed since she had a chest infection
two weeks previously. On examination, he had a
temperature of 38 C,left wrist was red, swollen
& painful.
What is the appropriate investigation for this pt?
1)Erythrocyte sedimentation rate.
2 )Full bl. count .
3 )Joint aspiration.
4 )Serum urate concentration.
5 )X ray of joint.
A 30-year-old woman is evaluated in the
endocrinology clinic for increased urine output. She
weighs 60 kg and has a 24-hour urine output of
3500 ml. Her basal urine osmolality is 210
mOsm/kg. She undergoes a fluid deprivation test
and her urine osmolality after fluid deprivation (loss
of weight 3 kg) is 350 mOsm/kg. A subsequent
injection of subcutaneous DDAVP (desmopressin
acetate) did not result in a further significant rise of
urine osmolality after 2 hours (355 mOsm/kg).
Which of the following is the likely diagnosis?
A. Normal
B. Primary polydipsia
C. Osmotic diuresis
D. Pituitary diabetes insipidus
E. Nephrogenic diabetes insipidus
A 60 years old woman diagnosed with giant
cell arteritis was commenced on high dose
prednisolone therapy. What is the best
appropriate treatment for prevention of
steroid- induced osteoporosis?
1 )Bisphosphonate therapy
2 )Calcium vitamin D
3 )Hormone replacement therapy
4 )Raloxifene.
5 )Salmon Calcitonin.
A 19 year old girl presents at the antenatal clinic. She is
approximately six weeks pregnant and the pregnancy
was unplanned. She has a two year history of grand mal
epilepsy for which she takes carbamazepine. She has
had no fits for approximately six months. She wants to
continue with her pregnancy if it is safe to do so. She is
worried about her anticonvulsant therapy and the effects
on the baby and enquires how she should be managed?
1) Advise termination due to drug teratogenicity
2) Continue with carbamazepine
3) Stop carbamazepine until the second trimester
4) Switch therapy to phenytoin
5) Switch therapy to sodium valproate
The answer is 2
The patient and fetus are at far more risk from uncontrolled seizures
than from any potential teratogenic effect of the therapy. In
pregnancy total plasma concentrations of anticonvulsants fall and so
the dose may need to be increased. The potential teratogenic
effects (particularly neural tube defects) of carbamazepine do need
to be explained and in an effort to reduce this risk she should
receive folate supplements. Screening with AFP and second
trimester ultrasound are required. Vitamin K should be given to the
mother prior to delivery. There is no point in switching therapies as
this could precipitate seizures in an otherwise stable patient.
Similarly, both phenytoin and valproate are again associated with
teratogenic effects.
Which of the following investigations best
supports a diagnosis of new variant CJD:
1) CSF analysis
2) CT brain
3) EEG
4) EMG
5) MRI brain
The answer is 5
MRI brain typically shows bilateral posterior
thalamic high signal abnormalities in
patient with new variant CJD. EEG , CSF
analysis only shows non-specific changes.
EMG and CT brain are normal. Sporadic
CJD (and not new variant CJD) is
associated with specific EEG changes.
A 65 year old lady has ischaemic cardiomyopathy and
symptoms of breathlessness walking up one flight of
steps. Her breath sounds are clear. Chest X ray shows
cardiomegaly and clear lung fields. She is currently on
frusemide 40mg bd and perindopril 4 mg at night. What
medication should be added?
A. Spironolactone
B. Carvedilol
C. Digoxin
D. Amiodarone
E. Diltiazem
Answer: b) carvedilol. The two best options are
carvedilol and spironolactone. Both B blocker
trials (CIBIS II, Merit HF, Copernicus) and
spironolactone trials (RALES) have shown
symptomatic improvement and decreased
mortality. In this patient with little signs of fluid
overload, a beta blocker can be started first, and
then spironolactone added as well.
A 33 year old epileptic female presents with visual
problems. Examination reveals a constriction of
visual fields to confrontation. Which of the
following may be responsible for her visual
deterioration?
1) Vigabatrin
2) Lamotrigine
3) Gabapentin
4) Phenytoin
5) Sodium Valproate
The answer is 1
Vigabatrin is associated with constricted
visual fields and when detected therapy
should be stopped.
A 45-year-old attends clinic complaining of
tiredness. She is hypothyroid & takes
thyroxine 150 micrograms daily. Which of
following useful test assessing
appropriateness of thyroid hormone
replacement in primary hypothyroidism?
1)Free T3 & T4 concentrations
2 )Skin biopsy
3 )Thyroid binding globulin
4 )Total T3 T4
5 )TSH
A 52 year old lady has palpitations. Her ECG
shows a broad complex tachycardia. Which of
these features suggests that the tachycardia is
more likely to be of ventricular origin?
A. QRS of 150 ms
B. Left bundle branch block and left axis
deviation
C. P wave for every QRS complex
D. History of atrial fibrillation
E. Heart rate of 150
Answer: a) QRS of 150ms. Features that
favour VT are : QRS of > 140ms,
dissociated p waves, history of ischaemic
heart disease, right bundle branch block
with left axis deviation, HR >170 beats per
minute.
A 59 year lady has a history of hypertension and
has recently been prescribed a new
antihypertensive agent. She now has fevers
and joint pains in the arms, shoulders, knees
and ankles. Which of the following drugs could
cause this?
A. Bendrofluazide
B. B. deltiazem
C. C. Methyldopa
D. D. Amlodipine
E. E. Lisinopril
Answer: c) methyldopa. The history would be
consistent with drug induced SLE. In drug-
induced lupus erythematosus, the features of
arthritis, systemic symptoms, and cardiac and
pulmonary (lung) symptoms may be present.
Other symptoms associated with SLE, such as
lupus nephritis and neurological disease, are
rare. The drugs which may cause this are
procainamide, isoniazid, chlorpromazine,
penicillamine, sulfasalazine, hydralazine,
methyldopa, and quinidine.
A 55 year old man has bitemporal hemianopia. He
also has elevated IGF-1 levels in the serum and an
enlarged pituitary seen on MRI. Which one of the
following is an associated feature?
A. Hypolipidemia
B. Hypoglycaemia
C. Rheumatoid arthritis
D. Myocarditis
E. Hypertension
Answer: e) hypertension. The diagnosis is
acromegaly. Carpal tunnel syndrome,
impaired glucose tolerance, carpal tunnel
syndrome and high cardiac output cardiac
failure are associated.

MRI showing a pituitary tumour in


acromegaly
An 18 year-old girl receives radioactive iodine
as treatment of thyrotoxicosis. Which of
following is likely long-term complication of
this treatment?
1)hypoparathyroidism.
2 )hypothyroidism.
3 )increased risk of developing cancer.
4 )recurrent laryngeal nerve damage.
5 )osteoporosis.
Which of following percentages
accurately reflects mortality associated
with modern management of diabetic
ketoacidosis?
1 )0.5%
2 )1%
3 )2-3%
4 )5-6%
5 )8-10%
A 30 year old lady has recurrent attacks of
dizziness and blackouts. A 72 hour fast
reveals periods where her plasma glucose is
2.5 mmol/l with elevated insulin and C
peptide levels. What is the next best
investigation?
A. Repeat 72 hour fast
B. B. Glucagon stimulation test
C. C. Glucose tolerance test
D. D. MRI of abdomen
E. E. Insulin antibodies
Answer: d) MRI of abdomen. The tests so
far suggest an insulinoma. Localisation of
the insulinoma can be done with MRI, CT,
superior mesenteric angiography or
pancreatic venous catheterisation.

Insulinoma
A 35 year old lady has a history of diarrhoea,
profuse sweating and irritability. Her parents
had similar symptoms and passed away in
their fourties after doing radical surgery .
Which one of the following is the most likely
diagnosis?
A. Phaeochromocytoma
B. Acromegaly
C. Diabetes
D. Ovarian carcinoma
E. Medullary thyroid carcinoma
Answer: e) medullary thyroid carcinoma.
Medullary thyroid carcinoma produces
peptides and neurohormones which lead
to symptoms of irritability, diarrhoea and
sweating. It can be inherited in association
with MEN type II.
A young woman has acne and is taking oral medication. She develops
polyarthritis and raised liver enzyme tests. Investigations show
AST 95
ALT 170
bilirubin 16
antinuclear antibodies strongly positive at 1/640, Which of the
following drugs is she most likely to have been prescribed?
1) erythromycin
2) isotretinoin
3) minocycline
4) oxytetracycline
5) trimethoprim
The answer is 3
Except trimethoprim all other drugs are used
in the treatment of acne. And all of these
can cause hepatotoxicity. Erythromycin
usually causes cholestasis. Minocycline
can cause drug induced SLE.
Which of the following features is
characteristic of early Alzheimer's
disease?
1) ataxic gait
2) impaired short term memory
3) myoclonic jerks
4) urinary incontinence
5) visual hallucinations
The answer is 2
Alzheimer's disease is characterised early in the
disease by short term memory loss. The other
features listed here would suggest an alternative
diagnosis such as normal pressure
hydrocephalus (gait ataxia and urinary
incontinence), Creutzfeld-Jacob disease
(myoclonic jerks) and delirium or vascular
dementia (visual hallucinations).
A 45-year-old man presents with an insidious onset of
binocular horizontal diplopia and left sided facial pain.
On examination ha has a left abducens nerve palsy and
numbness over the maxillary division of the left
trigeminal nerve. The most likely anatomical site of his
neurological lesion is:
1) Cavernous sinus
2) Petrous apex
3) Superior orbital fissure
4) Cerebellopontine angle
5) Midbrain
The answer is 2
In the pre-antibiotic era an abducens nerve palsy with
ipsilateral pain and numbness was due to petrous
osteitis (Gradenigo syndrome) but is now more likely the
result of a meningioma or nasopharyngeal carcinoma of
the petrous apex. The cavernous sinus syndrome
consists of variable involvement of: oculomotor,
trochlear, abducens, trigeminal (ophthalmic and
maxillary division) and oculo-sympathetic nerves. The
superior orbital fissure syndrome is similar to the
cavernous sinus syndrome except for the presence of
proptosis.
2. A 75-year-old male was admitted with chest pain &
dyspnoea. His pain subsides, he is generally well,
although dyspnoea restricts his mobilisation. He is
unable manage the stairs, but can mobilise solely
around the ward. Whilst being monitored his
telemetry demonstrates short runs of non sustained
ventricular tacchycardia, associated light-
headedness.
What is the next appropriate investigation for this
patient?
1 )24hr tape
2 )Coronary angiography
3 )Echocardiography
4 )Electrophysiological studies
5 )Outpatient Cardiology referral
Comments: 3 )Echocardiography
patients with reduced LV function or
asymptomatic VT may benefit from
implantation of an implantable cardiac
defibrillator . You need to know the
patients cardiac function, so we feel that
an ECHO would be the next appropriate
study in this patient, followed by coronary
angiography & inpatient cardiology
referral.
A 55-year-old female receiving 10 mg of Methotrexate & 5mg
of folate* weekly presents with a sore right finger after cutting
herself in the garden. On examination, she has a swollen,
erythematosus right ring finger up to the proximal
interphalangeal joint & you diagnose a cellulitis. You give her
a prescription of erythromycin as she is allergic to penicillins.
She has being receiving the Methotrexate over just one year
with no problems in all routine bl monitoring which are
normal.
Whilst monitoring the response of the infection to treatment,
what appropriate strategy regarding her Methotrexate
therapy?
1 )Continue Methotrexate unchanged & increase folate
supplements to 10mg daily.
2 )Continue Methotrexate & folate unchanged.
3 )Reduce dose of Methotrexate to 5mg weekly
4 )Stop Methorexate until the infection is resolved.
5 )Stop Methotrexate only if full bl. count reveals a neutropaenia.
Comments:
true 4. In the circumstances of infection, one should
consider temporarily stopping methotrexate as it is an
immunosuppressant. Any infection should be treated as
usual to response to treatment monitored. Once the
infection is successfully treated methotrexate can be
reinstated. However, if the patient has recurrent serious
infections while taking methotrexate, its continued long
term use should be discussed with the patient's
rheumatologist. *some local variations may exist
regarding dose & frequency of folate therapy.
14.A 26-year-old man with a history of alcohol &
drug abuse was admitted with a 14 day history
of fever, cough & fatigue. He was emaciated.
His temperature was 39.4C. Cervical & axillary
lymphadenopathy were present. Chest X-ray
showed bilateral areas of pulmonary
shadowing. Which of the following is the likely
diagnosis?
1 )alcoholic cardiomyopathy
2 )pneumococcal pneumonia
3 )pneumocystis pneumonia
4 )pulmonary tuberculosis
5 )tricuspid endocarditis
A 45 year old woman with breathlessness
presents for further investigation. She has a
known history of rheumatoid arthritis and is
on methotrexate and folate. She has a CXR
and lung function tests. Which one of the
following is a recognized respiratory
manifestation of rheumatoid arthritis?
A. Mesothelioma
B. B. Asthma
C. C. Bronchiolitis obliterans
D. D. Pulmonary eosinophilia
E. E. Pulmonary embolus
Answer: c) bronchiolitis obliterans. In
rheumatoid arthritis, exudative pleural
effusions, fibrotic lung disease, Caplans
syndrome (pneumoconiosis, pulmonary
nodules), and obstructive lung disease in the
form of bronchiolitis obliterans (obstruction of
bronchiolar lumen with inflammatory exudate)
may occur. Pulmonary eosinophilia may be
found in Churg Strauss or Wegeners vasculitis.
A 55 year old female has been on long-term steroids for
chronic obstructive pulmonary disease. She complains of
pain in her right groin radiating down the anteromedial
thigh. On examination of the hip, there is decreased
range of movement especially flexion, abduction and
internal rotation. What is the likely diagnosis?
A. Osteoarthritis
B. Rheumatoid arthritis
C. Metastatic hip lesion
D. Avascular necrosis of the femoral head
E. Hairline fracture
Answer: d) avascular necrosis of the femoral head.
In a patient on long term steroids presenting with groin
pains radiating to the thigh associated with an antalgic
gait and decreased range of movement of the hip, the
most likely diagnosis is avascular necrosis of the
femoral head.
In this condition, MRI is the most sensitive and specific
technique and is useful for early diagnosis before
collapse of bone occurs. CT scan and x-ray are useful to
rule out advanced disease if duration is not clear. Bone
scanning is more sensitive than x-ray but is non-specific.

MRI showing avascular necrosis of the femoral head


A patient who is 8 months pregnant has a
thyroid function test because she has a smooth
palpable goitre. Her Free T4 is 20 pmol/l and
TSH is 5 mU/l. Which of these diagnoses is most
likely?
A. Medullary thyroid carcinoma
B. Multinodular goitre
C. Grave's disease
D. Thyroid adenoma
E. Colloid goitre
Answer: e) colloid goitre. A colloid goitre is most
likely. Presentation is with mildly elevated TSH
(0.3-4.0 mU/l) and normal free T4. Mild iodine
deficiency or a pregnancy can cause the goitre
to become more noticeable.

Colloid Goitre -increased size of follicles and


flattening of follicular epithelial cells
A 55 year old patient with type 2 diabetes
is reviewed in the diabetic clinic. Which
of the following is a feature of diabetic
neuropathy to watch out for?
A. Cervical myelopathy
B. B. Brisk reflexes
C. C. Muscle hypertrophy
D. D. Loss of vibration sense
E. E. Myotonia
Answer: d) loss of vibration sense. Autonomic
neuropathy to the gut, bladder and sexual
organs (impotence) can occur. A 3rd nerve
mononeuropathy can occur. Motor neuropathy
can cause muscle wasting, and sensory
neuropathy causes vibration sensory loss. With
myotonia, prolonged contraction of muscle fibres
associated with muscle dystrophy is due to
genetic causes of muscle protein abnormality
and is not neurologically dependent.
A 40 year old lady has a pulmonary systolic
murmur which is louder on inspiration. She
also has a right ventricular heave. The second
heart sound is wide and fixed with splitting.
What is the likely diagnosis?
A. Mitral stenosis
B. Aortic stenosis
C. Atrial septal defect
D. Dilated cardiomyopathy
E. Hypertrophic cardiomyopathy
Answer: c) atrial septal defect.
In Atrial Septal Defect (ASD), a Primum defect
causes RBBB and LAD, whilst Secundum
causes RBBB and RAD on the ECG. A systolic
murmur is heard in the pulmonary area because
of increased pulmonary valve flow due to
pulmonary hypertension. Similarly, a left
parasternal heave is present due to RVH.
ECG showing RBBB and LAD
A 75 year old man with known moderate to severe
aortic stenosis presents to hospital with
breathlessness. Which of the following
features would indicate poor prognosis without
urgent treatment?
A.Early diastolic murmur
B. Displaced apex beat
C. Pulmonary oedema
D. Aortic valve calcification seen on X ray
E. Breathlessness
Answer: c) pulmonary oedema. Pulmonary
oedema and angina can occur with aortic
stenosis, these are symptoms which
indicate urgent assessment should be
made with a view towards surgery.
A 45 year old man has polyuria and abdominal
pains. His calcium is 2.9 mmol/l, phosphate 0.8
mmol/l and PTH 12 pmol/l. Which one of the
following would suggest that he has primary
hyperparathyroidism?
A. Serum alkaline phosphatase is increased
B. Normal skull
C. Arthritis
D. Low urinary phosphate excretion
E. Low urinary cAMP
Answer: a) serum alkaline phosphatase is increased.
Serum alkaline phosphatase is usually increased in
primary hyperparathyroidism.
Primary hyperparathyroidism does not respond to
steroids.
Salt and pepper changes on the skull can be a hallmark.
Gout, rather than arthritis, is worsened by
hyperparathyroidism.
High urinary phosphate excretion leads to serum
phosphate and a high cAMP in the urine suggests active
excretion.
A 40 year old lady has increasing hirsutism. She
is embarassed about having to shave her chin
and also her chest. Her voice is becoming
deeper. What is the most likely diagnosis?
A. Drug induced hirsutism .
B. Polycystic ovarian syndrome.
C. Adrenal tumour.
D. Congenital adrenal hyperplasia.
E. Ovarian carcinoma .
Answer: c) adrenal tumour. Rapid
development of hirsutism is usually
caused by an adrenal tumour. There are
high testosterone or DHEA levels in the
plasma.

MRI showing an adrenal tumour


A 60 year old man has worsening joint stiffness,
hip and back pains. He has spinal and hip X rays
for assessment. Which one of the following is a
recognised X ray change of osteoarthritis?
A. Osteitis fibrosaB.
B.Lytic lesions
C. Fibrosis
D. Subchondral cysts
E. Hairline fracture
Answer: d) subchondral cysts.
Osteophytes, loss of joint space,
subchondral sclerosis and subchondral
cysts are common in osteoarthritis.
A 55 year old lady has abdominal pains. She
has polyuria and polydipsia. Her free T4 is 16
pmol/l and TSH is 3.0 mU/l. Which of the
following is most likely?
A. Subclinical hypothyroidism
B. Renal tubular acidosis
C. MEN 1
D. Diabetes
E. Primary hyperparathyroidism
Answer: e) primary hyperparathyroidism.
Hypercalcaemia can cause abdominal
pain, polyuria and polydipsia. There are no
features suggestive of pituitary tumour,
phaeochromocytoma or medullary thyroid
involvement, hence MEN is unlikely.
A 10 year old child with joint pains is suspected
of having juvenile chronic arthritis. A full
rheumatological history is taken. Which of the
following is a recognised presenting feature of
juvenile chronic arthritis?
A. Low grade fever
B. Transient purpuric rash
C. Haematuria
D. Involvement of one or two large joints only
E. Ulcerative colitis
Answer: d) involvement of one or two large joints
only. Juvenile chronic arthritis is rheumatoid
factor negative. Commonest type is Stills
disease. Peaks of disease are about 5 years and
15 years of age. A high swinging fever is typical.
An erythematous rash can occur, but it is non
purpuric. Haematuria is not typical. Certain
forms can involve only one or two large joints
only. Different classifications are systemic,
pauciarticular and polyarticular. There is no
relation with ulcerative colitis.
La boratory evaluation of a 19-year-old male who is being worked up for
polyuria and polydipsia yields the following results:
S. Na+ 144, K+ 4.0, Cl 107, HCO3 25 BUN: 6.4 mmol/L.
Bl. gl: 5.7 mmol/L , Urine electrolytes (mmol/L): Na+ 28, K+ 32 Urine
osmolality: 195 mosmol/kg water After 12 h of fluid deprivation, body weight
has fallen by 5%. Laboratory testing now reveals the following:
Serum electrolytes (meq/L): Na+ 150, K+ 4.1, Cl 109, HCO3 25 BUN: 7.1
mmol/L (20 mg/dL) (98 mg/dL) Urine electrolyte : Na+ 24, K+ 35 Urine
osmolality: 200 mosmol/kg water One hour after the subcutaneous
administration of 5 units of arginine vasopressin urine values are as follows:
Urine electrolytes (meq/L): Na+ 30, K+ 30 Urine osmolality: 199 mosm
The likely diagnosis is
A. nephrogenic diabetes insipidus
B. osmotic diuresis
C. salt-losing nephropathy
D. psychogenic polydipsia
E. none of the above
Slide no 6
This female patient sufferred convuslions in childhood.
She experienced several attacks of hematuria recenly.
Proteinuria was less thn 1gm/24hrs and her RFTs
were normal.
1. What is her condition.
2. What is the most likely lesion of her CT scan
A 50 year man presents with lightheadness. He has
frequent nonsustained ventricular tachycardia on the
ECG and cardiac monitor. His bloods show a Hb 13.0
g/dl, WCC 7 x 10^9/l, platelets 230 x 10^9/l, urea
11mol/l, creatinine 80mol/l, sodium 134 mmol/l,
potassium 3.2 mmol/l, serum magnesium of 0.6 mmol/l
(0.75). Which one of following is likely to be responsible
for his arrhythmias?
A. Poor diet
B. Alcoholism
C. Frusemide
D. Diarrhoea
E. Hyperphosphataemia
Answer: c) frusemide. The likely cause of
the arrhythmias is hypomagnesaemia and
hypokalaemia, which is most commonly
associated with diuretic use.
A 38 year old lady presents with myalgia and
lethargy. Her blood tests show a positive
ANA with a titre of 1:1024 and rheumatoid
factor is negative. The CK is raised at 360
U/l. Extranuclear antigen tests show a
negative Ro and negative La, negative Scl70
and positive ribonuclear protein antibody at
160 units. What is the likely diagnosis?
A.Polymyalgia rheumatica.
B. Polymyositis.
C. Scleroderma.
D. Systemic lupus erythematosus .
E. Mixed connective tissue disease.
Answer: e) mixed connective tissue
disease. A positive ANA (speckled
pattern), raised CK and positive anti RNP
antibody suggests mixed connective
tissue disease.
A 29-year-old banker is said to have gone camping
three weeks ago. He reports having developed a skin
rash around the groin area. It began as a papule but
went on to become a macule with a bright outer
border and a clear centre. It was warm but not
painful. The macule is disappearing, but he has since
developed severe headache, mild neck stiffness,
fever chills, pains which were once in the knees but
are now in the ankles and elbows. He also has
malaise and fatigue.
What is the most probable diagnosis?
Infective endocarditis
Lyme disease
Meningococcal meningitis
Q fever
Rheumatic fever
A 40 year old man has genital ulceration and
uveitis. his GP suspects Behcet's syndrome
and is referred to the rheumatologist. Which
one of the following is a feature of Behcets
syndrome?
A. Facial asymmetry
B. Receding hair line
C. Hirsutism
D. Malignant melanoma
E. Arterial thrombosis
Answer: e) arterial thrombosis. Behcet's
syndrome is an inflammatory disorder
(associated with certain HLA B and DR types)
causing mouth ulceration, arthritis, eyes
(anterior uveitis, retinal vein occlusion), vasculitis
(thrombophlebitis) and thrombosis. CNS
vasculitis involvement may lead to TIA,
meningoencephalitis, parkinsons and dementia.
A 42-year-old male a 15 year history of type 1
diabetes presents a two month history of
deteriorating pain & stiffness of the right shoulder.
On examination he has painful limitation of internal
rotation but can abduct right arm only 90 degrees.
Flexion is relatively unimpaired. There some
weakness of movement of shoulder slight wasting
of shoulder muscles. He has some reduced
vibration sensation in both hands.
Which of following is likely diagnosis?
1 )Adhesive capsulitis
2 )Brachial plexopathy
3 )Calcium pyrophosphate arthropathy
4 )Diabetic arthropathy
5 )Rheumatoid arthritis
A 70 year old lady presents with bilateral shoulder
and neck pains which have been present for
several years. She also feels constantly
lethargic. Her ESR is 105 mm/hour. What is
the likely diagnosis?
A. Fibromyalgia
B. Supraspinatus tendinitis
C. Polymyalgia rheumatica
D. Temporal arteritis
E. Cushing's syndrome
Answer: c) Polymyalgia rheumatica. With
a raised ESR, polymyalgia rheumatica is
far more likely than fibromyalgia. Temporal
arteritis should always be considered
especially if a headache is present.
A 50-year-old male is admitted to the hospital with
pneumonia. He does well after the administration of
antibiotics,but his sodium is noted to rise from 140 to 154
meq/L over 2 days. He reports thirst and has had a urine
output of approximately 5 L per day. Which of the
following is the most appropriate next step to evaluate
the patients disorder?
A. Measurement of serum osmolality
B. Measurement of serum vasopressin level
C. 24-h measurement of urinary sodium
D. Trial of arginine vasopressin
E. Trial of free water restriction
A 55 year old patient presented with breathlessness and ankle oedema. The
blood pressure is 135/80 mmHg. On examination, her JVP rises with
inspiration. She has a soft systolic murmur and a third heart sound.
Blood tests reveal
Hb 10.5 g/dl
WCC 7.5 x 10^9/l
Platelets 150x 10^9/l
sodium 136 mmol/l
potassium 3.5 mmol/l
creatinine 140 mol/l
urea 6 mol/l
ECG shows poor R wave progression. An echocardiogram shows no
pericardial effusion, the ventricles are stiff and systolic function is mildly
impaired.
Which of the following is the likely diagnosis?
A.Restrictive cardiomyopathy
B. Dilated cardiomyopathy
C. Constrictive pericarditis
D. Ischaemic cardiomyopathy
E. Pulmonary embolus
Answer: a) restrictive cardiomyopathy. In this
scenario, there symptoms can be caused by any
form of cardiomyopathy. The rise in JVP with
inspiration suggests either constrictive or restrictive
cardiomyopathy. Echocardiography showing no
pericardial effusion and stiffness suggests restrictive
rather than constrictive cardiomyopathy. The
transmitral dopplers on the echo may show E/A
wave reversal and high velocities which may
suggest restrictive picture. This may be due to
infiltration due to haemochromatosis,
endomyocardial fibrosis, sarcoidosis, myeloma,
lymphoma or connective tissue disease.
A 35 year old female presents with
sweatiness, tremors and palpitations.
Examination reveals a exopthalmos and a
goitre. Her GP requests TFTs which show
a TSH of 0.01mU/l, FT4 35 pmol/l, FT3 3.1
nmol/l. She has positive antithyroid
antibodies. What is the likely diagnosis?
A. Hashimoto's thyroiditis
B. Grave's disease
C. Iodine deficiency
D. Post radioiodine treatment
E. Papillary thyroid carcinoma
Answer: b) Grave's disease.
Graves disease is the diagnosis - thyroid
autoantibodies are increased. Almost 80% of
patients have exopthalmos. Medical treatment
such as carbimazole or radioiodine treatment
are recommended rather than surgery. There
will be increased uptake on the thyroid
radioisotope scan.

Exopthalmos in Grave's disease


A 68-year-old female is referred to outpatients with a
three-month history of dyspnoea and significant
peripheral oedema. Clinical findings are solely of
the oedema. Echocardiography demonstrates
thickened myocardium with impaired relaxation and
a bright speckled appearance.
What is the likely diagnosis?
A : Hypertrophic cardiomyopathy.
B : Cor pulmonale.
C : Sarcoidosis.
D : Amyloidosis.
E : Secondary neoplastic myocardial deposition.
A 44-year-old woman presents to her GP for review.
She has attended the GP on a number of
occasions during the past year for sinusitis and is
now concerned that the bridge of her nose has
collapsed and that she may require cosmetic
surgery. Chest X-ray is abnormal and reveals
multiple nodules. Routine blood testing reveals a
creatinine of 205 mol/l. Renal biopsy reveals a
necrotising microvascular glomerulonephritis.
Which of the following is the best initial
treatment for this condition?
1. Corticosteroids
2. Cyclophosphamide
3. Corticosteroids and cyclophosphamide.
4. Azathioprine
5. Methotrexate
True 3. The clinical history is highly suggestive
of Wegeners Granulomatosis. Often patients
present with severe rhinorrhoea, complicated
by nasal ulceration and later cough with
haemoptysis. Chest X-ray reveals nodular
changes +/ pneumonic features. Renal
involvement is characterised by
microvascular glomerulonephritis. Treatment
is with cyclophosphamide in combination
with corticosteroids in patients with severe
disease.
A 45-year-old woman presented with a 1-year history of
intermittent swelling and pain in the small joints of her
hands. Maximal stiffness occurs on waking and eases
after an hour. On examination rheumatoid nodules on
the elbows and symmetrical soft tissue swelling over the
PIP and MCP joints were noted. Effusions of both wrists
were also noted. She is rheumatoid factor positive.
Radiological examination of the wrist and hands
showed erosions and bony decalcification. Her GP has
commenced her on NSAIDs and she been referred to a
rheumatologist for consideration of DMARD. She has a
past history of TB.
Which of the following is best avoided if possible?
1. Methotrexate
2. Chloroquine
3. Sulfasalazine
4. Infliximab
5. leflunomide
True 4. Anti-TNF-a is an effective
treatment for rheumatoid arthritis (RA).
It has been shown to give a sustained
clinical response over 2 years. However,
it can reactivate old TB and should be
used with caution in these patients.
A patient with a history of Sj grens syndrome has
the following laboratory findings: plasma sodium 139
meq/L, chloride 112 meq/L, bicarbonate 15 meq/L, and
potassium 3.0 meq/L; urine studies show a pH of 6.0,
sodium
of 15 meq/L, potassium of 10 meq/L, and chloride
of 12 meq/L. The most likely diagnosis is
A. type I renal tubular acidosis (RTA)
B. type II RTA
C. type III RTA
D. type IV RTA
E. chronic diarrhea
72-year-old male presents a 2 month history of
weight loss & weakness. Examination reveals a
BMI of 24.5 kg/m2 a pressure of 186/100 mmHg.
Examination of lower limbs reveals a bilateral
weakness of knee extension. He is unable to rise
from squatting position. There absence of knee
reflex but ankle reflexes are preserved both
plantars are flexor. There are no abnormalities on
sensory examination.
Which of following tests may be diagnostic?
1 )Vitamin B12 concentration
2 )Thyroid function test
3 )Oral glucose tolerance test
4 )Urine free cortisol concentration
5 )Vitamin D concentration
Which of the following would be correct in
keeping with a diagnosis of ploymyalgia
rheumatica?
1 )raised creatinine kinase
2 )increased alkaline phosphatase
3 )sudden loss of vision in one eye
4 )shoulder & pelvic girdle pain in 35-year-
old man
5 )erythema nodosum
Correct answer 2
A 45-year-old woman admitted a spiking temperature
sweats. She unwell last 3 weeks flitting arthralgia
lethargy. There a rash over her trunk which prevalent
in mornings. Blood cultures are sterile. Her recent
transthoracic echocardiogram normal. ESR
56mm/hour. Her ferritin elevated at 6000(g/l.
Autoimmune screen negative.
1 )bacterial endocarditis.
2 )systemic lupus erythematosus .
3 )rheumatoid arthritis
4 )adult onset Stills disease .
5 )meningitis .
A 52-year-old schoolteacher attends for weight loss & sweats.
Subsequent investigations show:
Free T4 of 40 pmol/L (9-23)
Free T3 ofs 9.8 nmol/L (3.5-6)
TSH of 7.0 mU/L (0.5-5)
She is clinically thyrotoxic a diffuse goitre & commenced on
Propranolol & Carbimazole.
What is appropriate test this patient?
1 )FNA of thyroid gland
]2 )MRI scan pituitary gland
3 )Radio-isotope uptake scan of thyroid gland
4 )Repeat TFT
5 )Thyroid auto antibodies
A 75-year-old man presents with 12 months history
of cognitive impairment, parkinsonism, intermittent
confusion and generalised myoclonus. He was
started on 62.5 tds of sinemet. In the following 2
months he was started experiencing visual
hallucinations.
The most likely diagnosis is:
1) Idiopathic Parkinson's disease
2) Alzheimer's disease
3) Diffuse Lewy body disease
4) Multiple system atrophy
5) Progressive supranuclear palsy
The answer is 3
Diffuse lewy body disease presents with cognitive
impairment, visual hallucinations, intermittent
confusion, parkinsonism, myoclonus and
marked sensitivity to neuroleptic treatment.
Visual hallucinations in parkinson's disease
treated with L-dopa usually appear late (>2
years after initiation of treatment). Visual
hallucinations are not features of multiple system
atrophy or progressive supranuclear palsy.
A female patient aged 30 has a 5 years history of difficulty
getting upstairs and out of a low chair and mild upper
limb weakness but no pain. There is no family history.
She presented with severe type 2 respiratory failure.
EMG showed evidence of myopathy. The most likely
diagnosis is:
1) Polymyositis
2) Inclusion body myositis
3) Acid Maltase Deficiency
4) Miller-Fisher Syndrome
5) Lambert-Eaton Myasthenic Syndrome
The answer is 3
Acid maltase deficiency typically presents with insidious
onset of proximal myopathy and early respiratory muscle
weakness. Respiratory failure in inflammatory
myopathies (polymyositis, dermatomyositis, inclusion
body myositis) and limb girdle muscular dystrophy is
rare. Muscle biopsy shows vacuolation in muscle fibres.
Miller-Fisher Syndrome, a variant of GBS, is
characterised by ophthalmoplegia, ataxia and areflexia.
Lambert-Eaton Myasthenic Syndrome, often a
paraneoplastic phenomenon, is associated with
hyporeflexia which returns after exercise, autonomic
symptoms and fatiguability.
Which statement is true regarding Gabapentin?
1) is a potent hepatic enzyme inducer
2) side effects typically include visual field defects
with long-term use
3) therapy is best monitored through measuring
plasma concentrations
4) is of particular value as monotherapy in
absence attacks (petit mal)
5) requires dose adjustment in renal disease
The answer is 5
Gabapentin does not induce cytochrome P450
unlike other anticonvulsants such as phenytoin
and phenobarbitone. Vigabatrin may cause
visual field defects, which may be irreversible.
Rarely have visual disturbances been
associated with gabapentin. No use in Petit Mal
and is used for add-on therapy in partial or
generalised seizures.
Which of the following clinical manifestations
suggests Guillain Barr Syndrome?
1) Weakness beginning in the arms
2) Asymmetrical involvement of distal
muscles
3) Bulbar involvement in about 50% of cases
4) Brisk tendon reflexes
5) Normal CSF protein
The answer is 3
GB is a post-infectious polyneuropathy causing demyelination in mainly motor
but also sensory nerves. It usually follows a non-specific viral infection.
Campylobacter and mycoplasma are recognised causes. Weakness begins
in the legs and progressively ascends to involve the trunk, upper limbs and
finally the bulbar muscles (Landry's ascending paralysis). Asymmetry is
present in only 9% of patients, with symmetrical involvement being typical.
Usually there is painless progression over days or weeks, but in cases of
abrupt onset, there may be tenderness or muscle pain. Bulbar involvement
occurs in 50%, with a risk of aspiration and respiratory insufficiency can be
problematic. In the Miller Fisher Syndrome there is external
ophthalmoplegia, ataxia and areflexia. In 20% of cases there is urinary
incontinence of retention. Clinical symptoms usually improve within 2-3
weeks, though a chronic relapsing form is recognised. CSF protein is
elevated to more than twice the upper limit of normal, with normal glucose
and no pleocytosis. Bacterial cultures are negative and viral cultures rarely
isolate anything. The dissociation between a high CSF protein and a lack of
cellular response in a person with an acute or subacute polyneuropathy is
diagnostic of Guillain Barr Syndrome
A 72-year-old lady has 4 months of memory loss,
urinary incontinence and falls. On examination
she has mild memory loss and a broad-based,
slow gait. Muscle tone is normal and both
plantar reflexes are downgoing. What is the
likely diagnosis?
1) Alzheimer's disease
2) Frontal lobe dementia
3) Mulit-infarct dementia
4) Normal-pressure hydrocephalus
5) Parkinson's disease
The answer is 4
Normal pressure hydrocephalus characterized by
abnormal gait, urinary incontinence, and
dementia. It is an important clinical diagnosis,
because it is a potentially reversible cause of
dementia. It is important to distinguish it from
Parkinson's Disease. The onset of gait
disturbance and urinary symptoms is unusual so
early in dementia. Frontal lobe dementia is
characterised by loss of 'executive' functions and
multi-infarct state usually has a step-wise
history.
A patient has, on examination, weakness in
plantar flexion and foot inversion on the left. He
also is unable to tiptoe on the same foot. Ankle
jerk is absent. Which of the following nerve
lesion is most likely?
A. Common peroneal nerve
B. L4 nerve root
C. Tibial nerve
D. Sciatic nerve
E. Femoral nerve
Answer: c) tibial nerve. Tibial nerve
supplies the gastrocnemius muscle and
leads to the above findings. The common
peroneal nerve causes weakness of
eversion and dorsiflexion.
A 25 year old secretary has had several episodes of brief
jerking of the right arm over the past few weeks. There is
no loss of consciousness. A CT scan of the head is
unremarkable. Which is the best medication to
commence?
A. Carbamazepine
B. Phenytoin
C. Lorazepam
D. Diazepam
E. Levodopa

Answer: a) carbamazepine. Brief episodes


of jerking suggests simple partial seizures.
Carbamazepine is first line therapy for this.
A 40 year old patient has a transthoracic
echocardiogram as a follow up. He has a diagnosis
of hypertrophic obstructive cardiomyopathy.
Clinically he has a systolic murmur heard loudest in
the right upper sternal edge. His ECG shows grossly
large QRS complexes with LVH strain pattern. Which
of the following suggests highest risk for sudden
death?
A left ventricular outflow tract gradient of 20
mmHg
B. Tricuspid regurgitation
C. Systolic anterior motion of mitral valve
D. ECG showing ventricular ectopics
E. Interventricular septal thickness of 4 cm
Answer: e) interventricular septal
thickness of 4 cm. The LV outflow tract
gradient is not significantly high in this
patient, however, this may vary with
exercise. The large interventricular septal
thickness of 4 cm (normal <1.3 cm)
suggests very hypertrophic myocardium
and high risk of outflow tract obstruction
with exertion.
A 18 year old man is referred for an
echocardiogram for further investigation of
a systolic murmur. This reveals a bicuspid
aortic valve. Which of the following is an
association?
A. Mitral stenosis
B. Mitral valve prolapse
C. Marfan's syndrome
D. Down's syndrome
E. Coarctation of aorta
Answer: e) coarctation of aorta. Bicuspic
aortic valve can be congenital. 5% of
cases demonstrate significant association
with coarctation of the aorta.

Coarctation of Aorta
A 50 year old woman with rheumatoid arthritis
has hand and spine X rays due to worsening
joint and back pains. Which of the following is
a recognised X ray change of rheumatoid
arthritis?
A. Juxta-articular osteosclerosis
B. Sacroilitis
C. Tendon swelling
D. Marginal erosions
E. Calcification of entheses
Answer: d) marginal erosions. Juxta-articular
osteoporosis or osteosclerosis occurs in OA.
Sacroilitis is seen in seronegative arthritis.
Marginal erosions are seen at the articular
cartilage and attachment of synovium. There is
associated loss of joint space. Calcification of
entheses also occurs in seronegative arthritis.

Marginal Erosions of the first MTP joint


A 35 year old lady has positive ANA,
and has a butterfly shaped rash on
her face. Her physician makes a
diagnosis of SLE. She has flare ups
of joint swellings and pains requiring
several months treatment with
prednisolone. 1 year later she
presents with hip pain limiting her
mobility. Which one of the following is
the likely cause?
A. Rheumatoid arthritis
B. Septic arthritis
C. Juvenile chronic arthritis
D. Avascular necrosis
E. Perthe's disease
Answer: d) avascular necrosis. 15% of
patients with SLE develop avascular
necrosis of the bone. Nephritis, vasculitis
and long term steroid use predispose to
avascular necrosis. Collapse of the
femoral head due to avascular necrosis
A 65 year old man has impaired hearing in his
left ear, an enlarged skull vault, bowing of the
left tibia which is warm to touch. In this
patient, one of the complications of
immobilisation would be:
A. Osteoarthritis
B. Hypercalcaemia
C. Peripheral vascular disease
D. Venous varicosities
E. Disseminated intravascular coagulation
Answer: b) hypercalcaemia. The patient has the
clinical features of Pagets disease of bone. In
this condition immobilisation is likely to cause
hypercalcaemia. Other complications are gout,
high output cardiac failure and coincidental
hyperparathyroidism.

Bowed tibia in Paget's disease


A 60 year old lady has an autosomal dominant
condition. She small red lesions on her face and
mouth. She has been on ferrous sulphate tablets
for chronic iron deficiency anaemia. In view of the
likely diagnosis, which feature may be
associated?
A. Oesophageal dysmotility
B. Pulmonary AV malformation
C. Pulmonary fibrosis
D. Hiatus hernia
E. Aortic aneurysm
Answer: b) pulmonary AV malformation. She has
hereditary haemorrhagic telangiectasia (Osler-
Rendu-Weber syndrome). Patients have a
chronic problem with recurrent GI bleeding and
require multiple transfusions and iron
supplementation. 10% of patients have
pulmonary AV malformation. Oesophageal
dysmotility is seen in scleroderma.

Telangiectatic lesions in HHT


A cardiothoracic surgeon is planning for surgery
in a 60 year old man with coronary artery
disease. In coronary artery bypass grafting,
which of the following vessels should be
considered for grafting to the Left anterior
descending artery?
A. Saphenous vein graft
B. Left internal mammary
C. Right internal mammary
D. Radial artery
E. Brachial artery
Answer: b) left internal mammary. The left
internal mammary artery supplies the anterior
chest wall. It has been shown to be superior to
saphenous vein grafts (from aorta to LAD) in
staying patent and hence is now the choice
artery (LIMA to LAD) graft. Although circumflex
and right coronary arteries are usually grafted
with veins, RIMA arteries are sometimes used to
graft the RCA.
A 45 year old man has developed pains and swelling over
his wrist joints bilaterally over 3 months. He has limited
wrist movements and clubbing. X rays show periosteal
reaction at the ends of the radius and ulnar bones
suggestive of periostitis. Which medical condition
predisposes to this?
Behcet's disease
B. Crohn's disease
C. Amyloidosis
D. Diabetes
E. Hyperthyroidism
Answer: b) Crohn's disease. Hypertrophic
osteoarthropathy (or hypertrophic pulmonary
osteoarthropathy when there is mesothelioma or
bronchogenic carcinoma associated) is
associated with conditions such as liver
cirrhosis, ulcerative colitis, whipple's disease
and crohn's disease.

Hypertrophic osteoarthropathy
A 30-year-old woman complains of severe headache, mainly in the occipital
region, as well as irritability, low mood and lack of energy. Her husband
informs you that he has noted her to be withdrawn and short tempered. She
also gives a history of a recent increase in hair loss, and joint pains,
predominantly in the wrist and knees. She has had amenorrhoea for the last
3 months. She works as a secretary, and is now finding it extremely difficult
to cope with her work and household activities. She and her husband are
requesting sick leave. On examination, she looks unwell. Her temperature is
38oC, she is pale and there is an erythematous rash on her face. Joint
examination is remarkable for tenderness in the wrists and knees, but no
swelling. Cardiovascular, Respiratory and abdominal examination are
unrevealing. Neurological examination does not show focal neurologic signs
or neck stiffness. Laboratory results were returned as follows: Hb 9.7
g/dl WBC3.8 x 10 9/l, N2.1 x 10 9/l,L0.8 x 10 9/l, PLT88 x 10 9/l, ESR 88
mm/1st hr, CRP8 mg/l, AST 44 IU/l, ALK-P156 IU/l, ALT 50 IU/l, ANA
positive, DNA and ENA awaited, Urine dipstick protein+
What is the most likely clinical problem that may explain the above?
1. Neuropsychiatric lupus
2. Viral meningitis
3. Lupus glomerulonephritis
4. Antiphospholipid antibody syndrome
5. Cranial angitis
True 1. The above clinical findings and investigation results are
suggestive of systemic lupus erythematosus (SLE). However,
the symptoms of headache and mood changes point towards
the possibility of neuropsychiatric lupus. Features of
neurological disease in lupus range from the common,
relatively harmless migraine headache, to major psychotic
episodes and grand mal seizures, recognised in some lupus
patients.
Lupus glomerulonephritis cannot be completely excluded but
active urinary sediment with proteinuria of ++ on dipstick will
be more suggestive.
Antiphospholipid antibody syndrome (APS) is unlikely even
though the patient has thrombocytopenia; lymphocytosis
rather than lymphopenia would be more consistent with APS.
A 65 year old man is assessed on the ward for
weakness in his legs. He is an ex smoker and drinks 15
units of alcohol in a week. His wife mentions that he is
confused. On examination, his MMSE score is 20/30. He
has an ataxic gait. There is bilateral pyramidal weakness
and coordination is impaired. Routine blood tests are
normal. An MRI scan of the head shows diffuse white
matter changes, more in the cerebellar region than the
cerebrum. Which of these tests would help most in
confirming the diagnosis?
A. CSF for oligoclonal bands
B. CSF for anti Hu and anti Yo antibodies
C. CSF for TB culture
D. EEG E. EMG
Answer: b) CSF for Anti Hu and anti Yo
antibodies. Anti Hu and anti Yo antibodies
would help confirm a diagnosis of
paraneoplastic syndrome. Multiple
sclerosis is unlikely in view of late
presentation and is not commonly
associated with dementia.
A 36 year old female gave a recent history of
sensory impairment and imbalance in left
half of the body. She also complained of
tingling .She had recurrent episodes of
ataxia in last year each episode resolved
spontaneously. What is the likely diagnosis
of this episode?
A. Transient ischaemic attack
B. Intracranial space occupying lesion
C. Multiple sclerosis
D. Cerebellar haemorrhage
Answer: C) Multiple sclerosis.
History of patchy sensory loss, and ataxia
which improves on occasion is suggestive
of an inflammatory disorder, in this case
most likely multiple sclerosis.
Which of the following diseases is
associated with an increased frequency
of HLA -DR4 on the cell membranes?
A. Ankylosing spondylitis
B. B. Rheumatoid arthritis
C. C. Porphyria
D. D. Congenital adrenal hyperplasia
E. E. Narcolepsy
Answer: b) rheumatoid arthritis. Lack of
HLA DR2 is associated with narcolepsy,
and increased HLA DR4 is associated with
rheumatoid arthritis.
11.Anti-neutrophilic cytoplasmic autoantibodies:

A positive only in Wegeners syndrome associated


with renal disease .
B cause neutropenia in SLE .
C present in inflammatory bowel disease .
D is invariably associated with Polyarteritis
Nodosa.
E ANCA positive glomerulonephritis characteristically
causes nephrotic syndrome.
25 year old lady complains of unilateral throbbing
headache for 10 months. Each time she has
headaches, she also complains of unilateral
weakness in the arm which resolve
spontaneously when the headaches improve.
CT head was normal. What is the diagnosis?
A. Cluster headache
B. Migraine
C. Tension headache
D. Somatoform disorder
E. Cavernous sinus thrombosis
Answer: B) migraine. Hemiplegic migraine
is a term used to describe the migraine
syndrome that is associated with a
weakness or sensory loss of the limbs on
one side of the body. The headache
usually precedes the weakness by a day
or more. The limbs gradually return to
normal over several days.
31 yrs old female coming for infertility.
PCO by US , BMI 35. what is the most
appropriate drug?
A. Testosterone
B. clomiphene
C. Finasteride
D. Rosiglitazone
E. Metformin
Answer: clomiphene Both metformin and
clomiphene has been shown to be
effective in stimulating ovulation in patients
with PCOS (clomiphene is more effective,
but not in the options). The reduction of
hormonal imbalance and treating insulin
resistance helps to restore the ovulatory
cycles and fertility.
A 65 year old lady is seen in the rhematology
clinic with the results of a DEXA scan. The T
score of the hip is -2.8 SD and in the spine is -
2.5 SD. Which of the following medications does
NOT improve bone mineral density?
A. Calcium and vitamin D
B. Oestrogen replacement
C. Selective oestrogen receptor modulator
(SERM)
D. Bisphosphonates
E. Phosphate replacement
59- 50-year-old old man is admitted to hospital
unconsious, and smelling of alcohol. One hour
after admission, he becomes suddenly sweaty
with a regular tachycardia of 110 bpm and a
BP of 100/50. What is the diagnosis?
1) Alcohol withdrawal.
2) Hepatic encephalopathy.
3) Hypoglycaemia.
4) Subdural haematoma.
5) Wernicke's encephalopathy.
The answer is 3
This is a bit early for alcohol withdrawal
particularly as the patient is admitted
smelling of alcohol. The most likely
diagnosis is hypoglycaemia. We do not
have any clinical findings to suggest any of
the other alternativies.
A 45 year old lady has had long standing
arthritis of her hands and feet. Which of the
following X ray changes suggests rheumatoid
arthritis instead of a seronegative
arthropathy?
A. Osteosclerosis
B. Osteophytes
C. Osteoporotic changes
D. Periarticular erosions
E. Loss of joint space
Answer: d) periarticular erosions.
Osteophytes and loss of joint space are
commonly found in osteoarthritis, and
can also be found in rheumatoid arthritis.
Periarticular erosions are suggestive of
rheumatoid arthritis.

Periarticular Erosions
A 60 year old man is brought to hospital having
collapsed to the ground suddenly and was unable to
move his left leg or arm. There was no loss of
consciousness. He has a past medical history of
hypertension only. The episode lasted a few seconds
and he has been relatively well. Examination reveals a
mild hemiparesis of the left arm and leg. Which is the
likely diagnosis?
A. Pontine haemorrhage
B. Primary epilepsy
C. Medullary haemorrhage
D. Right internal capsule infarct
E. Left internal capsule infarct
Answer: d) right internal capsule infarct.
This patient is likely to have a lacunar
infarct involving the internal capsule,
causing transient contralateral
hemiparesis.
A 35 year old man presents acutely with urethritis,
conjunctivitis and arthritis. Rheumatoid factor is
negative and he has raised inflammatory
markers. In view of the likely diagnosis, which
one of the following features is associated?
A. Myocardial infarction
B. B. Pulmonary embolus
C. C. Circinate balanitis
D. D. nail dystrophyE. Calcinosis
Answer: c) circinate balanitis. The diagnosis is Reiter's
syndrome. Reiters syndrome is urethritis,
conjunctivitis, seronegative arthritis (cannot see, cannot
pee, cannot climb a tree). The typical patient is a young
man with recent urethritis or dysentery. The seronegative
arthritis is usually a mono or oligoarthritis.
Other features are anterior uveitis, keratoderma
blenorrhagica (brown abscesses on palms and soles),
mouth ulcers, plantar fasciitis and archilles tendinitis
(enthesopathy), circinate balanitis (painless rash) and
aortic incompetence. The arthritis may relapse or remain
chronic. Management is usually with rest and NSAIDs.
A demyelinating polyneuropathy is typically
caused by:
1) Diabetes
2) Excessive alcohol
3) Hereditary motor-sensory neuropathy
4) Renal failure
5) Vitamin B12 deficiency
The answer is 3
The differential diagnosis of demyelinating
neuropathy includes: hereditary motor-sensory
neuropathy (Charcot-Marie Tooth disease),
Refsum's Disease, Guillain-Barre syndrome,
chronic inflammatory demyelinating
polyneuropathy (CIDP), paraprotein-related
disorder, leukodystrophies. Amiodarone,
Diabetes, alcohol, Vitamin deficiencies and renal
failure cause an axonal polyneuropathy.
A 35 year old lady has skin pigmentation,
hypotension, hyponatraemia. A short synacthen
test shows a rise in cortisol from 100 to 140
g/ml. Which one of the following conditions is
associated?
A. Papillary thyroid carcinoma
B. Ovarian fibroids
C. Phaeochromocytoma
D. Hepatocellular carcinoma
E. Pernicious anaemia
Answer: e) pernicious anaemia. Addison's
disease is described. Many autoimmune
diseases are associated e.g. vitiligo,
diabetes, primary ovarian failure and
pernicious anaemia.
A 70-year-old woman presented with episodic
impairment of consciousness.
Which of the following is the most likely cause?
1) Alzheimer type dementia
2) chronic sub-dural haematoma
3) Creutzfeldt-Jacob disease
4) depressive stupor
5) normal pressure hydrocephalus
The answer is 2
This is quite a grey question. The clinical scenario is very
brief with no mention of any neurological signs so a
logical deduction must be made.
Alzheimer's disease would be expected to have a
continous impairment of consciousness in its advanced
stages but could be episodic if there were variation in
drugs therapy or concurrent illnesses. Similarly Normal
Pressure Hydrocephalus, Creutzfeld-Jacob and
depression would present with dementia (or apparent
dementia) but not fluctuant.
Of all those listed subdural haematoma is classically
associated with fluctuating level of consciousness. This
would make it the most likely.
Temporal lobe lesions cause:
1) Apraxia
2) 2) Astereogenesis
3) Primitive reflexes
4) Visuospatial neglect
5) Wernike's (receptive) aphasia
The answer is 5
Lesions of the frontal lobe include difficulties with task sequencing and
executive skills. Expressive aphasia (receptive aphasias a temporal lobe
lesion), primitive reflexes, perseveration (repeatedly asking the same
question or performing the same task), anosmia and changes in personality.
Lesions of the parietal lobe include apraxias, neglect, astereognosis (unable
to recognise an object by feeling it) and visual field defects (typically
homonymous inferior quadrantanopia). They may also cause alcalculia
(inability to perform mental arithmetic). Lesions of the temporal lobe cause
visual field defects (typically homonymous superior quadrantanopia),
Wernike's (receptive) aphasia, auditory agnosia, and memory impairment.
Occipital lobe lesions include cortical blindness (blindness due to damage to
the visual cortex and may present as Anton syndrome where there is
blindness but the patient is unaware or denies blindness), homonymous
hemianopia, and visual agnosia (seeing but not percieving objects - it is
different to neglect since in agnosia the objects are seen and followed but
cannot be named).
A 72-year-old lady has 4 months of memory loss,
urinary incontinence and falls. On examination
she has mild memory loss and a broad-based,
slow gait. Muscle tone is normal and both
plantar reflexes are downgoing. What is the
likely diagnosis?
1) Alzheimer's disease
2) Frontal lobe dementia
3) Mulit-infarct dementia
4) Normal-pressure hydrocephalus
5) Parkinson's disease
The answer is 4
Normal pressure hydrocephalus characterized by
abnormal gait, urinary incontinence, and
dementia. It is an important clinical diagnosis,
because it is a potentially reversible cause of
dementia. It is important to distinguish it from
Parkinson's Disease. The onset of gait
disturbance and urinary symptoms is unusual so
early in dementia. Frontal lobe dementia is
characterised by loss of 'executive' functions and
multi-infarct state usually has a step-wise
history.
What is the diagnosis of the patient
whose hand on the right and suffers
hyponatremia

Slide no 10

Normal hand
A 2 year old girl with unrepaired tetralogy of Fallot presents to the
emergency department with increased cyanosis. Physical examination
shows a small, very cyanotic child lying on a stretcher with her knees drawn
up against her chest. Pulse oximetry shows an oxygen saturation of 58% in
room air. Her respirations are rapid and deep. Auscultation of the heart
discloses tachycardia but no murmur. Which of the following is the most
appropriate general treatment strategy for this patient?

Decrease systemic vascular resistance to increase right-to-left shunting


Decrease pulmonary vascular resistance to decrease left-to-right shunting
Decrease systemic vascular resistance to decrease right-to-left shunting
Increase pulmonary vascular resistance to increase left-to-right shunting
Increase systemic vascular resistance to decrease right-to-left shunting
Explanation

The correct answer is Choice E.


Understanding the anatomy of patients with congenital heart disease is crucial for understanding their care. Often,
the best way to think about the anatomy for a patient with congenital heart disease is to imagine the path of blood
flow. For example, imagine a single red blood cell in a normal patient. The red blood cell returns to the heart via
lesion: the pulmonary
the vena cava, deoxygenated. outflowthe
It then enters tract obstruction
right atrium,and thethe
then VSD.
right ventricle, then goes to the lungs via
the pulmonary artery. After getting oxygenated in the pulmonary capillaries, it returns to the left atrium via a
pulmonary vein, then enters the left ventricle, then moves out to the body to deliver its oxygen to the tissues.
Patients with Tetralogy of Fallot have four distinct lesions ("PROVe": pulmonary outflow tract obstruction, right
ventricular hypertrophy, an overriding aorta, and a large VSD) but only two of them are necessary for
understanding the pathophysiology of the lesion: the pulmonary outflow tract obstruction and the VSD.
Imagine now a single red blood cell returning to the heart in a patient with Tetralogy of Fallot. After leaving the
vena cava and the right atrium, the RBC enters the right ventricle. Here, the cell must make a "choice" - to pass
through the stenotic pulmonary valve and enter the lungs, or to pass through the VSD and into the left ventricle
(and on to the systemic circulation) without ever getting oxygenated? Remember, blood flows to the path of least
resistance, and thus how much blood goes to the lungs versus how much goes to the body without being
oxygenated depends on the balance between the pulmonary vascular resistance and the systemic vascular
resistance.
When the pulmonary vascular resistance is high and the systemic vascular resistance is low, it is much more
difficult for blood to get into the lungs than it is for it to pass through the VSD and out to the circulation without
being oxygenated. This leads to severe right-to-left shunting, called a "Tet spell," and is what is happening to the
patient in the vignette.
Alternately, if the systemic vascular resistance is high and the pulmonary vascular resistance is low, it is easier for
blood to pass through the pulmonary outflow tract obstruction and into the lungs, where it can be
oxygenated. This is the goal of treatment for a Tet spell. Thus, the correct answer is choice E - if the systemic
vascular resistance increases, then the right-to-left shunt should decrease - which will reduce the patient's
cyanosis.
Decreasing the systemic vascular resistance would lead to increased right-to-left shunting (choice A), but this will
cause more cyanosis and would not be the goal of treatment.
Decreasing pulmonary vascular resistance (as in choice B) is a good idea, as it would allow more blood to enter
the lungs - but the goal is to decrease the right-to-left shunt, not the left-to-right.
Decreasing systemic vascular resistance (choice C) will lead to more right-to-left shunting, which is not what this
patient needs.
Increasing pulmonary vascular resistance (choice D) would cause increased right-to-left shunting, which as
described previously, would be very bad in this situation.

A 67-year-old woman was referred with a 3-month history of painful legs,


malaise and weight loss. She had had type 2 diabetes mellitus and
hypertension for 18 years. Her medication was gliclazide 160 mg twice daily,
ramipril 2.5 mg daily and atorvastatin 20 mg daily. On examination, her
blood pressure was 145/90 mmHg. There was some tenderness over her
spine and lower legs.
Investigations:
serum sodium 138 mmol/L (137144)
serum potassium 5.5 mmol/L (3.54.9)
serum creatinine 240 mol/L (60110)
serum corrected calcium 1.80 mmol/L (2.202.60)
serum phosphate 1.6 mmol/L (0.81.4)
plasma parathyroid hormone 22.2 pmol/L (0.95.4)

What therapy is most likely to correct the calcium and parathyroid hormone
concentrations?

A alendronic acid
B alfacalcidol
C calcitonin
D cinacalcet
E ergocalciferol
Answer Key: B
A 53-year-old man presented with a 2-week history of diarrhoea associated
with cramping abdominal pain. He was passing up to 15 very loose and
watery stools per day. There was no blood in stools. He had had a heart
transplant 2 years previously, and his medication comprised ciclosporin,
prednisolone, aspirin and ramipril.
Investigations:
haemoglobin 110 g/L (130180)
white cell count 12.5 109 /L (4.011.0)
serum urea 14.4 mmol/L (2.57.0)
serum creatinine 135 mol/L (60110)
serum C-reactive protein 35 mg/L (<10)
stool culture negative
stool microscopy cysts identified on modified acid-fast stain
What is the most likely pathogen?
A Cryptosporidium parvum
B Entamoeba histolytica
C Giardia lamblia
D Pneumocystis jirovecii
E Toxoplasma gondii
Answer Key: A
A 56-year-old man was referred to the medical clinic from the psychiatric unit
because of weight gain of 20 kg over 4 months. He had a 6-month history of severe
depression with psychosis, which had required inpatient treatment. He was taking
olanzapine 20 mg daily and fluoxetine 40 mg daily.
On examination, he appeared Cushingoid, with centripetal obesity and a few
abdominal striae. He had reasonable proximal muscle strength. His blood pressure
was 170/100 mmHg.

Investigations:
fasting plasma glucose 8.5 mmol/L (36)
serum cholesterol 6.4 mmol/L (<5.2)
serum LDL cholesterol 5.01 mmol/L (<3.36)
serum HDL cholesterol 0.75 mmol/L (>1.55)
fasting serum triglycerides 2.42 mmol/L (0.451.69)
overnight dexamethasone suppression test (after 1 mg dexamethasone)
serum cortisol 55 nmol/L (<50) 24-h urinary cortisol 310 nmol (55250)

What is the most likely diagnosis?


A adrenal Cushings syndrome
B ectopic ACTH syndrome
C metabolic syndrome
D pituitary-dependent Cushings disease
E pseudo-Cushings syndrome
E
An 82-year-old woman with hypertension presented with
tiredness, ankle swelling and arthralgia. On examination, her
pulse was 92 beats per minute and her blood pressure was
150/90 mmHg. She had bilateral ankle oedema. Her serum
creatinine concentration had been normal 6 months previously.
Urinalysis showed protein 2+, blood 2+.
Investigations:
haemoglobin 103 g/L (115165) white cell count 10.5
109/L (4.011.0) platelet count 410 109/L (150400)
serum creatinine 252 mol/L (60110)
What is the most likely cause of her renal impairment?
A amyloidosis
B crescentic glomerulonephritis
C IgA nephropathy
D membranoproliferative glomerulonephritis
E membranous nephropathy
Answer Key: B
A 72-year-old man presented with a 4-year history of acute intermittent pain and swelling of the
knees. The problem affected one knee at a time and each episode lasted about a week. He
took naproxen for the pain. His serum urate was measured during one of the attacks and was
found to be normal. He had drunk 2428 units of alcohol per week for 30 years. There was no
family history of diabetes mellitus.
On examination, his body mass index was 34 kg/m2 (9499). His pulse was 64 beats per minute
and his blood pressure was 110/70 mmHg. His liver was enlarged to 5 cm below the costal
margin and his spleen to 3 cm. The metacarpophalangeal joints of the index and middle fingers
in both hands were swollen. Urinalysis showed glucose 3+.
Investigations:
haemoglobin 165 g/L (130180)
white cell count 9.5 109/L (4.011.0)
platelet count 135 109/L (150400)
serum sodium 128 mmol/L (137144)
serum potassium 3.1 mmol/L (3.54.9)
serum urea 3.5 mmol/L (2.57.0)
serum creatinine 56 mol/L (60110)
serum albumin 31 g/L (3749)
serum total bilirubin 32 mol/L (122)
serum alanine aminotransferase 133 U/L (535)
serum aspartate aminotransferase 110 U/L (131)
fasting plasma glucose 13.4 mmol/L (3.06.0)
What is the most likely diagnosis?
A alcoholic cirrhosis
B haemochromatosis
C palindromic rheumatism
D rheumatoid arthritis
E sarcoidosis
Answer Key: B
A 26-year-old woman was admitted as an emergency. She complained of right upper
quadrant pain, fever and shaking. She was 30 weeks pregnant with her first child.
On examination, her temperature was 39.5C, her pulse was 120 beats per minute and her
blood pressure was 105/80 mmHg. She was jaundiced and tender over her liver. The uterine
fundus was palpable above the umbilicus.

Investigations: haemoglobin 107 g/L (115165)


white cell count 14.9 109/L (4.011.0)
neutrophil count 12.1 109/L (1.57.0)
platelet count 257 109/L (150400) prothrombin time 15.5 s (11.515.5)
serum sodium 138 mmol/L (137144)
serum potassium 3.4 mmol/L (3.54.9)
serum urea 8.7 mmol/L (2.57.0)
serum creatinine 130 mol/L (60110)
serum albumin 34 g/L (3749)
serum total bilirubin 118 mol/L (122)
serum alanine aminotransferase 204 U/L (535)
serum alkaline phosphatase 609 U/L (45105)
serum gamma glutamyl transferase 748 U/L (435)

What is the most likely diagnosis?


A autoimmune hepatitis
B common bile duct stone
C HELLP syndrome
D hepatitis A infection
E primary sclerosing cholangitis
Answer Key: B
A 73-year-old retired man was admitted after he had been found to have
renal impairment by his general practitioner. He gave a 3-month history of
lethargy and back pain, with thirst and constipation over the past 4 weeks.
He had noticed that he had passed less urine than normal in the past 3
days. Abdominal examination was normal.
Investigations: haemoglobin 98 g/L (130180)
white cell count 5.6 109 /L (4.011.0)
platelet count 380 109 /L (150400)
erythrocyte sedimentation rate 115 mm/1st h (<20)
serum urea 36.3 mmol/L (2.57.0)
serum creatinine 651 mol/L (60110)
serum corrected calcium 2.92 mmol/L (2.202.60)
serum total protein 85 g/L (6176)
serum albumin 34 g/L (3749)
serum alkaline phosphatase 99 U/L (45105)
What is the most likely diagnosis?
A carcinoma of prostate
B myeloma
C primary hyperparathyroidism
D sarcoidosis
E tuberculosis
Answer Key: B
A 60-year-old woman was admitted with a 2-day
history of dysuria, loin pain and rigors. On
admission, she was unwell and confused. She was also
febrile and tachycardic. She was transferred to the
medical high-dependency unit for invasive monitoring.
Which set of haemodynamic values is most likely to be
present?
M Arterial pr M Right atrial pr M.PAP PA wedge Pr . COP
normal 85 3 15 9 5.0
A 80 8 22 20 3.0
B 110 18 20 11 4.0
C 85 6 16 8 2.5
D 66 20 22 20 2.0
E 60 2 15 8 4.5
A 34-year-old woman was admitted with shortness of breath,
and found to have a pulmonary embolus. On systemic enquiry,
she admitted to cold intolerance. She was taking no medication.

On examination, she had a livedo reticularis rash on the thighs.


Blood tests showed marked thrombocytopenia and a prolonged
activated partial thromboplastin time. Antinuclear antibodies
were negative and complement C3 and C4 levels were normal.
The blood film was normal, with no evidence of haemolysis.

What is the most likely diagnosis?


A antiphospholipid antibody syndrome
B cryoglobulinaemia
C mixed connective tissue disease
D systemic lupus erythematosus
E thrombotic thrombocytopenic purpura

Answer Key: A
A 19-year-old woman presented with a
widespread eruption 2 weeks after a sorethroat.
On examination, there were multiple 5-mm
diameter, scaly, erythematous papules over her
trunk and limbs.
What is the most likely diagnosis?
A atopic eczema
B dermatitis artefacta
C guttate psoriasis
D lichen planus
E pityriasis versicolor
Answer Key: C
A 27-year-old woman attended the emergency department with a 1-week history of
progressive dyspnoea and cough, and a 2-day history of left basal thoracic pain on
inspiration and haemoptysis. She had a history of bronchiectasis. She was a non-
smoker. She had recently returned from a holiday in New Zealand, and was taking
co-amoxiclav and prednisolone prescribed by her general practitioner for an
exacerbation of her bronchiectasis. On examination, she was thin. Her temperature
was 37.3C, her pulse was 115 beats per minute and regular, her blood pressure
was 128/78 mmHg and her respiratory rate was 22 breaths per minute. Further
examination was normal.
Investigations:
haemoglobin 157 g/L (115165)
white cell count 18.0 109/L (4.011.0) serum C-reactive protein 68 mg/L (<10)
arterial blood gases, breathing air:
PO2 8.9 kPa (11.312.6) PCO2 4.9 kPa (4.76.0)
pH 7.40 (7.357.45)
H+ 40 nmol/L (3545)
bicarbonate 22 mmol/L (2129)
ECG sinus tachycardia
chest X-ray cystic changes at left base

What is the most appropriate next investigation?


A CT pulmonary angiography
B D-dimer
C echocardiography
D ultrasound scan of legs and pelvis
E ventilation/perfusion isotope lung scan.
Answer Key: A
A 30-year-old woman had a 3-year history of
Crohns disease, which had required the
formation of an ileostomy. She presented with
a sore area around the stoma . This was
causing problems with adhesion of the stoma
pouch.
What is the most appropriate treatment?
A oral flucloxacillin
B oral prednisolone
C radiotherapy
D surgical debridement
E topical terbinafine
Answer Key: B
A 60-year-old man was admitted with a 3-day history of diarrhoea. A
diagnosis of lung and peritoneal metastases from an unknown primary
carcinoma had been made
4 months previously and he had been receiving platinum-based combination
chemotherapy. He had been discharged from hospital 5 days earlier after an
episode of neutropenic sepsis following his second cycle of chemotherapy.
On examination, he was apyrexial, his pulse was 98 beats per minute and his
lying blood pressure was 110/65 mmHg. He looked dehydrated. His
abdomen was soft but tender over the left iliac fossa.
Investigations: haemoglobin 113 g/L (130180) white cell count 6.5
109/L (4.011.0) neutrophil count 5.4 109/L (1.57.0) platelet count
170 109/L (150400) CT scan of abdomen thickened sigmoid colon
What is the most likely diagnosis?
A cryptosporidiosis
B flare-up of diverticular disease
C ischaemic colitis
D pseudomembranous colitis
E tumour progression
Answer Key: D
A 79-year-old woman was referred to the medical outpatient clinic with a 3-
month history of low back pain. She was taking regular paracetamol and
occasional ibuprofen. She was normally fit and active, with no other
complaints. Examination was normal.
Investigations:
full blood count normal
serum urea and electrolytes normal
Serum calcium normal
serum immunoglobulin (Ig) G 15.2 g/L (6.013.0)
serum IgA 2.5 g/L (0.83.0)
serum IgM 1.0 g/L (0.42.5)
serum protein electrophoresis IgG kappa paraprotein: 4.3 g/L
X-ray of lumbar spine generalised osteopenia; no focal collapse; mild
degenerative change
What is the most likely diagnosis?
A amyloidosis
B low-grade lymphoma
C monoclonal gammopathy of undetermined significance
D myeloma
E solitary plasmacytoma

Answer Key: C
A 52-year-old man presented with the sudden onset of
severe anterior chest pain. An acute myocardial
infarction was suspected and subsequently confirmed
by serum troponin concentration and ECG. Following
admission, he was noted to be hypotensive with
oliguria. A SwanGanz catheter was inserted. The
right atrial pressure was found to be 20 mmHg (48)
and the indirect left atrial mean pressure (wedge) was
2 mmHg (510). What is the most likely explanation
for these pressure measurements?
A acute left ventricular failure
B acute mitral regurgitation
C hypovolaemia
D pericardial tamponade
E right ventricular failure
Answer Key: E
A 50-year-old woman presented with a 24-hour
history of palpitations. An ECG revealed atrial
fibrillation with a ventricular rate of 130 beats per
minute.
Which drug is most likely to restore sinus rhythm?
A adenosine
B bisoprolol
C digoxin
D flecainide
E verapamil
2. Answer Key: D
A 48-year-old woman complained of leg weakness
and tenderness, which was worse on exercise. Her
serum creatine kinase was elevated, but a muscle
biopsy was inconclusive.
Antinuclear antibodies were negative, but antibodies
to gastric parietal cells and thyroid peroxidase were
both detected.
There was no anaemia but the MCV was raised.

What is the most likely diagnosis?


A alcohol abuse
B folate deficiency
C hypothyroidism
D pernicious anaemia
E polymyositis
Answer Key: C
A 25-year-old woman was reviewed 6 weeks after stopping
anticoagulant therapy. She had received anticoagulants for 3
months following a right iliofemoral vein thrombosis that had
developed 7 days post partum.
Investigations:
protein C 85 IU/dL (80135)
protein S 110 IU/dL (80120)
antithrombin 95 IU/dL (80120)
prothrombin 20210A allele negative
factor V Leiden mutation heterozygous
What is the most appropriate further management?
A anticoagulation for an indefinite period with target international
normalised ratio (INR) 2.5
B anticoagulation for an indefinite period with target INR 3.5
C anticoagulation for another 3 months with target INR 2.5
D long-term aspirin
E no further anticoagulation
Answer Key: E
Long-term anticoagulant therapy is not recommended in
patients with VTE provoked by surgery (1B) Long-term
anticoagulant therapy is not recommended in
patients with VTE provoked by non-surgical transient
trigger factors (1B).
Patients with unprovoked proximal DVT or PE should be
considered for long-term anticoagulation, taking
intoaccount information that may help predict risk of
recurrence and risk of bleeding in the individual
patient(2B).
Long-term anticoagulant therapy is not recommended in
patients with VTE conned to the calf (i.e. not extending
into the popliteal vein)(1A).
A 21-year-old man was admitted to hospital with a 5-day history of fevers
and vomiting. He also complained of knee and ankle pains. He was
homosexual, and had last had receptive anal intercourse 6 weeks
previously.
On examination, he had a widespread erythematous macular rash. His
temperature was 37.5C, his pulse was 85 beats per minute and his blood
pressure was 115/60 mmHg. There was no joint swelling.

Investigations:
haemoglobin 147 g/L (130180)
white cell count 6.5 109 /L (4.011.0)
serum urea 4.3 mmol/L (2.57.0)
serum total bilirubin 50 mol/L (122)
serum alanine aminotransferase 687 U/L (535)
serum alkaline phosphatase 110 U/L (45105)
serum gamma glutamyl transferase 89 U/L (<50)

What is the most likely diagnosis?


A acute hepatitis B
B acute hepatitis C
C acute HIV infection
D gonococcal bacteraemia
E secondary syphilis
A
A 24-year-old man presented with a 2-day history of fever and a generalised
blistering rash. He was taking prednisolone 10 mg daily for asthma. His son
had had chickenpox 2 weeks previously.
On examination, he was low in mood. His temperature was 38.5C, his
blood pressure was 118/76 mmHg and his respiratory rate was 14 breaths
per minute. His oxygen saturation was 96%, breathing air (9499). He had a
widespread eruption consisting of vesicles and pustules. Examination of his
chest revealed a few wheezes but no crackles.

Investigations:
haemoglobin 128 g/L (130180) platelet count 189 109/L(150400)
white cell count 15.2 109/L (4.011.0)
neutrophil count 13.8 109/L (1.57.0) lymphocyte count 1.0 109/L (1.54.0)
chest X-ray normal

What is the most appropriate next management step?


A antipyretic
B intravenous aciclovir
C intravenous flucloxacillin
D oral valaciclovir
E varicella zoster hyperimmune globulin
B
A 72-year-old woman presented with weakness of her right leg and
numbness in her right hand. She had a 20-year history of rheumatoid
arthritis and also had long-standing type 2 diabetes mellitus and
hypertension. Her medication comprised gliclazide, amlodipine, simvastatin
and sodium aurothiomalate.
On examination, she had chronic rheumatoid changes in her hands and
feet, and subcutaneous nodules at her elbows. She had bruises in the
nailfolds, and her right index fingertip was cold and discoloured.
Neurological examination revealed altered sensation in all of the fingers of
her right hand. She was unable to dorsiflex her right ankle.
Investigations:
erythrocyte sedimentation rate 110 mm/1st h (<30)
antinuclear antibodies positive at 1:80 dilution
c-ANCA negative
p-ANCA positive

What is the most likely cause of her symptoms?


A amyloidosis
B diabetes mellitus
C rheumatoid vasculitis
D systemic lupus erythematosus
E Wegeners granulomatosis

Answer Key: C
A 42-year-old man was admitted to hospital with severe abdominal pain. His
alcohol intake was 18 units per week. His serum amylase level was raised at
1346 U/L (60180) and a diagnosis of acute pancreatitis was made. There
was no evidence of gallstones. He made an uncomplicated recovery.

Investigations (after recovery):


fasting plasma glucose 5.7 mmol/L (3.06.0)
serum cholesterol 5.8 mmol/L (<5.2)
serum LDL cholesterol 3.41 mmol/L (<3.36)
serum HDL cholesterol 0.96 mmol/L (>1.55)
fasting serum triglycerides 22.63 mmol/L (0.451.69)

What is the most appropriate treatment to reduce his risk of recurrent pancreatit
A atorvastatin
B ciprofibrate
C ezetimibe
D nicotinic acid
E omega-3-marine triglycerides
Answer Key: B
A 65-year-old man had a 10-year history of dialysis-dependent renal failure
caused by renovascular disease. He began taking warfarin because of
recurrent arteriovenous fistula thrombosis. One month later, he presented
with livedo reticularis on his trunk, and areas of painful ulceration on his
shins.
Investigations:
haemoglobin 102 g/L (130180) white cell count 5.4 109 /L (4.011.0)
eosinophil count 0.78 109 /L (0.040.40)
platelet count 478 109 /L (150400)
international normalised ratio 1.9 (<1.4) serum corrected calcium 2.58
mmol/L (2.202.60) serum phosphate 1.9 mmol/L (0.81.4)
plasma parathyroid hormone 18.5 pmol/L (0.95.4)
anticardiolipin antibodies:
immunoglobulin G 32 U/mL (<23)
immunoglobulin M 24 U/mL (<11)

What is the most likely diagnosis?


A antiphospholipid antibody syndrome
B calciphylaxis
C cholesterol embolisation
D coumarin necrosis
E thromboembolism from arteriovenous fistula

Answer key C
A 35-year-old woman with early stage IV chronic
kidney disease (glomerular filtration rate 1529
mL/min) was being considered for erythropoietin
therapy.
On examination, her blood pressure was 140/90
mmHg and she had an arteriovenous fistula in situ for
planned haemodialysis.
Investigations: haemoglobin 95 g/L (115165) MCV
84 fL (8096)
What is the most likely outcome of erythropoit. therapy?
A improved blood pressure control
B improved exercise tolerance
C increased ventricular hypertrophy
D reduced likelihood of red cell aplasia
E stabilisation of renal function
3. Answer Key: B
A 51-year-old woman presented with a 2-year history of
intermittent episodes of arthralgia, rash and fevers. She
complained of increasing fatigue, breathlessness and swollen
ankles over the previous 6 months. On examination, she had a
purpuric rash on the lower extremities and a right-sided pleural
effusion. Urinalysis showed protein 3+, blood 1+.
Investigations:
erythrocyte sedimentation rate 140 mm/1st h (<30)
serum creatinine 140 mol/L (60110)
serum complement C4 <5 mg/dL (1550)
antinuclear antibodies positive at 1:600 dilution (negative at
1:20 dilution) rheumatoid factor 90 kIU/L (<30)
What is the most likely diagnosis?
A HenochSchnlein purpura
B microscopic polyangiitis
C mixed cryoglobulinaemia
D systemic lupus erythematosus
E systemic rheumatoid disease
4. Answer Key: D
5. A 27-year-old woman presented with a right-
sided thyroid swelling with associated cervical
lymphadenopathy.
What is the most likely cause?
A anaplastic carcinoma
B follicular adenoma
C follicular carcinoma
D Hashimotos thyroiditis
E papillary carcinoma
5. Answer Key: E
A previously well 64-year-old woman presented with haemoptysis.
Biopsy of a right upper-lobe endobronchial lesion confirmed the
diagnosis of non-small cell bronchogenic carcinoma.
Investigations:
forced expiratory volume in 1 s 61% of predicted
forced vital capacity 78% of predicted
CT scan of chest and abdomen 3.5-cm mass in right upper
lobe with ipsilateral hilar lymph node enlargement; no disease
below the diaphragm
PET scan increased uptake in right upper lobe and ipsilateral
hilar nodes
What is the most appropriate management?
A chemotherapy
B palliative radiotherapy
C pneumonectomy
D radical radiotherapy
E radiofrequency ablation
Answer Key: C
6. A 77-year-old man presented with anaemia and
diarrhoea. He had undergone emergency partial
gastrectomy for a bleeding ulcer 9 months previously.
He was having maintenance proton pump inhibitor
therapy.
Investigations: endoscopy active, benign-looking
stomal ulceration
What is the most appropriate next investigation?
A barium follow-through
B endoscopic ultrasound of gastric remnant
C fasting serum gastrin concentration
D Helicobacter pylori serology
E prolonged glucose tolerance test
6. Answer Key: C
A clinical trial compared medical versus
surgical treatment for stable angina. A large
number of patients withdrew from the study.
Which type of analysis should be used to
estimate the likely benefits in similar groups of
patients in future?
A intention-to-treat
B meta-analysis
C on-treatment
D post-hoc
E sensitivity
A
A 62-year-old man presented with a 2-day history of pleuritic right lower chest pain associated
with a cough productive of redbrown sputum. He had a 1-year history of microscopic
polyangiitis which had initially presented with pulmonary haemorrhage and rapidly
progressive glomerulonephritis. He had responded well to plasma exchange, methylprednisolone
and cyclophosphamide before being switched to azathioprine at 3 months. At his clinic visit 3
weeks previously, his serum C-reactive protein had been 5 mg/L (<10) and his eGFR 28
mL/min (>60). His current medication was prednisolone 7.5 mg daily and azathioprine 100 mg
daily. On examination, he was comfortable at rest. His temperature was 37.8C, his blood
pressure was 106/78 mmHg and his respiratory rate was 18 breaths per minute. There was
bronchial breathing and a pleural rub at the right lung base but no crackles.
Investigations:
haemoglobin 120 g/L (130180) white cell count 9.4 109 /L (4.011.0) platelet count 256
109 /L (150400) serum C-reactive protein 56 mg/L (<10) estimated glomerular filtration rate
(eGFR) 26 mL/min (>60)
arterial blood gases, breathing air:
PO2 10.6 kPa (11.312.6)
PCO2 4.2 kPa (4.76.0)
pH 7.44 (7.357.45)
H+ 36 nmol/L (3545)
bicarbonate 18 mmol/L (2129)
chest X-ray patchy shadowing at right base

What is the most appropriate treatment?


A intravenous cefotaxime and clarithromycin
B intravenous cefotaxime, clarithromycin and co-trimoxazole
C intravenous clarithromycin, amoxicillin and co-trimoxazole
D oral amoxicillin and clarithromycin
E oral amoxicillin, clarithromycin and co-trimoxazole
Answer key: D
7. A 70-year-old man presented with a 2-week history of
persistent fever. Some weeks previously, he had undergone
colonoscopy and biopsy for a suspected carcinoma of the
colon, and was awaiting surgery. He had a history of moderate
mitral regurgitation caused by mitral valve prolapse.

Investigations: echocardiogram a vegetation on the anterior


leaflet of the mitral valve.

What is the most likely causative organism?


A Bartonella henselae
B Staphylococcus aureus
C Streptococcus bovis
D Streptococcus viridans
E Yersinia enterocolitica
7. Answer Key: C
8. A 37-year-old woman with a history of depression was
brought to hospital having been found in a state of collapse.
She had written a suicide note, and had an empty bottle of
amitriptyline in her jacket. On examination, her pulse was 135
beats per minute and her blood pressure was 105/50 mmHg.
Investigations:
serum potassium 4.5 mmol/L (3.54.9)
serum bicarbonate 18 mmol/L (2028)
serum urea 8.5 mmol/L (2.57.0)
serum creatinine 110 mol/L (60110)
ECG sinus tachycardia; QRS duration of 135 ms; bursts of
non-sustained ventricular tachycardia
What is the most appropriate initial intravenous treatment?
A amiodarone
B esmolol
C magnesium
D sodium bicarbonate
E sodium chloride 0.9%
8. Answer Key: D
9. A 21-year-old man presented after he had injured his
back, resulting in hemisection of his spinal cord at T10
level. What is most likely to be present below the level
of the lesion 2 months after the injury?

A contralateral gross wasting


B contralateral loss of pain and temperature sensation
C contralateral loss of proprioception
D contralateral upper motor neurone weakness
E ipsilateral gross wasting
9. Answer Key: B
A 46-year-old man with profuse diarrhoea was admitted to measure his daily stool
weight for
3 days, with a further measurement on day 4 when fasting. He had previously had a
normal
colonoscopy and small bowel meal.

Investigations:
serum sodium 135 mmol/L (137144)
serum potassium 2.4 mmol/L (3.54.9)
serum urea 5.8 mmol/L (2.57.0)
serum creatinine 78 mol/L (60110)

daily stool weight:


day 1 1450 g (<200)
day 2 1200 g
day 3 1560 g
day 4 (fasting) 1400 g

What is the most likely diagnosis?


A coeliac disease
B irritable bowel syndrome
C lactose intolerance
D pancreatic insufficiency
E VIPoma

Answer Key: E
11. A 79-year-old woman was admitted for elective
hip replacement surgery. On examination, she was
pale. There was a 2-cm splenomegaly and there were
small discrete axillary lymph nodes.
Investigations:
haemoglobin 107 g/L (115165) white cell count
34.5 109/L (4.011.0) platelet count 183 109/L.
What is the most likely diagnosis?
A acute myeloid leukaemia
B chronic lymphocytic leukaemia
C chronic myeloid leukaemia
D myelodysplasia
E myelofibrosis
11. Answer Key: B
A 60-year-old man with diet-controlled type 2 diabetes
mellitus gave a 3-month history of numbness and pins and
needles sensations in his feet. He also felt unsteady. Two
years previously, he had undergone surgery for carcinoma of
the stomach.
On examination, he had mild weakness of hip flexion. His
ankle reflexes were absent and all of his other limb reflexes
were diminished. His plantar responses were extensor. There
was diminished sensation to pinprick and light touch below
the knees, vibration sense was impaired at the ankles, but
joint position sense was normal. Rombergs test was positive.

What is the most likely diagnosis?


A diabetic amyotrophy
B paraneoplastic sensory ataxic neuropathy
C spinal arteriovenous malformation
D subacute combined degeneration of the cord
E syringomyelia
Answer Key: D
A 23-year-old man presented to the emergency department
with acute severe asthma. His regular medication was inhaled
salbutamol and beclometasone, and oral theophylline.
On examination, he was dyspnoeic and in distress. His pulse
was 104 beats per minute, his blood pressure was 108/64
mmHg and his respiratory rate was 40 breaths per minute.
Auscultation of his lungs revealed expiratory wheezes
throughout. He was unable to perform a peak expiratory flow
reading and he failed to respond to initial therapy with oxygen,
nebulised salbutamol and ipratropium bromide, and oral
prednisolone 40 mg.
What is the most appropriate next step in treatment?
A intravenous aminophylline
B intravenous hydrocortisone
C intravenous magnesium sulphate
D non-invasive ventilation
E subcutaneous terbutaline
Answer Key: C
A 20-year-old Chinese woman presented with a 2-week history of ankle swelling.
She had suffered from asthma since childhood and was taking paracetamol regularly
for pains in her wrists and knees. She was also taking an oral contraceptive.
Examination showed bilateral pitting oedema to mid-shin level. Her blood pressure was
148/92 mmHg and her heart sounds were normal. Chest, abdominal and neurological
examinations were normal. Urinalysis showed blood 1+ and protein 4+, but was negative
for glucose.

Investigations:
haemoglobin 118 g/L (115165)
MCV 79 fL (8096)
white cell count 3.3 109 /L (4.011.0)
serum sodium 136 mmol/L (137144)
serum potassium 4.1 mmol/L (3.54.9)
serum urea 5.7 mmol/L (2.57.0)
serum creatinine 73 mol/L (60110)
24-h urinary protein 7.8 g (<0.2)

What is the most important diagnostic investigation?


A anti-glomerular basement membrane antibodies
B anti-neutrophil cytoplasmic antibodies
C antinuclear antibodies
D antistreptolysin O titre
E serum complement levels
Answer Key: C
12. A 74-year-old woman presented with
distressing restlessness in her legs, particularly
at night. She usually had to get out of bed
several times every night to reduce her
symptoms, and her sleep was being disturbed.
What is the most appropriate treatment?
A clonazepam.
B co-beneldopa .
C propranolol .
D quinine.
E ropinirole.
12. Answer Key: E
is a non-ergoline dopamine agonist. It is used in
the treatment of Parkinson's disease. Ropinirole
is one of three medications approved by the
FDA to treat Restless Legs Syndrome, the other
two being pramipexole and gabapentin.
Ropinirole can cause nausea, dizziness,
hallucinations, orthostatic hypotension, and
sudden sleep attacks during the daytime
13. A 34-year-old woman with a 6-year
history of Sjgrens syndrome found that she
was 14 weeks pregnant. Which autoantibody
in this condition is associated with an
increased risk of congenital heart block in the
fetus?
A anticardiolipin
B anticentromere
C anti-La .
D anti-neutrophil cytoplasmic (ANCA)
E anti-Ro
13. Answer Key: E
14. A 40-year-old woman presented for review of her
asthma. Her current treatment was inhaled beclometasone
800 micrograms per day and inhaled salbutamol as
required via a metered-dose inhaler. She used the
salbutamol two or three times a day and woke at night
wheezing once or twice a week.
What is the most appropriate management?
A add montelukast
B add salmeterol
C change to a powder inhaler
D double the dose of beclometasone
E maintain current treatment
14. Answer Key: B
15. A 21-year-old woman presented with watery
diarrhoea and cramping abdominal pain 5 days after
arriving in Mexico. On examination, her temperature was
37.8C and there was mild abdominal tenderness. There
was some mucus in the stools, but no blood. What is
the most likely pathogen?
A Aeromonas hydrophila
B Entamoeba histolytica
C Escherichia coli
D Giardia lamblia
E rotavirus
15. Answer Key: C
16. A 27-year-old man was referred with an acute
hepatic illness. Which laboratory finding would indicate
the need for inpatient management?
A aspartate aminotransferase:alanine aminotransferase
ratio >1.0
B prothrombin time 24 s (11.515.5)
C serum alanine aminotransferase 1400 U/L (535)
D serum alkaline phosphatase 1800 U/L (45105)
E serum conjugated bilirubin 110 mol/L (<3.4)
16. Answer Key: B
17. A 75-year-old woman presented with a 6-month
history of an ulcer over the right ankle. She had a history
of right deep venous thrombosis 5 years previously. On
examination, she had a superficial sloughing ulcer 6 cm
in diameter over the medial malleolus.
What is the most appropriate investigation?
A anklebrachial pressure index
B bacteriological swab of the ulcer
C bilateral lower limb arteriography
D right leg venography
E venous duplex ultrasound scan
17. Answer Key: A
18. A 66-year-old woman presented with a 4-
week history of cough and haemoptysis. She
was a lifelong non-smoker. Investigations:
chest X-ray right upper lobe tumour and an
enlarged right hilum, bronchoscopy normal
What is the most likely histological diagnosis?
A adenocarcinoma
B carcinoid tumour
C large cell carcinoma
D small cell carcinoma
E squamous cell carcinoma
18. Answer Key: A
A 75-year-old man presented to his general practitioner with worsening
palpitations and dyspnoea on exercise. He had lost about 3 kg in weight
during the past 2 months. He had developed a coarse tremor in both hands.
His past medical history included ischaemic heart disease and recurrent
supraventricular tachycardia. He was taking aspirin 75 mg daily, simvastatin
40 mg daily, bisoprolol 5 mg daily, ramipril 10 mg daily, amiodarone 200 mg
daily, glyceryl trinitrate spray as required, and warfarin.

On examination, there was a small palpable goitre. He had a tremor, warm


hands, bilateral upper eyelid retraction and proptosis.

Investigations:
plasma thyroid-stimulating hormone <0.1 mU/L (0.45.0)
plasma free T4 95.0 pmol/L (10.022.0)
plasma free T3 35.2 pmol/L (5.010.0)
ECG sinus tachycardia

What is the most likely cause of this patients thyroid dysfunction?


A amiodarone-induced thyrotoxicosis
B Graves disease
C Reidels thyroiditis
D solitary toxic nodule
E toxic multi-nodular goitre
Answer Key: B
19. A 65-year-old man presented with an acute coronary
syndrome. He was advised to take clopidogrel in addition
to aspirin, atenolol, glyceryl trinitrate and low-molecular-
weight heparin. What is the predominant mechanism
through which clopidogrel inhibits platelet aggregation?
A it enhances the effect of circulating antithrombin
B it inhibits binding of adenosine diphosphate
C it inhibits cyclooxygenase-1
D it irreversibly binds glycoprotein IIb/IIIa receptor sites
E it prevents production of thromboxane A2
19. Answer Key: B
20. In a clinical trial to assess the effectiveness of a new
antihypertensive drug, subjects were randomised to be
treated with either the new drug or an existing standard
antihypertensive one. The main outcome criterion was
the blood pressure after 2 months of treatment. What is
the most appropriate statistical technique to compare
mean blood pressure between the groups?
A chi-squared test
B Mann Whitney U test
C Pearsons correlation coefficient
D regression analysis
E two-sample t-test
20. Answer Key: E
21. A 21-year-old woman, undergoing chemotherapy for
non-Hodgkins lymphoma, was in contact with her
nephew for 2 hours on the day that he developed a
chickenpox rash. Investigations: varicella serology
negative What is the most appropriate management?
A advise her to seek treatment if she develops a
chickenpox rash
B interrupt chemotherapy for 2 weeks
C oral aciclovir 800 mg five times daily for 1 week
D varicella immunisation
E varicella zoster immunoglobulin
21. Answer Key: E
23. A 65-year-old woman presented with generalised
arthralgia for the previous 2 weeks. She had a past
medical history of urinary tract infections. She was advised
to take a non-steroidal anti-inflammatory drug. On
examination, her blood pressure was 139/98 mmHg and
she had pitting oedema below the knees. Investigations:
serum creatinine 458 mol/L (60110), 24-h urinary total
protein 2.3 g (<0.2)
What is the most likely cause of the renal impairment?
A acute tubular necrosis
B immunoglobulin A (IgA) nephropathy
C interstitial nephritis
D membranous glomerulonephropathy
E papillary necrosis
23. Answer Key: C
25. A 25-year-old woman presented with increasingly severe
headache and two generalised fits 36 hours after the normal vaginal
delivery of her first baby at 40 weeks gestation. She had been
treated with epidural analgesia during labour. On examination, her
temperature was 37.6C, she was drowsy, there was a mild left
hemiparesis and both plantar responses were extensor. What is the
most likely diagnosis?

A bacterial meningitis
B cortical thrombophlebitis
C pre-eclamptic toxaemia
D subarachnoid haemorrhage
E viral encephalitis
25. Answer Key: B
26. A 37-year-old woman with breast cancer
had a family history of breast and ovarian
cancer. Molecular genetic testing revealed a
BRCA1 mutation. What is the normal function of
BRCA1?
A angiogenesis
B apoptosis
C cell adhesion
D promotion of mitosis
E tumour suppression
26. Answer Key: E
27. A 66-year-old man, who was undergoing
maintenance haemodialysis through a subcutaneous
tunnelled catheter, presented with fever, chills and rigors
during haemodialysis. On examination, his temperature
was 38.6C, his pulse was 105 beats per minute and his
blood pressure was 100/60 mmHg. Examination of his
respiratory and cardiovascular systems was normal.
What is the most likely cause of his infection?
A Escherichia coli
B Pseudomonas aeruginosa
C Staphylococcus epidermidis
D Streptococcus pneumoniae
E Streptococcus viridans
27. Answer Key: C
28. A 35-year-old woman presented with a 6-month
history of episodes of sweating, joint pain and
headaches. On examination, her blood pressure was
160/90 mmHg. A clinical diagnosis of acromegaly was
suspected. Investigations: fasting plasma glucose 8.1
mmol/L (3.06.0) Which additional investigation would
confirm the diagnosis?
A fasting growth hormone
B growth hormone suppression test
C insulin-like growth factor 1
D insulin tolerance test
E MR scan of pituitary
28. Answer Key: B
29. A 54-year-old right-handed woman
presented with the sudden onset of reading
difficulties. Investigations: CT scan of head a
recent left parietal lobe infarct. Which additional
feature is most likely to be present?
A acalculia
B confabulation
C cortical deafness
D expressive dysphasia
E homonymous hemianopia
29. Answer Key: A
30. A 75-year-old man presented with a 2-day history of
recurrent fever and a cough productive of purulent
sputum. Investigations: chest X-ray right lower lobe
pneumonia
What additional finding would most indicate a poor
prognosis?
A respiratory rate of 25 breaths per minute
B serum sodium concentration of 130 mmol/L (137144)
C serum urea concentration of 9.0 mmol/L (2.57.0)
D systolic blood pressure of 95 mmHg
E temperature of 38.5C
30. Answer Key: C
31. A 30-year-old woman presented with a 6-month
history of tremor and difficulty in speaking. On
examination, she was found to have increased muscle
tone in all four limbs, bradykinesia and 4-cm
hepatomegaly. Which laboratory finding would best
support a diagnosis of Wilsons disease?
A increased incorporation of radioactive copper into
caeruloplasmin
B low hepatic copper content
C low serum caeruloplasmin concentration
D low serum copper concentration
E low urine copper concentration
31. Answer Key: C
A 65-year-old man presented with a 1-week history of
increasing drowsiness and confusion. He rapidly deteriorated
and was intubated, ventilated and transferred to the intensive
care unit. He had a history of recurrent chest infections in the
previous year, treated with several courses of oral antibiotics.
He had developed progressive difficulty in climbing the stairs
over the previous 5 years. His father had died of respiratory
failure in his mid-fifties.
On examination, he had bilateral ptosis and facial weakness. His
eye movements were normal. His neck flexors were weak. He
had predominantly distal weakness affecting his arms and legs.
The deep tendon reflexes were absent. The plantar responses
were flexor. Sensory examination was normal.
What is the most likely underlying diagnosis?
A Beckers muscular dystrophy
B GuillainBarr syndrome
C motor neurone disease
D myasthenia gravis
E myotonic dystrophy
Answer Key: E
33. A 72-year-old man presented with a 2-day
history of pain and swelling of the right knee.
Analysis of synovial fluid from the right knee
confirmed the presence of calcium
pyrophosphate crystals. What is the
microscopic appearance of these crystals?
A needle-shaped with negative birefringence
B needle-shaped with no birefringence
C needle-shaped with positive birefringence
D rhomboid with negative birefringence
E rhomboid with positive birefringence
33. Answer Key: E
34. The half-life of a novel anti-obesity drug
exhibiting first-order kinetics was calculated to
be 4 hours. What percentage of the drug will be
eliminated 20 hours after ingestion?
A 75%
B 80%
C 90%
D 97%
E 100%
34. Answer Key: D
35. A 16-year-old girl presented with non-scaly,
discrete areas of hair loss on the scalp. She had
a past history of atopic eczema and had a
number of depigmented areas on her hands and
around her eyes. What is the most likely
diagnosis?
A alopecia areata
B hypothyroidism
C lupus erythematosus
D seborrhoeic dermatitis
E trichotillomania
35. Answer Key: A
37. A 38-year-old woman required extraction of
her wisdom teeth. She gave a history of
haemorrhage after a dental extraction 10 years
previously, when she had required suturing.
There had been no history of excessive bleeding
before this. What is the most likely diagnosis?
A factor V Leiden
B factor IX deficiency
C factor XII deficiency
D primary antiphospholipid syndrome
E von Willebrands disease
37. Answer Key: E
38. A 25-year-old intravenous drug user
presented with an injection site abscess. On
examination, his temperature was 38.5C and
there was a pansystolic murmur. Which
organism is most likely to be cultured from both
the abscess and the blood?
A Clostridium novyi
B Klebsiella pneumoniae
C Staphylococcus aureus
D Staphylococcus epidermidis
E Streptococcus viridans
38. Answer Key: C
A 72-year-old man presented following an episode of collapse.
There had been two similar episodes recently, each lasting
about 1 minute. He had experienced an anterior myocardial
infarction 4 years previously. On examination, he was
orientated and symptom-free with a regular pulse of 80
beats per minute. His blood pressure was 140/80 mmHg
and the apex beat was displaced to the left. There was an
apical systolic murmur. There were no signs of trauma. ECG
showed sinus rhythm, anterior Q waves and anterior ST
segment elevation without reciprocal depression.
What is the most likely diagnosis?
A acute anterior myocardial infarction
B cerebral embolism
C epilepsy
D pulmonary embolism
E ventricular tachycardia
39. Answer Key: E
41. A 33-year-old man presented with a 6-month history
of cough and breathlessness that tended to worsen as
the week progressed and improve when he went on
holiday. He was a lifelong non-smoker and worked as a
paint sprayer in a car factory.Investigations: chest X-ray
normal. Which further investigation would be most useful
in establishing a diagnosis?
A cardiorespiratory exercise test
B histamine challenge
C immunoglobulin E antibody to isocyanate
D serial peak expiratory flow rate measurements
E transfer factor
D
43. A 30-year-old man presented with increasing breathlessness
and wheeze. He smoked five cigarettes per day. Investigations:
serum 1-antitrypsin 0.12 g/L (1.12.1)
forced expiratory volume in 1 s 0.85 L (3.45.1)
forced vital capacity 4.75 L (4.26.5)
transfer factor for CO (TLCO) 4.1 mmol/min/kPa (7.112.7)

Which protease inhibitor genotype is present?


A MM
B MZ
C SS
D SZ
E ZZ
E
44. A 25-year-old man presented with a 3-month history of hearing
two people commenting on his actions. For the past 2 years, he had
become increasingly withdrawn and had neglected his self-care. He
had graduated from university at 21 years of age, but had never
been employed and had few friends. He had taken an overdose of
paracetamol 2 months previously. He had smoked cannabis
regularly for 5 years. He drank 60 units of alcohol weekly. On
examination, he made poor eye contact and his speech was
disjointed.

What is the most likely diagnosis?


A alcoholic hallucinosis
B borderline personality disorder
C cannabis-induced psychosis
D paranoid schizophrenia
E psychotic depression
D
45. A 42-year-old man with a 20-year history of
ulcerative colitis was advised that he was at
increased risk of developing cancer of the colon. He
was taking sulfasalazine with intermittent courses of
corticosteroids for acute exacerbations. He had also
tried various diets in the past, including a lactose-
free diet and a low-fibre diet. Which factor is most
likely to increase his risk of developing cancer of the
colon?
A chronic colonic inflammation
B he is likely to be a carrier for the caspase
recruitment domain-containing protein 15(CARD 15)
gene mutation
C he is likely to have the APC (adenomatous
polyposis coli) gene
D long-term immunosuppression
E reduced intake of dietary fibre
A
46. A 75-year-old woman presented with a 2-month
history of generalised malaise and headaches that
had disturbed her sleep. For the past 24 hours, she
had been unable to see with her right eye. She had a
past medical history of glaucoma and was using
regular timolol eye drops. Fundoscopic examination
revealed a swollen, pale right optic disc and a normal
left optic disc.
What is the most likely diagnosis?
A central retinal vein occlusion
B closed-angle glaucoma
C giant cell arteritis
D optic neuritis
E raised intracranial pressure
C
48. A 78-year-old man presented after several
falls. He had a 20-year history of rheumatoid
arthritis and his only medication was sulfasalazine.
On examination, he had been incontinent of urine
and had difficulty in recalling recent events. He had
chronic rheumatoid hand deformities, a right-sided
grasp reflex and bilateral extensor plantar
responses. He had difficulty in rising from a chair,
walked with small paces and was unsteady on
turning, having to hold on to his wife.
What is the most likely diagnosis?
A Alzheimers disease
B cervical myelopathy
C multi-infarct dementia
D normal pressure hydrocephalus
E Parkinsons disease
D
51. A 28-year-old man presented with weight loss,
abdominal distension, flatulence and foul-smelling
diarrhoea for 1 month following a visit to India.
Investigations:
endomysial antibodies negative
stool cultures and microscopy negative
What is the most likely diagnosis?
A acute HIV seroconversion illness
B coeliac disease
C giardiasis
D hookworm infection (ancylostomiasis)
E viral gastroenteritis
c
52. A 45-year-old man presented with recurrent
epistaxis. Examination revealed telangiectasia on his lips
and in the mouth. A diagnosis of hereditary
haemorrhagic telangiectasia was made.

What is the most likely mode of inheritance?

A autosomal dominant
B autosomal recessive
C mitochondrial inheritance
D sporadic mutation
E X-linked recessive
A
53. A 37-year-old man with hypertension attended for
outpatient review. He had previously been treated with
several drugs that he could not name. He had been
obliged to stop taking each of these because of
unacceptable adverse effects, which included
lethargy, ankle swelling and severe gum swelling and
bleeding. He was currently taking
bendroflumethiazide. On examination, his blood
pressure remained elevated at 168/96 mmHg. In view
of his previous history of adverse drug effects, what is
the most appropriate additional therapy?

A amlodipine
B bisoprolol
C diltiazem
D minoxidil
E perindopril
E
54. A 17-year-old boy presented with a non-blanching rash
over his legs, a swollen knee and painless frank
haematuria. Urine dipstick analysis showed blood 3+,
protein 1+.
Investigations: serum creatinine 210 mol/L (60110)
urine culture negative
ultrasound scan of kidneys normal

Which glomerular abnormality is most likely to be present at


renal biopsy?
A focal and segmental sclerosis
B foot process fusion
C linear deposition of IgG on the basement membrane
D mesangial deposition of IgA
E thickening of basement membranes
D
55. A 77-year-old man presented with increasing pains
around the low back and lower limb girdle. He had
recently presented with symptoms of hesitancy and post-
micturition dribbling. Investigations:
erythrocyte sedimentation rate 28 mm/1st h (<20)
serum corrected calcium 2.34 mmol/L (2.202.60)
serum phosphate 0.8 mmol/L (0.81.4)
serum alkaline phosphatase 2985 U/L (45105)
serum prostate-specific antigen 6 g/L (<4)

What is the most likely cause of this mans pain?


A insufficiency fracture of the pelvis
B osteomalacia
C Pagets disease of the pelvis
D polymyalgia rheumatica
E prostatic carcinoma with metastases
C
56. A healthy 19-year-old medical student was
asked to perform a Valsalva manoeuvre (forced
expiration against a closed glottis) for
demonstration purposes in a physiology class.
What is the most likely initial haemodynamic
response?
A decreased jugular venous pressure
B decreased pulse
C decreased systolic blood pressure
D decreased venous return to the heart
E increased cardiac output
D
57. A 16-year-old boy with type 1 diabetes mellitus was
treated with a biphasic insulin preparation but
achievement of good blood glucose control proved
difficult. He was offered treatment with the insulin
analogue, insulin lispro. Which characteristic of insulin
lispro might improve his glycaemic control?
A low incidence of hypoglycaemia
B low incidence of lipoatrophy at the injection site
C low risk of immunogenic reaction
D rapid onset of action
E small injection volume
D
58. A 16-year-old boy presented within half an hour of
falling. His parents reported that he had fallen and hit the
right side of his head on the ground. He had lost
consciousness for a few seconds and then made a full
recovery. On examination, he was fully cooperative and
had no focal neurological signs. Two hours later, his
conscious level deteriorated.
What is the most likely diagnosis?
A cerebral oedema
B diffuse axonal injury
C extradural haematoma
D subarachnoid haemorrhage
E subdural haematoma
C
59. A 17-year-old girl presented with a single,
painless, enlarged cervical lymph node. She
was asymptomatic. Investigations:
chest X-ray enlarged mediastinal lymph nodes
What is the most likely diagnosis?
A angioimmunoblastic T-cell lymphoma
B extramedullary plasmacytoma
C follicular B-cell non-Hodgkins lymphoma
D Hodgkins lymphoma
E mantle cell lymphoma
D
60. A 55-year-old man was found to have an
abnormal chest X-ray at an employment
medical check. He was well and had no
respiratory symptoms or signs. He smoked 10
cigarettes per day and had been heavily
exposed to asbestos when working in a
shipyard 30 years previously.
Which abnormality is most likely to be seen on
his chest X-ray?
A asbestosis
B bronchial carcinoma
C calcified pleural plaques
D diffuse pleural thickening
E mesothelioma
C
On removal of the renal arterial clamp
following a donor kidney transplantation,
the surgeon noted changes suggestive of
hyperacute rejection.
Which immunoglobulin is likely to be
responsible?
A IgA
B IgD
C IgE
D IgG
E IgM
D
A 46-year-old man presented within 1 hour of
ingesting 40 tablets of slow-release
theophylline.

What is the most appropriate initial management?


A activated charcoal
B alkaline diuresis
C gastric lavage
D observation only
E whole bowel irrigation
A
A 17-year-old boy presented with breathlessness
and night sweats. Investigations; confirmed a
diagnosis of Burkitts lymphoma. What is the
most likely underlying abnormality of gene
expression?
A expression of BCR-ABL
B loss of p53
C over-expression of BCL-2
D over-expression of c-MYC
E over-expression of JAK2
D
A 50-year-old man was admitted to the coronary
care unit. In which circumstance would it be
most appropriate to use a glycoprotein IIb/IIIa
receptor inhibitor?
A acute life-threatening pulmonary embolism
B acute myocardial infarction with ST segment
elevation
C continuing pain following thrombolytic therapy
D continuing pain with positive troponin and
awaiting coronary angiography
E left ventricular thrombus post myocardial
infarction
D
66. A 45-year-old man had recurrent
nephrolithiasis. Renal function tests and serum
calcium measurements were normal.
Investigations:
24-h urinary calcium 15.0 mmol (2.57.5)
24-h urinary urate 3.0 mmol (<3.6)
24-h urinary oxalate 0.20 mmol (0.140.46)
24-h urinary citrate 2.0 mmol (0.33.4)
What is the most useful therapy to reduce stone
formation?
A allopurinol
B dietary calcium restriction
C penicillamine
D potassium citrate
E thiazide diuretic
E
A 29-year-old woman was admitted to hospital with a 12-hour history of
severe throbbing headache and right-sided weakness. She was otherwise
well with no significant past medical history and was taking no medication
apart from the oral combined contraceptive. She denied any regular illicit
drug use but admitted to taking an ecstasy tablet 2 days previously.

Examination confirmed decreased power on the right, with brisk tendon


reflexes and an extensor plantar response. Her pupils were equal and
reactive to light. Fundoscopy was normal.

Investigations:
CT scan of head (18 h after symptom onset) no evidence of haemorrhage
cerebrospinal fluid:
opening pressure 200 mmH2O (50180)
total protein 0.41 g/L (0.150.45)

What is the most likely diagnosis?


A cerebral infarction
B cerebral venous thrombosis
C hemiplegic migraine
D idiopathic intracranial hypertension
E subarachnoid haemorrhage
Answer Key: B
67. A 42-year-old man was found to have abnormal liver
function tests. He had had ulcerative colitis for 15 years. He
had been treated initially with mesalazine but this had been
stopped 8 years previously when his disease had gone into
remission. Investigations:
serum albumin 40 g/L (3749)
serum total bilirubin 15 mol/L (122)
serum alanine aminotransferase 63 U/L (535)
serum aspartate aminotransferase 41 U/L (131)
serum alkaline phosphatase 741 U/L (45105)
serum gamma glutamyl transferase 221 U/L (<50)
What is the most likely cause of these blood results?
A autoimmune hepatitis
B fatty liver
C liver metastases
D primary biliary cirrhosis
E primary sclerosing cholangitis
E
68. A 32-year-old woman returned from a
holiday in the Mediterranean with a suntan and
numerous hypopigmented, slightly scaly lesions
on the neck and upper trunk.What is the most
likely diagnosis?
A chronic plaque psoriasis
B discoid eczema
C pityriasis rosea
D pityriasis versicolor
E seborrhoeic dermatitis
D
69. A 72-year-old woman with rheumatoid arthritis
presented with dysuria of 2 days duration. Her regular
medication comprised methotrexate, prednisolone and
paracetamol. On examination, she had a body mass
index of 34 kg/m2 (1825). Investigations: urine culture
coliforms, sensitive to ciprofloxacin. Which condition is
most likely to be precipitated by the use of ciprofloxacin?
A gastric ulceration
B glucose intolerance
C hepatitis
D pneumonitis
E tendinopathy
E
70. A 50-year-old man presented with a 24-hour
history of agitation, confusion and suicidal thoughts. He
had started VADchemotherapy (vincristine 0.4 mg daily,
doxorubicin 9 mg/m2 daily and dexamethasone 40 mg
daily) 4 days previously, following a diagnosis of multiple
myeloma. What is the most likely cause of his
symptoms?
A adjustment disorder
B confusion secondary to sepsis
C corticosteroid psychosis
D reactive depression
E vincristine encephalopathy
C
A 30-year-old doctor presented following a needlestick
injury sustained while treating a patient with recently
diagnosed HIV infection. The patient was hepatitis C
negative and had been vaccinated against hepatitis B
virus, and was taking no antiretroviral treatment.
What is the most appropriate post-exposure
prophylaxis for the doctor?
A single-drug antiretroviral treatment for 1 month
B single-drug antiretroviral treatment for 3 months
C three-drug antiretroviral treatment for 1 month
D three-drug antiretroviral treatment for 3 months
E two-drug antiretroviral treatment for 1 month
C
A 65-year-old woman presented with a 12-hour
history of the sudden onset of gait unsteadiness,
vomiting and headache, followed by increasing
drowsiness. What is the most likely diagnosis?
A acute cerebellar haemorrhage
B acute subdural haemorrhage
C frontal subdural empyema
D herpes simplex encephalitis
E pituitary apoplexy
A
A 75-year-old woman with hypertension had been treated
with bendroflumethiazide for the past 5 years. On
examination, her blood pressure was 125/78 mmHg.
Investigations:
serum sodium 140 mmol/L (137144)
serum potassium 3.2 mmol/L (3.54.9)
serum creatinine 67 mol/L (60110)
What is the most likely mechanism for the hypokalaemia?
A increased flow in the distal tubule
B increased potassium secretion in the proximal tubule
C opening of potassium channels in the principal cells
D reduced aldosterone secretion
E reduced potassium reabsorption in the loop of Henle
A
74. A 47-year-old man presented with a 6-month history of
episodic sweating and hunger. He had gained 10 kg in weight
and drank 10 units of alcohol per week.
Investigations:
full blood count normal
fasting plasma glucose 4.0 mmol/L (3.06.0)
liver function tests normal
What is the most appropriate next investigation?

A CT scan of pancreas
B electroencephalography
C home blood glucose monitoring
D plasma glucose concentration following a prolonged (72-h) fast
E serum C-peptide concentration
D
A 27-year-old woman had a history of repeated
attendances for acute exacerbations of asthma. On
current presentation, she had been short of breath
for 4 hours and had features of acute severe
asthma. Which finding would indicate that this
exacerbation of her asthma should be regarded as
life-threatening?
A PaCO2 of 5.5 kPa
B PaO2 of 8.4 kPa
C peak flow 35% predicted
D pulse 114 beats per minute
E respiratory rate 30 breaths per minute
A
A 37-year-old woman with immune
thrombocytopenia failed to respond to
corticosteroid therapy. Splenectomy was
planned.
What is the optimum time for pneumococcal
vaccination?
A 1 month after surgery
B 1 month before surgery
C 1 week after surgery
D 1 week before surgery
E perioperatively
A
A 32-year-old woman presented with feverishness and
myalgia for 2 days followed by headache, photophobia
and neck stiffness for the past 6 hours.
Investigations: white cell count 10.4 109/L (4.011.0)
random plasma glucose 5.2 mmol/L
cerebrospinal fluid: opening pressure 180mm (50180)
total protein 0.60 g/L (0.150.45), glucose 3.5 mmol/L
cell count 76/L ( 5) Lymphocyte count 46/L (3.5)
neutrophil count 30/L
What is the most likely causative organism?
A enterovirus
B Listeria monocytogenes
C Mycobacterium tuberculosis
D Neisseria meningitidis
E Streptococcus pneumoniae
A
78. A 25-year-old woman presented complaining of
loose stools five to six times a day. One year
previously, she had undergone resection of her
terminal ileum for Crohns disease. Her only drug
therapy was mesalazine. On examination, there was
no abdominal tenderness. Investigations:
haemoglobin 126 g/L (115165)
serum albumin 38 g/L (3749)
serum C-reactive protein 4 mg/L (<10)
What is the most likely diagnosis?
A active Crohns disease
B adverse effect of mesalazine
C bile salt-induced diarrhoea
D enteric infection
E irritable bowel syndrome
C
79. A 71-year-old woman presented with a 2-
hour history of severe pain and loss of vision in
her right eye. She had noticed halos around
lights during the previous 24 hours. On
examination, there was ciliary vessel
hyperaemia and a dilated unreactive pupil on
the affected side.
What is the most likely diagnosis?
A anterior uveitis
B closed-angle glaucoma
C diffuse scleritis
D orbital cellulitis
E retinal artery occlusion
B
A 65-year-old man presented with chest discomfort
consistent with stable angina pectoris after
climbing two flights of stairs.
Investigations: serum cholesterol 4.8 mmol/L
(<5.2). echocardiogram good left ventricular
systolic function Bruce protocol exercise test 3-
mm ST segment depression in stage 3. Which
treatment has been shown to improve the
prognosis in this situation?
A aspirin
B diltiazem
C isosorbide mononitrate
D ivabradine
E nicardipine
A
Which dietary substrate is broken down into
glucose and galactose by the action of
intestinal enzymes?
A fructose
B lactose
C maltose
D mannose
E sucrose
A
A 47-year-old woman was being treated with
lithium for bipolar affective disorder. On
examination, her blood pressure was
168/104 mmHg. What drug is the most
appropriate antihypertensive?
A amlodipine
B bendroflumethiazide
C doxazosin
D losartan
E ramipril
A
A 50-year-old man with tophaceous gout presented with painful joints 2
days after starting treatment with allopurinol. His only other
medication was paracetamol. He had a history of alcohol abuse. On
examination, his temperature was 38.5C and there was acute
inflammation of the finger joints, wrists, knees and ankles.
Investigations:
serum gamma glutamyl transferase 90 U/L (<50)
serum urate 0.65 mmol/L (0.230.46)
serum C-reactive protein 180 mg/L (<10)
What is the most likely cause of his symptoms?
A acute pseudogout
B alcoholic binge
C allergic reaction to allopurinol
D allopurinol therapy
E joint sepsis
D
A 45-year-old woman presented with right flank pain. She
had a 4-year history of hypertension and progressive
cognitive impairment. On examination, she had livedo
reticularis and tenderness in the right flank. Her blood
pressure was 185/105 mmHg. Urinalysis showed blood
3+, protein 1+. Investigations:
haemoglobin 129 g/L (115165)
white cell count 8.7 109/L (4.011.0)
platelet count 83 109/L (150400)
serum creatinine 106 mol/L (60110)
Which antibody test is most likely to be positive?
A anticardiolipin
B anticentromere
C anti-glomerular basement membrane
D antimitochondrial
E anti-neutrophil cytoplasmic
A
A 20-year-old man presented with a 7-day history of
bloody diarrhoea, abdominal pain and fevers. He had
recently returned from a backpacking holiday in South
America. On examination, his temperature was 38.5C
and the abdomen was tender on palpation.
Investigations:
stool microscopy pus cells and red blood cells
What is the most likely causative organism?
A Cryptosporidium parvum
B rotavirus
C Salmonella typhi
D Schistosoma mansoni
E Vibrio cholerae
C
A 65-year-old woman with a 40-year history of rheumatoid
arthritis presented with a 4-month history of lower leg
oedema. She was taking methotrexate weekly and
prednisolone daily. Urinalysis showed protein 3+.
Investigations:
haemoglobin 94 g/L (115165)
white cell count 10.6 109/L (4.011.0)
erythrocyte sedimentation rate 105 mm/1st h (<30)
serum albumin 24 g/L (3749)
Which investigation is most likely to be of diagnostic value?
A lower leg venography
B rectal biopsy
C renal angiography
D serum protein electrophoresis
E ultrasound scan of kidneys
B
A 71-year-old woman required a transfusion of 2
units of blood after a hip replacement. One week
later, her haemoglobin concentration had fallen
by 42 g/L. Which associated finding is most
likely to indicate a delayed transfusion reaction?
A conjugated hyperbilirubinaemia
B elevated D-dimer
C haemoglobinuria
D haemosiderinuria
E positive direct antiglobulin test
E
75-year-old man presented with weight loss, lethargy
and repeated haemoptysis. He had been treated for
pulmonary tuberculosis 10 years previously.
Investigations:
Aspergillus fumigatus precipitins positive
chest X-ray a solid lesion at the left lung apex
What is the most likely diagnosis?
A allergic bronchopulmonary aspergillosis
B aspergilloma
C bronchial carcinoma
D invasive aspergillosis
E reactivation of tuberculosis
B
A 36-year-old HIV-positive man presented with a 1-week history of
generalised pruritus. He had taken part in unprotected anal sex 2
months previously while on holiday in Spain. He had previously been
vaccinated against hepatitis B virus, with an adequate antibody
response. On examination, his temperature was 37.8C and he was
jaundiced. Examination was otherwise normal.
Investigations:
CD4 count 550 106/L (4301690)
serum total bilirubin 99 mol/L (122)
serum aspartate aminotransferase 754 U/L (131)
serum alkaline phosphatase 173 U/L (45105)
hepatitis A immunoglobulin M negative
What is the most likely diagnosis?
A acute hepatitis C
B acute hepatitis D
C cytomegalovirus infection
D syphilis
E toxoplasmosis
A
92. A post-marketing observational study of a
new drug was conducted on 5000 patients
following clinical trials.
What best describes the data generated from
this type of study?
A comparative efficacy
B costbenefit
C cost effectiveness
D potency
E profile of adverse effects
To help protect y our priv acy , PowerPoint has block ed automatic download of this picture.

E
Clinical trials involving new drugs are commonly classified into four phases. Each
phase of the drug approval process is treated as a separate clinical trial. The drug-
development process will normally proceed through all four phases over many years.
Preclinical trial .It involves in vitro (test tube or cell culture) and in vivo (animal)
experiments using wide-ranging doses
Phase 0 trials are also known as human microdosing studies .
Phase I trials are the first stage of testing in human subjects. Normally, a small
group of 20-100 healthy volunteers will be recruited. This phase is designed to
assess the safety (pharmacovigilance), tolerability, pharmacokinetics, and
pharmacodynamics of a drug.
Phase II trials are performed on larger groups (100-300) and are designed to assess
how well the drug works, as well as to continue Phase I safety assessments in a
larger group of volunteers and patients.
Phase III studies are randomized controlled multicenter trials on large patient
groups (3003,000 or more depending upon the disease/medical condition studied)
and are aimed at being the definitive assessment of how effective the drug is, in
comparison with current 'gold standard' treatment. Because of their size and
comparatively long duration, Phase III trials are the most expensive, time-consuming
and difficult trials to design and run, especially in therapies for chronic medical
conditions.
Phase IV trial is also known as Postmarketing surveillance Trial. Phase IV trials
involve the safety surveillance The safety surveillance is designed to detect any rare
or long-term adverse effects over a much larger patient population and longer time
period than was possible during the Phase I-III clinical trials.
93. A 20-year-old woman presented with a 2-month
history of an intensely pruritic rash on her trunk and
limbs. She was otherwise well. On examination, she had
multiple, violaceous, flat-topped papules over the flexor
surfaces of the wrists, and on the ankles and lower back.
The papules were aggregated in a linear fashion at one
site on her left forearm.
Which clinical feature is most likely to be present?
A asymmetric oligoarthritis
B burrows in finger webs
C conjunctival scarring
D involvement of the buccal mucosa
E non-scarring alopecia
D
95. In a large prospective trial of a statin in patients with angina, 1000
patients were treated with a statin and 1000 with a placebo. A total of 150
patients taking the placebo experienced a cardiovascular event, compared
with 100 taking the active drug.

placebo statin total


cardiovascular event 150 100 250
no cardiovascular event 850 900 1750

What is the number-needed-to-treat (NNT) to prevent one cardiovascular


event?

A 2.5 (250/100)
B 5 (250/50)
C 10 (1000/100)
D 20 (1000/50)
E 40 (2000/50)
D
A 58-year-old man with congestive heart failure
remained oedematous despite treatment with
furosemide 120 mg daily. Investigations:
serum sodium 134 mmol/L (137144)
serum potassium 3.4 mmol/L (3.54.9)
serum urea 10.6 mmol/L (2.57.0)
serum creatinine 156 mol/L (60110)
What is most likely to be present?
A high plasma aldosterone concentration
B high serum cortisol concentration
C low plasma angiotensin II concentration
D low plasma atrial natriuretic peptide concentration
E low plasma renin concentration
A
A 43-year-old woman presented with a 1-week history of polyuria
and nocturia without weight loss. Clinical examination was
normal. Investigations:
serum sodium 142 mmol/L (137144)
serum potassium 3.5 mmol/L (3.54.9)
serum corrected calcium 2.60 mmol/L (2.202.60)
random plasma glucose 7.0 mmol/L
plasma osmolality 310 mosmol/kg (278300)
urinary osmolality 200 mosmol/kg (3501000)

What is the most likely diagnosis?


A compulsive polydipsia
B diabetes insipidus
C diabetes mellitus
D hyperparathyroidism
E syndrome of inappropriate antidiuretic hormone secretion
B
A 55-year-old woman presented with a 2-day history
of diplopia and progressive unsteadiness while
walking. On examination, she had limited eye
movement in all directions although her pupils were
normal. Muscle power in the limbs was normal but
tendon reflexes were absent. There was prominent
bilateral finger nose ataxia. Plantar responses were
flexor. Sensation was normal.
What is the most likely diagnosis?
A brainstem stroke
B Miller Fisher syndrome
C multiple sclerosis
D myasthenia gravis
E Wernickes encephalopathy
B
A 60-year-old woman with rheumatoid arthritis presented with
an acutely inflamed right knee. The joint was aspirated and
methylprednisolone 80 mg was injected after the removal of
20 mL of non-purulent synovial fluid. The knee improved
following this procedure but her symptoms recurred and
became much worse a week later. Investigations:
haemoglobin 105 g/L (115165)
white cell count 15.5 109/L (4.011.0)
neutrophil count 13.5 109/L (1.57.0)
erythrocyte sedimentation rate 55 mm/1st h (<30)
What is the most appropriate immediate management?
A arthroscopic washout of right knee
B re-aspirate right knee
C re-inject right knee with corticosteroid
D rest right knee in a splint
E start intravenous flucloxacillin
B
A 52-year-old man presented with a 4-month history of
altered bowel habit with occasional bright-red blood
per rectum. Colonoscopy revealed an annular sigmoid
tumour and histology confirmed an adenocarcinoma.
A staging CT scan of abdomen revealed two 1-cm
adjacent lesions in the right lobe of the liver, highly
suspicious for liver metastases.

What is the most appropriate next management step?

A biopsy of liver lesions


B palliative chemotherapy
C palliative radiotherapy
D stenting of colorectal carcinoma
E surgical resection of carcinoma and liver lesions
Answer Key: E
42. A 26-year-old woman presented with unequal pupils.
On examination, the right pupil was larger than the left
and did not react to light, directly or consensually. On
convergence, the right pupil reacted very slowly, but
eventually became smaller than the left pupil. What
type of pupillary abnormality does this woman have?
A Adies tonic pupil
B afferent pupillary defect
C Argyll Robertson pupil
D Horners syndrome
E third cranial nerve palsy
A

Вам также может понравиться